Sie sind auf Seite 1von 728

y

o
u
rs
m
a
h
b
o
o
b
.w
o
rd
p
re
s
s
.c
SSC
o
m
Ultimate Guide to

Combined Graduate Level Exams


Test of
F General Intelligence & Reasoning
F Quantitative Aptitude
F English Language
F General Awareness
Covers :
F Latest Current Affairs
F Includes Past Year Questions in
Exercise
y
o
u
rs
m
a
• Head Office : B-32, Shivalik Main Road, Malviya Nagar, New Delhi-110017

h
b
o
o
• Sales Office : B-48, Shivalik Main Road, Malviya Nagar, New Delhi-110017

b
.w
Tel. : 011-26691021 / 26691713

o
rd
p
re
s
s
.c
o
m
Typeset by Disha DTP Team

DISHA PUBLICATION
All RightS Reserved

© Copyright Publisher

No part of this publication may be reproduced in any form without prior permission of the publisher. The author and the
publisher do not take any legal responsibility for any errors or misrepresentations that might have crept in. We have
tried and made our best efforts to provide accurate up-to-date information in this book.

For further information about the books from DISHA,


Log on to www.dishapublication.com or info@dishapublication.com
y
o
u
rs
m
a
h
Contents

b
o
o
b
.w
o
rd
p
Combined Graduate Level (Tier-I) Exam-2016 (11-09-2016) 2016-1 – 2016-8

re
s
s
Combined Graduate Level (Tier-II) Exam-2015 (25-10-2015) 2015-2-1 – 2015-2-24

.c
o
m
Combined Graduate Level (Tier-I) Exam-2015 (09-08-2015) 2015-1 – 2015-20

Section A : general intelligence & Reasoning

Verbal Reasoning

1. Analogy A-1
- A-9
2. Classification A-10 - A-14
3. Series A-15 - A-23
4. Coding-Decoding A-24 - A-33
5. Blood Relation A-34 - A-40
6. Direction & Distance A-41 - A-53
7. Time Sequence, Number & Ranking Test A-54 - A-59
8. Number Puzzle A-60 - A-67
9. Logical sequence of words A-68 - A-71
10. Clock & Calendar A-72 - A-78
11. Venn Diagrams A-79 - A-86
12. Syllogism A-87 - A-99
13. Mathematical Operation A-100 - A-103
14. Arithmetical Reasoning A-104 - A-107

15. Matrix Based Reasoning A-108 - A-111


16. Statement & Conclusions A-112 - A-116

non-verbal Reasoning
17. Cube & Dice A-117 - A-125
18. Completion of Figure A-126 - A-133
19. Figure Formation & Analysis A-134 - A-139
20. Paper Cutting & Folding A-140 - A-145
21. Visual Reasoning A-146 - A-151
22. Mirror and Water Image A-152 - A-157
23. Embedded Figure A-158 - A-161
24. Non-Verbal Series A-162 - A-166
y
o
u
rs
m
a
section b : quantitative aptitude

h
b
o
o
1. Number System B-1 - B-14

b
.w
2. HCF & LCM B-15 - B-22

o
rd
3. Algebraic Expressions and Inequalities B-23 - B-42

p
re
4. Average B-43 - B-52

s
s
5. Percentage B-53 - B-64

.c
o
6. Profit & Loss B-65 - B-78

m
7. Simple & Compound Interest B-79 - B-89
8. Ratio, Proportion & Partnership B-90 - B-105
9. Mixture & Alligation B-106 - B-116
10. Time and Work & Pipes and Cisterns B-117 - B-137
11. Speed, Time & Distance (Trains, Boat & Stream) B-138 - B-152
12. Mensuration B-153- B-171
13. Trigonometry B-172 - B-189
14. Geometry B-190 - B-218
15. Data Interpretation B-219 - B-228

section C : English Language


1. English Grammar C-1 -C-20
2. Vocabulary C-21 - C-35
3. Synonyms C-36 - C-47
4. Antonyms C-48 - C-62
5. Idioms and Phrases C-63 - C-72
6. Spelling Test C-73 - C-77
7. Spotting Errors C-78 - C-87
8. Sentence Improvement C-88 - C-98
9. Sentence Completion C-99 - C-110
10. Cloze Test C-111 - C-116
11. Comprehension Test C-117 - C-132

section D : General awareness


1. History D-1 - D-22
2. Geography D-23 - D-42
3. Polity D-43 - D-60
4. General Science D-61 - D-103
5. Economy D-104 - D-117
6. Miscellaneous D-118 - D-132
7. Computer Fundamentals D-133 - D-140
8. Current Affairs D-141- D-147
y
o
u
rs
m
a
SSC

h
Held on 11-09-2016

b
o
o
b
.w
o
rd
p
re
Combined Graduate Level (Tier-I) Exam-2016

s
s
.c
o
m
Time : 1 Hour Max. Marks : 100

PART-A : GENERAL INTELLIGENCE & REASONING 12. From the given alternative words , select the word which
cannot be formed using the letters of the given word:
DIRECTIONS (Qs. 1-3) : Select the related word/letters/numbers IN TELL IGEN CE
from the given alternatives: (a) CANCEL (b) NEGLECT
1. Medicine : Patient : : Education : ? (c) GENTLE (d) INCITE
(a) Teacher (b) School 13. If C is coded 3 , DASH is coded as 32 , then DANCE will be
(c) Student (d) Tuition coded as
2. LAMP : IXJM :: FISH : ? (a) 20 (b) 25
(a) CGPF (b) CFQE (c) 26 (d) 27
(c) CFPE (d) CGQF 14. If ' + ' means minus , ' - ' means multiplication, ' ÷ ' means plus,
3. 13 : 20 : : 17 : ? and ' x ' means division, then 15 - 3 + 10 x 5 ÷ 5
(a) 25 (b) 26 (a) 52 (b) 48
(c) 27 (d) 28 (c) 22 (d) 5
15. A certain system is followed to solve the problem.
DIRECTIONS (Qs. 4-6) : Find the odd word/letter/ number Accordingly find out the correct answer from the alternatives
form the given alternative. for the unsolved equations.
4. (a) Rival (b) Opponent 7 × 4 × 9 = 479
(c) Foe (d) Ally 9 × 5 × 2 = 592
5. (a) POCG (b) KLIZ 6 × 9 × 5 = 965
(c) BUDX (d) FQMV 8× 6×2=?
6. (a) 36 - 48 (b) 56 - 44 (a) 286 (b) 682
(c) 78 - 66 (d) 33 - 64 (c) 628 (d) 268
7. Arrange the following words as per order in the dictionary 16. Find the missing number from the given alternatives
1. Ambitions 2. Ambiguous 7 10 5
3. Ambiguity 4. Animation
5. Animal 16 40 8
(a) 3 ,2 , 4 , 1 , 5 (b) 3 , 2 , 5 , 4 , 1 15 ? 9
(c) 3 , 2 , 1 , 5 , 4 (d) 3 , 2 , 4 , 5 , 1 (a) 75 (b) 45
8. A series is given, with one term missing. Choose the correct (c) 20 (d) 30
alternative from the given ones that will complete the series. 17. Sanmitra walked 8 m towards the north. He turned to his
CAT , DBT , ECT , ? right and walked 16 m, then he turned to his left and walked
(a) DCT (b) FDT 5 m and again he turned to his left and walked 16 m. In which
(c) FCT (d) FAT
direction is he from his starting point?
9. A series is given , with one term missing . Choose the correct
(a) South (b) East
alternative from the given ones that will complete the series.
(c) North (d) West
5 , 11 , 24 , 51 , 106 ,____?
18. Consider the given statement/s to be true and decide which
(a) 122 (b) 217
of the given conclusions/assumptions can definitely be
(c) 120 (d) 153 drawn from the given statement.
10. Showing a man on the stage , Rita said , " He is the brother Statements :
of the daughter of the wife of my husband . How is the man 1. Some teacher are followers.
on stage related to Rita ? 2. Some followers are famous.
(a) SON (b) HUSBAND Conclusions :
(c) COUSIN (d) NEPHEW I. Some teachers are famous.
11. The age of Dr. Pandey is four times the age of his son. After II. Some followers are teachers.
10 years , the age of Dr. Pandey will be twice the age of his
(a) Only conclusion I follows
son. The present age of Dr. Pandey's son is
(b) Only conclusion II follows
(a) 4 years (b) 5 years
(c) Both conclusion I and II follows
(c) 6 years (d) 8 years
(d) Neither conclusion I nor II follows
y
o
u
rs
2016 - 2 Combined Graduate Level Exam

m
a
h
19. How many triangles are there in the question figure? 23. A piece of paper is folded and cut as shown below in the

b
given answer figures, indicate how it will paper when opened.

o
o
b
.w
o
(a) 18 (b) 24

rd
(c) 28 (d) 30

p
re
20. Identify the diagram that best represents the relationship

s
among classes given below

s
.c
Delhi , Sri Lanka , Asia (a) (b)

o
m
(a) (b)

(c) (d)

24. If a mirror is place on the line MN , then which of the answer


(c) (d) figures is the right image of the figure?

21. Which answer figure complete the pattern in the question figure?

(a) (b)
(a) (b)

(c) (d)

(c) (d)
25. In the question, a word is represented by only one set of
numbers as given in any one of the alternatives. The sets of
22. From the given figure, select the one in the question the numbers given in the alternatives are represented by two
figure is hidden / embedded classes of alphabets as in two matrices given below. The
columns and rows of Matrix I are numbered from 0 to 4 and
that of Matrix II are numbered from 5 to 9. A letter these
matrix can be represented first by row and next by its column,
e. g., ' A ' can be represented by 01 , 13 , etc. ., and ' B ' can be
represented by 58 , 69 , etc. . Similarly , you have to identify
the set for the word ' FINE '
MATRIX-I MATRIX-II
0 1 2 3 4 5 6 7 8 9
(a) (b) 0 F A N O I 5 S E H B T
1 I O F A N 6 H S E T B
2 A N O I F 7 B T S E H
3 O F I N A 8 E H T B S
4 N I A F O 9 T S E H B
(c) (d)
(a) 00 , 04 , 02 , 56 (b) 12 , 10 , 13 , 67
(c) 24 , 19 , 31 , 78 (d) 31 , 32 , 33 , 87
y
o
u
rs
CGL (Tier-I) 2016 Solved Paper 2016 - 3

m
a
h
PART-B : GENERAL AWARENESS 40. The famous Vishnu temple at Angkor Wat in Cambodia was

b
built by ?

o
o
26. Who propounded the "Doctrine of Passive Resistance"? (a) Shrutavarman (b) Suryavarman II

b
.w
(a) Balgangadhar Tilak (b) Aurobindo Ghosh (c) Indravarman (d) Aniruddha
(c) Lala Lajpat Rai (d) Bipin Chandra Pal 41. In IT, the technique of delaying outgoing acknowledgements

o
rd
27. At Rio Olympics, what was the final position of Dipa temporarily is known as

p
Karmakar in her category? (a) AR Acknowledgement (b) AR request

re
(a) 4 (b) 5 (c) Piggybacking (d) Piggyframing

s
s
(c) 6 (d) 7 42. If speed of rotation of the earth increases, the weight of the

.c
o
28. The 14th Finance Commission has recommended increase body

m
in States share in net proceeds from tax collection from 32% (a) decreases
to (b) increases
(a) 35% (b) 40% (c) remains the same
(c) 42% (d) 45% (d) may increase or decrease
29. Fiscal policy in India is formulated by? 43. Which among the following countries is facing debt crisis?
(a) Reserve Bank of India (b) Planning Commission (a) China (b) Canada
(c) Finance Ministry (d) SEBI (c) Germany (d) Greece
30. Which of the following has pH value 7? 44. The new symbol of Indian currency is designed by?
(a) Pure Water (b) H2SO4 (a) Santosh Kumar (b) Y.V. Reddy
(c) Sodium Carbonate (d) HCl (c) Udayakumar (d) Dr. Rangarajan
31. When ice cubes are made, the entropy of water 45. What is MUDRA?
(a) does not change (a) Development and Refinance Agency
(b) decreases (b) Scheme for Agricultural Insurance
(c) increases (c) New Planet Discovered
(d) may either increase or decrease depending on the (d) Development and Regulatory Authority for Urban
process used Township
32. Two vectors are said to be equal if 46. Which of the following is not correctly matched?
(a) only their magnitudes are same (a) Sania Mirza - Tennis
(b) only their directions are same (b) Sania Nehwal - Badminton
(c) both magnitude and direction are same (c) Mary Kom - Boxing
(d) magnitudes are same but directions are opposite (d) Pankaj Advani - Wrestling
33. The chemical component that is invariably found in all 47. Which of the following State is surrounded by Bangladesh
viruses is from three sides?
(a) proteins (b) lipids (a) Nagaland (b) Assam
(c) DNA (d) RNA (c) Arunachal Pradesh (d) Tripura
34. Which is used as an Air pollution indicator? 48. Which enzyme digests proteins in the stomach?
(a) Algae (b) Fungi (a) Trypsin (b) Pepsin
(c) Bacteria (d) Lichens (c) Salivary amylase (d) Pancreatic canal
35. The study of mountains is known as 49. Fermentation is a type of _____ _______ process.
(a) Oncology (b) Lithology (a) Aerobic Respiration (b) Anaerobic Respiration
(c) Orology (d) Ornithology (c) Exothermic Reaction (d) Transpiration
36. Salination of soil is caused by 50. The first Muslim to be elected President of 'Indian National
(a) Pesticides (b) soil erosion Congress' was?
(c) excess irrigation (d) crop rotation (a) Maulana Azad (b) Mohammed Ali
37. ICC World T20 in 2016 was won by (c) Badruddin Tyabji (d) Shah Wali-ullah
(a) India (b) England PART-C : QUANTITATIVE APTITUDE
(c) New Zealand (d) West Indies
38. Which one of the following was established with a definite 51. A,B and C contract a work for ` 440. Together A and B are to
provision under an Article of the Constitution of India? do 9/11 of the work. The share of C should be:
(a) Union Public Service Commission (a) 75 (b) 90
(b) National Human Rights Commission (c) 100 (d) 80
(c) Election Commission 52. A dealer marks a washing machine for ` 7500, and allows a
(d) Central Vigilance Commission discount of 6% on it. Find the selling price
39. The President can advance money to meet unforeseen (a) 6850 (b) 7050
expenses from the (c) 7250 (d) 6950
(a) Consolidated Fund of India 53. The sum of the cubes of two numbers in the ratio 3 : 4 is
(b) Grants of the Central Government 5824. The sum of the numbers is:
(c) Aid from the Union Government (a) (5824)1/3 (b) 28
(d) Contingency Fund (c) 24 (d) 14
y
o
u
rs
2016 - 4 Combined Graduate Level Exam

m
a
h
54. Loss of 20% on selling price is equal to x% loss in cost 68. PQR is an equilateral triangle. MN is drawn parallel to QR

b
price. What is x? such that M is on PQ and N is on PR. If PN = 6 cm, then the

o
o
(a) 20% (b) 20 length of MN is

b
(a) 3 cm (b) 6 cm

.w
2
(c) 16 % (d) 16 (c) 12 cm (d) 4.5 cm

o
3 69. If cosec q + sin q = 5/2 then the value of cosecq – sinq is

rd
55. In a motor of 120 machine parts, 5% parts were defective. In (a) –3/2 (b) 3/2

p
another motor of 80 machine parts, 10% parts were defective.

re
(c) – Ö3/2 (d) Ö3/2
For the two motors considered together, the percentage of

s
70. Two men are on opposite sides of a tower. They measure

s
defective machine parts were

.c
the angles of elevation of the top of the tower as 30° and 45°

o
(a) 7 (b) 6.5 respectively. If the height of the tower is 50 m, the distance

m
(c) 7.5 (d) 8 between the two men is (Take Ö3 = 1.7)
56. If (23)2 = 4x then 3x is equal to (a) 136.5 m (b) 50 Ö3 m
(a) 3 (b) 6 (c) 100 Ö3 m d) 135.5 m
(c) 9 (d) 27 71. The radius and the height of a cone are each increased by
57. A train, 200 m long, is running at a speed of 54 km/hr. The 20%. Then the volume of the cone increases by
time in seconds that will be taken by train to cross a 175 m (a) 20% (b) 20.5%
long bridge is (c) 62% (d) 72.8%
(a) 12.5 (b) 20 DIRECTIONS (Qs. 72-75) : A bar graph showing the heights of
(c) 25 (d) 10 six mountain peaks. Study the bar graph and answer questions
58. Which of the following ratios can be the ratio of the sides of H
9000
8200 8600 8800
a right angled triangle? e 8000 7500
i
7000 6500
(a) 9 : 6 : 3 (b) 13 : 12 : 5 g
h 6000
6000
(c) 7 : 6 : 5 (d) 5 : 3 : 2 t
i 5000
59. Number of circles that can be drawn through three non- n
m
4000
colinear points is e
t
3000
2000
(a) exactly one (b) two e
r 1000
(c) three (d) more than three s
0
A B C D E F
60. Value of (cos 53° – sin 37°) is
Mountain peaks
(a) 0 (b) 1 72. The average height of all the peaks (in meters) is
(c) 2 sin 37° (d) 2 cos 53° (a) 7601.5 (b) 7600
1 5x (c) 7599.5 (d) 7610
61. If 4x + = 5, × ¹ 0, then the value of is
x 4x 2
+ 10x + 1 73. Which peak is the second highest?
(a) 1/2 (b) 1/3 (a) B (b) C
(c) 2/3 (d) 3 (c) A (d) E
74. Write the ratio of the heights of the highest peak and the
1 3 1 lowest peak
62. If C + = Ö 3, then the value of C + 3 is equal to
C..... C (a) 22 : 15 (b) 15 : 22
(a) 0 (b) 3Ö3 (c) 20 : 13 (d) 13 : 22
(c) 1/Ö3 (d) 6Ö3 75. When the heights of the given peaks are written in
63. What least value must be assigned to '*' so that the numbers ascending order, what is the average of the middle two
451 * 603 is exactly divisible by 9? peaks?
(a) 7 (b) 8 (a) 7950 m (b) 7560 m
(c) 5 (d) 9 (c) 7650 m (d) 7850 m
64. The average of 9 observations was found to be 35. Later on,
it was detected that an observation 81 was misread as 18. PART-D : ENGLISH LANGUAGE
The correct average of the observations is DIRECTIONS (Qs. 76-77) : In the following question, out of the
(a) 28 (b) 42 four alternatives, choose the word which best expresses the
(c) 32 (d) 45 meaning of the given word.
65. If x = 222, y = 223, z = 225 then the value of x3 + y3 + z3 – 3xyz is 76. CREDULOUS
(a) 4590 (b) 4690 (a) FUNNY (b) SILLY
(c) 4950 (d) 4960 (c) INNOCCENT (d) GULLIBLE
66. If x = 31/3 – 3–1/3 then 3x3 + 9x is equal to 77. REASSURE
(a) 5 (b) 6 (a) COMFORT (b) CONSOLE
(c) 7 (d) 8 (c) INSPIRE (d) DISCOURAGE
67. Two circles touch each other internally. The radius of the 78. Four words are given, out of which only one word is spelt
smaller circle is 6 cm and the distance between the centre of correctly. Choose the correctly spelt word and click the
two circles is 3 cm. The radius of the larger circle is button corresponding to it.
(a) 7.5 cm (b) 9 cm (a) Defianse (b) Defyance
(c) 8 cm (d) 10 cm (c) Difiance (d) Defiance
y
o
u
rs
CGL (Tier-I) 2016 Solved Paper 2016 - 5

m
a
h
DIRECTIONS (Qs. 79-81) : In the following questions, one part and click the button corresponding to it. In case no improvement

b
of the sentence may have an error. Find out which part of the is needed, "No improvement" is the answer.

o
o
sentence has an error and click the button corresponding to it. 91. They have had a real good time.

b
If the sentence is free from error, click the "No error" option.

.w
(a) have had a (b) have had really
79. Too great a (A)/ variety of studies (B)/ distract the mind (C)/ (c) have had a really (d) No improvement

o
rd
No Error (D) 92. Electricity has modernized the march of modern events.

p
(a) A (b) B (a) revolutionised (b) developed

re
(c) C (d) D (c) created (d) No improvement

s
80. The severe cyclonic storm (A)/ has left behind (B)/ a trial of 93. The sales boy told the consumer not to touch the products

s
.c
misery (C)/ No Error (D) on display.

o
(a) A (b) B (a) buyer (b) shopper

m
(c) C (d) D (c) customer (d) No improvement
81. My sister has been (A)/ interested in medicine (B)/ ever 94. Please make it a point to send the letter at my address.
since she was a child (C)/ No Error (D) (a) on (b) to
(a) A (b) B (c) in (d) No improvement
(c) C (d) D 95. Sunil and I helped to sell tickets.
(a) we (b) us
DIRECTIONS (Qs. 82-84) : The sentences given with blanks
(c) me (d) No improvement
are to be filled with an appropriate word(s). Four alternatives
are suggested for each question. For each question, choose the DIRECTIONS (Qs. 96-100) : A passage is given with 5 questions
correct alternative. following it. Read the passage carefully and choose the best
answer to each question out of the four alternatives and click
82. It is __________ university of excellence.
(a) a (b) an the button corresponding to it.
(c) also (d) one To know language is to be able to speak it; even a child who does
83. They have not had much time for social activities and I not yet attend school can speak his or her language. In order to
have not __________ . speak a language it is important to listen to it and to read a few
(a) done so (b) either pages in it everyday. A child picks up language and learns to talk
(c) also (d) gone just as (s) he learns to walk. Walking and talking comes naturally
84. Food was packed, and they__________, a merry, delighted to a child as it grows. In our country, a child may grow up speaking
party. more than one language, if these languages are spoken in the
(a) went off (b) start off home and in the neighbourhood. we call this multilingualism. A
(c) cut off (d) set off child speaks a language or languages much before (s)he starts
going to school. To know a language then is first of all to be able
DIRECTIONS (Qs. 85-87) : In each of the questions, four to speak it as easily and naturally as a tree year old child does.
alternatives are given for the Idiom/Phrase. Choose the Later on the child will learn to read and write in that language. In
alternative which best expresses the meaning of the Idiom/Phrase order to read and write in a language, one has to speak it. But it is
and click the button corresponding to it. possible to speak a language but not able to read or write in it. A
85. pull a fast one baby does not speak until it is nine months old but it understands
(a) as fast as lightning a few words at six months of age. It has been listening ever since
(b) play a trick it was born, and even a little before that. So the first strategy in
(c) carry a heavy burden speaking a language is to listen.
(d) take a deep breath 96. One of the activities of a child before it is even born is
86. Grease the palm __________ .
(a) dirty one's hands (b) work in a garage (a) seeing (b) listening
(c) slip and fall (d) bribe (c) understanding (d) talking
87. turn turtle 97. It is necessary for one to __________ the language before
(a) slow like a turtle (b) turn like a turtle (s)he writes in that language.
(c) over-turn (d) a game turtles play (a) sing (b) spell
DIRECTIONS (Qs. 88-90) : Out of the four alternatives, choose (c) speak (d) none of the above
the one which can be substituted for the given words/sentences 98. Multi-lingualism means
and click the button corresponding to it. (a) speaking more than one language
88. Action that is likely to make people very angry (b) speaking only one language
(a) Inflationary (b) Inflammable (c) speaking any language
(c) Commensurable (d) Inflammatory (d) speech
89. A humorous drawing dealing with current events or politics. 99. A child has been __________ ever since it was born
(a) Sketch (b) Illustration (a) speaking (b) reading
(c) Cartoon (d) Skit (c) walking (d) listening
90. Act of mercy killing 100. To know a language one must be able to?
(a) Suicide (b) Euthanasia (a) Speak it as easily and naturally as a three year old
(c) Immolation (d) Asphyxiation child.
DIRECTIONS (Qs. 91-95) : A sentence/a part of the sentence is (b) Read it well all the time.
underlined. Four alternatives are given to the underlined part (c) Write it quickly
which will improve the sentence. Choose the correct alternative (d) Sing in the language
y
o
u
rs
2016 - 6 Combined Graduate Level Exam

m
a
h
Hints & Explanations

b
o
o
b
.w
1. (c) Medicine is given to patient similarly Education is given

o
16. (d) 7 10 5

rd
to student.

p
16 40 8

re
2. (a) L A M P
15 ? 9

s
s
–3 ¯ –3 ¯ –3 ¯ –3 ¯

.c
(7 – 5) × 5 = 10

o
I X J M

m
(16 – 8) × 5 = 40
(15 – 9) × 5 = 30
F I S H
–3 ¯ –3 ¯ –3 ¯ –3 ¯ 17. (c)
C F P E
3. (d) As 20 = 13 × 2 – 6
\ 28 = 17 × 2 – 6
4. (d) Ally is Antonym of Remaining three.
5. (d) Only FQMV is without any Vowel He is in north direction from starting point
6. (d) Difference between 36 – 48, 56 – 44 and
18. (b)
78 – 64 is 12 where as 33 – 64 is different.
7. (c) Arrangement as per Dictionary rules is 3, 2, 1, 5, 4
i.e Ambiguity, Ambiguous, Ambitions, Animation
8. (b)

Only Conclusion II follows.


19. (c)

20. (b)
9. (b) 5 × 2 + 1 = 11
11 × 2 + 2 = 24
24 × 2 + 3 = 51
51 × 2 + 4 =106
106 × 2 + 5 = 217
Delhi and Srilanka both are in Asia.
10. (a) 21. (a) 22. (b) 23. (d) 24. (a) 25. (a)
26. (b) Doctrine of Passive Resistance is authored by
Aurobindo Ghosh. It is based on a series of articles by
Aurobindo Ghosh which were published in April 1907
That man is son of Reeta in the journal Bande Matram. The articles were written
11. (b) Age of Son = x, Age of doctor = 4x when the Bengal was burning with indignation after its
4x + 10 = 2(x + 10) partition October 16, 1905 by Viceroy Curzon. The main
4x – 2x = 20 – 10 theme of the articles is the methods and ideology which
2x = 10 could be adopted in face of oppression and injustice
x= 5 as perpetrated by the contemporary British rule in India.
Age of Son = 5 year 27. (a) Dipa Karmakar returned empty handed from the Vaults
12. (a) CANCEL cannot be Form from final at the Gymnastics event of Rio 2016 Olympics
INTELLIGENCE after she finished the event at fourth rank.
13. (d) D A S H Þ 4 + 1 + 19 + 8 = 32 28. (c) 29. (c) 30. (a)
D A N C E Þ 4 + 1 + 14 + 3 + 5 = 27 31. (b) 32. (c) 33. (a)
14. (b) Putting + = – , – = ×; ÷ = – and × = ÷
34. (d) Lichens can be used asair pollution indicators,
in 5 – 3 + 10 × 5 ÷ 5
especially of the concentration of sulfur dioxide in the
Þ 15 × 3 – 10 ÷ 5 + 5
atmosphere. Lichens are plants that grow in exposed
45 – 2 + 5 = 48
places such as rocks or tree bark. They need to be very
15. (b) = 4 7 9, = 5 9 2, good at absorbing water and nutrients to grow there.
35. (c) Orology is the study of mountains.An example of
=965 =682 orology is research on how a particular mountain came
to be formed.
y
o
u
rs
CGL (Tier-I) 2016 Solved Paper 2016 - 7

m
a
h
36. (c) Soil salinity is the salt content in the soil; the process
Distance

b
of increasing the salt content is known as salinization.

o
57. (c) Time =
Speed

o
Salts occur naturally within soils and water. Salinization

b
.w
can be caused by natural processes such as mineral
200 + 175 375
weathering or by the gradual withdrawal of an ocean.

o
Time = 5 = 15

rd
37. (d) 38. (c) 39. (d) 54 ×

p
18

re
40. (b) It was built by the Khmer King Suryavarman II in the
= 25 sec

s
early 12th century.

s
58. (b) As only 13, 12 and 5 follows Pythagorous theorem

.c
41. (c) 42. (a) 43. (d)

o
59. (a)

m
44. (d) The Indian rupee sign is the currency sign for the Indian
60. (a) cos 53° – sin 37° Þ cos 53° – sin (90° – 53°)
rupee, the official currency of India. Designed by D.
Þ cos53° – cos53°
Udaya Kumar, it was presented to the public by the
=0
Government of India on 15 July 2010, following its
selection through an "open" competition among Indian 1
61. (b) 4x + =5
residents. x
45. (a) Micro Units Development and Refinance Agency Bank Þ 4x2 + 1 = 5x
(or MUDRA Bank) is a public sector financial
5x 5x
institution in India. 2
=
4x + 10x +1 10x + 5x
46. (d) 47. (d) 48. (b) 49. (b)
50. (c) Badruddin Taiyabji became the Ist Indian Barrister in 5x 1
Bombay; became the 2nd Indian Chief Justice; was the Þ =
15x 3
founding member of Bombay presidency association
and INC and also presided over the 3rd congress 1
62. (a) C+ = 3
session in Madras in 1887. C
9 2 Cubing both Sides
51. (d) Remaining work = 1– =
11 11 1
C3 + + 3( 3 ) = 3 3
C3
2
C will get = × 440 = 2× 40 = 80 1
11 C3 + =0
52. (b) Market Price = 7500, Discount = 6% C3
63. (b) To divide 451 * 603 by 9
94 (4 + 5 + 1 + * + 6 + 0 + 3) = (19 + *)
Selling Price = × 7500 = 7050 `
100 (19 + *) must be multiple of 9
53. (b) Let the number be 3x and 4x \ 19 + * = 27
(3x)3 + (4x)3 = 5824 *=8
27x3 + 64x3 = 5824 64. (b) Average of 9 observations = 35
91x3 = 5824 Total = 9 × 35 = 315
x3 = 64 Misread Difference = 81 – 18 = 63
x =4 New total = 315 + 63 = 378
Sum of numbers are = x(4 + 3) = 4× 7 = 28 378
Average = = 42
54. (c) Let Selling Price = 100 Loss = 20% 9
Cost price = 120 Shortcut Method:
81 – 18 = 63
20 2
Loss% of cost price = ×100 = 16 %
120 3 63
\ =7
9
5 10
55. (a) Total deffective part = ×120 + × 80 35 + 7 = 42
100 100
= 6 + 8 = 14 1
65. (b) x3 + y3 + z3 – 3xyz = (x + y + z)
2
14
Deffective % = ×100 = 7% [(x – y)2 + (y – z)2 + (z – x)2]
200
1
56. (d) (23)2 = 4x = (222 + 223 + 225)
26 = 22x 2
6 = 2x [(222 – 223)2 + (223 – 225)2 + (225 – 222)2]
x = 3 1
Þ [670][1 + 4 + 9] = 335 [14] = 4690
33 = 27 2
y
o
u
rs
2016 - 8 Combined Graduate Level Exam

m
a
h
31/3 – 3–1/3
= 50 (1+ 3 )
66. (d) x =

b
Cubing on both sides

o
o
= 50 (2.73) = 136.5 m

b
1 1 1
x3 = 3 – – 3(3) - éë31 3 – 3–1 3 ùû

.w
71. (d) Let radius and height of cone = 100
3 3 3 New radius and height of cone = 120

o
rd
1 120 6
x3 = 9 – – 3(x)

p
Ratio = =

re
3 100 5

s
s
9 –1

.c
3
x3 + 3x = æ 6 ö 216

o
3 Volume Ratio = ç ÷ =

m
3x3 + 9x = 8 è 5 ø 125
67. (b) O¢O = 3 Difference = 216 – 125 = 91
OA = 6 91 4
Difference % = ×100 = 91×
125 5
O¢A = 6 + 3 = 9 cm
364
68. (b) In Given equilateral D, MN || QR = = 72.8%
5
PN MN P
= 8200 + 6000 + 8600 + 7500 + 8800 + 6500
PQ QR 72. (b) Average =
M N 6
PN = MN Q (PR = QR) = 7600
MN = 6 cm 73. (b)
Q R 74. (a) Highest peak = 8800
1 5 lowest peak = 6000
69. (b) cosecθ + =
cosec θ 2 8800
Ratio = = 22 : 15
Squaring on both sides 6000
1 25 75. (d) Ascending order 6000, 6500, 7500, 8200, 8600, 8800
cosec2 θ + +2=
cosec2 θ 4 7500 + 8200
Middle two average =
1 25 2
cosec2θ + + 2–4= –4
cosec2 θ 4 15700
= = 7850 m
1 169 2
cosec 2 θ + –2 =
cosec 2 θ 4 76. (d) Credulous- having or showing too great a readiness
to believe things.
169 Gullible- easily persuaded to believe something.
(cosec q – sin q) =
4 77. (d) Reassure-say or do something to remove the doubts
165 and fears of (someone).
(cosec° q – sin° q) = Discourage- cause (someone) to lose confidence or
4
enthusiasm.
3 78. (d) Here variety, the main nation, is Singular in form. If the
cosec θ – sin q =
2 writer intends it for a collective Noun, he may
70. (a) In D ADC reasonably use the Plural distract. But it seems better
AC = 50 m to treat variety as a quality and therefore really Singular,
in which case the verb should be distract. At all events
AC the Plural studies, being only part of a secondary Noun-
= tan30°
DC phrase, does not excuse the Plural distract.
50 1 79. (c) 80. (b) 81. (d)
= 82. (a) 83. (b) 84. (d)
DC 3
85. (b) pull a fast one- to succeed in an act of deception.
D C = 50 3 86. (d) grease somebody's palm also grease the palm of
in D ABC somebody- to give someone money to persuade them
to do what you want.
AC
= tan45° 87. (c) turn turtle- to turn upside down.
BC
88. (d) 89. (c) 90. (b) 91. (b) 92. (a)
BC = 50
93. (c) 94. (b) 95. (d) 96. (b) 97. (c)
BD = 50 + 50 3 98. (a) 99. (d) 100. (a)
y
o
u
rs
m
a
SSC

h
Held on 25-10-2015

b
o
o
b
.w
o
rd
p
re
Combined Graduate Level (Tier-II) Exam-2015

s
s
.c
o
m
Time : 4 Hours Max. Marks : 400

ENGLISH LANGUAGE & COMPREHENSION (c) No error


(d) about the political situation while you were in that
DIRECTIONS (Qs. 1-20) : In the following Twenty questions, country?
some parts of the sentences have errors and some are correct, 10. I was convinced that the only thing I wanted to do ever, is
Find out which part of a sentence has an error and blacken the to write novels.
circle [ ] corresponding to the appropriate correct option. If a (a) the only thing I wanted to do
sentence is free from errors, ‘No error’ in the question. (b) I was convinced that
1. He looked upon me eye to eye for a few moments before he (c) ever, is to write novels
spoke. (d) No error
(a) before he spoke. 11. The heavy-weight fighter was knocking out in one punch.
(b) No error (a) No error
(c) eye to eye for a few moments (b) The heavy-weight fighter
(d) He looked upon me (c) was knocking out
2. Until you are in the habit of putting of things you cannot (d) in one punch.
ensure yourself of a good future. 12. There be a long queue for entry into the exhibition ground.
(a) you cannot ensure yourself of a good future (a) No error (b) the exhibition ground
(b) the habit of putting off things (c) There be a long queue (d) for entry into
(c) Until you are in 13. Riches are uncertain things, and the fame that power give
(d) No error or is won by foul means is as short-lived as the dewdrop.
3. I meant to repair the radio but hadn’t time to do it today, (a) No error
(a) but hadn’t time (b) No error (b) Riches are uncertain things, and the fame
(c) to do it today. (d) meant to repair the radio (c) that power give or is won by foul means
4. This book is different than that (d) is as shortlived as the dewdrop
(a) This book (b) is different 14. The results of the survey were fairly disappointing.
(c) No error (d) than that (a) The results (b) fairly disappointing
5. The doctor asked his patient to regularly take his medicine. (c) No error (d) of the survey were
(a) The doctor 15. The article suggests that when a person is under unusual
(b) No error stress you should be especially careful to eat a well-
(c) asked his patient balanced diet.
(d) to regularly take his medicine (a) The article suggests that when a person is under
6. Do you think you could lend me good pair of gloves to
unusual stress
wear to the wedding?
(b) to eat a well-balanced diet.
(a) Do you think you could lend me
(c) No error
(b) No error
(d) you should be especially careful
(c) good pair of gloves
16. Some members of the committee were opposed to use the
(d) to wear to the wedding?
7. I forbid you not to go there again. club members’ money to redecorate the meeting hall.
(a) there again (b) not to go (a) Some members of the committee
(c) No error (d) I forbid you (b) No error
8. In twenty wrecks five lives were fortunately lost. (c) were opposed to use
(a) In twenty wrecks (b) five lives were (d) the club members’ money to redecorate the meeting
(c) No error (d) fortunately lost hall.
Please note : The sentence mentions twenty wrecks. This 17. Thanks to an insight and persistence of the local doctor,
is incorrect as wreckage can’t be counted. This could also hundreds of victims have been able to resume normal life.
be a possible error. (a) No error
9. Did you hear many news about the political situation while (b) Thanks to an insight
you were in that country? (c) hundreds of victims have been able to resume normal
(a) Did you hear life.
(b) many news (d) and persistence of the local doctor
y
o
u
rs
2015-2 – 2 Combined Graduate Level Exam

m
a
h
18. We received much help from the neighbours. and named P, Q, R and S. These four parts are not given in their

b
(a) from the neighbours (b) much help proper order. Read the sentence and find out which of the four

o
o
(c) We received (d) No error combinations is correct and indicate it by blackening the

b
19. Let us vie to one another in doing good. appropriate circle [ ] in the Answer Sheet.

.w
(a) No error (b) Let us vie

o
33. 1. Our pleasures should be healthy so that they can

rd
(c) to one another (d) in doing good. impart a sense of well-being.

p
20. As per the invitation card Rahim marries with Sayra on 13th P. This applies very much to the passion for sports.

re
December, Monday. Q. Some people become slaves to an enthusiasm and

s
s
(a) No error regard it as their real and only pleasure in life.

.c
(b) On 13th December, Monday R. It is quite possible that indulging this passion is doing

o
m
(c) As per the invitation card them great harm.
(d) Rahim marries with Sayra S. Modern sports have become so exaggerated that they
DIRECTIONS (Qs. 21-22) : In the following questions, out of can damage and sometimes destroy one’s health
the four alternatives, choose the one which can be substituted 6. An enthusiasm for violent sports may well dig an early
for the given words/sentences and indicate it by blackening the grave for the participant.
appropriate circle in the Answer Sheet. (a) QPRS (b) QSPR
21. An error or misprint in printing or writing (c) QRPS (d) QRSP
(a) axiom (b) erratum 34. 1. The Pyramids are beautiful enormous structures.
(c) jargon (d) agenda P. A mummy is the dead body of a human being to which
22. The period between two reigns oils and spices have been applied to prevent it from
(a) era (b) interregnum decaying.
(c) intermission (d) anachronism Q. They are the tombs of the old kings of Egypt who were
23. Too willing to obey other people called the Pharaohs.
(a) subjugate (b) subservient R. These mummies were placed inside these great
(c) sublimate (d) subaltern Pyramids.
24. A person extremely desirous of money S. The bodies of the Pharaohs were made into mummies
(a) avaricious (b) fervent when they died
(c) extravagant (d) miser 6. Near them, Were placed, gold, silver, food, furniture
25. To destroy completely and other things because it was believed that the
(a) chide (b) annihilate mummies might require them after death.
(c) dawdle (d) bluster (a) SPRQ (b) QPRS
26. One who is known widely but usually unfavourable is (c) QSPR (d) QSRP
(a) notorious (b) famous 35. 1. One of the gifts of independence is the awakening of
(c) illustrious (d) tarnished women of our country.
P. Besides, their talent is recognized and they are
27. A person who is very selective, disgusted easily, and is appointed to high posts in the State.
hard to please. Q. Free India has seen women as Governors, Ministers
(a) selector (b) misanthrope and Ambassadors.
(c) ambiguous (d) fastidious R. That is because our government is making efforts to
raise their status.
28. A person who has had one or more limbs removed S. Women have a bright future in independent India.
(a) limber (b) amputee 6. We even had a woman Prime Minister.
(c) fatalist (d) handicap (a) QPSR (b) QSPR
(c) SPQR (d) SRPQ
36. 1. In a fact, a robotic exoskeleton device has enabled a
29. A song sung at the death of a person
39-year old former athlete, who had been completely
(a) liturgy (b) elegy
paralysed for four years.
(c) sonnet (d) ode
P. This is the first time that a person with chronic,
30. Rub or wipe out
complete paralysis has regained enough voluntary
(a) remove (b) terminate
control to actively work with a robotic device.
(c) efface (d) plunder
Q. The athlete’s leg movement also resulted in other
31. No longer in existence or use
health benefits.
(a) obsolete (b) invincible R. To control his leg muscles and take thousands of
(c) delete (d) inefficient steps.
32. A statement in which you say the same thing twice in S. In addition to the device, the man was aided by a
different words novel non-invasive spinal stimulation technique that
(a) tautology (b) temerity does not require surgery .
(c) repetition (d) tarragon 6. Including improved cardio vascular function and
DIRECTIONS (Qs. 33-52): In the following Twenty questions, muscle tone.
the 1st and the last part of the sentence passage are numbered 1 (a) SPQR (b) RPSQ
and 6. The rest of the sentence passage is split into four parts (c) QRPS (d) PQRS
y
o
u
rs
Combined Graduate Level Exam 2015-2 – 3

m
a
37. 1. Mandela led the battle of freedom against slavery.

h
43. 1. Technology is meant for bringing comfort to the body

b
P. The way was fighting with non-violence and truth. and spirituality brings comfort to the mind.

o
o
Q. He fought it in a unique way. P. But in India that never happened, religion has always

b
R. This struggle brought the racists down to the ground. encouraged Science.

.w
S. Many nations got their freedom in this way. Q. Scientists were persecuted.

o
6. But some nations still wouldn’t get the desired freedom.

rd
R. Here religion and Philosophy were never isolated
(a) SPQR (b) PQRS

p
practices.

re
(c) QPRS (d) RSPQ
S. In the West, religion was always opposed to Science.

s
38. 1. India is rapidly coming under the influence of western

s
6. They intertwined with life, in music, art, drama, dance,

.c
culture.
painting and sculpture.

o
P. Earlier India had its own moral and social values.

m
Q. This change can be seen in our education system, (a) SRQP (b) PRSQ
marriages, food habits and daily routine. (c) SQPR (d) PQSR
R. But today they have changed enormously, 44. 1. The world leaders,
S. The effect of westernization is visible everywhere in P. That the despotic regime will try to get.
India Q. by unleashing cruder violence that may come
6. It may be harmful to forget our culture and values R. gathered in the peace-conference, were unanimous in
completely. voicing their fears
(a) QSRP (b) PSRQ S. over its palpable sense of insecurity
(c) PQSR (d) SPRQ 6. In the form of the use of chemical weapons.
39. 1. The world is stunned with the news that 44 year old (a) RPSQ (b) SQRP
Steeve Irwin is no more. (c) QPRS (d) PQSR
P. He was shooting in the Great Barrier Reef of 45. 1. The Cinema is a very valuable teaching aid.
Queensland. P. The cinema is entertaining also.
Q. He is killed by the poisoned barb of a huge sting ray.
Q. Historical films help the teacher of History.
R. The incident took place at about 11 a.m.
S. He was shooting an underwater documentary film. R. Science can also be taught very effectively through
6. Irwin was brought to the surface unconscious. Cinema.
(a) RSPQ (b) SPQR S. Large number of people like social films.
(c) PRSQ (d) QSPR 6. Thus Cinema has a great impact on our society.
40. 1. Designing is as natural to me as breathing (a) SQRP (b) RQSP
P. It was then that I launched my own label. (c) QRPS (d) PQRS
Q. It’s been a good season so far. 46. 1. Grandpa had some old clothes.
R. But my carrer took off only after the birth of my P. But my mother took them out and kept them neatly
second child . folded in the cupboard again.
S. Right now, I’m busy with my first store. Q. So he threw them into the waste basket.
6. Next month, I’ll be going to New York. R. So he put the clothing into the family’s bag of items to
(a) PSQR (b) RPSQ donate to charity.
(c) SQRP (d) QRSP S. My mother found them and put them back in his
41. 1. Katherine Mansfield was born in Wellington, New basket.
Zealand
6. Grandpa finally put the items in my mother’s mending
P. In 1908 she went back to the London which she felt to
be her spiritual home. basket and never saw them again.
Q. She was sent to Queen’s College School, London in (a) SRPQ (b) QSRP
her fifteenth year to finish’ her education (c) PQSR (d) RSQP
R. She remained there for four years. 47. 1. Children like to celebrate their birthday.
S. Soon after returning to New Zealand, she became P. In addition to birthday presents, they also receive
dissatisfied. greetings.
6. She hoped to make a literary carreer there. Q. It provides an opportunity for them to enjoy
(a) PQRS (b) PSRQ themselves with their friends.
(c) QRSP (d) RSQP R. The birthday presents also add to their joy.
42. 1. A couple in my neighbour is known for shouting at S. Nobody can deny that company of friends is joyful.
each other 6. All these factors make birthday worth celebrating.
P. With an apprehension of a serious fight between the (a) SRPQ (b) RPQS
two I went closer to the door and peeped in. (c) QSRP (d) QRSP
Q. I intended to calm them down. 48. 1. Rola Sleiman parked her car in the only empty spot
R. I could guess both of them lay peacefully in their bed. outside Tripoli’s evangelical church.
S. Last night at about 11 O’clock I could hear shrieks and
P. But that’s not the only thing makes her unique.
sounds.
6. The fight was going on in a T.V. Serial on a channel. Q. She’s a pastor, and at 37, she’s younger than most of
(a) RPSQ (b) SPQR her colleagues.
(c) PSRQ (d) RPQS R. It is a small sand-coloured building with a simple
Please note: Statement Q is unnecessary in the sequence façade and large wooden doors in the middle of the
and can be done away with. Choice (b) city.
y
o
u
rs
2015-2 – 4 Combined Graduate Level Exam

m
a
h
S. It was Sunday morning and, like every Sunday at (c) breath taking enough to be unreal .

b
around this time, Rola was headed to work. (d) very realistic and unbelievable.

o
o
6. Rola is also, as far as she knows, the only female 54. The old car Thumbed along over the story road.

b
pastor in Lebanon - and perhaps even in the entire

.w
(a) chugged (b) No improvement
Middle East. (c) creaked (d) crawled

o
rd
(a) SRPQ (b) RSQP 55. Honesty is more superior than riches.

p
(c) SQPR (d) PRSQ (a) more superior to (b) superior to

re
49. 1. Researches say that jogging alone is unhealthy. (c) far superior than (d) No improvement

s
s
P. It was found that communal joggers have double the 56. What does agonise me most is not this criticism, but the

.c
number of brain cells as solo runners.

o
trivial reason behind it.

m
Q. These positive effects are suppressed when running (a) most agonising me (b) No improvement
occurs in isolation. (c) most agonised me (d) agonises me most
R. Experiments indicated that running alone stifles brain 57. A hand pump is very easy to work and it can be fitted in
cell regeneration. every house.
S. Experienced in a group, running stimulates brain cell (a) can be fitting (b) No improvement
growth.
(c) can be fits (d) could be fitting
6. However, joggers around the world should remember
58. I was greatly shocked by the most ruthless murder.
that jogging is healthier than the rat race.
(a) gruesome (b) No improvement
(a) RPSQ (b) RQPS
(c) SPQR (d) PQSR (c) shocking (d) loathsome
50. 1. The heart is the pump of life. 59. If the room had been brighter. I would have been able to
P. They have even succeeded inherat transplants. read for a while before going to bed.
Q. Now-a-days surgeons are able to stop a patient’s (a) Had the room been brighter
heart and carry out complicated operations. (b) If the room was brighter
R. A few years ago, it was impossible to operate on a (c) No improvement
patient whose heart was not working properly. (d) If the room are brighter
S. If the heart stops we die in about five minutes. 60. Listen attention to what I say.
6. All this was made possible by the invention of the (a) attentive (b) attend
heart lung machine. (c) attentively (d) No improvement
(a) QRSP (b) RSQP 61. They have not and cannot be in the good books of the
(c) PQRS (d) SRPQ coach because they lack discipline.
51. 1. Nalanda became India’s famous centre of education. (a) have not and can never been
P. Ten thousand Buddhist monks used to live there. (b) have not and can never be
Q. It is situated near the town of Bihar Sharif. (c) No improvement
R. The ruins of Nalanda can still be seen. (d) have not been and can never be
S. It was visited by the Chinese Pilgrim Hiuen Tsang. 62. Sincerity is always appreciated
6. He stayed there for several years. (a) No improvement (b) enjoyed
(a) QPSR (b) RPSQ (c) enhanced (d) waited
(c) RQPS (d) SPQR 63. The Criminal was hung to death
52. 1. Once upon a time I went to Scotland. (a) hang (b) hanged
P. I found my flesh creep as I Walked down its sinister (c) hunged (d) No improvement
corridor. 64. He has composed a beautiful song.
Q. There, in a castle in the dark, misty highlands, actually (a) jotted down (b) No improvement
a modest hotel in Edinburgh. (c) written (d) penned
R. and the next morning he’d been found with (his throat
65. This matter admits no excuse.
slit)
(a) No improvement (b) admits to
S. We’d had dinner with Jock Mc Arthur only the
(c) admits of (d) admits for
previous night.
66. No sooner had he reaching the station than the train began
6. The chill finger of suspicion pointed at all of us.
(a) QPSR (b) SRQP to move.
(c) PQSR (d) QRSP (a) No sooner had he reach the station than the train
DIRECTIONS (Qs. 53-74) : In the following Twenty Two begins to move
questions, a part of the sentence sentence is underlined. Below (b) No sooner did he reaching the station than the train
are given alternatives to the underlined part which may began to move
improve the sentence. Choose the correct alternative. In case (c) No sooner had he reached the station than the train
no improvement is required, choose “No Improvement” option. began to move
Mark your answer in the Answer Sheet. (d) No improvement
53. He was a failure at art but his last piece was so beautiful (c)
that no one could believe he had painted it. 67. My father has been advised to reduce smoking.
(a) indeed a masterpiece (a) lower down (b) No improvement
(b) No improvement (c) cut down (d) reduce down
y
o
u
rs
Combined Graduate Level Exam 2015-2 – 5

m
a
h
68. The Louvre, a museum known to everyone in the world, is 78. “What an excellent piece of art” she said.

b
in Paris. (a) She exclaimed with surprise was it an excellent piece of

o
o
(a) No improvement art.

b
(b) She had exclaimed that it was an excellent piece of art.

.w
(b) a world-renowned museum
(c) a world-famous museum (c) She exclaimed what an excellent piece of art was it.

o
(d) She exclaimed with appreciation that it was an

rd
(d) a globally known museum
excellent piece of art.

p
69. My sister does not know whether tomorrow can be a

re
79. Dr. Ratan said to the patient, “Take light food and do not go
holiday for her.

s
out in the Sun.”

s
(a) whether tomorrow is a holiday to her

.c
(a) Dr. Ratan advised the patient to take light food and

o
(b) No improvement not go out in the sun.

m
(c) whether tomorrow is a holiday for her (b) Dr. Ratan advised the patient to take light food and to
(d) whether tomorrow will be a holiday for her. do not go out in the sun.
70. The sun will not rise before an hour (c) Dr. Ratan advised the patient to take light food and
(a) arise in an hour (b) rises for an hour not go out in the Sun.
(c) No improvement (d) rise in an hour (d) Dr. Ratan advised the patient to take light food and do
71. Many a man have died of cancer. not go out in the Sun.
(a) has died of (b) No improvement 80. Rajiv said to me, “He plays with right hand.”
(c) have been dying of (d) have died from (a) Rajiv said that he played with right hand.
(b) Rajiv told me that he could play with right hand
72. I want to admit in a university in the US.
(c) Rajiv told me that he plays with right hand
(a) enroll (b) No improvement (d) Rajiv asked if he could play with right hand.
(c) enter (d) go 81. “Oh I how foolish I have been in money matters!”
73. Heartfelt prayers to God will always have expected results. (a) He expressed with regret that he could be so foolish in
(a) have desired results money matters.
(b) have good results (b) He confessed regretfully that he had been very
(c) have amazing results foolish in money matters.
(d) No improvement (c) He confessed to himself that he is being very foolish
74. She is annoying that her father had not accepted her in money matters.
suggestion. (d) He exclaimed regretfully that he was very foolish in
(a) had to annoy (b) was to annoy money matters.
(c) was annoyed (d) No improvement 82. I said to my friend, “Can I borrow your dictionary for one
day only?”
DIRECTIONS (Qs. 75-101) : In the following Twenty seven (a) I asked my friend if I might borrow his dictionary for
questions, a sentence has been given in Direct/indirect Speech. one day only.
Out of the four alternatives suggested, select the one which best (b) I told my friend I could borrow his dictionary for one
expresses the same sentence in Indirect/Direct Speech and day only.
mark your answer in the Answer sheet. (c) I asked my friend if I could borrow his dictionary for
75. Vipul said, “Alas! How thoughtless I have been!” one day only.
(d) I requested my friend if I can borrow his dictionary for
(a) Vipul exclaimed how thoughtless I have been
one day only.
(b) Vipul regretted upon my thoughtlessness. 83. I’ll say to my friends, “I have started learning computer”.
(c) Vipul confessed with regret that he had been (a) I’ll tell my friends that I learn computer.
thoughtless. (b) I’ll tell my friends that I am learning computer.
(d) Vipul admitted that he had been thoughtless. (c) I’ll tell my friends that I have started learning
76. The officer said, “Cowards! you shall soon be put to computer.
death.” (d) I’ll tell my friends that I had started learning computer.
(a) The officer said that they were cowards and should be 84. The commander said, “Let the gates be left open.”
put to death soon. (a) The commander ordered that the gates will be left
(b) The officer called them cowards and said that they open.
would soon be put to death. (b) The commander ordered for the gates to be left open.
(c) The officer called them cowards and said that they (c) The commander ordered that let the gates be left open.
(d) The commander wanted that the gates be left open.
must be put to death soon.
(b)
(d) The officer said that they would soon be put to death. 85. I said to my mother. “Please warm this milk. Don’t boil it”.
77. My father said to me. “Will you sit and study at least now.” (a) I requested my mother to warm that milk and asked her
(a) My father asked me if I will sit and study at least then not boil it.
(b) My father enquired me if I would sit and study at least (b) I asked my mother that she may warm that milk but not
now boil it.
(c) My father asked me whether I can sit and study at (c) I requested my mother that she warm that milk and not
least then. to boil it.
(d) My father asked me if I would sit and study at least (d) I told my mother respectfully that she will warm that
then. milk and not to boil it.
y
o
u
rs
2015-2 – 6 Combined Graduate Level Exam

m
a
h
86. “They are late” she has already told us. 95. “Sam, will you help me carry my bags?” Asked the old

b
(a) She has already told us (that) they are late. woman.

o
o
(b) She told us that they are late already. (a) The old woman told Sam to helped her carry her bags.

b
(c) She told us that they are already late. (b) The old woman asked Sam whether he would help her

.w
(d) She has already been telling us that they are late. carry her bags.

o
rd
87. He said to me, “Where are you going?” (c) The old woman said to Sam to help her carried her

p
(a) He asked that where I was going bags.

re
(b) He asked me where I was going (d) The old woman asked to Sam to help her carry her

s
s
(c) He said me to where I was going bags.

.c
(d) He said to me that where I was going 96. The teacher said to his student, “Do you have any excuse

o
m
88. The teacher said to me, “I hope you will bring credit to my for coming late to the school, today?”
school.” (a) The teacher asked his student whether he had any
(a) The teacher hoped that I would bring credit to his excuse for coming late to the school that day.
school (b) The teacher asked his student whether did he have
(b) The teacher desired that I bring credit to his school. any excuse for coming late to the school that day.
(c) The teacher said that I will bring credit to his school. (c) The teacher asked his student whether he has any
(d) The teacher wished that I would bring credit to my excuse for coming late to the school that day.
school. (d) The teacher asked his student whether he did have
89. “There is no need for you to be so angry I” said Mrs. Sen to any excuse for coming late to the school that day.
her husband. 97. The boy thanked the Principal for granting him a fee-
(a) Mrs. Sen told to her husband that there was no need concession.
for his anger. (a) The boy said to the Principal, “I express my thanks
(b) Mrs. Sen told her husband to not be so angry. that you have granted me a fee-concession.”
(c) Mrs. Sen told her husband that there was no need for (b) The boy said to the Principal, “I am thankful to you for
him to be so angry. a fee-concession.”
(d) Mrs. Sen told her husband that no need was there to (c) The boy told the Principal, “You are kind enough to
grant me a fee-concession.”
be so in anger.
(d) The boy said to the Principal, “Thank you, madam, for
90. He wished me respectfully and told that he had come to join
granting me a fee-concession.”
the office.
98. The teacher said to him, “Do not go out.”
(a) He said to me, “Good morning, Sir! I’m here to join the
(a) The teacher commanded him that he should not go
office.”
out.
(b) He said to me, “Good morning, Sir! have come to join
(b) The teacher told him do not go out.
the office.” (c) The teacher ordered him to not to go out.
(c) He said to me, “Good morning Sir! I’ll join the office (d) The teacher advised him not to go out.
today”. 99. He asked me if I would like to have lunch with him that day.
(d) He said “Good morning Sir! I want to join the office.” (a) He said to me, “You can if you like to have lunch with
91. I said, “Good-bye, my beloved friends.” me today.”
(a) I wished my beloved friends good-bye. (b) He said to me, “Would you like to have lunch with me
(b) I bade my beloved friends good-bye. today?”
(c) I said good-bye to my beloved friends. (c) He said to me, “can you have lunch with me today?”
(d) I bided good-bye to my belowed friends. (d) He said to me, “Do you like to have lunch with me
92. They said, “We were living in Paris.” today?”
(a) They said they would have lived in Paris. 100. Rahul said, “Yes I am mistaken”
(b) They told me they had lived in Paris. (a) Rahul admitted that he was mistaken.
(c) They told me that they had been living in Paris. (b) Rahul said that he was on mistake.
(d) They said they might have been living in Paris. (c) Rahul admitted that I was mistaken.
93. The teacher said, “Don’t come to school on Monday (d) Rahul said that he is mistaken.
because it is a holiday.” 101. The guide said to the swimmer, “Don’t swim out too far.”
(a) The teacher asked us don’t come to school on (a) The guide told the swimmer to not swim out too far.
Monday because it is a holiday. (b) The guide suggested the swimmer that don’t swim out
(b) The teacher told us don’t come to school on Monday too far.
because it is a holiday. (c) The guide asked the swimmer not swim out too far.
(c) The teacher asked if we were coming to school on (d) The guide forbade the swimmer to swim out too far.
Monday because it is a holiday. (d)
(d) The teacher told us not to come to school on Monday DIRECTIONS (Qs. 102-111) : Read the passage carefully and
because it is a holiday. choose the correct answer to each question out of the four
94. He said to his father, “Shall I go to market now?” alternatives and fill in the blanks.
(a) He asked his father if he would go to market now. Passage (Ten questions)
(b) He asked his father if he shall go to market now. Every month scientists (i) new gadgets and new ways to make
(c) He asked his father if he will go to market then. technology faster and better. Our homes are full of hardware
(d) He asked his father if he should go to market then. (such as DVD players and computers) (ii) and (such as computer
y
o
u
rs
Combined Graduate Level Exam 2015-2 – 7

m
a
h
games and Mp3s) (iii) suggests, however, that it is the young 113. (v) = ?

b
people who are best able to deal with this change. Whereas (a) revolving (b) involving

o
o
teenagers have no problem (iv) a DVD player, their parents and (c) employing (d) devolving

b
grandparents often find using new technology (v) and different. 114. (vi) = ?

.w
But if you’re a teenager who criticizes your parents for their (vi) (a) fussy (b) finicky

o
rd
of technological awareness, don’t be too hard on them? (c) stingy (d) fastidious
115. (ii) = ?

p
Sometime (vii) the future, when you’ve got children of your own,

re
your (viii) to deal with new technology will probably (ix) and your (a) concurred (b) exposed

s
(c) interred (d) revealed

s
children will feel more (x) with new technology then you do.

.c
102. (v) = ? 116. (iii) = ?

o
(a) ground (b) deliberation

m
(a) unique (b) automatic
(c) dilemma (d) content
(c) sudden (d) complicated
117. (ix) = ?
103 (i) = ? (a) presume (b) attract
(a) estimate (b) found (c) repulse (d) distract
(c) discover (d) invent 118. (iv) = ?
104. (vii) = ? (a) restored (b) revised
(a) to (b) in (c) remarked (d) recommended
(c) at (d) on 119. (viii) = ?
(b) (a) unsuspected (b) unnerving
105. (x) = ? (c) unexpected (c) unfortunate
(a) able (b) pleased 120. (i) = ?
(c) comfortable (d) easy (a) employed (b) maintained
106. (iii) = ? (c) circulated (d) conducted
(a) experiment (b) research 121. (x) = ?
(c) industry (d) program (a) disturbances (b) dilemmas
107. (ii) = ? (c) discomfort (d) considerations
(a) gadgets (b) screens DIRECTIONS (Qs. 122-141) : In the following Twenty
(c) laptops (d) software questions, a sentence has been given in Active/Passive Voice.
108. (vi) = ? Out of the four alternatives suggested, select the one which best
(a) effect (b) amount expresses the same sentence in “Passive/Active Voice and mark
(c) lack (d) storage your answer in the Answer Sheet.
109. (viii) = ? 122. Did he plan an excursion to mountains?
(a) talent (b) ability (a) Was an excursion to mountains planned by him?
(c) possibility (d) master (b) Had an excursion to mountains planned by him?
110. (iv) = ? (c) Is an excursion to mountains planned by him?
(d) His an excursion to mountains planned by him?
(a) operating (b) inventing
123. That too many cooks spoil the broth is known to all.
(c) discovering (d) explaining
(a) All know that too many cooks spoil the broth.
111. (ix) = ? (b) All knew that too many cooks spoilt the broth.
(a) please (b) easy (c) All know that the broth is spoilt by too many cooks.
(c) decrease (d) able (d) All knows that the broth is spoilt by too many cooks.
DIRECTIONS (Qs. 112-121) : Read the passage carefully and 124. The Swiss regarded him as an imposter and called him a
choose the correct answer to each question out of the four Villain.
alternatives and fill in the blanks. (a) He was regarded as an imposter and was called a
Villain by the Swiss.
Passage (Ten questions)
(b) He is regarded as an imposter and was called a Villain
In a survey (i) by a library, it was recently (ii) that parents wish
by the Swiss.
their children to read books with high moral (iii). Around two (c) He was regarded as an imposter and is called a Villain
thousand parents were interviewed and most of them (iv). by the Swiss.
Dickens’ ‘The Christmas Carol’ as a must-read for children. This (d) He has been regarded as an imposter and called a
Christmas tale (v) the filthy rich but (vi) Scrooge and the poor but villain by the Swiss.
contented Cratchit family offers lessons in moral duties. Another 125. Ratan is performing an experiment.
book which many parents marked out as a (vii) read was Austen’s (a) Experiments were performed by Ratan.
“Pride and Prejudice”. The choice of this book was rather (viii), (b) An experiment was performed by Ratan.
because, this romantic novel is more likely to (ix) teenagers, than (c) An experiment is performed by Ratan.
children. Since Elizabeth’s final choice of Darcy is deeply rooted (d) An experiment is being performed by Ratan.
in strong moral (x), the parents, probably thought, she offers a 126. We shall write a novel.
good example for the girl child to follow. (a) A novel will have been written by us.
112. (vii) = ? (b) A novel will be written by us.
(a) condusive (b) reconciliatory (c) A novel would be written by us.
(c) mandatory (d) credible (d) A novel is written by us.
y
o
u
rs
2015-2 – 8 Combined Graduate Level Exam

m
a
h
127. Sita learns her lessons daily. 137. These days most of the students do their assignments with

b
(a) Her lessons is learnt daily by Sita. the help of the interest.

o
o
(b) Her lessons are learnt daily by Sita. (a) These days the assignments are being done with the

b
(c) Her lessons was learnt daily by Sita. help of the internet by most of the students.

.w
(d) Her lessons were learnt daily by Sita. (b) These days the assignments are being done with the

o
rd
128. Who tore the curtains yesterday? help of the internet by most of the students.

p
(a) By whom were the curtains teared yesterday? (c) These days most of the assignments are done with the

re
(b) By whom were the curtains torn yesterday? help of the internet by the students.

s
s
(c) By whom were the curtain tore yesterday? (d) These days most of the students are doing their

.c
(d) By whom are the curtains torn yesterday? assignments with the help of the internet.

o
m
129. We prohibit smoking 138. Our vehicle had been stolen from our garage by the same
(a) Smoking will be prohibited. burglary team.
(b) Smoking is prohibited. (a) The same burglary team had stolen our vehicle from
(c) Smoking has been prohibited. our garage.
(d) Smoking is being prohibited. (b) The same burglary team has stolen our vehicle from
130. The judge ordered the murdered to be hanged. our garage.
(a) The murdered was ordered by the judge to be hanged. (c) The same burglary team is stealing our vehicle from
(b) The murdered was to be hanged according to the our garage.
order of the judge. (d) The same burglary team will have stolen our vehicle
(c) The order was given by the judge to hang the from our garage.
murderer. 139. Who gave you the money?
(d) It was ordered by the judge to hang the murderer. (a) By whom was you given the money?
131. I shall be obliged to go. (b) By whom were you given the money?
(a) Circumstances could oblige me to go. (c) Who gave the money to you?
(b) Circumstances may oblige me to go. (d) Who was given the money by you?
(c) Circumstances would oblige me to go. 140. The situation could hardly have been altered by the
(d) Circumstances will oblige me to go. interference of the police force.
132. I didn’t speak even a single word in the meeting. (a) The interference of the police force could hardly have
altered the situation.
(a) Even a single word was not spoken by me in the
(b) The interference of the police force can hardly alter
meeting.
the situation.
(b) Even a single word has not been spoken by me in the
(c) The interference of the police force had altered the
meeting.
situationl
(c) Even a single word had not been spoken by me in the
(d) The interference of the police force could alter the
meeting.
situation with hardness.
(d) Even a single word have not been spoken by me in the
141. Shut the door
meeting. (a) Let the door be shutted.
133. Who had laughed at you? (b) Let the door be shut by you.
(a) By whom were you been laughed at? (c) Let door be shut.
(b) By whom had you been laughed at? (d) Let the door be shut.
(c) Who had you been laughed at?
(d) By whom you had been laughed at? DIRECTIONS (Qs. 142-146) : Read the passage carefully and
134. Kindly offer your remarks. choose the best answer to each question out of the four
alternatives.
(a) You are being requested to offer your remarks.
(b) You are requested to offer your remarks. Passage (Five questions)
(c) You were requested to offer your remarks. Nobody can argue that the acquisition of knowledge is more fun
(d) You have been requested to offer your remarks. and easier with computers. The mere activity of touching and
135. Rajesh could not complete his paper because he had a exploring this device constitutes an enjoyable task for a child.
headache. This, accompanied by the relaxing attitude and software
(a) Rajesh’s paper would not be completed because he interactivity, usually contributes to a better grasping of new
had a headache. knowledge. At a higher educational level the availability of
(b) The paper could not be completed by Rajesh because digital books, simulators and other academic materials provide
he had a headache. the student with an ever accessible source of information, that
(c) Rajesh’s paper could not be completed because he otherwise would not be at hand. But, besides the increasing
had headache complexity and behaviour of intelligent software, which is
(d) The paper of Rajesh could not be completed by him usually embedded in the academic digital material, the need for
because he had a headache. human interaction in the learning process will always be present,
at least in the foreseeable future. There is the necessity for a
136. It is time to water the plants.
(a) It is time for the plants to be watered. human being to be able to determine what the specific needs of
(b) It is time the plants be watered. each individual are. A computer, no matter how sophisticated its
software is, can hardly mimic the expertise of a teacher in how to
(c) It is time when plants are watered.
(d) The plants should be watered. explain and adapt complex concepts to different individuals.
y
o
u
rs
Combined Graduate Level Exam 2015-2 – 9

m
a
h
142. According to the author, human intervention will always be 150. How had the bird been injured?

b
required in order to (a) While flying through the woods

o
o
(a) update old software (b) By a great storm

b
(b) built bigger machines (c) By wild animals

.w
(c) determine the specific needs of the individual (d) She had been shot by hunters

o
rd
(d) repair broken down machines 151. Why did the girl take the bird to Rhayader?

p
143. What other factors related to computers contribute to a (a) So that he would tell her if it was still alive

re
deeper acquisition of knowledge? (b) So that he would send it to where it came from

s
(a) Relaxing attitude and software interactivity

s
(c) So that he would tell her what to do

.c
(b) Prompt response and accuracy (d) So that he would heal it

o
(c) Convenience of usage and design

m
DIRECTIONS (Qs. 152-156) : In the following Five questions,
(d) User friendliness and easy accessibility sentences are given with blanks to be filled in with an
144. According to this essay, what new developments in the appropriate word(s). Four alternatives are suggested for each
world of computers have helped students gain more access question. Choose the correct alternative out of the four and
to information? indicate it by blackening the appropriate circle [•] in the
(a) Availability of general knowledge software Answer Sheet.
(b) Availability of printing facilities 152. He set up institutions of international..........
(c) Availability of word processing applications (a) reputation (b) famous
(d) Availability of digital books (c) repute (d) renown
145. In what way are computers inadequate even inspite of their 153. If you do well in this exam next year then my efforts
sophistication? …………. successful.
(a) They keep breaking down after much use. (a) has been (b) would have been
(b) They can hardly imitate a teacher’s ability to explain (c) have been (d) will be
the most difficult of concepts. 154. We ………… not hurry, we have got plenty of time.
(c) They still need humans to turn them on and off. (a) must (b) need
(d) They require humans to update them periodically. (c) would (d) should
146. That computers make learning easier is a fact 155. The noise of the drum beats ……………and frightened the
(a) accepted by all (b) rejected by some tiger.
(c) welcomed by all (d) contested by a few (a) perplexed (b) pertained
DIRECTIONS (Qs. 147-151) : Read the passage carefully and (c) prevented (d) persisted
choose the best answer to each question out the four 156. Masked men …………… a security van on the motorway.
alternatives. (a) held out (b) held up
Passage (five questions) (c) held forth (d) held on
One November afternoon, a child came to Rhayader’s light house
studio. She was about twelve, slender, dirty, nervous and timid. DIRECTIONS (Qs. 157-161) : Read the passage carefully and
In her arms she carried something. She was desperately afraid of choose the correct answer to each question out of the four
the ugly man she had come to see, but she had heard somewhere alternatives and fill in the blanks.
that this man would heal injured things. The man’s voice was Passage (Five Questions)
deep and kind when he spoke to her. What is it, child? She How the domestication of animals began is not known. Perhaps,
stepped forward timidly, and in her arms was a large white bird there were large numbers of animals in areas near water where
which was quite still. There were stains of blood on its men also were (i) Here man could observe the animals and study
whiteness. The girl placed it in his arms, “I found it, Sir. It’s hurt Is their habits, and this knowledge must have. (ii) him to tame them.
it still alive?” “Yes. Yes, I think so.” Rhayader went inside with It was again, easy for an (iii) people to domesticate animals and
the bird in his arms. He placed it upon a table. The girl covered. feed them on the husks of the grain that were left after threshing.
The bird fluttered Rhayader spread one of its large white wings. In any event, sheep and goats, pigs and cattle and later horses
“Child where did you find it?” “In the wash, Sir. and asses were tamed and kept in pens. Man, thus, (iv) food from
Hunters had been there.’ It is a snow goose from Canada. the soil and also from animals. In the pens, the animals could be
But how did it get here?” The girl’s eyes were fixed on the injured observed even more closely. Calves suckling milk must have
bird. She said “Can you heal it, Sir? “Yes” said Rhayader, “We given man the idea that he too could get food other than meat
will try. You can help me, She has been shot, poor thing, her leg is from cows and goats. This practice which combines agriculture
broken and the wing too.” with the raising of animals is known as (v) farming. Animals,
147. Where did Rhayader live? however, were chiefly used to provide meat and milk; they were
(a) In a cabin by the marsh yet to be used as beasts of burden or to do draw the plough.
(b) An old cottage marsh 157. (iii) = ?
(c) In a barn (a) farming (b) rural
(d) In a light house (c) business (d) agricultural
148. Where did the bird come from? 158. (v) = ?
(a) From Australia (b) From North America (a) mixed (b) joined
(c) From the South (d) From Canada (c) united (d) blended
149. Which of the phrases best describes the girl? 159. (i) = ?
(a) Shy and clean (b) Bold and brave (a) amassed (b) intensified
(c) Loud and forceful (d) Slender and dirty (c) concentrated (d) strengthened
y
o
u
rs
2015-2 – 10 Combined Graduate Level Exam

m
a
h
160. (iv) = ? DIRECTIONS (Qs. 172-174) : In the following Three questions,

b
(a) gained (b) obtained four words are given in each question, out of which only one

o
o
(c) achieved (c) attained word is correctly spelt. Find the correctly spelt word and mark

b
161. (i) = ?

.w
your answer in the Answer Sheet.
(a) rendered (b) modified

o
172. Find the correctly spelt words.

rd
(d) enabled (d) authorised (a) Refresher (b) Curiculum

p
DIRECTIONS (Qs. 162-171) : In the following Ten questions, (c) Professor (d) Aggreable

re
four alternatives are given for the Idiom/phrase underlined in

s
173. Find the correctly spelt word.

s
the sentence. Choose the alternative which best expresses the

.c
(a) Nigardy (b) Emminent
meaning of the Idiom/phrase and mark it in the Answer Sheet.

o
(c) Magnificent (d) Nuisence

m
162. He was punished for taking French leave 174. Find the correctly spelt word.
(a) giving lame excuse (a) Fascination (b) Vindicasion
(b) leaving without intimation (c) Assemilation (d) Compitation
(c) being absent without permission
DIRECTIONS (Qs. 175-177) : In the following Three question,
(d) resigning without giving notice
choose the word opposite in meaning to the given words and
163. If the speaker continues with his boring speech, the
audience will probably vote with their feet. mark it in the Answer Sheet.
(a) choose him as leader 175. Minuscule
(b) kick him out (a) impressive (b) Minute
(c) stop him from speaking (c) Massive (d) Menial
(d) show their disapproval 176. Profound
164. Shyam Prasad was a chip of the old block. (a) Superficial (b) Large
(a) a common boy of the locality (c) Less (d) Special
(b) an experienced old man 177. Wary
(c) a being of the old generation (a) Free (b) Careless
(d) someone similar in character to one’s father. (c) Watchful (d) Kind
165. Russel Peters really brought the house down with his
DIRECTIONS (Qs. 178-182) : Read the passage carefully and
spectacular performance at the Club yesterday.
(a) amused the audience greatly choose the best answer to each question out of the four
(b) defamed a family alternatives.
(c) pulled down a building Passage (Five Questions)
(d) passed a bill unanimously The field of medicine forces a confrontation between scientific
166. My father is undoubtedly an armchair critic. and everyday language. Outside the world of the research
(a) someone who gives advice based on practice not laboratory and clinic, there exists the daily routine of medical
theory practice, a situation where a doctor tries to understand the
(b) someone who never gives advice problems of a patient, and the patient attempts to understand the
(c) someone who gives advice based on theory not doctor’s diagnosis. The initial statement of the symptoms of any
practice disease is of critical importance as it guides the doctor’s search
(d) someone who gives advice based on experience for the clinical signs of the condition. Similarly, the doctor’s
167. Sunil’s dog in a manger attitude did not help to settle the explanation of a problem, and the recommendations for
matter. treatment, is need to be clear and complete if the patient to
(a) warm (b) selfish understand and follow the correct course of action.
(c) cold (d) selfless The need for careful listening and expression by both
168. He worked hard to feather his nest.
parties should be obvious in a field as sensitive and serious as
(a) decorate his home
health. Patients worried about their health are often uncertain
(b) be envy of others
(c) trap birds and confused in their accounts. Busy doctors will not have the
(d) provide for his own needs time to take up every point the patient has referred to. Moreover,
169. I threw up my cards for want of funds. the tradition of medical interviewing hinders the development of
(a) gambled all my money a genuine communication.
(b) gave up my plan 178. The world Confrontation’ is closest in meaning to
(c) put my whole income at stake (a) competition (b) confluence
(d) looked out for help (c) trouble (d) conflict
170. He advised her to give a wide berth to malicious people. 179. Which of the following statements is true?
(a) give a larger bed to (b) to stay away from (a) Doctors should use medical terms in their instructions
(c) share her seat with (d) pay more attention to as these are specific.
171. Tripti gave her report quoting chapter and verse. (b) Medical communication is a specialized branch of
(a) provided minute details professional communication.
(b) spoke like a preacher (c) Patients must learn medical terms.
(c) referred to religious books (d) The patient’s relatives should be involved in the
(d) taught like a teacher interaction.
y
o
u
rs
Combined Graduate Level Exam 2015-2 – 11

m
a
h
180. Doctors are sometimes unable to treat the patient properly 186. “Life, liberty, and the pursuit of happiness” are:

b
because. (a) Gifts of our Creator (b) Pillars of equality

o
o
(a) They do not have the time to consider every point (c) Ideals of a nation (d) Rights of all men

b
.w
made by the patient. 187. The source of the strength and power of a nation depends
(b) They are too busy in performing surgeries. on

o
rd
(c) They do not know the language used for (a) freedom of the people

p
communicating with patients. (b) its people.

re
(d) They have a superiority complex. (c) the physical health of the people.

s
s
181. Which of the following statements best reflects the theme (d) rights of the people.

.c
188. Citizens of a country should work and cherish

o
of the passage?

m
(a) Communication should not be made a component of (a) love
medical education. (b) liberty, justice and opportunity
(b) For proper treatment, patients should listen carefully (c) liberty and happiness
to the instructions. (d) opportunity and justice
(c) There must be little research in the field of doctor- 189. The passage leads the reader to think about
patient interaction. (a) Ruling nation
(d) Medical profession requires a careful use of medical (b) What a country needs
and everyday language. (c) The Rights of the people
182. Patients are often unable to give a proper account of their (d) The morals of a nation
ailment to the doctor because. 190. The ending part of the passages is about
(a) they do not know how to communicate in medical (a) Hope (b) Want
terms (c) Perfection (d) A promise
(b) they are worried about their health, DIRECTIONS (Qs. 191-195) : Read the passage carefully and
(c) they want to hide certain symptoms of the disease. choose the best answer to each question out of the four
(d) they have an inferiority complex. alternatives.
DIRECTIONS (Qs. 183-185) : In the following Three questions, Passage (Five Questions)
out of the four alternatives, choose the one which best expresses The world is becoming a dangerous place to live in, and this is
the meaning of the given word and mark it in the Answer Sheet despite our claims of being civilized, of having evolved from the
183. Viable primitive to the modern man and from the cave man to cultured
(a) Capable (b) Useless being.
(c) Workable (c) Bright Many reasons can be attributed to this. A man longs to be
184. Sinuous the king of all kingdoms, but is too extravagant and idle. He
(a) Serene (b) Straight desires that his writ should run through the whole world. But
(c) Serpentine (d) Transparent then he is lazy and lethargic Man is mean, far inferior to other
185. Erroneous species. We are more human than hamane. We have negative
(a) Wrong (b) Unfair qualities such as anger, ego, envy, greed, hatred, and jealously,
(c) False (d) Inaccurate that we should consider overcoming.
We have allowed these qualities to become our consuming
DIRECTIONS (Qs. 186-190) : Read the passage carefully and
passions. We think that we are mightier than most we think we
choose the best answer to each question out of the four are capable of destroying anything by using our might. Today,
alternatives. we have acquired weapons of mass destruction, which are
Passage (Five Questions) capable of obliterating all life from the face of this planet.
If a country should have a message for its people, it should be a As men we arm men. Then we destroy people without arms.
message of human dignity. The ideals of a nation should be of Then why are we giving vent to this anger? We let our wrath take
the freedom of ideas, speech, press, the right to assemble and the over our senses. We fight to satisfy our egos. The overpowering
right to worship. A country should boldly proclaim to a world obsession of a man with himself motivates him to grab
dominated by tyrants that “all men are created equal and they are everything and to fulfill his greed.
endowed by their Creator with certain inalienable rights” “and 191. Why has the world became a dangerous place to live in ?
among these are life, liberty, and the pursuit of happiness”. This (a) Because man has evolved from primitive to modern.
should be the source of the strength and power of a nation. If (b) Because man has become civilized.
people have the freedom to live their lives in dignity, they can (c) Because man desires to be king of all kingdoms but is
work with a sound mind and physical health. The moral, political, idle and extravagant.
and economic stature of a country lies in the strength of its (d) Because man has become a cultured being.
people. A nation should strive to be a more perfect, not the 192. What are the qualities that have become man’s consuming
perfect country where the people are given a promise and a hope passion?
in their minds to work and cherish liberty, justice, and (a) Idleness and laziness
opportunity. We do not always get what we want when we want (b) Anger, ego, greed, envy, hatred and jealousy
it but it is always better to believe that someday somehow, (c) The desire to be king
someway, we will get what we want. (d) Laziness and lethargy
y
o
u
rs
2015-2 – 12 Combined Graduate Level Exam

m
a
h
193. What does man think of himself today?

b
(a) That he is more humane than human Type P Type Q

o
o
(b) That he is mightier than most. 275 275

b
300
(c) That he can rule the world

.w
Number of Vehicles (in thousands)
275
(d) That he can be king of all kingdoms. 250 225

o
225 200

rd
194. What is man capable of achieving today? 200 175 225

p
(a) The power to conquer the world 175 150

re
(b) The power to obliterate life from this planet. 150 175 175

s
s
(c) The ability to destroy everything 125

.c
100 125 125
(d) The ability to have his writ run through the whole

o
75 100

m
world 50
195. Give the antonym of “Obliterate”. 25
(a) Prevent (b) Erase 0
(c) Destroy (d) Create 2009 2010 2011 2012 2013 2014
Years
DIRECTIONS (Qs. 196-200) : Read the passage carefully and
choose the best answer to each question of the four alternatives.
2. The total production of Type P vehicles in the years 2009
Passage (Five Questions) and 2011 is what percent of total production of Type Q
The question of race has caused bloodbaths throughout history. vehicles in 2010 and 2014?
Take the case of the Negro, a negro is someone with black skin (a) 80 (b) 69.25
who comes from Africa. It is an old-fashioned word and is (c) 81.25 (d) 75
offensive. Some people used to write that way deliberately. The 3. The production of Type Q vehicles in 2010 was
word “nigger” is also very offensive. The word was later approximately what percent of Type P vehicles in 2014?
replaced by “coloured” which gave way to “back”. Black is a (a) 75 (b) 60
colour with negative suggestions. (c) 45.5 (d) 54.5
So we have expressions like “black deed”, “black” day and 4. Approximate percentage decrease in production of Type Q
“Blackmail”. So no wonder the word “black” too assumed vehicles from 2010 to 2011 is
unfavourable meanings. (Although in the 1960’s the famous (a) 16.7 (b) 14.3
slogan “Black is beautiful” was coined, and it did not help). The (c) 10.1 (d) 12.5
Blacks of the United States therefore came to be called Afro- 5. The ratio of total production of Type P vehicles to total
Americans. Now, the politically correct phrase is African production of Type Q vehicles over the years is
American. (a) 8 : 5 (b) 48 : 41
196. Which word is old-fashioned and offensive? (c) 41 : 48 (d) 5 : 8
(a) Skin (b) Negro 6. In how many of the given years, was the production of
(c) Africa (d) Black Type P vehicles of the company more than the average
197. Which impression does the word ‘black’ carry? production of this type vehicles in the given years?
(a) All of the above (b) Contemptible (a) 5 (b) 4
(c) Unfavourable (d) Negative (c) 3 (d) 2
198. Which is the politically correct phrase? 7. It 3(a2 + b2 + c2) = (a + b + c)2, then the relation between a,
(a) Coloured (b) African American b and c is
(c) Nigger (d) Afro-American (a) a ¹ b = c (b) a = b = c
199. Why was “Black” is beautiful coined? (c) a ¹ b ¹ c (d) a = b ¹ c
(a) To combat the pre-judiced feelings against the Blacks 8. A car covers four successive 7 km distances at speeds of 10
(b) To encourage Racal and Caste bias km/hour, 20 km/hour, 60 km/hour respectively. Its average
(c) To enhance the confidence of aboriginals speed over this distance is
(d) To discourage negro slavery system (a) 40 km/hour (b) 20 km/hour
200. Give the synonym of “Offensive”. (c) 60 km/hour (d) 30 km/hour
(a) Courteous (b) Aggressive 9. A cylinder with base radius 8 cm and height 2 cm is melted
(c) Sympathetic (d) Defending to form a cone of height 6 cm. The radius of the cone will be
(a) 6 cm (b) 5 cm
QUANTITATIVE APTITUDE (c) 4 cm (d) 8 cm
10. A dealer fixed the price of an article 40% above the cost of
6 2 + 7 2 + 82 + 9 2 + 10 2 production. While selling it he allows a discount of 20%
1. is equal to
7+4 3- 4+2 3 and makes a profit of ` 48. The cost of production (in `) of
(a) 305 (b) 355 the article is
(c) 366 (d) 330 (a) 420 (b) 360
DIRECTIONS (Q. 2-6) : The following graph shows production (c) 400 (d) 320
(in thousands) of two types (P and Q) of vehicles by a factory 11. Average of n numbers is a. The first number is increased by
over the years 2009 to 2014. Study the graph and answer five 2, second one is increased by 4, the third one is increased
questions : by 8 and so on. The average of the new number is
y
o
u
rs
Combined Graduate Level Exam 2015-2 – 13

m
a
h
21. ABCD is a cyclic quadrilateral. AB and DC when produced
2n - 1 2n - 1

b
(a) a+2 (b) a+ meet at P, if PA = 8 cm, PB = 6 cm, PC = 4 cm, then the length

o
n n

o
(in cm) of PD is

b
2n + 1 2n +1 - 1 (a) 6 (b) 12

.w
(c) a+2 (d) a+ (c) 8 (d) 10

o
n n

rd
22. In a school there were 1554 students and the ratio of the
12. If x = a sin q - b cos q, y = a cos q + b sin q, then which of

p
number of the boys and girls was 4 : 3. After few days, 30

re
the following is true? girls joined the school but few boys left; as a result the ratio

s
s
of the boys and girls became 7 : 6. The number of boys who
x2 a2

.c
x 2 + y 2 = a 2 + b2 + =1 left the school is

o
(a) (b) 2
y b2

m
(a) 84 (b) 76
(c) 86 (d) 74
x2 y2
(c) x 2 + y 2 = a 2 - b2 (d) + =1 23. If 7sin q + 3cos q = 4 , then the value of tan q is (q is
2 2
a2 b2
acute)
13 + 11 1
13. Let x = and y = , th en the value of 1 1
13 - 11 x (a) (b)
3 2
3 x2 - 5 xy + 3 y 2 is (c) 1 (d) 3
(a) 1717 (b) 1771
24. If (3x - 2 y ) : (2 x + 3 y) = 5 : 6 , th en one of value of
(c) 1171 (d) 1177
2
14. If 64 buckets of water are removed from a cubical shaped æ3 x+3 yö
water tank completely filled with water, 1/3 of the tank remains çç 3 ÷÷ is
è x- yø
3
filled with water. The length of each side of the tank is 1.2 m.
Assuming that all buckets are of the same measures then 1
the volume (in litres) of water contained by each bucket is (a) 25 (b)
5
(a) 16 (b) 18
(c) 12 (d) 15 1
(c) (d) 5
15. In trapezium ABCD, AB || CD and AB = 2CD. Its diagonals 25
intersect at O. If the area of DAOB = 84 cm2, then the area of 25. If tan A = n tan B and sinA = m sin B, then the value of cos2A is
DCOD is equal to
(a) 42 cm2 (b) 21 cm2 m2 + 1 m2 - 1
2 (a) (b)
(c) 72 cm (d) 26 cm2 n2 + 1 n2 - 1
16. Water tax is increased by 20% but its consumption is
decreased by 20%. Then the increase or decrease in the m2 + 1 m2 - 1
(c) (d)
expenditure of the money is n2 - 1 n2 + 1
(a) 5% decrease (b) 4% decrease 26. In an office, 40% of the staff is female. 70% of the female
(c) No change (d) 4% increase staff and 50% of the male staff are married. The percentage
17. A number when divided by 361 gives a remainder 47. If the of the unmarried staff in the office is
same number is divided by 19, the remainder obtained is (a) 42 (b) 60
(a) 1 (b) 3 (c) 54 (d) 64
(c) 9 (d) 8 27. In an examination average mark obtained by the girls of a
1 1 class is 85 and the average mark obtained by the boys of
æ p -1 q 2 ö 3
æ p 6 q -3 ö 3 the same class is 87. If the girls and boys are in the ratio 4 :
18. If çç 3 -2 ÷÷ ¸ çç -2 3 ÷÷ = p a q b , then the value of a + b, 5, average marks of the whole class (approx.) is closest to
èp q ø èp q ø (a) 86.4 (b) 86.1
where p and q are different positive primes, is (c) 85.9 (d) 86.5
(a) 1 (b) 2 28. Articles are marked at a price which gives a profit of 25%.
(c) 0 (d) –1 After allowing a certain discount the profit reduces to
19. In triangle ABC, DE || BC where D is a point on AB and E is 1
a point on AC. DE divides the area of DABC into two equal 12 % . The discount percent is
2
parts. Then DB : AB is equal to
1
(a) ( 2 + 1) : 2 (b) 2 : ( 2 + 1) (a) 12 % (b) 10%
2
(c) 2 : ( 2 - 1) (d) ( 2 - 1) : 2 (c) 12% (d) 11.1%
20. A and B have their monthly incomes in the ratio 8 : 5, while 2 4
29. If sin A + sin2 A = 1, then the value of cos A + cos A is
their monthly expenditures are in the ratio 5 : 3. If they have
saved ` 12,000 and ` 10,000 monthly respectively, then the 2
(a) 1 (b) 2
difference in their monthly income is 3
(a) ` 42,000 (b) ` 44,000 1
(c) ` 46,000 (d) ` 52,000 (c) 1 (d) 1
2
y
o
u
rs
2015-2 – 14 Combined Graduate Level Exam

m
a
h
30. A manufacturer fixes his selling price at 33% over the cost 40. Three science classes A, B and C take a Life Science test.

b
of production. If cost of production goes up by 12% and The average score of class A is 83. The average score of

o
o
manufacturer raises his selling price by 10%, his percentage class B is 76. The average score of class C is 85. The average

b
profit is score of class A and B is 79 and average score of class B

.w
and C is 81. Then the average score of classes A, B and C is

o
5

rd
(a) 35% (b) 36 % (a) 80.5 (b) 81.5
9

p
(c) 80 (d) 81

re
3 5 41. Two blends of a commodity costing `35 and `40 per kg

s
28 % (d) 30 %

s
(c) respectively are mixed in the ratio 2 : 3 by weight. If one-

.c
8 8

o
fifth of the mixture is sold at `46 per kg and the remaining at

m
1 the rate of `55 per kg, the profit percent is
31. A boat moves downstream at the rate of 1 km in 7 minutes
2 (a) 50 (b) 20
and upstream at the rate of 5 km an hour. What is the speed (c) 30 (d) 40
(in km/hour) of the boat in the still water?
(a) 8 (b) 4 42. If x2 + y 2 + z 2 = xy + yx + zx , then the value of
1
3
1 3 x4 + 7 y 4 + 5 z 4
(c) (d) 6 is
2 2 5x 2 y 2 + 7 y 2 z 2 + 3z 2 x 2
32. The greatest number among 350, 440, 530, 620 is
(a) 440 (b) 530 (a) 1 (b) 2
(c) 620 (d) 350 (c) –1 (d) 0
33. Give in ratio that the ratio of altitudes of two triangles is 4 : 43. Ram sold two horses at the same price, in one he gets a
5 and ratio of their areas is 3 : 2. The ratio of their profit 10% and in the other he gets a loss of 10%. Then Ram
corresponding bases is gets
(a) 8 : 15 (b) 5 : 8 (a) no loss or profit (b) 1% profit
(c) 15 : 8 (d) 8 : 5 (c) 2% loss (d) 1% loss
44. A and B can do a given piece of work in 8 days, B and C can
1 do the same work in 12 days and A, B and C complete it in 6
34. If sec q - tan q = , the value of secq tanq is
3 days. Number of days required to finish the work by A and
2 2 C is
(a) (b) (a) 8 (b) 16
3 3
(c) 24 (d) 12
1 4
(c) (d) 45. Pipe A can fill an empty tank in 6 hours and pipe B in 8
3 3 hours. If both the pipes are opened and after 2 hours pipe A
35. A man sells an article at 5% above its cost price. If he had is closed, how much time B will take to fill the remaining
bought it at 5% less than what he had paid for it and sold it tank?
at ` 2 less, he would have gained 10%. The cost price of the
article is 2 1
(a) 2 hours (b) 7 hours
(a) ` 100 (b) ` 300 5 2
(c) ` 200 (d) ` 400 1 1
(c) 2 hours (d) 3 hours
(0.67 ´ 0.67 ´ 0.67) ´ (0.33 ´ 0.33 ´ 0.33) 3 3
36. The value of
(0.67 ´ 0.67) + (0.67 ´ 0.33) ¸ (0.33 ´ 0.33) 46. There is a number consisting of two digits, the digit in the
(a) 11 (b) 0.34 units place is twice that in the tens place and if 2 be
(c) 1.1 (d) 3.4 subtracted from the sum of the digits, the difference is equal
to 1/6th of the number. The number is
37. If a + b = 1, find the value of a3 + b3 - ab - (a 2 - b2 )2
(a) 26 (b) 23
(a) 0 (b) 1 (c) 25 (d) 24
(c) –1 (d) 2 47. The value of cot 41° × cot 42° × cot 43° × cot 44° ×
38. AB and CD are two parallel chords of a circle of lengths 10
cm and 4 cm respectively. If the chords are on the same side cot 45° × cot 46° × cot 47° × cot 48° × cot 49°
of the centre and the distance between them is 3 cm, then (a) 0 (b) 1
the diameter of the circle is 3 1
(a) 2 29 cm (b) (c) (d)
21 cm 2 2
(c) 2 21 cm (d) 29 cm 48. A man purchases some oranges at the rate of 3 for ` 40 and
39. Let x be the least number, which when divided by 5, 6, 7 and the same quantity at 5 for ` 60. If he sells all the oranges at
8 leaves a remainder 3 in each case but when divided by 9 the rate of 3 for ` 50, find his gain or loss percent (to the
leaves no remainder. The sum of digits of x is nearest integer).
(a) 22 (b) 21 (a) 32% profit (b) 34% loss
(c) 18 (d) 24 (c) 31% profit (d) 31% profit
y
o
u
rs
Combined Graduate Level Exam 2015-2 – 15

m
a
h
49. The perimeter of a rhombus is 60 cm and one of its diagonal 59. If tan q - cot q = 0 and q is positive acute angle, then the

b
is 24 cm. The area (in sq. cm.) of the rhombus is

o
tan(q + 15°)

o
(a) 206 (b) 432

b
value of is
(c) 108 (d) 216 tan(q - 15°)

.w
50. A sum of money is paid back in two annual instalments of `

o
1

rd
17,640 each, allowing 5% compound interest compounded (a) (b) 3

p
annually. The sum borrowed was 3

re
(a) ` 32,400 (b) ` 32,800

s
1

s
(c) ` 32,000 (d) ` 32,200 (c) (d) 3

.c
51. A man starts from a place P and reaches the place Q in 7 3

o
m
hours. He travels 1/4th of the distance at 10 km/hour and the 60. The portion of a ditch 48 m long, 16.5 m wide and 4 m deep
remaining distance at 12 km/hour. The distance, in kilometre, that can be filled with stones and earth available during
between P and Q is excavation of a tunnel, cylindrical in shape, of a diameter 4
(a) 72 (b) 80
é 22 ù
(c) 90 (d) 70 m and length 56 m is ê Take p = ú
52. If O is the circumcentre of a triangle ABC lying inside the ë 7û
triangle, then ÐOBC + ÐBAC is equal to 1 2
(a) 110° (b) 90° (a) part (b) part
9 9
(c) 120° (d) 60° 1 1
(c) part (d) part
8 2 4
53. The simple interest on a sum of money is of the sum. If
25 61. If ( x3 - y3 ) : ( x 2 + xy + y 2 ) = 5 :1 and
the number of years is numerically half the rate percent per
annum, then the rate percent per annum is ( x 2 - y 2 ) : ( x - y) = 7 :1 , the n the ratio 2x : 3y equals
(a) 8 (b) 5 (a) 2 : 3 (b) 4 : 1
1 (c) 4 : 3 (d) 3 : 2
(c) 6 (d) 4
4 62. If x = a1/ 2 + a -1/ 2 , y = a1/ 2 - a -1/ 2 , then value of
54. In DABC, ÐBAC = 90° and AD ^ BC . If BD = 3 cm and
( x4 - x2 y 2 - 1) + ( y 4 - x 2 y 2 + 1)
CD = 4 cm, then the length (in cm) of AD is
(a) 16 (b) 14
(a) 2 3 (b) 6 (c) 12 (d) 13
(c) 3.5 (d) 5 63. The marked price of a tape recorder is ` 12,600. A festival
55. Three glasses of equal volume contains acid mixed with discount of 5% is allowed on it. Further for cash payment, a
water. The ratio of acid and water are 2 : 3, 3 : 4 and 4 : 5 second discount of 2% is given. The cash payment, in
respectively. Contents of these glasses are poured in a large rupees, that is to be made for buying it is
vessel. The ratio of acid and water in the large vessel is (a) 11,730.60 (b) 11,073.60
(a) 407 : 560 (b) 417 : 564 (c) 11,703.60 (d) 11,370.60
(c) 411 : 540 (d) 401 : 544 64. If a man walks at the rate of 5 km/hour, he misses a train by
56. If A : B = 2 : 3 and B : C = 3 : 7 then A + B : B + C : C + A is 7 minutes. However, if he walks at the rate of 6 km/hour, he
(a) 4 : 8 : 9 (b) 5 : 8 : 9 reaches the station 5 minutes before the arrival of the train.
(c) 4 : 10 : 9 (d) 5 : 10 : 9 The distance covered by him to reach the station is
57. The numerical values of the volume and the area of the (a) 6 km (b) 7 km
lateral surface of a right circular cone are equal. If the height (c) 4 km (d) 6.25 km
1 1 65. If x - 3 - 2 = 0 and y - 3 + 2 = 0 then value of
of the cone be h and radius, be r, the value of 2 + 2 is
h r
( x3 - 20 2) - ( y3 + 2 2)
3 9 (a) 3 (b) 2
(a) (b)
1 1 (c) 0 (d) 1
1 1 66. The radii of two solid iron spheres are 1 cm and 6 cm
(c) (d) respectively. A hollow sphere is made by melting the two
9 3
spheres. If the external radius of the hollow sphere is 9 cm,
58. Two places P and Q are 162 km apart. A train leaves P for Q then its thickness (in cm) is
and simultaneously another train leaves Q for P. They meet (a) 0.5 (b) 2
at the end of 6 hours. If the former train travels 8 km/hour (c) 1.5 (d) 1
faster than the other, then speed of train from Q is
67. There is a wooden sphere of radius 6 3 cm. The surface
1 5 area of the largest possible cube cut out from the sphere
(a) 9 km/hour (b) 10 km/hour
2 6 will be
5 1 (a) 464 3 cm2 (b) 646 3 cm2
(c) 12 km/hour (d) 8 km/hour 2
6 2 (c) 462 cm (d) 864 cm2
y
o
u
rs
2015-2 – 16 Combined Graduate Level Exam

m
a
h
68. If 60% of A = 30% of B, B = 40% of C and C = x% of A, then (a) increase by 1

b
value of x is (b) remains the same

o
o
(a) 200 (b) 500 (c) increase by 2

b
.w
(c) 300 (d) 800 (d) increase by 1.5
69. A and B can do a piece of work in 30 and 36 days respectively.

o
1

rd
They began the work together but A leaves after some days 79. If a - = 5, then the value of
a -3

p
and B finished the remaining work in 25 days. After how

re
many days did A leave?

s
1

s
(a) 6 days (b) 5 days ( a - 3)3 -

.c
is
( a - 3)3

o
(c) 11 days (d) 10 days

m
70. A sum of money placed at compound interest doubles itself (a) 14 (b) 5
in 5 years. It will amount to eight times itself at the same rate (c) 2 (d) 7
of interest in 80. A plane divides a right circular cone into two parts of equal
(a) 10 years (b) 20 years volume. If the plane is parallel to the base, then the ratio, in
(c) 12 years (d) 15 years which the height of the cone is divided, is
71. Quadrilateral ABCD is circumscribed about a circle. If the (a) 1: 3 2 (b) 1: 2
lengths of AB, BC, CD are 7 cm, 8.5 cm and 9.2 cm respectively,
(c) 1: 3 2 - 1 (d) 1: 3 2 + 1
then the length (in cm) of DA is
(a) 16.2 (b) 7.2 81. Let x be the smallest number, which when added to 2000
(c) 7.7 (d) 10.7 makes the resulting number divisible by 12, 16, 18 and 21.
The sum of the digits of x is
72. A right prism has a triangular base whose sides are 13 cm,
20 cm and 21 cm. If the altitude of the prism is 9 cm, then its (a) 4 (b) 7
volume is (c) 6 (d) 5
(a) 1134 cm3 (b) 1314 cm3 82. The diameter of each wheel of a car is 70 cm. If each wheel
3 rotates 400 times per minute, then the speed of the car (in
(c) 1413 cm (d) 1143 cm3
73. 300 grams of sugar solution has 40% of sugar in it. How æ 22 ö
km/hr) is ç Take p = ÷
much sugar should be added to make it 50% in the solution? è 7 ø
(a) 60 gms (b) 10 gms (a) 52.8 (b) 0.528
(c) 80 gms (d) 40 gms (c) 528 (d) 5.28
74. The area of isosceles trapezium is 176 cm2 and the height is 83. The average age of 30 students of a class is 14 years 4
2 months. After admission of 5 new students in the class the
th of the sum of its parallel sides. If ratio of the length of average becomes 13 years 9 months. The youngest one of
11
the parallel sides is 4 : 7, then the length of a diagonal (in the five new students is 9 years 11 months old. The average
cm) is age of the remaining 4 new students is
(a) 13 years 6 months (b) 10 years 4 months
(a) 2 137 (b) 137 (c) 11 years 2 months (d) 12 years 4 months
(c) 24 (d) 28 ss 84. P and Q together can do a job in 6 days. Q and R can finish
75. A and B are centres of two circles of radii 11 cm and 6 cm
60
respectively. PQ is a direct common tangent to the circles. the same job in days. P started the work and worked for
7
If AB = 13 cm, then length of PQ will be 3 days. Q and R continued for 6 days. Then the difference
(a) 8.5 cm (b) 12 cm of days in which R and P can complete the job is
(c) 13 cm (d) 17 cm (a) 8 (b) 12
76. A, B and C can do work separately in 16, 32 and 48 days (c) 10 (d) 15
respectively. They started the work together but B leaving 85. A telegraph post is bent at a point above the ground due to
off 8 days and C six days before the completion of the work. storm. Its top just touches the ground at a distance of 10 3
In what time is the work finished?
m from its foot and makes an angle of 30° with the horizontal.
(a) 12 days (b) 10 days Then height (in metres) of the telegraph post is
(c) 14 days (d) 9 days (a) 24 (b) 20
77. AD is perpendicular to the internal bisector of ÐABC of (c) 25 (d) 30
DABC. DE is drawn through D and parallel to BC to meet
86. If 5cos q + 12sin q = 13, 0° < q < 90°, then the value of
AC at E. If the length of AC is 12 cm, then the length of AE
(in cm) is sinq is
(a) 3 (b) 6 12 5
(c) 8 (d) 4 (a) - (b)
13 13
78. The average of five consecutive positive integers is n. If
the next two integers are also included, the average of all 12 6
(c) (d)
these integers will 13 13
y
o
u
rs
Combined Graduate Level Exam 2015-2 – 17

m
a
h
1 1 1 1

b
87. If a + = b + = c + , where a ¹ b ¹ c ¹ 0 then the (d) th of the radius of the hemisphere

o
b c a 4

o
b
94. 60 kg of an alloy A is mixed with 100 kg of alloy B. If alloy A

.w
value of a2 b2 c 2 is
has lead and tin in the ratio 3 : 2 and alloy B has tin and

o
(a) –1 (b) 1 copper in the ratio 1 : 4, the amount of tin in the new alloy is

rd
(c) abc (d) 0 (a) 53 kg (b) 80 kg

p
88. The HCF and LCM of two numbers are 21 and 84 respectively.

re
(c) 44 kg (d) 24 kg
If the ratio of the two numbers is 1 : 4, then the larger of the

s
95. Base of a right pyramid is a square of side 10 cm. If the

s
two numbers is

.c
height of the pyramid is 12 cm, then its total surface area is

o
(a) 48 (b) 108 (a) 260 cm2 (b) 460 cm2

m
(c) 12 (d) 84 (c) 400 cm2 (d) 360 cm2
89. If 90 men can do a certain job in 61 days, working 12 hours/ 96. If a shopkeeper wants to give 20% discount on a toy, he
day, then the part of that work which can be completed by
has to sell it for ` 300. If he sells it at ` 405, then his gain
70 men in 24 days, working 8 hours/day is percent is
2 7 (a) 6% (b) 4%
(a) (b) (c) 8% (d) 5%
3 9
97. The unit digit in the product (2467)153 × (341)72 is
1 5 (a) 1 (b) 3
(c) (d)
3 8 (c) 7 (d) 9
90. A sum of ` 7,930 is divided into 3 parts and given on loan at 98. The interior angle of a regular polygon exceeds its exterior
5% simple interest to A, B and C for 2, 3 and 4 years angle by 108°. The number of sides of the polygon is
respectively. If the amounts of all three are equal after their (a) 10 (b) 16
respective periods of loan, then A received a loan of (c) 14 (d) 12
(a) ` 2,750 (b) ` 2,800 99. The value of
(c) ` 2,760 (d) ` 3,050 4– 4 .
91. The value of 1+ 1 .
(coseca – sina)(seca – cosa) (tana + cota) 3+ 1 .
(a) 4 (b) 2 1
(c) 1 (d) 6 2+
4
92. There would be a 10% loss, if rice is sold at ` 54 per kg. To
earn a profit of 20%, the price of rice per kg will be 1 1
(a) ` 65 (b) ` 63 (a) (b)
8 64
(c) ` 70 (d) ` 72
93. If a hemisphere is melted and four spheres of equal volume 1 1
(c) (d)
are made, the radius of each sphere will be equal to 16 32
(a) radius of the hemisphere 100. The centroid of a DABC is G. The area of D abc is 60 cm2.
1 The area of DGBC is
(b) th of the radius of the hemisphere (a) 40 cm2 (b) 30 cm2
6
(c) 20 cm2 (d) 10 cm2
1
(c) of the radius of the hemisphere
2
y
o
u
rs
2015-2 – 18 Combined Graduate Level Exam

m
a
h
Hints & Explanations

b
o
o
b
.w
ENGLISH LANGUAGE 150 - 125

o
´ 100 » 16.7%

rd
4. (a)
1. (d) 2. (a) 3. (a) 4. (d) 5. (d) 150

p
re
6. (c) 7. (b) 8. (d) 9. (b) 10. (c) 100 + 125 + 200 + 225 + 275 + 275

s
11. (c) 12. (c) 13. (c) 14. (b) 15. (d) 5. (b)

s
175 + 150 + 125 + 175 + 175 + 225

.c
16. (c) 17. (b) 18. (b) 19. (c) 20. (d)

o
21. (b) 22. (b) 23. (a) 24. (a) 25. (b) 1200 48

m
= =
26. (a) 27. (d) 28. (b) 29. (b) 30. (c) 1025 41
31. (a) 32. (a) 33. (a) 34. (c) 35. (d)
36. (b) 37. (c) 38. (d) 39. (b) 40. (b) 1200
6. (c) Average production = = 200
41. (c) 42. (b) 43. (c) 44. (a) 45. (c) 6
46. (d) 47. (c) 48. (b) 49. (a) 50. (b) In 2012, 2013 and 2014, the production is more than
51. (c) 52. (a) 53. (b) 54. (a) 55. (b) 200.
56. (d) 57. (b) 58. (a) 59. (a) 60. (c)
61. (d) 62. (a) 63. (b) 64. (b) 65. (c) 7. (b) 3(a 2 + b2 + c2 )
66. (c) 67. (c) 68. (b) 69. (d) 70. (d) = (a 2 + b2 + c2 ) + 2(ab + bc + ca)
71. (a) 72. (a) 73. (a) 74. (c) 75. (c)
76. (b) 77. (d) 78. (d) 79. (a) 80. (c) Þ 2a2 + 2b2 + 2c 2 - 2ab - 2bc - 2ac = 0
81. (b) 82. (c) 83. (d) 84. (b) 85. (a)
86. (a) 87. (b) 88. (a) 89. (d) 90. (b) Þ (a - b)2 + (b - c)2 + (c - a)2 = 0
91. (b) 92. (c) 93. (d) 94. (d) 95. (b) Þa=b=c
96. (a) 97. (d) 98. (a) 99. (b) 100. (a)
101. (d) 102. (d) 103. (d) 104. (b) 105. (a) 7+7+7+7
8. (b) Average speed =
106. (d) 107. (c) 108. (b) 109. (b) 110. (c) 7 7 7 7
+ + +
111. (c) 112. (d) 113. (b) 114. (d) 115. (d) 10 20 30 60
116. (b) 117. (c) 118. (d) 119. (c) 120. (b) = 20 km/hr.
121. (d) 122. (a) 123. (a) 124. (a) 125. (d)
1
126. (b) 127. (b) 128. (b) 129. (b) 130. (d) 9. (d) p(8) 2 (2) = p(r ) 2 × (6) Þ r = 8 cm.
131. (d) 132. (a) 133. (b) 134. (b) 135. (b) 3
136. (a) 137. (a) 138. (a) 139. (d) 140. (a) 10. (c) Let the cost price be ` 100x.
141. (d) 142. (c) 143. (a) 144. (d) 145. (b) æ 100 + 40 öæ 100 - 20 ö
146. (a) 147. (d) 148. (d) 149. (d) 150. (d) Þ 100 x ç ÷ç ÷ - 100 x = 48
è 100 øè 100 ø
151. (d) 152. (c) 153. (d) 154. (b) 155. (a)
156. (b) 157. (d) 158. (a) 159. (c) 160. (b) Þ 100x = 400.
161. (c) 162. (c) 163. (d) 164. (d) 165. (a) na + 2 + 4 + 8 + .... + 2n
166. (c) 167. (b) 168. (d) 169. (b) 170. (b) 11. (a) Average =
n
171. (a) 172. (c) 173. (c) 174. (a) 175. (c)
176. (a) 177. (b) 178. (d) 179. (b) 180. (c) æ 2n - 1 ö
181. (d) 182. (b) 183. (c) 184. (c) 185. (a) na + 2 çç ÷÷
è 2 -1 ø = a + 2 × 2 -1
n
186. (d) 187. (d) 188. (b) 189. (b) 190. (a) =
191. (c) 192. (b) 193. (b) 194. (b) 195. (d) n n
196. (b) 197. (a) 198. (b) 199. (b) 200. (b)
12. (a) x 2 = a 2 sin 2 q + b2 cos2 q - 2ab sin q cos q

QUANTITATIVE APTITUDE y 2 = a 2 cos2 q + b2 sin 2 q + 2ab sin q cos q


Þ x 2 + y 2 = a 2 (sin 2 q + cos 2 q) +b 2 (cos2 q + sin 2 q)
2 2 2 2 2 2)
(1 + 2 + ...... + 10 ) - (1 + 2 + ........ + 5 = a2 + b2.
1. (d)
(2 + 3) - ( 3 + 1)
13 - 11 13 - 11
13. (a) x + y = +
385 - 55 13 + 11 13 + 11
= = 330
1
2(13 + 11)
100 + 200 = = 24
2. (a) ´ 100 = 80% 13 - 11
150 + 225
3 x2 - 5xy + 3 y 2 = 3( x + y)2 - 11xy
150
3. (d) ´ 100 = 54.5% = 3(24)2 - 11(1) = 1717.
275
y
o
u
rs
Combined Graduate Level Exam 2015-2 – 19

m
a
h
2
20. (a) 8x – 5y = 12,000 ...(1)

b
14. (b) rd of the tank is emptied using 64 buckets. 5x – 3y = 10,000 ...(2)

o
3

o
From (1) and (2)

b
.w
3 : x = 14,000
Þ Volume of the tank = 64 ´ i.e., 96 buckets of water..

o
2 \ Difference in their incomes

rd
\ Volume of each bucket = 8x – 5x = 3x = ` 42,000.

p
re
1.2 ´ 1.2 ´1.2 ´1000 litres 21. (b) PA × PB = PC × PD
=

s
= 18 litres.

s
96 8´6

.c
Þ PD = = 12 cm.

o
D C 4

m
15. (b)
4
O 1554 ´ - x
B 7 7
= =
22. (b) G 3 6
1554 ´ + 30
7
A B Þ x = 76.
ÐAOB = ÐCOD ü 23. (a) 4sin 2 q + 3sin 2 q + 3cos 2 q = 4
ï
ÐCAB = ÐDCA ý DAOB is similar to DCOD
1
ÐDBA = ÐCDB ïþ Þ 4sin 2 q = 1 Þ sin q =
2
AB 2 Area of DAOB æ 2 ö
2
Þ q = 30° (Acute)
= Þ =ç ÷
CD 1 Area of DCOD è 1 ø 1
\ tan q = tan 30° = .
1 2
3
Þ Area of DCOD = 84 ´ = 21 cm
4 24. (a) 6(3 x - 2 y ) = 5(2 x + 3 y)
(20)(-20) y 8
16. (b) (20) + ( -20) + = -4 Þ =
100 x 27
\ 4% decrease.
17. (c) N = 361x + 47 = 19 (19x + 2) + 9 é3 æ
2
y öù æ 2ö
2
æNö ê x çç1 + 3 ÷÷ ú 1 +
x øú ç 3 ÷ æ 5 ö2
Remainder ç ÷ = 9. ê è
= ç ÷ = ç ÷ = 25.
è 19 ø ê æ ú
ê 3 x ç1 - 3 y öú çç 1 - 2 ÷÷ è 1 ø
÷
1 êë èç x ø÷ úû è 3ø
( p -4 q 4 ) 3 -4
= p a × qb = p 3 q10 / 3
18. (b) 1 25. (b) tanA = ntanB Þ cotB = ncotA ...(1)
( p 8 q -6 ) 3 sinA = msinB Þ cosecB = mcosecA ...(2)
Squaring and subtracting eq (1) from eq(2)
4 10 6
Þ a +b = - + = = 2 : cosec2B – cot2B = m2cosec2A – n2cot2A
3 3 3
m 2 - n2 cos 2 A
19. (d) A Þ =1
sin 2 A
Þ m2 - n2 cos 2 A = 1 - cos 2 A

m2 - 1
D \ cos 2 A = .
E n2 - 1

26. (a) Total 100

B C
DADE is similar to DABC and
Males Females
Area of DADE 1 AD 1 60 40
= Þ =
Area of DABC 2 AB 2 Married ® 30 28
Unmarried ® 30 12
DB AD 1 2 -1
\ = 1- = 1- = . Total unmarried = 30 + 12 = 42
AB AB 2 2
y
o
u
rs
2015-2 – 20 Combined Graduate Level Exam

m
a
h
4(85) + 5(87)

b
27. (b) = 86.1.

o
4+5

o
38. (a)

b
28. (b) Cost price = 100x

.w
Þ Marked price = 125x

o
After allowing some discount Selling price = 112.5x.

rd
r

p
12.5 x

re
\ Discount (%) = ´ 100 = 10% x

s
125 x

s
5

.c
29. (d) sinA = 1 – sin2A = cos2A 3
2

o
cos2A + cos4A = sinA + sin2A = 1.

m
30. (d) Cost price = 100 Þ Selling price = 133
New cost price = 112 and
New selling price = 133 + 13.3 = 146.3 r 2 = 52 + x2 = (3 + x) 2 + 22
34.3 5
\ Profit (%) = = ´ 100 = 30 % Þ x = 2 Þ r = 52 + 22 = 29
112 8
\ Diameter = 2r = 2 29 .
1 39. (c) LCM (5, 6, 7, 8) = 840
31. (d) U + V = = 8 km/hr..
æ 7.5 ö x = 840k + 3, is divisible by 9 when k = 2.
ç ÷
è 60 ø \ x = 1683 Þ sum of the digits
U – V = 5 km/hr. = 1 + 6 + 8 + 3 = 18.
8+5 1 40. (b)
\U = = 6 km/hr.. A B C D
2 2
32. (a) (35)10, (44)10, (53)10, (62)10 83 76 76 85
(243)10, (256)10, (125)10, (36)10
\ Greatest = 440. 79 81
1
´ b1 ´ 4 x
3 b 15 3 4 4 5
33. (c) 2 = Þ 1 =
1 2 b2 8
´ b2 ´ 5 x A : B = 3 : 4; B : C = 4 : 5
2
Þ A: B :C = 3: 4:5
1
34. (a) sec q - tan q = 3(83) + 4(76) + 5(85)
3 Average = = 81.5.
Þ sec2 q– tan2 q = 1 3+ 4 + 5
Þ (sec q – tan q) (sec q + tanq) = 1 41. (d) Let the two blends be 2 kg and 3 kg.
Total cost price = 2(35) + 3(40) = ` 190.
1 Selling price = 1(46) + 4(55) = ` 266.
Þ (sec q + tan q) =
3 266 - 190
\ Profit (%) = ´ 100 = 40%.
(sec q + tan q)2 - (sec q - tan q) 2 190
Þ sec q × tan q =
4 42. (a) x2 + y 2 + z 2 - xy - yz - zx = 0
1
3-
3=2. 1
= Þ [( x - y ) 2 + ( y - z )2 + ( z - x) 2 ] = 0
4 3 2
35. (d) Cost price = 100x Þx= y=z
110
Þ 95 x ´ = 105 x - 2 Þ x = 4 3x 4 + 7 y 4 + 5z 4 3+7 +5
100 = = 1.
2 2 2 2
5x y + 7 y z + 3z x 2 2 5+ 7 +3
\ cost price = ` 400
43 (d) Since the selling price is same for each article there is a
a 3 - b3
36. (b) = a - b = 0.67 - 0.33 = 0.34 (10)2
a 2 + ab + b2 loss of % = 1%
100
37. (a) a3 + b3 - ab - (a 2 - b2 )2 \ 1% loss.
= (a + b)(a 2 - ab + b2 ) - ab - ( a + b) 2 (a - b) 2 44. (a) ( A + C ) = 2( A + B + C ) - ( A + B) - ( B + C )
Dividing eq by (a + b)
æ1ö 1 1 1
= 2ç ÷ - - =
= (a 2 - 2ab + b2 ) - (a - b)2 è 6 ø 8 12 8
= (a - b)2 - (a - b)2 = 0 [Q a + b = 1] \ A and C can finish the work in 8 days.
y
o
u
rs
Combined Graduate Level Exam 2015-2 – 21

m
a
h
2 2+ x

b
45. (d) A + B =1Þ + =1 A

o
6 8 54. (a)

o
b
.w
1
Þ x=3 hours.

o
3

rd
46. (d) Number = xy Þ y = 2x

p
re
1

s
x + 2 x - 2 = (10 x + 2x) Þ x = 2 Þ y = 4

s
.c
6

o
\ Number = 24. B 3 cm D 4 cm C

m
47. (b) cot 41 . cot 42 . cot 43 . cot 44 . cot 45 . cot(90 – 44) .
cot(90 – 43) . cot(90 – 42) . cot(90 – 41) AD = BD × CD = 3 ´ 4 = 12 = 2 3 cm
= 1 . 1 . 1 . 1 . 1 = 1. [ Q tanq.cotq = 1] 2 3 4
48. (a) CP of 15 oranges = 5 (40) = ` 200 + +
5 7 9 = 401 .
CP of 15 oranges = 3 (60) = ` 180 55. (d) 3 4 5 544
Total CP of 30 oranges = ` 580 + +
5 7 9
SP of 30 oranges = 10(50) = ` 500
56. (d) A : B = 2 : 3; B : C = 3 : 7
500 - 380 ÞA:B:C=2:3:7
\ Profit (%) = ´ 100 » 32%
380 \ ( A + B ) : ( B + C ) : (C + A) = 5 :10 : 9.
60 1 2 l 1
49. (d) Side of the rhombus, a = = 15 cm. 57. (c) pr h = prl Þ =
4 3 hr 3
d12 + d22 = 4a 2 Þ (24)2 + d 22 = 4(15)2 1 1 r 2 + h2 æ l ö æ1ö
2
1
2
Þ + = =ç ÷ =ç ÷ = .
Þ d2 = 18 h2 r2 h2 r 2 è hr ø è 3 ø 9
58. (a) Speed of train Q = S km/hr.
1
\ Area of the rhombus = (24)(18) 162 1
2 Þ = 6 Þ S = 9 km/hour..
= 216 cm2. ( S + 8) + S 2
2
59. (d) tan q = cot q Þ q = 45°
æ 5 ö æ 5 ö
50. (b) 17640 ç1 + ÷ + 17640 = P ç1 + ÷ tan(q + 15°) tan 60° 3
è 100 ø è 100 ø = = = 3.
tan(q - 15°) tan 30° 1
205 100 100
Þ P = 17640 ´ ´ ´ = ` 32,800. 3
100 105 105
d 3d æ 22 ö 2
ç ÷ (2) (56) 2
4 + 4 = 7 Þ d = 80 km. è 7 ø = .
51. (b) 60. (b)
10 12 48 ´ 16.5 ´ 4 9

A x3 - y3
61. (b) = x- y =5 ...(1)
52. (b) x 2 + xy + y 2

x2 - y 2
O = x+ y = 7 ...(2)
x- y
From (1) and (2), x = 6, y = 1

B C 2 x 2(6) 4
\ = = .
3 y 3(1) 1
ÐBAC = x Þ ÐBOC = 2 x 1 1
62. (a) x= a+ ; y= a-
180 - 2 x a a
Þ ÐOBC = = 90 - x
2
2
\ÐOBC + ÐBAC = 90 - x + x = 90°. Þ x + y = 2 a; x - y =
a
R
8
P´ ´R x4 - x2 y 2 - 1 + y 4 - x 2 y 2 + 1
53. (a) P= 2 Þ R1 = 8%.
25 100 = x4 + y 4 - 2 x2 y 2
y
o
u
rs
2015-2 – 22 Combined Graduate Level Exam

m
a
h
= ( x 2 - y 2 )2 = [( x + y)( x - y)]2 \ CQ = CR = x + 1.5 (Q BC = 8.5)

b
o
\ DS = DR = 7.7 - x (Q CD = 9.2)

o
2
é æ 2 öù

b
2
= ê(2 a ) ç ÷ ú = 4 = 16.

.w
\ AD = AS + DS = x + 7.7 - x = 7.7 cm.
ë è a øû 72. (a) Volume of the prism = base area × height

o
rd
æ 100 - 5 öæ 100 - 2 ö = 27(27 - 13)(27 - 20)(27 - 21) ´ 9

p
63. (a) 12600 ç ÷ç ÷ = `11, 730.60.

re
è 100 øè 100 ø = 1134 cm3.

s
s
73. (a) 60% of 300 gm = 180 gm (quantity other than sugar)

.c
d d 12 Now, 180 gm is 50% of the new quantity.
- = Þ d = 6 km.

o
64. (a)
5 6 60 \ New quantity = 100% = 360 gm.

m
\ Sugar added = 360 – 300 = 60 gm.
65. (c) x = 3 + 2; y = 3 - 2
4x
Þ xy = 1 and x - y = 2 2 74. (a) A B

x3 - 20 2 - y3 - 2 2
= ( x - y)3 + 3xy ( x - y) - 22 2 2x
= (2 2)3 + 3(1)(2 2) - 22 2 = 0
4 3 3 4
66. (d) p(1 + 6 ) = p(93 - x3 ) D C
3 3 1.5x 4x 1.5x
Þ 729 - x 3 = 217 Þ x = 8 Given,
\ Thickness = 9 – 8 = 1 cm. Area of trapezium = (4x)(2x) + (2x) (1.5x)
67. (d) Side of the cube = a = 176
Þx=4
Þ 3a = 2(6 3) Þ a = 12 \ Diagonal
\ Total surface area = 6a2
= 6 (12)2 = 864 cm2. = (2(4)) 2 + ((5.5)(4))2 = 2 137 .
68. (b) 60% A = 30% Þ B = 2A 75. (b) Length of direct common tangent
2A = 40% C Þ C = 5A
5A = x% A Þ x = 500. = d 2 - (R - r)2
69. (b) A + B =1
= (13) 2 - (11 - 6)2 = 12 cm.
x x + 25 76. (a) Let work be completed in x days
+ =1Þ x = 5
30 36 ATQ
5 x x -8 x -6
æ R ö + + =1
70. (d) 2 p = p ç1 + ÷ 76 32 48
è 100 ø
x -8 x -6
éæ
3 x+ + = 16
R ö ù
5 5
æ R ö 3 2 3
Þ ç1 + ÷ = 2 Þ êç 1 + ÷ ú = (2)
è 100 ø êëè 100 ø úû By solving x = 12 days
A
15
æ R ö 77. (b)
Þ 8 p = p ç1 + ÷ Þ 15 years
è 100 ø
E
P D
B x
71. (c) A x
C
B
F
AD extended meets BC at F.
S ÐADB = ÐBDF = 90°
Q
ÐADB = ÐFDB (BD is the angle bisector)
\ÐBAD = ÐBFD
C Þ DABD and DFBD are congruent.
D R
Þ AD = DF.
Let PQRS be the points of contact of the circle with And DADE is similar to DAFC (Q DE||BC)
sides AB, BC, CD and AD respectively.
AE AD 1 1
Let AP = AS = x Þ BP = BQ = 7 - x (Q AB = 7) = = Þ AE = (12) = 6 cm.
AC AF 2 2
y
o
u
rs
Combined Graduate Level Exam 2015-2 – 23

m
a
h
78. (a) Let the five consecutive integers be 82. (a) Required speed

b
x – 2, x – 1, x, x + 1 and x + 2.

o
22 70 1 1

o
\ Average = x. = 400 ´ 60 ´ 2 ´ ´ ´ ´

b
7 2 100 1000

.w
Average after adding next two numbers
= 52.8 kmph.

o
rd
5x + x + 3 + x + 4
= = x + 1. æ 1 ö 11 æ 3ö

p
(b) 30 ç14 ÷ + 9 + 4a = 35 ç 13 ÷

re
7 83.
è 3 ø 12 è 4ø

s
\ increases by 1.

s
.c
1

o
1 \ a = 10 years = 10 years 4 months.

m
79. (a) (a - 3) - = 5-3 = 2 3
(a - 3)
84. (c) Part of the work done by Q and R in 6 days
3 1
\ ( a - 3) - 7 7
( a - 3)3 6´ = .
60 10
3
æ 1 ö 1 æ 7ö 3
= ç (a - 3) - ÷ + 3( a - 3) \ Remaining ç 1 - ÷ = th of work done by P in 3
è (a - 3) ø ( a - 3) è 10 ø 10
days.
æ 1 ö \ P alone can do the work in 10 days.
ç ( a - 3) - ÷
è ( a - 3) ø
10 ´ 6
= 23 + 3(2) = 14. \ Q alone can do the work in = 15 days.
10 - 6
80. (c) As shown in the figure,
60
15 ´
O 7 = 20
\ R alone can do the work in 60 days.
15 -
7
h \ Difference = (20 – 10) = 10 days.

H
r 85. (d)
C D

h l
A A B
R
30°
h H h( R )
= Þr= ...(1) 10 3
r R H
1 2 1 1
Given that, pr h = pR 2 H - pr 2 h tan 30° =
h
=
1
Þ h = 10 m
3 3 3
10 3 3
Þ R 2 H = 2r 2 h
h 1
sin 30° = = Þ l = 2h
æ h 2 R2 ö l 2
\ R2 H = 2 ç 2 ×h
ç H ÷÷ [from (1)] \ Length of the telegraph post = l + h
è ø
= 3h = 30 m.
1 86. (c) Given 5 cos q + 12sin q = 13 ...(1)
Þ H 3 = 2h3 Þ h = 3
H.
2 Let 12 cos q - 5sin q = k ...(2)
\ The required ratio = h : H – h Squaring and adding eq (1) and (2)
H H : 52 + 122 = 132 + k 2 Þ k = 0
= 3
:H-3
= 1: ( 3 2 - 1) .
2 2 12
\12 cos q = 5sin q tan q =
81. (b) L.C.M. (12, 16, 18, 21) = 1008 5
\ Number to be added = 2(1008) – 2000 = 16
12 12
\ Sum of digits = 1 + 6 = 7. \ sin q = =
2
12 + 5 2 13 .
y
o
u
rs
2015-2 – 24 Combined Graduate Level Exam

m
a
h
1 1 1 O

b
87. (b) a+ =b+ = c+

o
b c a

o
b
1 1 b-c b-c

.w
Þ a -b = - = Þ bc =
a -b 12

o
c b bc D C

rd
a -b c-a

p
Similarly, ab = ; ca =

re
c-a b-c O' 5 E

s
s
2 2 2
\ a b c = (ab)(bc)(ca) A

.c
10 B

o
\ The total surface area

m
a -b b-c c-a
= ´ ´ =1
c - a a -b b - c æ1 ö
88. (d) Let the number be x and 4x = 4 ç ´10 ´ 13 ÷ + (10)(10)
x × 4x = 84 × 21 è2 ø
x2 = (21)2 = 260 + 100 = 360 sqcm.
x = 21 96. (c) The marked price of the toy
Larger number = 4 × 21 = 84 300
89. (b) Part of work completed = =` 375
(1 - 0.2)
70 ´ 24 ´ 8 7
= = . Required profit percentage = 405 – 375
90 ´16 ´12 9
90. (c) 110% of a = 115% of b = 120% of c = k (say). ` 30
= ´ 100% = 8%
`375
1 1 1
\a :b : c = : : 97. (c) The units digit of the product
110 115 120
= (units digit of 7153) × (units digit of 172)
1 1 1 = units digit of 71 = 7 (As 7 is a four cycle).
= : : = 23 ´ 24 : 22 ´ 24 : 22 ´ 23
22 23 24 98. (a) Let the interior and the exterior angles of the regular
\ Loan received by A polygon be l and e respectively.
Þ i - e = 108° ...(1)
23 ´ 24
= ´ 7930 =` 2760. i + e = 180° ...(2)
1586
91. (c) (coseca – sina)(seca – cosa) (tana + cota) (2) – (1) Þ 2e = 7i Þ e = 36°
\ Number of sides of the polygon
æ 1 öæ 1 ö æ sin a cos a ö
=ç - sin a ÷ ç - cos a ÷ ç + ÷ 360°
è sin a ø è cos a ø è cos a sin a ø = = 10°.
36°
æ 1 - sin 2 a öæ 1 - cos 2 a ö æ sin 2 a + cos 2 a ö

ç sin a ÷ç ÷ç cos a ÷÷ çç ÷÷ 5 5
è øè ø è (cos a)(sin a) ø 99. (a) 4- = 4-
1 1
1+ 1+
æ cos 2 a öæ sin 2 a ö æ 1 ö 1 4
=ç =1 3+ 3+
ç sin a ÷ç÷ç cos a ÷÷ ç (cos a)(sin a) ÷ 1 9
è øè øè ø 2+
92. (d) Let the cost price of rice be 100 c per kg. 4
Given that, (1 – 0.1)(100c) = ` 54
Þ 90 c = ` 54 Þ c = ` 0.6. 5 5 ´ 31 32 - 31 1
= 4- = 4- = = .
The required selling price 9 40 8 8
1+
= (1 + 0.2) (100) (` 0.6) = ` 72. 31
93. (c) Let R be the radius of the hemisphere and r be the
radius of the smaller sphere 1
100. (c) Area of DGBC = (Area of DABC )
3
2 3 é4 ù
Þ pR = 4 ê pr 3 ú
3 ë 3 û 1
= (60) = 20 cm2.
3
R3 R
Þ r3 = Þr= . A
8 2
94. (c) Amount of tin in the new alloy
2 1
=
(60) + (100) = 24 + 20 = 44 kg.
5 5
95. (d) The slant height of the pyramid
G
= (OO ')2 + (O ' E )2 = (12)2 + (5) 2
= 13 cm. B C
y
o
u
rs
m
a
SSC

h
Held on 09-08-2015

b
o
o
b
.w
o
rd
p
re
Combined Graduate Level (Tier-I) Exam-2015

s
s
.c
o
m
Time : 2 Hours Max. Marks : 200

PART-A : GENERAL INTELLIGENCE & REASONING 9. Name a single letter, which can be prefixed to the following
words in order to obtain entirely new words ?
DIRECTIONS (Qs. 1-2): In the following two Questions, which TILL TABLE PILE TAB PRING
one set of letters when sequentially placed at the gaps in the (a) S (b) B
given letter series shall complete it? (c) H (d) C
1. ccbab _ caa _ bccc _ a _ 10. Unscramble the following letters to frame a meaningful word.
(a) babb (b) bbba Then find out the correct numerical position of the letters :
(c) baab (d) babc B C U S M E L R N A
2. a_ _ __dba_ _bcad__ _da__ _cd
(a) bccdbcab (b) abcddcba 1 2 3 4 5 6 7 8 9 10
(c) cbcddcba (d) aabbccdd (a) 6 1 4 3 2 5 8 7 9 10
(b) 3 1 5 7 10 4 2 6 9 8
DIRECTIONS (Qs. 3-15): In the following two Questions, a series
(c) 3 9 4 2 8 10 5 1 7 6
is given, with one term missing. Choose the correct alternative
(d) 2 1 3 4 6 8 9 7 5 10
from the given ones that will complete the series.
11. Using the following code and key decode the given coded
3. 4, 6, 10, 16, 24, ? word:
(a) 28 (b) 30 Code : L X P Z J Y Q M N B
(c) 34 (d) 40
4. 3, 5, 9, 17, _____?____ K , Key : b a e s p r h i g t
(a) 26 (b) 65 Coded word : Z B Y X M N Q B
(c) 33 (d) 42 (a) height (b) struggle
5. A train starts from station A and reaches B 15 minutes late (c) straight (d) strength
when it moves with 40 km/hr and 24 minutes late when it 12. In a certain code "MOUSE" is written as "PRUOC". How is
goes 30 km/hr. The distance between the two stations is "SHIFT" written in that code ?
(a) 16 km (b) 18 km (a) VJIDR (b) VIKRD
(c) 21 km (d) 24 km (c) RKIVD (d) VKIDR
6. In a row of men, Manoj is 30th from the right and Kiran is 13. In a certain code, '253' means 'books are old'; '546' means
20th from the left. When they interchange their position, 'man is old' and '378' means 'buy good books'. What stands
Manoj becomes 35th from the right. What is the total number for "are" in that code?
of men in the row? (a) 2 (b) 4
(a) 45 (b) 44 (c) 5 (d) 6
(c) 54 (d) 34 14. If, + stands for division; × stands for addition ; – stands for
7. Seven persons A, B, C, D, E, F and G are standing in a multiplication : ¸ stands for subtraction, which of the
straight line. following is correct ?
D is to the right of G. A. 46 × 6 ¸ 4 – 5 + 3 = 74
C is between A and B. B. 46 – 6 + 4 × 5 ¸ 3 = 71
E is between F and D. C. 46 ¸ 6 × 4 – 5 + 3 = 75.5
There are three persons between G and B. Who is on the D. 46 × 6 – 4 + 5 ¸ 3 = 70.1
extreme left? (a) D (b) B
(a) A (b) B (c) A (d) C
(c) D (d) G 15. If + = ×, – = ¸, × = +, ¸ = –, then which is the correct equation
8. From the given alternative words, select the word which out of the following?
cannot be formed using the letters of the given word: (a) 18 ¸ 6 + 4 – 2 ¸ 3 = 22
CUMBERSOME (b) 18 + 6 – 4 × 2 ÷ 3 = 26
(a) MOUSE (b) SOBER (c) 18 × 6 – 4 + 7 × 8 = 47
(c) ROME (d) MERCY (d) 18 – 6 × 7 ¸ 2 + 8 = 63
y
o
u
rs
2015 - 2 Combined Graduate Level Exam

m
a
h
DIRECTIONS (Qs. 16-22): In the following Five Questions, select 23. Statement : Every school should promote partnerships that

b
the missing number from the given responses. will increase parental involvement and participation for

o
o
16. promoting the growth of children.

b
.w
? Conclusions I : For the growth of the children, parents

o
should be involved in various school activities.

rd
Conclusions II. Involvement of parents in school activities

p
re
137 8 2 has no influence on the growth of the children.

s
(a) Only I follows (b) Only II follows

s
.c
(c) Neither I nor II follows (d) Both I and II follow

o
m
17 24. Statement : Aggressive animals can be trained with care
and affection to behave as the occassion demands.
(a) 97 (b) 907 Conclusions I : Trained dogs cannot be aggressive.
(c) 1097 (d) 9107 Conclusions II : Animals are always aggressive unless care
17. and affection is given to them.
14 12 9 (a) Only I follows (b) Only II follows
11
(c) Neither I nor II follows (d) Both I and II follow
36 54 DIRECTIONS (Qs. 25-30): In the following Six Questions, select
7 3 16 ? the related word/letters/number from the given alternatives.
25. Haematology : Blood : : Phycology : ?
(a) 12 (b) 17 (a) Fungi (b) Fishes
(c) 18 (d) 16 (c) Algae (d) Diseases
18. 9 11 13 26. Pride of Lions : : ________ of cats
13 15 17
(a) Herd (b) School
10 12 14
(c) Clowder (d) Bunch
14 16 18
27. MAN : PDQ : : WAN : ?
11 13 ?
(a) ZDQ (b) NAW
(a) 22 (b) 14
(c) 15 (d) 21 (c) YQD (d) YDQ
28. AEFJ : KOPT : : ? : QUVZ
19. 3 2 2 -1 6 5 (a) GLKP (b) GKLP
4 24 -2 4 0 ? (c) HLKP (d) HKQL
29. 2 : 32 : : 3 : ?
(a) 30 (b) 11 (a) 243 (b) 293
(c) 0 (d) 1 (c) 183 (d) 143
20. 7 5 3
30. D × H : 4 × 8 as M × Q: ?
8 4 9
(a) 12 × 17 (b) 12 × 16
2 8 ?
(c) 13 × 17 (d) 14 × 18
112 160 162
(a) 4 (b) 6 DIRECTIONS (Qs. 31-44): In the following Six Questions, find
(c) 8 (d) 12 the old word/ number/ letters / number pair from the given
21. Pinky walks a distance of 600 mtr. towards east, turns left alternatives.
and moves 500 mtr, then turns left and walks 600 mtr. and 31. (a) Morning (b) Noon
then turns left again and moves 500 mtr and halts. At what (c) Evening (d) Night
distance in metres is the from the starting point? 32. (a) Liberty (b) Society
(a) 2200 (b) 500 (c) Equality (d) Fraternity
(c) 0 (d) 600
33. (a) DWFU (b) EVHS
22. Sunita rode her scooty northwards, then turned left and
(c) HSKP (d) KQNN
then again rode to her left 4 km. She found herself exactly 2
34. (a) CBEF (b) EDGH
kms west of her starting point. How far did she ride
(c) IHKL (d) GFHJ
northwards initially ?
(a) 2 km (b) 4 km 35. (a) 4025 (b) 7202
(c) 5 km (d) 6 km (c) 6023 (d) 5061
DIRECTIONS (Qs. 23-24) : In the following Two Questions, one 36. (a) 96 : 80 (b) 64 : 48
statement is given followed by two conclusions, I and II. You (c) 80 : 60 (d) 104 : 78
have to consider the statement to be true, even if it seems to be at 37. Choose the correct alternative to complete the series.
variance from commonly known facts. You are to decide which Lily, Daisy, Datura, ?
of the given conclusions can definitely be drawn from the given (a) Sun Flower (b) Hibiscus
statement. Indicate your answer. (c) Marigold (d) Jasmine
y
o
u
rs
CGL (Tier-I) 2015 Solved Paper 2015 - 3

m
a
h
38. Which one of the given responses would be a meaningful 44. Q

b
order of the following ?

o
o
1. Elephant 2. Cat P R K

b
.w
3. Mosquito 4. Tiger

o
5. Whale

rd
(a) 5, 3, 1, 2, 4 (b) 1, 3, 5, 4, 2 In the fig.

p
(c) 3, 2, 4, 1, 5 (d) 2, 5, 1, 4, 3 Q represents all quadrilaterals

re
s
39. How many triangles are there in the figure ? K represents all Kites

s
.c
R represents all Rhombus

o
P represents all Parallelogram

m
The statement 'Rhombus is also a Kite' can be described as
(a) P and K is nothing but R
(a) 24 (b) 14 (b) P or K is nothing but R
(c) 28 (d) 20 (c) P and R is nothing but K
40. The figure given on the left hand side is folded to form a (d) P or R is nothing but K
box. Choose from the alternatives (1), (2), (3) and (4) the DIRECTIONS (Qs. 45-50): In the following Two Questions, which
boxes that is similar to the box formed. answer figure will complete the pattern is the questiion figure ?
45. Question figure

(1) (2) (3) (4)

(a) (2) and (3) only (b) (1), (3) and (4) only
(c) (2) and (4) only (d) (1) and (4) only
Answer figures :
41. Find out the number of circles in the given figure :

(a) (b)

(c) (d)
(a) 14 (b) 16
(c) 17 (d) 18 46. Question figure
42. Identify the diagram that best represents the relationship
among the classes given below :
Animals and animals, sea animals

(a) (b)

(c) (d) Answer figures :

1 (a) (b)
Doctors
43. 5 6 2 3 Professors
4
Married People
Which number indicates doctors who are not married ?
(a) 6 (b) 4 (c) (d)
(c) 2 (d) 1
y
o
u
rs
2015 - 4 Combined Graduate Level Exam

m
a
h
47. From the given answer figures, select the one which is 50. Direction : A word is represented by only one set of

b
hidden/embedded in the question figure : numbers as given in any one of the alternatives. The sets of

o
o
Question figure numbers given in the alternatiives are represented by two

b
.w
classes of alphabets as in two matrices given below. The
columns and rows of Matrix I are numbered from 0 to 4 and

o
rd
that of Matrix II are numbered from 5 to 9. A letter from these

p
matrices can be represented first by its row and next by its

re
column, e.g.. 'A' can be represented by 01, 14 etc. and 'O'

s
s
Answer figures : can be represented by 59, 67 etc. similarly, you have to

.c
o
identify the set for the word 'PEARL'.

m
Matrix – I Matrix – II
(a) (b) 0 1 2 3 4 5 6 7 8 9
0 P A G R Z 5 E M L N O
1 G R Z P A 6 L E O M N
(c) (d) 2 Z P A G R 7 O N E L M
3 A G R Z P 8 N O M E L
48. A piece of paper is folded and cut as shown below in the 4 R Z P A G 9 M L N O E
question figures. From the given answer figures indicate
(a) 00, 55, 22, 11, 96 (b) 00, 66, 14, 32, 56
how it will appear when opened.
(c) 13, 77, 30, 14, 88 (d) 12, 88, 43, 32, 89
Question figure
PART-B : GENERAL AWARENESS
51. To whom the line 'A thing of beauty is a joy for ever' is
attributed ?
(a) John Keats (b) Dr. Charles Dickens
Answer figures : (c) Dr. Jonathan Swift (d) William Wordsworth
52. The birthday of which of the following leaders is celebrated
as 'Teachers Day' in India?
(a) Dr. Rajendra Prasad (b) S. Radhakrishnan
(a) (b) (c) C. Rajgopalachari (d) Lala Lajpat Rai
53. The award given for outstanding performance in sports is
(a) Bharat Ratna (b) Padma Shri Award
(c) Arjuna Award (d) Dronacharya Award
54. Which hill station's name means place of the thunderbolt'?
(a) Shillong (b) Oottacamand
(c) (d)
(c) Darjeeling (d) Gangtok
55. The ship building yard––Mazgaon Dock is located at –
49. If a mirror is places on the line MN, then which of the answer (a) Kochi (b) Kolkata
figures is the right image of the given figure ? (c) Mumbai (d) Vishakhapatnam
Question figure: 56. Electric current is measured using which of the following
instrument ?
M
(a) Voltmeter (b) Anemometer
(c) Wattmeter (d) Ammeter
57. 'Agha Khan Cup' is related with which of the following
sport event ?
N (a) Cricket (b) Hockey
Answer figures : (c) Table Tennis (d) Football
58. Where was the first conference of SAARC (South Asian
Association for Regional Cooperation) held ?
(a) Dhaka (b) New Delhi
(a) (b) (c) Colombo (d) Kathmandu
59. Which among the following is not a Bretton Woods
Institution ?
(a) International Monetary Fund (IMF)
(b) World Bank
(c) Organisaiton of Economic Cooperation an d
(c) (d) Development (O.E.C.D.)
(d) None of these
y
o
u
rs
CGL (Tier-I) 2015 Solved Paper 2015 - 5

m
a
h
60. Equilibrium price in the market is determined by the 72. Jallianwala incident took place at

b
(a) equality between total cost and total revenue (a) Lucknow (b) Surat

o
o
(b) equality between average cost and average revenue. (c) Amritsar (d) Allahabad

b
.w
(c) equality between marginal cost and marginal revenue 73. Who was the founder of Lodhi dynasty ?
(d) equality between marginal cost and average cost.

o
(a) Sikandar Lodhi (b) Bahlol Lodhi

rd
61. In the national context which of the following indicates (c) Ibrahim Lodhi (d) Daulat Khan Lodhi

p
Macro Approach ? 74. Which one of the following pair is not correctly matched ?

re
(a) Sales of Bata Shoe Company

s
(a) Akbar – Todarmal

s
(b) Exports of Mangoes to U.K.

.c
(b) Chanakya – Chandragupta

o
(c) Income from Railways (c) Vikramaditya – Chaitanya

m
(d) Inflation in India (d) Harshvardhan – Hiuen Tsang
62. Internal economies 75. The South East trade winds are attracted towards the Indian
(a) arise in an economy as it makes progress sub continent in the rainy season due to
(b) accrue to a firm when it expands its output (a) the effect of easterlies
(c) arise when there is expansion in internal trade
(b) the effect of Northern–East trade winds
(d) arise when there is expansion in an industry
(c) the presence of low atmospheric pressure over North–
63. One of the features of a free market economy is
West India
(a) public ownership of factors of production
(d) the development of cyclone over the equator
(b) rationing and price control
76. The 'graded profile' of a river course is a
(c) consumer's sovereignty
(a) smooth curve in the upper course
(d) active state intervention
64. Gross National Product – Depreciation Allowance = ? (b) smooth curve in the middle course
(a) Gross Domestic Product (c) smooth curve in the lower course
(b) Personal Income (d) smooth curve from source to mouth
(c) Net National Product 77. Sink hole is a phenomenon of _______ topography.
(d) Per Capita Income (a) Desert (b) Tundra
65. The Panchayat Samiti remains accountable for its functions (c) Karst (d) Plain
to 78. Kerala is famous for the cultivation of
(a) The Gram Panchayats and Gram Sabhas 1. Coconut 2. Black pepper
(b) Zilla Parishads 3. Rubber 4. Rice
(c) Anchal Panchayats (a) 1, 2 and 4 (b) 2, 3 and 4
(d) Janpad Panchayats (c) 1 and 4 (d) 1, 2 and 3
66. The legislature gains a priority over the executive in 79. The longest continental Railway in the world is
(a) A Federal Government (a) Trans Siberian Railway
(b) An Authoritarian Government (b) Canadian Pacific Railway
(c) A Parliamentary Government (c) Canadian National Railway
(d) A Presidential Government (d) Trans Atlantic Railway
67. The legislature in a democratic country can influence public 80. Photoperiodisrn affects
opinion by (a) Flowering (b) Vegetative growth
(a) Granting rights (c) Fruiting (d) All of these
(b) Enacting non controversial laws 81. Match the following :
(c) Defining the duties of the citizens I II
(d) Focusing attention on public issues A. Ascorbic acid 1. Photosynthetic
68. If the President wants to resign from his office, he may do pigment
so by writing to the B. Chlorophyll 2. Quencher
(a) Vice President (b) Chief Justice of India C. Carotenoid 3. Enzyme
(c) Prime Minister (d) Speaker of Lok Sabha D. Superoxide 4. Vitamin–C
69. Which of the following is not a Union Territory ? dismutase
(a) Lakshadweep (b) Puducherry A B C D
(c) Nagaland (d) Dadra and Nagar Haveli (a) 4 2 1 3
70. The greatest king of the Pratihara dynasty was (b) 2 4 1 3
(a) Bhoj (Mihir–Bhoj) (b) Dantidurga (c) 4 1 3 2
(c) Nagbhatta II (d) Vatsaraj (d) 4 1 2 3
71. In 1939 Subhash Chandra Bose was elected as President of 82. Allantois of Embryo helps in
the Congress Party defeating (a) respiration (b) excretion
(a) Jawaharlal Nehru (c) protection (d) digestion
(b) Maulana Abul Kalam Azad 83. Which one of the following animals belongs to mollusca ?
(c) V.B. Patel (a) Hare (b) Hydra
(d) Pattabhi Sitharamayya (c) Hyla (d) Haliotis
y
o
u
rs
2015 - 6 Combined Graduate Level Exam

m
a
h
84. Outside the nucleus DNA is found in 97. The term 'brown air' is used for

b
(a) Mitochondria (b) Ribosome (a) Photochemical smog (b) Sulfurous smog

o
o
(c) Endoplasmic reticulum (d) Golgi bodies (c) Industrial smog (d) Acid fumes

b
.w
85. Animal protein is called first class protein because it is 98. Which of the following is FALSE with respect to rain water
(a) delicious in taste harvesting?

o
rd
(b) cheaper in the market (a) It helps raising water table

p
(c) rich in essential amino acids (b) It helps meet rising water demand

re
(d) easily digestible (c) It increases run–off losses

s
s
86. It is easy to burst a gas filled balloon with a needle than (d) It is a device of water conservation

.c
o
with a nail. It is because 99. Peroxyacetyl nitrate is a

m
(a) nail exerts more pressure than needle on the balloon (a) Plant hormone (b) Vitamin
(b) needle exerts more pressure than nail on the balloon (c) Secondary pollutant (d) Acidic dye
(c) gas is reactive with the needle 100. Which of the following river does not originate in Indian
(d) nail is more longer than needle territory ?
87. The velocity of sound in moist air is more than in dry air (a) Mahanadi (b) Brahmaputra
because the moist air has (c) Satluj (d) Ganga
(a) less pressure than dry air
PART-C : QUANTITATIVE APTITUDE
(b) more pressure than dry air
(c) more density than dry air 101. In DABC, a line through A cuts the side BC at D such that
(d) less density than dry air BD : DC = 4 : 5. If the area of DABD = 60 cm2 , then the area
88. X–rays can be used of DADC is
(a) to detect heart diseases. (a) 50 cm2 (b) 60 cm2
(c) 75 cm 2 (d) 90 cm2
(b) to detect defects in precious stones and diamonds.
(c) to detect gold under the earth. 102. A tangent is drawn to a circle of radius 6cm from a point
(d) for cutting and welding of metals. situated at a distance of 10 cm from the centre of the circle.
89. Ice is packed in saw dust because The length of the tangent will be
(a) saw dust is poor conductor of heat. (a) 4 cm (b) 5 cm
(b) saw dust is a good conductor of heat. (c) 8 cm (d) 7 cm
(c) saw dust does not stick to the ice. 103. A ship after sailing 12km towards south from a particulari
(d) saw dust will not get melted easily. place covered 5 km more towards east. Then the straightway
90. What is used to identify whether a data word has an odd or distance of the ship from that place is
even number of 1's ? (a) 18 km (b) 15 km
(a) Sign bit (b) Zero bit (c) 13 km (d) 11km
(c) Parity bit (d) Carry bit 104. Two poles of height 7 m and 12 m stand on a plane ground.
91. Rearranging and allocating space in memory to provide for If the distance between their feet is 12 m, the distance
multiple computing tasks is called between their top will be
(a) Multiprogramming (b) Multitasking (a) 13 m (b) 19 m
(c) Memory Management (d) Networking (c) 17 m (d) 15 m
92. What happens when a drop of glycerol is added to crushed 105. The maximum value of sin4q + cos4q is
KMnO4 spread of a paper ? (a) 1 (b) 2
(a) There is a violent explosion 1
(c) 3 (d)
(b) There is no reaction 3
(c) The paper ignites 106. Find the value of
(d) There is a crackling sound. tan 4° tan 43° tan 47° tan 86°
93. Most commonly used bleaching agent is 1
(a) Alcohol (b) Carbon dioxide (a) 1 (b)
2
(c) Chlorine (d) Sodium chloride
94. The least penetrating power ray is 2
(c) 2 (d)
(a) a–Ray (b) b–Ray 3
(c) g–Ray (d) X–Ray 107. The sum of four numbers is 48. When 5 and 1 are added to
95. Hydrogen peroxide is an effective sterilizing agent. Which the first two; and 3 & 7 are subtracted from the 3rd & 4th,
one of the following product results when it readily loses the numbers will be equal. The numbers are
active oxygen ? (a) 4, 12, 12, 20 (b) 5, 11, 13, 19
(a) Water (b) Hydrogen (c) 6, 10, 14, 18 (d) 9, 7, 15, 17
(c) Ozone (d) Nasant Hydrogen 108. The least number that should be added to 2055 so that the
96. The maximum fixation of solar energy is done by sum is exactly divisible by 27 :
(a) Bacteria (b) Fungi (a) 24 (b) 27
(c) Green plants (d) Protozoa (c) 31 (d) 28
y
o
u
rs
CGL (Tier-I) 2015 Solved Paper 2015 - 7

m
a
h
109. A and B together can do a piece of work in 6 days. If A can 119. 729 ml of a mixture contains milk and water in the ratio 7 : 2.

b
alone do the work in 18 days, then the number of days How much more water is to be added to get a new mixture

o
o
required for B to finish the work is containing milk and water in the ratio 7 : 3 ?

b
.w
(a) 12 (b) 9 (a) 60 ml (b) 71 ml
(c) 52 ml (d) 81 ml

o
(c) 15 (d) 10

rd
110. A pipe can fill a tank in x hours and another can empty it in 120. The average weight of 15 oarsmen in a boat is increased by

p
1.6 kg when one of the crew, who weighs 42 kg is replaced

re
y hours. They can together fill it in (y > x)
by a new man. Find the weight of the new man (in kg).

s
(a) x – y (b) y – x

s
(a) 65 (b) 66

.c
xy xy (c) 43 (d) 67

o
m
(c) (d) 121. What is the Arithmetic mean of the first 'n' natural numbers?
x-y y-x
111. A tap can empty a tank in 30 minutes. A second tap can n +1 n 2 (n + 1)
empty it in 45 minutes. If both the taps operate (a) (b)
2 2
simultaneously, how much time is needed to empty the tank?
(a) 18 minutes (b) 14 minutes n(n + 1)
(c) 2 (n + 1) (d)
(c) 15 minutes (d) 30 minutes 2
112. The perimeter of one face of a cube is 20 cm. Its volume will 122. A shopkeeper bought 30 kg of rice at the rate of `70 per kg
be and 20 kg of rice at the rate of `70.75 per kg. If he mixed the
(a) 100 cm3 (b) 125 cm3 two brand of rice and sold the mixture at `80.50 per kg. his
(c) 400 cm3 (d) 625 cm3 gain is
(a) `510 (b) `525
113. If the area of a circle is A, radius of the circle is r and
(c) `485 (d) `450
circumference of it is C, then
123. The population of a town increases by 5% every year. If the
C r present population is 9261, the population 3 years ago was
(a) rC = 2A (b) =
A 2 (a) 5700 (b) 6000
(c) 7500 (d) 8000
r2 A 124. A farmer travelled a distance of 61 km in 9 hrs. He travelled
(c) AC = (d) =C partly on foot at the rate of 4 km/hr and partly on bicycle at
4 r
the rate of 9 km/hr. The distance travelled on foot is
114. A square is inscribed in a quarter–circle in such a manner
(a) 17 km (b) 16 km
that two of its adjacent vertices lie on the two radii at an
(c) 15km (d) 14 km
equal distance from the centre, while the other two vertices
125. Walking at the rate of 4 kmph a man covers certain distance
lie on the circular arc. If the square has sides of length x,
in 2hrs 45 min. Running at a speed of 16.5 kmph the man will
then the radius of the circle is
cover the same distance in how many minutes ?
16x 2x (a) 35 min. (b) 40 min.
(a) (b) (c) 45 min. (d) 50 min.
p+4 p
1
5x 126. In certain years a sum of money is doubled itself at 6 %
(c) (d) 4
2x
2 simple interest per annum, then the required time will be
115. 10% discount and then 20% discount in succession is 1
equivalent to total discount of (a) 12 years (b) 8 years
2
(a) 15% (b) 30%
(c) 24% (d) 28% 2
(c) 10 years (d) 16 years
116. The marked price of a watch was `720. A man bought the 3
same for `550.80 after getting two successive discounts, 127. The length of the portion of the straight line 3x + 4y = 12
the first being 10%. The second discount rate is intercepted between the axes is
(a) 12% (b) 14% (a) 3 (b) 4
(c) 15% (d) 18% (c) 7 (d) 5
117. Allowing 20% and 15% successive discounts, the selling 128. The value of
price of an article becomes `3,060; then the marked price 1 1 1 1 1
will be - + - + is
7- 6 6- 5 5-2 8 - 7 3- 8
(a) `4,400 (b) `5,000
(a) 0 (b) 1
(c) `4,500 (d) `4,000
(c) 5 (d) 7
118. Eighteen years ago, the ratio of A's age to B's age was 8 : 13.
129. If m = –4, n = –2, then the value of m3 – 3m2 + 3m + 3n + 3n2
Their present ratio's are 5 : 7. What is the present age of A? + n3 is
(a) 70 years (b) 50 years (a) 124 (b) –124
(c) 40 years (d) 60 years (c) 126 (d) –126
y
o
u
rs
2015 - 8 Combined Graduate Level Exam

m
a
h
130. 2x – ky + 7 = 0 and 6x – 12y + 15 = 0 has no solution for
cos2 45° cos2 60° tan 2 30°

b
(a) k = – 4 (b) k = 4

o
141. The numerical value of + – –

o
(c) k = 1 (d) k = – 1 sin 2 60° sin 2 45° cot 2 45°

b
.w
131. Choose the incorrect relation(s) from the following :
sin 2 30°

o
(i) 6+ 2 = 5+ 3

rd
is
cot 2 30°

p
(ii) 6+ 2 < 5+ 3

re
3 1

s
(iii) 6+ 2 > 5+ 3 (a) (b)

s
.c
4 4
(a) (i) (b) (ii)

o
m
(c) (i) and (iii) (d) (ii) and (iii) 1 1
132. If x = 332, y = 333, z = 335, then the value of x3 + y3 + z3 – (c) (d) 1
2 4
3xyz is 142. If x cosq– sin q = 1, then
(a) 7000 (b) 8000 x2 + (1 + x2) sinq equals
(c) 9000 (d) 10000 (a) 1 (b) –1
1 (c) 0 (d) 2
133. If 2 + x 3 = , then the simplest value of x is 143. A 10 m long ladder is placed against a wall. It is inclined at
2+ 3
an angle of 30° to the ground. The distance (in m) of the
(a) 1 (b) –2
(c) 2 (d) –1 foot of the ladder from the wall is (Given 3 = 1.732)
(a) 7.32 (b) 8.26
m - a 2 m - b2 m - c 2 (c) 8.66 (d) 8.16
134. If 2 2 + 2 + = 3, then the value of m is
b +c c + a 2 a 2 + b2 DIRECTIONS (Qs. 144-150) : Study the following bar diagram
(a) a2 + b2 (b) a2 + b2 + c2 carefully and answer the following Four Questions.
2
(c) a – b – c2 2 (d) a2 + b2 – c2 The number of the production of electronic items (TVs and LCDs)
135. The measure of an angle whose supplement is three times in a factory during the period from 2009 to 2013.
as large as its complement, is
TVs
(a) 30° (b) 45°
(c) 60° (d) 75° LCDs
136. The sides of a triangle having area 7776 sq. cm are in the
ratio 3:4:5. The perimeter of the triangle is 13,000
(a) 400 cm (b) 412 cm 12,000
(c) 424 cm (d) 432 cm
11,000
137. Two chords of length a unit and b unit of a circle make
angles 60° and 90° at the centre of a circle respectively, then 10,000
9,400
No. of productions

the correct relation is 9,000


(a) b = 2a (b) b = 2a 8,000
7 ,000
3
(c) b = 3a a
(d) b = 6,000
2
138. In a parallelogram PQRS, angle P is four times of angle Q,
then the measure of ÐR is
(a) 36° (b) 72°
(c) 130° (d) 144°
139. If sin q + sin 2 q = 1 then cos 2 q + cos 4 q is equal to
2009
20112010 2012 2013
sin q Years
(a) 1 (b)
cos 2 q 144. The total number of production of electronic items is
maximum in the year
cos 2 q (a) 2009 (b) 2010
(c) (d) None
sin q (c) 2011 (d) 2013
140. If a clock started at noon, then the angle turned by hour 145. The ratio of production of LCDs in the year 2011 and 2013 is
hand at 3.45 PM is (a) 3 : 4 (b) 4 : 3
1° 1° (c) 2 : 3 (d) 1 : 4
(a) 104 (b) 97 146. The difference between averages of production of TVs and
2 2
LCDs from 2009 to 2012 is
1° 1° (a) 600 (b) 700
(c) 112 (d) 117
2 2 (c) 800 (d) 900
y
o
u
rs
CGL (Tier-I) 2015 Solved Paper 2015 - 9

m
a
h
147. The ratio of production of TVs in the years 2009 and 2010 is DIRECTIONS (Qs. 156-165): In the following Ten Questions, a

b
(a) 7 : 6 (b) 6 : 7 sentence/a part of the sentence is underlined. Below are given

o
o
(c) 2 : 3 (d) 3 : 2 alternatives to the underlined part which may improve the

b
.w
The following pie–chart shows the sources of funds to sentence. Choose the correct alternative. In case no improvement
collected by the National Highways Authority of India is needed blacken the circle [ ] corresponding to "No

o
rd
(NHAI) for its Phase II projects. Study the pie–chart and improvement".

p
answer the following Three Questions:

re
156. It's high time that you go home.

s
(a) are going (b) went

s
.c
External (c) gone (d) No improvement

o
Assistance 157. Drinking tea is an English habit.

m
11,486 (a) tradition (b) convention
Market SPVS (c) custom (d) No improvement
Borrowing 5252
158. He is wanting ina little common sense.
29,952 Toll (a) lacks (b) is lacking in
4910 (c) needs some (d) No improvement
Annuity
6000 159. The second pigeon flew just at the first pigeon had flown.
(a) one had done (b) one had flown away
(c) had done (d) No improvement
148. If the toll is to be collected through an outsourced agency 160. The old man has acquired experience through age.
by allowing a maximum 10% commission, how much amount (a) developed experience (b) experienced
should be permitted to be collected by the outsourced (c) got experience (d) No improvement
agency, so that the project is supported with ` 4,910 crores? 161. Water and soil pollutants find their entry into the body
(a) ` 6,213 crores (b) ` 5,827.crores through ingestion of contaminated water or food.
(c) ` 5,401crores (d) ` 5,316 crores (a) digestion of contaminated
149. If NHAI could receive a total of `9,695 crores as External (b) injection of contaminated
Assistance, by what percent (approximately) should it (c) passage of contaminated
increase the Market Borrowing to arrange for the shortage (d) No improvement
of funds? 162. He lives far from the station.
(a) 4.5% (b) 7.5% (a) away from the station
(c) 6% (d) 8% (b) a long way from the station
150. The central angle corresponding to Market Borrowing is (c) off the station
(a) 52° (b) 137.8° (d) N o improvement
(c) 187.2° (d) 192.4° 163. Mumbai is famous because of its textiles.
(a) at (b) in
PART-D : ENGLISH COMPREHENSION (c) for (d) No improvement
164. They spoke about the weather.
DIRECTIONS (Qs. 151-155): In the following Five Questions, (a) said (b) talked
four alternatives are given for the Idiom/Phrase underlined in (c) expressed (d) No improvement
the sentence. Choose the alternative which best expresses the 165. Mr. Mukherjee knows ten languages, isn't it'?
meaning of the Idiom\Phrase and mark it in the Answer Sheet. (a) doesn't Mr. Mukherjee (b) hasn't he
151. At his wit's end, he turned to his old trade. (c) doesn't he (d) No improvement
(a) After careful thinking DIRECTIONS (Qs. 166-172): In the following Seven Questions,
(b) Confidently out of the four alternatives, choose the one which can be
(c) Not knowing what to do substituted for the given words/sentences and indicate it by
(d) Overjoyed blackening the appropriate circle [ ] in the Answer Sheet
152. Then it comes to dancing, he is all–thumbs.
166. A recurrent Compulsive urge to steal.
(a) an expert (b) a trainer
(a) Pneumonia (b) Insomnia
(c) clumsy (d) lazy (c) Nymphomania (d) Kleptomania
153. He usually goes to bed very early and rises with the lark. 167. Act of injuring another's reputation by any slanderous
(a) very late (b) very early communication.
(c) after sunrise (d) at midnight (a) Orchestration (b) Aberration
154. Their attempt to get back the stolen necklace proved to be (c) Misrepresentation (d) Defamation
a wild goose chase. 168. A story in which animals or objects speak and give
(a) wise decision (b) useless search wholesome moral lesson.
(c) timely action (d) delayed action (a) Fable (b) Parable
155. The day the new product was launched, people made a (c) Allegory (d) Legend
beeline to purchase it. 169. Medical Study of skin and its diseases.
(a) rushed (b) were doubtful (a) Dermatology (b) Endocrinology
(c) refused (d) went online (c) Gynaecology (d) Orthopaedics
y
o
u
rs
2015 - 10 Combined Graduate Level Exam

m
a
h
170. A process involving too much official formality. 179. How, accordmg to the author, can one suppress anger ?

b
(a) Diplomacy (c) Bureaucracy (a) By holding one's anger.

o
o
(c) Red–tapism (d) Nepotism (b) By thinking about one's anger

b
.w
171. A person who enters without any invitation. (c) By converting anger.
(d) By redirecting anger.

o
(a) Burglar (b) Intruder

rd
(c) Thief (d) Vandal PASSAGE –II (5 Questions)

p
172. Not suitable for eating

re
The crowd surged forward through the narrow streets of Paris.
(a) Tasteless (b) Uneatable

s
There was a clatter of shutters being closed hastily by trembling

s
.c
(c) Inedible (d) Spicy hands the citizens of Paris knew that once the fury of the people

o
DIRECTIONS (Qs. 173-174): In the following Two Questions, was excited there was no telling what they might do. They came

m
four words are given each question, out of which only one word to an old house which had a workshop on the ground floor. A
is correctly spelt. Find the correctly spelt word and mark your head popped out of the door to see what it was all about "Get
answer, in the Answer Sheet. him! Get Thimonier! Smash his devilish machines!" yelled the
crowd.
173. (a) Acquariam (b) Aquarium They found the workshop without its owner. M. Thimonier
(c) Acquerium (d) Aquarim had escaped by the back door. Now the fury of the demonstrators
174. (a) Facsimile (b) Facsimilee turned against the machines that were standing in the shop, ready
(c) Fasimile (d) Fasimmile to be delivered to buyers. They were systematically broken up
DIRECTIONS (Qs. 175-184): In the following Ten Questions, and destroyed – dozens of them. Only when the last wheel and
you have two passages with 5 questions following each passage. spindle had "been trampled under foot did the infuriated crowd
Read the passages carefully and choose the best answer to each recover their senses.
question out of the four alternatives and mark it by blackening "That is the end of M'Sieur Thimonier and his sewing
the appropriate circle [ ] in the Answer Sheet. machines," they said to one another and went home satisfied.
Perhaps now they would find work, for they were all unemployed
PASSAGE – I (5 Questions) tailors and seamstresses who believed that their livelihood was
The instinctive, natural way to express anger is to respond threatened by that new invention.
aggressively. Anger is a natural, adaptive response to threats; It 180. The passage throws light on
inspires powerful, often aggressive, feelings and behaviours, (a) why inventions should be avoided.
which allow us to fight and to defend ourselves when we are (b) how a well meant invention can be misunderstood
attacked. On the other hand, we can't physically lash out at every (c) what mischief an inventor can do to ordinary people.
person or object that irritates or annoys us; laws, social norms (d) how dangerous an invention can be.
and common sense place limits on how far our anger can take us. 181. The crowd was protesting against
People use a variety of both conscious and unconscious (a) the closings of workshops.
processes to deal with there angry feelings. The three main (b) the misdoings of Thimonier.
approaches are expressing, suppressing and calming. Expressing (c) the newly invented sewing machine
your angry feelings in an assertive – not aggressive manner is (d) Thimonier for keeping the invention a secret
182. The aim of the crowd was to
the healthiest way to express anger. Being assertive doesn't '
(a) kill Thimonier
mean being 'pushy or demanding; It means being respectful of
(b) drive Thimonier away
yourself and others. Anger can be suppressed and then converted (c) humiliate Thimonier
or redirected. This happens when you hold in your anger, stop (d) destroy the sewing machines
thinking about it and focus on something positive. 183. The people thought that
175. How does a person naturally express anger? (a) their lives were in danger.
(a) By inspiring powerful feelings (b) Thimonier was mad.
(b) By responding aggressively (c) the sewing machine was dangerous.
(c) By defending oneself (d) they would be deprived of their livelihood.
(d) By adapting strong behaviour. 184. Shutters were being closed because the shopkeepers
176. Which one of the following places limits on how far we can (a) wanted to attack the crowd.
take our anger? (b) wanted to protect Thimonier.
(a) Behaviour (b) Feelings (c) feared their shops would be destroyed.
(c) Instinct (d) Law (d) wanted to support the crowd.
177. According to the author, how should people deal with their DIRECTIONS (Qs. 185-189) : In the following Five Questions,
anger? some parts of the sentences have errors and some are correct.
(a) Express it assertively Find out which part of a sentence has an error and blacken the
(b) Express it aggressively circle [ ] corresponding to the appropriate correct option. If a
(c) Expressing consciously sentence is free from error, blacken the circle [ ] corresponding
(d) Expressing unconsciously to "No Error" in the Answer Sheet.
178. What does the author mean by being assertive? 185. I shall look forward to being with you sometime next month.
(a) Being pushy (b) Being demanding (a) I shall look forward (b) to being with you
(c) Being respectful (d) Being calm (c) sometime next month (d) No error
y
o
u
rs
CGL (Tier-I) 2015 Solved Paper 2015 - 11

m
a
h
186. I really do regret not to learn to play the violin when I had 192. The deserted garden was infested ________ weeds.

b
so many opportunities to learn the practice in school. (a) for (b) into

o
o
(a) I really do regret not to learn to play the violin . (c) on (d) with

b
.w
(b) when I had so many opportunities 193. He went there _______ .
(a) walking on foot (b) by foot

o
(c) to learn and practice in school.

rd
(d) No error (c) with foot (d) on foot

p
187. The government granted relief payments of `5,000 each to 194. After your long illness I am happy to see you.

re
(a) up and about (b) by and by

s
those who injured in the fire accident.

s
(c) up and down (d) up and round

.c
(a) The government granted

o
(b) relief payments of `5,000 each DIRECTIONS (Qs. 195-197): In the following Three Questions,

m
(c) to those who injured in the fire accident. out of the four alternatives, choose the one which best expresses
(d) No error the meaning of the given word and mark it in the Answer Sheet.
188. By May next year I have been working in this college for 195. Devout
twenty years. (a) Solemn (b) Loyal
(a) By May next year
(c) Dedicated (d) Pious
(b) I have been working in this college
196. Predilection
(c) for twenty years.
(a) Favour (b) Whim
(d) No error
189. Everyone of us should realize that any act of negligence (c) Prejudice (d) Preference
will cause a great harm to our country's security. 197 Effigy
(a) Everyone of us should realize that (a) Dummy (b) Imagery
(b) any act of negligence will cause (c) Reflection (d) Organ
(c) a great harm to our country's security DIRECTIONS (Qs. 198-200): In the following three Questions,
(d) No error choose the word opposite in meaning to the given word and
DIRECTIONS (Qs. 190-194): In the following Five Questions, mark it in the answer sheet.
sentences given with blanks to be filled in with an appropriate 198. Tremulous
word(s). Four alternatives are suggested for each questton. (a) Steady (b) Obese
Choose the correct alternative out of the four and indicate it by (c) Young (d) Healthy
blackening the appropriate circle [ ] in the Answer Sheet.
199. Fake
190. You must ______ your career with all seriousness. (a) Fanciful (b) Real
(a) follow (b) complete
(c) Ideal (d) Wrong
(c) direct (d) pursue
200. Disconsolate
191. Making pies and cakes _____ Mrs. Reddy's speciality.
(a) are (b) were (a) Joyous (b) Thankful
(c) is (d) has (c) Unprejudiced (d) Prominent

ANS WER KEY


1 (a) 21 (c) 41 (c) 61 (d) 81 (d) 101 (c) 121 (a) 141 (a) 161 (d) 181 (c)
2 (a) 22 (b) 42 (d) 62 (d) 82 (b) 102 (c) 122 (a) 142 (a) 162 (d) 182 (d)
3 (c) 23 (a) 43 (d) 63 (c) 83 (d) 103 (c) 123 (d) 143 (c) 163 (c) 183 (d)
4 (c) 24 (c) 44 (a) 64 (c) 84 (a) 104 (a) 124 (b) 144 (c) 164 (b) 184 (c)
5 (b) 25 (c) 45 (a) 65 (b) 85 (c) 105 (a) 125 (b) 145 (a) 165 (c) 185 (b)
6 (c) 26 (c) 46 (c) 66 (c) 86 (b) 106 (a) 126 (d) 146 (d) 166 (d) 186 (a)
7 (d) 27 (a) 47 (b) 67 (d) 87 (c) 107 (c) 127 (d) 147 (c) 167 (d) 187 (c)
8 (d) 28 (b) 48 (c) 68 (a) 88 (b) 108 (a) 128 (c) 148 (c) 168 (a) 188 (b)
9 (a) 29 (a) 49 (a) 69 (c) 89 (a) 109 (b) 129 (d) 149 (c) 169 (a) 189 (d)
10 (c) 30 (c) 50 (a) 70 (a) 90 (c) 110 (d) 130 (b) 150 (c) 170 (c) 190 (d)
11 (c) 31 (d) 51 (a) 71 (d) 91 (c) 111 (a) 131 (c) 151 (c) 171 (b) 191 (c)
12 (d) 32 (b) 52 (b) 72 (c) 92 (a) 112 (b) 132 (a) 152 (c) 172 (c) 192 (d)
13 (a) 33 (a) 53 (c) 73 (b) 93 (c) 113 (a) 133 (d) 153 (b) 173 (b) 193 (d)
14 (b) 34 (d) 54 (c) 74 (c) 94 (a) 114 (d) 134 (b) 154 (b) 174 (a) 194 (a)
15 (b) 35 (d) 55 (c) 75 (c) 95 (a) 115 (d) 135 (b) 155 (a) 175 (b) 195 (d)
16 (c) 36 (a) 56 (d) 76 (d) 96 (c) 116 (c) 136 (d) 156 (b) 176 (b) 196 (d)
17 (c) 37 (d) 57 (d) 77 (c) 97 (a) 117 (c) 137 (a) 157 (d) 177 (a) 197 (a)
18 (c) 38 (c) 58 (a) 78 (d) 98 (c) 118 (b) 138 (d) 158 (a) 178 (c) 198 (a)
19 (c) 39 (c) 59 (c) 79 (a) 99 (c) 119 (d) 139 (a) 159 (a) 179 (a) 199 (b)
20 (b) 40 (b) 60 (c) 80 (d) 100 (b) 120 (b) 140 (c) 160 (c) 180 (b) 200 (a)
y
o
u
rs
2015 - 12 Combined Graduate Level Exam

m
a
h
Hints & Explanations

b
o
o
b
.w
8. (d) MERCY cannot be formed from the given word
PART-A : GENERAL INTELLIGENCE & REASONING

o
CUMBERSOME, as there is no ‘Y’ in the word.

rd
1. (a) c c b a/b b c a/a a b c/c c b a/b 9. (a) Only ‘S’ can be prefixed to the given words.

p
re
2. (a) a b c d/b a c d/b c a d/b c d a/a b c d. New words are:

s
STILL, STABLE, SPILE, STAB, SPRING

s
3. (c) 4 6 10 16 24 34

.c
10. (c) Meaningful word is:

o
m
U N S C R AM B L E
+2 +4 +6 +8 +10 3 9 4 2 8 10 5 1 7 6
4. (c) 3 5 9 17 33 11. (c) Coded word: Z B Y X M N Q B
¯ ¯ ¯ ¯ ¯ ¯ ¯ ¯
Key : s t r a i g h t
+2 +4 +8 +16 12. (d) M O U S E
5. (b) Let ‘D’ be the distance between A and B and T be the ¯ +3 ¯ +3 ¯ +0 ¯ –2 ¯ –2
time taken by them Coded as: P R U Q C
Then, Similarly,
Distance = Speed × Time S H I F T
¯ +3 ¯ +3 ¯ +0 ¯ –2 ¯ –2
æ 15 ö
D = 40 × ç T + ÷ ...(1) Coded as: V K I D R
è 60 ø

æ 24 ö 13. (a) 2 5 3 Þ are old


D = 30 × ç T + ÷ ...(2) books
è 16 ø
Equating (1) and (2) 5 4 6 Þ man is old

æ 1ö æ 2ö
40 ç T + ÷ = 30 ç T + ÷
è 4ø è 5ø 3 7 8 Þ buy good
books
4 6 Codes are :
= 4T = 1 = 3T +
3 5 5 Þ old4 Þ man or is 8 Þ buy or good
6 3 Þ books 6 Þ man or is
T = -1
5 2 Þ are7 Þ buy or good
1 2 stands for “are” in that code.
T = hour 14. (b) 46 × 6 ¸ 4 + 5 – 3 = 46 × 1.5 + 5 – 3 = 69 + 5 – 3 = 71
5
15. (b) 18 × 6 ¸ 4 + 2 – 3 = 18 × 1.5 + 2 – 3 = 27 + 2 – 3 = 26
Putting ‘T’ value in equation (1), we get
16. (c) 8 × 2 + 1 = 17
æ 1 1 ö 40 ´ 9 8 × 17 + 1 = 137
D = 40 × ç ´ ÷ = = 18 km.
è5 4ø 20 8 × 137 + 1 = 1097
Hence, the distance between the two stations is 18 km. 17. (c) 36 – 14 + 12 + 7 = 3
6. (c) Original Position: 54 – 9 + 11 + 16 = 18
Kiran Manoj 18. (c) + 4 -3 + 4 -3
9 ¾¾¾ ® 13 ¾¾¾ ® 10 ¾¾¾ ® 14 ¾¾¾ ® 11
+4 -3 +4 -3
20th from the left 30th from the right 11 ¾¾¾ ®15 ¾¾¾ ®12 ¾¾¾ ®16 ¾¾¾®13
Position after Interchange: +4 -3 +4 -3
13 ¾¾¾ ®17 ¾¾¾®14 ¾¾¾ ®18 ¾¾¾® 15
Manoj
Hence, 15 is the correct answer.
19. (c) 3 × 4 × 2 = 24
35th from the right
2 × –2 × –1 = 4
Total Number of Students = 35 + 20 – 1 6× 0×5=0
= 55 – 1 = 54 20. (b) 7 × 8 × 2 = 112
7. (d) Standing Arrangement: 5 × 4 × 8 = 160
3 × 9 × ? = 162
27 × ? = 162
G F E D B C A 162
?= =6
G is standing on the extreme left. 27
y
o
u
rs
CGL (Tier-I) 2015 Solved Paper 2015 - 13

m
a
h
21. (c)

b
D 600 mtr. C Option (c): –3

o
o
b
H S K P

500 mtr.

.w
500 mtr.

o
rd
+3

p
re
Pinky’s
starting point A 600 mtr. B Option (d): –3

s
s
.c
Hence, Pinky is 0 metres from the starting point.

o
K Q N N

m
22. (b) C B
+3

Option (a): –2
4 km 4 km
D W F U

+2
D A Sunita’s
2 km starting point Hence, option (a) is the group of odd letters.
-1
(d) Option (a): C ¾¾¾ +3 +1
Hence, Sunita rode 4 km northwards initially. 34. ® B ¾¾¾® E ¾¾¾®F
23. (a) Parental involvement and participation is necessary -1 +3 +1
Option (b): E ¾¾¾® D ¾¾¾® G ¾¾¾®H
for promoting the growth of children.
24. (c) Conclusion I is not based upon the given statement. -1 +3 +1
Option (c): I ¾¾¾® H ¾¾¾® K ¾¾¾®L
Conclusion II does not prove the given statement so,
neither I nor II follows. Option (d): G ¾¾¾ -1 +1 +1
® F ¾¾¾ ® H ¾¾¾ ®J
25. (c) Hematology is the branch of medicine concerned with Hence, option (d) is the odd group of letters.
the study and prevention of diseases related to the 35. (d) 4 + 0 + 2 + 5 = 11
blood. 6 + 0 + 2 + 3 = 11
Similarly, phycology is the scientific study of algae. 7 + 2 + 0 + 2 = 11
26. (c) A group of Lions is called a pride.
5 + 0 + 6 + 1 = 12
A group of Cats is called a clowder.
Hence, (d) is the odd one out.
27. (a) M A N W A N
¯ +3 ¯ +3 ¯ +3 Similarly, ¯ +3 ¯ +3 ¯ +3 36. (a) 64 : 48
P D Q Z D Q ¯ ¯
28. (b) A E F J (8 ´ 8) (8 ´ 6)
¯ +10 ¯ +10 ¯ +10 ¯ 10 80 : 60
K O P T ¯ ¯
Similarly, (10 ´ 8) (10 ´ 6)
G K L P 104 : 78
­ –10 ­ –10 ­ –10 ­ –10 ¯ ¯
Q U V Z (13 ´ 8) (13 ´ 6)
29. (a) 25 = 32 Hence, option (a) is odd one out
35 = 243 37. (d) Lily, Daisy, Datura all have outer part white and inner
30. (c) D × H M × Q part yellow.
¯ ¯ Similarly, ¯ ¯ Jasmine also has outer part white and inner part
4 × 8 13 × 17 yellow.
Respective place value of letters in English alphabet. 38. (c) Meaningful Order:
Hence, option (c) in the correct answer. Mosquito ® Cat ® Tiger ® Elephant ® Whale
31. (d) Except (d), all others belong to the period of before (3) (2) (4) (1) (5)
sunset.
32. (b) Except (b), all others are principles of society. B F I
–3 G
33. (a) Option (b): D E L
39. (c) A J
E V H S

+3 C H K
y
o
u
rs
2015 - 14 Combined Graduate Level Exam

m
a
DABD, DADC, DBDE, DDEC,

h
PART-B : GENERAL AWARENESS

b
DABC, DEBC, DACE, DABE,

o
o
DBEF, DFIG, DCEH, DHGK, 51. (a) Endymion is a poem,written by John Keats. It begins

b
with the line "A thing of beauty is a joy forever". The

.w
DFGE, DEGH, DGIL, DILJ,
DGLK, DLJK, DGIJ, DGKJ, poem tells about how nature and its wonder mesmerize

o
rd
DGIK, DIJK, DBCH, DIHK, us and take away all the sorrow that surrounds us

p
DBFC, DFIK, DFCK, DHBI from time to time.

re
Total Triangles = 28 52. (b) The birthday of Sarvepalli Radhakrishnan is celebrated

s
s
40. (b) The given figure can be numbered like this: as Teacher's Day. The day commemorates the birthday

.c
o
1 of Dr Sarvepalli Radhakhrishnan, a philosopher and a

m
teacher par excellence, and his contribution towards
2
Indian education system. As a tribute to this great
3 4 teacher, his birthday is observed as Teacher's Day
5 across India on 5th September.
6 53. (c) The Arjuna Awards are given by the Ministry of Youth
In this figure: Affairs and Sports, government of India to recognize
1 lies opposite 3; outstanding achievement in National sports.
2 lies opposite 5; 54. (c) "Place of the Thunderbolt" is associated with
4 lies opposite 6. Darjeeling(west Bengal). The word Darjeeling is a
When the sheet of question figure is folded to form a combination of two words 'dorje,' which means
cube, then the face bearing a dot lies opposite to one 'thunderbolt' and 'ling', which means 'place'. Hence
of the shaded faces. the word Darjeeling means 'the Land of Thunderbolt'.
Therefore, figure (2) which has both the shaded faces 55. (c) The Ship building yard Mazagon Dock Limited (MDL)
adjacent to the face bearing the dot, cannot be formed. is located in Mumbai(Maharashtra).It is India's prime
Hence, the cubes shown in figures (1), (3) and (4) can shipyard. It manufactures warships and submarines
be formed. for the Indian Navy and offshore platforms and
41. (c) There are 17 number of circles in the given figure. associated support vessels for offshore oil drilling.
42. (d) Animals 56. (d) Electric current is measured using a device called an
ammeter.
57. (d) Aga Khan Cup is related to Football. When Prince
Sea Animals Aga Khan IV of Iran visit in 1958, expressed his interest
Land
to start a major international football club tournament
animals
in the region. The football authorities of East Pakistan
43. (d) Persons Numbers in collaboration with Asian Football Confederation
1 2 3 4 5 6
(AFC) decided to start this event.
Doctors 3 3 5 5 3 3 58. (a) The first summit was held in Dhaka, Bangladesh on 7-
5 5 5 3 3 3 8 December 1985 and was attended by president of
Married 5 3 3 5 5 3 Bangladesh, Maldives, Pakistan and Sri Lanka, the
kings of Bhutan and Nepal, and the prime minister of
Number 1 indicates doctors who are not married India.
44. (a) P and K is nothing but R. 59. (c) The Bretton Woods Institutions are the World Bank,
47. (b) and theInternational Monetary Fund (IMF). They were
set up at a meeting of 43 countries in Bretton Woods,
New Hampshire, USA in July 1944.
60. (c) The price and output under monopoly are determined
by equality between marginal cost and marginal
revenue and not by the intersection of demand and
M supply curves.
49. (a)
61. (d) Macroeconomics is a branch of economics dealing with
the performance, structure, behavior, and decision-
making of an economy as a whole, rather than
individual markets. Macroeconomists develop models
that explain the relationship between such factors as
N Mirror image national income, output,consumption, unemployment,
inflation etc.
50. (a) P E A R L
62. (d) Internal economies arise within the firm because of the
¯ ¯ ¯ ¯ ¯
expansion of the size of a particular firm. They are called
00 55 22 11 96
the economies of scale.
y
o
u
rs
CGL (Tier-I) 2015 Solved Paper 2015 - 15

m
a
h
63. (c) In a free market economy there is a freedom of choice 76. (d) A river profile is a curve which shows the slop of a

b
for the consumers to buy goods and services which river from source to mouth.A stream flowing over

o
o
suit their tastes and preferences. This is generally called irregular terrain may have waterfalls, rapids and lakes

b
.w
the principle of consumer sovereignty. This means in along its course.Though the stream will wear away the
a market economy the consumers are just like a king or irregularities to leave a smoothly curving profile called

o
rd
sovereign who dictate what goods and services and a graded profile.

p
what quantities of them are produced. 77. (c) Karst is a landscape formed from the dissolution of

re
64. (c) Net national product (NNP) refers to gross national soluble rocks including limestone, dolomite and

s
s
product (GNP), i.e. the total market value of all final gypsum. It is characterized by sinkholes, caves, and

.c
o
goods an d servicesproduced by th e factors of underground drainage systems.

m
production of a country or other polity during a given 78. (d) Kerala is famous for the cultivation of coconut, tea,
time period, minus depreciation. coffee, cashew and spices.
NNP = GNP – Depreciation 79. (a) The Trans-Siberian Railway is the longest railway line
65. (b) Zilla Parishad is the apex body of the PR(Panchayati in the world. It has a length of 9,289 km which connects
Raj) system located at the district level. Chairpersons/ Moscow to Vladivostok.
Presidents of Panchayat Samitis come within its 80. (d) Photoperiodism is the physiological reaction of
jurisdiction. organisms to the length of day or night. It occurs in
66. (c) A parliamentary system is a system of democratic plants and animals. It affects Flowering,Vegetative
governance of a state in which the executive branch growth and fruiting in plants.
derives its democratic legitimacy from, and is held 81. (d) The correct match is as follows:
accountable to the legislature(parliament). Ascorbic acid- Vitamin C
67. (d) Public opinion is important in a democracy because Chlorophyll- Photosynthetic pigment
the people are the ultimate source of political power.
Carotenoid- Quencher
The legislature in a democratic country can influence
Superoxide dismutase- Enzyme
public opinion byFocusing attention on Public issues.
82. (b) Allantois is a part of a developingamniote's conceptus
68. (a) Article 56 of the Indian Constitution says that the
which primarily involved in nutrition and excretion,
President shall hold office for a term of five years from
and is webbed withblood vessels.
the date he takes up his post. He may resign from his
office by writing his resignation to the Vice-President 83. (d) Haliotis belongs to Mollusca.
of India. 84. (a) Although most DNA is packaged in chromosomes
69. (c) Nagaland is a state in Northeast India. The state capital within the nucleus, mitochondria also have a small
is Kohima. Nagaland became the 16th state of India amount of their own DNA. This genetic material is
from 1 December 1963. known as mitochondrial DNA or mtDNA
70. (a) Mihira Bhoja was a ruler of the Gurjara Pratihara 85. (c) First class proteins contain all the essential amino acids
dynasty of India. Bhoja's empire extended to Narmada in sufficient amounts.Animal proteins are obtained from
River in the South, Sutlej River in the northwest, and milk,egg,fish,meat etc.are first classproteins. These are
up to Bengal in the east. also called adequate proteins.
71. (d) Bose appeared at the 1939 Congress meeting and was 86. (b) Needle exerts more pressure than nail on the balloon.
elected president over Gandhi's preferred candidate 87. (c) The speed of sound is greater in moist air than in dry air.
Pattabhi Sitaramayya. 88. (b) X rays are frequently used to check the defects in
72. (c) The Jallianwala Bagh Massacre happened in Amritsar, Diamonds and other precious stones.
in 1919. It is named after the Jallianwala Bagh (Garden) 89. (a) When ice is kept on saw dust then it does not melt
atAmritsar. On April 13, 1919, British, Indian Army quickly as it's an insulator of heat and air does not
soldiers started shooting an unarmed gathering of circulate in good insulators . So, it prevents ice from
men, women and children. melting quickly .
73. (b) Bahlol Lodi was the founder of the Lodi dynasty.He 90. (c) A parity bit, or check bit is a bit added to the end of a
ruled for long thirty-nine years (1451-89). He was the string of binary code that indicates whether the number
governor of Lahore and Sirhind during the rule of of bits in the string with the value one is even or odd.
Muhammad Shah of Sayyid dynasty. Parity bits are used as the simplest form of error
detecting code.
74. (c) Sri Chaitanya Mahaprabhu arrived in the empire at the
91. (c) Memory management is the process of controlling and
time of Emperor Prataparudra (Gajapatis)and stayed
coordinating computer memory, assigning portions
for 18 long years at Puri.
called blocks to various running programs to optimize
75. (c) The instance heat that prevails in the Indian Sub
overall system performance.It involves components
continent causes a low pressure region over the that physically store data, such as RAM (random
northern plains. It is intense enough to attract the access memory) chips, memory caches, and flash-
moisture bearing winds from the Indian Ocean .Thus based SSDs (solid-state drives).
the south east trade winds from the southern 92. (a) When a drop of Glycerol is added to crushed KMnO4
hemisphere are attracted towards India. spread on a paper there is a violent explosion.
y
o
u
rs
2015 - 16 Combined Graduate Level Exam

m
a
h
93. (c) A bleaching agent is a material that lightens or whitens
A

b
a substrate through chemical reaction. The most

o
102. (c)

o
common bleaching agents generally fall into two

b
6 cm
categories: chlorine and its related compounds (such

.w
as sodium hypochlorite) and the peroxygen bleaching B

o
O 10 cm

rd
agents, such as hydrogen peroxide and sodium

p
perborate.

re
94. (a) Alpha particles are the least penetrating as they are

s
AB2 + OA2 = OB2

s
the most densely ionizing. The penetrating power of

.c
nuclear radiation depends upon the ionizing power of AB2 = (10)2 – (6)2

o
m
the radiation. The more localised the ionization the = 100 – 36 = 64
less penetrating power it will possess. AB = 8cm
95. (a) Hydrogen- peroxide is an effective sterilizing agent. 103. (c) Given, AB = 12 km, BC = 5 km
Water results when it readily loses active oxygen. A
96. (c) The maximum fixation of solar energy is done by green
plants.The energy is stored in the plants as
carbohydrates for their metabolic activities as
growth,respiration etc. 12 km
97. (a) The term 'brown air' is used for photochemical smog.
The brown color is cause by the presence of a various
Nitrous Oxides.
98. (c) One of the Major objectives and advantages of
rainwater harvesting is to reduce run off loss. B 5 km C
99. (c) Peroxyacetyl nitrate is a secondary pollutant present Straight way distance of ship
in photochemical smog. It is thermally unstable and
AC = AB2 + BC2 = 122 + 52 = 169 = 13 km
decomposes into peroxyethanoyl radicals and nitrogen
dioxide gas. 104. (a) E
100. (b) Brahmaputra River is one of the largest rivers in the
world. The origin of Brahmaputra River is in
southwestern Tibet as the Yarlung River. 5m
PART-C : QUANTITATIVE APTITUDE
D C
101. (c)
A
12 m
7m 12 m

A B
B 4x D 5x C 12 m
1 Let DE be the distance between the poles
Area of DABD = ´ BD ´ AD Distance between poles top
2
1 DE = DC2 + CE 2 = 122 + 52 = 144 + 25 = 13 m
60 = ´ 4x ´ AD ... (1) 105. (a) The maximum value of sin 4q + cos4q is 1.
2
106. (a) tan 4° tan 43° tan47° tan 86°
1 tan (90° – 86°) × tan (90° – 47°) × tan 47° × tan 86°
Area of DADC = ´ DC ´ AD
2 Þ cot 86° × cot 47° × tan 47° × tan 86°
Þ1
1
Area of DADC = ´ 5x ´ AD ... (2) 107. (c) Let four numbers are a, b, c, d, then
2 a + b + c + d = 48 ...(i)
Dividing eqn. (1) and (2) and a + 5 = b + 1 ...(ii)
1 or, a = b – 4 ...(iii)
60 ´ 4x ´ AD and c – 3 = d – 7 ...(iv)
= 2 c=d–4 ...(v)
Area of DADC 1
´ 5x ´ AD Substituting equation (iii) and (v) in equation (i) we
2
get
5x ´ 60 b – 4 + b + d – 4 + d = 48
Þ Area of D ADC = = 75 cm
4x b + d = 28 ...(vi)
y
o
u
rs
CGL (Tier-I) 2015 Solved Paper 2015 - 17

m
a
h
But we know, OA = AD = x

b
b+ 1= d–7

o
\ OD = x 2 + x 2 = 2x

o
\ b=d–8

b
Substituting in equation (vi) we get

.w
So (d) option is correct.
d – 8 ¹ d= 28

o
xy

rd
d = 18 115. (d) Successive discount can be given by = x + y +

p
Solving this way we get a = 6, b = 10, c = 14 and d = 18 100

re
108. (a) Number has to be less than 27. Let the number be x. On

s
( -10 ´ -20)

s
Dividing 2055 by 27, we get remainder as 3 = -10 - 20 +

.c
Now, 3 + x = 27 100

o
m
\ x = 24 = –30 + 2 = 28%
109. (b) A and B can complete work in 6 days Hence, the successive dicount in equal to 28%
A can complete in 18 days 116. (c) Let the second discount be x%. Then
Let B can complete in x days (100 – x)% of 90% of 720 = 550.80
\ One day work be equal to
100 - x 90 55080
1 1 1 Þ ´ ´ 720 =
= + 100 100 100
6 18 x
1 x + 18 55080 ´ 100
= Þ (100 - x) = = 85
6 18x 90 ´ 720
3x = x + 18 Þ x = 100 - 85 = 15%
x = 9 days 117. (c) S.P. of an article = 20% and 15% successive discount
1 × marked price of an article
110. (d) Work done by A in one hour =
x 80 85
3060 = ´ ´ marked price of an article
1 100 100
Work done by B in one hour =
y 3060 ´ 100 ´ 100
Both A & B together in work one hour \ Marked of an article = = 4500
80 ´ 85
1 1 y-x 118. (b) Let the A’s age and B’s age was 8x and 13x
= - =
x y xy According to question
xy 8x + 18 5
Both A & B fill tank in hours. =
y-x 13x + 18 7
Þ 56x + 18 ´ 7 = 65x + 18 ´ 5
1
111. (a) Work done by 1st tap in one minute = Þ 65x – 56x = 18 ´ 7 – 18 ´ 5
30 Þ 9x = 18 ´ 2
1 Þ x=4
Work done by 2nd tap in one minute =
45 Hence, the present age of A
1 1 45 + 30 = 8 ´ 4 + 18 = 50 yrs.
Both tap one minute work = + =
30 45 1350 7
75 1 119. (d) Quantity of milk = ´ 729 = 567 ml
= = 9
1350 18
Both tap will empty the tank in 18 minutes. 2
Quantity of water = ´ 729 = 162 ml
112. (b) Perimeter of one face, 4a = 20 cm 9
Therefore, side of cube, a = 5 cm Let ‘x’ be the quantity that should be added to make
Volume of cube= a3 = 53 = 125 cm3 the ratio 7 : 3
113. (a) Area of circle, A = pr2 ...(i) According the question
Circumference of circle, C = 2pr ...(ii)
Multiplying eq. (i) by 2, we get, 2A = 2pr2 567 7
=
Multiplying eq. (ii), by ‘r’, we get rC = 2pr2 162 + x 3
\ rC = 2A Þ 1701 = 1134 + 7x
114. (d) D C Þ 7x = 1701 – 1134
Þ x = 81 ml
120. (b) Let the average weight of 15 Oarsmen at the start = x kg
2x 2x Let the new man’s weight = y kg
According to question
x x 15x – 42 = 15 (x + 1.6) – y
15x – 42 = 15x + 24 – y
y = 24 + 42 = 66 kg
A x O x B
y
o
u
rs
2015 - 18 Combined Graduate Level Exam

m
a
h
121. (a) Arithmetic mean of first ‘n’ natural number
1 1 1 1 1

b
- + - +

o
Sum of 'n' natural number 128. (c)
7- 6 6- 5 5 -2 8- 7 3- 8

o
=

b
Number of observations

.w
1 7+ 6 1 6+ 5 1
(n)(n + 1) n + 1 Þ ´ - ´ +

o
=

rd
= 7- 6 7+ 6 6- 5 6+ 5 5-2
2´n 2

p
re
122. (a) Solving by alligation

s
5+2 1 8+ 7 1 3+ 8

s
70 70.75 ´ - ´ + ´

.c
x 5+2 8- 7 8+ 7 3- 8 3+ 8

o
m
30 20
Þ
7+ 6
-
( 6+ 5 )+ 5+2
70 ´ 30 + 70.75 ´ 20 2100 + 1415 7 -6 6-5 5-4
x= = = 70.3
50 50
Hence, cost price = 70.3 × 50 = ` 3515
-
( 8+ 7 ) + 3+ 8
Selling price = 80.5 × 50 = ` 4025
8-7 9-8
Required gain = 4025 – 3515 = 510
Þ 7 + 6 - 6 - 5 + 5 + 2- 8 - 7 +3+ 8
9261
123. (d) Population 3 yrs. ago = 3 Þ5
æ 5 ö 129. (d) m3 – 3m2 + 3m + 3n + 3n2 + n3
çè 1 + ÷
100 ø Þ (– 4)3 – 3 (– 4)2 + 3 (– 4) + 3 (– 2) + 3 (– 2)2 + (– 2)3
Þ – 64 – 48 – 12 – 6 + 12 – 8
9261´ 20 ´ 20 ´ 20 Þ – 126
= = 8000
21´ 21´ 21
124. (b) Let the distance travelled as foot be x km. 130. (b) For no solution, a = b
Then, distance travelled by bicycle = (61 –x) km
2 -k
x 61 - x =
So, + =9 6 -12
4 9
9x + 4 (61 –x) = 9 × 36 -12 ´ 2
k= =4
9x – 4x = 324 – 244 -6
5x = 80 131. (c) By squaring the given relations, we get (i) and (iii) are
x = 16 km incorrect relations from the given statement.
125. (b) When distance is contant, then speed is inversely 132. (a) Using the formula,
proportional
S1 : S2 = T2 : T1 1
x 3 + y 3 + z 3 - 3xyz = ´ (x + y + z)
4 : 16.5 = T2 : 165 2
4 T é( x - y )2 + ( y - z )2 + ( z - x )2 ù
or = 2 ë û
16.5 165
165 ´ 4 1
T2 = = 40 min Þ ´ (332 + 333 + 335 ) ´
16.5 2
126. (d) Let x be the principal amount é(332 - 333)2 + (333 - 335)2 + (335 - 332 )2 ù
‘y’ be the time to double the money. ë û
Then interest will also be ‘x’.
1
´1000 ´ é( -1) + ( -2 ) + ( -3) ù
2 2 2
x ´ 25 ´ y Þ ë û
\ x= 2
4 ´100
1
400 = 25y Þ ´1000 ´ [1 + 4 + 9]
y = 16 years 2
x y 1
127. (d) Intercept can represent in the form of + =1 Þ ´ 1000 ´14 = 7000
a b 2
To get x and y intercept, we have
3x + 4y = 12 1
133. (d) 2+ x 3 =
2+ 3
x y
+ =1
4 3 ( 2 + x 3 )(2 + 3 ) = 1
So, triplets of 3, 4 and 5.
Hence, 5 is the length of portion of straight line. 4 + 2 x 3 + 2 3 + 3x = 1
y
o
u
rs
CGL (Tier-I) 2015 Solved Paper 2015 - 19

m
a
h
2 x 3 + 3x = 1 - 4 - 2 3 = -3 - 2 3 C

b
143. (c)

o
( ) ( )

o
x 2 3 + 3 = - 3+ 2 3

b
.w
m
x=–1

10

o
135. (b) Let ‘x’ be the measure of an angle.

rd
According to question

p
re
3x + x = 180° 30°
B

s
A

s
4x = 180°

.c
AB

o
180 cos 30° =
AC

m
x= = 45°
4 AB = AC × cos 30°
136. (d) Let sides of D be 3x, 4x, 5x
3
a+b+c = 10×
s= = 6x 2
2 = 8.66 m
Area of D = s( s - a)( s - b)( s - c) 144. (c) Production of electronic items is highest in 2011
i.e. 13,000 + 9,000 = 22,000
7776 = 6 x.3 x.2 x.x 145. (a) Production of LCD in 2011 = 9,000
7776 = 6x2 Production of LCD in 2013 = 12,000
\ x = 36 9,000 3
Sides of D will be 108, 144 and 180 Ratio = =
12,000 4
Perimeter of D is 108 +144 + 180 = 432 cm
138. (d) P = 4Q 146. (d) Total production of TV from 2009 to 2012 = 39,000
P + Q = 180° Average of TV production = 9,750
4Q + Q = 180° Total production of LCD = 35,400
Average of TV production = 8,850
180 Their difference = 9,750 – 8,850 = 900
Q= = 36°
5
6,000
So, R = 180° – 36° = 144° 147. (c) Ratio of production of TV = 9, 000 = 2 : 3
139. (a) Given sinq + sin2q = 1
1 – sin2q = sinq 148. (c) Amount of product = 4910 crores
Then, cos2q + cos4q Þ cos2q + (cos2q)2 Amount to be collectd
Þ (1– sin2q) + (1– sin 2q)2 = 4910 crores + 10% of 4910 crores
Þ sinq + sin2q = 1 = 4910 + 491 = 5401 crores
140. (c) Clock started at 12 pm 149. (c) External assistance received = 9695 crores
Total required = 11486 crores
360
Angle turned by hour hand in one hour = = 30° Shortage of external assistance = 11486 – 9695
12 = 1791 crores
30 1º Previous Market borrwings = 29952 crores
Angle turned by hour hand in one minute = =
60 2 1791
Angle turned by hour hand in 3 hour 45 minutes % increase in market borrowing = ´ 100 = 6%
29952
1 1º 150. (c) Total of sources of funds = 11486 + 5252 + 4910
= 3 × 30° + 45 × = 112 + 6000 + 29952
2 2
= 57600 crores
cos 2 45° cos 2 60° tan 2 30º sin 2 30º Central angle corresponding to market borrowing
141. (a) + - -
sin 2 60° sin 2 45º cot 2 45º cot 2 30º 29952
= ´ 360 = 187.2
2 2 2 2 57600
æ 1 ö æ1ö æ 1 ö æ1ö
ç ÷ ç ÷ ç ÷ ç ÷
Þ è

+ è ø 2 - è 2ø -
2 3 è 2ø PART-D : ENGLISH COMPREHENSION
(1) ( )
2 2
æ 3ö æ 1 ö 3
çç ÷ ç ÷ 151. (c) At one's wit's end means at the limits of one's mental
è 2 ø÷ è 2ø resources. Example: I'm at my wit's end with choosing
the subject in my graduation. I cannot figure it out.
1 4 1 2 1 1 1 Anju could do no more. She was at her wit's end.
= ´ + ´ - - ´
2 3 4 1 3 4 3 152. (c) All thumbs means very awkward and clumsy,
especially with one's hands. Poor Sam can't play the
2 1 1 1 8 + 6 - 4 -1 9 3
Þ + - - Þ = = piano at all. He's all thumbs. Pamela is all thumbs when
3 2 3 12 12 12 4 it comes to gardening.
y
o
u
rs
2015 - 20 Combined Graduate Level Exam

m
a
h
153. (b) To rise with the lark entails to rise very early in the 172. (c) Inedible is something that is not suitable for food or

b
morning. eating.

o
o
154. (b) Wild-goose chase means a worthless hunt or chase; a 173. (b) Aquarium

b
.w
futile pursuit. I wasted all evening on a wild-goose 174. (a) Facsimile
chase. Raj was angry because he was sent out on a 175. (b) A person naturally expresses his anger by responding

o
rd
wild-goose chase. aggressively.

p
155. (a) Make a beeline for someone or something means 176. (b) Our feelings place limits on how far we can take our

re
heading straight toward someone or something. John anger.

s
s
came into the kitchen and made a beeline for the cookies. 177. (a) People should deal with their anger by expressing it

.c
After the game, we all made a beeline for Dhoni, who

o
assertively.

m
just played the winning run. 178. (c) According to author, being assertive means to be
156. (b) If you say it's high time that something happened, you respectful of yourself and others.
mean that it should already have been done. His 179. (a) One, according to author, can suppress his anger by
parents decided it was high time he started behaving holding his anger.
himself. (often + that) It's high time that workers were 180. (b) The passage throws light on how a well-means
given better pay and conditions. invention can be misunderstood.
157. (d) Habit is a particular practice, custom, or usage: the 181. (c) The crowd was protesting against the newly invented
habit of shaking hands. sewing machine.
158. (a) 'He lacks common sense' is a meaningful sentence. 182. (d) The aim of the crowd was to destroy the sewing
Others are not. machines.
159. (a) A single person or thing; a unit: This is the one I like 183. (d) People thought they would be deprived of their
best. Of his many movies, the best ones are the last livelihood.
two. 184. (c) Shutters were being closed because the shopkeepers
160. (c) Experience may be gotten, acquired or gathered; hence, feared their shops would be destroyed.
option c is correct. 185. (b) This part of the sentence should be 'to be with you.'
161. (d) Ingestion is the process of taking food into the body 186. (a) I really do regret not having learnt to play the violin…..
through the mouth (as by eating); hence, option d. should be the correct structure.
162. (d) The adverb far showing distance indicates at, to, or 187. (c) Injure cannot be an intransitive verb. You do not say,
from a great distance in space or time: for example, 'He injured in a car accident'. You say 'He
Ex: How far is it from Australia to New Zealand? He was injured in a car accident'. Fifty workers were injured
doesn't live far from here. in the collapse of the building.'
163. (c) If someone or something is famous, a lot of people 188. (b) 'I shall have been working in this college' is the right
know their name or have heard about them. Ex: The construction.
town of Moradabad is famous for brassware. Alexander 189. (d) No error
Fleming, the Scot famous for discovering penicillin. 190. (d) Pursue means carry out or participate in an activity; be
164. (b) If you talk, you are having a conversation with other involved in or go in search of or hunt for something.
people. Ex: They were all talking and laughing together. Ex: She pursued many activities.
We were talking about you just last night. 191. (c) Making is in singular; hence, 'is' the correct helping
165. (c) A tag question is one where a statement is made, but verb.
the speaker wants a response from the listener. The 192. (d) If something is infected, it means the insects or animals
given sentence is in simple present with third person, are present (in a place or site) in large numbers, typically
so response will be 'doesn't he?' so as to cause damage or disease. Ex: The house is
166. (d) A compulsion to steal having no relation to need or infested with cockroaches.
the monetary value of the object. Pneumonia is a 193. (d) On foot means running or walking using the feet. My
disease; insomnia is an inability to sleep; chronic car won't work so I have to travel on foot. We go
sleeplessness while nymphomania is an abnormally everywhere around the campus on foot. He went there
intense sexual desire in women. on foot.
167. (d) A false accusation of an offence or a malicious 194. (a) Up and about means no longer in bed (after sleep or an
misrepresentation of someone's words or actions. illness).
168. (a) A usually short narrative making an edifying or 195. (d) 'Devout' means deeply religious or pious.
cautionary point and often employing as characters 196. (d) 'Predilection' means a predisposition in favour of
animals that speak and act like humans.
something; A strong liking; preference.
169. (a) The branch of medicine that deals with the diagnosis 197. (a) Effigy means a representation of a person (especially
and treatment of diseases and disorders of the skin.
in the form of sculpture); hence, dummy.
170. (c) The practice of requiring excessive paperwork and
198. (a) Tremulous means unsteady; hence, steady is the
tedious procedures before official action can be
opposite.
considered or completed; also called red-tapery or red-
199. (b) The opposite of fake is real.
tapist.
200. (a) Disconsolate means sad beyond comforting; incapable
171. (b) Intruder is someone who intrudes, especially into a
of being consoled; hence, Joyous is the opposite.
building with criminal intent.
y
o
u
rs
SECTION - A : GENERAL INTELLIGENCE & REASONING

m
a
h
b
o
o
b
CHAPTER

.w
1

o
ANALOGY

rd
p
re
s
s
.c
o
m
The meaning of analogy is ‘similar properties’ or similarity. If an 4. Worker & product based analogy: This type of analogy
object or word or digit or activity shows any similarity with gives a relationship between a person of particular
another object or word or digit or activity in terms of properties, profession and his/her creations.
type, shape, size, trait etc., then the particular similarity will be Examples:
called analogy. For example, cricket : ground and chess: table
Batsman : Run
are the analogous pairs (why?). In fact, both pairs of words have
similar relationship in terms of place of playing as cricket is played Writer : Book
in the ground and similarly chess is played on the table. Author : Novel
TYPES OF ANALOGY. Singer : Song
1. Tool & object based analogy: This establishes a Poet : Poem
relationship between a tool and the object in which it works. Journalist : News
Similar relations has to be discovered from answer choices. 5. Cause & effect based analogy: In such type of analogy
Examples: 1st word acts and the 2nd word is the effect of that action.
Pencil : Paper Examples:
Pen : Paper Work : Tiredness
Scissors : Cloth Bath : Freshness
Saw : Wood Race : Fatigue
Eraser : Paper Shoot : Kill
2. Synonym based analogy : In such type of analogy two 6. Opposite relationship (Antonym) based analogy : In
words have similar meaning. such type of analogy the two words of the question pair are
Examples: opposite in meaning. Similar relations has to be discovered
Big : Large from the answer choice word pairs.
Huge : Gigantic Examples:
Endless : Eternal Poor : Rich
Thin : Slim Fat : Slim
Benevolent : Kind Tall : Short
Notion : Idea Big : Small
Huge : Big Light : Dark
3. Worker & tool based analogy: This establishes a Avoid : Meet
relationship between a particular tool and the person of that 7. Gender based analogy: In such type of analogy, one word
particular profession who uses that tool. is masculine and another word is feminine of it. In fact, it is
Examples: a ‘male and female’ or ‘gender’ relationship.
Writer : Pen Examples:
Painter : Brush Man : Woman
Cricketer : Bat Boy : Girl
Blacksmith : Hammer Nephew : Niece
Barber : Scissors Bull : Cow
Hunter : Gun Duck : Drake
y
o
u
A-2 Analogy

rs
m
8. Classification based analogy: This type of analogy is based 14. Adult & young one based analogy : In such type of

a
on biological, physical, chemical or any other classification. analogy, the 1st word is the adult one and 2nd word is the

h
In such problems the 1st word may be classified by the 2nd

b
young one of the 1st word or vice-versa.

o
word and vice-versa.

o
Examples:

b
Examples:

.w
Cow : Calf
Cow : Animal

o
rd
Girl : Human Human : Child

p
Oxygen : Gas Dog : Puppy

re
s
Water : Liquid Duck : Duck ling

s
.c
Snake : Reptile 15. Subject & specialist based analogy: In such type of

o
m
Parrot : Bird analogy the 2nd word is the specialist of 1st word (subject)
9. Function based analogy : In such type of analogy, 2nd or vice-versa.
word describes the function of the 1st word. Examples:
Examples:
Heart : Cardiologist
Singer : Sings
General : Commands Skin : Dermatologist
Player : Plays 16. Habit based analogy: In this type of analogy 2nd word is
Surgeon : Operates the habit of 1st and vice-versa.
10. Quantity and unit based analogy: In such type of analogy Examples:
2nd word is the unit of the first word and vice-versa. Cat : Omnivorous
Examples: Tiger : Carnivorous
Distance : Mile Cow : Herbivorous
Mass : Kilogram Goat : Herbivorous
Length : Meter 17. Instrument and measurement based analogy: We see in
11. Finished product & raw material based analogy : In such this type of analogy, the 1st word is the instrument to measure
type of analogy the 1st word is the raw material and 2nd the 2nd word and vice-versa:
word is the end product of that raw material and vice-versa. Examples:
Examples: Hygrometer : Humidity
Yarn : Fabric Barometer : Pressure
Milk : Curd Thermometer : Temperature
Flour : Bread Sphygmomanometer : Blood pressure
Latex : Rubber 18. Individual & group based analogy : Second word is the
Grape : Wine group of 1st word (or vice-versa) in such type of analogy.
Fruit : Juice Examples:
12. Utility based analogy : In such type of analogy the 2nd Cow : Herd
word shows the purpose of the 1st word or vice-versa. Sheep : Flock
Examples: Grapes : Bunch
Pen : Writing Singer : Chorus
Food : Eating 19. State & capital based analogy: 1st word is the state and
Chair : Sitting 2nd word is the capital of that state (1st word) (or vice-versa)
Bed : Sleeping in the analogy like this.
Bat : Playing Examples:
13. Symbolic relationship based analogy: In such type of Bihar : Patna
analogy, the 1st word is the symbol of the 2nd word and West Bengal : Kolkata
vice-versa. Maharashtra : Mumbai
Examples: Karnataka : Bangluru
White : Peace Note: Analogy based on country and capital is very similar
Red : Danger to this type of analogy in which we put name of the country
Black : Sorrow in place of the name of state and country capital in place
Red cross : Hospital of state capital. For example India: New Delhi and
Nepal : Kathmandu.
Swastika : Fortune
y
o
u
Analogy A-3

rs
m
20. Analogy based on individual & dwelling place : In such Case V: (Jumbled letters relation)

a
type of analogy 1st word is the individual & 2nd word is Example:

h
b
the dwelling place of that individual (1st word) and vice- (i) LAIN : NAIL : : EVOL : LOVE

o
o
versa.

b
Here the 1st term gets reveresed to produce the 2nd term

.w
Examples: and similar relation is shown in between 3rd and 4th term.

o
Horse : Stable

rd
(ii) ABCD : OPQR : : WXYZ : KLMN

p
Bee : Apiary

re
In (ii) each letter of the 1st group ‘ABCD’ is moved fourteen
Dog : Kennel

s
steps forward to obtain the corresponding letter of the 2nd

s
.c
Birds : Aviary group ‘OPQR’. A similar relation is established between

o
m
Monk : Monastery the third group ‘WXYZ’ and the fourth group ‘KLMN.’
Human : House
NOTE : Every type of analogy discussed in (23) may have
21. Analogy based on worker and working place: In this different variations of problems and you can get perfection on
type of analogy the 1st word represents a person of particular them by proper practice only.
profession and 2nd word represents the working place of
Format of the questions
that person (1st word) and vice-versa.
Examples : EXAMPLE 1. Lion is to flesh as cow is to …….
Doctor : Hospital (a) snake (b) grass
Clerk : Office (c) worm (d) animal
Cook : Kitchen Sol. Lion eats flesh, similarly, cow eats grass. Hence option (b)
Professor : College is the right answer.
Teacher : School EXAMPLE 2. Pen : Writer : : ……. : Batsman
22. Analogy based on topic study: 1st word is the study of the (a) Brush (b) Fighter
2nd word (or vice-versa) in the analogy like this.
(c) Stick (d) Bat
Examples:
Sol. Option (d) is the correct answer because a writer uses pen
Birds : Ornithology to write and similarly a batsman uses bat to play.
Earth quakes : Seismology
EXAMPLE 3. NCDP : ODEQ : : ……… : MPRO
Eggs : Zoology
23. Analogy based on letters (or meaningless words) (a) LOQN (b) NQOL
Case I : (Forward alphabetical sequence) (c) OQNL (d) QNOL
Examples: Sol. Option (a) is the correct answer as letters of 1st term go one
CD : FG : : PQ : UV step forward to be the 2nd term. Similarly, the letters of 3rd
term will go one step forward to be the 4th term (Letters of
Here CD and FG are in the natural alphabetical sequence.
step go one step backward to be the 3rd term).
Similarly, PQ & UV are in the natural alphabetical sequence.
Case II: (Backward or opposite alphabetical sequence) EXAMPLE 4. Bulky : Fat : : Happiness : ?
Example: (a) Bad (b) Ugly
DC : GF : : QP : VU (c) Joy (d) Sorrow
In fact this case is opposite of case I Sol. (c) is the correct option because ‘Bulky’ is the synonym
Case III: ( Vowel – consonant relation) of ‘Fat’ and similarly ‘Happiness’ is the synonym of
Example ‘joy’.
ATL : EVX : : IPR : ORS Now, we can say that we have discussed almost all
type of analogy to be asked frequently in the
Here, the 1st two words start with the 1st two vowels A & E
examinations. But examinees must prepare for any
and the next two words start with the next two vowels I & surprise kind of problems while solving the problems
O. Last two letter of every word are consonants. under this segment. But by practicing more & more,
Case IV: Example (Skip letter relation) you can be master in solving these problems. Only keep
ABC : FGH : : IJK : NOP in mind the following:
Here between ABC & FGH two letters skip and they are D (1) You must have strong word power.
& E. Similarly, between IJK & NOP two letters skip and (2) You must have good understanding & reasoning ability.
they are L & M. (3) You must have good general knowledge.
y
o
u
A-4 Analogy

rs
m
a
h
b
o
o
b
.w
1. Which of the following is related to ‘Melody’ in the same 13. 'Dream' is related to 'Reality' is the same way as 'Falsehood'

o
way as ‘Delicious’ is related to ‘Taste’? is related to which of the following.

rd
p
(a) Memory (b) Highness (a) Untruth (b) Truth

re
(c) Voice (d) Speak (c) Fairness (d) Correctness

s
s
2. In a certain way ‘Diploma’ is related to ‘Education’. Which 14. 'Frame work' is related to 'House' in the same way as

.c
'Skeleton' is related to which of the following?

o
of the following is related to ‘Trophy’ in a similar way?

m
(a) Sports (b) Athlete (a) Ribs (b) Skull
(c) Winning (d) Prize (c) Body (d) Grace
3. ‘Clock’ is related to ‘Time’ in the same way as ‘Vehicle’ is DIRECTIONS (Qs. 15 to 87) : In each of the following
related to which of the following? questions, there are two words / set of letters / numbers to the
(a) Driver (b) Road left of the sign :: which are connected in some way. The same
relationship obtains between the third words / set of letters /
(c) Passenger (d) Journey
numbers and one of the four alternatives under it. Find the correct
4. “Illness” is related to “Cure” in the same way as “Grief’ is alternative in each question.
related to 15. Import : Export :: Expenditure : ?
(a) Happiness (b) Ecstasy (a) Deficit (b) Income
(c) Remedy (d) Solicitude (c) Debt (d) Tax
5. ‘Bouquet’ is related to ‘Flowers’ in the same way as 16. Ocean : Water :: Glacier : ?
‘sentence’ is related to (a) Refrigerator (b) Ice
(a) Letters (b) Paragraph (c) Mountain (d) Cave
(c) Content (d) Words 17. Medicine : Sickness :: Book : ?
6. ‘Electricity’ is related to ‘Wire’ in the same way as ‘Water’ (a) Ignorance (b) Knowledge
is related to (c) Author (d) Teacher
(a) Bottle (b) Jug 18. Bank : River :: Coast : ?
(c) River (d) Pipe (a) Flood (b) Waves
7. Mathematics is related to Numbers in the same way History (c) Sea (d) Beach
is related to : 19. Thunder : Rain :: Night : ...
(a) People (b) Events (a) Day (b) Dusk
(c) Dates (d) Wars (c) Darkness (d) Evening
8. ‘Locker’ is related to ‘Jewellery’ in the same way as 20. Breeze : Cyclone : : Drizzle : ?
‘Godown’ is related to (a) Earthquake (b) Storm
(a) Storage (b) Grasim (c) Flood (d) Downpour
(c) Garments (d) Goods 21. Disease : Pathology :: Planet : ?
9. Distil is related to Whiskey in the same way as Brew is (a) Astrology (b) Geology
related to .....?..... (c) Astronomy (d) Palaeontology
(a) Ferment (b) Gin 22. Foresight : Anticipation :: Insomnia : ?
(c) Beer (d) Sugar (a) Treatment (b) Disease
10. 'Story' is related to 'Novel' in the same way as 'Sea' is related (c) Sleeplessness (d) Unrest
to which of the following? 23. Oasis : Sand :: Island : ?
(a) River (b) Sea
(a) Ocean (b) Water
(c) Water (d) Waves
(c) River (d) Pond
24. Major : Battalion :: Colonel : ?
11. ‘Hygrometer’ is related to ‘Humidity’ in the same way as (a) Company (b) Regiment
‘Sphygmomanometer’ is related to
(c) Army (d) Soldiers
(a) Pressure (b) Blood Pressure
25. Shout : Whisper :: Run : ?
(c) Precipitation (d) Heart Beat
(a) Stay (b) Stand
12. ‘Engineer’ is related to ‘Machine’ in the same way as (c) Walk (d) Hop
‘Doctor’ is related to
26. Smoke : pollution : : war : ?
(a) Hospital (b) Body
(a) victory (b) peace
(c) Disease (d) Medicine
(c) treaty (d) destruction
y
o
u
Analogy A-5

rs
m
27. Hour : second : : tertiary : ? 45. CJDL : FMGR :: IKJR: ?

a
(a) ordinary (b) secondary (a) OQPT (b) RSTU

h
b
(c) primary (d) intermediary (c) LSMT (d) KRMO

o
o
28. Safe : secure : : Protect : ? 46. BCDA : STUR :: KLMJ : ?

b
(a) guard (b) lock

.w
(a) VWXU (b) EFHG
(c) sure (d) conserve

o
(c) SRTU (d) QSRP

rd
29. Penology : Punishment : Seismology : ............ . 47. CEG : EGC : : LNP :

p
(a) Law (b) Earthquake

re
(a) LPN (b) UWY

s
(c) Liver (d) Medicine

s
(c) NPL (d) MOP

.c
30. DRIVEN : EIDRVN :: BEGUM : ?

o
48. KLM : PON : : NOP : ..........
(a) EUBGM (b) MGBEU

m
(c) BGMEU (c) UEBGM (a) LMK (b) MLK
31. NUMBER : UNBMER : : GHOST : ? (c) NML (d) KLN
(a) HOGST (b) HOGTS 49. ACE : FGH :: LNP : ?
(c) HGOST (d) HGSOT (a) QRS (b) PQR
32. MASTER : OCUVGT : : LABOUR : ? (c) QST (d) MOQ
(a) NCDQWT (b) NDERWT 50. 14 : 9 :: 26 : ?
(c) NBCRWT (d) NEDRWT (a) 12 (b) 13
33. RIDE : LNBE : : HELP : ? (c) 15 (d) 31
(a) NINP (b) BAJP 51. 11 : 17 : : 19 : ?
(c) JPCH (d) BJJP (a) 2 9 (b) 27
34. MUMBAI : LTLAZH : : DELHI : ____________ (c) 2 3 (d) 21
(a) CDKGJ (b) IHLED 52. 3 : 27 : : 4 : ?
(c) CDKGH (d) BCKGH (a) 140 (b) 75
35. RATIONAL : RATNIOLA :: _________ : TRILBA (c) 100 (d) 64
(a) TIRLAB (b) TRIBAL 53. 12 : 30 : : 20:?
(c) TRIALB (d) TIRBAL
(a) 25 (b) 32
36. HEATER : KBDQHO : : COOLER : ?
(c) 35 (d) 42
(a) ALRHV (b) FLRIHO
54. 3 : 10 :: 8, ?
(c) FLIRHO (d) FRLIHO
(a) 10 (b) 13
37. PAPER : SCTGW : : MOTHER : ?
(a) ORVLGW (b) PQVIGT (c) 14 (d) 17
(c) PQXJJT (d) PQXKJV 55. 13 : 19 :: ? : 31
38. POPULAR : QPQVMBS : : ? : GBNPVT (a) 21 (b) 23
(a) FAMOSU (b) FAMOUS (c) 25 (d) 26
(c) FASOUM (d) FOSAUM 56. 48 : 122 : : 168 : ?
39. CALCUTTA : GEPGYXXE : : ? : FSQFCE (a) 284 (b) 286
(a) BOMBYA (b) BOMBAY (c) 288 (d) 290
(c) BOMYAB (d) BOBAYM 57. 5 : 21 : : 7 : ?
40. PRLN : XZTV :: JLFH : ? (a) 25 (b) 49
(a) NPRT (b) NRPT (c) 43 (d) 30
(c) NTRP (d) RTNP 58. 182 : ? : : 210 : 380
41. ACFJ : OUZJ :: SUXB : ? (a) 342 (b) 272
(a) GNSA (b) GLQZ
(c) 240 (d) 156
(c) GKPY (d) GMRB
59. 16 : 56 : : 32 : ?
42. ACE : HIL :: MOQ : ?
(a) 96 (b) 112
(a) XVT (b) TVX
(c) 120 (d) 128
(c) VTX (d) TUX
60. 23 : 13 :: 54 : ?
43. ACBD : EFGH : : OQPR –
(a) STUV (b) RSTU (a) 45 (b) 39
(c) UVWX (d) QRST (c) 40 (d) 41
44. TSR : FED :: WVU ? 61. 49 : 21 :: 169 : ?
(a) CAB (b) MLK (a) 26 (b) 39
(c) PQS (d) GFH (c) 42 (d) 13
y
o
u
A-6 Analogy

rs
m
62. 25 : 37 :: 49 : ? 75. 20 : 50 :: 100 : ?

a
(a) 41 (b) 56 (a) 150 (b) 250

h
b
(c) 60 (d) 65 (c) 200 (d) 156

o
o
63. 2149370 : 2854610 : : 2735630 : ? 76. Voyage : Sea sickness : : Heights : ?

b
.w
(a) 7536501 (b) 832150 (a) Ship (b) Travel

o
(c) 880890 (d) 9437102 (c) Giddiness (d) Motion

rd
64. Petrology : Rocks : : Palaeontology : ? 77. Waitress : Restaurant : : ?

p
re
(a) Birds (b) Animals (a) Doctor : Nurse (b) Driver : Truck

s
(c) Fossils (d) Soil (c) Teacher : School (d) Actor : Role

s
.c
65. Melt : Liquid : : Freeze : ? 78. AROUND : RAUODN : : GROUND : ?

o
m
(a) Crystal (b) Solid (a) RGUODN (b) NDOOGR
(c) Ice (d) Evaporate (c) OUNDGR (d) DNUURG
66. 8 : 27 : : ? : 81 79. APPROACHED : ROACHEDAPP : : BARGAINED : ?
(a) 24 (b) 34 (a) AINEDBARG (b) GAINEDBAR
(c) 44 (d) 54 (c) GAINEDRAB (d) RABGAINED
67. Length : Metre :: Power : ? 80. 8 : 256 : : ?
(a) Calories (b) Degree (a) 7 : 343 (b) 9 : 243
(c) Watt (d) Kilogram (c) 10 : 500 (d) 5 : 75
68. Square : Cube :: Circle : ? 81. 21 : 3 : : 574 : ?
(a) Ellipse (b) Parabola (a) 23 (b) 82
(c) Cone (d) Sphere (c) 97 (d) 113
69. Paper : Tree :: Glass : ? 82. Saint : Meditation : : Scientist : ?
(a) Window (b) Sand (a) Research (b) Knowledge
(c) Stone (d) Mirror (c) Spiritual (d) Rational
70. ACFJ : ZXUQ :: EGIN : ? 83. King : Palace :: Eskimo : ?
(a) VUSQ (b) VRPM (a) Caravan (b) Asylum
(c) UTRP (d) VTRM (c) Monastery (d) Igloo
71. ACEG : DFHJ :: QSUW : ? 84. AFKP : DINS :: WBGL : ?
(a) TVXZ (b) TQST (a) ORUX (b) OSWA
(c) MNPR (d) EGIJ (c) OTYD (d) OQSU
72. EGIK : FILO :: FHJL : ? 85. SINGER : QGLECP :: MONSTER : ?
(a) JGMP (b) JGPM (a) KLNSCP (b) KLMSCP
(c) GJPM (d) GJMP (c) KMLQRCP (d) KLMTDO
73. 10 : 91 :: 9 : ? 86. 18 : 5 :: 12 : ?
(a) 69 (b) 72 (a) 4 (b) 10
(c) 89 (d) 97 (c) 3 (d) 6
74. 7 : 56 :: 9 : ? 87. 12 : 20 :: ?
(a) 63 (b) 81 (a) 15 : 37 (b) : 16 : 64
(c) 90 (d) 99 (c) : 27 : 48 (d) 30 : 42

ANSWER KEY
1 (c) 12 (c) 23 (c) 34 (c) 45 (c) 56 (d) 67 (c) 78 (a)
2 (a) 13 (b) 24 (b) 35 (b) 46 (a) 57 (c) 68 (d) 79 (b)
3 (d) 14 (c) 25 (c) 36 (b) 47 (c) 58 (a) 69 (b) 80 (c)
4 (c) 15 (b) 26 (d) 37 (c) 48 (b) 59 (b) 70 (d) 81 (b)
5 (d) 16 (b) 27 (c) 38 (b) 49 (a) 60 (d) 71 (a) 82 (a)
6 (d) 17 (a) 28 (a) 39 (a) 50 (c) 61 (b) 72 (d) 83 (d)
7 (b) 18 (c) 29 (b) 40 (d) 51 (a) 62 (d) 73 (b) 84 (c)
8 (d) 19 (c) 30 (b) 41 (d) 52 (d) 63 (d) 74 (c) 85 (c)
9 (c) 20 (d) 31 (d) 42 (d) 53 (d) 64 (c) 75 (b) 86 (c)
10 (a) 21 (c) 32 (a) 43 (a) 54 (d) 65 (c) 76 (c) 87 (d)
11 (b) 22 (c) 33 (d) 44 (b) 55 (b) 66 (a) 77 (c)
y
o
u
Analogy A-7

rs
m
a
h
b
o
o
b
.w
1. (c) ‘Delicious’ is the adjective used for ‘Taste’. Similarly, 30. (b) Fifth and third letters of the first term are first and

o
‘Melodious’ is the adjective used for ‘Voice’. second letters of the second term and first two letters

rd
of the first term are third and fourth letters of the second

p
2. (a) A successful finish of ‘Education’ equips one with

re
’Diploma’. Similary, a successful finish in ‘Sports’ term.

s
s
equips one with ‘Trophy’. 31. (d) First two letters of the first term are in reverse order in

.c
3. (d) The clock makes a journey of time. the second term and so are the next two letters.

o
m
4. (c) Cure ensures removal of illness in the same way as 32. (a) M A S T E R O C U V G T
remedy insures removal of grief. +2
5. (d) Bouquet is a bunch of flowers. Similarly, Sentence is +2
a set of words that is complete in itself. +2
6. (d) Wire is the medium to transmit Electricity. Similarly, +2
Pipe is the medium to carry Water. +2
7. (b) Mathematics is related to the numbers in the same way +2
History is related to Events.
8. (d) Goods are stored in godown. Similarly
9. (c) First is process of preparing the second. LABOUR NCDQWT
10. (a) First is the shorter form of second.
11. (b) First is an instrument to measure the second. 33. (d) R I D E L N B E
12. (c) First tackles the second. –6
13. (b) Dream is antonym of Reality. Similarly, falsehood is +5
antonym of truth. –2
14. (c) Framework is foundation on which house is built. +0
Similarly, body is built around skeleton.
15. (b) The words in each pair are antonyms. Similarly HELP BJJP
16. (b) First consists of the second. M U M B A I
17. (a) As medicine cures sickness, in the same way, books 34. (c) –1 –1 –1 –1 –1 –1
remove ignorance.
18. (c) Bank is the land beside a river. L T L A Z H
Similarly, coast is the land beside a sea. Similarly
19. (c) As ‘Rain’ is followed by ‘Thunder’, similarly
D E L H I
‘Darkness’ is followed by ‘Night’.
20. (d) Second is more intense than the first. –1 –1 –1 –1 –1
21. (c) Diseases are studied under Pathology. C D K G H
Similarly, planets are studied in Astronomy.
35. (b) RATIONAL : RATNIOLA TRIBAL : TRILBA
22. (c) The words in each pair are synonyms.
First 3 Remain Same Place TRI
23. (c) ‘Oasis’ is a water pool amidst sand. rd
Then coming III one LBA TRIBAL
Similarly, island is a piece of land amidst water.
Remember : ‘Sea’ would have been the answer if we Comes in IVth place. Then
had ‘desert’ in place of ‘sand’. left two took place and last
24. (b) As Major heads a battalion, the Colonel commands a if digit comes inter change them.
regiment. 36. (b) H E A T E R
25. (c) Whisper is of lesser intense than shouting, so is walking
to running. +3 –3 +3 –3 +3 –3
26. (d) Smoke cause pollution
similarly, destruction is the result of war. K B D Q H O
27. (c) Second is smallest of hour and similarly, primary is Similarly,
initial stage
28. (a) Safe is synonyms of secure and protect is synonyms C O O L E R
of guard. +3 –3 +3 –3 +3 –3
29. (b) Penology is the study of punishment in the same say
seismology is the study of earthquake. F L R I H O
y
o
u
A-8 Analogy

rs
m
37. (c) P A P E R +2 –1 +2 +1 +1 +1

a
43. (a) A C B D : E F G H

h
b
+3 +2 +4 +2 +5

o
+1

o
b
S C T G W Similarly,

.w
Similarly, +2 –1 +2 +1 +1 +1

o
rd
O Q P R : S T U V
M O T H E R

p
re
+1
+3 +2 +4 +2 +5 +2

s
s
44. (b) The letters are consecutive and written in reverse order.

.c
45. (c) In each set of letters, the 1st and 3rd letters are

o
P Q X J J T

m
consecutive.
38. (b) P O P U L A R CJDL:FMGR :: IKJR : LSMT
+1 +1 +1 +1 +1 +1 +1 46. (a) In each group the first three letters are consecutive
and they follows the fourth letter.
Q P Q V M B S
A BCD : S TUR : : J KLM : U VWX
Similarly,
G B N P V T 47. (c) The second set EGC is formed by simply putting the
first letter of CEG at last to form EGC, and so on.
–1 –1 –1 –1 –1 –1
48. (b) Because KLM are assigned No. 11, 12 & 13 from A
onwards, this corresponds to PON, which are also
F A M O U S numbered 11, 12 and 13 from Z to A in reverse order.
39. (a) C A L C U T T A Hence NOP will correspond to MLK.
49. (a) The three letters moved 5, 4, and 3 and steps forward
+4 +4 +4 +4 +4 +4 +4 +4 respectively.
50. (c) The relationship is (2x – 4) : x.
G E P G Y X X E 51. (a) 11 : 17 alternate prime number (skipping 13) 19 : 29
Similarly, alternate prime number (skipping 23)
F S Q F C E 52. (d) Second term = (First term)3
Fourth term = (Third term)3
–4 –4 –4 –4 –4 –4 53. (d) 12 = 32 + 3, 30 = 52 + 5 :
2
20 = 4 + 4 : ? = 62 + 6
B O M B Y A 2
54. (d) 3 = 2 – 1, 10 = 32 + 1
40. (d) As Similarly, 8 = 32 – 1, ? = 42 + 1
8 8 55. (b) 13 and 19 are primes with 17 left out in between.
P X J R
56. (d) 48 = 72 – 1, 122 = 112 + 1 :
8 8
R Z L T 168 = 132 – 1, ? = 172 + 1
8 8 57. (c) 5 × 5 = 25 – 4 = 21
L T F N
7 × 7 = 49 – 6 = 43
8 8
N V H P 58. (a) 210 = (15)2 – 15
15 + 5 = 20
41. (d) As, Similarly, 380 = (20)2 – 20
14 14 182 = (14)2 – 14
A O S G
(19)2 – 19 = 342 14 + 5
18 18
C U U M 59. (b) 16 = 8 × 2
20 20 56 = 8 × 7
F Z X R Similarly 32 = 16 × 2
J
0
J B
0
B = 16 × 7 = 112
42. (d) As, Similarly, 60. (d) 2 3 13 5 4 25 + 16 = 41
7 7
A H M T
6 6
C I O U
2 2 2 2
7 7 2 +3 5 +4
E L Q X
y
o
u
Analogy A-9

rs
m
77. (c) Waitress is a person whose job is to serve customers
61. (b) 49 21 169 39

a
in a restaurant.

h
b
Similarly, A teacher teaches students in a school.

o
o
b
(a) A R O U N D R A U O D N

.w
2 2 78.
7 7× 3 13 13 × 3

o
rd
Similarly,
62. (d) 25 37 49 65

p
re
G R O U N D R G U O D N

s
s
.c
o
2 2 2 2
(7 + 1) + 1

m
5 (5 + 1) + 1 7
79. (b) APP R O A C H E D R O A C H E D A PP
63. (d) 2 + 1 + 4 + 9 + 3 + 7 + 0 = 26
2 + 8 + 5 + 4 + 6 + 1 + 0 = 26 Similarly,
2 + 7 + 3 + 5 + 6 + 3 + 0 + 26
BAR G A I N E D G A I N E D BAR
Therefore, 9 + 4 + 3 + 7 + 1 + 0 + 2 = 26
64. (c) Petrology is the branch of geology that deals with the
80. (c) 8 8 64 4 256
origin, composition, structure and alteration of rocks.
Therefore, palacontology is the study of fossils to 10 10 100 5 500
determine the structue and evolution of extinct animals
and plants and the age and the conditions of deposition 21
81. (b) 7
of the rock strata in which they are found. 3
65. (c) First is the process of formation of the second. 574
574
66. (a) 27 × = 81 Similarly, 7, x 82
x 7
8 × 3 = 24
67. (c) Metre is a unit of length likewise watt is a unit of power. 82. (a) A saint practices meditation.
68. (d) As cube is 3-D of square. Similarly sphere is 3-D of Similarly,
circle. A scientist does research.
69. (b) As paper is product of Tree. Similarly glass is a product 83. (d) A palace is the official home of a king.
of sand. Similarly,
An igloo is a small round house of an Eskima.
70. (d) As, A C F J similarly, E G I N
5 5 5
84. (c) A F K P
V T R M 5 5 5
Z X U Q D I N S
71. (a) As, Similarly 5 5 5
W B G L
3 3
A D Q T 5 5 5
O T Y D
3 3
C F S V 85. (c) S
2
Q Similarly 2
M K
3 3
E H U X 2 2
I G O M
3 3
G J W Z 2 2
N L N L
72. (d) As, Similarly
2
1 1 G E S Q
E F F G
2
2 2 E C T R
G I H J
2
3 3 R P E C
I L J M
4 4 R P
K O L P
73. (b) The relationship is x : (x2 – 9). 86. (c) 18 3 1 5
74. (c) The relationship is x : x (x + 1)
12 3 1 3
75. (b) 50 = 20 × 2 + 20/2, So 100 × 2 + 100/2 = 250
76. (c) Sea sickness is to feel ill when you are travelling on a
87. (d) 12 : 20 : : 30 : 42
ship or boat.
Similarly, Giddiness is to feel that you are going to fall. (3 4) (4 5) (5 6) (6 7)
y
o
u
A-10 Classification

rs
m
CHAPTER

a
h
b
2

o
CLASSIFICATION

o
b
.w
o
rd
p
re
s
s
.c
What is classification?

o
m
You must have in your mind that what does classification mean. (c) D E F G H (d) I J K L M N
In fact, in classification we take out an element out of some given
elements and the element to be taken out is different from the rest 2 letter gap 3 letter gap
of the elements in terms of common properties, shapes, sizes, As it is clear that except option (d) all the other options have 2
types, nature, colours, traits etc. In this way the rest of the letters gap between 2nd and third letters and the 1st two letters
elements form a group and the element that has been taken out is are in consecutive order. While in case of option (d) 1st two
not the member of that group as this single element does not letters are in consecutive order but there is 3 letters gap between
possesses the common quality to be possessed by rest of the 2nd and third letter separating it out of the remaining group of the
elements. For example, if we compare the elements like, lion, cow, letters. Hence option (d) is the correct option.
tiger, panther, bear and wolf then we find that this is a group of
animals. How do we classify them? To understand this let us see EXAMPLE 2. Following are given four options and out of
the presentation given below :- them 3 form a group in terms of some similarity. Find out the
option which does not fit into that group.
Lion Cow Tiger Panther Bear Wolf (a) LMNO – ONML (b) PQRS – SRQP
(c) IJKL – LKJI (d) UVWX – VUXW
Wild Domestic Wild Wild Wild Wild L M N O O N M L
animal animal animal animal animal animal Sol. (a)
1 2 3 4 4 3 2 1
Here, if we want to separate out one animal then definitely that
P Q R S S R Q P
animal will be cow because cow is the only animal in the group (b)
which is a domestic animal. Rest of the animals (Lion, Tiger, 1 2 3 4 4 3 2 1
Panther, Bear and Wolf) are wild animals. Hence rest of the animals I J K L L K J I
(Lion, Tiger, Panther, Bear & Wolf) form a group of wild animals (c)
1 2 3 4 4 3 21
separating out the domestic animal (Cow).
Similarly, out of 6 letters A, M, N, S, P & Q, we will take out A and U V W X V U X W
form a group of 5 letters M, N, S, P & Q because out of given six (d) 1 2 3 4 2 1 4 3
letters only A is a vowel while rest of the letters form a group of Above presentation makes it clear that (d) does not fit into
consonants. the groups.
Types of classification : 2. Meaningful words based classification :- In such type of
(1) Letter/meaningless word based classification classification we have to take odd word out of the given
(2) Meaningful word based classification group of meaningful words.
(3) Digit based classification
EXAMPLE 3. Which one of the following words is not a part
(4) General knowledge based classification
Now we will discuss types of classifications one by one:- of the group formed by remaining words. The remaining words
form the group on the basis of certain similarity.
(1) Letter/meaningless word based classification :- Such
(a) Name (b) Game
classifications are based on letters of English alphabet. So
(c) Fame (d) Shame
many groups of letters are given in the question in which
Sol. (d) is the correct answer because this is the only word
one group is different from remaining groups and hence the
which has 5 letters while the remaining words have 4 letters
different group will be our answer.
and hence options (a), (b), & (c) form a group separating
EXAMPLE 1. Find the odd word out of the following options. out option (d).
(a) PQT (b) UVY
EXAMPLE 4. Out of the 4 words given below, three have
(c) DEH (d) IJN
certain thing common and so they form a group. Find out the
Sol. (a) P Q R S T (b) U V W X Y word which one is not a part of that group.
(a) Slim (b) Trims
2 letter gap 2 letter gap (c) Greets (d) Grid
y
o
u
Classification A-11

rs
m
Sol.: Option (c) is the correct option because this is the only Sol.: Option (c) is the correct answer because Madhya Pradesh

a
option which has two vowels while the other options have is an Indian state while all other options are capitals of

h
only one vowel. Let us see the following presentation:-

b
Indian states. Patna is the capital of Bihar; Mumbai is the

o
o
Sl i ms Tr i ms capital of Maharashtra and Bengaluru is the capital of

b
(a) (b)

.w
Karnataka. In case of Madhya Pradesh (it is an Indian state),
1 vowel 1 vowel

o
we can say that it has its capital in Bhopal.

rd
Gr ee ts Gr i d

p
(c) (d) EXAMPLE 8. Which of the following animals does not fit

re
2 vowels 1 vowel

s
into the group formed by remaining animals?

s
3. Digit based classification :- In such type of classifications

.c
digits or numbers are given to find out one number that is not a (a) Cat (b) Dog

o
m
part of the group of remaining numbers. (c) Tiger (d) Octopus
Sol.: Option (d) is the correct option as this is the only animal
EXAMPLE 5. Find the odd number out. out of given options which is a water animal. Rest of the
(a) 122 (b) 128 options are land animals.
(c) 199 (d) 200
Sol.: Option (c) is the correct option because this is an odd number EXAMPLE 9. Three of the following given options have some
while all the other options are even numbers. similarity and so they form a group separating out one option.
EXAMPLE 6. Four of the following numbers have some Find out that separate option.
similarity and hence they form a group. Find out the number (a) Tendulkar (b) Dravid
which does not suit in the group. (c) Sania Mirza (d) Yuvraj Singh
(a) 7842 (b) 8952 Sol.: Option (c) is the correct option because Sania Mirza is an
(c) 7122 (d) 7722 Indian tennis player. While the rest of the options are Indian
Sol.: Option (b) is the correct answer as except option (b) all cricketers.
other options start with 7 & end with 2. Now, this chapter has come to an end. Readers are advised
4. General knowledge based classification :- Such to move as per the following steps while solving the
classification is done on the basis of our general knowledge. problems related to classification :-
No doubts that this is a word based classification but Step I : See all the given options with a serious eye.
without having general knowledge this type of questions
Step II : Try to make relation of similarity among the given
can not be solved.
options.
EXAMPLE 7. Find the odd man out. Step III : Find out the one word not having the common
(a) Patna (b) Mumbai similarity like other options and that one word will be
(c) Madhya Pradesh (d) Bengaluru your answer.

DIRECTIONS (Qs. 1 to 66) : Find the odd word/letters/number 8. (a) Mathematics (b) Geometry
pair from the given alternatives. (c) Algebra (d) Trigonometry
1. (a) Rice (b) Wheat 9. (a) Advice (b) Counsel
(c) Barley (d) Mustard (c) Direct (d) Suggest
2. (a) Arrow (b) Sword 10. (a) Petrol (b) Acetone
(c) Knife (d) Axe (c) Mercury (d) Kerosene
3. (a) Listen (b) Feel 11. (a) Medium (b) Average
(c) Think (d) Sing (c) Mediocre (d) Terrible
4. (a) Volume (b) Size 12. (a) Travelled (b) Sailed
(c) Large (d) Shape (c) Walked (d) Rode
5. (a) Anxiety (b) Worry 13. (a) Telescope (b) Transistor
(c) Inhibition (d) Curiosity (c) Taperecorder (d) Telephone
6. (a) Seismograph (b) Earthquake 14. (a) Lord Dalhousie (b) Lord Mountbatten
(c) Cyclone (d) Tsunami (c) Lord Linlithgow (d) Lord Tennyson
7. (a) Nephrology (b) Entomology 15. (a) Microbe (b) Microflim
(c) Astrology (d) Mycology (c) Microphone (d) Microscope
y
o
u
A-12 Classification

rs
m
16. (a) Fore ward (b) Text 42. (a) 72, 60 (b) 108, 96

a
(c) Index (d) preface (c) 84, 72 (d) 60, 36

h
b
17. (a) Analysis (b) Search 43. (a) 12, 8 (b) 6, 16

o
o
(c) Conclusion (d) Investigation (c) 18, 6 (d) 32, 3

b
18. (a) Room (b) Chamber

.w
44. (a) 62 - 37 (b) 74 - 40
(c) Veranda (d) Cabin

o
(c) 85 - 60 (d) 103 - 78

rd
19. (a) ISLOJ (b) LUOQM 45. (a) 81 : 243 (b) 16 : 64

p
(c) AKDGB (d) FPILG

re
(c) 64 : 192 (d) 25 : 75

s
20. (a) HJN (b) JLP 46. (a) 52, 142 (b) 54, 126

s
.c
(c) PRU (d) QSW (c) 56, 168 (d) 58, 184

o
m
21. (a) PORM (b) WVYT 47. (a) 14, 9 (b) 17, 8
(c) KJMH (d) FEJC (c) 42, 3 (d) 21, 6
22. (a) GJOV (b) BFJQ 48. (a) 117, 13 (b) 162, 18
(c) ILQX (d) RUZG (c) 171, 19 (d) 304, 16
23. (a) BFJM (b) DEFG 49. (a) Fervent (b) Enthusiastic
(c) JMPS (d) PRTV (c) Apathetic (d) Ardent
24. (a) CXGT (b) EVBY 50. (a) FAA (b) OFF
(c) DXEY (d) AZDW (c) ATT (d) EPP
25. (a) AZMN (b) BYEF 51. (a) 117 – 143 (b) 142 – 156
(c) CXUV (d) IRDV (c) 64 – 78 (d) 103 – 169
26. (a) EBD (b) IFH 52. (a) Radio (b) Television
(c) QMO (d) YVX (c) Transistor (d) Tube
27. (a) BDGK (b) JLOS 53. (a) Pathology (b) Geology
(c) HJMQ (d) MORU (c) Cardiology (d) Radiology
28. (a) BCYZ (b) EFVW 54. (a) Rivulet (b) Stream
(c) DEUZ (d) ABZA (c) River (d) Pond
29. (a) PROQN (b) DECEG 55. (a) Konark (b) Madurai
(c) GIFHE (d) KMJLI (c) Dilwara (d) Ellora
30. (a) ACEGF (b) FHJLK 56. (a) RTW (b) QOM
(c) KMOQP (d) UWYZA (c) IKG (d) IKM
31. (a) CADGE (b) JHKNL 57. (a) EFH (b) OPQ
(c) ECFGI (d) XVYBZ (c) BCE (d) IJL
32. (a) AI (b) IU 58. (a) DH (b) FJ
(c) EO (d) OU (c) HK (d) PR
33. (a) PUT (b) OWL 59. (a) 24 (b) 49
(c) END (d) ARM (c) 80 (d) 15
34. (a) 126 (b) 122 60. (a) 121 (b) 324
(c) 65 (d) 50 (c) 523 (d) 729
35. (a) 226 (b) 290 61. (a) 704, 11 (b) 256, 4
(c) 360 (d) 170 (c) 832, 13 (d) 310, 5
36. (a) 20, 10 (b) 30, 18 62. (a) Tennis (b) Cricket
(c) 45, 27 (d) 60, 30 (c) Volleyball (d) Football
37. (a) 7, 26 (b) 8, 30 63. (a) artificial (b) insincere
(c) 10, 35 (d) 15, 30 (c) affected (d) naive
38. (a) 1/2, 1/8 (b) 1/3, 1/12 64. (a) 68 (b) 85
(c) 9, 3 (d) 24, 6 (c) 153 (d) 174
39. (a) 3, 5 (b) 7, 9 65. (a) 7 : 98 (b) 9 : 162
(c) 11, 13 (d) 17, 19 (c) 12 : 288 (d) 17 : 572
40. (a) 7 : 18 (b) 9 : 26
(c) 11 : 36 (d) 13 : 42
41. (a) 80 – 9 (b) 64 – 8 66.
(c) 36 – 6 (d) 7 – 49
(a) (b) (c) (d)
y
o
u
Classification A-13

rs
m
ANSWER KEY

a
h
b
1 (d) 8 (a) 15 (a) 22 (b) 29 (b) 36 (a) 43 (c) 50 (a) 57 (b) 64 (d)

o
o
2 (a) 9 (c) 16 (b) 23 (a) 30 (d) 37 (b) 44 (b) 51 (a) 58 (c) 65 (d)

b
(d) (c) (c) (c) (c) (c) (b) (d) (b)

.w
3 10 17 24 31 38 45 52 59 66 (c)
4 (c) 11 (d) 18 (c) 25 (d) 32 (d) 39 (b) 46 (c) 53 (b) 60 (c)

o
rd
5 (d) 12 (a) 19 (b) 26 (c) 33 (a) 40 (c) 47 (c) 54 (d) 61 (d)

p
re
6 (a) 13 (a) 20 (c) 27 (d) 34 (a) 41 (a) 48 (d) 55 (d) 62 (b)

s
7 (c) 14 (d) 21 (d) 28 (c) 35 (c) 42 (d) 49 (c) 56 (a) 63 (d)

s
.c
o
m
1. (d) Except ‘mustard’ each belongs to the same category, 24. (c) In all other groups, the first and second letters occupy
viz food grains. Mustard is an oilseed. the same positions from the beginning and end of the
2. (a) All others are held in the hand and not shot out. alphabet, and so do the third and fourth letters.
3. (d) All others are the features of sense organes. 25. (d)
4. (c) ‘Large’ is an adjective whereas others are noun. A Z M N B Y E F
5. (d) All others are negative. (a) (b)
6. (a) Except (a) all are natural disaster. Seismograph is an opposite +1 opposite +1
instrument scientists use to measure earthquakes.
C X U V I R D V
7. (c) All except Astrology are concerned with biology.
(c) (d)
8. (a) MATHEMATICS is STREAM/Main Category. opposite +1 opposite +19
Others are substream or sub category of Mathematics. Hence, 'IRDV' does not belong to that group.
9. (c) Direct is different from the other three words. Direct 26. (c) The pattern is –
means 'to give a formal order or command.'
10. (c) Except Mercury all others are organic compounds. E B D I F H
Mercury is a metal. ; ;
11. (d) All except terrible are synonyms. +3 +2 +3 +2
12. (a) All others are different modes of travel. Q M O Y V X
13. (a) All except Telescope are audio devices. ;
14. (d) All except Lord Tennyson were either the Governor- +4 +2 +3 +2
General or the Viceroy of India. Hence, QMO is the odd one out.
15. (a) Microbe is living organism other are scientific 27. (d) The pattern is –
apparatus. B D G K J L O S
16. (b) Text is different from other three. ;
17. (c) Conclusion is the final stage of analysis, search and +2 +3 +4 +2 +3 +4
investigation. H J M Q M O R U
18. (c) Except veranda, all others are surrounded by four walls. ;
+2 +3 +4 +2 +3 +3
19. (b) I S L O J L U O Q M Hence, MORU is odd one out.
28. (c) The pattern is
+10 –7 +3 –5 +9 –6 +2 –4
B CY Z E F V W
A K D G B F P I L G ;
+10 –7 +3 –5 +1 +1 +1 +1
+10 –7 +3 –5
D E U Z A BZ A
20. (c) In all other groups, the third letter is four steps ahead
of the second letter which, in turn, is two steps ahead ;
+1 +5 +1 +1
of the first letter.
Hence, DEUZ is odd one out.
21. (d) In all other groups, first and third letters as well as
fourth and second letters are alternate. 29. (b) Pattern is–
22. (b) In all other groups, the second, third and fourth letters 2 3 2 3
P R O Q N
are respectively three, five and seven steps ahead of So that, DECEG is out.
the first, second and third letters respectively. 30. (d) Pattern is–
23. (a) In all other groups, the number of letters skipped 2 2 2 1
between adjacent letters is uniform. A C E G F
So that, UWYZA is out.
y
o
u
A-14 Classification

rs
m
31. (c) 2 3 3 2 47. (c) 42 is a multiple of 3.
C A D G E

a
48. (d) 13 × 9 = 117

h
So that, ECFGI is out.

b
18 × 9 = 162

o
32. (d) VOWELS Only “OU” Comes in

o
19 × 9 = 171

b
A I O U E a sequence of cronical order.

.w
But, 16 × 9 = 144 not 304

o
33. (a) “PUT” Not start with vowel among these options. 49. (c) Fervent: having or displaying a passionate intensity.

rd
34. (a) The rest are based on the expression x2+ 1. Enthusiastic: having or showing intense and eager

p
re
But 126 = 112 + 5. enoyment.

s
s
35. (c) After a close look you will get that except 360 each Ardent: very enthusiastic or passionate

.c
number is one more than square of a natural number,

o
Apathetic: showing or feeling no interest, enthusiasm

m
i.e., 226 = 152 + 1 ; 290 = 172 + 1 ; 170 = 132 + 1 ; or concern.
122 = 112 + 1 Hence, apathetic is odd one out.
36. (a) 20, 10. There not divisible by 3. Other will. 50. (a) In all other groups, a vowel is followed by a consonant
37. (b) 8, 30 only there divisible by 2. repeated twice.
38. (c) 9, 3 3 :1 51. (a) Only 117-143 is divisible by 13. Therefore, it is odd one
out.
Others are 4 :1 52. (d) Radio, Transistor and television are the way of
39. (b) 7, 9 Here 9 is not Prime No. broadcasting. Hence, tube is odd one out.
Others pairs has all Prime Nos. 53. (b) As all terms given in question are medical terms except
40. (c) In all other paris, 2nd number = (1st number × 4) – 10. geology.
41. (a) In all other pairs, one number is the square of the other. 55. (d) All except Ellora are famous for temples, while Ellora is
42. (d) The difference in all the other cases is 12. famous for caves.
43. (c) The product in all other cases is 96. 56. (a) Consonants and vowel are used in option (b), (c) and
(d). Only consonants are used in option (a).
44. (b) In all other pairs of numbers the difference between
the two number is of 25. 57. (b) E F H O P Q
+1 +2 +1 +1
62 - 37 = 25 But, 74 - 40 = 34
85 - 60 = 25 B C E I J L
103 - 78 = 25 +1 +2 +1 +2
45. (b) The second numbers is three times the first number
except in the case of option (b) 58. (c) D H F J H K P R
81 × 3 = 243
64 × 3 = 192 4 8 6 10 8 11 16 18
25 × 3 = 75
But, 16 × 4 = 64
46. (c) 52 5 + 2 = 7 Pair of odd and even number
142 1 + 4 + 2 = 7 59. (b) 24 = 52 – 1 80 = 92 – 1
2
49 = 7 – 0 15 = 42 – 1
54 5 + 4 = 9
126 1 + 2 + 6 = 9 60. (c) All are squares except (c).
61. (d) 704 11 = 64 256 4 = 64
56 5 + 6 = 11 1+1= 2
832 13 = 64 310 5 = 62
168 1 + 6 + 8 = 15 1+5= 6 64. (d) Except (d) all others are divided by 17.
58 5 + 8 = 13 1 + 3 = 4 65. (d) Except (d) all others second number is divisible by the
184 1 + 8 + 4 = 13 1 + 3 = 4 first one.
y
o
u
Series A-15

rs
m
CHAPTER

a
h
b
3

o
SERIES

o
b
.w
o
rd
p
re
s
s
.c
A series is a sequence of numbers/alphabetical letters or both which follow a particular rule. Each element of series is called

o
m
‘term’. We have to analyse the pattern and find the missing term or next term to continue the pattern.
Types of series are explained in the following chart :

SERIES

Number Alphabet Alpha-numeric Mixed Letter Correspondence


series series series series series series

A series that is A series that is A series in A series which A series of letters, A series consists of
made by only made by only which both is created by which follow a three sequence with
alphabets and the combination certain pattern, is three different
number or digit alphabetic letters of two or more
numbers are given with four / elements (for ex.
than two series five times blank capital letters,
used
1. Ascending series spaces in between. numbers and small
The order of letters). An element
2. Descending series
missing letters of each sequence is
3. Oscillating series correspond to the
is correct answer.
element of other
sequence on the
basis of the similarity
in position.

In number series, relationship between the terms is of any kind. Sol. (b) 4 6 9 13 18 Correct answer
For example.
+2 +3 +4 +5
(1) Consecutive even numbers
(2) Consecutive odd numbers EXAMPLE 2. Choose the next term of series given below.
(3) Consecutive prime numbers 64, 32, 16, 8, ?
(a) 0 (b) 1 (c) 2 (d) 4
(4) Square of numbers Sol. (d) Each number is half of its previous number.
(5) Cubes of numbers
(6) Square root of numbers (II) To find the missing number of series :
(7) Omission of certain number of letter in any consecutive
order EXAMPLE 3. What will come in place of question mark in
(8) Addition /subtraction/ multiplication/ division by some the following series?
number ( For Ex. A.P & G.P) or any other relation. 79, 87, ? , 89, 83
TYPES OF QUESTIONS : (a) 80, (b) 81 (c) 82 (d) 88
(I) Complete the series
Sol. (b) 79 87 81 89 83
(II) Find Missing number of the series
(III) Find Wrong number of the series +8 -6 +8 -6
EXAMPLES ON NUMBER SERIES EXAMPLE 4. What will come in place of question mark in
(I) Complete the series the following series?
EXAMPLE 1. Which of the following is the next term of 37, 41, ? , 47, 53
series given below ?
(a) 42 (b) 43 (c) 46 (d) 44
4, 6, 9, 13, ....
(a) 17 (b) 18 (c) 19 (d) 20 Sol. (b) Consecutive prime numbers.
y
o
u
A-16 Series

rs
Sol. (a) Alphabets follow the sequence

m
EXAMPLE 5. What will come in place of question mark in

a
M

h
the following series? K P T Y

b
o
21, 34, ? , 89, 144

o
+2 +3 +4 +5

b
(a) 43 (b) 55 (c) 64 (d) 71

.w
And numbers are increasing by 2
Sol. (b) Each number is the sum of the two preceding

o
rd
numbers. EXAMPLES ON MIXED SERIES

p
21 + 34 = 55

re
EXAMPLE 10. Complete the series

s
34 + 55 = 89

s
Z, L, X, J, V, H, T, F, __, __

.c
55 + 89 = 144

o
(a) D, R (b) R, D (c) D, D (d) R, R

m
(III) To find the wrong term in the series : Sol. (b) The given sequence consists of two series
EXAMPLE 6. Find the wrong term in the series (i) Z, X, V, T, __
3, 8, 15, 24, 34, 48, 63. (ii) L, J, H, F, __. Both consisting of alternate letters in
the reverse order.
(a) 15 (b) 15 (c) 34 (d) 63
Sol. (c) 2 – 1, 3 – 1, 4 – 1, 5 – 1, 62 – 1
2 2 2 2 Next term of (i) series = R, and
Next term of (ii) series = D
EXAMPLES ON ALPHABETIC SERIES
EXAMPLE 11. What will come in place of question mark
EXAMPLE 7. What will come in place of question mark in in the following series?
the following series?
7, 5, 26, 17, 63, 37, 124, 65, ?, ?
G, H, J, M, ? (a) 101, 215 (b) 101, 101
(a) R (b) S (c) Q (d) P (c) 215, 101 (d) 215, 215
Sol. (c) G H J M Q Sol. (c) The given series consists of two series
(i) 7, 26, 63, 124 .....
+1 +2 +3 +4
(ii) 5, 17, 37, 65 .....
EXAMPLE 8. What will come in place of question mark in In the first series,
the following series? 7 = 23 – 1, 26 = 33 – 1, 63 = 43 – 1 ,
BF, CH, ? , HO, LT 124 = 53 – 1, 63 – 1 = 215
(a) FG (b) EK (c) CE (d) F J and in the second series.
5 = 22 + 1, 17 = 42 + 1,
+2 +3 +4 +5 37 = 62 + 1, 65 = 82 + 1,
Sol. (b) BF CH EK HO LT 102 + 1 = 101

+1 +2 +3 +4 EXAMPLES ON LETTER SERIES

EXAMPLES ON ALPHA-NUMERIC SERIES EXAMPLE 12. Which sequence of letters when placed at
the blanks one after another will complete the given letter
EXAMPLE 9. What will come in place of question mark in
series?
the following series?
baab–aba–bba– –
K 1, M 3, P 5, T 7, ? (a) b baa (b) aaaa (c) ab ab (d) b aba
(a) Y 9 (b) Y 11 (c) V 9 (d) V 11 Sol. (d) b a a b b a / b a a b b a / b a.
y
o
u
Series A-17

rs
m
a
h
b
o
o
b
.w
DIRECTIONS (Qs. 1 to 56) : Complete the given series. 21. MHZ, NIW, OKT, PNQ,?

o
rd
1. BDF, CFI, DHL, ? (a) RRN (b) QRN (c) QRM (d) QQN

p
(a) CJM (b) EIM (c) EJO (d) EMI 22. A, CD, GHI, ?, UVWXY

re
2. FAG, GAF, HAI, IAH, _______

s
(a) LMNO (b) MNO (c) NOPQ (d) MNOP

s
.c
(a) JAK (b) HAK (c) JAI (d) HAL 23. CAT, FDW, IGZ, ?

o
m
3. DILQTYBG? (a) KJA (b) KTC (c) LHD (d) LJC
(a) H (b) I (c) J (d) P 24. 1, 3, 8, 19, 42, 89, ?
4. BC FG JK __ RS VW (a) 108 (b) 184 (c) 167 (d) 97
(a) LM (b) OP (c) QR (d) NO 25. 5, 16, 51, 158, ..... ?
5. BYA, CXB, ?, EVD : (a) 1452 (b) 483 (c) 481 (d) 1454
(a) DVE (b) DCW (c) DXB (d) DWC
26. 8, 13, 10, 15, 12, 17, 14 ... ?
6. LXF, MTJ, NPN, OLR, ?
(a) 19 (b) 22 (c) 16 (d) 20
(a) PHV (b) PIU (c) PKX (d) PJW
27. 3, 15, 4, 16, 5, 17, 6, ?, 7
7. YANWY, DFMBD, IKNGI, NPMLN, (?), XZMVX
(a) 12 (b) 18 (c) 15 (d) 13
(a) RUMSR (b) SUNQS
28. 240, ... 120, 40, 10, 2
(c) UWNSU (d) VUMTV
(a) 480 (b) 240 (c) 220 (d) 120
8. PEXKW, RFWMU, TGVOS, VHUQQ, XITSO, (?)
29. 6.25, 9, 12.25, 16, 20.25, 25, 30.25, ?
(a) ZJSUM (b) YJSUZ (c) ZKSVJ (d) JZSTN
9. AYBZC, DWEXF, GUHVI, JSKTL, (?), POQPR (a) 36 (b) 32 (c) 28.25 (d) 40.25
(a) MQDRN (b) QMONR 30. 0, 7, 26, ?, 124, 215
(c) MQNRO (d) NQMOR (a) 51 (b) 37 (c) 63 (d) 16
10. ZYYZR, ABVUN, (?), BCUTM, XWABT, CDTSL 31. 2 12 36 80 150 ?
(a) YXZAS (b) ZYABT (a) 194 (b) 210 (c) 252 (d) 258
(c) XWYZR (d) YXZAB 32. 1, 8, 9, 64, 25, 216, ?, ?
11. deb, ijg, nol, (?), xyv (a) 49, 64 (b) 343, 64 (c) 49, 512 (d) 343, 512
(a) rsp (b) stp (c) rsq (d) stq 33. 3, 13, 53, 213 — ?
12. A, Q, C, L, F, H, J, ? (a) 553 (b) 653 (c) 753 (d) 853
(a) E (b) B (c) F (d) D 34. 4, 16, 36,?, 100, 144
13. BYCXW, CXDWV, EVFUT, GTHSR, ? (a) 72 (b) 68 (c) 81 (d) 64
(a) IRJQP (b) KPOLN (c) KPLON (d) JOKPO 35. 8, 15, 28, 53, ...?
14. FOX, GP?, HQZ (a) 98 (b) 106 (c) 100 (d) 102
(a) Y (b) Z (c) T (d) W 36. 6, 12, 21, ?, 48
15. QPO, SRQ, UTS, WVU, (?) (a) 38 (b) 40 (c) 45 (d) 33
(a) XVZ (b) YXW (c) ZYA (d) VWX 37. 1, 4, 27, 16, ?, 36, 343
16. YEB, WFD, UHG, SKI, (?)
(a) 87 (b) 120 (c) 25 (d) 125
(a) QOL (b) TOL (c) QGL (d) QNL
38. 28, 33, 31, 36, ? 39
17. AZ, CX, FU, (?)
(a) 32 (b) 34 (c) 38 (d) 40
(a) IR (b) JQ (c) IV (d) KP
39. 165, 195, 255, 285, 345, ?
18. ABD, DGK, HMS, MTB, SBL, ?
(a) 375 (b) 390 (c) 420 (d) 435
(a) ZAB (b) XKW (c) ZKU (d) ZKW
40. 1, 5, 14, 30, 55, 91, ?
19. OTE, PUF, QVG, RWH, ?
(a) SYJ (b) TXI (c) SXJ (d) SXI (a) 130 (b) 140 (c) 150 (d) 160
20. BEH, KNQ, TWZ, ? 41. 625, 5, 125, 25, 25, ?, 5
(a) IJL (b) CFI (c) BDF (d) ADG (a) 5 (b) 25 (c) 125 (d) 625
y
o
u
A-18 Series

rs
63. ba _ b _ aab _ a _ b

m
42. 6, 18, 3, 21, 7, 56, ?

a
(a) 8 (b) 9 (c) 63 (d) 64 (a) abaa (b) abba (c) baab (d) babb

h
b
43. 198, 194, 185, 169, ? 64. c _ bba _ cab _ ac _ ab _ ac

o
o
(a) abcbc (b) acbcb (c) babcc (d) bcacb

b
(a) 136 (b) 144 (c) 9 (d) 92

.w
44. 4, 9, 25, ?, 121, 169, 289, 361 65. aab – cc – daa – bbb – ccddd

o
rd
(a) 49 (b) 74 (c) 89 (d) 91 (a) bdbd (b) ddca (c) dbbc (d) bdac

p
45. 2, 5, 10, 17, 26, 37, ? 66. adb_ac_da_cddcb_dbc_cbda

re
s
(a) 61 (b) 58 (c) 50 (d) 72 (a) bccba (b) cbbaa (c) ccbba (d) bbcad

s
.c
46. 24, 60, 120, 210, ? DIRECTIONS (Qs. 67-76): In each of the following number series,

o
m
(a) 300 (b) 336 (c) 420 (d) 525 a wrong number is given. Find out that number.
47. 462, 420, 380, ?, 306 67. 10 15 24 35 54 75 100
(a) 322 (b) 332 (c) 342 (d) 352 (a) 35 (b) 75 (c) 24 (d) 15
48. 121, 225, 361, ? 68. 1 3 4 7 11 18 27 47
(a) 441 (b) 484 (c) 529 (d) 729 (a) 27 (b) 11 (c) 18 (d) 7
49. J2Z, K4X, I7V, ?, H16R, M22P 69. 3 2 3 6 12 37.5 115.5
(a) I11T (b) L11S (c) L12T (d) L11T (a) 37.5 (b) 12 (c) 6 (d) 2
50. 3F, 6G, 11I, 18L, ? 70. 2 8 32 148 765 4626 32431
(a) 21O (b) 25N (c) 27P (d) 27Q (a) 765 (b) 148 (c) 8 (d) 32
51. KM5, IP8, GS11, EV14, ? 71. 2 3 11 38 102 229 443
(a) BX17 (b) BY17 (c) CY18 (a) 11 (b) 229 (c) 120 (d) 38
(d) CY17 72. 5 10 17 27 37 50 65
52. Q1F, S2E, U6D, W21C,? (a) 10 (b) 17 (c) 27 (d) 37
(a) Y44B (b) Y66B (c) Y88B (d) Z88B 73. 108 54 36 18 9 6 4
53. 2Z5, 7Y7, 14X9, 23W11, 34V13, ? (a) 54 (b) 36 (c) 18 (d) 9
(a) 27U24 (b) 45U15 (c) 47U15 (d) 47V14 74. 2 3 5 8 14 23 41 69
54. W-144,?, S-100, Q-81, O-64 (a) 5 (b) 8 (c) 6 9 (d) 41
(a) U-121 (b) U-122 (c) V-121 (d) V-128 75. 0 1 9 36 99 225 441
55. 2 3 B __ 6 __ F G __ 5 D __ 8 __ H I (a) 9 (b) 36 (c) 99 (d) 225
(a) C, 7, 4, E, 9 (b) D, 8, 6, C, 7 76. 3 7. 5 15 37.5 75 167.5 375
(c) E, 8, 7, D, 9 (d) W, 8, 7, I, 6 (a) 167.5 (b) 75 (c) 37.5 (d) 15
56. 2 A 11, 4 D 13, 12 G 17 ? DIRECTIONS (Qs. 77-84): In a given series, with one term
(a) 36 J 21 (b) 36 I 19 (c) 48 J 21 (d) 48 J 23 missing. Choose the correct alternative from the given ones
that will complete the series.
DIRECTIONS (Qs. 57-66): Which sequence of letters when placed
at the blanks one after another will complete the given letter 77. AKU, FPZ, ?, PZJ, UEO, ZJT
series ? (a) JUE (b) KVE (c) KUE (d) JVE
57. ba _ b _ aab _ a _ b 78. MRS, LTU, KVW, ?
(a) abaa (b) abba (c) baab (d) babb (a) TQR (b) MOP (c) JXY (d) CDE
58. c _ bba _ cab _ ac _ ab _ ac 79. B, G, K, N, ?
(a) abcbc (b) acbcb (c) babcc (d) bcacb (a) P (b) O (c) H (d) L
59. abca — bcaab — ca — bbc — a 80. 1944, 108, ?, 6, 3
(a) ccaa (b) bbaa (c) abac (d) abba (a) 16 (b) 18 (c) 11 (d) 12
60. b — b — bb — bbb — bb — b 81. 251 (12) 107
(a) bbbbba (b) bbaabb (c) ababab (d) aabaab 381 (?) 125
61. aa – bb – aa – abbbb – a (a) 14 (b) 24 (c) 11 (d) 16
(a) bbaa (b) aabb (c) baba (d) abab 82. 354 (110) 526
62. – aba – cabc – dcba – bab – a 297 (?) 2631
(a) abdca (b) bcadc (c) abcdd (d) cbdaa (a) 128 (b) 116 (c) 135 (d) 143
y
o
u
Series A-19

rs
m
83. 0, 7, 26, 63, ? 90. 0, 5, 60, 615,?

a
(a) 125 (b) 126 (c) 217 (d) 124 (a) 6030 (b) 6170 (c) 6130 (d) 6000

h
b
84. 2, 5, 10, 19, 36, ?

o
91. Which one set of letters when sequentially placed at the

o
(a) 70 (b) 71 (c) 68 (d) 69 gaps in the given letter series shall complete it ?

b
.w
85. Which one set of letters when sequentially placed at the ac__c_cb_acbcacbca_bc

o
gaps in the given letter series shall complete it?

rd
(a) abbb (b) bacc (c) babc (d) bbcc

p
a _ cbc _ ca _ ab _ bca _ ab
92. rtx_sx_z_txy_ _yz

re
(a) b a b c c (b) b c a b b

s
(a) yyr x s (b) yys x r (c) yyr s x (d) yyx r s

s
.c
(c) a b c b c (d) b c a b c
DIRECTIONS (Qs. 93-96) : A series is given, with one term

o
DIRECTIONS (Qs. 86 -90) : In the following questions a series

m
missing. Choose the correct alternative from the given ones that
is given, with one term missing. Choose the correct alternative
will complete the series.
from the given ones that will complete the series.
86. BCFG, JKNO, RSVW, ? 93. 1, 2, 6, 24, ? ,720
(a) ZADE (b) HIKL (c) STUX (d) MNPQ (a) 3 (b) 5 (c) 120 (d) 8
87. CIM, HNR, MSW, ? 94. 156, 506, ?, 1806
(a) SXA (b) UYB (c) RXB (d) ZEH (a) 1056 (b) 856 (c) 1456 (d) 1506
88. 2, 3, 6, 7, 14, 15, ? 95. 3, 6, 9, 15, 24, 39, 63, ?
(a) 16 (b) 30 (c) 31 (d) 32 (a) 100 (b) 87 (c) 102 (d) 99
89. 3120, ? , 122, 23, 4 96. –1, 0, ?, 8, 15, 24
(a) 488 (b) 621 (c) 610 (d) 732 (a) 4 (b) 3 (c) 2 (d) 1

ANSWER KEY
1 (c) 14 (a) 27 (b) 40 (b) 53 (c) 66 (b) 79 (a) 92 (c)
2 (a) 15 (b) 28 (b) 41 (c) 54 (a) 67 (a) 80 (b) 93 (c)
3 (c) 16 (a) 29 (a) 42 (a) 55 (a) 68 (a) 81 (d) 94 (a)
4 (d) 17 (b) 30 (c) 43 (b) 56 (d) 69 (b) 82 (b) 95 (c)
5 (d) 18 (d) 31 (c) 44 (a) 57 (b) 70 (d) 83 (d) 96 (b)
6 (a) 19 (d) 32 (c) 45 (c) 58 (b) 71 (b) 84 (d)
7 (b) 20 (b) 33 (d) 46 (b) 59 (c) 72 (c) 85 (a)
8 (a) 21 (b) 34 (d) 47 (c) 60 (c) 73 (d) 86 (a)
9 (c) 22 (d) 35 (d) 48 (c) 61 (a) 74 (c) 87 (c)
10 (a) 23 (d) 36 (d) 49 (d) 62 (a) 75 (c) 88 (b)
11 (d) 24 (b) 37 (d) 50 (c) 63 (b) 76 (a) 89 (b)
12 (a) 25 (c) 38 (b) 51 (d) 64 (b) 77 (c) 90 (b)
13 (a) 26 (a) 39 (d) 52 (c) 65 (d) 78 (c) 91 (b)
y
o
u
A-20 Series

rs
m
a
h
b
o
o
b
.w
+2 +2 +3 +3 +4 +4 9. (c) The Pattern is–

o
1. (c) B D F, C F I, D H L,

rd
A Y B Z C D W E X F
+5 +5

p
+3

re
E J O

s
1 1 1 1 –2
2. (a)

s
F G H I J

.c
+3

o
0 0 0 0

m
A A A A A –2
+3
1 3 1 3
G F I H K Therefore,
3. (c) After D, 4 letters are skipped and I follows after which M Q N R O P O Q P R
2 letters are skipped Trend flows alternatively : +3
D(EFGH) I (JK) L(MNOP) Q (RS) T(UVWX) Y(ZA)
–2
B(CDEF) G(HI) J.
+3
3 3 3 3
4. (d) BC FG JK NO RS –2
5. (d) Sequence is : +3
1st letter E (next letter from B to E) 10. (a) This is two alternate series.
B C D
1 1
2nd letter Y X W V (reverse from V to Y) Z Y X
3rd letter A B C D (next letter from A to D) 1 1
Y X W
6. (a) Ist Letter:
1 1
1 1 1 1 Y Z A
L M N O P
1 1
2nd Letter : Z A B
4 4 4 4 1 1
X T P L H R S T
11. (d) Sequence is :
3rd Letter :
5 5 5 5
4 4 4 4 d i n s x
F J N R V
5 5 5 5
7. (b) Sequence is : e j o t y
5 5 5 5 5 5 5 5 5
Y D I N S X b g l q v
5 5 5 5 5 –5 –4 –3
A F K P U Z
1 1 1 1 1 A Q C L F H J E
N M N M N M 12. (a)
5 5 5 5 5 +2 +3 +4
W B G L Q V +1 +2 +2 +2
13. (a) B C E G I
5 5 5 5 5
Y D I N S X Y
–1
X
–2
V
–2
T
–2
R
8. (a) The Pattern is– C
+1
D
+2
F
+2
H
+2
J
P E X K W R F W M U –1 –2 –2 –2
X W U S Q
+2
–1 –2 –2 –2
W V T R P
+1
+1 +1
–1 14. (a) +1
+2 FOX GPY HQZ
–2
Therefore, +1 +1
X I T S O Z J S U M 15. (b) Each letter moves +2 steps.
+2 16. (a) 1st letter moves –2 steps each time.
+1 2nd letter moves +1, +2, +3, +4 steps respectively.
–1 3rd letter moves +2, +3, steps alternatively.
+2 17. (b) 1st letter moves +2, +3 and +4 steps respectively.
–2 2nd letter moves –2, –3 and –4 steps respectively.
y
o
u
Series A-21

rs
18. (d)

m
+5 +6 +7 +8 +9 31. (c) 13 + 12 = 2, 23 + 22 = 12, 33 + 32 = 36 and so on 63 +

a
62 = 252

h
b
32. (c) The odd terms are squares of the numbers 1,3,5,....
A BD D G K HMS MT B S B L ZKW

o
while the even terms are cubes of the numbers 2,4,6,....

o
b
+3 +4 +5 +6 +7 33. (d) The difference in consecutive nos. follows the

.w
+7 +8 +9 +10 +11 sequence, 10, 40, 160.

o
So the next difference = 160 × 4 = 640

rd
19. (d) The first letters of the terms are in alphabetical order,

p
and so are the second and third letters. Therefore number = 213 + 640 = 853.

re
34. (d) The numbers in the series are square of 2,4,6 ...,10,12 .

s
20. (b) All the letters of each term are moved nine steps Therefore, the missing number is square of 8 i.e. 82=64.

s
.c
forward to obtain the corresponding letters of the next
35. (d) 8 × 2 – 1 = 15, 15 × 2 – 2 = 28, 28 × 2 – 3 = 53,

o
term.

m
53 × 2 – 4 =102
21. (b) The first letters of the terms are consecutive letters. 36. (d) The common differences between the consecutive
The third letter of each term is moved three steps terms of the series are 6, 9, 12, 15, etc.
backward to obtain the third letter of the successive 37. (d) The series is 13, 22, 33, 42, 53, 62, 73.
term. The middle letters of the first, second, third and 38. (b) The pattern is +5, –2, +5, –2, .....
fourth terms are moved one, two, three, and four steps So, missing term = 36 –2 = 34.
forward respectively to obtain the middle letter of the 39. (d) Each number of the series is 15 multiplied by a prime
successive terms. number i.e. 15 × 11, 15 × 13, 15 × 17, 15 × 19, 15 × 23, .....
So, missing term = 15 × 29 = 435.
22 (d) The number of letters in the terms goes on increasing
40. (b) The pattern is + 4, +9, + 16, + 25, + 36, ..... i.e. + 22, + 32,
by one at each step. Also, there is a gap of one letter
+ 42, + 52, + 62, .....
between the last letter of the first term and first letter So, missing term = 91 + 72 = 91 + 49 = 140.
of the second term and a gap of two letters between 41. (c) The given sequence is a combination of two series:
the last letter of the second term and first letter of the I. 625, 125, 25, 5 and II. 5, 25, ?
third term. So, the first letter of the required term would The pattern in I is ÷ 5, while that in II is × 5.
be four steps ahead of the last letter of the third term. So, missing term = 25 × 5 = 125.
+3 +3 +3 42. (a) Each term at an even place in the series is the product
of its two adjacent terms. Thus, if the missing term be
23. (d) x, then we have:
C T F D W I G Z L J C 7 × x = 56
or x = 56 7 = 8
+3 +3 +3
+3 +3 +3 198 194 185 169 144
24. (b) Each of the numbers is doubled and 1, 2, 3, 4, 5, 6 is 43. (b)
2 2 2 2
added in turn, so 89 × 2 + 6 = 184. –2 –3 –4 –5
25. (c) 16 = 5 × 3 + 1, 51 = 16 × 3 + 3, 44. (a) The given series constitutes the squares of the
consecutive prime numbers : –
158 = 51 × 3 + 5
Next term = 158 × 3 + 7 = 481 22 , 32 , 52 , 7 2 , 132 , 17 2 , 19 2.......
26. (a) There are two atterate series-
2 5 10 17 26
+2 +2 +2 45. (c)
I series : 8 10 12 14 12
1 2 2
1 3 2
1 4 2
1 5 2
1
+2 +2 +2
II series : 13 15 17 19 37 50
27. (b) There are two alternate series. 2 2
6 1 7 1
1st series : 3, 4, 5, 6, 7 ........ and so on.
2nd series : 15, 16, 17,18, 19 ....... and so on. 46. (b) 24 60 120 210 336
28. (b) Ratios of two consecutive terms are 1, 1/2, 1/3, 1/4,
+36 +60 +90 +126
and 1/5 respectively.
29. (a) There are two alternate series +24 +30 +36
Series I- 6.25, 12.25, 20.25, 30.25 (sequence is +6, +8,
+10) +6 +6
Series II- 9, 16, 25, 36, (sequence is +7, +9, +11) 47. (c) 462 – 42 = 420
420 – 40 = 380
+6 +8 +10
380 – 38 = 342
6.25 9 12.25 16 20.25 25 30.25 36
342 – 36 = 306
+7 +9 +11 48. (c) The numbers are 112, 152, 192, ....., i.e., 112,
30. (c) Clearly the given series is 13 – 1, 23 – 1, 33 – 1, 43 – 1, (11 + 4 × 1)2, (11 + 4 × 2)2,.....
53 – 1, 63 – 1. Missing number = (11 + 4 × 3)2 = (23)2 = 529.
So, the missing number is 43 – 1 = 63.
y
o
u
A-22 Series

rs
m
–1 –1 56. (d)

a
49. (d) +3 +3 +3

h
b
J K I L H M

o
o
2 A 11 4 D 13 12 G 17 48 J 23

b
+1 +1

.w
o
2 3 4 5 6
2 4 7 11 16 22 ×2 ×3 ×4

rd
+2 +4 +6

p
–2 –2 –2 –2 –2
Z X V T R P 57. (b) baab/baab/baab

re
58. (b) cabbac/cabbac/cabbac.

s
+1 +2 +3 +4

s
.c
59. (c) The series is abc/aabc/aabbc/aabbcc/a.

o
50. (c) 3 F 6G 11 I 18 L 27 P 60. (c) The series is babb/bbab/bbba/bbbb.

m
Thus, in each sequence, ‘a’ moves one step forward
+3 +5 +7 +9
and ‘b’ takes its place and finally in the fourth
51. (d) The first letter of each term is moved two steps backward sequence, it is eliminated.
and the second letter is moved three steps forward to obtain 61. (a) a a b b / b b a a / a a b b / b b a a.
the corresponding letters of the next term. The number in
each term is 3 more than that in the preceding term. 62. (a) The series formed is :
52. (c) 1st letter: a a b a b c a b c d d c b a c b a b a a
in which the letters equi–distant from the beginning
2 2 2 2
Q S U W Y and end are the same.
Middle number: 63. (b) baab/baab/baab
1 1 2 2 3 3 4 4 64. (b) cabbac/cabbac/cabbac.
1 2 6 21 88
65. (d) a a b b c c d d / a a a b b b c c c d d d
3rd letter: 66. (b)
1 1 1 1 67. (a) The series is +5, +9, +13, +17 .... The difference in
F E D C B
successive nos. 9 – 5 = 13 – 9 = 17 – 13 = .... = 4. Hence,
53. (c) Ist number : 2
5
7
7
14 9 23 35 is wrong. It should be 37.
68. (a) The sum of the first two nos. is the third no. Hence, 27
11 13
34 47 is wrong. It should be 29.
Middle letter : 69. (b) The series is × 0.5 + 0.5, × 1 + 1, × 1.5 + 1.5 .... Hence, 12
is wrong. It should be 14.
1 1 1 1 1
Z Y X W V U 70. (d) The series is × 2 + 22, × 3 + 32, × 4 + 42, × 5 + 52 .....
3rd number : Hence, 32 is wrong. It should be 33.
71. (b) The series is + l3, + 23, + 33, + 43 ..... Hence, 229 is
2 2 2 2 2
5 7 9 11 13 15 wrong. It should be 227.
54. (a) Letters : 72. (c) The series is + 5, + 7, + 9, + 11, ...
2 2 2 2 73. (d) The series is 2, 1.5 alternately..
W U S Q O
74. (c) The series is an alternate series, having
Numbers : S1 = 2 5 14 41; × 3 – 1 in each term
144(122) 121 (112) 100(102) S2 = 3 8 23 69: × 3 – 1 in each term
75. (c) The differences are 1– 0 = 1 = 13 ; 9 – 1= 8
81 (9 ) 64 (8 ) 2 2 = 23; 36 – 9 = 27 = 33; 99 – 36 = 63 43,
55. (a) From the alternatives, it is clear that the series can be but 100 – 36 = 64 = 43; 225 – 100 = 125 = 53 ;
divided into groups of 4 terms each––each group 441– 225 = 216 = 63
comprising of two numbers followed by two letters, as 76. (a) The series is × 2.5,× 2 alternately.
shown below:
2 3 B __ / 6 __ F G / __ 5 D __ / 8 __ H I +5 +5 +5 +5 +5
77. (c) A F K P U Z
Clearly, the first number in each group represents the
position of the letter at the third place, in the English +5 5 +5 +5 +5
K P U Z E J
alphabet. Thus, the third missing term is the number
+5 +5 +5 +5 +5
corresponding to the postion of D in English alphabet U Z E J O T
i.e. 4, which is given in (a). Substituting other terms of
(a) into the series, we get:
1 1 1
2 3 B C / 6 7 F G / 4 5 D E / 8 9 H I 78. (c) M L K J
Observe that the second number in each group +2 +2 +2
represents the position of the letter at the fourth place, R T V X
in the English alphabet. S
+2
U
+2
W
+2 Y
y
o
u
Series A-23

rs
m
30

a
79. (a) 2 3 6 7 14 15
88. (b)

h
b
+1 ×2 +1 ×2 +1 ×2

o
89. (b) 4th term 23 = 5 × 4 + 3 = 23

o
80. (b) 6 × 3 = 18

b
3rd term 122 = 5 × 23 + 7 = 122

.w
18 × 6 = 108
108 × 18 = 1944

o
2nd term ? = 5 × 122 + 11 = 621

rd
Hence, 18 is the missing number in the sequence. 1st term 3120 = 5 × 621 + 15 =3120

p
81. (d) 251 – 107 = 144 = (12)2

re
381 – 125 = 256 = (16)2

s
0 5 60 615 6170

s
Hence, 16 is the missing number in the given question.

.c
90. (b)

o
+5 +55 +555 +5555

m
880
82. (b) 354 + 526 = =110
8 91. (b) ac b c / a c bc / a c bc / a c b c / a c b c
923 92. (c) r t x y
297 +631 = = 116 sx yz
8
r txy
83. (d) s x yz
93. (c) 1 × 2 = 2
2×3=6
6 × 4 = 24
84. (d) 24 × 5 = 120
120 × 6 = 720
94. (a) 156 + 350 = 506
85. (a) a b c b c a ca b / a b c bc a c ab. 506 + 550 = 1056
1056 + 750 = 1806
86. (a) B C F G 2, 3, 6, 7 95. (c) 3 + 3 = 6
6+3=9
JKNO 10, 11, 14, 15 9 + 6 = 15
15 + 9 = 24
RSV W 18, 19, 22, 23 24 + 15 = 39
Next sequence = 26, 27, 30, 31 = 26, 1, 4, 5 = Z A D E 39 + 24 = 63
(subtract the excess value by 26) 63 + 39 = 120
87. (c) –1 0 3 8 15 24
96. (b)
C I M, H N R, M S W, R X B
+1 +3 +5 +7 +9
+5
+5
+5
+5
+5
+5
+5
+5
+5
y
o
u
A-24 Coding-Decoding

rs
m
a
CHAPTER

h
b
CODING-DECODING 4

o
o
b
.w
o
rd
p
re
s
s
.c
In this segment of commonsense reasoning, secret messages or Example: If the word ‘FACT’ is coded as ‘IDFW’; then how will

o
words have to be decoded. They are coded as per a definite you code ‘DEEP’?

m
pattern/ rule which should be identified 1st. Then the same is Explanation: Here, you see that 2 letters are omitted in alphabetic
applied to decode another coded word. Under this segment you sequence. The following diagram gives you the more clear picture :
come across two types of coding letter coding and number coding.
Based on these two types of coding-decoding various types of F A C T
problems come your way. This chapter makes you familiar with +3 +3 +3 +3
every types of problems based on coding-decoding.
I D F W
TYPE I (CODING BY LETTER SHIFTING)
Clearly, ‘F’ (skip 2 letters) ‘I’
Pattern 1: Coding in forward sequence ‘A’ (skip 2 letters) ‘D’
Example: If ‘GOOD’ is coded as ‘HPPE’, then how will you code ‘C’ (skip 2 letters) ‘F’
‘BOLD’? ‘T’ (skip 2 letters) ‘W’
Explanation: Here,every letter of the word ‘Good’ shifts one place Similarly, ‘DEEP’ can be coded. Let us see :
in forward alphabetical sequence. Let us see: D E E P
G O O D +3 +3 +3 +3
+1 +1 +1 +1 G H H S
H P P E Code for ‘DEEP’ will be ‘GHHS’.
Similarly, every letter in the word ‘BOLD’ will move one place in TYPE II (CODING BY ANALOGY)
forward alphabetical sequence as given below:
Example: If ‘RPTFA’ stands for ‘BLADE’, how will you code
B O L D ‘BALE’.
+1 +1 +1 +1 Explanation: Here, ‘BLADE’ has been coded as ‘RPTFA’. You
C P M E will see that all the letters in the word ‘BALE’, which have to be
coded, are also there in the word ‘BLADE’. Hence, all that needs
Code for ‘BOLD’ will be ‘CPME’. to be done is to choose the relevant code letters from the code
Pattern 2: Coding in backward sequence. word ‘RPTFA’. Therefore, B becomes R, A becomes T, L becomes
Example: If ‘NAME’ is coded as ‘MZLD’, then how will code P, and E becomes A. Therefore, ‘BALE’ will be coded as ‘RTPA’.
SAME? Correct answer is ‘RPTA’.
Explanation: Here, every letter of the word ‘MZLD’ moves one
place in backward alphabet sequence. Let us see: TYPE III (CODING BY REVERSING LETTERS)
Example: If ‘TEMPERATURE’ is coded as ‘ERUTAREPMET’,
N A M E then how will you code ‘EDUCATION’ following the same scheme.
–1 –1 –1 –1 Explanation: Here, the word ‘TEMPERATURE’ has been reversed.
M Z L D Hence, the code for ‘education’ will be ‘NOITACUDE’.
Similarly, every letter of the word ‘SAME’ will move one place in TYPE IV (CODING IN FICTIONS LANGUAGE)
backward alphabet sequence. Let us see : In some cases of coding-decoding, fictions language is used to
S A M E code some words. In such questions, the codes for a group of
–1 –1 –1 –1 words is given. In such types of problems, codes for each word
can be found by eliminating the common words.
R Z L D Example: In a certain code language ‘over and above’ is written
Code for ‘SAME’ will be ‘RZLD’. as ‘da pa ta’ and ‘old and beautiful’ is written as ‘Sa na pa’. How
Pattern 3: Coding based on skipped sequence. is ‘over’ written in that code language?
y
o
u
Coding-Decoding A-25

rs
m
Explanation: As per the given condition
Explanation: Over and above = da Pa ta

a
h
3 4 5 6

b
Old and beautiful = Sa na Pa

o
R D N P

o
Clearly, ‘and’ is common in both and a common code is ‘Pa’.

b
.w
Code for ‘and’ must be ‘Pa’. 5 3 4 4 6

o
Code for ‘over’ = ‘da’ or ‘ta’. Now, N R D D P

rd
Code for above = ‘da’ or ‘ta’.

p
Code for 53446 = NRDDP.

re
Code for old = ‘Sa’ or ‘na’

s
Code for beautiful = ‘Sa’ or ‘na’

s
TYPE VI (MATHEMATICAL OPERATIONS WITH THE

.c
We can’t certainly say what will be exact code for ‘over’. But POSITION NUMBERS OF LETTERS)

o
it is sure that code for ‘over’ must be either ‘da’ or ‘ta’.

m
Example: In a certain code, if ‘TALE’ is written as 38, then how
TYPE V (CODING BASED ON NUMBERS) will you code ‘CAME’ using the same coding scheme?
Pattern 1: When numerical values are given to words. Explanation : Look at the numbered alphabet and write down the
Example: If in a certain language A is coded as 1, B is coded as 2. number corresponding to the letters of the word ‘TALE’.
C is coded as 3 and so on, then find the code for AEECD. T A L E
Explanation: As given the letters are coded as below: 20 1 12 5
A B C D E F G H I The fact that the code for ‘TALE’ is 38, gives you a clue that the
1 2 3 4 5 6 7 8 9 code is probably obtained by performing an arithmatical
operations of the numbers of each other. Let us see :
A E E C D 20 + 1 + 12 + 5 = 38
Now, 1 5 5 3 4 Thus, the code for ‘CAME’ is
Code for AEECD = 15534 C A M E
Pattern 2: When alphabetical code value are given for numbers. 3 + 1 + 13 + 5 = 22
Example: In a certain code 3 is coded as ‘R’, 4 is coded as ‘D’, 5 Code for ‘CAME’ = 22
is coded as ‘N’, 6 is coded as ‘P’, then find the code for ‘53446’.

1. In a certain code language BEAM is written as 5 % * K and 6. If in a certain code language ‘pen pencil’ is written as ‘$ £’,
COME is written as $ 7 K %. How is BOMB written in that ‘eraser sharpener’ is written as @ #’, and ‘pencil eraser’ is
code? written as ‘$ @’, then what is the code for ‘pen’?
(a) 5 % K5 (b) 5 7 K5 (a) £ (b) @
(c) $ 7 K $ (d) 5$ % 5 (c) $ (d) #
2. In a certain code PATHOLOGIST is written as 7. In a certain code language BORN is written as APQON and
PIUBQKSRHFN. How is CONTROVERSY written in that LACK is written as KBBLK. How will the word GRID be
code? written in that code language?
(a) SUOPDNXRQDU (b) SUOPDNZTSFW (a) FQHCD (b) FSHED
(c) QSMNBPXRQDU (d) QSMNBPZTSFW (c) HSJED (d) FSHCD
3. In a certain code language NATIONALISM is written as 8. In a certain code language STREAMLING is written as
OINTANMSAIL. How is DEPARTMENTS written in that CGTVUHOJMN. How will the word PERIODICAL be written
code? in that language?
(a) RADEPTSTMNE (b) RADPETSTMNE (a) PJSFQMNBJE (b) QKTGRMBDJE
(c) RADPESTMTNE (d) RADPETSTNME (c) QKTGRMCEKF (d) PJSFQMBDJE
4. In a certain code language OUTCOME is written as 9. If ‘green’ is called `white’, `white’ is called `yellow , ‘yellow’
OQWWEQOE. How is REFRACT written in that code? is called `red’, `red’ is called `orange’, then which of the
(a) RTGITCET (b) RTGTICET following represents the colour of sunflower?
(c) RTGITECT (c) RTGICTET (a) red (b) yellow
5. If B is coded as 8, F is coded as 6, Q is coded as 4, D is (c) brown (d) indigo
coded as 7, T is coded as 2, M is coded as 3, and K is coded 10. In a certain code BROUGHT is written as SGFVAQN. How
as 5, then what is the coded form of QKTBFM? is SUPREME written in that code?
(a) 452683 (b) 472683 (a) FNFSRTO (b) RTOSDLD
(c) 452783 (d) None of these (c) DLDSRTO (d) DLDSTVQ
y
o
u
A-26 Coding-Decoding

rs
m
11. In a certain code ‘CLOUD’ is written as ‘GTRKF’. How is 24. In a certain code language the word FUTILE is written as

a
SIGHT written in that code? HYVMNI. How will the word PENCIL be written in that

h
(a) WGJHV (b) UGHHT language?

b
o
(c) UHJFW (d) WFJGV (a) OIFRLT (b) OIFRLS

o
b
12. In a certain code CHAIR is written as # * • ÷ % and HIDE is (c) OLFRIT (d) None of these

.w
written as * ÷ + $. How is DEAR written in that code? 25. In a certain code language the word ‘NUMBER’ is written

o
(a) $ + • % 2 (b) + $ ÷ % as ‘UMHTEL’. How will the word ‘SECOND’ be written in

rd
(c) $ + % ÷ 4 (d) None of these that language?

p
re
13. In a certain code AROMATIC is written as BQPLBSJB. How (a) CTQDRB (b) GRQDRB

s
is BRAIN written in that code? (c) CTQFRB (d) GRQFRB

s
.c
(a) CQBJO (b) CSBJO 26. If the sentence “you must go early to catch the train” is

o
(c) CQBHO (d) CSBHO coded as “early catch train must to go the you”, what will

m
14. If ‘yellow’ means ‘green’, ‘green’ means ‘white’, white means be code for the sentence “morning exercise will help you to
‘red’, ‘red’ means ‘black’, ‘black’ means ‘blue’ and ‘blue’ keep fit”?
means ‘violet’, which of the following represents the colour (a) help to fit you exercise will keep morning
of human blood? (b) help to fit exercise you will keep morning
(a) black (b) violet (c) will help to fit you exercise keep morning
(c) red (d) None of these (d) will fit to exercise you help keep morning
15. In a certain code ‘FEAR’ is written as ‘ + × ÷ * ’ and ‘READ’ 27. In a code, CORNER is written as GSVRIV. How can
is written as ‘*× ÷ $ ’. How is ‘FADE’ written in that code? CENTRAL be written in that code?
(a) + ÷ $ × (b) × ÷ + $ (a) DFOUSBM (b) GIRXVEP
(c) $ ÷ + * (d) ÷ $ + × (c) GJRYVEP (d) GNFJKER
16. In a certain code BREAK is written as ASDBJ. How is SOLAR 28. If FRIEND is coded as HUMJTK, how is CANDLE written
written in that code? in that code?
(a) RPKBS (b) TPMBS (a) EDRIRL (b) DCQHQK
(c) RPKBQ (d) TPKBQ (c) ESJFME (d) FYOBOC
17. In a code language “1357” means “We are very happy”, 29. In a code STATION is denoted by URCRKMP then BRING
“2639” means “They are extremely lucky”, and “794” means is denoted in the same code by
“Happy and lucky”. Which digit in that code language (a) CSKLH (b) DSGLH
stands for “very”? (c) KSKPH (d) None of these
(a) 1 (b) 5 30. In a certain code ‘289’ means ‘Read from newspaper’, ‘276’
(c) 7 (d) Data inadequate means ‘tea from field’ and ‘85’ means ‘Wall newspaper’.
18. In a certain code language ‘CREATIVE’ is written as Which of the following number is used for ‘tea’?
‘BDSBFUJS’. How is ‘TRIANGLE’ written in that code? (a) 2 (b) 6
(a) BSHSFHKM (b) BHSSMHHF (c) Either 7 or 6 (d) Either 2 or 6
(c) BSSHFMKH (d) BHSSFKHM 31. If ‘black’ means ‘white’, ‘white’ means ‘red’, ‘red’ means
19. In a certain code ‘BROTHER’ is written as‘$%53#4%’ and ‘yellow’, ‘yellow’ means ‘blue’, ‘blue’ means ‘green’, ‘green’
‘DREAM’ is written as ‘9%47 ’. How is ‘THREAD’ written means ‘purple’ and ‘purple’ means ‘orange’ then what is
in that code? the colour of clean sky?
(a) #3%479 (b) 3#%479 (a) green (b) purple
(c) 3$%479 (d) 3#% 79 (c) blue (d) yellow
20. In a certain code language ‘OMNIPRESENT’ is written as 32. If ROSE is coded as 6821, CHAIR is coded as 73456 and
‘QJONPTSMDRD’. How is ‘CREDIBILITY’ written in that PREACH is coded as 961473, what will be the code for
code? SEARCH?
(a) JEFSDCXSHKH (b) JEFSDDXSHKH (a) 318826 (b) 214673
(c) DSFEJDDXSHKH (d) JEFSDXDSHKH (c) 214763 (d) 216473
21. If water is called air, air is called green, green is called brown, 33. If MOBILITY is coded as 46293927, then EXAMINATION
brown is called steel, steel is called red, red is called rain, is coded as
rain is called tree and tree is called road, what is the colour (a) 45038401854 (b) 56149512965
of human blood?
(c) 57159413955 (d) 67250623076
(a) Red (b) Water 34. In a certain code BANKER is written as LFSCBO. How will
(c) Rain (d) Tree
CONFER be written in that code?
22. In a certain code language ‘MOTHERS’ is written as
(a) GFSDPO (b) FGSDOP
‘OMVGGPU’. How is ‘BROUGHT’ written in that code?
(c) GFSEPO (d) FHSDPO
(a) CPRTIEV (b) DPQSIFV
35. In a certain code language, ‘3a, 2b, 7c’ means ‘Truth is
(c) DPRTIDV (d) DPQTIFV
Eternal’; ‘7c, 9a, 8b, 3a’ means ‘Enmity is not Eternal’ and
23. In a certain code ‘PENCIL’ is written as ‘RCTAMJ’ then in
‘9a, 4d, 2d, 8b’ means ‘Truth does not perish’. Which of the
that code ‘BROKEN’ is written as
following means ‘Enmity’ in that language?
(a) SPFLIM (b) SVFLIN
(a) 3a (b) 7c
(c) FVSMGL (d) None of these
(c) 8b (d) 9a
y
o
u
Coding-Decoding A-27

rs
m
36. If DELHI is coded as 73541 and CALCUTTA as 82589662, 51. If NOR is coded as 2-3-6, then how should REST be coded

a
how can CALICUT be coded? in the same code language ?

h
(a) 5279431 (b) 5978213 (a) 6-19-6-7 (b) 5-19-5-8

b
o
(c) 8251896 (d) 8543691 (c) 6-19-5-6 (d) 6-18-5-8

o
b
37. If REASON is coded as 5 and BELIEVED as 7, what is the 52. In a code language, 123 means ‘hot filtered coffee’, 356

.w
code number for GOVERNMENT? means ‘very hot day’, 589 means ‘day and night’. Which

o
(a) 6 (b) 8 numerical stands for ‘very’?

rd
(c) 9 (d) 10 (a) 5 (b) 6

p
re
38. If 341782 denotes MONKEY and 0596 denotes RAGS, then (c) 8 (d) 9

s
75195044 will denote 53. In a certain code WEAK is written as 5%9$ and WHEN is

s
.c
(a) KANGAROO (b) PALMANTT written as 5*%7. How HANK written in that code?

o
m
(c) HANGAMEE (d) KARNAGOO (a) *9$7 (b) 9*$7
39. In a certain code ‘BREAKTHROUGH’ is written as (c) $97* (d) *97$
‘EAOUHRGHKT’ How is DISTRIBUTION written in that 54. In a certain code MODE is written as #8%6 and DEAF is
code. written as %67$. How is FOAM written in that code?
(a) STTIBUDIONRI (b) STTIBUONRIDI (a) $87# (b) $#7%
(c) TISIBUONDIRI (d) RISTTIBUDION (c) #87% (d) $87%
40. If DANCE is coded as GXQZH then how will RIGHT be 55. In a certain code DESK is written as # $ 52, RIDE is written
coded ? as % 7#$. How is RISK written in that code?
(a) UFJEW (b) SGKFX (a) %725 (b) %752
(c) UFJWE (d) UFWJE (c) %7#2 (d) %7$#
41. If CAT is coded as 3120, what code number can be given to 56. In a certain code BEND is written as 5%7@ and DREAM is
NAVIN. written as @2%48. How is MADE written in that code?
(a) 14 1 22 9 14 (b) 4 9 2 7 4 6 5 4 (a) 84@% (b) 8@4%
(c) 7 3 9 5 7 6 14 (d) None of these (c) 85@% (d) 84%5
42. If ‘ ’ is coded as ‘ARGUE’ and is SOLVE, 57. If SMOOTH is coded as 135579, ROUGH as 97531 and
What is n is ? HARD as 9498, then SOFT will be coded as
(a) VAGUELY (b) VAGRANT (a) 1527 (b) 1347
(c) VAGUELE (d) VAGUER (c) 4998 (d) 8949
43. If NATION is coded as 467234 and EARN be coded as 1654 58. In a certain code OVER is written as $#%*. and VIST is
then ATTENTION should be coded as :- written as #+×–. How is SORE written in that code?
(a) 432769561 (b) 956143654 (a) ×$*% (b) %×$*
(c) 766412743 (d) 677147234 (c) ×*$% (d) × %*
44. If HONESTY is written as 5132468 and POVERTY as 7192068 59. If in a certain language LATE is coded as 8 & 4 $ and HIRE
how is HORSE written as in a certain code? is coded as 7*3$ then how will HAIL be coded in the same
(a) 50124 (b) 51042 language ?
(c) 51024 (d) 52014 (a) 7 & 8* (b) &7*8
45. If SEVEN is coded as 23136 and EIGHT as 34579. What will (c) 7*& 8 (d) 7&*8
be the code for NINE ? 60. If in a certain language WEAK is coded as 9%2$ and SKIT
(a) 6463 (b) 6364 is coded as #$7@, then how will WAIT be coded in the
(c) 6346 (d) 6436 same language ?
46. If RUSH is coded as 66 then how is GIRL coded as: (a) 9267 (b) 9276
(a) 75 (b) 64 (c) 92 @ 6 (d) None of these
(c) 47 (d) 46 61. In a certain code BROWN is written as 531 @% and MEAN
47. If LOVE is coded as 27 then how is COME coded as:- is written as 26©%. How is ROBE written in that code ?
(a) 38 (b) 18 (a) 3@16 (b) 3516
(c) 28 (d) 8 (c) 3156 (d) 3©16
48. If 841 = 3, 633 = 5, 425 = 7, then 217 = ? 62. If DFIN is coded as WURM, then HJMO can be coded as
(a) 6 (b) 7 (a) RPNO (b) SQNP
(c) 8 (d) 9 (c) SQNL (d) TRPO
49. If A = 1, PAT = 37 then TAP = ? 63. If RUMOUR can be written as QSJKPL, then how HERMIT
(a) 73 (b) 37 can be written?
(c) 36 (d) 38 (a) GEPKHR (b) GCOIDN
50. Certain letters are coded as : TODAY-457338 WROTE-10542. (c) GCPIDM (d) GCPIEN
DATE-7342 and DIRTH-79046. What does the code number 64. In a certain code language, GRAPE is written as 27354 and
‘5’ stand for ? FOUR is written as 1687. How is GROUP written in that
(a) D (b) R code?
(c) O (d) T (a) 27384 (b) 27684
(c) 27685 (d) 27658
y
o
u
A-28 Coding-Decoding

rs
m
65. WAYIN is written as TXVFX. How LBUK can be written in 69. In a certain code DEPUTATION is written as ONTADEPUTI.

a
that code? How is DERIVATION written in that code ?

h
(a) IYRH (b) KATJ (a) ONVADERITI (b) ONDEVARITI

b
o
(c) JZSI (d) NDWM (c) ONVAEDIRTI (d) ONVADEIRIT

o
b
66. In a certain code language, if the word PARTNER is coded 70. If MADRAS is coded as 517916 and TENANT is coded as

.w
as OZQSMDQ, then what is the code for the word 432121, how would you encode RMATSN?

o
SEGMENT? (a) 851353 (b) 951363

rd
(a) TFHNFOU (b) RDFLDMS (c) 951462 (d) 941562

p
re
(c) RDELDMS (d) RDFEDNS 71. Find the missing number from the given responses.

s
67. If DOCTOR is written as FQEVQT; how PATIENT can be

s
.c
written in that code?

o
260 132

m
(a) RVKGPV (b) RCKPGVV
(c) RCVKGPV (d) RVCKGVP ? 68
68. If ‘S’ is written as ‘H’ ‘R’ as ‘@’ ‘A’ as ‘ ’ ‘M’ as ‘#’ ,
20 36
‘T’ as ‘$’ and ‘E’ as ‘%’ then how is ‘MASTER’ written in
that code ?
(a) # H$%@ (b) #H $%@ (a) 12 (b) 10
(c) # $H%@ (d) # H%@$ (c) 9 (d) 8

ANSWER KEY
1 (b) 9 (a) 17 (d) 25 (b) 33 (b) 41 (a) 49 (b) 57 (a) 65 (a)
2 (a) 10 (c) 18 (d) 26 (b) 34 (a) 42 (a) 50 (c) 58 (a) 66 (b)
3 (b) 11 (a) 19 (b) 27 (b) 35 (c) 43 (d) 51 (c) 59 (d) 67 (c)
4 (a) 12 (d) 20 (b) 28 (a) 36 (c) 44 (b) 52 (b) 60 (d) 68 (a)
5 (d) 13 (c) 21 (c) 29 (d) 37 (c) 45 (a) 53 (d) 61 (c) 69 (a)
6 (a) 14 (a) 22 (d) 30 (c) 38 (a) 46 (d) 54 (a) 62 (c) 70 (c)
7 (b) 15 (a) 23 (d) 31 (a) 39 (a) 47 (b) 55 (b) 63 (b) 71 (d)
8 (b) 16 (c) 24 (d) 32 (b) 40 (a) 48 (d) 56 (a) 64 (c)
y
o
u
Coding-Decoding A-29

rs
m
a
h
b
o
o
b
.w
1. (b) Here, B 5, E %, A *, Similarly,

o
M K, C $,O 7 G R I D

rd
Therefore, BOMB 57K5 –1 +1 –1 +1

p
re
2. (a) F S H E –1 D

s
s
.c
I Group II Group

o
8. (b)

m
S T R E A M L I N G
Reverse
order
A E R T S G N I L M
+2 +2 +2 +2 +2 +1 +1 +1 +1 +1
C G T V U H O J M N
Hence, CONTROVERSY will be written as
SUOPDNXRQDU. Similarly,

I Group II Group

3. (b) P E R I O D I C A L
Reverse
order
O I R E P L A C I D
+2 +2 +2 +2 +2 +1 +1 +1 +1 +1
Q K T G R M B D J E
Similarly,
9. (a) The colour of sunflower is yellow and yellow is called
‘red’. Hence sunflower is red.

10. (c)

O U T C O M E
+2 +2 +3 +2 +2 +2 +0 S U P R E M E
4. (a) Reverse
Coded as O Q W W E Q O E order
E M E R P
U S
R E F R A C T –1 –1 –1 +1 –1 –1 –1
+2 +2 +3 +2 +2 +2 +0 D L D S O T R
Coded as Reverse
R T G I T C E T D L D S R T O order
5. (d) Q K T B F M = 4 5 2 8 6 3 11. (a) Here, each letter of the word CLOUD is written as three
6. (a) Pen pencil = $£ ...(i) letters forward and one letter backward alternately.
Eraser sharpner = @# ...(ii) Following this CLOUD becomes FKRTG. After that,
Pencil eraser = $@ ...(iii) reverse the order of the result obtained in the previous
From (i) and (iii), the code for ‘pencil’ is $. operation. Thus, FKRTG becomes GTRKF.
Hence, from (i), the code for ‘pen’ is £. Similarly, SIGHT will change its form as follows:
7. (b) B O R N SIGHT VHJGW WGJHV
–1 +1 –1 +1 12. (d)
A P Q O –1 N Code: # * • % + $
Letter: C H A I R D E
L A C K
Therefore, code for DEAR = + $ • %
–1 +1 –1 +1
13. (c) A R O M A T I C
K B B L –1 K +1 –1 +1 –I +1 –1 +1 –1
B Q P L B S J B
y
o
u
A-30 Coding-Decoding

rs
m
Similarly, B R A I N Reverse

a
+1 –1 +1 –1 +1 order

h
C Q B H O

b
O R B K N N

o
14. (a) The colour of human blood is red. Here red means

o
+4 –2 +4 –2 +4 –2

b
black. Therefore, black is our answer.

.w
15. (a) It is clear that F +, A , D $ and E × S P F I R L

o
FADE + $×

rd
24. (d)
16. (c) B R E A K

p
re
–1 +1 –1 +1 –1

s
s
A S D B J

.c
o
Similarly,

m
S O L A R
–1 +1 –1 +1 –1
R P K B Q
17. (d) C R E A T I V E 25. (b)
18. (d) C R E A T I V E
When the letters in both the halves are reversed, we
get
A E R C E V I T
+1 –1 +1 –1 +1 –1 +1 –1
B D S B F U J S
Next, the letters have been written as one place forward Similary, SECOND DSNEOC GRQDRB
and one place backward alternately. 26. (b) you must go early to catch the train
1 2 3 4 5 6 7 8
Similarly, TRIANGLE is coded as follows:
earlycatch train must to go the you
T R I A N G L E
4 6 8 2 5 3 7 1
A I R T E L G N
Similarly,
+1 –1 +1 –1 +1 –1 +1 –1
morning exercise will help you to keep fit
B H S S F K H M 1 2 3 4 5 6 7 8
Hence, code for TRIANGLE is BHSSFKHM 4 6 8 2 5 3 7 1
19. (b) help to fit exerciseyou will keep morning

27. (b)
Therefore, code for THREAD 3#% 479
20. (b) Similarly,

F R I E N D
28. (a) +2 +3 +4 +5 +6 +7
Coded as: H U M J T K
Hence code for CREDIBILITY is JEFS DDXSHKM Similarly,
21. (c) The colour of human blood is red. But here red is called
rain.
22. (d) M O T H E R S
+2 –2 +2 –1 +2 –2 +2
O M V G G P U 29. (d) As
Similarly, BROUGHT be coded as follows:
B R O U G H T –2
+2 –2 +2 –1 +2 –2 +2 –2
D P Q T I F V –2
23. (d) P E N C I L S T A T I O N U R C R K M P
+2
+2
+2
+2

Similarly, B R O K E N
y
o
u
Coding-Decoding A-31

rs
m
Similarly 37. (c) Code for the given word = (Number of letters in the

a
–2 word) –1.

h
So, code for GOVERNMENT = 10 – 1= 9.

b
–2

o
B R I N G D P K L I 38. (a) The number represent letters and to find the answer,

o
+2

b
select the respective letters.

.w
+2 3 4 1 7 8 2 0592 letters

o
+2 MO N K E Y RAGS code

rd
30. (c) ‘289’ means ‘Read from newspaper’ ..........(i) So,

p
re
‘276’ means ‘tea from field’ ..........(ii) 7 51 95 04 4 code

s
On comparing (i) and (ii), 2 is used for, ‘from’ KAN GA R O O answer letters

s
.c
From (ii) for tea the number is either 6 or 7. 39. (a)

o
m
31. (a) The colour of clean sky is blue and blue means green. BR EAKT HROU G H E AOU HR BR GHKT
1 2 3 4 5 6 7 8 9 10 11 12 3 4 9 10 7 8 1 2 1112 5 6
Hence, the colour of clean sky is green.
D I S T R I B U T I ON S T T I BU D I O N R I
32. (b) The alphabets are coded as shown: 1 2 3 4 5 6 7 8 9 10 1112 3 4 9 10 7 8 1 2 1112 5 6
R O S E C H A I P
D A N C E G X Q Z H
6 8 2 1 7 3 4 5 9 40. (a)
+3
So, in SEARCH, S is coded as 2, E as 1, A as 4 , R as 6,
–3
C as 7, H as 3. Thus, the code for SEARCH is 214673.
+3
33. (b) Let A = 1, B = 2, C = 3, ....., X = 24, Y = 25, Z = 26.
–3
Then, M = 13 = 1 + 3 = 4; O = 15 = 1 + 5 = 6
+3
L = 12 = 1 + 2 = 3; T = 20 = 2 + 0 = 2;
Y = 25 = 2 + 5 = 7. Similarly
So, MOBILITY = 46293927. RI GHT UFJE W
Similarly, EXAMINATION = 56149512965 41. (a)
34. (a) Here, the coding has been done in two steps. In the
C A T Similarly N A V I N 14 1 22 9 14
first step, the letters of the words are split into two
groups having equal number of letters, i.e.
B A N K E R
After that the position of the groups are interchanged. 3 1 20 14 1 22
9 14
i.e.
42. (b)
K E R B A N
And in the second step, each letter is moved one step 43. (d) N A T I O N
forward.
4
K E R B A N 6 7 2
3 4

L F S C B O E A R N
Thus the code for BANKAR is LFSCBO. Similarly, the
code of CONFER can be obtained as follows:
1 6 5 4
C O N F E R
A T T E N T I O N
F E R C O N

G F S D P O Coded word
6 7 7 1 4 72 3 4
35. (c) In the second and third statements, the common code is
‘9a’ and the common word is ‘not’. So, ‘9a’ means ‘not’. In 44. (b) H O N E S T Y
the first and second statements, the common codes are
‘7c’ and ‘3a’ and the common words are ‘is’ and ‘Eternal’’.
So, in the second statement, ‘8b’ means ‘Enmity’. 5 1 3 2 4 6 8
36. (c) The alphabets are coded as follows:
P O V E R T Y
D E L H I C A U T
7 3 5 4 1 8 2 9 6
So, in CALICUT, C is coded as 8, A as 2, L as 5, I as
1, U as 9 and T as 6. Thus, the code for CALICUT is 7 1 9 2 0 6 8
8251896. Therefore
y
o
u
A-32 Coding-Decoding

rs
m
H O R S E 50. (c) The manner of coding is

a
TODAY WROTE DATE DIRTH

h
45738 10542 7342 79046

b
o
The number ‘5’ is common in words TODAY and

o
b
5 1 0 4 2 WROTE, and so is the letter ‘O’. Letter ‘T’ cannot be

.w
the answer because it is common in all the four words

o
45. (a) S E V E N 23136 and so is its number code ‘4’.

rd
51. (c) Difference between alphabetical positions of

p
EI GHT 34579

re
Hence N and O = 1 = difference between 2 and 3

s
Difference between alphabetical positions of

s
.c
NIN E 6463 O and R = 3 = Diff. between 3 and 6.

o
m
Similarly, for REST,
46. (d) R U S H difference between R and E = 13,
difference between E and S = 14
and difference between S and T = 1
Here, only option (c) follows above condition
18 + 21+ 19 + 8 = 66
G I R L 52. (b) 1 2 3 hot filtered coffee
3 5 6 very hot day

5 8 9 day and night


7 + 9 + 18 +12 = 46
47. (b) L O V E Clearly, ‘6’ stands for ‘very’.
53. (d) W E A K W H E N

12 + 15 + 22 + 5 = 54 5 % 9 $ 5 * % 7
54 Therefore,
27 H A N K
2
Similarly,
C O M E * 9 7 $
54. (a) As,
M O D E and D E A F
3 + 15 + 13 + 5 = 36
36 # 8 % 6 % 6 7 $
18
2 Similarly,
8 F O A M
48. (d) 2 1 3
4
6 $ 8 7 #
3 5
3 55. (b) R %, I 7, S 5, K 2.
56. (a) BEND 5%7@
4
5 7 and DREAM @2%48
2 MADE 84@%
2 57. (a) SMOOTH 135579
2 7 9
1 ROUGH 97531
49. (b) A = 1 HARD 9498 O = 5
P A T Clearly, only the first option i.e. 1527 contains 5. Hence,
the correct code for SOFT in the code would be 1527.
O V E R V I S T
58. (a) $ # % * # + –
16 + 1 + 20 = 37
T A P From above table, SORE is coded as :
S O R E
$ * %
20 + 1 + 16 = 37
y
o
u
Coding-Decoding A-33

rs
m
59. (d) As L 8 and H 7 Similarly, L B U K

a
h
A & I * 3 3 3 3

b
o
T 4 R 3 I Y R H

o
b
.w
E $ E $
P A R T N E R

o
Similarly, 66. (b) As,

rd
1 1 1 1 1 1 1

p
H 7 A &

re
I * L 8 O Z Q S M D Q

s
s
Similarly,

.c
60. (d) As W 9 and S #

o
S E G M E N T

m
E % K $
1 1 1 1 1 1 1
A 2 I 7
R D F L D M S
K $ T @
67. (c) As, D O C T O R
Thus, W 9 2 2 2 2 2 2
A 2 F Q E V Q T
I 7
Similarly,
T @
P A T I E N T
61. (c) B R O W N and M E A N
2 2 2 2 2 2 2
5 3 1 @ % 2 6 © % R C V K G P V
R O B E
68. (a) Letter S R A M T E
3 1 5 6
62. (c) code H @ # $ %
MASTER = # $ % @

Respective place value of each letter from startig has 69. (a)
been coded as the respective place value of letter from
the last. Code
63. (b) Similarly,

Coded
70. (c) M A D R A S T E N T
64. (c) GRAPE = 2 7 3 5 4
FOUR = 1 6 8 7 coded as 5 1 7 9 1 6 4 3 2 1
So, G = 2, R = 7, A = 3, P = 5, E = 4, F = 1, O = 6, U = 8, Similarly,
R= 7 R M A T S N
GROUP = 2 7 6 8 5
65. (a) As W A Y I N Codedas 9 5 1 4 6 2
3 3 3 3 3
71. 8
T X V F K
y
o
u
A-34 Blood Relation

rs
m
a
CHAPTER

h
b
BLOOD RELATION 5

o
o
b
.w
o
rd
p
re
s
s
.c
MEANING OF BLOOD RELATION example also the sentence “the only daughter of the parents

o
m
Blood relation does mean biological relation. Remember a wife in laws of the husband of ‘Vandana’ has been given in the
and husband are met biologically related but they are biological form of indirect relation. Below are given some indirect
parents of their own children. Similarly, brother, sister, paternal relation in the form of a list. Examinees are required to learn
grandfather, paternal grandmother maternal grandfather, maternal them by heart. If are keeps this list in one’s mind, he/she will
grandmother, grandson, granddaughter, niece, cousin etc. are find it very easy to solve problems based on blood relations.
our blood relatives. 1. Son of father or mother : Brother
2. Daughter of father or mother : Sister
TYPES OF BLOOD RELATIONS
3. Brother of father : Uncle
There are mainly two types of blood relatives:
(i) Blood relation from paternal side 4. Brother of mother : Maternal uncle
(ii) Blood relation from maternal side 5. Sister of father : Aunt
Now, we will discuss both kind of relations one-by one. 6. Sister of mother : Aunt
(i) Blood relation from paternal side : This type of blood 7. Father of father : Grandfather
relation can be further subdivided into three types:
(a) Past generations of father : Great grandfather, great 8. Father of father's father : Great grand father
grandmother, grandfather, grandmother etc. 9. Father of grandfather : Great grandfather
(b) Parallel generations of father: Uncles (Brothers of father). 10. Mother of father : Grandmother
aunts (sisters of father) etc. 11. Mother of father's mother : Great grandmother
(c) Future generations of father: Sons, daughters, grandsons,
12. Mother of grandmother : Great grandmother
granddaughters etc.
(ii) Blood relation from maternal side: This type of blood 13. Father of mother : Maternal grandfather
relations can also be subdivided into three types: 14. Father of mother's father : Great maternal grand
(a) Past generations of mother: Maternal great grandfather, father
maternal great grandmother, maternal grandfather, maternal 15. Father of maternal : Great maternal
grandmother etc. grandfather grandfather
(b) Parallel generations of mother: Maternal uncles, maternal
16. Mother of mother : Maternal grandmother
aunts etc.
(c) Future generations of mother: Sons, daughters, 17. Mother of mother, mother : Great maternal
grandsons, granddaughters etc. grandmother
In the examinations, the questions are given in complicated 18. Mother of maternal : Great maternal
way. In other words, in the given questions, the easy grandmother grandmother
relationship takes the complicated form and examinees are 19. Wife of father : Mother
expected to solve this complication in order to find out the
20. Husband of mother : Father
correct answer. How does an examinee get aid of this
complication? For this, an examinee sees the given data in 21. Wife of Grandfather : Grandmother
the question with a serious eye; then tries to establish 22. Husband of Grandmother : Grandfather
relation among elements of given data on the basis of certain 23. Wife of son : Daughter-in-law
logic and finally finds out the required answer. In fact 24. Husband of daughter : Son-in-law
complications in the asked question occur because of the
given indirect relation. It does mean questions are in the 25. Brother of Husband : Brother-in-law
form of indirect relation & one has to convert this indirect 26. Brother of wife : Brother-in-law
relation into direct relation. For example “only son of my 27. Sister of Husband : Sister-in-law
father” does mean ‘me’ (myself). Here in place of ‘me’ indirect 28. Sister of wife : Sister-in-law
relation has been given in form of “only son of my father”.
29. Son of brother : Nephew
Similarly, “the only daughter of the parents in laws of the
husband of Vandana” does mean ‘Vandana’ herself. In this 30. Daughter of brother : Niece
y
o
u
Blood Relation A-35

rs
m
31. Wife of brother : Sister-in-law is the only daughter of mother of my brother’s sister.” How

a
32. Husband of sister : Brother-in-law is Mr. Sharma related to the lady in the photograph?

h
33. Son of sister : Nephew

b
(a) Cousin (b) Sister

o
34. Daughter of sister : Niece (c) Aunt (d) Daughter in law

o
b
35. Wife of uncle : Aunt Ans. Here we have to find relationship between Mr. Sharma &

.w
36. Wife of maternal uncle : Aunt the lady in the photograph.

o
37. Son/daughter of uncle/Aunt : Cousin

rd
Mother of my brother’s sister does mean my (Mr. Sharma’s)
38. Son/daughter of maternal : Cousin

p
mother. Only daughter of Mr. Sharma’s mother does mean

re
uncle/maternal aunt
“sister of Mr. Sharma”. Hence option (b) is the correct

s
39. Son/daughter of sister : Cousin

s
answer.

.c
of Father

o
m
40. Son/daughter of sister : Cousin (2) Blood relation based on family tree
of Mother Sample Q: Q is the brother of C and C is the sister of Q. R and D
41. Only son of grandfather : Father are brother and sister. R is the son of A while A & C are wife
42. Only daughter of maternal : Mother and husband. How is Q related with D.
grandfather Ans. For such type of question a family tree is made in which
43. Daughter of grandfather : Aunt some symbols are used as below:
44. Sons of grandfather other : Uncle ‘ ’ is used for husband & wife.
than father ‘___’ is used for brother & sister
45. Son of maternalgrandfather : Maternal Uncle. ‘ | ’ is used for parents (father or mother). Parents are put on
/maternal grand mother
top while children are put at the bottom.
46. Only daughter in law of : Mother
‘–’ or minus sign is used for female
grandfather/ grandmother
‘+’ or plus sign is used for male.
47. Daughters in law of : Aunt other than mother
grandfather/ grandmother Now adopting and using the above given symbols we can
48. Daughters-in-law of : Aunt maternal make a family tree and solve the given problem, let us see
maternal grandfather/ grandmother the family tree for sample question:
49. Neither brother nor sister : Self Family + tree :

SOME IMPORTANT INFORMATION ABOUT BLOOD – +


A+ C Q
RELATION
A. Without the information of gender, no relationship can be
established between two people. For example, If given that
R is the child of P & Q, then we can only say that P & Q are
the parents of R. But we can not find out:
(i) R is the son of P & Q or R is the daughter of P & Q. R+ D–
(ii) Who is mother of R and who is father of R.
But if we have given that P is a male, Q is a female and R is As per the question Q is the brother of c and c is the sister of Q.
male, then we can easily say that R is the son of P and Q. Hence relation between C & Q has been presented as C — Q

Further we can also say that P is father of R and Q is mother
where ‘–’ sign above C makes it clear that C is a female and ‘+’
of R.
B. Gender can not be decided on the basis of name. For example sign above ‘Q’ makes it clear that Q is a male. Similarly for R and
+
in Sikh community the names like Manjit, Sukhvinder etc. D. The presentation R—D has been made. Further
are the names of both male and female. Similarly, in the
according to the question.
Hindu Community ‘Suman’ is the name of both male and
female. A and C are having a husband and wife relationship and hence
Remember: Solution Tips
(a) While solving blood relation based question, first of all this has been presented as A C . As it is already given
find out that two persons between whom a relationship has that C is the sister of Q and A and C are wife and husband, this
to be established. becomes clear that A is the male member of the family and this is
(b) Next, try to find out middle relation the reason A has ‘+’ as its gender sign. Lastly, the vertical line
(c) Finally findout the relationship between two persons to be gives father and son relationship and has been presented as
identified for this purpose. A
Type of problems | . Now from this family tree it becomes clear that C is the
(1) General problems of blood relation R
(2) Blood relation based on family tree mother of R and D and as Q is the brother of C, then Q will
Now, we will discuss all the three types of problems one by one definitely be the maternal uncle of R & D. Hence we can say that
(1) General problem of blood relation Q is the maternal uncle of D and this is the required answer for
Sample Q: Pointing towards a photograph, Mr. Sharma said, “She our sample question.
y
o
u
A-36 Blood Relation

rs
m
a
h
b
o
o
b
.w
1. Anil, introducing a girl in a party, said, she is the wife of the 12. A is the brother of B. A is the brother of C. To find what is

o
rd
grandson of my mother. How is Anil related to the girl? the relation between B and C. What minimum information

p
(a) Father (b) Grandfather from the following is necessary?

re
(c) Husband (d) Father-in-law (i) Gender of C

s
s
2. A man said to a woman, “Your mother’s husband’s sister is (ii) Gender of B

.c
my aunt.” How is the woman related to the man ?

o
(a) Only (i) (b) Only (ii)

m
(a) Granddaughter (b) Daughter (c) Either (i) or (ii) (d) both (i) and (ii)
(c) Sister (d) Aunt 13. Mohan is son of Arun’s father’s sister. Prakash is son of
3. Introducing Rajesh, Neha said, “His brother’s father is the Reva, who is mother of Vikash and grandmother of Arun.
only son of my grand father”. How Neha is related to Rajesh? Pranab is father of Neela and grandfather of Mohan. Reva
(a) Sister (b) Daughter
is wife of Pranab.
(c) Mother (d) Niece
How is Vikash’s wife related to Neela?
4. Vinod is the brother of Bhaskar. Manohar is the sister of
(a) Sister (b) Niece
Vinod. Biswal is the brother of Preetam and Preetam is the
daughter of Bhaskar. Who is the uncle of Biswal? (c) Sister-in-law (d) Data inadaequate
(a) Bhaskar (b) Manohar 14. There is a family of 6 persons A, B, C, D, E and F. There are
(c) Vinod (d) Insufficient data two married couples in the family. The family members are
5. A man said to a woman, “Your brother’s only sister is my lawyer, teacher, salesman, engineer, accountant and doctor.
mother.” What is the relation of the woman with the maternal D, the salesman is married to the lady teacher. The doctor is
grandmother of that man? married to the lawyer. F, the accountant is the son of B and
(a) Mother (b) Sister brother of E. C, the lawyer is the daughter-in-law of A. E is
(c) Niece (d) Daughter the unmarried engineer. A is the grandmother of F. How is E
6. Pointing to a photograph, a man said, “ I have no brother or related to F?
sister but that man’s father is my father’s son.” Whose (a) Brother
photograph was it? (b) Sister
(a) His own (b) His son’s (c) Father
(c) His father’s (d) His nephew’s (d) Cannot be established (cannot be determined)
7. Pointing to a photograph, a lady tells Pramod, “I am the 15. Pointing to a girl, Abhishek said, “She is daughter of the
only daughter of this lady and her son is your maternal only child of my father.” How is Abhishek’s wife related to
uncle,” How is the speaker related to Pramod’s father? that girl?
(a) Sister-in-law (b) Wife (a) Daughter (b) Mother
(c) Neither (a) nor (b) (d) Aunt (c) Aunt (d) Sister
8. Introducing a man, a woman said, “His wife is the only 16. Introducing Sarita, Meena said, “She is the only daughter
daughter of my mother.” How is the woman related to that of my father’s only daughter.” How is Meena related to
man?
Sarita?
(a) Aunt (b) Wife
(a) Niece (b) Cousin
(c) Mother-in-law (d) Maternal Aunt
(c) Aunt (d) None of these
9. Deepak said to Nitin, “That boy playing with the football is
the younger of the two brothers of the daughter of my 17. A and B are married couple. X and Y are brothers. X is the
father’s wife.” How is the boy playing football related to brother of A. How is Y related to B?
Deepak? (a) Brother-in-law (b) Brother
(a) Son (b) Brother (c) Son-in-law (d) Cousin
(c) Causin (d) Nephew 18. A is the son of C; C and Q are sisters; Z is the mother of Q
10. A is the mother of B. C is the father of B and C has 3 children. and P is the son of Z. Which of the following statements is
On the basis of this information, find out which of the true?
following relations is correct : (a) P and A are cousins
(a) C has three daughters. (b) C has three sons. (b) P is the maternal uncle of A
(c) B is the son. (d) None of these. (c) Q is the maternal grandfather of A
11. A man pointing to a photograph says, “The lady in the (d) C and P are sisters
photograph is my nephew’s maternal grandmother.” How 19. Pointing out to a photograph, a man tells his friend, " she is
is the lady in the photograph related to the man’s sister the daughter of the only son of my father's wife. How is the
who has no other sister? girl related to the man in the photograph?
(a) Cousin (b) Sister-in-law (a) Daughter (b) Cousin
(c) Mother (d) Mother-in-law (c) Mother (d) Sister
y
o
u
Blood Relation A-37

rs
34. A, B, C, D, E, F and G are members of a family consisting of

m
20. Pointing to a boy, Meena says, “He is the son of my

a
grandfather’s only son.’’ How is the boy’s mother related 4 adults and 3 children, two of whom, F and G are girls. A

h
to Meena? and D are brothers and A is a doctor. E is an engineer married

b
o
(a) Mother (b) Aunt to one of the brothers and has two children. B is married to

o
D and G is their child. Who is C ?

b
(c) Sister (d) Data inadequate

.w
21. Pointing to a lady in the photograph, Shaloo said, “Her (a) G's brother (b) F's father

o
son’s father is the son-in-law of my mother”. How is Shaloo (c) E's father (d) A's son

rd
related to the lady? 35. Examine the following relationships among members of a

p
(a) Aunt (b) Sister family of six persons A, B, C, D, E and F.

re
(b) Cousin (d) Mother 1. The number of males equals that of females

s
s
22. Pointing to Kapil, Shilpa said, “His mother’s brother is the 2. A and E are sons of F.

.c
3. D is the mother of two, one boy and one girl

o
father of my son Ashish”. How is Kapil related to Shilpa's

m
Husband? 4. B is the son of A
(a) Sister-in-law (b) Nephew 5. There is only one married couple in the family at present
(c) Aunt (d) Niece Which one of the following inferences can be drawn from
23. A man said to lady, “Your mother’s husband’s sister is my the above?
aunt.” How is the lady related to the man? (a) A, B and C are all females
(a) Daughter (b) Grand daughter (b) A is the husband of D
(c) Mother (d) Sister (c) E and F are children of D
24. A family has a man, his wife, their four sons and their wives. (d) D is the grand daughter of F
The family of every son also have 3 sons and one daughter. 36. Introducing a man, a woman said, “He is the only son of my
Find out the total number of male members in the family. mother’s mother.” How is the woman related to the man?
(a) 4 (b) 8 (c) 12 (d) 17 (a) Mother (b) Cousin
25. A is B’s sister. C is B’s mother. D is C’s father. E is D’s (c) Niece (d) Aunt
mother. Then, how is A related to D? 37. Pointing to a man in the park, Naman said, “His son is my
(a) Grandmother (b) Grandfather son’s uncle.” How is the man related to Naman?
(c) Daughter (d) Granddaughter (a) Brother (b) Father
26. In a joint family, there are father, mother, 3 married sons and (c) Uncle (d) Grandfather
one unmarried daughter. Of the sons, two have 2 daughters 38. Following are some information about a family consisting
each and one has a son. How many females members are of 4 members P, Q, R and S.
there in the family? P is an adult male and has 2 children. Q is the son-in-law
(a) 2 (b) 3 (c) 6 (d) 9 of P. R is Q's brother–in–law. There is just one couple in
27. A, B and C are sisters. D is the brother of E and E is the the family.
daughter of B. How is A related to D? Who is the daughter of P?
(a) Sister (b) Cousin (a) P (b) R (c) Q (d) S
(c) Niece (d) Aunt 39. In a family of 5, P is the father of R. S is Q's son. S has
28. F is the brother of A. C is the daughter of A. K is the sister R as sister. Therefore, if U has P as brother, then the
of F. G is the brother of C. Who is the uncle of G? relationship between Q and U is as follows.
(a) A (b) C (c) F (d) K (a) Q is U's daughter
29. P is the brother of Q and R. S is the R’s mother. T is P’s (b) U is Q's wife
father. Which of the following statements cannot be defi- (c) Q is the sister-in-law of U
nitely true? (d) Q is U's brother-in-law
(a) T is Q’s father (b) S is P’s mother 40. Looking at a portrait of a man, Harsh said, "His mother
(c) T is S’s husband (d) S is T’s son is the wife of my father's son. Brothers and sisters I have
30. A party consisted of a man, his wife, his three sons and none. " At whose portrait was Harsh looking?
their wives and three children in each son’s family. How (a) His son (b) His cousin
many were there in the party? (c) His uncle (d) His nephew
(a) 24 (b) 22 (c) 13 (d) 17 41. Deepak said to Nitin, "That boy playing football is the
31. Nithya is Sam’s Sister. Mogan is Sam’s Father. Selvan is younger of the two brothers of the daughter of my father's
Rajan’s Son. Rajan is Mogan’s Brother. How is Nithya wife." How is the boy playing football related to Deepak?
related to Selvan?
(a) Daughter (b) Sister (a) Son (b) Brother
(c) Cousin (d) Wife (c) Cousin (d) Nephew
32. X' is the wife of 'Y' and 'Y' and Y' is the brother of 'Z' , 'Z' is 42. Pointing to Kapil, Shilpa said, "His mother's brother is the
the son of 'P' . How is 'P' related to 'X' ? father of my son Ashish. "How is Kapil related to Shilpa?
(a) Sister (b) Aunt (a) Sister-in-law (b) Nephew
(c) Brother (d) Data inadequate (c) Niece (d) Aunt
33. Suket has three daughters and each daughter has a brother. 43. Pointing to a man, a lady said "His mother is the only daughter
How many male members are there in the family ? of my mother". How is the lady related to the man?
(a) 4 (b) 2 (c) 3 (d) 1 (a) Mother (b) Daughter
(c) Sister (d) Aunt
y
o
u
A-38 Blood Relation

rs
m
ANSWER KEY

a
h
1 (d) 5 (d) 9 (b) 13 (d) 17 (a) 21 (b) 25 (d) 29 (d) 33 (b) 37 (b) 41 (b)

b
o
2 (c) 6 (b) 10 (d) 14 (d) 18 (b) 22 (b) 26 (d) 30 (d) 34 (d) 38 (d) 42 (b)

o
b
3 (a) 7 (b) 11 (c) 15 (b) 19 (a) 23 (d) 27 (d) 31 (c) 35 (b) 39 (c) 43 (a)

.w
4 (c) 8 (b) 12 (d) 16 (d) 20 (a) 24 (d) 28 (c) 32 (d) 36 (c) 40 (a)

o
rd
p
re
s
s
.c
o
m
1. (d) Clearly, the grandson of Anil’s mother is son of Anil 10. (d) A C
Mother
and wife of Anil’s son is daughter in-law of Anil. Thus,
Anil is the father-in-law of the girl. Father
B
2. (c) Woman’s Mother’s husband C has three children but we can’t say that he has
three daughters or three sons.
So, options (a) and (b) are incorrect.
Woman’s father
Also, we don’t know that B is a boy or girl.
Woman’s father’s sister Woman’s Aunt. So, option (c) is also incorrect.
Since, woman’s aunt is man’s aunt 11. (c) Clearly, the lady is the grandmother of man’s sister’s
woman is sister of man. son i.e., the mother of the mother of man’s sister’s son
3. (a) Father of Rajesh’s brother is the father of Rajesh. i.e., the mother of man’s sister.
Rajesh’s father is the only son of Neha’s grandfather. So, the lady is man’s mother.
Hence, Rajesh’s father is Neha’s father. So, Neha is 12. (d) Without knowing the sex of C, we can’t be determined
the sister of Rajesh. whether B is sister of C or B is brother of C. Similarly
4. (c) without knowing the sex of B we can’t be determined
Vinod
Sister Manohar whether C is sister of B or C is brother of B. Therefore,
Brother U ncl
e
both (i) and (ii) are necessary.
Bhaskar

Biswal

D a ug
13. (d) Pranab Reva
er

hter
Broth

( ) ( )
tam
Pree

Neela Prakash Vikash


Thus, Vinod will be uncle of Biswal. (–) ( )
5. (d) The only sister of the brother of the woman will be the
woman herself and she is the mother of that man. Thus, Mohan Arun
the woman is the daughter of the maternal ( ) ( )
grandmother of that man.
6. (b) Since the narrator has no brother, his father’s son is Data inadequate. As nothing is mentioned about vikash
wife in the question
he himself. So, the man who was talking is the father of
the man in the photograph, i.e. the man in the 14. (d) ' ' Male,'O' Female
photograph is his son. ' ' offspring, '=' couple
7. (b) Clearly, the speaker’s brother is Pramod’s maternal
uncle. So, the speaker is Pramod’s mother or his father’s ' ' Sibbling
wife.
8. (b) Clearly, only daughter of her mother is woman herself. Salesman D = A Lady
So, that woman is the wife of man.
9. (b) Father’s wife — Mother; Mother’s daughter — Sister; Doctor B = C Lawyer
Sister’s younger brother — His brother. So, the boy is
Deepak’s brother. Accountant F = E engineer
y
o
u
Blood Relation A-39

rs
m
As, sex of E is not clear, so E can be brother or sister of F.

a
Hence, relation between E and F can’t be established. 30. (d) a man his wife

h
15. (b) Girl is daughter of the only child of Abhishek’s father

b
o
or, Girl is daughter of Abhishek

o
b
Hence, girl is daughter of Abhishek’s wife.

.w
16. (d) Meena's father only daughter is Meena. Herself hence,
3 wives

o
3 sons
sarita is only daughter of Meena or Meena is the

rd
mother of Sarita.

p
re
17. (a) X and Y are brothers and

s
X is brother of A Y is brother of A.

s
.c
Y is brother-in-law of B.

o
3 childrens each (3 × 3)

m
18. (b) C and Q are sisters and A is the son of C. Hence, C is
the mother of A or Z is the mother Q. Total No. of people in the party
Hence, Z is the maternal grandmother of A. P is the =1+1+3+3+3×3
son of Z. Hence, P is the maternal uncle of A. = 8 + 9 = 17
19. (a) Father's wife means mother ; mother's only son means 31. (c) Nithya is Sam’s Sister and Mogan is Sam’s Father
himself and thus the girl is the daughter of the man. Nithya is Mogan’s Daughter.
20. (a) One’s grandfather’s only son one’ss father. And the Selvan is Rajan’s Son and Rajan is Mogan’s
son of one’s father One’ss brother or oneself. Hence, Brother
the mother of the boy is Meena’s mother. Selvan is Mogan’s Nephew.
21. (b) Lady’s son’s father is lady’s husband. So, the lady’s So, Nithya is Selvan’s Cousin.
husband is the son-in-law of Shaloo’s mother i.e., the 32. (d) The relationship chart, based on the given problem
lady is the daughter of Shaloo’s mother. Thus, Shaloo can be worked out as given below.
is the lady’s sister.
w
22. (b) Father of Shilpa’s son Shilpa’s husband. So, Kapil -la P
e r-in
is the son of Shilpa’s husband's sister,. Thus Kapil is oth
Shilpa’s Husband's Nephew. orM
-law

So
23. (d) Lady’s mother’s husband Lady’s father r-in

n
he
Lady’s father’s sister Lady’s Aunt. Fat
So, Lady’s aunt is man’s aunt and therefore lady is
man’s sister. X Y Z
24. (d) The make members in the family are:- Wife Brother
(i) The man himself The sex of P is not known
(ii) his four sons; and 33. (b) Suket has three daughters and each daughter has a
(iii) his (3 × 4) = 12 grandsons. brother.
Hence total numbers of male members
= 1 + 4 + 12 =17 Suket
25. (d) A is the sister of B and B is the son/daughter of C. So,
A is the daughter of C. Also, D is the father of C.
Thus, A is the granddaughter of D. D1 = B = D2 = D3
26. (d) The female members are:- (F) (M) (F) (F)
(i) mother Hence, there are 2 male members in a family.
(ii) Wives of 3 married sons 34. (d) ‘ ’ brothers, ‘=’ couple, ‘ ’ offspring, ‘ ’
(iii) unmarried daughter male, ‘ ’ female, ‘X’ unknown
(iv) 2 daughter of each of two sons
Total No of females E A D B
=1+3+1+2×2=9
27. (d) E is the daughter of B & B is the brother of E. So, D is
the son of B. Also, A is the sister of B. Thus, A is D’s G
Aunt. X X
28. (c) G is the brother of C and C is the daughter of A. So, G
Clearly, C and F are the remaining members to be adjusted
is son of A. Also, F is the brother of A. So, F is the
in place of two x. since, there are 3 children out of which two
uncle of G.
are girls, i.e. G and F, so clearly the third children C is a boy.
29. (d) P, Q, R are children of same parent. So, S, who is R’s
So C is the son of E and A.
mother and T, who is P’s father will be mother and
35. (b) ‘ ’ Female, ‘ ’ Male, ‘ ’ Couple, ‘ ’
father of all three. However, it is not mentioned weather
Offspring
Q is male or female. So, (d) cannot be definitely true.
y
o
u
A-40 Blood Relation

rs
m
38. (d) S is P's daughter, Q's wife and R's sister.
F

a
39. (c) P and Q are husband and wife, with R and S as their

h
children. P is U's brother, therefore Q is U's brother's

b
o
wife, that is, sister-in-law.

o
D A E

b
40. (a) Since Harsh has no brother or sister, so he is his father's

.w
only son.

o
So, wife of Harsh's father's son––Harsh's wife.

rd
C B
Thus, Harsh's wife is the man's mother or the man is

p
Since, there is only 1 married couple, so D must be married

re
Harsh's son.
to A, as D is the mother of two and B is the son of A. Also,

s
41. (b) Father's wife ––Mother; Mother's daughter __ Sister;

s
as number of males and females are equal, so F must be a

.c
Deepak's sister's younger brother––Deepak's younger

o
female. brother.

m
36. (c) Mother So, the boy is Deepak's brother.
42. (b) Father of Shilpa's son ––Shilpa's husband
So, Kapil is the son of sister of Shilpa's husband. Thus,
Mother Son (+) Kapil is Shilpa's nephew.
(–)
Niece 43. (a)
Women (–)

37. (b) Man (+)


Father
Son (+) Naman

Uncle
Son (+)
y
o
u
Direction & Distance A-41

rs
m
a
CHAPTER

h
DIRECTION &

b
6

o
o
b
.w
DISTANCE

o
rd
p
re
s
s
Concept of Direction

.c
o
In our day to day life, we make our concept of direction after Right turn

m
Left turn
seeing the position of sun. In fact, this is a truth that sun rises in

Right turn

Left turn
Right turn

Left turn
the East and goes down in the west. Thus when we stand facing
sunrise, then our front is called East while our back is called
West. At this position our left hand is in the Northward and the
right hand is in the Southward. Let us see the following direction Right turn Left turn
(i) (ii)
map that will make your concept more clear:
Direction Map:

Right turn Left turn


North
North-West North-East
(iii) (iv)

Important Point Regarding Direction


West East
(1) If our face is towards North, than after left than our face will
be it towards West while after right turn it will be towards
East.
South-West South-East (2) If our face is towards South, then after left turn our face will
South be towards East and after right turn it will be towards West.
(3) If our face is towards East, then after left turn our face will
be to North and after right turn it will be towards South.
Note: (4) If our face is towards West, then after left turn our face will
On paper North is always on top be while South is always in be towards South and after right turn it will be towards
bottom. North.
(5) If our face is towards North-West, then after left turn our
Concept of Degree face will be towards South-West and after right turn it will
Let us see the following picture: be towards North-East.
(6) If our face is towards South-West, then after left turn our
face will be towards South-East and after right turn it will be
towards North-West.
(7) If our face is towards South-East, then after left turn our
face will be towards North-East and after right turn it will be
towards South-West.
(8) If our face is towards North-East, then after left turn our
face will be towards North-East and after right-turn it will be
towards South-East. A
Concept of Minimum Distance
Minimum distance between initial and last point
h2 = b2 + p2, where
h = Hypotenuse
Concept of Turn
b = Base P h
Left turn = clockwise turn
P = Perpendicular
Right turn = Anti-clockwise turn. Remember this important rule is
Let us understand it through pictorial representation: known as ‘Pythogoras Theorem’

B b C
y
o
u
A-42 Direction & Distance

rs
m
EXAMPLE 1. Raman walked 2 km West from his office and (a) 6 km West (b) 7 km East

a
(c) 8 km North (d) 5 km North-East

h
then turned South covering 4 km. Finally, he waked 3 km towards
Sol. (d) It is clear, Rashmi moves from A 10 km Northwards

b
East and again move 1 km West. How far is Raman from his

o
upto B, then moves 6 km Southwards upto C, then

o
initial position?

b
turns towards East and walks 3 km upto D.

.w
(a) 4 km (b) 8 km Then, AC = (AB – BC) = 10 – 6 = 4 km

o
(c) 10 km (d) 7 km CD = 3km.

rd
Sol. (a) Raman starts from his office A, moves 2 km West upto

p
re
B, then 4 km to the South upto C, 3 km East upto D and B

s
finally 1 km West upto E, Thus his distance from the

s
6 km

.c
initial position A = AE = BC = 4 km.

o
3 km
Hence option (a) is the correct answer.

m
D
C
B 2 km
A

10 km

1 km
D
C 2 km E
A
EXAMPLE 2. Rashmi walks 10 km towards North. She walks
6 km towards South then. From here she moves 3 km towards Rashmi’s distance from starting point A
East. How far and in which direction is she with reference to her
starting point? =AD= AC 2 + CD 2 42 32 16 9 25 5km.
From figure, D is to the North-East of A, Hence (d) is the
correct option

1. Q travels towards East. M travels towards North. S and T saw that the shadow of his uncle was to his right side.
travel in opposite directions. T travels towards right of Q. Which direction was his uncle facing during their talk?
Which of the following is definitely true? (a) North (b) South
(a) M and S travel in the opposite directions. (c) East (d) Data inadequate
(b) S travels towards West. 6. A and B are standing at a distance of 20 km from each other
(c) T travels towards North. on a straight East-West road. A and B start walking
(d) M and S travel in the same direction. simultaneously, eastwards and westwards respectively, and
2. P, Q, R, S and T are sitting around a circular table. R is to the both cover a distance of 5 km. Then A turns to his left and
right of P and is second to the left of S. T is not between P walks 10 km. ‘B’ turns to his right and walks 10 km and at the
and S. Who is second to the left of R? same speed. Then both turn to their left and cover a distance
(a) S (b) T of 5 km at the same speed. What will be the distance between
(c) Q (d) Data inadequate them?
3. Of the five villages P, Q, R, S and T situated close to each (a) 10 km (b) 5 km
other, P is to west of Q, R is to the south of P, T is to the (c) 20 km (d) 25 km
north of Q, and S is to the east of T. Then, R is in which 7. Alok walked 30 metres towards east and took a right turn
direction with respect to S? and walked 40 metres. He again took a right turn and walked
(a) North-West (b) South-East 50 metres. Towards which direction is he from his starting
(c) South-West (d) Data Inadequate point?
4. M is to the East of D, F is to the South of D and K is to the (a) South (b) West
West of F. M is in which direction with respect to K? (c) South-West (d) South-East
(a) South-West (b) North-West 8. Ruchi's house is to the right of Vani's house at a distance of
(c) North-East (d) South-East 20 metres in the same row facing North. Shabana's house is
5. After 4 pm on a sunny day when Ramesh was returning in the North- East direction of Vani's house at a distance of
from his school, he saw his uncle coming in the opposite 25 metres. Determine that Ruchi's house is in which direction
direction. His uncle talked to him for some time. Ramesh with respect of Shabana's house?
y
o
u
Direction & Distance A-43

rs
m
(a) North-East (b) East walks 4 km while B turns left and walks 3 km. How far is each

a
(c) South (d) West from the starting point ?

h
9. Y is to the East of X, which is to the North of Z. If P is to the (a) 5 km (b) 4 km

b
o
South of Z, then P is in which direction with respect to Y? (c) 10 km (d) 8 km

o
b
(a) North (b) South 20. Anuj started walking positioning his back towards the sun.

.w
(c) South-East (d) None of these After sometime, he turned left, then turned right and then

o
10. One afternoon, Manisha and Madhuri were talking to each towards the left again. In which direction is he going now?

rd
other face to face in Bhopal on M.G. Road. If Manisha’s (a) North or South (b) East or West

p
re
shadow was exactly to the left of Madhuri, which direction (c) North or West (d) South or West

s
was Manisha facing? 21. From her home, Prerna wishes to go to school. From home,

s
.c
(a) North (b) South she goes towards North and then turns left and then turns

o
right, and finally she turns left and reaches school. In which

m
(c) East (d) Data inadequate
11. ‘X’ started walking straight towards South. He walked a direction her school is situated with respect to her home?
distance of 5 metres and then took a left turn and walked a (a) North-East (b) North-West
distance of 3 metres. Then he took a right turn and walked (c) South-East (d) South-West
a distance of 5 metres again. ‘X’ is facing which direction 22. One day, Ravi left home and cycled 10 km southwards,
now? turned right and cycled 5 km and turned right and cycled 10
(a) North-East (b) South km and turned left and cycled 10 km. How many kilometres
(c) North (d) South-West will he have to cycle to reach his home straight?
12. If A is to the south of B and C is to the east of B, in what (a) 10 km (b) 15 km
direction is A with respect to C? (c) 20 km (d) 25 km
(a) North-east (b) North- west 23. Rasik walks 20 m North. Then, he turns right and walks
(c) South-east (d) South-west 30 m. Then he turns right and walks 35 m. Then he turns left
13. One morning after sunrise, Gopal was facing a pole. The and walks 15 m. Then he again turns left and walks 15 m. In
shadow of the pole fell exactly to his right. Which direction which direction and how many metres away is he from his
original position?
was he facing?
(a) 15 metres West (b) 30 metres East
(a) South (b) East
(c) 30 metres West (d) 45 metres East
(c) West (d) Data inadequate
24. From his house, Lokesh went 15 km to the North. Then he
14. A boy rode his bicycle northwards, then turned left and
turned West and covered 10 km. Then, he turned South and
rode one km and again turned left and rode 2 km. He found
covered 5 km. Finally , turning to East, he covered 10 km. In
himself exactly one km west of his starting point. How far
which direction is he from his house?
did he ride northwards initially?
(a) East (b) West
(a) 1 km (b) 2 km
(c) North (d) South
(c) 3 km (d) 5 km
25. Kailash faces towards north. Turnings to his right, he walks
15. Ravi wants to go to the university which is opposite to 25 metres. He then turns to his left and walks 30 metres.
theatre. He starts from his home which is in the East and Next, he moves 25 metres to his right. He then turns to the
come to a crossing. The road to the left ends is a theatre, right again and walks 55 metres. Finally, he turns to the
straight ahead is the hospital. In which direction is the right and moves 40 metres. In which direction is he now
university? from his starting point ?
(a) North (b) South (a) South-West (b) South
(c) East (d) West (c) North-West (d) South-East
16. A rat runs 20' towards east and turns to right, runs 10' and 26. A clock is so placed that at 12 noon its minute hand points
turns to right, runs 9' and again turns to left, runs 5' and towards north-east. In which direction does its hour hand
then to left, runs 12' and finally turns to left and runs 6'. point at 1: 30 pm ?
Now, which direction is the rat facing? (a) North (b) South
(a) East (b) West (c) East (d) West
(c) North (d) South 27. One evening before sunset two friends Sumit and Mohit
17. If South-east becomes North, North-east becomes West were talking to each other face to face. If Mohit’s shadow
and so on, what will West become? was exactly to his right side, which direction was Sumit facing?
(a) North-east (b) North-west (a) North (b) South
(c) South-east (d) South-west (c) West (d) Data inadequate
18. P, Q, R and S are playing a game of carrom. P, R and S, Q are 28. Rohit walked 25 metres towards South. Then he turned to
partners. S is to the right of R who is facing west. Then, Q his left and walked 20 metres. He then turned to his left and
is facing walked 25 metres. He again turned to his right and walked
(a) North (b) South 15 metres. At what distance is he from the starting point
(c) East (d) West and in which direction?
19. A and B start walking, from a point, in opposite directions. (a) 35 metres East (b) 35 metres North
A covers 3 km and B covers 4 km. Then A turns right and (c) 40 metres East (d) 60 metres East
y
o
u
A-44 Direction & Distance

rs
m
29. One morning after sunrise, Reeta and Kavita were talking to 38. Sobha was facing East. She walked 20 metres. Turning left

a
each other face to face at Tilak Square. If Kavita’s shadow was she moved 15 metres and then turning right moved 25

h
b
exactly to the right to Reeta, which direction Kavita was facing? metres. Finally, she turned right and moved 15 metres more.

o
How far is she from her starting point?

o
(a) North (b) South

b
(c) East (d) Data inadequate (a) 25 metres (b) 35 metres

.w
30. I am facing east. I turn 100° in the clockwise direction and (c) 50 metres (d) 45 metres

o
rd
then 145° in the anticlockwise direction. Which direction 39. Jatin leaves his house and walks 12 km towards North. He

p
am I facing now? turns right and walks another 12 km. He turns right again,

re
walks 12 km more and turns left to walk 5 km. How far is he

s
(a) East (b) North-east

s
from his home and in which direction ?

.c
(c) North (d) South-west

o
31. A man is facing north-west. He turns 90° in the clockwise (a) 7 km east (b) 10 km east

m
direction, then 180° in the anticlockwise direction and then (c) 17 km east (d) 24 km east
another 90° in the same direction. Which direction is he 40. Deepak starts walking straight towards east. After walking
facing now? 75 metres, he turns to the left and walks 25 metres straight.
(a) South (b) South-west Again he turns to the left, walks a distance of 40 metres
(c) West (d) South-east straight, again he turns to the left and walks a distance of 25
32. A man is facing west. He runs 45° in the clockwise direction metres. How far is he from the starting point ?
(a) 25 metres (b) 50 metres
and then another 180° in the same direction and then 270° in
(c) 115 metres (d) 35 metres
the anticlockwise direction. Which direction is he facing now?
41. If a person is walking towards North, what direction should
(a) South (b) North-west
he follow so that he is walking towards West ?
(c) West (d) South-west
(a) right, right, left (b) left, left, right
33. Gan esh cycles towards South West a distance of
(c) left, right, left (d) left, left, left
8 m, then he moves towards East a distance of 20 m. From
42. A watch read 4.30. If the minute hand points East, in what
there he moves towards North East a distance of 8 m, then
direction will the hour hand point?
he moves towards west a distance of 6 m From there he
(a) North (b) North west
moves towards North-East a distance of 2m. Then he moves (c) South-east (d) North-east
towards west a distance of 4 m and then towards south 43. A person stood alone in a desert on a dark night and wanted
west 2 km and stop at that point. How far is he from the to reach his village which was situated 5 km east of the
starting point ? point where he was standing. He had no instruments to
(a) 12 m (b) 10 m find the direction but he located the polestar. The most
(c) 8 m (d) 6 m convenient way now to reach his village is to walk in the
34. From my house I worked 5 km towards North. I turned right (a) direction facing the polestar
and walked 3 km. Again I went one km to south How far am (b) direction opposite to the polestar
I from my house? (c) direction keeping the polestar to his left
(a) 7 km (b) 6 km (d) direction keeping the polestar to his right
(c) 4 km (d) 5 km 44. A person travels 12 km due North, then 15 km due East,
35. Ram left home and walked 5 km southward, turned right and after that 15 km due West and then 18 km due South. How
walked 2 km and turned right and walked 5 km and turned far is he from the starting point?
left and walked 5 km. How many km will he have to walk to (a) 6 km (b) 12 km
reach his home starting ? (c) 33 km (d) 60 km
(a) 5 (b) 7 45. Priya starts walking in the afternoon facing the Sun. After
(c) 17 (d) 15 some time, she turned to the right. Later again, she tunred
36. Going 60 m to the south of his house. Kiran turn left and to her left and again also left. At what direction is Priya
goes another 20 m, then turning to the North. moving now?
He goes 40 m and then starting walking to his house. In (a) East (b) West
which direction is his house from there? (c) North (d) South
(a) South-East (b) North 46. Asha drives 6 km towards West and turns to the right and
(c) East (d) North-West drives 3 km. Then, she turns again and drives towards right
37. Ram started walking towards East after 1 km. He turned hand and drives 6 km. How far is she from her starting
south and walked 5 km. Again he turned East and walked 2 point? In which direction would she be driving?
km. Finally he turned North and walked 9 km. How far is he (a) 6 km East (b) 3 km West
from the starting point? (c) 3 km East (d) 6 km North
(a) 7 km (b) 3 km 47. In the given figure, P is 300 km eastward of O and Q is 400
(c) 4 km (d) 5 km km north of O. R is exactly in the middle of Q and P. The
distance between Q and R is
y
o
u
Direction & Distance A-45

rs
m
North along a diagonal order to catch a rat. How much total

a
distance is covered by the cat?

h
(a) 10 (b) 14

b
o
(c) 38 (d) 48

o
Q

b
54. Two ladies and two men are playing bridge a card game and

.w
seated at North, East, South and West of a table. No lady is

o
facing East. Persons sitting opposite to each other are not

rd
400 km R
of the same gender. One man is facing South. Which

p
re
directions are the ladies facing?

s
(a) East and west (b) South and east

s
.c
East
O 300 km P (c) North and west (d) North and east

o
m
55 Consider the following statements :
(a) 250 km (b) 100 3 km There are six villages A, B, C, D, E and F.
(c) 500 km (d) 125 km F is 1 km to the west of D.
48. The houses of A and B face each other on a road going B is 1 km to the east of E.
north-south, A’s being on the western side. A comes out of A is 2 km to the north of E.
his house, turns left, travels 5 km, turns right, travels 5 km C is 1 km to the east of A.
D is 1 km to the south of A.
to the front of D’s house. B does exactly the same and Which three villages are in a line ?
reaches the front of C’s house. In this context, which one of (a) A, C, B (b) A, D, E
the following statements is correct ? (c) C, B, F (d) E, B, D
(a) C and D live on the same street. 56. If all the directions are rotated, i.e., if North is changed to
(b) C’s house faces south. West and East to North and so on, then what will come in
(c) The houses of C and D are less than 20 km apart. place of North-West ?
(d) None of the above (a) South-west (b) North-east
49. If M is in North-east of N and P in South-West of N then (i) (c) East-north (d) East-west
P is inthe South of N and (ii) N is between M and P. Out of 57. In a meeting, the map of a village was placed in such a
these two statements (i) and (ii) which is/are correct? manner that south-east becomes north, north-east becomes
(a) (i) and (ii) both are correct west and so on. What will south become?
(b) (i) and (ii) both are wrong (a) North (b) North-east
(c) only (i) is correct (c) North-west (d) West
(d) only (ii) is correct 58. A is 40 m south-west of B. C is 40 m south-east of B. Then,
50. Five persons A, B, C, D and E are standing in a row. B is C is in which direction of A?
between A and C and D is between C and E. If the distance (a) East (b) South
of C from B is equal to the distance of D from C, what is the (c) West (d) North
relation between the distances of A to B and B to E? 59. Gaurav walks 20 metres towards North. He then
(a) Both are equal turns left and walks 40 metres. He again turns left and
(b) A B is smaller than BE walks 20 metres. Further, he moves 20 metres after turning
(c) A B is larger than BE to the right. How far is he from his original position?
(d) There is no relation in AB and BE (a) 55 m (b) 60 m
51. The post office is in the East of the school while my house (c) 65 m (d) 50 m
is in the South of the school. The market is in the North of 60. My friend and I started walking simultaneously
the post office. If the distance of the market from the towards each other from two places 100 m apart. After
post-office is equal to the distance of my house from the walking 30 m, my friend turns left and goes 10 m, then he
school, in which direction is the market with respect to my turns right and goes 20 m and then turns right again and
house? comes back to the road on which he had started walking. If
(a) North (b) East we walk with the same speed, what is the distance between
(c) North-east (d) South-west us at this point of time?
52. A person stood alone in a desert on a dark night and wanted (a) 50 m (b) 60 m
to reach his village which was situated 5 km east of the (c) 40 m (d) 45 m
point where he was standing. He had no instruments to 61. A square field ABCD of side 90 m is so located
find the direction but he located the polestar. The most that its diagonal AC is from north to south and the corner B
convenient way now to reach his village is to walk in the is to the west of D. Rohan and Rahul start walking along the
(a) direction facing the polestar sides from B and C respectivelyin the clockwise and anti-
(b) direction opposite to the polestar clockwise directions with speeds of 8 km/hr and 10 km/hr.
(c) direction keeping the polestar to his left Where will they cross each other the second time ?
(d) direction keeping the polestar to his right (a) On AD at a distance of 30 m from A
53. The length and breadth of a room are 8 m and 6 m (b) On BC at a distance of 10 m from B
respectively. A cat runs along all the four walls and finally (c) On AD at a distance of 30 m from D
(d) On BC at a distance of 10 m from C
y
o
u
A-46 Direction & Distance

rs
m
62. If South East becomes North, then what will South West 67. An insect is walking in straight line. It covers a distance of

a
become? 15 cm per minute. It comes back 2.5 cm after every 15 cm.

h
(a) North (b) West How long will it take to cover a distance of 1 metre ?

b
o
(c) East (d) North West (a) 6.5 min (b) 8 min

o
b
63. A man coming out of the backdoor of his house which is (c) 10 min (d) 12 min

.w
facing East, walked for one kilometre, turned to his right 68. Four players P. Q. R and S are standing a play filed in such

o
and walked for another kilometre. Then he turned to his a way that Q is to East of P, R is to the South of P and S is to

rd
right and walked a kilometre again. Where was he from his the North of P. In which direction of Q is S Standing ?

p
re
house at the end? (a) North (b) South

s
(a) 1 km away in north (c) North-West (d) South-East

s
.c
(b) 1 km away in south 69. A cyclist goes 30 km to North and then turning to East he

o
m
(c) 1 km away in east goes 40 km. Again he turns to his right and goes 20 km.
(d) 1 km away in west After this he turns to his right and goes 40 km. How far is he
64. Two squads of soldiers A and B, facing East and West from his starting point ?
respectively received the following commands - Left Turn, (a) 0 km. (b) 10 km.
About Turn, Right Turn, Left Turn. Which directions would (c) 25 km. (d) 40 km.
the squads A and B face at the end? 70. A boy from his home, first walks 20 m in north - West direction
(a) East, West (b) West, East & then 20 m in South - West direction. Next, he walks 20 m
(c) North, South (d) South, North South - East direction. Finally. he turns towards his house.
65. A direction pole was situated on the crossing. Due to an In which direction is he moving ?
accident, the pole turned in such a manner that the pointer (a) North - West (b) North - East
which was showing East started showing South. One (c) South - West (d) South - East
traveller went to the wrong direction thinking it to be West. 71. A person walks towards his house at 8.00 am and observes
In what direction actually was he travelling? his shadow to his right. In which direction he is walking ?
(a) South (b) East (a) North (b) South
(c) West (d) North (c) East (d) West
66. Dinesh and Ramesh start together from a certain point in 72. A boat moves from a jetty towards East. After sailing for 9
the opposite direction on motorcycles. The speed of nautical miles, she turns towards right and covers another
Dinesh is 60 km per hour and Ramesh 44 km per hour. What 12 nautical miles. If she wants to go back to the jetty, what
will be the distance between them after 15 minutes ? is the shortest distance now from her present position ?
(a) 20 km (b) 24 km (a) 21 nautical miles (b) 20 nautical miles
(c) 26 km (d) 30 km (c) 18 nautical miles (d) 15 nautical miles

ANSWER KEY
1 (d) 9 (d) 17 (c) 25 (d) 33 (b) 41 (b) 49 (d ) 57 (b) 65 (d)
2 (c) 10 (a) 18 (a) 26 (c) 34 (d) 42 (d) 50 (b ) 58 (a) 66 (c)
3 (c) 11 (b) 19 (a) 27 (b) 35 (b) 43 (c) 51 (c) 59 (b) 67 (b)
4 (c) 12 (d) 20 (a) 28 (a) 36 (d) 44 (a) 52 (c) 60 (a) 68 (c)
5 (b) 13 (a) 21 (b) 29 (a) 37 (d) 45 (d) 53 (c) 61 (d) 69 (b)
6 (a) 14 (b) 22 (b) 30 (b) 38 (d) 46 (c) 54 (c) 62 (c) 70 (b)
7 (c) 15 (a) 23 (d) 31 (d) 39 (c) 47 (a) 55 (b ) 63 (a) 71 (b)
8 (c) 16 (c) 24 (c) 32 (d) 40 (d) 48 (c) 56 (a) 64 (d) 72 (d)
y
o
u
Direction & Distance A-47

rs
m
a
h
b
o
o
b
.w
1. (d) We have been given that Q travels towards East and 8. (c)
M travels towards North. Now, T travels towards right Shabana

o
N

rd
of Q implies that T travels towards South. Hence, S m
25

p
travels towards North (because S and T travel in

re
E
opposite directions). Therefore, it is definitely true that

s
s
M and S travel in the same direction i.e., North. Vani 20 m

.c
Ruchi

o
m
9. (d)
N

2. (c)
W E
Q is second to the left of R.
SW
3. (c) T S
P S 10. (a) In the afternoon the sun is in the west. Hence the
shadow is in the east. Now, east is to the left of
Q Madhuri. So, Madhuri is facing south. Therefore,
R Manisha, who is face to face with Madhuri, is facing
north.
Hence, R is to the South-West with respect to S. N
4. (c)
D M 5

11. (b) W E 3

K F S
M is to the North-East of K. 12. (d) Clearly, comparing the direction of A w.r.t. C in the
5. (b) After 4 pm the shadow will be towards East. Now, East second diagram with that in the first diagram, A will be
is to the right of Ramesh. So Ramesh faces North. And south-west of C.
his uncle, who is opposite him, faces South.
5 km B 5 km N
B C
6. (a) A

W E

10 km SW A
S

13. (a) The Sun rises in the east. So, in morning, the shadow
falls towards the west. Now, shadow of pole falls to
A B
5 km 5 km the right of Gopal.Therefore, Gopal’s right side is the
20 km west. So, he is facing South. C 1km B
Starting point 14. (b) Clearly, the boy rode from A to B,
7. (c) N then to C and finally up to D. Since 2km
30 m D lies to the west of A, so required
W E distance = AB = CD = 2 km. D 1km A
40 m 15. (a) Starting from his house in the East, Ravi moves
S westwards. Then, the theatre, which is to the left, will be
in the South. The hospital, which is straight ahead, will
be to the West. So, the University will be to the North.
50 m
y
o
u
A-48 Direction & Distance

rs
21. (b)

m
N University N
School
NE

a
NW

h
b
W

o
E W E

o
Hospital Home

b
.w
SW NW Home
S
Theatre

o
S It is clear from the diagram that school is in North-west

rd
16. (c) The movements of rat are as shown in figure. Clearly, it direction with respect to home.

p
22. (b) Here, Ravi starts from home at A, moves 10 km

re
is finally walking in the direction FG i.e. North.

s
20' southwards up to B, turns right and moves 10 km up to

s
B

.c
A C, turns right again and moves 10 km up to D and

o
finally turns left and moves 10 km up to E.

m
9' 10' G Thus, his distance from initial position A = AE
D C
= AD + DE
5' 6'
= BC + DE = (5 + 10) km = 15 km.
E 12' F 10 km
D A (Ravi)
17. (c) Here, each direction moves 90° + 45° = 135° E
(Anti-clockwise)
SW 10 km 10 km
S N
NW NE
C 5 km B
SE NW 23. (d) The movements of Rasik from A to F are as shown in
W E
figure.
SE Since CD = AB + EF, so F lies in line with A.
SW S Rasik’s distance from original position A = AF
E
NE = (AG + GF) = (BC + DE) = (30 + 15) m = 45m.
18. (a) Here, R faces towards West. S is to the right of R. So, Also, F lies to the east of A.
S is facing towards South. Thus, Q who is the partner 30 m
B C
of S, will face towards North.
N 20 m
S F
G
(Rasik) A 35 m 15 m
P R W E
D 15 m E
24. (c) The movements of Lokesh are as shown in figure. (A
Q S to B, B to C, C to D, D to E). Clearly, his final position is
19. (a) Here, O is the starting point.
E which is to the North of his house A.
C 10 km B 15 km

5 km
4
E
D 10 km
3
A (Lokesh)
A 3 O 4 B 25. (d)
25 m
Both A and B are 32 42 = 5 km from the starting N
point.
20. (a) Clearly, there are two possible movements of Anuj as 30 m
shown below:
W E
25 m 55 m
North

Sun SE
Sou

S
th-E
ast

40 m

Sun South End point


y
o
u
Direction & Distance A-49

rs
m
26. (c) The positions of the minute and hour hands at 12 noon 31. (d) As shown in figure, the man initially faces in the

a
and 1:30 p.m. are as shown in the diagram. Comparing direction OP. On moving 90° clockwise, he faces in the

h
with direction figure, we see that the hour hand at direction OX. On further moving 180° anticlockwise,

b
o
1:30 p.m. points towards the East. he faces in the direction OY. Finally, on moving 90°

o
b
NE anticlockwise, he faces in the direction OZ, which is

.w
N E South-east.
11 12 1

o
rd
10 2

p
NW SE
9 3

re
4

s
8

s
7 6 5

.c
W S

o
SW

m
11 12 1
10 2 32. (d) Clearly, the man initially faces in the direction OA. On
9 3 moving 45° clockwise, he faces in the direction OB. On
8 4 further moving 180° clockwise, he faces in the direction
7 6 5 OC. Finally, on moving 270° anticlockwise, he faces in
27. (b) In the evening, sun is in the west and so the shadows the direction OD, which is South-west. Hence, the
fall towards east. So, Mohit’s shadow fell towards east. answer is (d)
Now, since Mohit’s shadow fell towards right, therefore, B
Mohit is facing North. So Sumit, standing face to face 270°
with Mohit, was facing South. 180°
28. (a) The movements of Rohit are as shown in figure. 45°
O
A
Rohit’s distance from starting point A = AE
90°
= (AD + DE) = (BC + DE) = (20 + 15) m = 35 m.
Also, E is to the East of A.
(Rohit) 15 m D C
A E
D 33. (b)
G 4m
25 m 25 m F
2m
A 2m
EN
B C
20 m O 6m D
29. (a) In morning, sun rise in the east so shadow of a object
falls towards the west. Now, Kavita’s shadow falls to 8m 8m
the rights of Reeta. Hence, Reeta is facing South and
Kavita is facing North.
B H 20 m C
N N
Reeta

W E
W E
Kavita's Kavita
Shadow S
30. (b) As shown in figure, the man initially faces towards
east i.e., in the direction OA. On moving 100° clockwise, S
AO = 20 – (4 + 6) = 10 m
he faces in the direction OB. On further moving 145°
anti-clockwise, he faces the direction OC. Clearly, OC 34. (d) A 3 km B
makes an angle of (145° – 100°) i.e. 45° with OA and
so, the man faces in the direction North-east. 1 km
C
3
5 km
C
O A
100°
145° 4

O
B House
y
o
u
A-50 Direction & Distance

rs
m
37. (d)
N

a
h
b
o
o
W E

b
.w
9 km

o
1 km

rd
S 5 km

p
re
s
OC = 42 32 = 16 9 25 = 5 km

s
.c
2 km

o
m
Starting N
35. (b) 5 km

W E
5 km 5 km

S
Required distance = 42 32 = 5 km.
2 km
25 metres
38. (d)

15 metres
15 metres
N

Starting 20 metres
Point E Finishing Point
W E towards South is same, i.e., 15 metres. So, Shobha is
20 + 25 metres = 45 metres away from her starting point.
39. (c) (12 km + 5 km = 17 km)
S N

12km W E
Required distance = 5 + 2 = 7 km
12km S
12km
36. (d) 12km
Kiran’s House Finishing
A 5km Point
Home
North West
40. (d) The movements of Deepak are as shown in fig.
D 40 m C

25m
25m
40 m
60 m

A E 75 m B
Clearly, FB = DC = 40 m .
B C Deepak's distance from the starting point A
20 m = (AB – EB) = (75 – 40) m = 35m.
41. (b) The directions to be followed will be :
left left
N
N

W E W E

S
S

right
y
o
u
Direction & Distance A-51

rs
m
42. (d) Clearly, to show 4.30, the position of the minute and 48. (c) Given information diagrammatically can be shown as

a
hour hands of the clock will be as shown. So, again as follows:

h
shown, if the minute hand points East, the hour hand

b
o
will point in the North-east.

o
b
.w
W
D

o
SW NW 5km

rd
11 12 1

p
10 2 5km

re
S N

s
9 3

s
.c
8 A B
4

o
m
7 6 5 SE NE
E 5km

43. (c)
44. (a)
C
5km
N
15 km
P Q From the above diagram, it is clear that the houses of C
W E 12km and D are less than 20 km apart.
18 km
O N
49. (d) M
R
S
N W E
Let O be the starting point and P, Q and R the positions
after every movement. Hence, Distance from the starting
point =Distance of final position R from O = OR = 18 – 12 P S
= 6 km.
N is between M and P. Hence only (ii) statement is correct.
45. (d) 50. (b) The position of all the five persons is as follows:

A B C D E
Hence AB is smaller than BE
51. (c) The positions of school, house, post office and market
are as follows:

Market N

Hence, Priya is moving in the South direction. School


46. (c) W E
Post office

House S
Hence the market is in the North-east of my house.
52. (c)

8m
53. (c) A B

6m
Hence, Asha is 3km from starting point and in the east
direction.
D C
47. (a) Clearly, PQ OP2 OQ2 (300) 2 (400) 2
= 90000 160000 = 500 km Required distance = 8 + 6 + 8 + 6 + 82 62
1 = 28 + 100 = 28 + 10 = 38 m
Since, R is the midpoint of PQ, so QR PQ = 250 km.
2
y
o
u
A-52 Direction & Distance

rs
m
54. (c) The positions of the ladies and the men are shown in 58. (a) As clear from the adjoining diagram, C lies to the

a
the diagram given below east of A.

h
b
M1

o
N B

o
NW NE

b
N

.w
40 m 40 m
M2 L2

o
W E
W E

rd
p
A C

re
SW SE
S
L1 S

s
s
.c
59. (b) The movements for Gaurav are as shown in figure.

o
m
55. (b) 1 km Clearly, Gaurav’s distance from his initial position
A C
P = PX= (PS + SX) = (QR + SX) = (40 + 20) m = 60 m.
1 km 40 m
N R Q
1 km
F D

20m
W E 20m
1 km
S
1 km X 20m S P (Gaurav)
E B
60. (a)
Hence, A, D, E in a line.
56. (a) Original directions 70 m
100 m
North B' A'
North-East
A B
North-West 50 m 10 30 My friend
My self 10
20
West East
When my friend reaches on the previous track (i.e. on
B') again, he had travelled a distance of (30 + 10 + 20
+ 10) = 70 m. As I walk with the same speed as that of
South-West South-East
my friend I have walked 70m, but on the straight track.
South
Now, he is just [100 – (30 + 20)] = 50m from my starting
Changed directions point.
Hence, the distance between us = (70 – 50) = 20m
West
61. (d)
South-West North-West
g
A
tin

South North
ee
1s m
tm
30

90 m 90 m
South-East North-East

East 8 km/h Rohan


B D
57. (b) N
South West 90 m 90 m

Rahul 10 km/h
10 ng

W E
m

C
ti
ee
m
d
2n

East North
S
Speeds of Rohan and Rahul are in the ratio 4 : 5.
From the figure, it is clear that ‘S’ becomes ‘North-east’ in
the new figure (dotted line) 62. (c) If South East becomes North then south west becomes
east as shown in direction chart.
y
o
u
Direction & Distance A-53

rs
m
Distance travelled by them = Relative speed × Time

a
15 104×15

h
SW distance = 104km / hr hr = 26km

b
S NW NE W 60 60

o
N

o
67. (b)

b
.w
N

o
S

rd
68. (c) North North west

p
E W E

re
SE W
NW P East Q

s
s
.c
South

o
S

m
R
SE
Hence, Q is in North west disection of S.
SW S
NE N
E 69. (b) East
C 40km D right

63. (a) North N


20 km 20 km
Starting 1 km
point E
W E 30 km
1 km B
North right
Destination point
of cyclist
1 km A
S Starting point
of cyclist
64. (d) North (B) About turn Distance from starting point = AC – BC = 30 – 20 = 10 km

Left Left 70. (b) B


(South-West)
Starting point of B. Starting point of A 20 m (North-West)
Right Right 20 m
>

Left
Left
C A (Boy’s Starting Point)
South (A)
About turn
>
20 m
North-East
(South-East)
About turn-turning in reverse direction.
D
(Boy’s Destination Point)
65. (d)
71. (b) A pesson observes his shadow to his right. The sun
is to his left. He is walking towards the south.

72. (d)

As East started showing south then west was actually


north.

66. (c) The shortest distance

AB2 BC2

Relative speed of Dinesh and Ramesh's motorcycles 92 122


= (60 + 44) = 104 km/hr 225 = 15 nautical miles
y
o
u
A-54 Time Sequence, Number & Ranking Test

rs
m
CHAPTER

a
h
b
TIME SEQUENCE, NUMBER
7

o
o
b
.w
& RANKING TEST

o
rd
p
re
s
s
.c
TIME SEQUENCE Sol. (c) According to Vandana her father’s birthday is on one

o
m
To solve problems related to time sequence, let us gather 1st the of the days among 14 th and 15 th June. According to
following informations : Vandana’s brother, the father’s birthday is on one of the
1 Minute = 60 seconds days among 15 th 16th and 17th June.
1 Hour = 60 minutes It is obvious that the father’s birthday is on the day common
1 Day = 24 hours to both the above groups. The common day is 15th June.
1 Week = 7 days Hence, the father’s birthday falls on 15 th June.
1 Month = 4 weeks Option (c) is the correct option.
1 Year = 12 months NUMBER TEST
1 Ordinary year = 365 days In such test, generally you are given a long series of numbers.
1 Leap year = 366 days The candidate is required to find out how many times a number
1 Century = 100 years satifying the conditions specified in the question occurs.
Other facts to be remembered
• A day is the period of the earth’s revolution on its axis. EXAMPLE 3. How many 8s are there in the following number
• A ‘Solar year’ is the time taken the earth to travel round the sequence which are immediately preceded by 5 but not
1 immediately followed by 3?
sun. It is equal to 365 days, 5 hours, 48 minutes and 47 38584583988588893
2
(a) 1 (b) 4
seconds nearly.
(c) 3 (d) 2
• A ‘Lunar month’ is the time taken the moon to travel round
Sol. (d) Let use see the following :
the earth. It is equal to nearly 28 days.
3 8 5 8 4 5 8 3 9 88 5 8 88 93
Leap Year
clearly, such sequence occurs two lines
• If the number of a given year is divisible by 4, it is a leap
Option (d) is correct.
year. Hence, the years like 1996, 2008, 2012 are leap years.
But years like 1997, 1991, 2005, 2007 are not divisible by 4 EXAMPLE 4. What will be last digit of the 3rd number from
and therefore, such years are not leap years. top when the numbers given below are arranged in descending
• In a leap year, February has 29 days. order after reversing the position of the digits within each
• A leap year has 52 weeks and 2 days. number?
517 325 639 841 792
EXAMPLE 1. Neena returned house after 3 days earlier
(a) 2 (b) 5
than the time she had told her mother. Neena’s sister Veena (c) 7 (d) 3
reached five days later than the day Neena was supposed to return. Sol. (d) The given numbers are :
If Neena returned on Thursday, on what day did Veena return ? 517 325 639 841 792
(a) Friday (b) Saturday After reversing, the numbers becomes as follows :
(c) Wednesday (d) Sunday 715 523 936 148 297
Sol. (a) Neena returned home on Thursday. Neena was When arranged in descending order the numbers become
supposed to return 3 days later, i.e., on Sunday. as follows :
Veena returned five days later from Sunday. i.e., on Friday. 936 715 523 297 148
Option (a) is the correct option. Now, the third number from top is 523. Hence, the last digit
of 523 is 3.
EXAMPLE 2. Vandana remembers that her father’s birthday Option (d) is correct.
is between 13 th and 16 th of June. Whereas her brother RANKING TEST
remembers that their Father’s birthday is between 14th and In such problems, the ranks of a person both from the top and
18 th of June. On which day is their Father’s birthday ? from the bottom are given and on the basis of this the total number
(a) 14 th June (b) 16 th June of persons is asked. Sometimes question is twisted also and
(c) 15 th June (d) 18 th June position of a particular person is asked.
y
o
u
Time Sequence, Number & Ranking Test A-55

rs
m
EXAMPLE 5. Karishma ranks 10 th from the top and 15 th Sol. (d) As per the question; the class has

a
(i) 15 students higher than Karishma

h
from the bottom in an examination. Find the total number of

b
(ii) 14 students lower than Karishma

o
students in Karishma’s class.

o
(iii) Karishma

b
(a) 35 (b) 31

.w
Total number of students = 15 + 14 + 1 = 30
(c) 28 (d) 30

o
Hence, option (d) is correct.

rd
p
re
s
s
.c
o
m
1. Mohan and Suresh study in the same class. Mohan has 7. If the positions of the first and the sixth digits of the group
secured more marks than Suresh in the terminal examination. of digits 5904627813 are interchanged, similarly, the
Suresh’s rank is seventh from top among all the students in positions of the second and the seventh are interchanged,
the class. Which of the following is definitely true? and so on, which of the following will be the fourth from the
(a) Mohan stood first in the terminal examination. right end after the rearrangement?
(b) There is at least one student between Mohan and (a) 4 (b) 9 (c) 1 (d) 0
Suresh in the rank list. 8. In a row of boys Akash is fifth from the left and Nikhil is
(c) There are at the most five students between Mohan eleventh from the right. If Akash is twenty-fifth from the
and Suresh in the rank list. right then how many boys are there between Akash and
(d) Suresh is five ranks lower than Mohan in the rank list. Nikhil?
2. Fifteen children are standing in a row facing north. Ravi is (a) 14 (b) 13 (c) 15 (d) 12
to the immediate left of Prabha and is eighth from the left 9. The positions of the first and the sixth digits in the number
end. Arjun is second from the right end. Which of the 3597280164 are interchanged. Similarly, the positions of the
following statements is not true? second and the seventh digits are interchanged, and so on.
(a) Prabha is 7th from right end. Which of the following will be the fourth digit from the right
(b) There are four children between Prabha and Arjun. end after the rearrangement?
(c) There are five children between Ravi and Arjun. (a) 5 (b) 3 (c) 9 (d) 4
(d) Arjun is 13th from the left end. 10. In a shop, there were 4 dolls of different heights M, N, O
3. Rajnish is older than Rajesh and Raman. Ramesh is older and P. ‘P’ is neither as tall as ‘M’ nor as short as ‘O’. ‘N’ is
than Rajesh but younger than Rajeev. Raman is older than shorter than ‘P’ but taller than ‘O’. If Anvi wants to purchase
Rajeev. Who among them is oldest? the tallest doll, which one should she purchase?
(a) Rajeev (b) Rajesh (a) Either M or P (b) Either P or N
(c) Rajnish (d) Ramesh (c) Only P (d) Only M
4. If ‘P’ means ‘division’, ‘T’ means ‘addition’, ‘M’ means 11. Ketan takes casual leave only on first working day of every
‘subtraction’, and ‘D’ means ‘multiplication’ then what will month. The office has weekly offs on Saturday and Sunday.
be the value of the following expression? In a month of 30 days, the first working day happened to be
12 M 45 D 28 P 7 T 15 = ? Tuesday. What will be the day for his next casual leave?
(a) – 15 (b) 45 (a) Wednesday (b) Thursday
(c) – 30 (d) None of these (c) Friday (d) Monday
5. If the positions of the first and the fifth digits of the number 12. Abhay gave an application for a new ration card to the clerk
83721569 are interchanged, similarly, the positions of the on Monday afternoon. Next day was a holiday. So the clerk
second and the sixth digits are interchanged, and so on, cleared the papers on the next working day on resumption
which of the following will be the third from the right end of duty. The senior clerk checked it on the same day but
after the rearrangement? forwarded it to the head clerk on next day. The head clerk
(a) 6 (b) 3 decided to dispose the case on the subsequent day. On
(c) 2 (d) 7 which of the following days was the case put up to the
6. In a class some students play cricket only, some other head clerk by the senior clerk?
1 (a) Wednesday (b) Thursday
students play football only and remaining th students (c) Friday (d) Saturday
6
play both cricket and football. Which of the following 13. Five newly born babies were weighed by the doctor. In her
statements is definitely true? report, she stated that child A is lighter than child B, child C
(a) Two-thirds of the students play cricket. is lighter than D and child B is lighter than child D, but
(b) Three-fourths of the students play football only. heavier than child E. Which child is the heaviest?
(c) One-thirds of the students play football only. (a) E (b) D
(d) None of these (c) C (d) A
y
o
u
A-56 Time Sequence, Number & Ranking Test

rs
m
14. If it is possible to make a number which is perfect square of 24. In a row of students, Ramesh is 9th from the left and

a
a two-digit odd number with the second, the sixth and ninth Suman is 6th from the right. When they both interchange

h
their positions then Ramesh will be 15th from the left. What

b
digits of the number 187642539. which of the following is

o
the digit in the unit’s place of that two-digit odd number ? will be the position of Suman from the right?

o
b
(a) 1 (b) 7 (a) 12th (b) 13th

.w
(c) 9 (d) None of these (c) 15th (d) 6th

o
25. There are five steel boxes labelled A, B, C, D and E. E is

rd
15. A, B, C, D and E, when arranged in descending order of
heavier only than A. C is heavier than A and E and is less

p
their weight from top, A becomes third, E is between D and

re
heavier than B which is not the heaviest. Which of the
A, C and D are not at the top. Who among them is the following boxes is the heaviest?

s
s
second? (a) A (b) B

.c
o
(a) C (b) B (c) C (d) D

m
(c) E (d) Data inadequate 26. P, Q, R and S are four men. P is the oldest but not the
poorest. R is the richest but not the oldest. Q is older than
16. There are seven persons up on a ladder, A, B, C, D, E, F and
S but not than P or R. P is richer than Q but not than S. The
G (not in that order). A is further up than E but is lower than four men can be ordered (descending) in respect of age and
C. B is in the middle. G is between A and B. E is between B richness, respectively, as
and F. If F is between E and D, the person on the bottom (a) PQRS, RPSQ (b) PRQS, RSPQ
step of the ladder will be (c) PRQS, RSQP (d) PRSQ, RSPQ
(a) B (b) F 27. In a class of 45 students, among those students who passed,
(c) D (d) E Anmol secured 11th position from upwards and 15th from
17. Rakesh is on 9th position from upwards and on 38th position downwards. How many students failed?
from downwards in a class. How many students are in class? (a) 19 (b) 20
(c) 15 (d) 18
(a) 47 (b) 45
28. In a row at a bus stop, A is 7th from the left and B is 9 th from
(c) 46 (d) 48
the right. Both of them interchange their positions and thus
18. Sarita is on 11th place from upwards in a group of 45 girls. If
A becomes 11th from the left. How many people are there in
we start counting from downwards, what will be her place? that row?
(a) 36th (b) 34th (a) 18 (b) 19
(c) 35th (d) Cannot be determined (c) 20 (d) 21
19. Raman is 9th from downwards in a class of 31 students. 29. In a row of boys facing the North, A is sixteenth from the
What will be his position from upwards? left end and C is sixteenth from the right end. B, who is
(a) 21st (b) 22nd fourth to the right of A, is fifth to the left of C in the row.
(c) 23rd (d) 24th How many boys are there in the row ?
20. Some boys are sitting in a line. Mahendra is on 17th place (a) 39 (b) 40
from left and Surendra is on 18th place from right. There are (c) 41 (d) 42
8 boys in between them. How many boys are there in the 30. In a class of 60, where girls are twice that of boys, kamal
line? ranked seventeenth from the top. If there are 9 girls ahead
(a) 43 (b) 42 of Kamal, how many boys are after him in rank ?
(c) 41 (d) 44 (a) 3 (b) 7
21. In a line of boys, Ganesh is 12th from the left and Rajan is (c) 12 (d) 23
15th from the right. They interchange their positions. Now, 31. Priya is taller than Tiya and shorter than Siya. Riya is shorter
Rajan is 20th from the right. What is the total no. of boys in than Siya and taller than Priya. Riya is taller than Diya, who
the class? is shorter than Tiya. Arrange them in order of ascending
(a) 30 (b) 29 heights.
(c) 32 (d) 31 (a) Priya – Siya – Riya – Tiya – Diya
22. In a queue, Vijay is fourteenth from the front and Jack is (b) Riya – Siya – Priya – Diya – Tiya
seventeenth from the end, while Mary is in between Vijay (c) Siya – Riya – Priya – Tiya – Diya
(d) Siya – Priya – Riya – Diya – Tiya
and Jack. If Vijay be ahead of Jack and there be 48 persons
32. In a queue, A is eighteenth from the front while B is six-
in the queue, how many persons are there between Vijay
teenth from the back. If C is twentieth from the front and is
and Mary?
exactly in the middle of A and B, then how many persons
(a) 8 (b) 7
are there is the queue ?
(c) 6 (d) 5 (a) 45 (b) 46
23. Malay Pratap is on 13th position from the starting and on (c) 47 (d) 48
17th position from the end in his class. He is on 8th position 33. In a row of 21 girls, when monika was shifted by four place
from the starting and on 13th position from the end among towards the right, she became 12 th from the left end. What
the students who passed. How many students failed? was her earlier positions from the right end of the row ?
(a) 7 (b) 8 (a) 9th (b) 10th
(c) 9 (d) Cannot be determined (c) 11th (d) 14 th
y
o
u
Time Sequence, Number & Ranking Test A-57

rs
m
34. Gita is prettier than Sita but not as pretty as Rita. Then, 37. On one side of a street are even numbers and on the other

a
(a) Sita is not as pretty as Gita. side are odd numbers. No. 1 is exactly in front of No. 2. My

h
(b) Sita is prettier than Rita. House is No. 9. From my house , a man comes up from No.

b
o
(c) Rita is not as pretty as Gita. 2 and knocks at the door, five doors beyond the house

o
b
(d) Gita is prettier than Rita. infront of me. What is the No. of that house ?

.w
35. Ram and Sham are ranked 13th and 14th respectively is a (a) 18 (b) 20

o
class of 23. What are their ranks from the last respectively?

rd
(c) 22 (d) 26
(a) 10 th : 11th (b) 11 th; 12 th

p
38. Five policemen are standing in a row facing south. Shekhar

re
(c) 11th ; 10 th (d) None of these is to the immediate right of Dhanush. Bala is between Basha

s
36. Five coaches P, L, R, M, O are in a row. R is to the right of M

s
and Dhanush. David is at the extreme right end of the row.

.c
and left of P. L is to the right of P and left of O. Which coach

o
Who is standing in the middle of the row?

m
is in the middle?
(a) Bala (b) Basha
(a) P (b) L
(c) Shekhar (d) Dhanush
(c) R (d) O

ANSWER KEY
1 (a) 6 (d) 11 (b) 16 (c) 21 (d) 26 (b) 31 (c) 36 (a)
2 (d) 7 (b) 12 (b) 17 (c) 22 (b) 27 (b) 32 (c) 37 (b)
3 (c) 8 (b) 13 (b) 18 (c) 23 (c) 28 (b) 33 (d) 38 (d)
4 (d) 9 (a) 14 (b) 19 (c) 24 (a) 29 (b) 34 (a)
5 (b) 10 (d) 15 (a) 20 (a) 25 (d) 30 (c) 35 (c)

1. (a) 11. (b) If the first working day happened to be Tuesday then
2. (d) 8th 9th 14th 8th, 15th, 22nd and 29th of the month will be Tuesday.
Ravi Prabha Arjun Hence, the last day of the month will be Wednesday
3. (c) Rajnish > Rajesh, Raman... (i) (since, number of days in the month is 30). Thus, the
Rajeev > Ramesh > Rajesh ... (ii)
next casual leave will be on Thursday.
Raman > Rajeev ... (iii)
Combining all, we get 12. (b) (i) Submitted application form : Monday
Rajnish > Raman > Rajeev > Ramesh > Rajesh (ii) Holiday : Tuesday
4. (d) 12 – 45 × 28 ÷ 7 + 15 (iii) Clearance from clerk : Wednesday
= 12 – 45 × 4 + 15 = 27 – 180 = – 153 (iv) Clearance from senior clerk : Wednesday
5. (b) New arrangement of numbers is as follows: 15698372
(v) Submitted to the head clerk : Thursday
Hence, third number from right end is 3.
6. (d) We can’t find the proportion of those students out of 13. (b) Even if we cannot determine the exact sequence of the
the total students who play only cricket. Similarly, we weights of the children, we can conclude on the basis
can’t find the proportion of those students out of the of the information provided that D is the heaviest.
total students who play only football. But 5/6th of the 14. (b) The specified digits are 8, 2 and 9. Now, we know a
total strength play either cricket only or football only. perfect square number does not have 8 and 2 at unit’s
7. (b) In the original group of digits ‘7’ is fourth from the place. Therefore, we can make only two three-digit
right, which is interchanged with ‘9’. The new series is
numbers from it, i.e., 829 and 289. Among these two
2781359046.
8. (b) There are (25 – 11– 1 =) 13 boys between Akash and numbers, 289 is a perfect square number, i.e., square of
Nikhil. 17. Thus, unit’s digit is 7 and ten’s digit is 1.
9. (a) After interchanging the number becomes as follows: 15. (a) ------
8 01 6 43 5972 ----A----
Hence, the fourth digit from the right end is 5. DEA - - [It is not possible as D is not at the top.]
10. (d) The correct order of dolls according to descending
--- AED
order of their heights are:
M> P>N >O BCAED
Therefore, Anvi will purchase the doll M. Hence, C is second among them.
y
o
u
A-58 Time Sequence, Number & Ranking Test

rs
m
25. (d) D > B > C > E > A
16. (c) Top

a
C 26. (b) Q > S, P > Q, R > Q

h
Age: As, Q > S, P > Q, R > Q

b
o
A Also, P is the oldest.

o
b
G P> R> Q>S

.w
Richness : As, P > Q, S > P

o
Also R is he richest.

rd
Middle B
R> S> P>Q

p
re
E 27. (b) Failed Students

s
= [Total students] – [(Anmol’s position from upwards)

s
.c
F + (Anmol’s position from downwards) – 1]

o
m
Bottom D = 45 – [(11 + 15) – 1] = 20
28. (b) After interchanging their positions, position of A from
D is at the bottom step of the ladder. left = 11
17. (c) Total students then positions of A form right = 9.
= [Rakesh’s position from upwards + Rakesh’s position The total no. of people in the row
from downwards] – 1 = (9 + 11) – 1 = 19.
= [9 + 38] – 1 = 46 29. (b)
18. (c) Sarita’s place from downwards
Total Sarita’s place
1 = [45 – 11] + 1 = 35th N
girls from upwards
19. (c) Raman’s position from upwards A B C
15 3 4 15

Raman’s Clearly, according to the given conditions, there are 15


Total position 1 boys to the left of A , as well as to the right of C. Also, B
students
from down lies between A and C such that there are 3 boys between
A and B and 4 boys between B and C. So, number of
= [31 – 9] + 1 = 23rd boys in the row = (15 + 1 + 3 + 1 + 4 + 1 + 15) = 40.
20. (a) Total boys 30. (c) Let the number of boys be x.
Then, number of girls = 2x.
Mahendra’s Surendra’s Boys between x + 2x = 60 or 3x = 60 or x = 20.
place place + So, number of boys = 20 and number of girls = 40.
from left from right them
Number of students behind Kamal in rank (60 – 17)
= 43.
= [17 + 18] + 8 = 43 Number of girls ahead of Kamal in rank = 9.
21. (d) Total students Number of girls behind Kamal in rank = (40 – 9) = 31
= [First position of Ganesh + Second position of Rajan] Number of boys behind Kamal in rank
–1 = (43 – 31) = 12.
= [12 + 20] – 1 = 31 31 (c) From statement (1) when we arrange them in order of
22. (b) Number of persons between Vijay and Jack ascending heights.
= 48 – (14 + 17) = 17 Siya > Priya > Tiya ....(i)
Now, Mary lies in middle of these 17 persons i.e., at the Similarly from statement (2)
eighth position. Siya > Riya > Priya ....(ii)
So, number of persons between Vijay and Mary = 7. Similarly from statement (3)
23. (c) Total students Riya > Tiya > Diya ....(iii)
= [Malay’s place from starting + Malay’s place from From (i), (ii) and (iii) if they are arranged in order of
end] –1 ascending heights.
= [13 + 17] – 1 = 29 Diya < Tiya < Priya < Riya < Siya
Number of passed students Option (c) states the descending order correctly and
= [Malay’s place from starting + Malay’s place from hence, should be the correct option.
end] –1 32. (c) A is 18th from front and C is 24th
= [8 + 13] – 1= 20 Number of persons between A and C = 6.
Number of failed students = 29 – 20 = 9 Since C is exactly in middle of A and B, so number of
24. (a) Position of Suman from right persons between C and B = 6.
17 6 6 15
Difference of First position
Ramesh’s position of Suman A C B
Number of persons in the queue
= [(15 – 9) + 6 ] = 12th = ( 17 + 1 + 6 + 1 + 6 + 1 + 15 ) = 47.
y
o
u
Time Sequence, Number & Ranking Test A-59

rs
m
33. (d) The change of place by Monika can be shown as under. 37. (b) My Home

a
h
1 2 3 4 5 6 7 8 9 10 11 M 13 14 15 16 17 18 19 20 21

b
o
1 3 5 7 9 11 13 15 17 19 21

o
b
Clearly, Monika's earlier position was 8th from the left Man from no.2

.w
and 14th from the right end. 5 houses beyond my house.

o
rd
34. (a) Rita > Gita > Sita 2 4 6 8 10 12 14 16 18 20 22

p
35. (c) Rank of Ram from the last = 23 – 13 + 1 = 11

re
Hence, 20 is the number of that house.

s
and Rank of Shyam from the last = 23 – 14 + 1 = 10

s
38. (d) Standing Arrangement : (Facing South)

.c
36. (a)

o
Right Left

m
End End
Hence, P coach is in the middle of the five coaches. David Shekhar Dhanush Bala Basha
Hence, Dhanush is standing at the middle of the row.
y
o
u
A-60 Number Puzzles

rs
m
a
CHAPTER

h
b
8

o
NUMBER PUZZLES

o
b
.w
o
rd
p
re
s
s
.c
o
m
This question are based on different number. This type of problem (a) 195 (b) 61
having figure which follows a particular rule for their different (c) 99 (d) 120
number. We have then asked to find a missing number by using
Sol. (b) 110 + 30 – 75 = 65
same rule.
103 + 25 – 67 = 61
Direction:- (Sample problem)
97 + 82 – 80 = 99
EXAMPLE 1. Find the missing number in the following
diagram:
EXAMPLE 3. 24 29 21 41 .
22
4
82
11

7 35 13 11 9
4 13

(a) 37 (b) 39 53 43 ?
(c) 38 (d) 33 (a) 123 (b) 121
(c) 63 (d) 33
Sol. (a) 4 + 7 + 11 + 13 = 35
Similarly, 11 + 13 + 4 + 9 = 37. Sol. (a) 29 + 24 = 53
21 + 22 = 43
EXAMPLE 2. 41 + 82 = 123

110
25
75 67 EXAMPLE 4. 2 5 7
65 ? 6 15 21
10 19 ?
30 103 (a) 52 (b) 48
82 (c) 25 (d) 28
80
99 Sol. (c) 2 × 3 + 4 = 10
5 × 3 + 4 = 19
7 × 3 + 4 = 25
97
y
o
u
Number Puzzles A-61

rs
m
a
h
b
o
o
b
.w
DIRECTIONS: In the following questions find missing number 8. 8 9 10

o
1. 16

rd
12 21 9 10
? 5 4 3

p
re
28 ? 16

s
s
12 25 14

.c
o
(a) 28 (b) 11

m
(c) 32 (d) 18
14 15 16 9.
(a) – 21 (b) 12 1
(c) 32 (d) 22
2. 1 2 3 2 10 12
2 5 12 10 16 13
4 30 2
1 2 1 ? 10 24
(a) 5 (b) 11
(c) 13 (d) 8

3. 3
? 9
3
33 17

(a) 60 (b) 68
(c) 55 (d) 65
6 286 4
4. 2 7 9
7 3 4
9 8 ?
126 168 216 15

(a) 8 (b) 3
4
(c) 6 (d) 36
5. 169 64 81 30
625 ? 49 50
1296 576 100 70 3 218 ?
(a) 324 (b) 289
(c) 441 (d) 361
6. 1 2 3
12
4 5 6
7 8 9 (a) 6 (b) 7
(c) 9 (d) 12
27 38 ?
(a) 49 (b) 51 28 50
(c) 50 (d) 52 10. 2 3 4 5
7. 3 4 5
2 3 4
1 2 3 ?
3 5
14 29 ?
(a) 50 (b) 30 (a) 35 (b) 40
(c) 40 (d) 32 (c) 49 (d) 53
y
o
u
A-62 Number Puzzles

rs
11.

m
3 3 1 2 3

a
16.

h
2 3 4

b
o
6 10 ?

o
b
12 20

.w
1 5 3 7 (a) 18 (b) 24
(c) 14 (d) 16

o
rd
17. 3 4 5

p
re
6 8 4 5 3

s
3

s
4 3 ?

.c
o
48 60 105

m
(a) 2 (b) 6
? (c) 7 (d) 9
2 7
18.
? 5
6
(a) 10 (b) 15 11 7
(c) 20 (d) 25
(a) 1 (b) 9
12. 21 24 36 (c) 12 (d) 17
11 14 12 19.
3 ? 4 594 198
77 112 108
(a) 2 (b) 4 ? 66
(c) 3 (d) 5
(a) 22 (b) 33
13. 5 9 15
(c) 11 (d) 44
16 29 ?
20. 6 15 20
49 89 147
8 4 5
(a) 45 (b) 48 3 5 20
(c) 51 (d) 54
51 65 ?
14. (a) 56 (b) 120
17 8
7 9 (c) 51 (d) 44
21.
40 54
7

4 5 6 3 ? 15

10 21
127 31
63
?

3 (a) 190 (b) 255


6 (c) 221 (d) 236
22.
(a) 60 (b) 62 36 25
(c) 64 (d) 66
? 27
2 3 1
33 23
1 2 1
15. 21 30
3 4 ?
(a) 5 (b) 2 (a) 35 (b) 32
(c) 1 (d) 4 (c) 22 (d) 19
y
o
u
Number Puzzles A-63

rs
m
23. 30.
12 16 18

a
4 7

h
16 16 20

b
5 3

o
5 7 ? 1 64

o
b
11 27 ? 8
197 263 356

.w
8 2

o
(a) 9 (b) – 4

rd
(c) 4 (d) – 8

p
24. (a) 0 (b) 8

re
(c) 125 (d) 216

s
5 7

s
.c
31.

o
1

m
3 23 4 3 54 6
15 2 17
? 16
1 4 4 6
3 18 5
19
11
(a) 13 (b) 14
? 40 2 (c) 20 (d) 21
32.
9 5 6 6 7 4 8
12 21 ?
(a) 4 (b) 1
(c) 2 (d) 3 4 5 10
25. The diagram below is a ‘magic square’ in which all rows
(a) 14 (b) 22
and columns and both diagonals add up to 34. Find xy
(c) 32 (d) 320
1 8 13 12 33.
14 93 27 79 38 67 16
4 x 16 y
15 63 37 42

(a) 77 (b) 60
3 4 ?
(c) 45 (d) 63
26. 8 10 12 A B C
5 7 9
(a) 5 (b) 6
12 14 ?
(c) 8 (d) 9
(a) 16 (b) 15
(c) 18 (d) 17 34. 4 8 0
5 0 6 7 1 6 11 ? 2
27. 1 12 10
2
7 4
15 2 ? A B C
8 5 3 (a) 0 (b) 2
(c) 11 (d) 12
(a) 9 (b) 11
(c) 4 (d) 6 35. 101 43 48 34
28. 38 ?
6 9 15 15
35 56 184
8 12 20 A B
4 6 ? (a) 127 (b) 142
(a) 5 (b) 10 (c) 158 (d) 198
(c) 15 (d) 21 27 30 59 40
29 29
29. 36. 39 72 33 42 73 31 ? 79 10
72 24 6
96 16 12 45 43 43 44 39 20
108 ? 18 A B C
(a) 49 (b) 50
(a) 12 (b) 16
(c) 18 (d) 20 (c) 60 (d) 69
y
o
u
A-64 Number Puzzles

rs
m
1 2 4

a
37. ? 2

h
9 3

b
7 8 ?

o
44. 7 4

o
b
5 5

.w
9 6 5 6 8 4

o
(a) 6 (b) 9 (a) 10 (b) 11

rd
(c) 10 (d) 11 (c) 12 (d) 13

p
re
45. 4 9 2

s
38. Z ? S

s
3 5 7

.c
R O ?

o
8 1 ?

m
? G C (a) 9 (b) 6
(a) WJK (b) KWT (c) 15 (d) 14
(c) WKJ (d) JKW 46. 7 6 6
8 6 ?
39. B G N 3 4 5
D J R 168 144 120
G N ? (a) 8 (b) 10
(a) U (b) V (c) 5 (d) 4
(c) W (d) X 47. 8 5 6
3 7 5
40. 2 4 0 1 4 2
1 2 4 74 90 ?
3 1 3 (a) 65 (b) 85
36 ? 91 (c) 52 (d) 76
48. 22 46 24
(a) 24 (b) 48
27 58 31
(c) 59 (d) 73
32 68 ?
36 9 25 (a) 46 (b) 36
41. 49 26 64 81 21 25 64 ? 144 (c) 32 (d) 38
25 16 36
49.
A B C 260 132
(a) 19 (b) 23 ? 68
(c) 25 (d) 31
20 36
3 5 4 7 3 5
42. ? (a) 12 (b) 10
39 51
(c) 9 (d) 8
6 3 5 4 5 4
50.
A B C 7 9 8
(a) 47 (b) 45 2 4 3
(c) 37 (d) 35 5 7 6
43. 16 32 ?
7
286 16
(a) 17 (b) 23
(c) 47 (d) 73

142 34 51. 3 9 2 8 4 7
? 7 4 ?
81 5 64 6 49 5
(a) 72 (b) 70
(c) 68 (d) 66 (a) 1 (b) 8
(c) 6 (d) 16
y
o
u
Number Puzzles A-65

rs
m
ANSWER KEY

a
h
b
1 (d) 7 (a) 13 (b) 19 (a) 25 (c) 31 (b) 37 (d) 43 (b) 49 (a)

o
o
2 (c) 8 (b) 14 (b) 20 (b) 26 (a) 32 (c) 38 (c) 44 (b) 50 (b)

b
.w
3 (d) 9 (b) 15 (a) 21 (b) 27 (c) 33 (d) 39 (c) 45 (b) 51 (c)

o
4 (c) 10 (b) 16 (c) 22 (d) 28 (b) 34 (c) 40 (d) 46 (d)

rd
5 (a) 11 (b) 17 (c) 23 (b) 29 (a) 35 (b) 41 (d) 47 (a)

p
re
6 (b) 12 (c) 18 (d) 24 (c) 30 (d) 36 (d) 42 (c) 48 (b)

s
s
.c
o
m
1. (d) (16 + 12) 2 = 14 11. (b) (5 – 1) × (6 – 3) = 12
(21 + 9) 2 = 15 (7 – 3) × (8 – 3) = 20
(10 + x) 2 = 16 (7 – 2) × (6 – 3) = 15
x = 22 12. (c) As, 3 × 7 = 21, 11 × 7 = 77
2. (c) 4 × 9 = 36, 12 × 9 = 108
1 2 3 2 10 12 Therefore,14 × 8 = 112
2 5 12 10 16 13 ? × 8 = 24
1 2 1 ? 10 24
? 3
4 9 16 25 36 49
13. (b) 5 × 3 + 1 = 16 9 × 3 + 2 = 29
Hence, the missing term is 25 – (2 + 10) = 13. 16 × 3 + 1 = 49 29 × 3 + 2 = 89
3. (d) 9 × 2 – 1 = 17 15 × 3 + 3 = 48
17 × 2 – 1 = 33 48 × 3 + 3 = 147
33 × 2 – 1 = 65 14. (b) (7 + 9 + 5 + 4) × 2 – 10 = 40
4. (c) 2 × 7 × 9 = 126 (17 + 8 + 3 + 6) × 2 – 14 = 54
7 × 3 × 8 = 168 (10 + 21 + 6 + 3) × 2 – 18 = 62
9 × 4 × x = 216 15. (a) 22 – 3 = 1
x=6 12 – 2 = – 1
5. (a) 32 – 4 = 5
169 64 81 30
16. (c) (1 + 2) × 2 = 6
625 x 49 50 x = 324 (2 + 3) × 2 = 10
(3 + 4) × 2 = 14
1296 576 100 70
17. (c) 3 × 4 × 4 = 48
6. (b) 4 × 7 – 1 = 27
4 × 5 × 3 = 60
5 × 8 – 2 = 38
4 × 3 × x = 105
9 × 6 – 3 = 51
x=7
7. (a) 32 + 22 + 12 = 14 18. (d) 5 + 2 = 7
42 + 32 + 22 = 29 7 + 4 = 11
52 + 42 + 32 = 50 11 + 6 = 17
8. (b) 8 × 5 – 12 = 28 19. (a) 594 198 = 3
9 × 4 – 25 = 11 198 66 = 3
10 × 3 – 14 = 16 66 x = 3
9. (b) 42 + 12 + 22 + 32 = 30 x = 22
62 + 32 + 42 + 15 = 286 20. (b) 6 × 8 + 3 = 51,
32 + 42 + x + 122 = 218 15 × 4 + 5 = 65,
169 + x = 218 20 × 5 + 20 = 120
21. (b) 7 × 2 + 1 = 15
x = 218 – 169 = 49
15 × 2 + 1 = 31
10. (b) 23 + 5 = 28
31 × 2 + 1 = 63
45 + 5 = 50 63 × 2 + 1 = 127
35 + 5 = 40 127 × 2 + 1 = 255
y
o
u
A-66 Number Puzzles

rs
32. (c) The number inside the triangle is obtained by dividing

m
22. (d) 25 + 27 = 52

a
23 + 30 = 53 the product of the numbers outside of the triangle by

h
10. Thus,

b
21 + 33 = 54

o
In I triangle, (5 × 6 × 4) 10 = 12

o
36 + x = 55

b
In II triangle, (6 × 7 × 5) 10 = 21

.w
x = 19
In III triangle, missing number

o
23. (b) 12 × 16 + 5 = 197

rd
16 × 16 + 7 = 263 = (4 × 8 × 10) 10 = 32.

p
33. (d) In fig. (A), 93 – (27 + 63) = 3

re
18 × 20 + x = 356

s
In fig, (B), 79 – (38 + 37) = 4

s
x = –4

.c
24. (c) 5 × 4 + 3 × 1 = 23 In fig. (C), missing number

o
m
7 × 6 + 3 × 4 = 54 = 67 – (16 + 42) = 9.
11 × 2 + 9 × x = 40 34. (c) The number inside the circle is equal to the difference
x=2 between the sum of the numbers at the extremities of
the horizontal diameter and the sum of numbers at the
25. (c) extremities of the vertical diameter.
1 8 13 12
In fig. (A), (5 + 6) – (7 + 4) = 0
14 11 2 7
In fig. (B), (7 + 6) – (8 + 4) = 1.
4 5 16 9
In fig. (C) missing number
15 10 3 6 = (11 + 2) – (0 + 2) = 11
35. (b) In fig. (A),
26. (a) (101 + 15) – (35 + 43) = 116 – 78 = 38.
In fig. (B), Missing number
= (48 + 184) – (56 + 34) = 232 – 90 = 142.
36. (d) The sum of numbers at the extremities of the three
line segments in each figure is same.
27. (c) This is a multiplication magic square. The product of
In fig. (A), 39 + 33 = 29 + 43 = 27 + 45 = 72
each set of three numbers in any column or row is
the constant 120. In fig. (B), 42 + 31 = 29 + 44 = 30 + 43 = 73
Let the missing number in fig. (C) be x.
3 5
28. (b) In the first row, 6 9, 6 15 Then, x + 10 = 59 + 20 = 40 + 39 = 79
2 2 or x = 69.
3
In the second row, 8 12 , 8 5 20 . 37. (d) When we arrange the digits on the vertices of every
2 2 triangle separately, starting from upper vertices,
In the third row, missing number we get numbers 196, 256 and 484 respectively.
5
= 4 10 196 256
2 Now, 7, 8
2 2
24
29. (a) In the first row, 72 ÷ = 72 ÷12 = 6 484
2 Hence, 11
2
16 38. (c) The letter in the second column is three steps behind
In the second row, 96 ÷ = 96 ÷ 8 = 12 that in the first column, and the letter in the third
2
column is four steps behind that in the second
Let the missing number in the third row be x. Then, column. So, the missing letter in the first row will be
x x 108 three steps behind Z, which is W. The missing letter
108 18 6 x = 12. in the second row will be four steps behind O, which
2 2 18
is K. The missing letter in the third row will be three
30. (d) Clearly, (5 – 4)3 = 1;
steps ahead of G, which is J.
(7 – 3)3 = 64; (11 – 8)3 = 27.
39. (c) The letters in the first row follow the sequence + 5,
So, missing number = (8 – 2)3 = 63 = 216. + 7.
31. (b) The given figure contains numbers 1 to 6 in three The letters in the second row follow the sequence +
alternate segments, the smaller number being towards 6, + 8.
the outside and the numbers 14 to 19 in the remaining
In the third row, the first letter G moves 7 steps forward
three alternate segments with the smaller number
to give the second letter N. Clearly, the missing letter
towards the inside.
will be 9 steps ahead of N i.e. W.
y
o
u
Number Puzzles A-67

rs
m
40. (d) Clearly, (1st row)3 + (2nd row)3 + (3rd row)3 = 4th row 46. (d) 7 8 3 168

a
So, in the first column,

h
6 4 4 144

b
23 + 13 + 33 = 8 + 1 + 27 = 36

o
6 x 5 120

o
b
In the third column,

.w
30 x 120
03 + 43 + 33 = 0 + 64 + 27 = 91

o
120

rd
In the second column, missing number = 43 + 23 + 13 x 4

p
30
= 64 + 8 +1 = 73

re
47. (a) 82 32 12 74

s
41. (d) In fig (A), 62 = 36, 82 = 64, 52 = 25, 72 = 49 and 6 + 8 +5

s
.c
+ 7 = 26 54 72 42 90

o
m
In fig (B), 32 = 9, 52 = 25, 42 = 16, 92 = 81. and 3 + 5 + 4 2 2 2
6 5 2 65
+ 9 = 21
48. (b) 46 – 22 = 24
In fig (C), 52 = 25, 122 = 144, 62 = 36, 82 = 64.
58 – 27 = 31
So, missing number = 5 + 12 + 6 + 8 = 31.
68 – 32 = 36
42. (c) In fig (A), (3 × 3) + (6 × 5) = 39
In fig (B), (4× 4) + (5 × 7) = 51 8 16 32 64
49. (a) 12 20 36 68 132
In fig (C), missing number 128
260
= (3 × 4) + (5 × 5) = 37
50. (b) 7 + 22 + 5 = 16
43. (b) Clearly, we have : 9 + 42 + 7 = 32
7 × 2 + 2 = 16; 16 ×2 + 2 = 34 and so on.
8 + 32 + 6 = 23
so missing number = 34 ×2 + 2 = 70
51. (c) 3+9–5=7
44. (b) The numbers in the right half form the series : 2, 3, 4, 5. 2+8–6=4
The numbers in the left half form the series : 5, 7, 9, 11. 4 + 7 –5 = 6
y
o
u
A-68 Logical Sequence of Words

rs
m
a
CHAPTER

h
LOGICAL SEQUENCE

b
9

o
o
b
.w
OF WORDS

o
rd
p
re
s
s
.c
SEQUENCE OF WORDS : Sol. (a) Clearly illness occurs first. One then goes to a doctor

o
m
In this particular type of problems, certain inter-related words are and after consultation with him, undergoes treatment
given and numbered, followed by various sequences of the num- to finally attain recovery.
bers denoting them, as alternatives. The candidate is required to So, the correct order becomes 2, 3, 1, 4, 5... (a)
arrange these words in a logical sequence based on a common FORMATION OF WORDS :
property and then choose the correctly graded sequence from In these types of questions, certain words are given. The candidate
the given data is required to observe them in the order in which they are asked.
I. Sequence of occurrence of events or various stages in a
process EXAMPLE 2. Select the combination of numbers so that
letters arranged accordingly will form a meaningful word.
EXAMPLE 1. Arrange the following in a meaningful sequence R A C E T
1. Consultation 2. Illness 1 2 3 4 5
3. Doctor 4. Treatment (a) 1, 2, 3, 4, 5 (b) 3, 2, 1, 4, 5
5. Recovery (c) 5, 2, 3, 4, 1 (d) 5, 1, 2, 3, 4
(a) 2 3 1 4 5 (b) 2 3 4 1 5 Sol. (d) Clearly, the given letters, when arranged in the order 5, 1, 2,
3, 4 from the word ‘TRACE’.
(c) 4 3 1 2 5 (d) 5 1 4 3 2

DIRECTIONS (Qs. 1-12): Arrange the following in a logical order: 6. 1. Amoeba 2. Oyster
1. 1. Birth 2. Death 3. Worm 4. Cow
3. Funeral 4. Marriage (a) 1, 3, 2, 4 (b) 1, 2, 3, 4
5. Education (c) 4, 3, 2, 1 (d) 3, 2, 4, 1
(a) 1, 3, 4, 5, 2 (b) 1, 5, 4, 2, 3
7. 1. Conference 2. Registration
(c) 2, 3, 4, 5, 1 (d) 4, 5, 3, 1, 2
2. 1. Police 2. Punishment 3. Participate 4. Invitation
3. Crime 4. Justice 5. Representative
5. Judgement (a) 1, 2, 4, 5, 3 (b) 1, 4, 5, 2, 3
(a) 1, 2, 3, 4, 5 (b) 3, 1, 2, 4, 5 (c) 4, 1, 5, 2, 3 (d) 4, 5, 1, 3, 2
(c) 3, 1, 4, 5, 2 (d) 5, 4, 3, 2, 1 8. 1. Doctor 2. Fever
3. 1. Foetus 2. Child 3. Prescribe 4. Diagnose
3. Baby 4. Adult
5. Medicine
5. Youth
(a) 1, 2, 4, 3, 5 (b) 1, 3, 2, 4, 5 (a) 2, 1, 3, 4, 5 (b) 1, 4, 3, 2, 5
(c) 2, 3, 5, 4, 1 (d) 5, 4, 2, 3, 1 (c) 2, 1, 4, 3, 5 (d) 2, 4, 3, 5, 1
4. 1. Probation 2. Interview 9. 1. Line 2. Angle
3. Selection 4. Appointment 3. Square 4. Triangle
5. Advertisement 6. Application (a) 2, 1, 4, 3 (b) 3, 4, 1, 2
(a) 5 6 2 3 4 1 (b) 5 6 3 2 4 1 (c) 4, 2, 1, 3 (d) 1, 2, 4, 3
(c) 5 6 4 2 3 1 (d) 6 5 4 2 3 1 10. 1. Childhood 2. Adulthood
5. 1. Ocean 2. Rivulet
3. Infancy 4. Adolescence
3. Sea 4. Glacier
5. River 5. Babyhood
(a) 5, 2, 3, 1, 4 (b) 4, 2, 5, 3, 1 (a) 4, 1, 3, 2, 5 (b) 3, 5, 1, 4, 2
(c) 5, 2, 3, 4, 1 (d) 4, 2, 1, 3, 5 (c) 2, 5, 1, 4, 3 (d) 5, 4, 2, 3, 1
y
o
u
Logical Sequence of Words A-69

rs
m
11. 1. Books 2. Words 23. INVESTIGATION

a
3. Letters 4. Sentences (a) INSTIGATION (b) GESTATION

h
b
5. Chapter 6. Pages (c) VEST (d) STIGMA

o
(a) 3, 2, 6, 5, 4, 1 (b) 3, 2, 5, 4, 6, 1 24. SANCTION

o
b
(c) 3, 2, 4, 5, 6, 1 (d) 3, 2, 4, 6, 5, 1 (a) ACTION (b) NATION

.w
12. 1. Millenium 2. Diamond Jubilee (c) NOTION (d) NION

o
rd
3. Silver Jubilee 4. Centenary 25. RESEARCH

p
5. Golden Jubilee (a) SEARCH (b) REACH

re
(a) 2, 3, 5, 4, 1 (b) 2, 5, 3, 1, 4 (c) HEAR (d) READ

s
s
(c) 3, 5, 2, 4, 1 (d) 2, 3, 5, 1, 4

.c
DIRECTIONS (Qs. 13-16) : Arrange the following words as per DIRECTIONS (Qs. 26-30) : In each of the following questions, a

o
group of letters is given which are numbered 1, 2, 3, 4, 5 and 6.

m
order in the dictionary
13. 1. Noble 2. Nobilitary Below a re given four alternatives containing combinations of
these numbers. Select that combination of numbers so that letters
3. Noblesses 4. Nobility
arranged accordingly form a meaningful word.
5. Nobble
(a) 1, 4, 3, 2, 5 (b) 3, 4, 1, 2, 5 26. I N LA S G
(c) 5, 2, 4, 1, 3 (d) 2, 4, 3, 5, 1 123456
14. 1. Pick 2. Pith (a) 6, 1, 3, 5, 4, 2 (b) 5, 1, 6, 2, 4, 3
3. Pile 4. Perk (c) 3, 4, 6, 1, 2, 5 (d) 2, 4, 3, 6, 1, 5
5. Pour 27. T LPNAE
(a) 4, 1, 2, 3, 5 (b) 4, 1, 3, 2, 5 1 23 4 56
(c) 4, 3, 2, 1, 5 (d) 5, 4, 3, 2, 1 (a) 3, 2, 5, 4, 6, 1 (b) 3, 2, 5, 4, 1, 6
15. 1. Infricate 2. Interview (c) 4, 5, 3, 6, 2, 1 (d) 4, 6, 1, 3, 5, 2
3. Intransigent 4. Interrogation 28. RMN BUE
5. Intravenous 1 2 34 56
(a) 2, 4, 5, 3, 1 (b) 5, 3, 1, 2, 4 (a) 2, 6, 3, 4, 1, 5 (b) 4, 6, 3, 2, 1, 5
(c) 1, 4, 2, 3, 5 (d) 3, 5, 2, 1, 4 (c) 3, 5, 2, 4, 6, 1 (d) 1, 5, 4, 2, 6, 3
16. 1. Preposition 2. Preparatively 29. E HRAS P
3. Preposterous 4. Preponderate 1 2 34 56
5. Prepossess (a) 5, 2, 4, 6, 1, 3 (b) 6, 2, 3, 4, 5, 1
(a) 2, 4, 1, 5, 3 (b) 1, 5, 2, 4, 3 (c) 2, 4, 6, 1, 3, 5 (d) 3, 4, 2, 1, 6, 5
(c) 5, 4, 2, 3, 1 (d) 4, 2, 5, 1, 3 30. AMDENR
17. Arrange the following words in their descending order: 1 2345 6
1. Weekly 2. Bi-annual (a) 2, 1, 3, 5, 4, 6 (b) 6, 4, 2, 1, 5, 3
3. Fortnightly 4. Monthly (c) 3, 4, 5, 2, 1, 6 (d) 1, 6, 2, 4, 5, 3
5. Annual 31. If with the third, fourth, fifth, seventh and tenth letters of
(a) 1, 3, 4, 2, 5 (b) 2, 5, 4, 1, 3 the word ‘PERSONALITY’, a meaningful word is formed,
(c) 4, 1, 2, 3, 5 (d) 5, 2, 4, 3, 1 then first letter of the word is the answer. If no word is
possible then X is the answer.
DIRECTIONS (Qs. 18-25): From the given alternatives select
(a) O (b) T
the word which cannot be formed using the letters of the given
(c) R (d) S
word.
32. If by arranging the letters of the word NABMODINT, the
18. CARPENTER name of a game is formed, what are the first and the last
(a) NECTAR (b) CARPET letters of the word so formed?
(c) PAINTER (d) REPENT (a) B, T (b) B, N
19. TRIBUNAL (c) N, D (d) M, T
(a) LATIN (b) BRAIN 33. Which one of the given responses would be a meaningful
(c) URBAN (d) TRIBLE order of the following ?
20. ORGANISATION 1. apartment 2. town
(a) NATION (b) GRANT 3. street 4. building
(c) RECOGNISE (d) SATAN 5. complex
21. DEPARTMENT (a) 1, 5, 4, 3, 2 (b) 4, 5, 3, 2, 1
(a) ENTER (b) PERMIT (c) 2, 1, 3, 4, 5 (d) 1, 4, 5, 3, 2
(c) TEMPER (d) RENTED 34. If the following words are arranged in reverse dictionary
22. CONSTITUTIONAL order, which word comes second ?
(a) LOCATION (b) TUTION (a) Explosion (b) Express
(c) TALENT (d) CONSULT (c) Exploit (d) Expulse
y
o
u
A-70 Logical Sequence of Words

rs
m
35. A group of alphabets are given with each being assigned a 1. Orange 2. Indigo 3. Red 4. Blue 5. Green 6. Yellow 7. Violet

a
number. These have to be unscrambled into a meaningful (a) 7, 2, 4, 5, 6, 1, 3 (b) 7, 2, 4, 6, 5, 1, 3

h
word and correct order of letters may be indicated from the

b
(c) 7, 2, 6, 4, 5, 1, 3 (d) 7, 2, 6, 4, 1, 5, 3

o
given responses.

o
39. Arrange the following words as per order in the dictionary.

b
TMHREO

.w
1. Forecast 2. Forget 3. Foreign 4. Forsook 5. Force
5 4321 0

o
(a) 3, 5, 1, 2, 4 (b) 5, 1, 3, 2, 4
(a) 025314 (b) 315402

rd
(c) 405312 (d) 504231 (c) 5, 1, 3, 4, 2 (d) 5, 1, 2, 3, 4

p
re
36. From the given alternative words, select the word which 40. From the given alternatives select the word which can be

s
cannot be formed using the letters of the given word : formed using the letters given in the word.

s
.c
TRIVANDRUM ULTRANATIONALISM

o
m
(a) RAIN (b) DRUM (a) ULTRAMONTANE (b) ULTRAMODERN
(c) TRAIN (d) DRUK (c) ULTRAIST (d) ULULATE
37. How many meaningful English words can be made with the 41. From the given alternatives select the word which cannot
letters ' OEHM ' using each letter only once in each word ? be formed using the letters of the given word.
(a) FOUR (b) THREE LEGALIZATION
(c) TWO (d) ONE
(a) ALERT (b) ALEGATION
38. Which one of the given responses would be a meaningful
order of the following ? (c) GALLANT (d) NATAL

ANSWER KEY
1 (b) 6 (b) 11 (d) 16 (a) 21 (b) 26 (b) 31 (c) 36 (d) 41 (a)
2 (c) 7 (a) 12 (c) 17 (d) 22 (c) 27 (a) 32 (b) 37 (d)
3 (b) 8 (c) 13 (c) 18 (c) 23 (d) 28 (c) 33 (d) 38 (a)
4 (a) 9 (d) 14 (b) 19 (d) 24 (c) 29 (b) 34 (b) 39 (b)
5 (b) 10 (b) 15 (c) 20 (c) 25 (d) 30 (b) 35 (c) 40 (c)

1. (b) Clearly, the given words when arranged in the order of 6. (b) 1. Amoeba
various events as they occur in man’s life, term the 2. Oyster
sequence: Birth – Education – Marriage – Death – 3. Worm
Funeral. 4. Cow
So the correct order becomes 1 5 4 2 3 7. (a) 1. Conference
2. (c) The correct sequence 2. Registration
Crime – Police – Justice – Judgement – Punishment 4. Invitation
So sequence will 3, 1, 4, 5, 2. 5. Representative
3. (b) The correct sequence is 3. Participate
Foetus – Baby – Child – Adult – Youth 8. (c) Correct sequence is :
4. (a) The correct sequence is–
Advertisement – Application – Interview – Selection – Fever Doctor Diagnose Prescribe Medicine
Appointment – Probation. (2) (1) (4) (3) (5)
So, correct option is (a). 9. (d) Line < Angle < Triangle < Square
5. (b) Glacier 1, 2, 4, 3
10. (b) 3, 5, 1, 4, 2
Rivulet 11. (d) Letters Words Sentences Pages Chapter Book
(3) (2) (4) (6) (5) (1)
River 12. (c) Silver jublee - 25 yr.
Golden jublee - 50 yr.
Sea Diamond jublee - 75 yr.
Centenary - 100 yr
Ocean Millennium - 1000 yr.
y
o
u
Logical Sequence of Words A-71

rs
m
13. (c) 5. Nobble 28. (c) NUMBER

a
2. Nobilitary 29. (b) PHRASE

h
4. Nobility 30. (b) REMAND

b
o
1. Noble 31. (c) The respective letters of the given word are R, S, O, A

o
b
3. Noblesses and T. The word formed is ROAST. So the first letter is

.w
14. (b) Perk > Pick > Pile > Pith > Pour R.

o
15. (c) 1. Infricate 32. (b) The name of the game is BADMINTON.

rd
4 Interrogation

p
re
2. Interview 33. (d) 2. Town

s
3. Street
3. Intransigent

s
.c
5. Complex
5. Intravenous

o
4. Building

m
16. (a) 2. Preparatively
4. Preponderate 1. Apartment
1. Preposition
5. Prepossess
3. Preposterous
17. (d) Annual - 1 year Monthly - 30 days
Weekly - 7 days Biannual - 6 month
Fortnightly - 15 days.
18. (c) Painter cannot be made from the letters of the given 34. (b) Arrangement in Reverse dictionary order—
word as I letter is not mentioned in the given word. Expulse Express Explosion Exploit
19. (d) Trible cannot be made from the letters of the given 1 2 3 4
word as E letter is not mentioned in the given word. 4 0 5 3 1 2
20. (c) Recognise cannot be made from the letters of the given 35. (c)
word as C letter is not mentioned in the given word. M O T H E R
21. (b) Permit cannot be made from the letters of the given 36. (d) DRUK cannot be formed using TRIVAN DROM as it
word as I letter is not mentioned in the given word. does not contain letter 'K'.
22. (c) Talent cannot be made from the letters of the given 37. (d) Home, only one meaningful word is formed.
word as E letter is not mentioned in the given word. 38. (a) These all are colours of the rainbow.
23. (d) STIGMA cannot be formed usin g word Hence, meaningful order is VIBGYOR.
'INVESTIGATION' because letter M is not in the given 39. (b) Force Forecast Foreign Forget Forsook
reference word. 40. (c) By options,
24. (c) NOTION cannot be formed because two Os are not in (a) can not be formed as there is no ‘E’ in the given word.
the reference word. (b) can not be formed as there is no ‘D’ in the given word.
25. (d) READ cannot be fomed as letter D is not in the reference (d) can not be formed as there is no ‘E’ and only ‘U’ in the
word. given word.
26. (b) SIGNAL 41. (a) ALERT can not be formed as there is no ‘R’ in the word
27. (a) PLANET LEGALIZATION. Hence, (a) is the correct option.
y
o
u
A-72 Clock and Calendar

rs
m
a
CHAPTER
CLOCK AND

h
b
10

o
o
b
.w
CALENDAR

o
rd
p
re
s
s
.c
CLOCK • In every 12 hours, the hands of clock are at right angles

o
m
22 times.
A clock has two hands : Hour hand and Minute hand.
• In every hour, the two hands are at right angles 2 times.
The minute hand (M.H.) is also called the long hand and the hour
• In every hour, the two hands are in opposite direction
hand (H.H.) is also called the short hand. once.
The clock has 12 hours numbered from 1 to 12. • In a day, the two hands are at right angles 44 times.
Also, the clock is divided into 60 equal minute divisions. Therefore, • If both the hands coincide, then they will again coincide
each hour number is separated by five minute divisions. Therefore,
5
Important Points – after 65 minutes. i.e. in correct clock, both hand
11
360
One minute division = = 6° apart. i.e. In one minute, the 5
60 coincide at an interval of 65 minutes.
minute hand moves 6°. 11
One hour division = 6° × 5 = 30° apart. i.e. In one hour, the 5
hour hand moves 30° apart. • If the two hands coincide in time less than 65
11
30 1 minutes, then clock is too fast and if the two hands
Also, in one minute, the hour hand moves = = apart.
60 2 5
coincides in time more than 65 minutes, then the
Since, in one minute, minute hand moves 6° and hour hand 11
clock is too slow.
1
moves , therefore, in one minute, the minute hand gains
2 NOTE :
1 ANOTHER SHORT-CUT FORMULA FOR CLOCKS
5 more than hour hand.
2 11
Angle made by Hands = 30H M
1 2
In one hour, the minute hand gains 5 60 330 over the
2 where H = Hour
hour hand. i.e. the minute hand gains 55 minutes divisions M = minute
over the hour hand.
EXAMPLE 1.
Relative position of the hands – At what time between 4 and 5 O’ Clock will the hands of a
The position of the M.H. relative to the H.H. is said to be the watch
same, whenever the M.H. is separated from the H.H. by the same (i) coincide, and
(ii) point in opposite directions.
number of minute divisions and is on same side (clockwise or
Sol.
anticlockwise) of the H.H.
(i) At 4 O’ clock, the hands are 20 minutes apart. Clearly the
Any relative position of the hands of a clock is repeated 11 times
minute hand must gain 20 minutes before two hands can be
in every 12 hours. coincident.
(a) When both hands are 15 minute spaces apart, they are at But the minute-hand gains 55 minutes in 60 minutes.
right angle. Let minute hand will gain x minute in 20 minutes.
(b) When they are 30 minute spaces apart, they point in opposite
directions. 55 60
So, =
(c) The hands are in the same straight line when they are 20 x
coincident or opposite to each other. 20 × 60 240 9
• In every hour, both the hand coincide once. x= = = 21 min.
55 11 11
• In a day, the hands are coinciding 22 times.
• In every 12 hours, the hands of clock coincide 11 times. 9
The hands will be together at 21 min past 4.
• In every 12 hours, the hands of clock are in opposite 11
direction 11 times.
y
o
u
Clock and Calendar A-73

rs
m
(ii) Hands will be opposite to each other when there is a space But, they are together after 65 min.

a
of 30 minutes between them. This will happen when the

h
5 5
minute hand gains (20 + 30) = 50 minutes.

b
Gain in 65 min 65 65 min .

o
11 11

o
50 × 60 6

b
Now, the minute hand gains 50 min in or 54 min. 5 60 24 10

.w
55 11 Gain in 24 hours min 10 min .
11 65 143

o
rd
6
The hands are opposite to each other at 54 min past 4. 10
The clock gains 10 minutes in 24 hours.

p
11

re
143

s
2.

s
EXAMPLE 5.

.c
EXAMPLE
What is the angle between the hour hand and minute hand

o
A man who went out between 5 or 6 and returned between 6

m
when it was 5 : 05 pm. and 7 found that the hands of the watch had exactly changed
Sol. 5.05 pm means hour hand was on 5 and minute hand was on place. When did he go out?
1, i.e. there will be 20 minutes gap.
Sol. Between 5 and 6 to 6 and 7, hands will change place after
Angle = 20 × 6° = 120° [ 1 minute = 6°] crossing each other one time. ie. they together will make 1 +
INCORRECT CLOCK 1 = 2 complete revolutions.
If a clock indicates 6 : 10, when the correct time is 6 : 00, it is
said to be 10 minute too fast and if it indicates 5 : 50 when 60 120
H.H. will move through 2 or minute divisions.
the correct time is 6 : 00, it is said to be 10 minute too slow. 13 13
• Also, if both hands coincide at an interval x minutes
120
5 Between 5 and 6 minute divisions.
and x 65 , then total time gained 13
11 At 5, minute hand is 25 minute divisions behind the hour-
5 hand.
65 x
11 minutes and clock is said to be ‘fast’. 120
x Hence it will have to gain 25 minute divisions on the
13
• If both hands coincide at an interval x minutes and
445
hour-hand minute divisions on the hour hand.
5 13
5 x 65
x 65 , then total time lost 11 minutes 445 445 12
11 x The minute hand gains minute divisions in
13 13 11
and clock is said to be ‘slow’. 5340 49
minutes 37 minutes
EXAMPLE 3. 143 143
My watch, which gains uniformly, is 2 min slow at noon on 49
The required time of departure is 37 min past 5.
Sunday, and is 4 minutes 48 seconds fast at 2 pm on the 143
following Sunday. When was it correct.
Sol. From Sunday noon to the following Sunday at 2 pm = 7 days CALENDAR
2 hours = 170 hours. The solar year consists of 365 days, 5 hrs 48 minutes, 48 seconds.
48 4 In 47 BC, Julius Ceasar arranged a calendar known as the Julian
The watch gains 2 4 = 6 minutes in 170 hours. 1
60 5 calendar in which a year was taken as 365 days and in order to
4
2 get rid of the odd quarter of a day, an extra day was added once in
The watch gains 2 minutes in 170 50 hours
4 every fourth year and this was called as leap year or Bissextile.
6
5 Nowadays, the calendar, which is mostly used, is arranged by
Now, 50 hours = 2 days 2 hours Pope Gregory XII and known as Gregorian calendar.
2 days 2 hours from Sunday noon = 2 pm on Tuesday. In India, number of calendars were being used till recently. In
1952, the Government adopted the National Calendar based on
EXAMPLE 4. Saka era with Chaitra as its first month. In an ordinary year, Chaitra
The minute hand of a clock overtakes the hour hand at 1 falling on March 22 of Gregorian Calendar and in a leap year it
intervals of 65 minutes of the correct time. How much a day falls on March 21.
does the clock gain or lose?
Sol. In a correct clock, the minute hand gains 55 min. spaces
over the hour hand in 60 minutes. In an ordinary year,
To be together again, the minute hand must gain 60 minutes 1 year = 365 days = 52 weeks + 1 day
over the hour hand. In a leap year,
60 5 1 year = 366 days = 52 weeks + 2 days
55 min. are gained in 60 min 65 min .
55 11
y
o
u
A-74 Clock and Calendar

rs
m
NOTE : First January 1 A.D. was Monday. So we must count EXAMPLE 7.

a
days from Sunday.

h
How many times does the 29th day of the month occur in

b
100 years or one century contains 76 ordinary years and 24 400 consecutive years?

o
o
leap years.
Sol. In 400 consecutive years, there are 97 leap years. Hence,

b
[76 × 52 weeks + 76 odd days]

.w
in 400 consecutive years, February has the 29th day 97 times
+ [24 × 52 weeks + 24 × 2 odd days]

o
and the remaining eleven months have the 29th day

rd
= (76 + 24) × 52 weeks + (76 + 48) odd days 400 × 1100 = 4400 times

p
The 29th day of the month occurs (4400 + 97) or 4497

re
= 100 × 52 weeks + 124 odd days

s
= 100 × 52 weeks + (17 × 7 + 5) odd days times.

s
.c
= (100 × 52 + 17) weeks + 5 odd days

o
EXAMPLE 8.

m
100 years contain 5 odd days.
Today is 5th February. The day of the week is Tuesday. This
Similarly, 200 years contain 3 odd days,
is a leap year. What will be the day of the week on this date
300 years contain 1 odd days, after 5 years?
400 years contain 0 odd days. Sol. This is a leap year. So, next 3 years will give one odd day
Year whose non-zero numbers are multiple of 4 contains no each. Then leap year gives 2 odd days and then again next
odd days; like 800, 1200, 1600 etc. year give 1 odd day.
The number of odd days in months Therefore (3 + 2 + 1) = 6 odd days will be there.
The month with 31 days contains (4 × 7 + 3) i.e. 3 odd days Hence the day of the week will be 6 odd days beyond
and the month with 30 days contains (4 × 7 + 2) i.e. 2 odd Tuesday, i.e., it will be Monday.
days.
EXAMPLE 9.
NOTE : What day of the week was 20th June 1837 ?
• February in an ordinary year gives no odd days, but in a leap Sol. 20th June 1837 means 1836 complete years + first 5 months
year gives one odd day. of the year 1837 + 20 days of June.
1600 years give no odd days.
• Day of the week related to ODD days –
200 years give 3 odd days.
No. of Days 0 1 2 3 4 5 6 36 years give (36 + 9) or 3 odd days.
Days Sun Mon Tues Wed Thur Fri Sat 1836 years give 6 odd days.
From 1st January to 20th June there are 3 odd days.
Odd days :
EXAMPLE 6. January :3
What day of the week was 15th August 1949? February : 0
Sol. 15th August 1949 means March :3
1948 complete years + first 7 months of the year 1949 April :2
+ 15 days of August. May :3
1600 years give no odd days. June :6
---------
300 years give 1 odd day. 17
48 years give {48 + 12} = 60 = 4 odd days. Therefore, the total number of odd days = (6 + 3) or 2 odd
[ For ordinary years 48 odd days and for leap year 1 days.
more day (48 ÷ 4) = 12 odd days; 60 = 7 × 8 + 4] This means that the 20th of June fell on the 2 nd day
From 1st January to 15th August 1949 commencing from Monday. Therefore, the required day was
Odd days : Tuesday.
January – 3
EXAMPLE 10.
Feburary – 0
Prove that the calender for 1990 will same for 2001 also.
March – 3
Sol. It is clear that the calender for 1990 will serve for 2001 if first
April – 2 January of both the years is the same weekdays. For that the
May – 3 number of odd days between 31st December 1989 and 31st
June – 2 December 2000 must be zero. Odd days are as given below.
July – 3 Year 1990 1991 1992 1993 1994
August – 1 Odd days 1 1 (Leap) 1 1
17 3 odd days. 2
15th August 1949 1 + 4 + 3 = 8 = 1 odd day. 1995 1996 1997 1998 1999 2000
This means that 15th Aug. fell on 1st day. Therefore, the 1 (Leap) 1 1 1 (Leap)
required day was Monday. 2 2
Total number of odd days = 14 days = 2 weeks + odd days.
y
o
u
Clock and Calendar A-75

rs
m
a
h
b
o
o
b
.w
1. If the two hands in a clock are 3 minutes divisions apart, 12. A watch which gains uniformly is 2 minutes low at noon on

o
then the angle between them is Monday and is 4 min 48 sec. fast at 2 p.m. on the following

rd
(a) 3° (b) 18° (c) 24° (d) 60° Monday. When was it correct?

p
re
2. At what approximate time between 4 and 5 am will the (a) 2 p.m. on Tuesday (b) 2 p.m. on Wednesday

s
hands of a clock be at right angle? (c) 3 p.m. on Thursday (d) 1 p.m. on Friday

s
.c
(a) 4 : 40 am (b) 4 : 38 am 13. If a clock strikes 12 in 33 seconds, it will strike 6 in how many

o
(c) 4 : 35 am (d) 4 : 39 am seconds?

m
3. What will be the acute angle between hands of a clock
33
at 2 : 30? (a) (b) 15 (c) 1 2 (d) 2 2
(a) 105° (b) 115° (c) 95° (d) 135° 2
4. In 16 minutes, the minute hand gains over the hour 14. A watch which gains 5 seconds in 3 minutes was set right at
hand by 7 a.m. In the afternoon of the same day, when the watch
(a) 16° (b) 80° (c) 88° (d) 96° indicated quarter past 4 O’clock, the true time is
5. A clock is set right at 1 p.m. If it gains one minute in an hour, 7
then what is the true time when the clock indicates 6 p.m. in (a) 4 p.m. (b) 59 minutes past 3
12
the same day?
5 7 3
(a) 55 minutes past 5 (b) 5 minutes past 6 (c) 58 minutes past 3 (d) 2 minutes past 4
61 11 11
1 15. At what time between 8 and 9 o’clock will the hands of a
(c) 5 minutes to 6 (d) 59 minutes past 5 watch be in straight line but not together?
64
6. Two clocks were set right at noon on Sunday. One 11 10
(a) 10 min. past 8 (b) 10 min. past 8
gains 2 min and the other loses 3 min in 24 hours. 10 11
What will be the true time when the first clock
indicates 3 pm on Wednesday? 10 10
(c) 11 min. past 8 (d) 12 min. past 8
(a) 2:38 pm (b) 2:54 pm (c) 2:23 pm (d) 2:48 pm 11 11
7. At what time between 9’O clock and 10’O clock will the 16. At what time between 5.30 and 6 will the hands of a clock be
hands of a clock point in the opposite directions? at right angles?
4 4
(a) 16 minutes past 9 (b) 16 minutes past 8 5 7
11 11 (a) 43 min. past 5 (b) 43 min. past 5
11 11
5 5
(c) 55 minutes past 7 (d) 55 minutes to 8 (c) 40 min. past 5 (d) 45 min. past 5
61 61 17. Find the angle between the hour hand and the minute hand
8. A clock gains 15 minutes per day. It is set right at 12 noon. of a clock when the time is 3.25.
What time will it show at 4.00 am, the next day?
(a) 4 : 10 am (b) 4 : 45 am 1 1
(c) 4 : 20 am (d) 5 : 00 am (a) 45° (b) 37 (c) 47 (d) 46°
2 2
9. Find the exact time between 7 am and 8 am when the two
18. How much does a watch lose per day, if its hands coincide
hands of a watch meet ?
every 64 minutes?
(a) 7 hrs 35 min (b) 7 hrs 36.99 min
(c) 7 hrs 38.18 min (d) 7 hrs 42.6 min 8 5
(a) 32 min. (b) 36 min.
10. In a watch, the minute hand crosses the hour hand for the 11 11
third time exactly after every 3 hrs., 18 min., 15 seconds of (c) 90 min. (d) 96 min.
watch time. What is the time gained or lost by this watch in 19. An accurate clock shows 8 O’clock in the morning. Through
one day? how many degrees will the hour hand rotate when the clock
(a) 14 min. 10 seconds lost shows 2 O’clock in the afternoon?
(b) 13 min. 50 seconds lost (a) 144° (b) 150° (c) 168° (d) 180°
(c) 13min. 20 seconds gained 20. A clock is set right at 5 a.m. The clock loses 16 min. in 24
(d) 14 min. 40 seconds gained hours. What will be the true time when the clock indicates
11. At what time between 3 and 4’O clock, the hands of a clock 10 p.m. on the 4th day ?
coincide? (a) 11 p.m. (b) 10 p.m. (c) 9 p.m. (d) 8 p.m.
4 5 21. The reflex angle between the hands of a clock at 10:25 is?
(a) 16 minutes past 3 (b) 15 minutes past 3
11 61
1º 1º
5 4 (a) 180º (b) 192 (c) 195º (d) 197
(c) 15 minutes to 2 (d) 16 minutes to 4 2 2
60 11
y
o
u
A-76 Clock and Calendar

rs
m
22. A clock gains 5 minutes. in 24 hours. It was set right at 10 26. If 09/12/2001 happens to be Sunday, then 09/12/1971 would

a
a.m. on Monday. What will be the true time when the clock have been at

h
indicates 10:30 a.m. on the next Sunday ? (a) Wednesday (b) Tuesday

b
o
(a) 10 a.m. (c) Saturday (d) Thursday

o
b
(b) 11 a.m. 27. Find the day of the week on 16th July, 1776.

.w
(c) 25 minutes past 10 a.m. (a) Tuesday (b) Wednesday

o
(d) 5 minutes to 11 a.m. (c) Monday (d) Thursday

rd
28. On January 12, 1980, it was Saturday. The day of the week

p
23. At what angle the hands of a clock are inclined at 15 minutes

re
on January 12, 1979 was –
past 5 ?

s
(a) Saturday (b) Friday

s
.c
1 1 1 (c) Sunday (d) Thursday
(a) 72 (b) 64° (c) 58 (d) 67

o
2 2 2 29. The year next to 1991 having the same calendar as that of

m
24. What will be the day of the week on 1st January, 2010 ? 1990 is –
(a) 1998 (b) 2001 (c) 2002 (d) 2003
(a) Friday (b) Saturday (c) Sunday (d) Monday
30. Today is Monday. After 61 days it will be :
25. The calendar for the year 2005 is the same as for the year : (a) Wednesday (b) Saturday
(a) 2010 (b) 2011 (c) 2012 (d) 2013 (c) Tuesday (d) Thursday

ANSWER KEY
1 (b) 5 (a) 9 (c) 13 (b) 17 (c) 21 (d) 25 (c) 29 (c)
2 (b) 6 (b) 10 (b) 14 (a) 18 (a) 22 (a) 26 (d) 30 (b)
3 (a) 7 (a) 11 (a) 15 (b) 19 (d) 23 (d) 27 (a)
4 (c) 8 (a) 12 (b) 16 (b) 20 (a) 24 (c) 28 (b)

1. (b) In a clock, each minute makes 6° 5. (a) Time interval indicated by incorrect clock
3 minutes will make 6 × 3 = 18° = 6 p.m – 1 p.m = 5 hrs.
2. (b) Here H × 30 = 4 × 30 = 120°. Time gained by incorrect clock in one hour
(Since initially the hour hand is at 4. H = 4). 1
Required angle A = 90° and since, H × 30 > A° so, = + 1 min = hr..
60
there will be two timings. True time interval
2 Using the formula,
Required time T = (H × 30 ± A) minutes past H. Time interval in incorrect clock
11 1
2 1 + hour gained in 1 hour by incorrect clock
One timing = (4 × 30 + 90) minutes past 4
11 True time interval 1
2 5 1
= 38 minutes past 4. 1
11 60
Or 4 : 38 approx. 5 60 56
3. (a) At 2'O Clock, Minute Hand will be 10 × 6 = 60° True time interval 4
61 61
behind the Hour Hand.
56
1 True time = 1 p.m. + 4 hrs.
In 30 minutes, Minute Hand will gain 5 × 30 61
2
= 150 + 15 = 165° 56 56
= 5 p.m. + hrs. = 5 p.m. + × 60 min.
Angle between Hour Hand and Minute Hand 61 61
= 165 – 60 = 105° 5
4. (c) In 1 hour, the minute h and gains 330º over the = 55 minutes past 5.
61
hour hand.
i.e. in 60 minute, the minute hand gains 330º over 6. (b) T i m e fr om n oon on S u n d a y t o 3 pm on
the hour hand. Wednesday = 75 hours.
In 16 minutes, the minute hand gains over the 24 hours 2 minutes of the first clock
= 24 hours of the correct one.
330º
hour hand by 16º 88º 1 hour of the first clock = 24 × (30/721) hours
60 of correct one.
y
o
u
Clock and Calendar A-77

rs
m
75 ours of the first clock 4
The watch gains 2 4 min .

a
= 24 × 30 × (75/721) hours of correct one
5

h
= 54000/721 hours = 74 hours 53.7 min.

b
34

o
Hence the answer is 2:54 pm. or min in 170 hrs.

o
5

b
7. (a) At 9’O clock, the Minute Hand is ahead of Hour Hand

.w
by 45 minutes. The hands will be opposite to each 34
Now, min are gained in 170 hrs.

o
other when there is a space of 30 minutes between 5

rd
them.

p
5

re
This will happen when the Minute Hand gains 15 2 min are gained in 170 2 hrs 50 hrs.
34

s
minutes’ space over Hour Hand.

s
Watch is correct 2 days 2 hrs. after 12 p.m. on

.c
Time taken by Minutes Hand to gain 15 minutes

o
Monday i.e. it will be correct at 2 p.m. on Wednesday.

m
1 15 4 4 13. (b) In order to strike 12, there are 11 intervals of equal time
15 1 15 15 1 16 minutes.
11 11 11 11 33
= = 3 seconds each
4 11
Hence the Hands are opposite to each other at 16 Therefore, to strike 6 it has 5 equal intervals, it requires
11
5 × 3 = 15 sec.
minutes past 9.
14. (a) Time from 7 a.m. to quarter past 4
8. (a) The clock gains 15 min in 24 hours.
= 9 hours 15 min = 555 min
Therefore, in 16 hours, it will gain 10 minutes.
Hence, the time shown by the clock will be 4.10 am. 37
Now, min of this watch = 3 min of the correct watch.
9. (c) 55 min spaces are gained in 60 min 12
35 min spaces will be gained in 38.18 min 3 12
Answer = 7 hrs + 38.18 min 555 min of this watch = 555 min
37
10. (b) In a watch than is running correct the minute hand
3 12 555
5 = hrs. 9 hrs of the correct watch.
should cross the hour hand once in every 65 min. 37 60
11
So they should ideally cross 3 times once in Correct time is 9 hours after 7 a.m. i.e., 4 p.m.
15. (b) At 8 o’clock, the hands of the watch are 20 min. spaces
720 2160 apart.
3 min = 196.36 minutes.
11 11 To be in straight line but not together they will be 30
But in the watch under consideration, they meet after min. space apart.
every 3hr, 18 min and 15 seconds, Minute hand will have to gain 10 min. spaces
55 min. spaces are gained in 60 min.
15 793 10 min. spaces will be gained in
i.e. 3 60 18 min.
60 4 60 10
50 min or 10 min .
Thus, our watch is actually losing time (as it is slower 55 11
than the normal watch). Hence when our watch elapsed
10
196.36 Required time = 10 min. past 8
1440 = 1426.27. 11
198.25 16. (b) At 5 O’clock, the hands are 25 min. spaces apart.
Hence the amount of time lost by our watch in one day To be at right angles and that too between 5.30 and 6,
the minute hand has to gain (25 + 15) = 40 min. spaces
1440 ~ 1426.27 13.73 i.e. 13 min an d 50s
55 min. spaces are gained in 60 min.
(approx).
60 7
11. (a) Since, in one hour, two hands of a clock coincide only 40 min. spaces are gained in 40 min . 43 min .
once, so, there will be value. 55 11
7
2 Required time = 43 min past 5
Required time T = (H 30 A ) minutes past H. 11
11
17. (c) Angle traced by the hour hand in 12 hours = 360°
Here H = initial position of hour hand = 3
41
(Since 3 O’clock) Angle traced by it in 3 hrs 25 min. i.e. hrs
A° = required angle = 0° (Since it coincides) 12
2 360 41 1
T= (3 30 0) minutes past 3 102 .
11 12 12 2

4 360
= 16 minutes past 3. Angle traced by it in 25 min. 25 150 .
11 60
12. (b) Time from 12 p.m. on Monday to 2 p.m. on the following
Required angle 1 1
Monday = 7 days 2 hours = 170 hours. 150 102 47 .
2 2
y
o
u
A-78 Clock and Calendar

rs
m
18. (a) 55 min spaces are covered in 60 min 23. (d) At 15 minutes past 5, the minute hand is at 3 and hour

a
60 hand slightly advanced from 5. Angle between their

h
60 min spaces are covered in 60 min 3rd and 5th position.

b
o
55 Angle through which hour hand shifts in 15 minutes is

o
b
5

.w
65 min 1 1
11 15 7

o
2 2

rd
5 16

p
Loss in 64 min 65 64 min. 1 1

re
11 11 Required angle = 60 7 67

s
2 2

s
.c
Loss in 24 hrs 16 1 8 24. (c) 2000 years have 2 odd days.
24 60 min 32 min .

o
11 64 11

m
Year 20 01 20 02 20 03 20 04 20 05 20 06 20 07 20 08 20 09
19. (d) Angle traced by the hour hand in 6 hours Odd days 1 1 1 2 1 1 1 2 1
= 11 odd days = 4 odd days.
360 1st January, 2010 has 1 odd day. Total number of odd
6 180
12 days = (2 + 4 + 1) = 7 = 0.
20. (a) Time from 5 a.m. on a day to 10 p.m. on 4th day is 89 1st January, 2010 will be Sunday.
hours. 25. (c) Count the number of days from 2005 onwards to get 0
Now, 23 hrs 44 min of this clock are the same as 24 odd day.
hours of the correct clock. Year 2005 2006 2007 2008 2010 2011
Odd days 1 1 1 2 1 1
i.e., 356 hrs. of this clock = 24 hrs. of correct clock. = 7 or 0 odd day.
15 Calendar for the year 2005 is the same as that for the
24 15 year 2012.
89 hrs. of this clock = 89 hrs. of correct 26. (d) 09/12/2001—— Sunday
356
No. of days between 9/ 12/ 71 & 9 / 12/ 2001
clock we know every year has 1 odd days
= 90 hrs of correct clock.
we know leap year has 2 odd days
So, the correct time is 11 p.m.
Here, No. of normal years = 22
125 And no. of leap years = 8
21. (d) Angle traced by hour hand in hrs.
12 So odd days = 22 + 16 = 38 i.e. 3odd days
(remainder when 38 is divided by 7, i.e. 3)
360 125 º 1º
312 Hence it was a Thursday
12 12 2 27. (a) 16th July, 1776 mean (1775 years + 6 months + 16 days)
Angle traced by minute hand in 25 min. Now, 1600 years have 0 odd days.
360 º 100 years have 5 odd days
25 150º 75 years contain 18 leap years and 57 ordinary years
12
and therefore (36 + 57) or 93 or 2 odd days.
1 º 1º 1775 years given 0 + 5 + 2 = 7 and so 0 odd days.
Reflex angle 360 312
150 197 Also number of days from 1st Jan. 1776 to 16th July,
2 2
1776
22. (a) Time between 10 a.m. on Monday to 10:30 a.m. on
Jan. Feb. March April May June July
1 31 + 29 + 31 + 30 + 31 + 30 + 16
Sunday = 144 hours. = 198 days = 28 weeks + 2 days = 2 odd days
2
Total number of odd days = 0 + 2 = 2.
1
24 hours of incorrect clock = 24 hours of correct Hence the day on 16th July, 1776 was ‘Tuesday’.
2 28. (b) The year 1979 being an ordinary year, it has 1 odd day.
time. So, the day on 12th January 1980 is one day beyond
1 on the day on 12th January, 1979.
144 hours of incorrect clock = x hours of correct But, January 12, 1980 being Saturday.
2
time. January 12, 1979 was Friday.
29. (c) We go on counting the odd days from 1991 onwards
1 till the sum is divisible by 7. The number of such days
144 24
2 are 14 upto the year 2001. So, the calendar for 1991 will
x= = 144 hours i.e.,
1 be repeated in the year 2002.
24
2 30. (b) Each day of the week is repeated after 7 days. So, after
The true time is 10 a.m. on Sunday. 63 days, it will be Monday.
After 61 days, it will be Saturday.
y
o
u
Venn Diagrams A-79

rs
m
a
CHAPTER

h
b
11

o
VENN DIAGRAMS

o
b
.w
o
rd
p
re
s
s
.c
Venn diagrams are pictorial way of represent the set of article.

o
Engineer Doctor

m
There are different regions which needs proper understanding
for solving problems based on given Venn diagrams. 2
1 5
TYPE- I
Ex:–
Case - I: 7
6 4
P Q
3

Two articles: IA IIAB IB Farmer


1 Engineer
2 Doctor
here IA represents only A 3 Farmer
IB represents only B 4 Doctor who is farmer also
IIAB represents A and B 5 Engineer who is doctor also
6 Engineer who is farmer also
7 Person who is Engineer, doctor and farmer.
2
For example : 1 EXAMPLE 1. In the following venn diagram identify the
3
number which denotes Doctor who know both swimming and
dancing.
– represents student passed in English
– represents student passed in Reasoning.
1 – represents student passed in English only.
2 – represents student passed in Reasoning only
3 – represents student passed in both English Reasoning 7 5
both.
Case: - II Three articles 6 4
2
Q 1
P

1 5 2

7 swimmers
6 4
doctors
3
dancers
R
(a) 5 (b) 3
1 – represents P only (c) 4 (d) 6
2 – represents Q only
3 – represents R only Sol. (c) Area 4 which is common to and
4 – represents Q and R (not P)
represents the required condition.
5 – represents P and Q (not R)
6 – represents P and R (not Q)
7 – represents P, Q and R
y
o
u
A-80 Venn Diagrams

rs
Note 5:- If three group of things are related to each other.

m
TYPE – II

a
Note 1- When one class of items is completely included in the

h
b
another class of item and is represented by the given diagram.

o
I II

o
b
.w
II

o
rd
I III

p
re
s
s
.c
Ex:- Graduate, Engineer and Doctor

o
Graduate may be Engineer and Doctor.

m
eg:- I – Mango Note 6: When two group of items are completely unrelated to
II – Fruit each other while they are partly related with third group of item
Here all mango are fruit. and is shown in.
Note 2: If two classes of item are completely different from each
other but they all are completely included in third class then the
relationship is represent of the diagram. I II III
III
I II Ex:- Cloth, Red, Flowers.
Some cloth are Red and also some Flowers are red.

Note 7:- When group of items are completely different from each
eg: I – represent potato other
II – represent onion
III – represent vegetable
Note 3 : If two group of items having some common relationship I II
and both of them are all included in third class then the
relationship is represented by the diagram.
III
III

I II Red, Yellow, Black


These are all different colour.

EXAMPLE 2. Diagrammatic representation of the following:


Ex:-Brother, Father, Male.
(1) Women, Doctors, Anchors
I Brother
(2) Tall men, Black haired people, Indians
II Father
(3) Mars, Solar System, Universe
III Male
(4) Girls, Boys, Students
Some Brother may be Father and all are male.
(5) Children, Men, Women
Sol.
Note 4: When one class of item is completely included in another
(1) D W
group while third is not related to both of them then such condition
are diagrammatically represented by D Doctor
W Women
A Anchors
II A
Some women are doctors and some are anchors. Among
I these women, some are both doctors and anchors.
III (2) T B
T Tall men
B Black haired people
I Indians
Ex:- Cricketer, player and farmer
I – Cricketer I
II – Player Some Indians are tall men. Some Indians have black hair.
III – Farmer Among these Indians, some Indians are tall men with black
hair.
All cricketers are players but farmers not.
y
o
u
Venn Diagrams A-81

rs
m
(3) U

a
S

h
U Universe

b
M

o
S Solar System 1

o
b
M Mars 3 6

.w
5

o
2 4

rd
Universe contains Solar System and Solar System contains 7

p
Mars.

re
s
Give the answers of the following questions :

s
(4) S

.c
o
(1) Which region represents young, uneducated and employed

m
G B
G Girls persons?
B Boys
S Students (2) Which region represents educated, employed and young
persons?
(3) Which region represents young, educated and unemployed
Girls and boys are students.
persons?
(5) C (4) Which region represents young, uneducated and
C Children unemployed persons?
W M W Women Sol.
M Men
(1) Region – 4, because this region is common to all three.
(2) Region – 2, because this region is common between young
Children, Women and Men are three different groups. persons and employed persons but not with uneducated.
Some times venn diagram can be represented by different (3) Region – 1, because this region contains only young
types of geometrical figures. persons not a part of uneducated region and employed
region.
EXAMPLE 3. ‘Triangle’ represents young persons, ‘circle’
(4) Region – 3, because this region is common between young
represents uneducated persons and ‘rectangle’ represents
persons and uneducated persons but not included in
employed persons.
employed region.

1. Which one of the following Venn diagrams represents the


best relationship between Snake, Lizard, Reptiles?
(a) (b)

(a) (b)
(c) (d)

3. Which of the following diagram represents the relationship


among sun, moon and stars?

(c) (d)

2. Which diagram correctly represents the relationship


between Human beings, Teachers, Graduates? (a) (b) (c) (d)
y
o
u
A-82 Venn Diagrams

rs
m
4. Which of the following diagram represents correct

a
relationship Judge, Thief, Criminal?

h
(a) (b)

b
o
o
b
.w
(a) (b)
(c) (d)

o
rd
p
re
10. Which of the diagram represents the given statements:-

s
Some of the players are tennis players, some tennis players

s
.c
(c) (d) are hockey players. No Cricket player is a hockey players?

o
m
5. Which diagram correctly represent the relationship between
politician, poets and women? (a) (b)

(a) (b)
(c) (d)

Directions (Qs. 11-20) : In each of these questions, three words


(c) (d) are related in some way. The relationship among the words in
question can best represents by one of the five diagram.
6. Which of the following four diagram represent correctly
the relationship between musician, instrumentalists,
violinists? (a) (b)

(a) (b)
(c) (d)

11. Cabinet, Home Minister, Minister.


(c) (d) 12. Professors, Researchers, Scientists.
13. Men, Rodents, Living beings.
7. Choose the diagram which represent the relationship among 14. Parents, Mothers, Fathers.
the following :- Capsules, Antibiotics, Injection. 15. Nitrogen, Ice, Air.
16. Musicians, Singers, Women
(a) (b) 17. Elephants, Carnivore, Tiger.
18. Rings, Ornaments, Diamond Rings.
19. Indoor games, Chess, Table tennis.
20. Fish, Herring, Animal living in water
21. Which one of the following diagrams represents the
(c) (d) relationship among Delhi, Lucknow, Uttar Pradesh?

8. Selects the figures which represents the relationship (a) (b)


between athletes, football players and cricket-players.

(c) (d)
(a) (b) 22. Which one of the following diagrams represents the correct
relationship among day, week, year?

(a) (b)
(c) (d)

9. Which of the following diagram represents the country, (c) (d)


states and District?
y
o
u
Venn Diagrams A-83

rs
m
23. Which one of the following diagrams best depicts the 28. Which are the numbers that makes their presence felt in

a
relationship among Homan Society - Youth Club. Political only one of the geometric figures.

h
Party and Youths ?

b
o
o
1

b
(a) (b) 6

.w
2

o
rd
(c) (d) 3 8 7 9

p
24. Which one of the following diagramsre represent correct

re
4

s
relationship among pigeon, birds, dogs ?

s
.c
o
m
(a) 4, 6, 7 (b) 1, 2, 9
(c) 3, 7, 9 (d) 2, 3, 8
(1) (2) (3) (4) 29. In the following venn diagram identify the better which
(a) 1 (b) 2 denotes players who are also doctors but not artist.
(c) 3 (d) 4 A Player
25. In the given figure in a garden, square represent the area
where jackfruit trees are grown, circle represent mango B G
trees and triangle represent coconut trees. Which number C
represent the common area in which all types of trees are Doctor
D E
grown.
F
1
8 2
7 3 5
9 H

Artist
(a) 4 (b) 3
(a) B + E (b) E
(c) 7 (d) 8
(c) B (d) A
26. In the following venn diagram, Identify the politicians from
30. In the given figure, the circle represents boys, triangle
urban areas who are corrupt.
represents players and square represents rurals. What
Rural people portion represents rural sports boys.
1
2

3 4 B C
5 A
Politician 7 8 9 Urban people D
E F
(a) E (b) F
11
12 (c) D (d) B

Corrupt people 31. School children


(a) 4 (b) 8 a
(c) 9 (d) 10 b
27 In the following figure, the boys who are cricketer and sober
d c
are indicated by which number ? f
Cricketer
Artist
e
10
7 Sober
g
6
Singers
2 9 Boys Above diagram represents school children, artist and
Girls 4 1 singers. Study the diagram and identify the region. Which
8 represents those school children who are artists not singers.
(a) a (b) b
(a) 6 (b) 5
(c) f (d) e
(c) 4 (d) 2
y
o
u
A-84 Venn Diagrams

rs
32.

m
39. Who among the following is a male urban oriented and also
1 3

a
2 a civil servant but not educated?

h
5 (a) 13 (b) 12

b
10 11 4 6

o
(c) 6 (d) 10

o
9

b
7

.w
Directions (Qs. 40-43) : In the figure below there are three
intersecting circles each represents certain section of people

o
8

rd
different region are marked a.g. Read the statement in each of the

p
– Girls following questions and choose the letter of the region which

re
– Athletics correctly represent the statements.

s
s
.c
– Boys A B

o
m
– Disciplined
a b f
The boys who are athletic and are disciplined are indicated Chinese Painter
by which numbers. c
(a) 1 (b) 4 d e
(c) 6 (d) 10
g Musicians
Directions (Qs. 33-39) : The following question are based on
the diagram given below.
C
40. Chinese who are painters but not musicians?
3 10 7 (a) b (b) c
(c) d (d) g
8 12 6 13 41. Painter who are neither Chinese nor musician?
(a) b (b) c
4 (c) f (d) g
14 42. Chinese who are musicians but not painters?
9 11
(a) d (b) c
5 (c) b (d) a
43. Chinese who are painters as well as musicians?
(a) a (b) b
Rectangle represents males (c) c (d) d
Triangle represents educated Directions (Q s. 44-47) : Refer to the followin g Venn
Circle represents urban diagram :
Square represents civil servants
33. Who among the following is an educated male who is not HISTORY
an urban resident?
(a) 4 (b) 5 16 GEOGRAPHY
(c) 11 (d) 9
34. Who among the following is neither a civil servant nor 14 13 12
educated but is urban and not a male?
9 18 20 18 9
(a) 2 (b) 3
(c) 6 (d) 10 13 16
35. Who among the following is a female urban resident and 15
MATHEMATICS
also a civil servant?
19 SCIENCE
(a) 6 (b) 7
(c) 10 (d) 13
36. Who among the following is an educated male who hails 44. The number of students who took any three of the above
from urban? subjects was
(a) 4 (b) 7 (a) 62 (b) 63
(c) 10 (d) 13 (c) 64 (d) 66
37. Who among the following is uneducated and also an urban 45. The number of students in total, who took History or
male? Mathematics or Science, was
(a) 2 (b) 3 (a) 183 (b) 190
(c) 11 (d) 12 (c) 424 (d) 430
38. Who among the following is only a civil servant but neither 46. The number of students who took both History and
a male or urban oriented and uneducated? Geography among other subjects was
(a) 7 (b) 8 (a) 62 (b) 63
(c) 9 (d) 14 (c) 65 (d) 66
y
o
u
Venn Diagrams A-85

rs
m
47. Which subject was taken by the largest number of students?

a
(a) Mathematics (b) Science

h
(a) (b)
(c) Geography (d) History

b
o
48. Identify the diagram that best represents the relationship

o
b
among classes given below :

.w
Food, Curd, Spoons (c) (d)

o
rd
p
re
ANSWER KEY

s
s
.c
1 (d) 7 (c) 13 (c) 19 (b) 25 (b) 31 (b) 37 (d) 43 (c)

o
m
2 (a) 8 (b) 14 (c) 20 (c) 26 (c) 32 (b) 38 (a) 44 (a)
3 (d) 9 (c) 15 (d) 21 (c) 27 (d) 33 (c) 39 (c) 45 (a)
4 (c) 10 (c) 16 (a) 22 (a) 28 (b) 34 (b) 40 (a) 46 (b)
5 (d) 11 (b) 17 (d) 23 (b 29 (c) 35 (c) 41 (c) 47 (b)
6 (a) 12 (a) 18 (b) 24 (a) 30 (c) 36 (a) 42 (a) 48 (d)

1. (d) Snake is different from Lizard but both are reptiles.


Reptiles Instrumentalists
Snake
6. (a)
Lizard Violinists

2. (a) Some teachers may be graduates and vice-versa all


teachers and all graduates are human beings.
Musicians

Human being
7. (c) Capsules are different from injection but both are uses
as antibiotics.
Teacher Graduate 8. (b) Some athletes may be football players and vice-versa.
Some athletes may be cricket players and vice-versa.
Some athletes may be both football players and cricket
players.
3. (d) Sun is stars. Moon is a satellite.
Some cricket players may be football players and vice-
Star versa.

9. (c)
Sun States
Moon
District Country
4. (c) Judge is different from both the thief and criminal.

Judge Thief
10. (c) Hockey
Criminal
5. (d) Some politician may be poets and vice-versa. Tennis
Cricket
Some politician may be women and vice-versa.
11. (b) Home minister is a minister, minister is part of cabinet.
No poets can be women as women poets is called
12. (a) Some professor may be scientist or researcher.
poetess.
13. (c) Men, Rodents are entirely differents, but both are living
beings.
Poets Politician Women 14. (c) Mother and Father are entirely different but both are
parents.
y
o
u
A-86 Venn Diagrams

rs
m
15. (d) Nitrogen is air but ice is different.
29. (c) Area common to and .

a
16. (a) All three items are partly related.

h
17. (d) Tiger is a carnivore, while elephant is not.

b
30. (c) Area common to , and

o
18. (b) Herring is type of fish, fish belongs to the class of

o
b
water animals. 33. (c) Area common to and

.w
19. (b) All diamonds rings are rings, all rings are ornaments. 34. (b) circle only

o
20. (c) Chess and table tennis are differents but both are indoor 35. (c) Area not common to rectangle but common to square

rd
games. and circle.

p
re
36. (a) Area common to Rectangle, Circle, Triangle

s
37. (d) Area common to Rectangle and Circle.

s
.c
38. (a) Only area of square.

o
39. (c) Area common to rectangle, square and circle.

m
21. (c)
40-43

Delhi is separate state while Lucknow is part of Uttar Athletes Football players
Pradesh.

22. (a) Year


Cricket players
Week

Day 40. (a) Required region is one which is common to circle A


and B.
41. (c) Area that contain only B.
42. (a) Area common to A and C.
43. (c) Common area of A, B and C
Youths 44. (a) The required set of students is denoted by region
common to any three circle only
Required number = (13 + 13 + 18 + 18) = 62.
23. (b) Political 45. (a) The required set of students is denoted by regions
Youth club
Party lying inside the circles representing History,
Mathematics and Science. Required number = (9 +
Birds 14 + 18 + 15 + 16 + 13 + 13 + 20 + 18 + 13 + 16 + 19) = 183.
24. (a)
Pigeon Dogs 46. (b) The required set of students is denoted by the regions
common to the circles representing History and
25. (b) 3 represents the area common to all types. Geography.
Required number = (20 + 13 + 12 + 18) = 63.
47. (b) Number of students who took History
26. (c) Required area common to , , . = (16 + 12 + 18 + 20 + 18 + 14 + 13) = 111.
27. (d) Regioins Number of students who took Geography
1 2 4 6 7 8 9 10 = (9 + 16 + 13 + 20 + 13 + 12 + 18) = 101.
Persons Number of students who took science
Boys = (19 + 15 + 18 + 20 + 18 + 16 + 13) = 119.
Girls Number of students who took mathematics
= (9 + 14 + 13 + 20 + 13 + 15 + 18) = 102.
Sober
Cricketer Food
Region 2 presents the boys who are cricketer and sober. 48. (d) Curd Spoons
28. (b) 1 represent only Circle.
2 represent only Rectangle.
9 represent only another Rectangle.
y
o
u
Syllogism A-87

rs
m
a
CHAPTER

h
b
12

o
SYLLOGISM

o
b
.w
o
rd
p
re
s
s
.c
Syllogism is a Greek word that does mean ‘inference’ or

o
Categorical proposition

m
‘deduction’. The problems of syllogism are based on two parts :
1. Proposition / Propositions
2. Conclusion / Conclusions drawn from given proposition/
propositions Universal Particular

WHAT IS A PROPOSITION?
Just consider the sentences given below:
Positive Negative Positive Negative
(i) “All lions are pigs ”
All M are P No M are P Some M are P Some M are not P
Subject Predicate (A type) (E type) (I type) (O type)
Therefore, it is clear, that universal propositions either completely
(ii) “No cat is rat ” include the subject (A type) or completely exclude it (E type). On
the other hand, particular propositions either only partly include
the subject (I type) or only partly exclude the subject (O type).
Subject Predicate
Now we can summarise the four types of propositions to be used
while solving the problems of syllogism :
(iii) Format Type
All M are P – A
No M are P – E
Some M are P – I
(iv) Some M are not P – O

Venn Diagram:
All A, E, I and O type of propositions can also be represented in
All the sentences mentioned above give a relation between subject pictorial way and this method is known as Venn diagram.
and predicate. Here, it is clear from the sentences that a subject is (i) Representation of “All M are P” (A type):
the part of a sentence something is said about, while a predicate
is the term in a sentence which is related to the subject. P
Now, let us define the proposition :
A proposition is a sentence that makes a statement giving a relation M
between two terms. It has three parts :
(a) the subject
(b) the predicate Here, the whole circle denoting M (all M) lies inside the
(c) the relation between subject and predicate circle denoting P. The other possibility is as picture given
WHAT ISA CATEGORICAL PROPOSITION? below :
Let us see the sentences given below :
“All M are P” M, P
“No M are P”
“Some M are P”
“Some M are not P” (ii) Representation of “No M are P” (E type):
What we notice in all above-Mentioned sentences that they are
condition free. These type of sentences are called Categorical M P
Propositions. In other words a categorical proposition has no
condition attached with it and it makes direct assertion. It is Here, the circle denoting M and P do not intersect at all and
different from non-categorical proposition which is in the format therefore, truely represents
“If M then P” “No M are P”
Types of categorical proposition: (iii) Representation of “Some M are P” (I type):
It can be understood by the diagram given below : This representation will be in two ways :
y
o
u
A-88 Syllogism

rs
m
Either (a): Or :

a
h
M

b
M P

o
P

o
b
.w
Here it is clear from the picture that shaded part of M is Some M are P
some part of P and shaded part of P is some part of M. Thus

o
[All P are M]

rd
“Some M are P”. Similarly, unshaded part of M is not P and
Some M are not P (O type):

p
unshaded part of P is not M. Thus it represents “Some M

re
are not P”. Either:

s
s
Or (b):

.c
o
M M P

m
P
Some M are not P
Here, only shaded part of M is P also. Thus we can say [Some M are P]
“Some M are P.” Or:
(iv) Representation of “Some M are not P” (O type):
This representation will be in three ways : P
Either (a):
M

M P Some M are not P


[All P are M]
Here, unshaded part of M is not a part of P. Thus it represents How to identify hidden propositions ?
“Some M are not P.” But the shaded part represents “Some (i) A type: Apart from ‘all’ it starts with every, each and any.
M are P”. EXAMPLE 1.
Or (b) :
Every girl is beautiful.
[All girls are beautiful.]
P Each of them is healthy.
[All (of them) are healthy.]
M Any one could kill the lion.
[All can kill the lion.]
Further, let us see the sentences given below :
Here, shaded part of M is not a part of P. Thus it represents
“Some M are not P” and the circle denoting P represents He should be amended Bharat Ratna
“All P are M”.
Or (c):
Subject Predicate
M P
Amitabh Bacchan is a great actor.
It is clear from this pictorial representation that this
represents “Some M are not P” and “No M are P” as well.
Now we can make a summary of Venn diagram: Subject Predicate
All M are P (A type): Thus, a positive sentence with a particular person as its
P subject is A type.
M, P Also, a sentence in the following format is A type :
and
M definite exception
[Possibility]
No M are P (E type): “All girls except Reeta are healthy.”

M P (ii) E type: Apart from ‘no’ this type of propositions starts from
‘no one’, ‘none’, ‘not a single’ etc.
Some M are P (I type): EXAMPLE 2.
Either:
No one (student) is studious.
[No student is studious]
M P None of the girl is beautiful.
[No girl is beautiful]
Some M are P Not a single girl is healthy.
[Some M are not P] [No girl is healthy].
y
o
u
Syllogism A-89

rs
Further, let us see the sentences given below :

m
EXAMPLE 4.

a
All girls are not beautiful.

h
He does not deserve Bharat Ratna

b
[Some girls are not beautiful]

o
o
Every boy is not present.

b
Subject Predicate [Some boys are not present.]

.w
Further, let us see the following sentences :

o
Poor are usually not healthy.

rd
Amitabh Bacchan is not a great actor.
[Some poor are not healthy]

p
re
Almost all the girls are not beautiful.

s
Subject Predicate [Some girls are not beautiful.]

s
.c
Thus, a negative sentence with a particular person as its subject Most of the garments are not handmade.

o
is E type proposition. [Some of the garments are not handmade.]

m
Also, sentences in following formats are E type : Girls are not frequently short tempered.
definite exception [Some girls are not short tempered].
Now, it is clear from the above mentioned examples that negative
propositions with words such as ‘almost’, ‘frequently’, ‘most’,
“No student except Reena has failed” ‘mostly’, ‘a few’, generally, etc. are to be reduced to the O–type
propositions.
“Is there any truth left in the world”
Again, positive propositions starting with words like ‘few’,
[No truth is left in the world.]
‘scarcely’, ‘rarely’, ‘little’, ‘seldom’ etc. are said to be O–type.
(iii) I type: Apart from some it also starts with words such as
often, frequently, almost, generally, mostly, a few, most etc. EXAMPLE 5.
3. Seldom are women jealous.
EXAMPLE
[Some women are not jealous]
Almost all the girls are beautiful. Few girls are beautiful.
[Some girls are beautiful]. [Some girls are beautiful]
Most of the garments are handmade. Rarely is a wealthy person worried.
[Some of the garments are handmade]. [Some wealthy person are not worried.]
Usually girls are beautiful. Also, see the following formates :
[Some girls are beautiful.]
A few money are left in my wallet. No definite exception as name of
[Some money are left in my wallet]. girls are not given.
Further, let us see the sentences given below :
Few girls are not studious.
[Some girls are studious.] No girls except three are beautiful.
Rarely is a girl not beautiful. [Some girls are not beautiful.]
[Some girls are beautiful].
Seldom are women not housewife. No definite exception as name of
[Some women are housewife]. women are not given.
It is clear from the above examples that negative sentences
begining with words like ‘few’, ‘rarely’, ‘seldom’, etc. (Also
‘hardly’, ‘scarcely’, ‘little’ etc.) are to be reduced to I type. No women except a few are housewife.
Just see the other formates given below : Therefore, a negative proposition with an indefinite exception, is
Not a definite exception as name of reduced to O type.
girls are not given. IDENTIFYING EXCLUSIVE PROPOSITIONS :
Such propositions start with ‘only’, ‘alone’, ‘none else but’, ‘none
but’ etc. and they can be reduced to either A or E or I format.
All girls except a few are beautiful. EXAMPLE 6.
[Some girls are beautiful] Only graduates are Probationary Officers.
No graduate is Probationary Officer (E type)
Not a definite exception as name of All Probationary Officers are graduates. (A type)
girls are not given. Some graduates are Probationary Officers (I type)
General format of sentences given in the examinations :
All M are P (A type)
All girls except 5 have passed No M are P (E type)
Some M are P (I type)
[Some girls have passed] Some M are not P (O type)
Therefore, a positive proposition with an indefinite exception NOTE : General format given above are frequently asked formats
is reduced to I type. in the examinations. But students must be ready for other hidden
(iv) O type : Apart from “Some ....... not’ this type of statements formates of A, E, I and O types of propositions as problems in
start with words like ‘all’, ‘every’, ‘any’, ‘each’, etc. hidden formates can also be given in question papers.
y
o
u
A-90 Syllogism

rs
m
Conversion of propositions : Conclusion Table

a
Before solving the problems of syllogism it is must to know the I Proposition II Proposition Conclusion

h
conversion rules of all A, E, O, and I types of propositions :

b
A A A

o
Conversion of A type :

o
A E E

b
Subject Predicate E A (O)R

.w
E I (O)R

o
rd
“All M are P ” (A type) I A I

p
I E O

re
After conversion it becomes. Note :

s
Subject Predicate

s
(a) Apart from above 6 pairs of propositions, no other pair will

.c
give any conclusion.

o
m
“Some P are M ” (I type) (b) The conclusion drawn out of two propositions is itself a
Therefore, it is clear that A type of propositions get converted proposition and its subject is the subject of the Ist statement
into I type. while its predicate is the predicate of the 2nd statement.
Conversion of E type : The common term get disappeared.
Subject Predicate (c) (O)R does mean that the conclusion is O type but is in
reverse order. In this case, the subject of the inference or
conclusion is the predicate of the 2nd proposition and the
“No M are P ”(E type) predicate of the conclusion is the subject of the Ist sentence
After conversion it becomes or statement.
Subject Predicate (d) The conclusion table gives correct conclusions or inference
if and only if the two propositions are aligned properly.
WHAT IS ALIGNING ?
“ No P are M ” (E type)
Let us see the following examples :
Therefore, E get converted into E. “sss” EXAMPLE 7.
Conversion of I type :
Statements :
Subject Predicate
I. All girls are beautiful.
“Some M are P ” (I type)
II. Some girls are Indian.
After conversion it becomes
EXAMPLE 8.
Subject Predicate
Statements :

“Some P are M ” (I type) I. No pen is chair..

Therefore, I get converted into I. II. Some tables are pen .


Conversion of O type :
O type of proposition can’t be converted. EXAMPLE 9.
Note : In each conversion, subject becomes predicate and Statements :
predicate become subject.
In fact, conversion is an immediate inference that is drawn from a I. Some women are men .
single proposition while inference drawn from two propositions
are called mediate inference. II. No men is chair..
Now we can make a short table of conversion to remember.
Table of conversion : In all the above mentioned example, we notice that in two
Type of proposition Get converted into statements of every example, there is a common term. In
A .................................. I example 7 the word ‘girl’ is common; in example 8 the word
E .................................. E ‘pen’ is common while in example 9 the word ‘men’ is
I .................................. I common.
O .................................. Never get converted Now, the aligning of the two statements (propositions) does
Rule to draw conclusion : mean that the pair of statements must be written in such a
After knowing conversion of propositions, we must learn the way that the common term is the predicate of the 1st sentence
rules to draw conclusions. In problems of syllogism, conclusions and the subject of the 2nd.
are drawn either from single propositions or from two proposition Just think over the following examples :
or from both. But a conclusion from single proposition is just a Statements :
conversion of that proposition while to get conclusion from two
propositions a certain table is used that tells us what type of I. Some girls are cute .
conclusion (in form of proposition) we get out of two propositions.
To understand it, let us see the following conclusion table : II. All cute are tall.
y
o
u
Syllogism A-91

rs
m
Here, the common term cute is the predicate of the I EXAMPLE 10.

a
statement and subject of the 2nd statement. Therefore, the

h
Statements :
two statements (I & II) are properly aligned.

b
I. All rats are cats.

o
But see another example.

o
II. All rats are men.

b
Statements :

.w
When aligned it takes the form as
I. Some bats are chairs.

o
rd
I. Some cats are rats [I type]

p
II. Some cats are bats .

re
Here, the sentences are not aligned as the predicate of the II. All rats are men [A type]

s
s
.c
1st statement is not the subject of the 2nd. Now we use the conclusion table given in this chapter that

o
Then how to align it ? In such type of cases we change the says

m
order of sentences. In another words we put I sentence in I + A = I type of conclusion.
place of II and II in place of I : Therefore, the drawn conclusion must be
II. Some cats are bats . “Some cats are men”
It is clear that the conclusion drawn “Some cats are men” is
I. Some bats are chairs. a mediate inference as it is the result of two propositions.
Let us consider another pair of statements. But in actual problem immediate inferences are also given
I. All bats are chair. in conclusion part and that format is given below :
II. All bats are cats. EXAMPLE 11.
Then how to align it ? In fact, in such cases we do alignment Statements :
in two ways : I. All rats are cats.
(a) by converting statement I as
II. All rats are men.
I. Some chair are bats . Conclusion :
(i) Some cats are men.
II. All bats are cats. (ii) Some men are cats.
and (iii) Some rats are cats.
(b) by changing the order of the sentences and then (iv) Some cats are rats.
converting the statement II. (v) Some rats are men.
Now 1st change the order as : (vi) Some men are rats.
II. All bats are cats. Answer options :
I. All bats are chair. (a) only (iii) follows
Again we do conversion for II and the aligned pair (b) only (i), (ii) and (iii) follow
takes the form as
(c) only (iv) follows
II. Some cats are bats . (d) all follow
(e) none of these
I. All bats are chair.. Here, the correct option is (d).
Therefore, as per the requirement and nature of the Conclusion (i) follows because it is the mediate inference of
sentence the alignment is done. statements I & II.
(i) only by changing the order of sentences. Conclusion (ii) is the conversion of conclusion (i).
or Conclusion (iii) is the immediate inference (conversion) of
(ii) only by converting of the sentences. statement I while conclusion (iv) is the conversion of
or conclusion (iii).
(iii) By changing the order of the statements and then Conclusion (v) is the immediate inference (conversion) of
converting one of the sentences. statement II while conclusion (vi) is the conversion of
IEA Rule : conclusion (v).
Alignment must be done in IEA order. It does mean that if the two Further, in some problems complementary pairs are also
statements are I & E then the conversion must be done for I and seen in the conclusion part in the forms of sentence given
for E & I it will be done for E.
below :
After discussing all the minute things about this chapter, now we
have come at the position of solving the problems of syllogism. (i) Some cats are rats.
This chapter suggests two methods: I-O pair
(ii) Some cats are not rats.
(1) By Analytical Method
(2) By Venn Diagram (i) All cats are rats.
A-O pair
(1) Analytical method : (ii) Some cats are not rats.
This method has two main steps :
(a) Aligning the pair of sentences. (i) Some cats are rats.
I-E pair
(b) Using conclusion table to draw conclusion. (ii) No cats are rats.
y
o
u
A-92 Syllogism

rs
m
Apart from I-O, A-O and I-E pair the two sentences must EXAMPLE 13.

a
have same subject and predicate as are the above mentioned

h
Statements :

b
pairs. For these pairs we write the form

o
A. All chairs are books. B. All books are ties.

o
Either (i) or (ii) follows

b
Conclusions :

.w
For example, see the following format :
I. Some ties are books. II. Some ties are chairs.

o
EXAMPLE 12. 1st Step :

rd
Statements :

p
re
I. Some dogs are cats. II. Some cats are rats.

s
b c t b

s
Conclusions :

.c
(i) Some cats are dogs.

o
m
(ii) Some rats are cats.
1A 1B
(iii) All cats are rats.
(iv) Some dogs are rats.
(v) Some dogs are not rats. c, b b, t
Answer options :
(a) All follow.
(b) Only (i) follows. 2A 2B
(c) Only (ii) and (iii) follow. Here, 1A and 2A are representations for statement A while
(d) Either (iv) or (v) and (i) & (ii) follow. 1B and 2B are representations for statement B. In these
Here, option (d) is correct because conclusion (i) is the representations
immediate inference (conversion) of statement I while b = books
conclusion (ii) is the immediate inference of II. Conclusion c = chairs
(iv) & (v) make complementary pair of I-O type. t = ties
Conclusion (iii) is not correct because I and II are I type of 2nd step :
statements and I + I does not give any conclusion. Further, Let us combine all the possible pairs of this pictorial
A type of conclusion can not be find from the immediate representations :
inferences (conversion) of I type of statements as I & II are.
Now, the complete process of solving syllogism problems t
b c b c t
can be summaried as below :
(a) 1st step is aligning the sentences.
(b) 2nd step is using conclusion table.
(1A + 1B) (1A + 2B)
(c) 3rd step is checking immediate inferences.
(d) 4th step is checking through the conversion of
immediate inferences & mediate inferences. t c, b c, b t
(e) Checking the complementary pairs.
(2) Venn diagram method for solving problems :
Students will have to adopt three steps to solve the (2A + 1B) (2A + 2B)
syllogism problems through Venn diagram method : 3rd step :
(a) 1st step is sketching all possible pictorial When we interpret the pictures in step II, we find that all the
representation for the statements separately. pictures support both the conclusions. Therefore,
(b) 2nd step is combining possible pairs of these conclusion I :
representations of all the statements into one. “Some ties are books” and
(c) 3rd and final step is making interpretation of this conclusion II.
combined figure. Conclusions are true if they are “Some ties are chairs”
supported by all the combined figures in 2nd step. both are true.
Now let us solve a problem. Important Note : In the Venn diagram method, any conclusion
given with any problem will be true if and only if it is supported
by all the combined pictorial representations through 2nd step.
If any pictorial representation contradicts the given conclusion,
it will be put in the category of incorrect or wrong conclusion.
y
o
u
Syllogism A-93

rs
m
a
h
b
o
o
b
.w
DIRECTIONS (Qs.1): In each of the questions below are given (c) Only either I or II and III follow

o
three statements followed by four conclusions numbered I, II, III (d) None of these

rd
and IV. You have to take the given statements to be true even if 5. Statements:

p
re
they seem to be at variance with commonly known facts. Read all Some shirts are coats.

s
the conclusions and then decide which of the given conclusions All coats are jackets.

s
.c
logically follows from the given statements disregarding Some jackets are trousers.

o
commonly known facts. Conclusions:

m
1. Statements: I. Some shirts are jackets.
Some boxes are trees. II. Some jackets are shirts.
Some trees are horses. III. All trousers are jackets.
All horses are fruits. IV. Some trousers are jackets.
Conclusions: (a) All follow
I. Some fruits are boxes. (b) Only I, II and III follow
II. Some fruits are trees. (c) Only I, II and IV follow
III. Some horses are boxes (d) Only II, III and IV follow
IV No fruits are boxes. 6. Statements:
(a) None follows All bikes are scooters. .
(b) Only either II or IV follows All scooters are scooties.
(c) Only either I or IV and II follow All scooties are mopeds.
Conclusions:
(d) Only either I or III and IV follow
I. All mopeds are scooties.
2. Statements:
II. All scooties are scooters.
All flowers are buses.
III. All scooters are bikes.
Some buses are cats.
IV. All bikes are mopeds.
All cats are tigers.
(a) None follows (b) All follow
Conclusions:
(c) Only III and IV follow (d) Only IV follows
I. Some tigers are buses.
7. Statements:
II. Some tigers are flowers.
Some pots are buckets.
III. Some cats are flowers.
Some buckets are bags.
IV. Some buses are tigers.
Some bags are purses.
(a) None follows (b) Only I and II follow
Conclusions:
(c) Only III and IV follow (d) Only I and IV follow
I. Some purses are buckets.
3. Statements: II. Some bags are pots.
No man is sky.
III. Some purses are pots.
No sky is road.
IV. Some pots are bags.
Some men are roads.
(a) All follow (b) None follows
Conclusions:
(c) Only I and III follow (d) Only II and IV follow
I. No road is man. 8. Statements:
II. No road is sky. All biscuits are chocolates.
III. Some skies are men.
Some chocolates are breads.
IV. All roads are men.
All breads are pastries.
(a) None follows (b) Only I follows Conclusions:
(c) Only I and III follow (d) None of these I. Some biscuits are pastries.
4. Statements: II. Some pastries are chocolates.
All windows are doors. III. Some biscuits are not pastries.
No door is a lock. IV. All pastries are breads.
Some locks are keys. (a) Only I and II follow
Conclusions: (b) Only I, lI and III follow
I. Some locks are doors. (c) Only either I or III and II follow
II. Some locks are not doors. (d) Only either I or III and IV follow
III. Some keys are not doors. 9. Statements:
IV. All doors are windows. All birds are fruits.
(a) All follow. Some fruits are towers.
(b) Only either I or II follows All towers are windows.
y
o
u
A-94 Syllogism

rs
m
Conclusions: 14. Statements:

a
I. Some birds are towers. Some candles are houses.

h
Some houses are trains.

b
II. Some windows are birds.

o
III. Some windows are fruits. Some trains are roads.

o
b
IV. Some towers are birds. Conclusions:

.w
(a) None follows (b) Only I and II follow I. Some roads are candles.

o
II. Some trains are candles.

rd
(c) Only II and III follow (d) Only III follows
III. Some roads are houses.

p
10. Statements: IV. Some candles are roads.

re
Some buses are trains.

s
(a) None follows (b) All follow

s
No train is a dog.

.c
(c) Only I and II follow (d) Only II and III follow

o
All dogs are parrots. 15. Statements:

m
Conclusions: No tree is fruit.
I. No bus is a parrot. All fruits are stones.
II. Some parrots are trains. All stones are rains.
III. Some parrots are buses. Conclusions:
IV. No dog is a bus. I. No stone is tree.
II. No rain is tree.
(a) Only either I or III follows
III. Some rains are fruits.
(b) Only II follows IV. Some rains are trees.
(c) Only IV follows (a) None follows
(d) Only I and III follow (b) Only either II or IV and III follow
11. Statements: (c) Only either II or III and I follow
Some cups are flowers. (d) All follow
Some flowers are boxes. 16. Statements:
All boxes are tigers. All books are stars.
Conclusions: Some stars are windows.
I. Some tigers are cups. All windows are hills.
II. Some tigers are flowers. Conclusions:
III. Some boxes are cups. I. Some windows are books.
II. Some hills are stars.
IV. No tiger is a flower.
III. Some hills are books.
(a) None follows IV. Some stars are books.
(b) Only either II or IV follows (a) None follows (b) Only I and III follow
(c) Only III follows (c) All follows (d) Only II and IV follow
(d) None of these 17. Statements:
12. Statements: Some cats are rats.
All glasses are roads. All rats are bats.
No road is a stick. Some bats are jungles.
Some sticks are pens. Conclusions:
Conclusions: I. Some jungles are cats.
I. Some glasses are sticks. II. Some bats are cats.
II. Some pens are sticks. III. Some jungles are rats.
III. Some roads are sticks. IV. No jungles is cat.
IV. No glass is a stick. (a) None follows
(b) Only III follows
(a) None follows
(c) Only either I or IV and III follow
(b) Only I or IV and II follow
(d) Only either I or IV and II follow
(c) Only either I or III or II follows 18. Statements:
(d) None of these All flowers are clouds.
13. Statements: No cloud is sky.
All buses are trains. All skies are tigers.
All trains are rickshaws. Conclusions:
All rickshaws are cycles. I. Some clouds are flowers.
Conclusions: II. All clouds are flowers.
I. All cycles are buses. III. Some tigers are skies.
II. All rickshaws are buses. IV. All tigers are skies.
III. All buses are rickshaws. (a) Only II and IV follow
IV. All trains are cycles. (b) Only either I or II follows
(a) All follow (b) None follows (c) Only either III or IV follows
(c) Only I and II follow (d) None of these (d) None of these
y
o
u
Syllogism A-95

rs
m
19. Statements: III. Some papers are plates.

a
Some spectacles are boxes. IV. Some books are records.

h
b
No bat is a ball. (a) Only I follows (b) Only II and III follow

o
o
Some boxes are balls. (c) Only I and Ill follow (d) Only IV follows

b
24. Statements:

.w
Conclusions:
Some bricks are trees.

o
I. Some boxes are not bats.

rd
All trees are pens.
II. Some bats are spectacles.

p
All pens are boats.

re
III. No bat is a box.

s
Conclusions:

s
IV. No ball is a spectacle.

.c
I. Some boats are bricks.

o
(a) Only I follows (b) Only I and III follow II. Some pens are bricks.

m
(c) Only II and III follow (d) Only IV follows III. Some trees are bricks.
20. Statements: IV. Some bricks are boats.
All fans are tubelights. (a) None follows (b) All follow
No pen is a bulb. (c) Only I and II follow (d) Only III and IV follow
Some bulbs are fans. 25. Statements:
Conclusions: All cups are tables.
I. Some pens are tubelights. No table is water.
II. No pens are tubelights. Some waters are clothes.
Conclusions:
III. Some tubelights are fans.
I. No cloth is cup.
IV. All tubelights are fans.
II. No cloth is table.
(a) Only I and II follow III. Some clothes are waters.
(b) Only I, II and III follow IV. Some waters are cups.
(c) Either I or II and III follow (a) None follows (b) All follow
(d) Only III and IV follow (c) Only III follows (d) Only I and II follow
21. Statements: 26. Statements:
Some fruits are vegetables. Some flowers are rods.
All liquids are drinks. Some rods are doors.
All drinks are fruits. Some doors are houses.
Conclusions: Conclusions:
I. Some drinks are vegetables. I. Some houses are flowers.
II. Some doors are flowers.
II. Some fruits are liquids.
III. Some flowers are doors.
III. All liquids are fruits.
IV. No house is flower.
IV. No liquids are vegetables.
(a) Only I and IV follow
(a) Only I and II follow (b) Only II and III follow
(b) Only II and III follow
(c) Only III and IV follow (d) Only I, II and III follow
(c) Only either I or II follows
22. Statements:
(d) Only either I or IV follows
All shirts are trousers.
27. Statements:
Some socks are shoes.
All shoes are shirts. All trucks are vans.
Conclusions: All vans are cars.
I. Some socks are shirts. All cars are trains.
II. Some socks are trousers. Conclusions:
III. All shoes are trousers. I. All trains are trucks.
IV. All shoes are socks. II. All cars are trucks.
(a) Only I and II follow (b) Only I or II or III follows III. All trucks are trains.
(c) Only II and IV follow (d) None of these IV. All vans are trains.
23. Statements: (a) All follow (b) Only I and II follow
Some books are papers. (c) Only II and III follow (d) None of these
All plates are records. 28. Statements:
Some records are books. No table is fruit.
Conclusions: No fruit is window.
I. Some plates are books. All windows are chairs.
II. Some records are papers.
y
o
u
A-96 Syllogism

rs
m
Conclusions: 31. Statements:

a
I. No window is table. All leaders are good team workers.

h
b
II. No chair is fruit. All good team workers are good orators.

o
III. No chair is table.

o
Conclusions:

b
IV. All chairs are windows.

.w
I. Some good team workers are leaders.
(a) None follows (b) All follow II. All good orators are leaders.

o
rd
(c) Only I and II follow (d) Only III and IV follow 32. Statements:

p
29. Statements: All terrorists are human.

re
s
I. Some towers are windows. All humans are bad.

s
.c
II. All windows are houses. Conclusions:

o
III. Some houses are temples. I. All terrorists are bad.

m
Conclusions: II. No human can be a terrorist.
I. Some towers are temples. 33. Statements:
II. Some houses are towers. Some teachers are followers.
(a) Only conclusion I follows. Some followers are famous.
(b) Only conclusion II follows. Conclusions:
(c) Both conclusions I & II follow. I. Some teachers are famous.
(d) Neither conclusion I nor II follows. II. Some followers are teachers.
30. Statement : 34. Statements:
I. The rich must live more simply. Some books are pens.
II. All poor people are simple. No pen is pencil.
Conclusions : Conclusions:
I. Rich people waste money. I. Some books are pencils.
II. Poor people save money. II. No book is pencil.
(a) Only I follows. (b) Only II follows. 35. Statements:
(c) Neither I nor follow. (d) Both I and II follow. Some dedicated souls are angels.
DIRECTIONS (Qs. 31-35) : In each questions belows are given All social workers are angels.
two statements followed by two conclusions. You have to take Conclusions:
the given statements to be true even if they seem to be at variance I. Some dedicated souls are social workers.
with commonly known facts. Read all the conclusions and then II. Some social workers are dedicated souls.
decide which of the given statements disregarding commonly 36. (1) All roses in sita's garden are red.
know facts. Given Answer. (2) All marigold flowers in Sita's garden are orange.
Give answer (a) If only conclusion I follows. (3) All flowers in Sita's garden are either red or orange.
Give answer (b) if only conclusion II follows. If lst two statements are true, the third is
Give answer (c) if either I or II follows. (a) True (b) False
Give answer (d) if neither I nor II follows. (c) Uncertain (d) Vague

ANSWER KEY
1 (c) 5 (c) 9 (d) 13 (d) 17 (d) 21 (b) 25 (c) 29 (b) 33 (b)
2 (d) 6 (d) 10 (a) 14 (a) 18 (d) 22 (d) 26 (d) 30 (c) 34 (c)
3 (d) 7 (b) 11 (d) 15 (b) 19 (a) 23 (d) 27 (d) 31 (a) 35 (d)
4 (d) 8 (c) 12 (d) 16 (d) 20 (c) 24 (b) 28 (a) 32 (a) 36 (c)
y
o
u
Syllogism A-97

rs
m
a
h
b
o
o
b
.w
1. (c) Conclusion II follows from conversion of the Venn-Diagram Representation:
conclusion obtained from statement (b) and statement

o
rd
(c) [ I + A = I]. Conclusion I, III and IV do not follow

p
Pots Buckets Bags Purses
because statement (a) + statement (b) gives no

re
conclusion. But the conclusion I and IV make a

s
s
.c
complementary pair IE-type. Hence, either of the two Conclusions:

o
follows. I. False

m
2. (d) Conclusion IV follows from statement (b) and statement II. False
(c). [ I +A = 1]. Conclusion I follows from conversion III. False
of conclusion IV. Statement (a) and statement (b) give IV. False
no conclusion [ A + I = no conclusion ]. Therefore, 8. (c) Some chocolates are breads + All breads are pastries
conclusion II and III do not follow. Some chocolates are pastries on conversion
3. (d) Only II follows. Statement (a) + statement (b) gives no Some pastries are chocolates. Hence, II follows. 1st
conclusion [ E + E = no conclusion]. Hence, I does statement + Some chocolates are pastries gives no
not follow. Note that I does not follow from statement conclusion. Hence, I and III do not follow but they
(c) either. Conclusion II follows from conversion of make a complementary (1-O) pair. Hence, either I or III
statement (b). Conclusion III does not follow from follows. IV does not follow from the last statement.
statement (a). Conclusion IV does not follow from OR
statement (c). Venn-Diagram Representation:
4. (d) No door is a lock ® on conversion ® No lock is a
door ® implication ® Some locks are not doors.
Hence, II follows and I does not. No door is a lock +
Some locks are keys = Some keys are not doors. Hence,
III follows. IV does not follow from the first statement.
5. (c) First + second statement gives conclusion I.
Conclusion II follows as conversion of conclusion I.
Third statement, on conversion, gives conclusion IV
but not conclusion III. Conclusions:
6. (d) I does not follow from the last statement, on conversion.
II does not follow from the second statement, on I False
conversion. III does not follow from the first statement, II. True –Either (Complementary
on conversion. 1st + 2nd + 3rd statement gives III. False pair) I–O type
conclusion IV. IV. False
OR
Venn-Diagram Representation: 9. (d) Some fruits are towers (I) + All towers are windows
(A) Some fruits are windows (I) ( I + A = I). Now,,
Mopeds conversion of “Some fruits are windows” gives
Scooties conclusion III.
Scooters OR
Bikes
Venn-Diagram Representation:

Windows
Fruits

Birds Towers

Conclusions:
I. False
II. False Conclusions:
III. False I. False
IV. True II. False
7. (b) As all the statements are I-type, no conclusion is III. True
possible from their combinations. Hence, none follows. IV. False
y
o
u
A-98 Syllogism

rs
m
10. (a) Conclusions I and III make a complementary pair. Conversion of “Some stars are hills” gives conclusion

a
Hence, either I or III follows. Conclusion II does not II. Conclusion III does not follow because statement

h
b
follow since “No train is a dog” (E) + “All dogs are (a) + statement (b) gives no conclusion. Conclusion

o
parrots” (A) gives the conclusion “Some parrots are IV follows from statement (a).

o
b
not trains” [ E + A O ]. Conclusion IV does not 17. (d) Statement (a) + Statement (b) gives the conclusion

.w
follow because “Some buses are trains” + “No train is “Some cats are bats” [ I + A = I]. “Some cats are

o
rd
a dog” gives the conclusion “Some buses are not bats” on conversion “Some bats are cats”.

p
dogs” [ I + E = O]. Hence, conclusion II follows. Conclusion III does not

re
11. (d) “Some flowers are boxes” (I) + “All boxes are tigers” follow because statement (b) + Statement (c) gives no

s
s
conclusion [ A + I = no conclusion]. Conclusions I

.c
(A) gives conclusion “Some flowers are tigers” (I)

o
[ I + A = I]. On conversion, we get “some tigers are and IV do not follow because statement (b) + statement

m
flowers”. Hence, conclusion II follows but IV does not (c) gives no conclusions. But conclusions I and IV
follow. “Some cups are flowers” (I) + “Some flowers make a complementary pair (IE-type). Hence, either I
are boxes” (I) gives no conclusion [ I + I= No or II follows.
conclusion]. Hence, III does not follow. No relation is OR
given between tigers and cups and hence, I does not Venn-Diagram Representation:
follow.
Bats
12. (d) Only II and IV follows.
13. (d) 1st + 2nd statements gives conclusion III. 2nd + 3rd Jungles Rats Cats
statements gives conclusion IV. I and II do not follow.
14. (a) Statement (a) + Statement (b) gives no conclusion
[ I + I = no conclusion]. Hence, conclusion II does Conclusions:
not follow. Conclusions I and IV do not follow because I. False
no conclusion can be obtained regarding candles and II. True
trains. Statement (b) + Statement (c) gives no –Either Complementary
III. False
conclusion [ I + I = no conclusion]. Hence, conclusion IV. False pair (IE type)
III does not follow. 18. (d) Only I and III follow. Conclusions I and III follow from
15. (b) Statement (a) + Statement (b) gives the conclusion statement (a) and statement (c) respectively [on
“Some stones are not trees.” [ E + A = O ]. Hence, conversion]. But conclusions II and IV do not follow
conclusion I does not follow. Statement (b) + Statement because A-type statement can’t be converted into A-
(c) gives the conclusion “All fruits are rains”. On type.
conversions it gives conclusion III. Now, statement 19. (a) Some boxes are balls + No ball is a bat (conversion of
(a) + “All fruits are rains” gives the conclusion “Some 2nd statement) = Some boxes are not bats [I + E = O].
rains are not trees” [ E + A = O ]. Conclusions II Hence, I follows and III does not. Now, some spectacles
and IV do not follow but these two conclusions make are boxes and conclusion I give no conclusion. Hence,
a complementary pair (El-type). Hence, either II and IV do not follow.
conclusion II or conclusion IV follows. 20. (c) Some bulbs are fans + All fans are tubelights = Some
OR bulbs are tubelights ... (a) [I + A = I]. Now, statement (b)
Venn-Diagram Representation: + (a) gives: .Some tubelights are not pens. Hence,
conclusions I and II can’t be established. III follows
Rains from first statement on conversion. But IV does not.
Stones But I and II make a complementary pair [I – E pair].
Tree
Hence, either I or II follows.
Fruits
OR
Venn-Diagram Representation:

Tubelights
Conclusions: Fans Pen
Bulbs
I. False
II. False
– Either Complementary
III. True
pair (EI type) Conclusions:
IV. False
16. (d) Statement (a) + Statement (b) gives no conclusion I. False –Either Complementary
[ A + I = no conclusion]. Hence, conclusion I does II. False pair (I-E pair)
not follow. Statement (b) + Statement (c) gives the III. True
conclusion “Some stars are hills” [ I + A = I]. IV. False
y
o
u
Syllogism A-99

rs
m
21. (b) conversion, gives: Some trains are trucks. Hence I does

a
not follow. The last two statements gives conclusion

h
Fruits IV.

b
o
Drinks 28. (a) 1st + 2nd statement gives no conclusion. Hence I does

o
b
not follow. 2nd + last statement gives: Some chairs are

.w
Liquids Vegetables
not fruits. Hence II does not follow. III does not follow

o
from combining all. IV does not follow from the last

rd
statement, on conversion.

p
re
29. (b)

s
s
Conclusions:

.c
I. False

o
m
II. True
III. True
IV. False
22. (d)
Trousers
Shirts

Shoes Socks

By looking at above venn diagram, we can concluded


that only conclusions II follow.
Conclusions:
I. True 31. (a) Conclusion I is the conversion of first statement, hence
I follows. But II does not follow because A + A = A i.e.
II. False
All leaders are good orators but not vice versa.
III. True 32. (a) A + A = A; i.e. All terrorists are human.
IV. False 33. (b) I does not follow. But II follows because it is conversion
23. (d) Only IV follows from statement (c) on conversion. of the first statement.
Records
Books 34. Books Pens Pencil
Plates
Papers
Conclusions:
Conclusions: I. False
I. False II. False
II. False
Angles
III. False 35. (d)
IV. True. Conversion of III Statement Social Dedicated
Workers souls
24. (b) III follows from the first statement, on conversion. 1st
statement + 2nd statement gives: Some bricks are
pens...(A) on conversion Some pens are bricks.
Hence II follows. (A) + 3rd statement gives conclusion Conclusions:
IV, which on conversion, give conclusion I. I. False –Either Complementary
25. (c) III follows from the last statement, on conversion. 1st II. False pair (I-E pair)
statement + 2nd statement gives: No cup is water ... 36. (c) Uncertain
(A). Hence IV does not follow from A, on conversion.
Red
(A) + last statement gives: Some clothes are not cups.
Hence, I does not follow. 2nd statement + last statement Statement 1: Roses True ( )
gives: Some clothes are not tables. Hence II does not
follow.
26. (d) As all the statements are I-type, hence no conclusion
follows from their combinations. But I and IV make a Yellow
complementary pair, hence either I or IV follows.
27. (d) 1st statement + 2nd statement gives: All trucks are Statement 2: marigold True ( )

cars ... (A). Hence II does not follow, on conversion.


(A) + last statement gives conclusion III. III, on
y
o
u
A-100 Mathematical Operation

rs
m
a
CHAPTER

h
MATHEMATICAL

b
13

o
o
b
.w
OPERATION

o
rd
p
re
s
s
.c
Under this type of problem, usually mathematical symbol are 3. Select correct combination of mathematical sign

o
EXAMPLE

m
converted into another form by either interchanging the symbol to replace ‘*’ sign to balance the equation.
or using different symbol in place of usual symbol and then 9 * 4 * 22 * 14
calculate the equation according to the given condition. (a) × = – (b) × – =
EXAMPLE 1. If ‘+’ stands for division, ‘×’ stands for addition, (c) = – × (d) – × =
Sol. (b) 9 * 4 * 22 * 14
‘–’ stands for multiplication, and ‘ ’ stands for subtraction, then
9 × 4 – 22 = 14
which of the following equation is correct?
(a) 36 × 6 + 7 2 – 6 = 20 EXAMPLE 4. means is ‘bigger than’
(b) 36 + 6 – 3 × 5 3 = 24 means is ‘smaller than’
(c) 36 6 + 3 × 5 – 3 = 45 means is ‘equal to’
(d) 36 – 6 + 3 × 5 3 = 74 × means ‘plus’
Sol. (d) 36 × 6 3 + 5 – 3 = means ‘minus’
36 × 2 + 5 – 3 = 74
If a c and b× d c, then
EXAMPLE 2. If P denotes ‘+’, Q denotes ‘–’, R denotes ‘×’
(a) d a (b) a d
and S denotes ‘ ’ then, which of the following statement is correct?
(a) 16 R 12 P 49 S 7 Q 9 = 200 (c) b c (d) d a
(b) 32 S 8 R 9 = 160 Q 12 R 12 Sol. (d) a > c and b + d = c
(c) 8 R 8 P 8 S 8 Q = 57 a > b + d and this is true only if, d < a.
(d) 36 R 4 S 8 Q 7 P 4 = 10 d a
Sol. (c) 8 R 8 P 8 S 8 Q EXAMPLE 5. Given interchange : sign ‘+’ and ‘–’and numbers
=8×8+8 8–8 5 and 8. Which of the following is correct?
8 (a) 82 – 35 + 55 = 2 (b) 82 – 35 + 55 = 102
=8×8+ –8
8 (c) 85 – 38 + 85 = 132 (d) 52 – 35 + 55 = 72
= 64 + 1 – 8 = 57 Sol. (a) 52 + 38 – 88 = 2

1. If ‘+’ means ‘–’ ‘–’ means ‘×’ ‘ ’ means ‘+’ and ‘×’ means ‘ ’, 5. Find out the correct answer for the unsolved equation on
then the basis of the given equations.
10 × 5 3 – 2 + 3 = ? If 6 * 5 = 91, 8 * 7 = 169, 10 * 7 = 211, then11 * 10 = ?
(a) 331 (b) 993 (c) 678 (d) 845
53
(a) 5 (b) 21 (c) (d) 18 6. If ‘–’ stands for division, ‘+’ for multiplication ‘ ’for
3 subtraction and ‘×’ for addition. Which one of the following
2. If ‘+’ means ‘ ’ , ‘–’ means ‘×’, ‘ ’ means ‘+’ and ‘×’ means equation is correct?
‘–’, then 63 × 24 + 8 4 + 2 – 3 = ? (a) 6 20 × 12 + 7 – 1 = 70 (b) 6 + 20 – 20 7 × 1 = 62
(a) 54 (b) 66 (c) 186 (d) 48 (c) 6 – 20 12 × 7 + 1 = 57 (d) 6 + 20 – 20 7 – 1 = 38
3. Which one of the following is correct? 7. In an imaginary mathematical operation ‘+’ means
6 * 4 * 9 * 15 multiplication, ‘×’ means subtraction, ‘ ’means addition and
(a) ×, = , – (b) ×, –, = (c) =, ×, – (d) –, ×, = ‘–’ means division. All other rules in mathematical operation
4. If > = , = ×, < = +, = –, + = <, × = =, – = > are the same as in the existing system.
(a) 6 > 2 > 3 8 4 + 13 (b) 6 2 < 3 > 8 < 4 – 13 Which one of the following gives the result of
(c) 6 2 < 3 8 > 4 × 13 (d) 6 > 2 3 < 8 4 + 13 175 – 25 5 + 20 × 3 + 10 = ?
(a) 160 (b) 2370 (c) 7 7 (d) 240
y
o
u
Mathematical Operation A-101

rs
m
8. If L stands for +, M stands for –, N stands for ×, P stands for 21. If ‘+’ means ‘×’ ; ‘–’ means ‘÷’ ; ‘ ×’ means ‘–’ and ‘÷’ means

a
, then 14 N 10 L 42 P 2 M 8 = ? ‘+’ then 9 + 8 ÷ 8 – 4 × 9 = ?

h
(a) 153 (b) 216 (c) 248 (d) 251 (a) 26 (b) 17 (c) 65 (d) 11

b
o
9. It being given that: > denotes +, < denotes –, + denotes , – 22. If ‘÷’ means ‘+’ ; ‘–’ means ‘×’ ; ‘+’ means ‘÷’ and ‘×’ means

o
b
denotes =, = denotes ‘less than’ and × denotes ‘greater ‘–’ then 20 ÷ 12 × 4 + 8 – 6 = ?

.w
than’. Find which of the following is a correct statement.
2

o
(a) 3 + 2 > 4 = 9 + 3 < 2 (b) 3 > 2 > 4 = 18 + 3 < 1 8

rd
(a) (b) 29 (c) 32 (d) 26
(c) 3 > 2 < 4 × 8 + 4 < 2 (d) 3 + 2 < 4 × 9 + 3 < 3 3

p
re
10. If ‘–’ stand for addition, ‘+’ stands for subtraction, ‘÷’ stands 23. If stands for ‘addition’ stands for ‘subtraction’

s
for multiplication and ‘×’ stands for division, then which

s
stands for ‘division ; stands for’ multiplication' stands

.c
one of the following equations is correct?

o
for equal to' then which of the following alternatives is

m
(a) 25 × 5 ÷ 20 – 27 + 7 = 120
(b) 25 + 5 × 20 – 27 ÷ 7 = 128 correct?
(c) 25 + 5 – 20 + 27 × 7 = 95 (a) 7 43 6 1 4
(d) 25 – 5 + 20 × 27 ÷ 7 = 100 (b) 3 6 2 3 6 5
11. If ‘×’ stands for ‘addition’, ‘<’ for substraction, + for division,
> for multiplication, – for ‘equal to’, for ‘greater than’ and (c) 5 7 3 2 5
‘=’ for ‘less than’, then state which of the following is true? (d) 2 5 6 2 6
(a) 3 × 4 > 2 – 9 + 3 < 3 (b) 5 × 3 < 7 8 + 4 × 1 24. If ‘×’ Stands for ‘ addition’ ‘<’ for subtraction’ ‘+’ for division'
(c) 5 > 2 + 2 = 10 < 4 × 8 (d) 3 × 2 < 4 16 > 2 + 4 > for multiplication’ ‘–’ for equal to’ ‘+’ for ‘ greater than’
DIRECTIONS (Qs. 12 - 15): In an imaginary language, the digits and '=' for ' less than' state which of the following is true. ?
0, 1, 2, 3, 4, 5, 6, 7, 8 and 9 are substituted by a, b, c, d, e, f, g, h, (a) 3 × 4 > 2 – 9 + 3 < 3 (b) 5 × 3 < 7 8 + 4 × 1
i and j. And 10 is written as ba. (c) 5 > 2 + 2 = 10 < 4 × 8 (d) 3 × 2 < 4 16 > 2 + 4
12. (cd + ef) × bc is equal to 25. If means + – means ÷ × means – and + means × then
(a) 684 (b) 816 (c) 916 (d) 1564
36 x 4 8x 4
13. dc × f – (bf – d) × d is equal to =?
(a) abb (b) abe (c) bce (d) bcf 4 8 x 2 16 1
14. baf + fg – (ca × h/be) is equal to (a) 0 (b) 8 (c) 12 (d) 16
(a) 141 (b) 145 (c) 151 (d) 161 26. If × means +, – means × , ÷ means + and + means – then
15. baf bf × d is equal to (3 – 15 ÷ 19) × 8 + 6 = ?
(a) df (b) cb (c) be (d) d (a) – 1 (b) 2 (c) 4 (d) 8
16. In the following question, some relationship have been 27. If + means , – means ×, × means +, means –, then
expressed through symbols which are 90 + 18 – 6 × 30 4 = ?
× = greater than = not less than (a) 64 (b) 65 (c) 56 (d) 48
÷ = less than = not greater than 28. Given below are numbers in the first line and symbols in the
+ = equal to = not equal to, second line. Numbers and symbols are code for each other.
then A B × C implies Choose the correct code for given symbols.
(a) B C (b) A ÷ C (c) A C (d) B C 1 2 3 4 5 6 7 8 9
17. If the given interchanges namely : signs + and ÷ and + – ×
numbers 2 and 4 are made in signs and numbers, which one Which number can be decoded from the following :
of the following four equations would be correct ?
×
(a) 2 + 4 ÷ 3 = 3 (b) 4 + 2 ÷ 6 = 1.5
(c) 4 ÷ 2 + 3 = 4 (d) 2 + 4 ÷ 6 = 8. (a) 5 8 6 3 7 (b) 5 6 8 7 3
(c) 5 7 8 6 3 (d) 5 8 3 6 7
18. If L denotes ×, M denotes ÷ , P denotes + and Q denotes
29. If '–' stands for '+', '+' stands for '×', '×' stands for '–' then
–, than 8 P 36 M 6 Q 6 M 2 L 3 = ?
which one of the following is not correct ?
13 1 1 (a) 22 + 7 – 3 × 9 = 148 (b) 33 × 5 – 10 + 20 = 228
(a) (b) – (c) 14 (d) 5
6 6 2 (c) 7 + 28 –3 × 52 = 127 (d) 44 – 9 + 6 × 11 = 87
19. If × stands for ‘addition’, < for ‘substraction’, + stands for 30. Some equations are solved on the basis of a certain system.
‘division’, > for ‘multiplication’, –, stands for ‘equal to’, Find the correct answer for the unsolved equation on that
for ‘greater than’ and = stands for ‘less than’, state which basis.
of the following is true ? 5 * 6 = 35, 8 * 44 = 28, 6 * 8 = ?
(a) 3 × 2 < 4 16 > 2 + 4 (b) 5 > 2 + 2 = 10 < 4 × 2 (a) 46 (b) 34 (c) 23 (d) 38
(c) 3 × 4 > 2 – 9 + 3 < 3 (d) 5 × 3 < 7 8 + 4 × 1 31. Select the correct combination of mathematical signs to
20. If ‘20 – 10’ means 200, ‘8 ÷ 4’ means 12, ‘6 × 2’ means 4 and replace * signs and to balance the following equation.
‘12 + 3’ means 4, then 12 * 3 * 4 = 6 * 8 * 8
100 – 10 × 1000 ÷ 1000 + 100 × 10 = ? (a) +, ×, –, × (b) ×, +, –, ×
(a) 1090 (b) 0 (c) 1900 (d) 20 (c) ×, +, ×, – (d) ×, –, ×, +
y
o
u
A-102 Mathematical Operation

rs
m
32. Which of the following interchange of signs would the (a) + × = (b) + ×=

a
equation correct ? (c) + × – = (d) × =

h
6 × 4+2=16 34. Select the correct combination of mathematical sings to

b
o
(a) + and ×, 2 & 4 (b) +and ×, 4 & 6 replace * sings and to balance the following equation :

o
b
(c) + and ×, 2 & 6 (d) +and ×, 3 & 4 8 *5 * 10 * 2 * 25

.w
33. Select the correct combination of mathematical sings to (a) + × = (b) + – =

o
replace the * sings and to balance the following equation : (c) × + = × (d) × – = ×

rd
(45*3*6)*6*15

p
re
s
s
.c
ANSWER KEY

o
m
1 (a) 6 (a) 11 (c) 16 (a) 21 (c) 26 (b) 31 (c)
2 (b) 7 (c) 12 (b) 17 (d) 22 (b) 27 (c) 32 (b)
3 (b) 8 (a) 13 (c) 18 (d) 23 (d) 28 (a) 33 (d)
4 (c) 9 (c) 14 (c) 19 (b) 24 (c) 29 (c) 34 (c)
5 (a) 10 (a) 15 (b) 20 (b) 25 (a) 30 (a)

– – 7. (c) , –
1. (a) ,–
10 × 5 3 – 2 + 3 = ? Given expression 175 – 25 5 + 20 × 3 + 10
or, ? = 10 5 + 3 × 2 – 3 After conversion 175 25 + 5 × 20 – 3 × 10
or, ?= 2+6–3 =5 = 7 + 100 – 30
= 77
– 8. (a) Using the proper signs, we get
2. (b)
– Given expression = 14 × 10 + 42 2 – 8
= 140 + 21 – 8
63 × 24 + 8 4 + 2 – 3 = ?
= 153
or, ? = 63 – 24 8 + 4 2 × 3
9. (c) Using proper notations, we have:
or, ? = 63 – 3 + 2 × 3
or, ? = 66 11
(a) Given statement is 3 2 + 4 < 9 3 – 2 or < 1 not true
3. (b) 6 × 4 – 9 = 15 2
4. (c) (b) 3 + 2 + 4 < 18 3 – 1 or 9 < 5, which is not true.
(c) 3 + 2 – 4 > 8 4 – 2 or 1 > 0, which is true
5
6 × 2 + 3 – 8 4 = 13 (d) 3 2 – 4 > 9 3 – 3 or > 0, which is not true.
2
6 × 2 + 3 – 2 = 13
10. (a) Solve by options, we can check all the options one by
12 + 3 – 2 = 13
one.
15 – 2 = 13
25 5 × 20 + 27 – 7 5 × 20 + 27 – 7 100 + 27 – 7
5. (a) 6 × 5 = 30, 30 × 3 + 1 = 91, 8 × 7 = 56, 56 × 3 + 1 =
120 = 120
169, 10 × 7 = 70, 70 × 3 + 1 = 211
11. (c) Using the proper notations in (c), we get the statement
Similarly 11 × 10 = 110, 110 × 3 + 1 = 331
as:-
6. (a) – , 5 × 2 2 < 10 – 4 + 8
or, 5 × 1 < 18 – 4
–,
or 5 < 12 which is true.
Option (a) : 6 20 × 12 + 7 – 1 = 70 12. (b) Using the correct symbols, we have
L.H.S. = 6 – 20 + 12 × 7 1 Given expression = (23 + 45) × 12 = 68 × 12 = 816.
= 6 – 20 + 84 13. (c) Given expression = 32 × 5 – (15 – 3) × 3
= 90 – 20 = 70 R. H.S. = 160 – 12 × 3
= 160 – 36 = 124 = bce
y
o
u
Mathematical Operation A-103

rs
m
14. (c) Given expression = 105 + 56 – (20 × 7/14) 29. (c) By options –

a
= 105 + 56 – 10 (a) 22 × 7 + 3 – 9 = 148

h
= 151

b
154 + 3 – 9

o
15. (b) Given expression = 105 15 × 3 157 – 9 = 148 (correct)

o
b
= 7 × 3 = 21 = cb (b) 33 – 5 + 10 × 20 = 228

.w
16. (a) A B × C 33 – 5 + 200

o
A B;B×C

rd
200 + 33 – 5
A B; B> C

p
233 – 5 = 228 (correct)

re
Hence, option (a) implies the given equation. (c) 7 × 28 + 3 – 52 = 127

s
17. (d) Interchanging (+ and ÷) and (2 and 4), we get :

s
196 + 3 – 52

.c
(1) 4 ÷ 2 + 3 = 3 or 5 = 3, which is false

o
199 – 52 = 147 (incorrect)

m
(2) 2 ÷ 4 + 6 = 1.5 or 6.5 = 1.5, which is false.
(d) 44 + 9 × 6 – 11 = 87
10 44 + 54 – 11
(3) 2 + 4 ÷ 3 = 4 or = 4, which is false.
3 98 – 11 = 87 (correct)
(4) 4 ÷ 2 + 6 = 8 or 8 = 8, which is true. 5 * 6/2 = 3 5
18. (d) Using the correct symbols, we have: 30. (a)
Given expression = 8 + 36 ÷ 6 – 6 ÷ 2 × 3
8 * 4/2 = 2 8
=8+6–3×3=5
19. (b) Using the proper notations in (2), we get the statement
as 5 × 2 ÷ 2 < 10 – 4 + 2 or 5 < 8 , which is true. 6 * 8/2 = 4 6
20. (b) Since, 20 × 10 = 200, therefore, – means ×
8 + 4 = 12, therefore, ÷ means +. 31. (c) By options,
6 – 2 = 4, therefore, × means – . (a) 12 + 3 × 4 = 6 – 8 × 8
and 12 ÷ 3 = 4, therefore, + means ÷. 12 + 12 = 6 – 64
Now, given expression 24 = 58 (incorrect)
= 100 × 10 – 1000 + 1000 ÷ 100 – 10 58 > 24
= 100 0 – 1000 + 10 – 10 = 0 (b) 12 × 3 + 4 = 6 – 8 × 8
21. (c) 9 × 8 + 8 ÷ 4 – 9 = 65 36 + 4 = 6 – 64
22. (b) 20 + 12 – 4 ÷ 8 × 6 = 29 40 = 58 (incorrect)
23. (d) Using the proper notations in (4) we get the statement 58 > 48
as 2 × 5 – 6 + 2 = 6 or 10 – 6 + 2 = 6 or 6 = 6, which is (c) 12 × 3 + 4 = 6 × 8 – 8
true.
36 + 4 = 48 – 8
24. (c) Using the proper notations in (3), we get the statement
40 = 40 (correct)
as 5 × 2 2 < 10 – 4 + 8 or 5 × 1 < 18 – 4 or 5 < 14,
(d) 12 × 3 – 4 = 6 × 8 + 8
which is true.
25. (a) Using the correct symbols, we have 36 – 4 = 48 + 8
Given expression 32 = 56 (incorrect)
56 > 32
36 4 8 4 32 8 4 4 4 32. (b) 4 6 2 16
= = = = 0.
4 8 2 16 1 32 32 1 0 1 33. (d) (45 3 × 6) 6 = 15
26. (b) Using the correct symbols, we have 34. (c) 8 × 5 + 10 = 2 × 25
Given expression = (3 × 15 + 19) ÷ 8 – 6 50 = 50
= 64 ÷ 8 – 6 = 8 – 6 = 2.
27. (c) 90 18 × 6 + 30 – 4 = 56
28. (a) #

Decoded as 5 8 6 3 7
y
o
u
A-104 Arithmetical Reasoning

rs
m
a
CHAPTER

h
ARITHMETICAL

b
14

o
o
b
.w
REASONING

o
rd
p
re
s
s
.c
Arithmetical Reasoning tests the ability to solve basic arithmetic 3.

o
EXAMPLE
problems encountered in everyday life. These problems require

m
basic mathematical skills like addition, subtraction, multiplication, A student got twice as many sums wrong as he got right. If
division etc. The tests include operations with whole numbers, he attempted 48 sums in all, how many did he solve correctly?
rational numbers, ratio and proportion, interest and percentage, (a) 12 (b) 16
and measurement. Arithmetical reasoning is one factor that helps (c) 18 (d) 24
characterize mathematics comprehension, and it also assesses
logical thinking. Sol. (b) Suppose the boy got x sums right and 2x sums wrong.
Then, x + 2x = 48, 3x = 48, x = 16.
EXAMPLE 1.
The total of the ages of Amar, Akbar and Anthony is 80 EXAMPLE 4.
years. What was the total of their ages three years ago ? In a group of cows and hens, the number of legs are 14 more
(a) 71 years (b) 72 years than twice the number of heads. The number of cows is
(c) 74 years (d) 77 years (a) 5 (b) 7
Sol. (a) Required sum = (80 – 3 x 3) years = (80 – 9) years (c) 10 (d) 12
= 71 years. Sol. (b) Let the number of cows be x and the number of hens
be y.
EXAMPLE 2. Then, 4x + 2y = 2 (x + y) + 14, 4x + 2y = 2x + 2y + 14,
Two bus tickets from city A to B and three tickets from city 2x = 14, x = 7.
A to C cost Rs. 77 but three tickets from city A to B and two
tickets from city A to C cost Rs. 73. What are the fares for EXAMPLE 5.
cities B and C from A ? Rani, Reeta , Sukhada, Jane and Radhika are friends. Reeta
(a) `4, `23 (b) `13, `17 is 18 years of her age, Radhika is younger to Reeta, Rani is
in between Radhika and Sukhada while Reeta is in Between
(c) `15, `14 (d) `17, `13 Jane and Radhika. If there be a difference of two years
Sol. (b) Let Rs. x be the fare of city B from city A and Rs. y be between the ages of girls from eldest to the youngest, how
the fare of city C from city A. old is Sukhada?
Then, 2x + 3y = 77 ...(i) (a) 10 years (b) 12 years
and 3x + 2y = 73 ...(ii) (c) 14 years (d) 16 years
Multiplying (i) by 3 and (ii) by 2 and subtracting, we get: Sol. (b) Arranging them on the basis of their ages,
5y = 85 or y = 17. Jane>Reeta>Radhika>Rani>Sukhada
Putting y = 17 in (i), we get: x = 13. If Reeta is 18 years old then Sukhada is 12 years

1. The 30 members of a club decided to play a badminton 2. A tailor had a number of shirt pieces to cut from a roll of
singles tournament. Every time a member loses a game he is fabric. He cut each roll of equal length into 10 pieces. He cut
out of the tournament. There are no ties. What is the at the rate of 45 cuts a minute. How many rolls would be cut
minimum number of matches that must be played to in 24 minutes?
determine the winner? (a) 32 rolls (b) 54 rolls
(a) 15 (b) 29 (c) 108 rolls (d) 120 rolls
(c) 61 (d) None of these
y
o
u
Arithmetical Reasoning A-105

rs
m
3. In a class of 60 students, the number of boys and girls 12. I have a few sweets to be distributed. If I keep 2, 3 or 4 in a

a
participating in the annual sports is in the ratio 3 : 2 pack, I am left with one sweet. If I keep 5 in a pack, I am left

h
respectively. The number of girls not participating in the with none. What is the minimum number of sweets I have to

b
o
sports is 5 more than the number of boys not participating pack and distribute ?

o
b
in the sports. If the number of boys participating in the (a) 25 (b) 37 (c) 54 (d) 65

.w
sports is 15, then how many girls are there in the class? 13. Mr. X, a mathematician, defines a number as 'connected

o
rd
(a) 20 (b) 25 with 6 if it is divisible by 6 or if the sum of its digits is 6, or

p
(c) 30 (d) Data inadequate if 6 is one of the digits of the number. Other numbers are all

re
4. At a dinner party every two guests used a bowl of rice 'not connected with 6'. As per this definition, the number of

s
s
between them, every three guests used a bowl of daal integers from 1 to 60 (both inclusive) which are not

.c
connected with 6 is

o
between them and every four used a bowl of meat between

m
them. There were altogether 65 dishes. How many guests (a) 18 (b) 22 (c) 42 (d) 43
were present at the party ? 14. A player holds 13 cards of four suits, of which seven are
(a) 60 (b) 65 black and six are red. There are twice as many diamonds as
(c) 90 (d) None of these spades and twice as many hearts as diamonds. How many
5. In a family, each daughter has the same number of brothers clubs does he hold ?
as she has sisters and each son has twice as many sisters (a) 4 (b) 5 (c) 6 (d) 7
as he has brothers. How many sons are there in the family? 15. Nitin's age was equal to square of some number last year
(a) 2 (b) 3 (c) 4 (d) 5 and the following year it would be cube of a number. If
6. In a garden, there are 10 rows and 12 columns of mango again Nitin's age has to be equal to the cube of some number,
trees. The distance between the two trees is 2 metres and a then for how long he will have to wait?
distance of one metre is left from all sides of the boundary (a) 10 years (b) 38 years
of the garden. The length of the garden is (c) 39 years (d) 64 years
(a) 20 m (b) 22 m (c) 24 m (d) 26 m 16. At the end of a business conference the ten people present
7. In a family, the father took 1/4 of the cake and he had 3 times all shake hands with each other once. How many
as much as each of the other members had. The total number handshakes will there be altogether ?
of family members is (a) 20 (b) 45 (c) 55 (d) 90
(a) 3 (b) 7 (c) 10 (d) 12 17. Anand, David , Karim and Mano are fans of games. Each
8. In three coloured boxes - Red, Green and Blue, 108 balls are has a different favourite game among hockey, chess, cricket
placed. There are twice as many balls in the green and red and football. David doesn't watch cricket and hockey
boxes combined as there are in the blue box and twice as matches. Anand doesn't like hockey, chess and cricket.
many in the blue box as there are in the red box. How many Mano doesn't watch cricket. Which is favourite game of
balls are there in the green box ? Karim?
(a) 18 (b) 36 (a) chess (b) cricket (c) football (d) hockey
(c) 45 (d) None of these 18. David gets onthe elevator at the 11th floor of a building and
9. A, B, C, D and E play a game of cards. A says to B, "If you rides up at the rate of 57 floors per minute. At the same time.
give me 3 cards, you will have as many as I have at this Albert gets on an elevator at the 51st floor of the same
moment while if D takes 5 cards from you, he will have as building and rides down at the rate of 63 floors per minute.
many as E has." A and C together have twice as many cards If they continue travelling at these rates, then at which
as E has. B and D together also have the same number of floor will their paths cross?
cards as A and C taken together. If together they have 150 (a) 19 (b) 28 (c) 30 (d) 37
cards, how many cards has C got ? 19. A fibres 5 shots to B’s 3 but A kills only once in 3 shots
(a) 28 (b) 29 (c) 31 (d) 35 while B kills once in 2 shots. When B has missed 27 times,
10. A man wears socks of two colours - Black and brown. He A has killed
has altogether 20 black socks and 20 brown socks in a (a) 30 birds (b) 60 birds
drawer. Supposing he has to take out the socks in the dark, (c) 72 birds (d) 90 birds
how many must he take out to be sure that he has a matching 20. First bunch of bananas has (1/4) again as many bananas as
pair ? a second bunch. If the second bunch has 3 bananas less
(a) 3 (b) 20 than the first bunch, then the number of bananas in the first
(c) 39 (d) None of these bunch is
11. Nithya is Sam’s Sister. Mogan is Sam’s Father. Selvan is (a) 9 (b) 10 (c) 12 (d) 15
Rajan’s Son. Rajan is Mogan’s Brother. How is Nithya 21. A boy’s age is one fourth of his father’s age. The sum of the
related to Selvan? boy’s age and his father’s age is 35. What will be father’s
(a) Daughter (b) Sister age after 8 years?
(c) Cousin (d) Wife (a) 15 (b) 28 (c) 35 (d) 36
y
o
u
A-106 Arithmetical Reasoning

rs
m
22. If 1 candle in box number 1 is placed in box number 2, then 23. A boat moves from a jetty towards East. After sailing for 9

a
box-2 has twice the number of candles that box 1 has. nautical miles, she turns towards right and covers another

h
If 1 candle from box-2 is placed in box-1, the box-2 and box- 12 nautical miles. If she wants to go back to the jetty, what

b
o
1 have the same number of candles. is the shortest distance now from her present position ?

o
b
How many candles were there in box-1 and box-2 ? (a) 21 nautical miles (b) 20 nautical miles

.w
Box--1 Box-2 Box-1 Box-2
(c) 18 nautical miles (d) 15 nautical miles

o
rd
(a) 5:3 (b) 7:5

p
re
(c) 6:4 (d) 5:7

s
s
.c
o
ANSWER KEY

m
1 (b) 4 (a) 7 (c) 10 (a) 13 (d) 16 (b) 19 (a) 22 (d
2 (d) 5 (b) 8 (d) 11 (c) 14 (c) 17 (b) 20 (d) 23 (d)
3 (c) 6 (c) 9 (a) 12 (a) 15 (b) 18 (c) 21 (d)

1. (b) Clearly, every member except one (i.e. the winner) must Then, we have :
lose one game to decide the winner. Thus, minimum d – 1 = s and 2 (s – 1) = d.
number of matches to be played = 30 – 1 = 29. Solving these two equations, we get: d = 4, s = 3.
2. (d) Number of cuts made to cut a roll into 10 pieces = 9. 6. (c) Each row contains 12 plants.
Therefore required number of rolls = (45 × 24)/9 = 120. There are 11 gapes between the two corner trees
3. (c) Let the number of boys and girls participating in sports (11 × 2) metres and 1 metre on each side is left.
be 3x and 2x respectively. Therefore Length = (22 + 2) m = 24 m.
Then, 3x = 15 or x = 5. 7. (c) Let there be (x + 1) members. Then,
So, number of girls participating in sports = 2x = 10. 1
Number of students not participating in sports Father’s share = , share of each other member
4
= 60 – (15 + 10) = 35.
3
Let number of boys not participating in sports be y. = .
Then, number of girls not participating in sports 4x
= (35 – y). 3 1
Therefore (35 – y) = y + 5 3 4x 36 x 9
4x 4
y = 15. Hence, total number of family member = 10.
So, number of girls not participating in sports 8. (d) Let R, G and B represent the number of balls in red,
= (35 – 15) = 20. green and blue boxes respectively.
Hence, total number of girls in the class Then, R + G + B = 108 ...(i)
= (10 + 20) = 30. G + R = 2B ...(ii)
4. (a) Let the number of guests be x. Then number of bowls B = 2R ...(iii)
x x From (ii) and (iii), we have G + R = 2 × 2R = 4R or G = 3R.
of rice = ; number of bowls of dal = ; number of Putting G = 3R and B = 2R in (i), we get:
2 3
R + 3R + 2R = 108 6R = 108 R = 18.
x Therefore Number of balls in green box = G = 3R
bowls of meat = .
4 = (3 × 18) = 54.
9. (a) Clearly, we have :
x x x
65 A=B–3 ...(i)
2 3 4 D + 5=E ...(ii)
6x 4x 3x A + C = 2E ...(iii)
65 13x 65 12 B + D = A + C = 2E ...(iv)
12
A+B + C + D + E=150 ...(v)
65 12 From (iii), (iv) and (v), we get: 5E = 150 or E = 30.
x 60 Putting E = 30 in (ii), we get: D = 25.
13
Putting E = 30 and D = 25 in (iv), we get: B = 35.
5. (b) Let d and s represent the number of daughters and
Putting B = 35 in (i), we get: A = 32.
sons respectively.
Putting A = 32 and E = 30 in (iii), we get: C = 28.
y
o
u
Arithmetical Reasoning A-107

rs
m
10. (a) Since there are socks of only two colours, so two out 1
Number of floors covered by David in min

a
of any three socks must always be of the same colour.

h
3
11. (c) Nithya is Sam’s Sister and Mogan is Sam’s Father

b
o
Nithya is Mogan’s Daughter. 1
57 19 .

o
=

b
Selvan is Rajan’s Son and Rajan is Mogan’s 3

.w
Brother So, their paths cross at (11 + 19)th i.e., 30th floor.

o
Selvan is Mogan’s Nephew. 19. (a) Let the total number of shots be x. then,

rd
So, Nithya is Selvan’s Cousin. 5 3

p
Shots fired by A = x ; Shots fired by B = x

re
12. (a) Clearly, the required number would be such that it leaves 8 8

s
a remainder of 1 when divided by 2, 3 or 4 and no

s
1 5 5x

.c
remainder when divided by 5. Such a number is 25. Killing shots by A = of x = ;

o
13. (d) Numbers from 1 to 60, which are divisible by 6 are : 3 8 24

m
6,12,18, 24, 30, 36,42, 48, 54, 60.There are 10 such 1 3 3x
numbers. Shots missed by B = of x = .
2 8 16
Numbers from 1 to 60, the sum of whose digits is 6 are
3x 27 16
: 6, 15, 24, 33, 42, 51, 60. 27 or x 144
There are 7 such numbers of which 4 are common to 16 3
the above ones. So, there are 3such uncommon 5x 5
numbers. Birds killed by A = 144 30
24 24
Numbers from 1 to 60, which have 6 as one of the
20. (d) Let the number of bananas in the second bunch be x
digits are 6, 16, 26, 36, 46, 56, 60.
Then, number of bananas in the first bunch
Clearly, there are 4 such uncommon numbers.
So, numbers 'not connected with 6' 1 5
=x+ x x
= 60 – (10 + 3 + 4) = 43. 4 4
14. (c) Clearly, the black cards are either clubs or spades while 5
the red cards are either diamonds or hearts. So, x x 3 5x – 4x = 12 x = 12
4
Let the number of spades be x. Then, number of clubs Number of bananas in the first bunch
= (7 – x).
Number of diamonds = 2 x number of spades = 2x; = 5 12 15
Number of hearts = 2 x number of diamonds = 4x. 4
Total number of cards = x + 2x + 4x + 7 – x – 6x + 7. 21. (d) Let father’s age is x yr.
Therefore 6x + 7 = 13 6x = 6 x – 1. x
Hence, number of clubs = (7 – x) = 6. Son’s age is yr..
4
15. (b) Clearly, we have to first find two numbers whose dif- x
ference is 2 and of which the smaller one is a perfect x 35 x = 28 yr..
4
square and the bigger one a perfect cube. Father’s age after 8 year is 36 years.
Such numbers are 25 and 27.
22. (d) Going by options; Box 1 Box 2
Thus, Nitin is now 26 years old. Since the next perfect
cube after 27 is 64, 5 : 7
so required time period = (64 – 26) years = 38 years. If 1 cande in box number is placed in box number 2
16. (b) Clearly, total number of handshakes = (9+ 8 + 7 + 6 + 5 then
+ 4 + 3 + 2+1) = 45. Box 1 Box 2
17. (b) 4 : 8
Therefore, Box 2 has twice the number of candles than
Games box 1.
Person Hockey Chess
If 1 candle from box 2 is palced in box-1
Cricket Football
Box 1 Box 2
Then- Hence, Both boxer have the
6 : 6:
Anand × × ×
same numbers of candles.
David × × × 23. (d) The shortest distance
Karim × × × Starting point 9 nm
AB2 BC2
Mano × × × A B
92 122
18. (c) Suppose their paths cross after x minutes. 225 12 nm
1 = 15 neutical miles
Then, 11 + 57x = 51 – 63 x 120 x = 10 x=
3 C
Ending point
y
o
u
A-108 Matrix Based Reasoning

rs
m
a
CHAPTER
MATRIX BASED

h
b
15

o
o
b
.w
REASONING

o
rd
p
re
s
s
.c
o
In this type of questions two matrices are given. In each matrix 3. LANKA

m
there are 25 cells and these cells contain two classes of alphabets.
The columns and rows of matrix I are numbered from 0 to 4 and (a) 97, 43, 59, 55, 01 (b) 97, 21, 58, 87, 01
that of matrix II from 5 to 9. A letter from these matrices can be (c) 69, 31, 76, 75, 01 (d) 78, 32, 89, 86, 12
represented first by its row number and next by its column number.
For example. ‘A’ Can be represented by 32 or 43. 4. LIKE

Matrix I (a) 57, 14, 76, 10 (b) 78, 00, 67, 03

(c) 78, 33, 87, 03 (d) 85, 00, 75, 40


0 1 2 3 4
0 I A U E O 5. RULE
1 E U O A I (a) 95, 02, 58, 30 (b) 79, 24, 85, 98
2 O A I E U
(b) 66, 11, 78, 41 (d) 79, 11, 86, 30
3 E U A O I
4 E I O A U 6. KRONE

Matrix II (a) 87, 79, 03, 89, 23 (b) 75, 97, 20, 89, 23

(c) 87, 56, 04, 76, 23 (d) 58, 88, 12, 59, 10
5 6 7 8 9
5 K R L M N Answers with Explanation
6 M R K N L
For questions 1 to 6 -
7 K N M L R
8 M L K R N A 01, 13, 21, 32, 43
9 N R L K M
E 03, 10, 23, 30, 40
EXAMPLE
I 00, 14, 22, 34, 41
Directions: In each of the following questions find out the correct
set of number pairs for the given word from the two matrices O 04, 12, 20, 33, 42
given above.
U 02, 11, 24, 31, 44
1. MONK
K 55, 67, 75, 87, 98
(a) 58, 33, 67, 98 (b) 65, 02, 59, 67
L 57, 69, 78, 86, 97
(c) 65, 04, 89, 75 (d) 65, 20, 89, 68
M 58, 65, 77, 85, 99
2. NAIL N 59, 68, 76, 89, 95
(a) 95, 01, 00, 77 (b) 89, 21, 33, 97 R 56, 66, 79, 88, 96
(c) 95, 32, 14, 86 (d) 95, 21, 34, 68 1. (c) 2. (c) 3. (a) 4. (b) 5. (d) 6. (c)
y
o
u
Matrix Based Reasoning A-109

rs
m
a
h
b
o
o
b
.w
DIRECTIONS : (Qs. 1-24) For following questions, two matrices Matrix II

o
are given below. In each matrix there are 25 cells and these cells 5 6 7 8 9

rd
contain two classes of alphabets. The columns and rows of matrix.

p
5 W R T E Y

re
I are numbered from 0 to 4 and that of matrix II from 5 to 9. A letter
6 R W Y T E

s
from these matrices can be represented first by its now number

s
.c
and next by its column number. Identify one set of number pairs 7 E Y W R T

o
out of (a), (b), (c) and (d) which represents the number.

m
8 Y T E W R
Matrix I 9 T E R Y W
6. DAILY
0 1 2 3 4
(a) 10, 31, 21, 42, 85 (b) 22, 43, 00, 32, 67
0 A P O R E (c) 41, 14, 13, 24, 98 (d) 34, 44, 21, 01, 76
1 R E P O A 7. RATE
2 O A E P R (a) 89, 20, 96, 69 (b) 65, 03, 86, 96
(c) 78, 14, 95, 58 (d) 56, 31, 58, 87
3 E O R A P
8. CIAT
4 P R A E O (a) 11, 21, 31, 87 (b) 32, 44, 20, 68
(c) 23, 34, 43, 95 (d) 04, 14, 02, 57
Matrix II 9. WEAR
5 6 7 8 9 (a) 77, 87, 32, 89 (b) 99, 78, 14, 56
(c) 88, 69, 21, 65 (d) 66, 87, 31, 97
5 F N M S T
10. DWART
6 T M F N S (a) 10, 77, 32, 89, 68 (b) 22, 88, 02, 89, 57
7 M S T F N (c) 34, 99, 43, 66, 79 (d) 41, 66, 20, 78, 88
8 N F S T M For (Qs. 11-15) matrices are given below.
Matrix I
9 S T N M F
0 1 2 3 4
1. MORT 0 E H J O A
(a) 66, 21, 32, 77 (b) 75, 44, 02, 65 1 H O E A J
(c) 57, 13, 03, 88 (d) 68, 31, 41, 96
2 O J A E H
2. FASE
(a) 67, 33, 95, 43 (b) 56, 21, 59, 11 3 J A O H E
(c) 86, 42, 77, 22 (d) 99, 00, 87, 31 4 A E H J O
3. SERAF Matrix II
(a) 69, 04, 10, 21, 88 (b) 87, 04, 24, 42, 78 5 6 7 8 9
(c) 76, 43, 24, 32, 99 (d) 58, 30, 32, 44, 67 5 L F S K T
4. PANT
6 T K L S F
(a) 12, 33, 69, 77 (b) 23, 43, 79, 88
(c) 01, 21, 56, 66 (d) 01, 21, 85, 77 7 S L F T K
5. TORN 8 K S T F L
(a) 96, 13, 24, 69 (b) 77, 31, 42, 85 9 F T K L S
(c) 65, 20, 32, 79 (d) 88, 02, 11, 56 11. JOLE
For (Qs. 6 - 10) matrices are given below. (a) 21, 44, 89, 34 (b) 30, 11, 68, 41
Matrix I (c) 14, 33, 98, 23 (d) 43, 20, 76, 13
12. FATH
0 1 2 3 4 (a) 69, 13, 79, 24 (b) 95, 31, 99, 12
0 I L A D C (c) 88, 21, 87, 42 (d) 77, 22, 96, 33
1 D C I L A 13. KHOLE
(a) 66, 24, 20, 76, 24 (b) 85, 10, 32, 67, 41
2 A I D C L (c) 97, 33, 12, 89, 34 (d) 79, 43, 03, 98, 12
3 L A C I D 14. FEAST
4 C D L A I (a) 77, 00, 14, 86, 96 (b) 56, 12, 22, 69 65
(c) 69, 23, 31, 75, 87 (d) 88, 33, 40, 99, 59
y
o
u
A-110 Matrix Based Reasoning

rs
m
15. LOHA Matrix II

a
(a) 98, 20, 42, 04 (b) 76, 32, 01, 41

h
(c) 67, 12, 24, 31 (d) 55, 03, 11, 22

b
o
o
For (Qs. 16-20) matrices are given below. 5 6 7 8 9

b
Matrix I

.w
0 1 2 3 4 5 H K L I N

o
rd
0 A S U E O

p
6 I N H K L

re
1 U O E S A

s
s
2 S A O U E 7 K L I N H

.c
o
3 E U A O S

m
4 O E S A U 8 L I N H K
Matrix II
9 N H K L I
5 6 7 8 9
5 C R T P N
6 N P C T R (a) 24, 21, 99, 57, 01 (b) 12, 79, 99, 57, 01
7 P C R N T (c) 33, 57, 99, 57, 01 (d) 41, 79, 99, 57, 11
8 T N P R C 22. Matrix I Matrix II
9 R T N C P
16. TUNE 0 1 2 3 4 5 6 7 8 9
(a) 68, 31, 79, 41 (b) 85, 10, 97, 22 0 R H E L I 5 B S N A D
(c) 79, 23, 86, 30 (d) 96, 42, 65, 12 1 I E L R H 6 D N B S A
17. CROPS 2 H L R I E 7 A B D N S
(a) 89, 95, 11, 75, 20 (b) 67, 77, 32, 87, 34 3 E R I H L 8 S D A B N
(c) 76, 88, 40, 67, 13 (d) 98, 56, 04, 58, 43 4 L I H E R 9 N A S D B
18. SENT
(a) 20, 31, 86, 79 (b) 34, 24, 59, 89 (a) 58, 02, 13, 01 (b) 85, 42, 31, 14
(c) 13, 03, 66, 85 (d) 42, 12, 78, 96 (c) 68, 20, 13, 32 (d) 85, 02, 44, 30
19. PART
23. A word is represented by only one set of numbers as given
(a) 75, 21, 89, 96 (b) 66, 14, 88, 57
(c) 87, 00, 95, 69 (d) 99, 43, 78, 85 in any one of the alternatives. The sets of numbers given in
20. SPENT the alternatives are represented by two classes of alphabets
(a) 13, 66, 25, 86, 96 (b) 20, 75, 30, 98, 57 as in two matrices given below. The columns and rows of
(c) 34, 87, 41, 59, 85 (d) 42, 58, 03, 65, 78 Matrix I are numbered from 0 to 4 and that of Matrix II are
21. A word is represented by only one set of numbers as given numbered from 5 to 9. A letter from these matrices can be
in any one of the alternatives. The sets of numbers given in represented first by its row and next by its column, e.g., 'F'
the alternatives are represented by two classes of alphabets can be represented by 30, 22, etc. and 'N' can be represented
as in two matrices given below. The columns and rows of by 97, 89, etc. Similarly, you have to identify the set for the
Matrix I are numbered from 0 to 4 and that of Matrix II are given word.
numbered from 5 to 9. A letter from these matrices can be
"DAKU"
represented first by its row and next by its column, e.g., 'G'
can be represented by 04, 40 etc. and 'K' can be represented Matrix-I Matrix-II
by 56, 75 etc. Similarly, you have to identify the set for the
word 'CHILD'.
Matrix I 0 1 2 3 4 5 6 7 8 9
4 A F K P U 9 D I N S X
0 1 2 3 4
3 F K A U P 8 X S I D N
0 C D E F G 2 P U F K A 7 N X S I D
1 F G C D E 1 K P U A F 6 S D X N I
0 U A P F K 5 I N D X S
2 D E F G C

3 E F G C D (a) 95, 40, 04, 42 (b) 24, 95, 20, 27


(c) 88, 24, 10, 34 (d) 57, 13, 23, 21
4 G C D E F
y
o
u
Matrix Based Reasoning A-111

rs
24. A word is represented by one set of numbers as given in

m
Matrix II
any one of the alternatives. The sets of numbers given in

a
h
the alternatives are represented by two classes of alphabets 5 6 7 8 9

b
o
as in two matrices given below. The columns and rows of 9 D I N S X

o
Matrix I are numbered from 0 to 4 and that of Matrix II are

b
8 X S I D N

.w
numbered from 5 to 9. A letter from these matrices can be
represented first by its row and next by its column e.g., 'A'

o
7 N X S I D

rd
can be represented by 40, 01, 13, 32, and 'N' can be
6 S D X N I

p
represented by 56, 68, 89 etc. Similarly, you have to identify

re
the set for the word given below : 5 I N D X S

s
s
SIX-KIDS

.c
Matrix I (a) 86, 87, 99 — 40, 41, 86, 64

o
(b) 98, 96, 85 — 42, 78, 88, 77

m
0 1 2 3 4 (c) 77, 69, 76 — 22, 95, 28, 31
4 A F K P U (d) 65, 55, 67 — 05, 25, 91, 40
3 F K A U P
2 P U F K A
1 K P U A F
0 U A P F K

ANSWER KEY
1 (c) 4 (d) 7 (c) 10 (b) 13 (b) 16 (c) 19 (b) 22 (c)
2 (a) 5 (c) 8 (b) 11 (a) 14 (c) 17 (a) 20 (c) 23 (d)
3 (b) 6 (a) 9 (d) 12 (d) 15 (a) 18 (d) 21 (b) 24 (b)

Solutions for 1 to 5 L 55, 67, 76, 89, 98


A 00, 14, 21, 33, 42 F 56, 69, 77, 88, 95
P 01, 12, 23, 34, 40 S 57, 68, 75, 86, 99
O 02, 13, 20, 31, 44 K 58, 66, 79, 85, 97
R 03, 10, 24, 32, 41 T 59, 65, 78, 87, 96
E 04, 11, 22, 30, 43 11. (a) 12. (d) 13. (b) 14. (c) 15. (a)
F 55, 67, 78, 86, 99 Solutions for 16 to 20
N 56, 68, 79, 85, 97 A 00, 14, 21, 32, 43
M 57, 66, 75, 89, 98 S 01, 13, 20, 34, 42
S 58, 69, 76, 87, 95 U 02, 10, 23, 31, 44
T 59, 65, 77, 88, 96 E 03, 12, 24, 30, 41
1. (c) 2. (a) 3. (b) 4. (d) 5. (c) O 04, 11, 22, 33, 40
Solutions for 6 to 10 C 55, 67, 76, 89, 98
I 00, 12, 21, 33, 44 R 56, 69, 77, 88, 95
L 01, 13, 24, 30, 42 T 57, 68, 79, 85, 96
A 02, 14, 20, 31, 43 P 58, 66, 75, 87, 99
D 03, 10, 22, 34, 41 N 59, 65, 78, 86, 97
C 04, 11, 23, 32, 40 16. (c) 17. (a) 18. (d) 19. (b) 20. (c)
W 55, 66, 77, 88, 99 21. (b) C = 00, 12, 24, 33, 41
R 56, 65, 78, 89, 97 H = 55, 67, 79, 88, 96
T 57, 68, 79, 86, 95 I = 58, 65, 77, 86, 99
E 58, 69, 75, 87, 96 L = 57, 69, 76, 85, 98
Y 59, 67, 76, 85, 98 D = 01, 13, 20, 34, 42
6. (a) 7. (c) 8. (b) 9. (d) 10. (b) CHILD = 12, 79, 99, 57, 01
Solutions for 11 to 15 22. (c) S 56, 68, 79, 85, 97
E 00, 12, 23, 34, 41 H 01, 14, 20, 33, 42
H 01, 10, 24, 33, 42 R 00, 13, 22, 31, 44
J 02, 14, 21, 30, 43 I 04, 10, 23, 32, 41
O 03, 11, 20, 32, 44 24. (b)
A 04, 13, 22, 31, 40
y
o
u
A-112 Statement & Conclusions

rs
m
a
CHAPTER
STATEMENT &

h
b
16

o
o
b
.w
CONCLUSIONS

o
rd
p
re
s
s
.c
In this type of questions, a statement is given followed by two II. The price of the cooking gas will

o
m
conclusions. We have to find out which of these conclusions increase at least by 33% from the next
definitely follows from the given statement. month.
What is a ‘conclusion’? Sol. (d) I does not follow because a govt’s policy is not
‘Conclusion’ means a fact that can be truly inferred from the determined merely by people’s needs.
contents of a given sentence. II does not follow. Let the present price be x
DIRECTIONS (for Examples 1 to 3) : In each of the following x
questions, a statement is given followed by two conclusions I Price if subsidy is removed = 1.49x
0.67
and II. Give answer :
Hence increase in price will be around 49%
(a) if only conclusion I follows;
DIRECTIONS (for Examples 4 to 5) : In each of the following
(b) if only conclusion II follows;
questions, a statement is given followed by two conclusions I and
(c) if either I or II follows;
II. Give answer :
(d) if neither I nor II follows;
(a) if only conclusion I follows;
EXAMPLE 1. (b) if only conclusion II follows;
Statement : The oceans are a storehouse of practically (c) if either I or II follows;
every mineral including uranium. But like most other (d) if both I and II follow.
minerals, it is found in extremely low concentration – about EXAMPLE 4.
three gms per 1000 tonnes of water.
Conclusions : I. The oceans are a cheap source of Statement : Interest rate will be fixed on the basis of
uranium. our bank’s rate prevailing on the date of
II. The oceans harbour radiation hazards. deposit and refixed every quarter
Sol. (d) I can not be concluded as most of the minerals are thereafter.
available in similar concentration levels in oceans. II is Conclusions : I. It is left to the depositors to guard their
out of context of the statement. interest.
EXAMPLE 2. II. The bank’s interest rates are subject
Statement : Today, out of the world population of several to change on a day-to-day basis
thousand million, the majority of men have to live under depending on market position.
government which refuse them personal liberty and the right Sol. (b) I does not follow because the statement is silent about
to dissent. the depositors. II follows from the
Conclusions : I. People are indifferent to personal liberty
phrase “bank’s rate prevailing on the
and the right to dissent.
II. People desire personal liberty and the date of deposit” which means the rates
right to dissent. are subject to day-to-day changes.
Sol. (b) It is mentioned in the statement that most people are
EXAMPLE 5.
forced to live under Governments which refuse them
personal liberty and right to dissent. This means that Statement : The government of country X has recently
they are not indifferent to these rights but have a desire announced several concessions and
for them. So, only II follows. offered attractive package tours for
foreign visitors.
EXAMPLE 3. Conclusions : I. Now, more number of foreign tourists
Statement : It has been decided by the Government to will visit the country.
withdraw 33% of the subsidy on cooking gas from the II. The government of country X seems
beginning of next month—A spokesman of the Government. to be serious in attracting tourists.
Conclusions : I. People no more desire or need such Sol.
subsidy from government as they can
(d) Clearly, the government has taken the step to attract
afford increased price of the cooking
more tourists. So, both I and II follow.
gas.
y
o
u
Statement & Conclusions A-113

rs
m
a
h
b
o
o
b
.w
Directions (Q. 1-19): In each questions below is given a statement 6. Statement: Mrs X is nominated for one of the two posts of

o
followed by two conclusions numbered I and II. You have to which one post is reserved by the Managing Committee for

rd
assume everything in the statement to be true, then consider the the female of other religious minority community and the

p
re
two conclusions together and decide which of them logically other for the female of scheduled Castes or Scheduled

s
follows beyond a reasonable doubt from the information given in Tribes.

s
.c
the statement. Give answer Conclusions:

o
(a) if only conclusion I follows. I. Mrs X is the member of religious minority community.

m
(b) if only conclusion II follows. II. Mrs X is the member of Scheduled Castes or Scheduled
(c) if either I or II follows. Tribes.
(d) if neither I nor Il follows 7. Statement: We do not need today in India extraordinary
1. Statement: Many people and media alleged that Mr. X, the specialists but those trained ordinary doctors who are
opposition leader, met the Chief Minister yesterday to seek dedicated to their profession.
certain favours, an allegation which was strongly rejected Conclusions:
by Mr X.
I. We should promote medical profession with dedicated
Conclusions:
ordinary doctors rather than promoting high
I. Mr X did meet the Chief Minister yesterday to seek
certain favours. specialised medical education.
II. Mr X did not meet the Chief Minister to seek certain II. Extraordinary specialists are not dedicated to their
favours. profession.
2. Statement: ‘Our approach of fund management is based on 8. Statement: People in metropolitan city ‘X’ have welcomed
science as much as on common sense and discipline because the recent Supreme Court order banning the registration of
our goal is consistent performance in the long term. – private vehicles that do not conform to Euro II emission
Advertisement of a mutual fund company. norms with immediate effect for metropolitan city ‘Y’ only.
Conclusions: Conclusions:
I. Only the approach of science of investment can lead I. City ‘X’ has quite lower level of vehicular pollution
to high gains in short-term investment. than city ‘Y’.
II. It is not necessary to go for long-term investment when II. Public vehicles do not contribute to the vehicular
low-return short-term investment is available. pollution.
3. Statement: ‘We follow some of the best and effective 9. Statement: The government of State ‘A’ has sought a
teaching learning practices used by leading institutes all
waiver of outstanding Central loans of ` 4,000 crores and a
over the world.’ — A statement of a Professor of MN
moratorium on repayment pending the waiver.
Institute.
Conclusions:
Conclusions:
I. The MN Institute is one of the leading institutes of the I. Unlike other states, State ‘A’ has no desire to make
world. repayment of its loans.
II. State ‘A”s financial condition does not appear to be
II. Whatever is being followed by world’s leading
satisfactory.
institutes will definitely be good and useful.
10. Statements:
4. Statement: The minister questioned the utility of the space
research programme and suggested its replacement by other I. Man can find ultimate reality by dedication to God
areas of felt national needs. through meditation.
Conclusions: II. Meditation is the process to enrich the peace of mind.
I. Exploring the space does not contribute to critical Conclusions:
national needs. I. Meditation is the process to realize the God.
II. Research should be oriented to national needs. II. Meditation does not help to find out the Mukti of Soul.
5. Statement: The laws and statutes framed by the Government III. Perfect dedication to God is essential to realize the
for the purpose of providing equal treatment to every existence of ultimate reality.
citizens, on implementation perpetuate corrupt working (a) Only conclusion I follows
system. (b) Only conclusion II & III follow.
Conclusions: (c) Only conclusion I and III follow.
I. The laws and statutes should be framed but they (d) All conclusions follow.
should not be implemented to avoid corrupt working 11. Statement : The eligibility for admission to the course is
system. minimum second class master’s degree. However, the
II. There should be obvious method to investigate corrupt candidates who have appeared for the final year examination
working system. of master’s degree can also apply.
y
o
u
A-114 Statement & Conclusions

rs
m
Conclusions: 19. Statements :

a
I. All candidates who have yet to get their master’s (1) Due to contamination of water, large number of people

h
degree will be there in the list of selected candidates.

b
were admitted to hospital.

o
II. All candidates having obtained second class master’s

o
(2) The symptoms were of Typhoid.

b
degree will be there in the list of selected candidates.

.w
12. Statement: The government-run company had asked its (a) Only conclusion I is true

o
employees to declare their income and assets but it has (b) Only conclusion II is true

rd
been strongly resisted by employees union and no employee (c) Both conclusions I and II are true

p
re
is going to declare his income. (d) Both conclusions I and II are false

s
Conclusions: Conclusions : I. Contamination of water may lead to

s
.c
I. The employees of this company do not seem to have Typhoid.

o
any additional undisclosed income besides their salary.

m
II. Typhoid is a contagious disease.
II. The employees’ union wants all senior officers to
declare their income first. 20. Statements :
13. Statement: Good health is a luxury in country ‘P’ where rate (1) 60% of the government employees went on strike.
of death is very high compared to other nations of that (2) Mr. Gopal is a government employee.
region. Conclusions : I. Mr. Gopal went on strike.
Conclusions: II. Mr. Gopal did not participate in the
I. People in country ‘P’ cannot afford to have many strike.
luxuries of life. (a) Only conclusion I follows
II. Good health is a gift of nature. (b) Only conclusion II follows
14. Statement: Company “Y” will improve the manufacturing (c) Both conclusions I and II follow
facilities for the production of shaving kits as a result of (d) Either conclusion I and II follows
which capacity would increase and cost would be reduced.
21. Statements: Video libraries are flourishing very much these
A spokesperson of the Company “Y”
days.
Conclusions :
Conclusions:
I. The products of Company “Y” will compete the market
I. People in general have got a less video craze.
norms in the quality and cost factor.
II. It is much cheaper to see as many movies as one likes
II. There will be demand of shaving kits of Company “Y”.
on videos rather than going to the cinema hall.
15. Statement: Population increase coupled with depleting
resources is going to be the scenario of many developing 22. If all players play to their full potential, we will win the
countries in days to come. match. We have won the match.
Conclusions : Conclusions:
I. The population of developing countries with not I. All players played to their full potential.
continue to increase in future. II. Some players did not play to their full potential.
II. It will be very difficult for the governments of 23. Statements: Leaders, who raise much hue and cry about
developing countries to provide its people decent the use of Hindi, generally send their children to English
quality of life. medium schools.
16. Statement: An advertisement / Interest rate will be fixed Conclusions:
on the basis of our bank’s rate prevailing on the date of I. India lacks good Hindi medium schools.
deposit and refixed every quarter thereafter. II. There is a world of difference between preaching and
Conclusions: practising.
I. It is left to the depositors to guard their interest. 24. Statements: The best evidence of India's glorious past is
II. The bank’s interest rates are subject to change on a the growing popularity of Ayurvedic medicines in the West.
day-to-day basis depending on market position. Conclusions:
17. Statement: In order to enforce discipline on transport I. Ayurvedic medicines are not popular in India.
operators, the state government has decided to impose a II. Allopathic medicines are more popular in India.
fine of ` 5,000 for the first excess tonne loaded in transport 25. Statements: Company X has marketed the product. Go
vehicle and ` 1,000 for each subsequent tonne. ahead; purchase it if price and quality are your
Conclusions :
considerations.
I. People will follow some discipline when severe fine is
Conclusions:
imposed.
I. The product must be good in appearance.
II. The state government has failed to understand the
problem of transport operators. II. The price of the product must be reasonable.
18. Statement: Book your flat before 15th June and avail 26. Statements: Only good singers are invited in the conference.
interest free loan from the builders. An advertisement No one without sweet voice is a good singer.
Conclusions: Conclusions:
I. No flat will be booked afterwards. I. All invited singers in the conference have sweet voice.
II. After 15th June, no loan will be provided. II. Those singers who do not have sweet voice are aslo
invited in the conference.
y
o
u
Statement & Conclusions A-115

rs
m
27. Statements: All those political prisoners were released on Conclusions:

a
bail who had gone to jail for reasons other than political I. The residents in these wards should economise on

h
dharnas. Bail was not granted to persons involved in water on Friday.

b
o
murders. II. The residents in these wards should not store some

o
b
Conclusions: water on the previous day.

.w
I. No political - prisoner had committed murder. 30. Statements: Company X has a record of manufacturing

o
II. Some politicians were not arrested. cameras of quality and the latest design so that you do not

rd
28. Statements: Modern man influences his destiny by the spoil even a single shot irrespective of the weather

p
re
choice he makes unlike in the past. conditions.

s
Conclusions: Conclusions:

s
.c
I. Earlier there were fewer options available to man. I. No other company except X is reputed in the camera

o
II. There was no desire in the past to influence the destiny. industry.

m
29. Statements: Water supply in wards A and B of the city will II. Anyone can take an acceptable shot with camera X.
be affected by about 50% on Friday because repairing work
of the main lines is to be carried out.

ANSWER KEY
1 (c) 4 (d) 7 (a) 10 (c) 13 (a) 16 (b) 19 (d) 22 (a) 25 (b) 28 (a)
2 (d) 5 (d) 8 (d) 11 (d) 14 (a) 17 (a) 20 (d) 23 (c) 26 (a) 29 (a)
3 (d) 6 (c) 9 (b) 12 (d) 15 (b) 18 (d) 21 (b) 24 (d) 27 (a) 30 (b)
y
o
u
A-116 Statement & Conclusions

rs
m
a
h
b
o
o
b
.w
o
3. (d) II may be an assumption which the professor is booking will continue even afterwards. And even loans

rd
assuming before passing his statement. Hence, II does will be given, though on interest.

p
not follow. I may or may not be possible. Hence, I does 21. (b) II provide suitable explanations to the given statement,

re
not follow.

s
so II follows.

s
8. (d) I does not follow. It is probable that city ‘X’ welcomed

.c
22. (a) The statement asserts that match can be won only if

o
the order with the expectation that the order would all the players play to their full potential. So, only I

m
soon be extended in some manner to city ‘X’ as well. II follows while II does not.
also does not follow. That public vehicles have been 23. (c) Clearly, either I or II could be the reason for the
left out of the order is probably due to other reasons. situation expressed in the statement.
For example, causing inconvenience to the commuters. 24. (d) The popularity of Ayurvedic or allopathic medicines
9. (b) I does not follow because it makes a comparison with in India is not being talked about in the statement. So,
“other states”. Now, in the statement, other states are neither I nor II follows.
nowhere in the picture. II follows because seeking such
25. (b) It is mentioned in the statement that one who considers
loan waivers indicates poor financial condition.
price and quality before buying a product should buy
12. (d) I does not follow. On the contrary, it must be their the product of company X. Statement II tells about
“additional undisclosed income” which is causing
quality. Hence only II follows.
hesitation on their part to declare their income. II also
26. (a) The statement asserts that a good singer always has a
does not follow. The question of seniority simply does
sweet voice and only good singers are invited in the
not arise.
conference. This implies that all those invited in the
13. (a) I follows. A country where even good health is
conference have sweet voice and those who do not
considered to be a luxury certainly can’t afford luxuries.
II does not follow. Man may strive towards good have sweet voice are not invited. So, only I follows.
health. 27. (a) According to the statement, the political prisoners can
be divided into two groups - those who were released
14. (a) Improvement in the manufacturing facilities will
and those who were put in jail for political dharnas.
automatically enhance the quality of its product and
reduce the cost. These two things are important to However, no person involved in murder was released.
compete in the market. Hence, I follows. II may be an This means that no political prisoner had committed
assumption but is it not a conclusion. murder. So, I follows. Clearly, II is not directly related
15. (b) With the limited resources and overpopulation it is to the statement and does not follow.
very hard to provide decent quality of life. Hence, II 28. (a) Clearly, I directly follows from the statement while II
follows. cannot be deduced from it.
16. (b) I does not follow because the statement is silent about View Answer Workspace Report Discuss in Forum
the depositors. II follows from the phrase “bank’s rate 29. (a) Clearly, the information has been given beforehand so
prevailing on the date of deposit”. Which means the that the residents can collect water on the previous
rates are subject to day-to-day changes. day and use less water on Friday. But conclusion II is
17. (a) I follows from the policy laid down and the reason just opposite to the given statement. So only I follows.
given thereof. II does not follow because the decision 30. (b) Clearly, the statement talks of Company X only and no
is expected to have been taken after a proper other company. So, I does not follow. Also, it is
assessment of the problem. mentioned that one can take a good shot even in bad
18. (d) Neither follows. From the statement, it is clear that weather conditions with a camera of Company X. So,
15th June is a deadline only for interest free loans. But II follows.
y
o
u
Cube & Dice A-117

rs
m
a
CHAPTER

h
b
CUBE & DICE 17

o
o
b
.w
o
rd
p
re
s
s
.c
What is a cube? Layer II or middle layer : The central cube has no face

o
m
A cube is three dimensional figure whose length, breadth and coloured, the four cubes at the corner have two faces
height are equal and any two adjacent faces are inclined to each coloured and the remaining 4 cubes have only face coloured.
other at 90°. It has 6 faces, 8 corners and 12 edges.
G H
Middle layer Top unpainted
D C
Unpainted cube
E
F
Bottom unpainted
A B
• Corners of the cube are A, B, C, D, E, F, G and H.
• Edges of the cube are AB, BE, EF, AF, AD, CD, BC, EH, CH, Layer III or bottom layer : The central cube has only one
GH, DG and FG. face coloured, four cubes at the corner have three faces
• Faces of the cube are ABCD, EFGH, CDGH, BCHE, ABEF coloured and the remaining 4 cubes have two faces
and ADFG. coloured.
When a cube is painted on all of its faces with any colour
and further divided into various smaller cubes of equal size, Bottom layer
we get following results :
(i) Smaller cubes with no face painted will present inside faces
of the undivided cube.
(ii) Smaller cubes with one face painted will present on the
faces of the undivided cube.
(iii) Smaller cubes with two faces painted will present on the
Bottom painted
edges of undivided cube.
(iv) Smaller cubes with three faces painted will present on the
corners of the undivided cube.
Also, if n = no. of divisions on the faces of cube
Length of the edge of undivided cube
= .
Length of the edge of one smaller cube
Cube with Cube with
two sides three sides Then,
painted painted (i) Number of smaller cubes with no face painted
= (n – 2)3
Cube with (ii) Number of smaller cubes with one face painted
one side
= (n – 2)2 × 6
painted
(iii) Number of smaller cubes with two faces painted
The above figure may be analysed by dividing it into three = (n – 2) × 12
horizontal layers : (iv) Number of smaller cubes with three faces painted = 8
Layer I or top layer : The central cube has only one face EXAMPLE 1. A cube is painted blue on all faces is cut into
coloured, four cubes at the corner have three faces coloured
and the remaining 4 cubes have two faces coloured. 125 cubes of equal size. Now, answer the following questions :
Top layer (i) How many cubes are not painted on any face?
(a) 8 (b) 16
(c) 18 (d) 27
(ii) How many cubes are painted on one face only?
(a) 8 (b) 16
Bottom unpainted (c) 36 (d) 54
y
o
u
A-118 Cube & Dice

rs
m
Sol. Since there are 125 smaller cubes of equal size, therefore, (i) Number of cubes with two surfaces painted with black

a
n = no. of divisions on the face of undivided cube = 5. and blue colour = 2(cubes along with edge AB) +

h
2(cubes along with edge CD) + 2(cubes along with

b
(i) (d) Number of cubes with no face painted = (n – 2)3

o
= (5 – 2)3 = 27 edge GH) + 2(cubes along with edge EF) = 8

o
b
(ii) (d) Number of cubes with one face painted = (n – 2)2 × 6 (ii) Number of cubes with two surfaces painted with blue

.w
= (5 – 2)2 × 6 and red colour = 2(cubes along with edge AD) + 2(cubes

o
along with edge BC) + 2(cubes along with edge FG) +

rd
= 54
2(cubes along with edge EH) = 8

p
re
EXAMPLE 2. A cube of side 4 cm is painted black on the pair
(iii) Number of cubes with two surfaces painted with black

s
of one opposite surfaces, blue on the pair of another opposite

s
and red colour = 2(cubes along with edge DE) + 2(cubes

.c
surfaces and red on remaining pair of opposite surfaces. The along with edge CF) + 2(cubes along with edge BG) +

o
m
cube is now divided into smaller cubes of equal side of 1 cm each. 2(cubes along with edge AH) = 8
Then, III. Number of smaller cubes with one surfaces painted
I. Find the number of smaller cubes with three surfaces = (4 – 2)2 × 6 = 24
painted. (i) Number of cubes with one surface painted with black
II. Find the number of smaller cubes with two surfaces painted. colour = 4(cubes on face ABGH) + 4(cubes on face
And out of this CDEF) = 8
(i) Find the number of cubes with two surfaces painted (ii) Number of cubes with one surface painted with blue
with black and blue colour. colour = 4(cubes on edge face ABCD) + 4(cubes on
(ii) Find the number of cubes with two surfaces painted face EFGH) = 8
with blue and red colour. (iii) Number of cubes with one surface painted with red
(iii) Find the number of cubes with two surfaces painted colour = 4(cubes on edge face ADEH) + 4(cubes on
with black and red colour. face BCFG) = 8
III. Find the number of smaller cubes with one surface painted. EXAMPLE 3. A cube is painted red on two adjacent faces and
And out of this on one opposite face, yellow on two opposite faces and green on
(i) Find the number of cubes with one surface painted the remaining face. It is then cut into 64 equal cubes.
with black colour. How many cubes have only one red coloured face?
(ii) Find the number of cubes with one surface painted (a) 4 (b) 8
with blue colour. (c) 12 (d) 16
(iii) Find the number of cubes with one surface painted Sol. (c)
with red colour.
Sol. (c) E F
Blue (back)
Black (top) C
E D
F G
H
C Red (side)
D A B
G
H Let faces ABCD, ABGH and CDEF are painted with red
colour.
A B Faces BCFG and ADEH are painted with yellow and EFGH
Blue is painted with green colour.
(front) Black (bottom) Clearly the cubes which have only one red coloured face
4 and all other faces uncoloured are the four central cubes at
Here, n 4 each of the three faces ABCD, ABGH and CDEF. Thus,
1
there are 4 × 3 = 12 such cubes.
I. Number of smaller cubes with three surfaces painted = 8
DIRECTIONS (for Examples 4 to 7) : Read the information given
(All three surfaces painted with different colours black,
below to answer the questions that follow.
blue and red)
I. A cube has six sides, each of which has a different colour :
II. Number of smaller cubes with two surfaces painted
black, blue, brown, green, red and white.
= (4 – 2) × 12 = 24
II. The red side is opposite the black.
Now, let faces ABCD and EFGH are painted with Blue.
III. The green side is between the red and the black.
Faces BCFG and ADEH are painted with Red. IV. The blue side is adjacent to the white.
Faces ABGH and CDEF are painted with Black. V. The brown side is adjacent to the blue.
Therefore, VI. The red side is the bottom face.
y
o
u
Cube & Dice A-119

rs
Number 1 is opposite to 6.

m
EXAMPLE 4.The four colours adjacent to green are :

a
Number 2 is opposite to 4.

h
(a) black, blue, brown, red Number 3 is opposite to 5.

b
o
(b) black, blue, brown, white Form 3:

o
(c) black, blue, red, white 1

b
.w
(d) black, brown, red, white 2

o
Sol. (d) When the cube is unfolded, it will look like as

rd
3 4

p
Black

re
5

s
s
6

.c
White Blue Brown

o
Number 1 is opposite to 3.

m
Red Number 2 is opposite to 5.
Number 4 is opposite to 6.
Green Form 4:
1
The four colours adjacent to green are black, brown, 2 3
red and white.
4
EXAMPLE 5. Which of the following can be deduced from the
5 6
statements I, II and VI ?
(a) Black is on the top Number 1 is opposite to 4.
(b) Blue is on the top Number 2 is opposite to 6.
(c) Brown is on the top Number 3 is opposite to 5.
(d) Brown is opposite to black Form 5:
1
Sol. (a) The red side is opposite to the black. Therefore, if red
is at the bottom, black will be at the top. 2
EXAMPLE 6. Which of the following statements given above 3 4
adds no information ? 5
(a) II (b) III
(c) V (d) VI 6
Sol. (d) VI does not add to the information provided by I – V. In this case:
EXAMPLE 7. If the red side is exchanged for the green side 1 lies opposite 3;
and the blue is swapped for black, then which of the following is 2 lies opposite 5;
4 lies opposite 6.
false ?
(a) Red is opposite to black. Form 6:
(b) White is adjacent to brown. 1
(c) Green is opposite to blue.
2 3 4
(d) White is adjacent to blue.
Sol. (b) Adjacent to white, we have brown. A dice is a cube 5
with all of its faces numbered from 1 to 6. When a dice
is unfolded, it will look like in any of the following In this case:
forms :
Form 1: will be the one of the faces of the cube and it
1
lies opposite 3;
2 3 4 2 lies opposite 4;
5 1 lies opposite 5.
Form 7:
6
1
Number 1 is opposite to 5.
Number 2 is opposite to 4. 2 3 4
Number 3 is opposite to 6. 5
Form 2:
1 2
In this case:
3
4 will be the one the faces of the cube and it lies
5 6 opposite 3;
y
o
u
A-120 Cube & Dice

rs
m
2 lies opposite 4; 2 lies opposite 4;

a
1 lies opposite 5. 1 lies opposite 5.

h
b
Form 8: EXAMPLE 8. Two positions of a dice are shown, when 4 is at

o
1

o
the bottom, what number will be on the top?

b
.w
2
1 1

o
3 2 6

rd
3 5

p
4

re
s
In this case: (i) (ii)

s
.c
(a) 1 (b) 2 (c) 5 (d) 6

o
Sol. (a) From the two figures it is clear that the numbers 2, 3, 5

m
will be the one the faces of the cube and it lies
and 6 cannot appear opposite 1. So, 4 appears opposite
opposite 3; 1. Therefore, when 4 is at the bottom, 1 will be on the
top.

DIRECTIONS (Qs. 1-4) : Read the following informations and 8. How many cubes have three faces coloured ?
answer the quesstions based on it. (a) 24 (b) 16
I. The length, breadth and height of a rectangular piece (c) 8 (d) 4
of wood are 4 cm , 3 cm and 5 cm respectively. DIRECTIONS (Qs. 9 - 11) : Three adjacent faces of a cube are
II. Opposite sides of 5 cm × 4 cm piece are coloured in coloured blue. The cube is then cut (once horizontally and once
red. vertically) to form four cuboids of equal size, each of these cuboids
III. Opposite sides of 4 cm × 3 cm are coloured in blue. is coloured pink on all the uncoloured faces and is then cut (as
IV. Rest sides of 5 cm × 3 cm are coloured on green in both before) into four cuboids of equal size.
sides. 9. How many cuboids have two faces coloured pink ?
V. Now the piece of is cut in such a way that cubes of 1 (a) 1 (b) 3 (c) 4 (d) 6
cm × 1 cm × 1 cm will be made. 10. How many cuboids have three faces coloured pink?
1. How many cubes shall have all the three colurs ? (a) 9 (b) 7 (c) 5 (d) 3
(a) 8 (b) 10 11. How many cuboids have three faces coloured blue?
(c) 12 (d) 14 (a) 4 (b) 2 (c) 1 (d) 0
2. How many cubes shall not have any colour ? DIRECTIONS (Qs. 12 - 16) : Following questions are based on
(a) No any (b) 2 the figures given below which represent different positions of
(c) 4 (d) 6 the same dice.
3. How many cubes shall have only two colours red and green
on their two sides ? 2 1 5
(a) 8 (b) 12 3 4 2
(c) 16 (d) 20 cv 1 6 6
4. How many cubes shall have only one colour ?
(a) 12 (b) 16 (i) (ii) (iii)
(c) 22 (d) 28 12. Which number lies at the bottom face of the dice (i) ?
DIRECTIONS (Qs. 5 -8) : A cube is coloured red on all faces. It (a) 4 (b) 2
is cut into 64 smaller cubes of equal size. Now, answer the (c) 1 (d) 3
following questions based on this statement : 13. Which number lies at the bottom face of the dice (iii) ?
5. How many cubes have no face coloured ? (a) 1 (b) 2
(a) 24 (b) 16 (c) 6 (d) 4
(c) 8 (d) 0 14. Which number lies opposite 6 ?
6. How many cubes are there which have only one face (a) 2 (b) 5
coloured ? (c) 3 (d) 1
(a) 4 (b) 8 15. Which of the following combinations shows the numbers
(c) 16 (d) 24 at the adjacent surfaces of the number 4 ?
7. How many cubes have two red opposite faces ? (a) 3, 2 (b) 6, 2
(a) 0 (b) 8 (c) 2, 3 (d) 6, 3
(c) 16 (d) 24
y
o
u
Cube & Dice A-121

rs
m
16. Which of the following numbers does not appear on any

a
one of the adjacent surfaces of the number 3 ?

h
(a) 2 (b) 6

b
o
(c) 4 (d) 1

o
b
17. On the basis of two positions of dice, find what number will (A) (B) (C) (D)

.w
be on the opposite face of number 5 ? (a) A only (b) B only

o
(c) A and C only (d) A, B, C and D

rd
6 5 22. How many dots are their on the dice face opposite the one

p
re
4 3 with three dots ?

s
3 1

s
.c
o
(i) (ii)

m
(a) 1 (b) 3
(c) 4 (d) 5
18. From the following positions of dice, find which number
will come in place of ‘?’.
(i) (ii) (iii) (iv)
4 5 1 (a) 2 (b) 4
3 2 ? (c) 5 (d) 6
2 1 6 23. Which number is on the face 4, if the four different positions
of a dice are as shown in the figures given below.
(i) (ii) (iii)
(a) 4 (b) 5 (c) 2 (d) 3
19. Three positions of the same dice are given below. Observe 4 2 1 2
the figures carefully and find which number will come in
place of ‘?’. 6 5 5 1 5 6 4 3

1 3 4
3 4 ? (i) (ii) (iii) (iv)
6 5 2 (a) 5 (b) 3
(i) (ii) (iii)
(c) 2 (d) 1
(a) 1 (b) 6 (c) 3 (d) 5 24. Six squares are coloured, front and back, red (R), blue (B),
20. Select from the alternative, the box that can be formed by yellow (Y), green (G), white (W) and orange (O) and are
folding the sheet shown in figure (X) : hinged together as shown in the figure given below. If they
are folded to form a cube, what would be the face opposite
the white face?
R B
Y O G
W
(a) R (b) G
(c) B (d) O
25. Three views of a cube following a particular motion are
(X) given below:

K H B
B K H
(A) (B) (C) (D) A M P
(a) A only (b) A and C only
(c) A , C and D only (d) A, B, C and D What is the letter opposite to A?
21. Select from the alternative, the box that can be formed by (a) H (b) P
folding the sheet shown in figure (X) : (c) B (d) M
F 26. A cube has six numbers marked 1, 2, 3, 4, 5 and 6 on its faces.
Three views of the cube are shown below:
A E
1 3 3
B
4 2 6
C D 6 1 5
(X)
y
o
u
A-122 Cube & Dice

rs
m
What possible numbers can exist on the two faces marked
3 2

a
A and B , respectively on the cube?

h
(c) 6 (d) 3

b
4 5

o
o
B

b
32. Select from alternative, the box that can be formed by folding

.w
5 the sheet shown.
A

o
rd
p
re
(a) 2 and 3 (b) 6 and 1

s
s
(c) 1 and 4 (d) 3 and 1

.c
27. The four different position of dice are given below ×

o
m
=
6 6 5 1
3 2 2 4 6 4 4 2
+
(i) (ii) (iii) (iv)

Which number is on the face opposite of 6?
(a) 1 (b) 2
(c) 3 (d) 4 ×
(a) + (b) =
=
28. =

(i) (ii) (iii) (iv)


(c) × (d) – ×
Symbol at bottom of (iv)
(a) (b)
(c) (d) 2 5 1 6
33. 5 2 6 4
4 1 5 5
E C D B
29. C E B A Which No. is opposite to 1?
A D C C (a) 4 (b) 6
Which letter will be opposite of letter D ? (c) 2 (d) 3
(a) A (b) B 34. Four views of a dice have been shown below, which of the
(c) E (d) F following symbols is on the face opposite to the face having
the symbol ÷ ?
× ?
30. =
Which symbol appear in place of ?
(a) × (b) = (a) $ (b) 0
(c) (d) (c) = (d)
31. Select from alternative, the box that can be formed by fold- 35. Choose the box that is similar to the box formed from the
ing the sheet shown. given sheet of paper (X).

3 2 5
1

4 1
(a) 3 (b) 6
2 2 (x)
y
o
u
Cube & Dice A-123

rs
m
a
h
(1) (2)

b
o
o
b
.w
(3) (4)

o
rd
(a) 1 only (b) 2 and 3 only

p
(c) 1 and 3 only (d) 1, 2 and 4 only

re
36. A dice is numbered from 1 to 6 in different ways.

s
(X)

s
If 1 is adjacent to 2, 3 and 5, then which of the following

.c
o
statements is necessarily true?

m
(a) 4 is adjacent to 6 (1) (2)
(b) 2 is adjacent to 5
(c) 1 is adjacent to 6
(d) 1 is adjacent to 4
37. The four different positions of a dice are given below: Find
the number on the face opposite the face showing 6? (3) (4)

(a) 1 and 2 only (b) 2 and 3 only


(c) 2 and 4 only (d) 1, 2, 3 and 4
40. Choose the cube that will be formed by folding the sheet of
paper shown in the problam figure.
(a) 1 (b) 2 Question figure :
(b) 4 (d) 5
38. Choose the box that is similar to the box formed from the
given sheet of paper (X).

Answer figures :

(X)

(1) (2) (a) (b) (c) (d)


41. Two positions of a dice are shown below. If 1 is at the
bottom, which number will be on top?
(3) (4)

(a) 1 only (b) 1, 2 and 3 only


(c) 2 and 3 only (d) 1, 2, 3 and 4 only (a) 4 (b) 3
39. Choose the box that is similar to the box formed from the (c) 8 (d) 5
given sheet of paper (X). 42. Four positions of a dice are given below, Identify the number
at the bottom when top is 6.
Question figures
2 4 6 5
1 3 1 2 4 2 4 6
(a) 1 (b) 3
(c) 4 (d) 5

ANSWER KEY
1 (a) 6 (d) 11 (c) 16 (b) 21 (b) 26 (a) 31 (a) 36 (a) 41 (b)
2 (d) 7 (a) 12 (a) 17 (c) 22 (c) 27 (a) 32 (c) 37 (c) 42
3 (b) 8 (c) 13 (a) 18 (d) 23 (d) 28 (d) 33 (a) 38 (d)
4 (c) 9 (d) 14 (c) 19 (a) 24 (d) 29 (a) 34 (a) 39 (d)
5 (c) 10 (a) 15 (d) 20 (a) 25 (a) 30 (a) 35 (c) 40 (b)
y
o
u
A-124 Cube & Dice

rs
m
a
h
b
o
o
b
.w
o
(1-4): Now, all uncoloured faces of each cuboid are coloured with

rd
pink and then again cut each cuboid into four cuboids.

p
Red
Blue

re
s
Green

s
.c
o
Blue

m
I
1. (a) Three surfaces coloured is constantly 8. Blue
2. (d) No. surfacecoloured = ( – 2) (b – 2) (h – 2) II
=3×1×2=6
3. (b) There are three cubes on each red-green interface
(barring comer cubes). So, 4 × 3 = 12 cubes. III
IV
4. (c) One surface coloured
= 2(1–2) (b – 2) + 2 (1 – 2) (h – 2) + 2 (b – 2) (h –2 ) Blue
= 2{3 × 1 + 3 × 2 + 1 × 2} = 22 In set I and IV : 2 cuboids have 2 faces blue, 2 faces pink
5-8. Since, there are 64 smaller cubes of equal size, therefore, and 2 faces uncoloured each. 2 cuboids have 1 face blue, 3
n = no. of divisions on the face of undivided cube = 4 faces pink and 2 faces uncoloured each.
5. (c) no. of cubes with no face coloured = (n – 2)3 In set II : 2 cuboids have 2 faces blue, 2 faces pink and 2
= (4 – 2)3 = 8 faces uncoloured each.
6. (d) no. of cubes with one face painted = (n – 2) 2 × 6 1 cuboid has 3 faces blue, 1 face pink and 2 faces uncoloured
= (4 – 2)2 × 6 = 24 each.
7. (a) Number of cubes with two red opposite faces = 0 1 cuboid has 1 face blue, 3 faces pink and 2 faces uncoloured
(none of the cubes can have its opposite faces each.
In set III : All the four cuboids have 1 face blue, 3 faces pink
coloured)
and 2 faces uncoloured each.
8. (c) Number of cubes with three faces coloured
9. (d) There are 2 cuboids in set I, 2 cuboids in set II and 2
= 4(cubes at top corners) + 4(cubes at bottom corners) cuboids in set IV having 2 faces pink in each. Thus,
=8 there are 2 + 2 + 2 = 6 such cubes.
9-11. The adjoining figure shows the cube coloured and cut 10. (a) There are 2 cuboids in set I, 1 cuboid in set II, 4 cuboids
into four cuboids as stated in the question. in set III and 2 cuboids in set IV having 3 faces pink
Blue (top) each. Thus, there are 9 such cuboids.
11. (c) There is only one cuboid having three faces blue. This
cuboid lies in set II.
Blue (side) 12. (a) Since 1, 3, 5 and 6 are adjacent to 2. Therefore, 4 lies
opposite 2 i.e. at the bottom face of dice (i).
13. (a) Since 2, 3, 6 and 4 are adjacent to 1. Therefore, 5 lies
I II opposite 1. Hence, 1 lies at the bottom face of dice (iii).
14. (c) Since 1, 4, 2 and 5 are adjacent to 6. Therefore, 3 lies
III IV opposite 6.
15. (d)
Blue (front) 2
Four cuboids are obtained as shown below :
6 1 3
p)
p)

(to
(to

e 4
blu
ue

e)
sid
bl

(
ue

5
bl

blue blue
(front) (front)
Hence, 1, 5, 3 and 6 are adjacent to 4.
I II 16. (b) From the figure in previous solution, 6 is not adjacent
to 3.
17. (c) Common number i.e. 3 to both the dice is placed on the
central position of the figure. Now place the numbers
e)

in the anticlock wise direction in block I, II, III and IV


( sid

respectively.Remaining number i.e. 2 will come in the


ue
bl

blue blue
(front) (front) block V. Hence number 4 is opposite to number 5.

III IV
y
o
u
Cube & Dice A-125

rs
The numbers adjacent to ‘1’ are ‘2’ and ‘3’ in fig. (2).

m
II

a
6 From these two statements, we can’t deduce that the

h
number opposite to ‘1’ is ‘5’.

b
o
III 5 3 4 I Possible value of (A) and (B) can be 2 and 3.

o
27. (a) 2, 3, 4, 5, appear adjacent to 6.

b
.w
1 IV 29. (a) It is clear that letter F will be opposite to C and A will be

o
opposite to letter D.

rd
2 V 30. (a) From Figure

p
18. (d)

re
4 =

s
s
.c
o
5 2 3

m
1 ×
32. (c) is opposite to =
6 × will opposite to +
Hence, the numbers that are adjacent to 1 are 5, 2, will opposite to –
6 and 3. hence (a) (b) (d) are not identical.
3 will come in place of ‘?’. 33. (a) It is clear from figure that when 1 appear 4 is not there.
19. (a) 34. (a) ÷ + O$
6 All other symbols are adjacent to . Therefore, $ symbol
is opposite to it.
5 3 1 35. (c) The fig. (X) is similar to form V. So, when the sheet
shown in fig. (X) is folded to form a cube, then the face
4 with shading lies opposite to the free bearing a square,
the face bearing a dot lies opposite to a blank face and
2 the face bearing a circle (with a '+' sign inside it) lies
Since 2, 3, 5 and 1 are adjacent to 4. Therefore, 1 will opposite to another blank face. The cubes in figures (2)
come in place of ‘?’. and (4) have the shaded face adjacent to the face bear-
20. (a) When the sheet shown in fig. (X) is folded to from a ing a square. Therefore, the cubes in these two figures
cube, then the face bearing the dot lies opposite to the cannot be formed. Hence, only cubes in figures (1) and
shaded face, the face bearing a circle (With ‘+’ sign (3) can be formed.
inside it) lies opposite to a blank face and the remain- 36. (a) If 1 is adjacent to 2, 3 and 5, then either 4 or 6 lies
ing two blank faces lie opposite to each other. Clearly, opposite to 1. So, the numbers 4 and 6 cannot lie oppo-
the cubes shown in figures (B) and (D) cannot be site to each other. Hence, 4 necessarily lies adjacent to 6.
formed since they have the shaded face adjacent to 37. (c) From figures (i), (ii) and (iii), we conclude that 3, 4, 2 and
the face bearing a dot and the cube shown in fig. (C) 6 lie adjacent to 5. Therefore, 1 must lie opposite 5.
cannot be formed since it shown all the three blank From figures (i), (iii) and (iv), we conclude that 4, 5, 6
face adjacent to each other. Hence, only the cube and 1 lie adjacent to 3. Therefore, 2 must lie opposite 3.
shown in fig.(A) can be formed. Now, we have 1 opposite 5 and 2 opposite 3. Hence, 4
must lie opposite 6.
21. (b) When the sheet in fig. (X) is folded to form a cube ,
38. (d) The fig. (X) is similar to the Form V. So, when the sheet
then ‘F’ appears opposite ‘C’ and ‘A’ appears oppo-
in fig. (X) is folded to form a cube, then the face bearning
site ‘D’. Therefore, the cube in fig. (A) which shows
a dot appears opposite to a blank face, the face bear-
‘F’ adjacent to ‘B’, the cube in fig.(C) which shows ‘E’ ing a '+' sign appears opposite to another blank face
adjacent to ‘C’ and the cube in fig. (D) which shows and the face bearing a circle appears opposite to the
‘A’ adjacent to ‘D’ cannot be formed. third blank face. Clearly, all the four cubes shown in
22. (c) From figure (i) (ii) and (iv), we conclude that 6, 4, 1 and figures (1), (2), (3) and (4) can be formed.
2 dots appear adjacent to 3 dots. Clearly, there will be 39. (d) The fig. (X) is similar to the Form II. So, when a cube is
5 dots on the face opposite the face with 3 dots. formed by folding the sheet shown in fig. (X), then the
23. (d) From figures (i) and (iv) we conclude that 6, 5, 2 and 3 two half-shaded faces lie opposite to each other and
lie adjacent to 4. It follows that lies opposite 4. one of the three blank faces appears opposite to the
24. (d) G–O 1 R face bearing a dot. Clearly, each one of the four cubes
R–W G B shown in figures (1), (2), (3) and (4) can be formed by
W Y 2
B– Y O 3 folding the sheet shown in fig. (X).
25. (a) B & K can’t opposite to A 41. (b) By looking, the dice position, we can say that 2, 4, 5
M & K can’t opposite to H and 6 are adjacent faces of 3. therefore, if 1 number is
B & P can’t opposite to H at the bottom then 3 will be on the top.
From above statements 42. (a) Number opposite to 4 will be 1, 2 , 3, 5, 6
H can’t be opposite to B, K, M, P opposite to (4) will be 3
Thus H will in opposite of A. 6 1, 2,3, 4, 5
26. (a) By soing options. Now, if 4 is opposite to 3 then
The numbers adjacent to ‘1’ are ‘4’ and ‘6’ in fig. (1). 1 will be opposite to 6.
y
o
u
A-126 Completion of Figure

rs
m
a
CHAPTER

h
COMPLETION OF

b
18

o
o
b
.w
FIGURE

o
rd
p
re
s
s
.c
In this section, an incomplete figure is given, in which some

o
part is missing. We have to choose the segment, given in

m
choices, that exactly fits into the blank portion of figure so that (c) (d)
the main figure is completed.
Note : If you observe carefully, you notice that the missing Sol.
portion may be the mirror image of any one of the quarters. (c) Here, if you see carefully, you observe that the missing
EXAMPLE 1. portion will be the mirror image of quarter I or III. Hence,
Select from alternatives the figure (X) that exactly fits in option (c) completes the given figure.
the main figure to complete its original pattern.

?
(X)
EXAMPLE 3.
Select from alternatives the figure (X) that exactly fits in
(a) (b) the main figure to complete its original pattern.

(c) (d) ?
Sol.
(b) In this question, half shaded leaf is moved clockwise. So,
option (b) is right one.

(X)

(a) (b)

EXAMPLE 2.
Select from alternatives the figure (X) that exactly fits in
the main figure to complete its original pattern. (c) (d)

Sol.
? (d) Here, missing portion will be the mirror image of quarter
I.

(X)

(a) (b)
y
o
u
Completion of Figure A-127

rs
m
EXAMPLE 4. EXAMPLE 5.

a
Select from alternatives the figure (X) that exactly fits in Select from alternatives the figure (X) that exactly fits in

h
b
the main figure to complete its original pattern. the main figure to complete its original pattern.

o
o
b
.w
o
rd
p
re
?

s
s
.c
o
(X)

m
(X)

(a) (b) (a) (b)

(c) (d)
(c) (d)
Sol.
(d) Only option (d) is right one. (It is the mirror image of each
Sol.
quarter).
(a)

DIRECTIONS : In each of the following questions, a part of the


figure is missing. Find out from the given options (a, b, c and d)
the right figure to fit in the missing place. (a) (b)

1.

? (c) (d)

3.

(a) (b)

(c) (d)

2. (a) (b)

(c) (d)
y
o
u
A-128 Completion of Figure

rs
m
4.

?
8.

a
h
b
o
o
b
.w
o
rd
p
re
s
(a) (b)

s
.c
(a) (b)

o
m
(c) (d)
(c) (d)
5.
9.
?

(a) (b)

(a) (b) (c) (d)

(c) (d) 10.

6.

?
(a) (b)
(a) (b)

(c) (d)
(c) (d)
11.
7.

(a) (b) (a) (b)

(c) (d)
(c) (d)
y
o
u
Completion of Figure A-129

rs
m
a
12.

h
(a) (b)

b
o
o
b
.w
? (c) (d)

o
rd
p
16.

re
s
s
.c
o
(a) (b)

m
(c) (d)
(a) (b)

13.
(c) (d)

17.

(a) (b)
(a) (b) (c) (d)
18.
(c) (d)

14.

(a) (b)

(c) (d)

(a) (b) 19.

(c) (d)

15.
(a) (b)

(c) (d)
y
o
u
A-130 Completion of Figure

rs
m
20.

a
24.

h
?

b
o
o
b
.w
o
rd
p
re
s
(a) (b) (c) (d)

s
(a) (b)

.c
o
21.

m
(c) (d)

25.

(a) (b)

(c) (d)
(a) (b)

22.

(c) (d)

26.
?
(a) (b)

(a) (b)
(c) (d)

(c) (d)
23.
27.
?

(a) (b)
(a) (b)

(c) (d)
(c) (d)
y
o
u
Completion of Figure A-131

rs
m
28. 31.

a
h
b
o
o
b
.w
?

o
rd
p
re
(a) (b) (c) (d)
(a) (b)

s
s
.c
32.

o
m
(c) (d)

29.

(a) (b) (c) (d)

33.

(a) (b) (c) (d)

30.

(a) (b) (c) (d)

(a) (b)

(c) (d)

ANSWER KEY
1 (a) 5 (c) 9 (d) 13 (b) 17 (d) 21 (a) 25 (c) 29 (d) 33 (b)
2 (a) 6 (c) 10 (d) 14 (d) 18 (b) 22 (c) 26 (c) 30 (c)
3 (a) 7 (d) 11 (c) 15 (b) 19 (d) 23 (a) 27 (d) 31 (c)
4 (a) 8 (c) 12 (a) 16 (c) 20 (c) 24 (a) 28 (c) 32 (b)
y
o
u
A-132 Completion of Figure

rs
m
a
h
b
o
o
b
.w
1. (a) 8. (c)

o
rd
p
re
s
s
.c
o
m
2. (a) 9 (d)

3. (a)
10. (d)

4. (a) 11. (c)

12. (a)
5. (c)

13. (b)
6. (c)

14. (d)
7. (d)
y
o
u
Completion of Figure A-133

rs
m
15. (b) 24. (a)

a
h
b
o
o
b
.w
o
rd
p
16. (c) 25. (c)

re
s
s
.c
o
m
18. (b) 26. (c)

19. (d) 27. (d)

21. (a) 28. (c)

22. (c)
29. (d)

23. (a) 30. (c)


y
o
u
A-134 Figure Formation & Analysis

rs
m
a
CHAPTER

h
FIGURE FORMATION

b
19

o
o
b
.w
& ANALYSIS

o
rd
p
re
s
s
.c
o
In this topic, a question is one of the following types : Sol. (c) Since we have to construct a square, therefore, first

m
I. Formation of triangles/square/rectangle etc. either by joining select a piece which contains a right angle between two
of three figures after choosing them from the given five adjacent edges. Then try to fit another piece in its hollow
figures or by joining any other pieces after selecting them spaces. If it doesn’t fit then select another piece. Repeat
from given alternatives. this procedure with different pieces. Similarly find the third
II. Making up a figure from given components. piece to get a completed square.
III. Making up a three dimensional figure by paper folding.
IV. Rearrangement of the parts of given figure. C
V. Fragmentation of key figure into simple pieces. E
A
TYPE-I : Formation of triangles/square/rectangle etc. either by TYPE-II : Making up a figure from given components
joining of three figures after choosing them from the given five
figures or by joining any other pieces after selecting them from EXAMPLE 3.
given alternatives. Find out which of the alternatives (a), (b), (c) and (d) can be
1. formed from the pieces given in box ‘X’.
EXAMPLE
A set of five figures (A), (B), (C), (D) and (E) are followed by
four combinations as the alternatives. Select the alternative
which represents the combination of figures which if fitted
together, will form a complete triangle.

(X)

(A) (B) (C) (D) (E) (a) (b) (c) (d)


(a) BCD (b) ABD
(c) CDE (d) ABE
Sol. Sol. (b)
(d) If figures A, B and E are fitted together, the resultant

EXAMPLE 4.
Find out which of the alternatives (a), (b), (c) and (d) can be
figure will be a triangle. B formed from the pieces given in box ‘X’.
A E
EXAMPLE 2.
A set of five figures (A), (B), (C), (D) and (E) are followed
by four combination as the alternatives. Select the
alternative which represents the combination of figures (X)
which if fitted together, will form a complete square.

(a) (b) (c) (d)


(A) (B) (C) (D) (E)
(a) ABC (b) ACD
Sol. (b)
(c) ACE (d) CDE
y
o
u
Figure Formation & Analysis A-135

rs
m
TYPE-III : Making up a three dimensional figure by paper folding.

a
In this type, we have to analyze when a paper folded along the

h
b
lines, how a three dimensional figure look like. Sometimes, a key

o
figure is given which is made by folding one of the four figures

o
(a) (b) (c) (d)

b
given in alternatives. We have to determine which figure can be

.w
Sol. (a)
used to create the key figure. TYPE-V : Fragmentation of key figure into simple pieces.

o
rd
This type is opposite to TYPE-II. In this type, a key figure is
EXAMPLE 5.

p
given and every alternatives has different pieces. We have to

re
A figure ‘X’ is given. You have to choose the correct figure, select the set of pieces that can make the given key figure.

s
s
given in the alternatives, when folded along the lines, will

.c
EXAMPLE 7.
produce the given figure ‘X’.

o
Find out which of the alternatives will exactly make up the

m
key figure (X)

(X)
(X)

(a) (b) (c) (d)


(a) (b) (c) (d) Sol. (a)

Sol. (a) EXAMPLE 8.


In the following question a key figure is given. Each
TYPE-IV : Rearrangement of the parts of given figure. alternatives contains various pieces. Find out which of
In this type of questions, a key figure is given. We have to identify the alternatives will exactly made up the key figure.
the figure from alternatives that is a rearrangement of parts of key
figure.

EXAMPLE 6. (X)
Which figure is the rearrangement of the parts of the given
figure.

(a) (b) (c) (d)

Sol. (a)
(X)

TYPE-I. 2.
DIRECTIONS (Qs. 1 to 3) : In each of following questions select
that combination of parts (A), (B), (C), (D) and (E) which if
fitted together will form an equilateral triangle. (A) (B) (C) (D) (E)
(a) ABC (b) ACE
1. (c) BCD (d) BDE

3.
(A) (B) (C) (D) (E)
(A) (B) (C) (D) (E)
(a) ABC (b) BCD
(a) ABE (b) BCE
(c) ABD (d) ABE
(c) ADE (d) BDE
y
o
u
A-136 Figure Formation & Analysis

rs
m
DIRECTIONS (Qs. 4 to 8) : In each of following questions, select 10.

a
that combination of parts (A), (B), (C), (D) and (E) which if

h
fitted together will form a square.

b
o
o
b
4.

.w
o
rd
(X)

p
(A) (B) (C) (D) (E)

re
s
(a) ABD (b) BCE

s
.c
(c) ACD (d) BDE

o
m
5. (a) (b) (c) (d)
11. In this question a key figure (X) is given followed by five
(A) (B) (C) (D) (E) alternative figures. You will have to select one figure from
the alternatives which fits exactly into key figure (X) to
(a) ACD (b) ABD form a rectangle.
(c) BCD (d) CDE
6.

(X)
(A) (B) (C) (D) (E)
(a) ABC (b) BCD
(c) ACD (d) CDE

7. (a) (b) (c) (d)


12. In this question, a key figure is given followed by four
alternative figures. You will have to select one figure from
(A) (B) (C) (D) (E) the alternatives which fits exactly into key figure (X) to
form a complete circle.
(a) ABD (b) BCD
(c) BDE (d) ADE

8.

(X)
(A) (B) (C) (D) (E)
(a) ABC (b) BCE
(c) BDE (d) ADE

DIRECTIONS (Qs. 9 to 10): a key figure (X) is given followed by (a) (b) (c) (d)
four alternative figures. You will have to select one figure from
the alternatives which fits exactly into key figure (X) to form a TYPE-II
perfect square. DIRECTIONS (Qs. 13 to 16) : In each of following questions
9. find out which of the figures (a), (b), (c) and (d) can be formed
from the pieces given in (X).
13

(X) (a) (b) (c) (d)


(X)
14.

(a) (b) (c) (d)


(a) (b) (c) (d)
(X)
y
o
u
Figure Formation & Analysis A-137

rs
m
15.

a
h
b
o
o
b
(c) (d)

.w
(a) (b)
(X)

o
rd
p
16.

re
(a) (b)

s
s
.c
o
m
(a) (b) (c) (d)
(X)

17.
(c) (d)

TYPE IV
(X) (1) (2) (3) (4)
(X) (a) (b) (c) (d) DIRECTIONS (Qs. 22 to 24) : Which figure is the rearrangement
of the parts of key figure X.
18.

22.
(1) (3) (4)
(X) (a) (b) (c) (d)
(X)

TYPE-III

DIRECTIONS (Qs. 19 & 20) : In the following questions how does


the figure look when folded into a cube along the marked line?
19. (a) (b)

(1) (2) (3) (4)

(X) (c) (d)


23.
(a) 1 and 3 (b) 3 and 4
(c) 2 and 3 (d) 2 and 4
20.

(1)
(X)
(2) (3) (4)
(X)

(a) 1, 3 and 4 (b) 2, 3 and 4


(c) 1, 2 and 3 (d) 1, 2 and 4
21. In this question, which of the following figures (a), (b), (c),
(d) when folded along the lines, will produce the given figure
(a) (b)
‘X’.

(X)
(c) (d)
y
o
u
A-138 Figure Formation & Analysis

rs
m
24. 28.

a
h
b
o
o
b
.w
(X)
(X)

o
rd
p
re
s
s
.c
(a) (b) (c) (d)

o
m
29.
(a) (b) (d)
(c)

TYPE V
DIRECTIONS (Qs. 25 - 34) : In each of the following questons (X)
select the option that can make up the key figure (X)
25.

(X) (a) (b) (c) (d)


30.

(a) (b) (c) (d)


26. (X)

(X)

(a) (b) (c) (d)

31.
(a) (b) (c) (d)
27.

(X)

(X)

(a) (b) (c) (d)


32.

(a) (b)
(X)

(c) (d) (a) (b) (c) (d)


y
o
u
Figure Formation & Analysis A-139

rs
34. Find out which answer figure will exactly make up the

m
33.

a
question figure.

h
Question figure :

b
o
o
b
.w
(X)

o
rd
p
Answer figures :

re
s
s
.c
o
m
(a) (b) (c) (d) (a) (b) (c) (d)

ANSWER KEY
1 (b) 5 (a) 9 (c) 13 (b) 17 (c) 21 (a) 25 (c) 29 (a) 33 (a)
2 (d) 6 (d) 10 (b) 14 (c) 18 (c) 22 (c) 26 (d) 30 (d) 34 (c)
3 (c) 7 (b) 11 (c) 15 (c) 19 (b) 23 (a) 27 (c) 31 (d)
4 (b) 8 (c) 12 (b) 16 (a) 20 (a) 24 (d) 28 (a) 32 (d)

17. (c) All of the components of figure (X) are present in the
figure (c)
B
1. (b) 2. (d) E 18. (c) All of the components of figure (X) are present in the
C B
D D figure (c)

28. (a) 29. (a)


D E
3. (c) E 4. (b) B
A C

D 30. (d) 31. (d)


C D
5. (a) A 6. (d)
C E

B B
7. (b) 8. (c) D
C D E 32. (d) 33. (a)

11. (c) 12. (b)


34. (c)
y
o
u
A-140 Paper Cutting & Folding

rs
m
a
CHAPTER

h
PAPER CUTTING

b
20

o
o
b
.w
& FOLDING

o
rd
p
re
s
s
.c
In this section, a sheet of paper is folded in given manner and

o
m
cuts are made on it. A cut may be of verying designs. We have
to analyze how this sheet of paper will look when paper is Sol. (b) Unfolded Step I
unfolded.
Note that when a cut is made on folded paper, the designs of
the cut will appear on each fold.
In each of following examples, figures A and B show a sequence Step II
of folding a square sheet. Figure C shows the manner in which
folded paper has been cut. You have to select the appropriate Since a quarter circle is made on folded paper.
figure from alternatives which would appear when sheet is opened. Therefore, when the paper is opened, a complete
EXAMPLE 1. circle will appear on paper.
EXAMPLE 3.

(A) (B) (C )

(A) (B) (C)

(a) (b) (c) (d)


Sol. (a) Step I-When sheet C is unfolded once, it will appear as
(a) (b) (c) (d)
follows
Sol. (d) Here, a circular cut is made on the quarter circle. Hence,
this sheet, when completely unfolded, will contain small
circle on each quarter and will appear as option (d).

EXAMPLE 4.
Step II -

Clearly, the circle will appear in each of the triangular


quarters of the paper.
EXAMPLE 2.
(A) (B) (C)

(A) (B) (C)


(a) (b) (c) (d)

Sol. (a) When sheet (c) is unfolded once, it will appear as shown
below :
(a) (b)

This sheet, when completely unfolded will appear as


option (a).
(c) (d)
y
o
u
Paper Cutting & Folding A-141

rs
m
EXAMPLE 5. Sol. (d) We unfold the paper step by step as follows

a
h
Step I

b
o
o
b
.w
Step II

o
(A) (B) (C)

rd
p
Step III

re
s
s
So, option (d) is right one.

.c
o
(a) (b) (c) (d) 7.

m
EXAMPLE

Sol. (a) When sheet C is unfolded once, it will appear as shown


below

(A) (B) (C)

This sheet, when completely unfolded will appear as


option (a).
EXAMPLE 6.
(a) (b) (c) (d)
Solution :
(d) We unfold the paper step by step as follows:
P (A) (B) (C)

Step I

(a) (b)

Step II
(c) (d)

DIRECTIONS (Qs. 1 to 3) : If a square sheet of paper is folded 2.


two times from the centre and cuts are made as shown in the
problem figure how will it appear when it is opened ? Select
the appropriate figure from the answer choices marked (a),
(b), (c) and (d). (X)

1.

(a) (b) (c) (d)


3.
(X)

(X)

(a) (b) (c) (d)

(a) (b) (c) (d)


y
o
u
A-142 Paper Cutting & Folding

rs
DIRECTIONS (Qs. 4 to 24) : In each of the following questions

m
9.

a
a set of three figures A, B and C showing a sequence of folding

h
of a piece of paper. Fig. (C) shows the manner in which the

b
o
folded paper has been cut. These three figures are followed by

o
b
four answer figures from which you have to choose a figure (A) (B) (C)

.w
which would most closely resemble the unfolded form of fig.

o
(C).

rd
p
4.

re
s
(a) (b) (c) (d)

s
.c
o
10.

m
(A) (B) (C)

(A) (B) (C)

(a) (b) (c) (d)

5. (a) (b) (c) (d)


11.

(A) (B) (C)

(A) (B) (C)

(a) (b) (c) (d)


(a) (b) (c) (d)
6.
12.

(A) (B) (C)

(a) (b) (c) (d)


(a) (b) (c) (d)
13.
7.

(A) (A) (B) (C)


(B) (C)

(a) (b) (c) (d) (a) (b) (c) (d)

8. 14.

(A) (B) (C)

(a) (b) (c) (d) (a) (b) (c) (d)


y
o
u
Paper Cutting & Folding A-143

rs
m
15. 21.

a
h
b
o
(A) (B) (C)

o
(A) (B) (C)

b
.w
o
rd
(a) (b) (c) (d)

p
re
(a) (b) (c) (d) 22.

s
s
.c
16. B

o
m
(A) (B) (C) (P) (X) (Y) (Z)
B B B B B
B B B

B
B
B
B B B B
B (a) (b)
(a) (b) (c) (d)
17.
(c) (d)

(P) (A) (B) (C)


23.

(a) (b) (c) (d)


(X) (Y) (Z)
18.

(A) (B) (C) (a) (b)

(a) (b) (c) (d) (c) (d)


19.
24.

(A) (B) (C) X Y Z

(a) (b)

(a) (b) (c) (d)


20. (c) (d)
DIRECTIONS (Qs. 25 to 34) : In the following questions, a
square sheet of paper is folded along the dotted lines and
then cuts are made on it. How would the sheet look when
(A) (B) (C) opened? Select the correct figure from the given choices.
25.

(a) (b)

(c) (d)
(a) (b) (c) (d)
y
o
u
A-144 Paper Cutting & Folding

rs
m
31. A piece of paper is folded and punched as shown below in
26.

a
the question figures. From the given answer figures, indicate

h
how it will appear when opened.

b
o
Question Figures :

o
b
.w
o
rd
p
re
(a) (b) (c) (d)
Answer Figures :

s
s
.c
o
27.

m
(a) (b) (c) (d)
32. Question figure

(a) (b) (c) (d)

28. Answer figures

(a) (b) (c) (d)


33. A piece of paper is folded and cut as shown below in the
(a) (b) (c) (d) question figures. From the given answer figures, indicate
how it will appear when opened.
Question figures :
29.
(cut)

Answer figures :

(a) ( b) (c ) (d)

30. A square paper is folded in a particular manner and a (a) (b) (c) (d)
punch is made. When unfolded, the paper appears as 34. A piece of paper is folded and cut as shown below in the
question figures. From the given answer figures, indicate
given below :
how it will appear when opened.
Question figures

Choose the manner in which the paper is folded and


punch is made. Answer figures

(a) (b) (c) (d)


(a) (b) (c) (d)
y
o
u
Paper Cutting & Folding A-145

rs
m
ANSWER KEY

a
h
b
1 (b) 5 (c) 9 (c) 13 (b) 17 (a) 21 (d) 25 (c) 29 (a) 33 (c)

o
o
2 (b) 6 (b) 10 (d) 14 (d) 18 (d) 22 (d) 26 (d) 30 (a) 34 (b)

b
.w
3 (a) 7 (a) 11 (c) 15 (c) 19 (b) 23 (d) 27 (d) 31 (c)

o
4 (b) 8 (c) 12 (b) 16 (d) 20 (c) 24 (b) 28 (c) 32 (d)

rd
p
re
s
s
.c
o
m
1. (b) We unfold the paper step by step as follows 5. (c) Unfolded step I

2. (b) Unfolded

step II
Step I - Step -II

31. (c)
4. (b) Unfolded step I - step II -

32. (d)
step III -

(a) (b) (c) (d)


y
o
u
A-146 Visual Reasoning

rs
m
a
CHAPTER

h
b
21

o
VISUAL REASONING

o
b
.w
o
rd
p
re
s
s
.c
VISUALIZATION Counting of figures problem

o
m
Visual intelligence measures the ability to process visual material
and to employ both physical and mental images in thinking. As a EXAMPLE 2.
result people with a high visualization find it easier to comprehend How many triangles are there in the given diagram?
information and communicate it to others. Your visualization skills
determine how well you perceive visual patterns and extract
information for further use. Visualization also facilitates the ability
to form associations between pieces of information something
which helps improve long term memory.
Odd-man out type problem
(a) 18 (b) 17 (c) 16 (d) 15
EXAMPLE 1. Sol. (b) B C
Choose the figure which is different from the others.
H
G
A D

(a) (b) (c) (d) F E


Sol. (d) The figures form a series. The complete figure rotates The triangles are:
90º CW in each step. Fig. (d) does not fit in the series ABF: AGB : AGF : BFC;
as it is the same as fig. (b). BCE : CEF : BFE : HBC;
Hence fig. (d) is the answer. HCE : HEF : HBF : BGH;
FGH : HCD : HDE : AFH;
ABH ;

DIRECTIONS (Qs. 1-3) : Choose the figure which is different 4. Group the following figures into three classes on the basic
from the others. of identical properties.

1.
1 2 3
(a) (b) (c) (d)

4 5 6
2.

(a) (b) (c) (d)


7 8 9

3. (a) 1, 3, 9 ; 2, 5, 8 ; 4, 6, 7
(b) 4, 8, 9 ; 1, 2, 5 ; 3, 6, 7
(c) 2, 5, 9 ; 1, 3, 8 ; 2, 6, 7
(a) (b) (c) (d) (d) 1, 8, 9 ; 4, 6, 7 ; 2, 3, 5
y
o
u
Visual Reasoning A-147

rs
m
5. How many triangles are there in the following figure? 11. How many cubes are there in the following figure?

a
h
b
o
o
b
.w
o
rd
p
re
(a) 6 (b ) 10

s
s
(c) 12 (d) 8

.c
(a) 29 (b) 27

o
(c) 23 (d) 30 12. What is the number of squares in figure?

m
6. How many triangles are there in the given figure?
A B

C D
(a) 16 (b) 14 (a) 12 (b) 13
(c) 8 (d) 12 (c) 15 (d) 17
7. How many triangles are there in the given figure?
13. What is the number of triangles in figure?

(a) 29 (b) 38
(c) 40 (d) 35
8. How many squares are there in a given figure? (a) 16 (b) 28
(c) 32 (d) 38

14.

(a) 12 (b) 13
(c) 10 (d) 11
9. How many triangles are there in the figure ABCDEF?
A B

F C
How many triangles are there ?
E D
(a) 20 (b) 21
(a) 24 (b) 26 (c) 26 (d) 28
(c) 28 (d) 30
15. How many Rectangles are there in the given figure?
10. How many parallelograms are there in the figure ?

(a) 14 (b) 15
(a) 13 (b) 14
(c) 16 (d) 18
(c) 15 (d) 17
y
o
u
A-148 Visual Reasoning

rs
m
16. How many Semicircles are there in the given figure? 20. What is the number of triangles in figure ?

a
h
b
1

o
o
b
.w
o
3 4 5 6

rd
2
7 8

p
re
9 13
12

s
10

s
(a) 16 (b) 14 11

.c
(c) 12 (d) 10 14 15

o
16

m
17. Count the number of squares in the given figure.
(a) 32 (b) 39
(c) 46 (d) 60
21. How many triangles are there in the given figure?

(a) 22 (b) 20
(c) 18 (d) 14
18. How many circles are there in the adjoining figure. (a) 10 (b) 12
(c) 14 (d) 11
22. How many triangles are there in the give figure ?

(a) 48 (b) 60
(a) 11 (b) 12
(c) 56 (d) 52
(c) 13 (d) 14
19. What is the number of triangles in figure ? 23. Find out the number of triangles in the given figure.

(a) 32 (b) 36 (a) 13 (b) 15


(c) 40 (d) 56
(c) 16 (d) 17

ANSWER KEY
1 (a) 5 (b) 9 (c) 13 (d) 17 (c) 21 (c)

2 (b) 6 (a) 10 (c) 14 (d) 18 (c) 22 (c)

3 (c) 7 (c) 11 (b 15 (a) 19 (d) 23 (c)

4 (d) 8 (a) 12 (d) 16 (c) 20 (d)


y
o
u
Visual Reasoning A-149

rs
m
a
h
b
o
o
b
.w
1. (a) All other figures can be rotated into each other. (In 7. (c)

o
A B C
each figure except fig. (a) the middle element is obtained

rd
by rotating the outer element through 90ºCW and the

p
re
inner element is obtained by rotating the middle element Q

s
P D

s
through 90ºCW) .

.c
R S
2. (b) Each one of the figures except fig. (b), contains – one

o
m
complete square, one cup-shaped element having side,
O N M G F E
one ‘L’-shaped element having two sides and one
T
straight line. L H
3. (c) In all other figures, the square has two line segments
inside and one line segment outside.
K I
4. (d) 1, 8, 9 are figures bisected by a straight line. J
4, 6, 7 are figures having an extended arm.
The simplest triangles are:
2, 3, 5 are figures intersected by a line. PNO; PNM; MPQ;
Thus the given figure containing nine figures may be MQR; AQP; AQR;
divided into three pairs : (1, 8, 9), (4, 6, 7) and (2, 3, 5). BRA; BRC; SRC;
Hence the answer is (d). SCD; SGR; SGD;
5. (b) A DFG; DFE; TLM;
TJK; TLK; TIH;
The triangles composed of two components are:
D E
PON; PMA; APR;
G RAM; RAC; RGC;
F H
DGC; DGE; MPR;
J K
GRD; DGE; TMK;
I L
TKI; TIG
The triangles composed of four components are:
B M O C
AMO; AMC; CAG;
N
CGE; MKI; GIK;
The triangles are: Other triangles are : SPI; DQK
ABC; ADE; AFH; AIL; Total number of triangles
DFG; DIK; DBO; GDE; 18 + 14 + 6 + 2 = 40
EGH; EJL; EMC; FIJ; 8. (a) A B
FBN; JFG; GJK; KGH;
HKL; HNC; NFH; GMO;
IBM; MIJ; JMN; NJK; H P I
KNO; OKL; LOC;
L Q M
6. (a) A E B E T U V W
D C
I
H F O R N
K S J

C G D
F G
The triangles are:
AIH; AIE; EIB; BFI; Squares are:
IHC; IGC; IGD; DFI; ABCD, DEFG, HIJK, LMNO,
IAB; IBD; ICD; IAC; HPDT, TDSK, PIWD, DWJS,
LQDU, UDRO, QMVD, DVNR.
BAC; ACD; BDC; BDA;
y
o
u
A-150 Visual Reasoning

rs
m
9. (c) FDE, ACD, ABD,
A B

a
G FBD, ABC, BCD,

h
J D, BLD, BJD,

b
I
F C

o
JCD, JKD, LDJ, LCD,

o
b
H LKD, HDC, KDC, EDC,

.w
E D
HKC, EKC, AEC, EHC,

o
The triangles are: AEH, AGH, AIH, AGI.

rd
FAE; FAI; FIE; CBD; AFB, AGF, FBG.

p
re
CBJ; CJD; AIJ; BJI; Total 28 triangles.

s
15. (a) The rectangles are —

s
BJA; AIB; IED; JDE;

.c
JDI; IEJ; GAB; GAI; ABKJ, BCDK, JLPI, LMNP,

o
m
MDEN, IOGH, OEFG, ACDJ,
GJI; GJB; HJI; HDE;
JMNI, LDEP, JDEI, IERH, ACFH
HEI; HJD; AJF; EFJ;
BCI; CDI; IBD; JEA; H
G
F
10. (c) We can label the figure as shown.
I P N
A B C D E
O

F J D
G W K M
E H
A C
B
I J K L
16. (c) According to the figure there are 12 semicircles.
The simplest parallelogram are ABFE, BCGF, CDHG,
EFJI, FGKJ and GHLK. These are 6 in number.
The parallelograms composed of two components
each, are ACGE, BDHF, EGKI, FHLJ, ABJI, BCKJ and
CDLK. Thus, there are 7 such parallelograms. The
12 11
parallelogram composed of four components each are 10 9
ACKI and BDLJ i.e. 2 in number. There is only one 1 35 7
parallelogram composed of six components, namely, 2 64 8
ADLI. Thus, there are 6 + 7 + 2 + 1 = 16 parallelograms
in the figure.
17. (c) The figure may be labelled as shown.
11. (b) There are 10 cubes.
12. (d) We have three squares with vertical and horizontal W X
sides. Each such square has 12 + 22 = 5 squares in it. O P
E B F
Thus there are 15 such squares. V Y
I J
In addition, we have two obliquely placed squares. N Q
A
Hence total no. of squares = 17 U M C
R
L K
13. (d) Each small square is bisected by its diagonals to give C1 Z
H D G
4 triangles of half the size of the square and 4 triangles T S
of 1/4th the size of the square. B1 A1
Thus there are 8 × 4 = 32 triangles in the four squares.
Then there are six triangles as shown in the adjoining The squares composed of two components each are
figure. Total no. of triangles = 32 + 6 = 38 BJMI, CKMJ, DLMK and AIML i.e. 4 in number.
The squares composed of three components each are
EBMA, BFCM, MCGD and AMDH i.e. 4 in number.
14. (d) F A E
The squares composed of four components each are
VWBA, XYCB, ZA1DC and B1C1AD i.e. 4 in number.
G The squares composed of seven components each
I H are NOJL, PQKI, RSLJ and TUIK i.e. 4 in number.
L There is only one square i.e. ABCD composed of eight
B J K C components.
There is only one square i.e. EFGH composed of twelve
components.
Total number of squares in the figure
= 4 + 4 + 4 + 4 + 1 + 1 = 18.
D 18. (c) The figure may be labelled as shown.
y
o
u
Visual Reasoning A-151

rs
m
triangle 1, 15, 16.
8

a
9 In the triangle with vertices 1, 6, 14 there are 3 triangles

h
3

b
and the pattern is repeated for the triangle with vertices
4

o
2
1, 2, 16.

o
13 1 10

b
7 In the parallelogram, there are 4 triangles each half the

.w
5
6 size, 4 triangles of quarter size and 6 triangles each

o
12

rd
11 made up of two small triangles.

p
There are 13 circles in the given figure. This is clear Finally, there two triangles with vertices 1, 10, 12 and 1,

re
14, 16 respectively

s
from the adjoining figure in which the centres of all the

s
Hence total no. of triangles.

.c
circles in the given figure have been numbered from

o
1 to 13. = 10 + 2 (6 + 8) + 2 × 3 + (4 + 4 + 6) + 2 = 60

m
19. (d) With vertex no. 1 we have four triangles on one side of
the diagonal and four triangles on the other side. 21. (c) E
C
1 2 D F G
O
5 6

9 A H B

8 7 There 14 triangles in the given figure. These are


AHO, ACB, BHO, BAD, ABE, ABD, BAF, ABG, AOF,
AFD, BOG, BGC, ADO and BOC.
4 3
Thus from four vertices. We have in all 4 × 8 = 32 22. (c) A E D F B
triangles. I K
Next consider square (5 – 6 – 7 – 8).
There are four triangles from each vertex. G J H
Thus we have another 4 × 4 = 16 triangles.
Lastly, we have oblique triangles with vertex 9 such as
(9 – 6 – 7), (9 – 2 – 3) and so on.
There are 8 such triangles.
Hence, total no. of triangles = 32 + 16 + 8 = 56 C
1
(5) (6) (7) (8) ABC, ADC, DBC, AEG, BFH
(1) EIG, EID, IGJ, IDJ,
DKJ, DFK, KJH, KFH
(2) EDG, DJG, EGJ, DJG
(3)
DFJ, FHJ, DHJ, DFH
ADG, DGH, DBH, JEF
(4) GJC, HJC, GHC = 28 Triangles
28 × 2 = 56 Triangles.
23. (c) A
3
20. (d) Within the triangle with vertices 1, 2, 6, there are
4 + 3 + 2 + 1 = 10 triangles. In the triangle with vertices
B C
1, 2, 14 there are 6 triangles.
In the triangle with vertices 1, 14, 15 there are 8 E
triangles. D F
1

M N

3 4 5 6 L
2 G H I J K
7 8
ABC, BDE, BCE, CEF, ADF,
9 13 DGH, DEH, MHI, EMN, NKJ,
10 12
11 FKE, EHK, AGL, FKL, CHL, BGK
14 15 16 Number of triangles = 16
This pattern is repeated for triangle 1, 6, 16 and for
y
o
u
A-152 Mirror and Water Images

rs
m
a
CHAPTER
MIRROR AND

h
b
22

o
o
b
.w
WATER IMAGES

o
rd
p
re
s
s
Mirror Images

.c
I. Mirror Images of Capital Letters

o
m
In this category questions are based on the criteria that a few
figures are given and you have to find out which one is the exact A N
image of the given figure in a mirror placed in front of it. This B O
image formation is based on the principle of ‘lateral inversion’
C P
which implies that size of the image is equal to the size of the
object but both sides are interchanged. The left portion of the D Q
object is seen on the right side and right portion of the object is E R
seen on the left side. For example, mirror image of ABC = F S
Note : There are ‘11’ letters in English Alphabet which have G T
identical mirror images: A, H, I, M, O, T, U, V, W, X, Y. H U
Characteristics of Reflection by plane mirror I V
J W
1. Perpendicular distance of object from mirror = Perpendicular
distance of image from mirror. K X
2. The image is laterally inverted. L Y
M Z

II Mirror Images of Small Letters


a n
b o
c p
d q
e r
f s
g t
h u
i v
Object Image j w
3. The line joining the object point with its image is normal to k x
the reflecting surface. l y
4. The size of the image is the same as that of the object. m z

Mirror-image of certain words are given below :


III. Mirror Images of Numbers
(1) FUN :
(2) GOLKONDA : 0 6
Mirror-image of certain combinations of alphabets and
1 7
numbers are given below :
2 8
(1) BMC49JN2317 : 3 9
(2) 15bg82XQh : 4 10
5
y
o
u
Mirror and Water Images A-153

rs
m
IV. Mirror Images of Clock: therefore the mirror image will start from the mirror images

a
There are certain questions in which the position of the of R, i.e.; .Thus the mirror image for water is

h
b
hour-hand and the minute-hand of a clock as seen in a

o
o
mirror are given. On the basis of the time indicated by the

b
mirror-image of the clock we have to detect the actual time WAT E R

.w
in the clock. In the solution of such questions we use the

o
Thus option (d) is the correct answer.

rd
fact that if an object A is the mirror-image of another object

p
B then B is the mirror-image of A. Water Image

re
Time of image in plane mirror The reflection of an object as seen in water is called its water

s
s
(a) Real time = XH, Image time = 12H – XH (H = hours) image. It is the inverted image obtained by turning the object

.c
o
(b) Real time = XHYM, Image time = 11H60M – XHYM upside down.

m
(M = minutes) Water-images of capital letters
(c) Real time = XHYMZS, Image time = 11H59M60S – XHYMZS
(S = seconds) Letters A B C D E F G H I J K L M
(d) if XHYMZS> 11H59M60S, image time = 23H59M60S – XHYMZS Water-image
Letters N O P Q R S T U V W X Y Z
––––––––––––––––– Shortcut Method –––––––––––––– Water-image
Whenever you have to solve a mirror image question, imagine a
mirror placed in front of the object and then try to find its inverted
Water-images of small letters
image. The portion of the object that is near the mirror will now be Letters a b c d e f g h i j k l m
the portion of the image near to the mirror in the inverted form. Water-image
EXAMPLE 1. Letters n o p q r s t u v w x y z
By looking in a mirror, it appears that it is 6 : 30 in the clock. Water-image
What is the real time ? Water-images of numbers
(a) 6 : 30 (b) 5 : 30
(c) 6 : 00 (d) 4 : 30 Letters 0 1 2 3 4 5 6 7 8 9
Sol. (b) Water-image
Note :
1. The letters whose water-images are identical to the
letter itself are : C, D, E, H, I, K, O, X
Time = 6 : 30 Time = 5 : 30 2. Certain words which have water-images identical to
the word itself are :
KICK, KID, CHIDE, HIKE, CODE, CHICK
(Fig A) (Fig B)
Clearly, fig (A) shows the time (6 : 30) in the clock as it –––––––––––––––– Shortcut Method –––––––––––––––
appears in a mirror. Then its mirror-image i.e. Fig (B) shows Whenever we have to analyze the water image of an object,
the actual time in the clock i.e. 5 : 30. You can solve it quickly imagine a mirror or a surface that forms an image just under the
if you remember that the sum of actual time and image time is given object. The portion of the object that is near the water
always 12 hours. surface will be inverted but will be near the water surface in the
image as well.
EXAMPLE 2.
Find the correct option for the mirror image for the following EXAMPLE 3.
word: Find the correct option for the water images below:
STORE
water surface
WAT E R ? ?
(a) (b)
(c) (d)
mirror Sol. (d) In case of water image, the water reflection will usually
surface be formed under the object / word.
(a) (b) In this case, the water image of the word will be an outcome of the
(c) (d) water images of each of the letters like, the water images of S is
Sol. (d) We have to find the correct mirror image for the word , T is , O is , R is and E is . Thus the water image of
‘WATER’ for which we need to find the mirror image for theword ‘STORE’ is ‘ .’
each letter separately and then arrange it, like the mirror STORE
image for the letters W is W, A is A, T is T, E is and R is .
Since, the word ends with R, i.e., where the mirror is placed,
y
o
u
A-154 Mirror and Water Images

rs
m
EXAMPLE 4. –––––––––––––––– Shortcut Method –––––––––––––––

a
h
Find the correct option for the water image below: (i) While solving a question, try eliminating some options and

b
solving the questions will become easier.

o
o
1 6 8 9 2 To eliminate options, keep in mind the pattern used in the

b
object (given diagram whose image is to be formed) as well

.w
? as the position of mirror or water such that the portion of

o
the object near to the mirror / water will produce the same

rd
(a) (b)
portion near the mirror / water in an inverted form.

p
(c) 5 (d)

re
(ii) Images are images, be it water or mirror, in both the cases an

s
Sol. (b) The water image of ‘1’ is , ‘6’ is ‘ ’, ‘8’ is , ‘9’ is ‘ ’ inverted image of the alphabets / numerals / clocks / any

s
.c
and ‘2’ is ‘ ’. other object are formed by inverting the object. Inverting of

o
the object solely depends upon the position of mirror or

m
Thus, the water image of 1 6 8 9 2 is
water surface w.r.t. the object.

DIRECTIONS (Qs. 1-13) : In each of the following questions,


choose the correct mirror-image of the Fig. (X) from amongst
5.
the four alternatives (a), (b), (c) and (d) given along with it.
X (a) (b) (c) (d)
6.

1.

X (a) (b) (c) (d)


(X)

7.

(a) (b) (c) (d)

(a) (b)
2.

(X) (c) (d)

(a) (b) (c) (d) 8.

3.

(X) (a) (b) (c) (d) (a) (b)

4.
(c) (d)
(X) (a) (b) (c) (d)
y
o
u
Mirror and Water Images A-155

rs
m
9. K OH L I

a
h
(a) (b)

b
(a) (b)

o
o
(c) (d)

b
.w
M
10.

o
rd
(c) (d)

p
re
s
N

s
.c
o
DIRECTIONS (Qs. 15-22) : In each of the following questions,

m
choose the correct water image of the figure (X) from amongst
the four alternatives (a), (b), (c), (d) given alongwith it.
(a) (b) (c) (d) 15.
absence
?
11. (a) (b)
(c) (d)
16. ZEBRA
?
(a) (b)
(d) (d)
17.

(a) (b) (c) (d)


(X)
12. M
JACK
(a) (b)
N

(c) (d)

(a) (b) (c) (d)


18.
M
13.
APPROACH
(X)
N
(a) (b)

(c) (d)
(a) (b) (c) (d)
14. The following figure is rotated in anticlockwise direction 19.
through 90° after its mirror image is obtained. Select the
correct mirror image from four alternatives given. (X)

(a) (b)

(d) (d)
y
o
u
A-156 Mirror and Water Images

rs
m
20.

a
h
b
o
o
22.

b
.w
X (a) (b) (c) (d)

o
rd
p
re
21.

s
s
.c
(a) (b)

o
m
(a) (b)

(c) (d)

(c) (d)

ANSWER KEY
1 (c) 5 (d) 9 (b) 13 (c) 17 (b) 21 (a)

2 (c) 6 (a) 10 (c) 14 (c) 18 (c) 22 (d)

3 (a) 7 (d) 11 (b) 15 (a) 19 (a)

4 (c) 8 (a) 12 (c) 16 (c) 20 (a)


y
o
u
Mirror and Water Images A-157

rs
m
a
h
b
o
o
b
.w
o
1. (c) Assume that the mirror is placed on the right hand 15. (a) The water image of ‘a’ is ‘ ’, ‘b’ is ‘ ’, ‘S’ is ‘ ’, ‘e’ is

rd
side of the given object, unless mentioned or drawn ‘ ’, ‘n’ is ‘ ’, ‘c’ is ‘ ’ and ‘e’ is ‘ ’.

p
near the object.

re
16. (c) The water image of ‘Z’ is ‘ ’, ‘E’ is ‘ ’, ‘B’ is , ‘R’ is

s
‘ ’ and ‘A’ is ‘ ’

s
.c
o
m
mirror 17. (b)

13. (c) APPROACH

14. (c)

21. (a)
0
90° anticlockwise
Question figure

22. (d)

Mirror image
y
o
u
A-158 Embedded Figure

rs
m
a
CHAPTER

h
b
23

o
EMBEDDED FIGURE

o
b
.w
o
rd
p
re
s
s
.c
A figure (X) is said to be embedded in a figure Y, if figure Y contains

o
m
figure (X) as its part. Thus problems on embedded figures contain
a figure (X) followed by four complex figures in such a way that
fig (X) is embedded in one of these. The figure containing the
figure (X) is your answer.
Hence, the answer is (c)
DIRECTIONS : In each of the following examples, fig (X) is
embedded in any one of the four alternative figures (a), (b), (c) EXAMPLE 2.
or (d). Find the alternative which contains fig. (X) as its part.
EXAMPLE 1.

(X)

(X)

(a) (b) (c) (d)


Sol. (b) Clearly, fig. (X) is embedded fig. (b) as shown below :

(a) (b) (c) (d)


Sol. (c) On close observation, we find that fig. (X) is embedded
in fig. (c) as shown below :
Hence, the answer is (b)

DIRECTIONS (Qs. 1-20) : In each of the following questions, you


are given a figure (X) followed by four alternative figures (a), (b), 2.
(c) and (d) such that fig. (X) is embeded in one of them. Trace out
the alternative figure which contains fig. (X) as its part.
1.

(X)
(X)

(a) (b) (c) (d) (a) (b) (c) (d)


y
o
u
Embedded Figure A-159

rs
m
3. 8.

a
h
b
o
o
b
(X) (a) (b) (c) (d)

.w
(X)

o
rd
9.

p
re
s
s
(X) (a) (b) (c) (d)

.c
(a) (b) (c) (d)

o
10.

m
4.

(X) (a) (b) (c) (d)

(X) 11.

(x)

(a) (b) (c) (d)


5.

(a) (b) (c) (d)


12.
(X)

(X)

(a) (b) (c) (d) (a) (b) (c) (d)


6. 13.

(X)
(X)

(a) (b) (c) (d) (a) (b) (d)


(c)
7.
14.

(X) (X)

(a) (b) (c) (d)


(a) (b) (c) (d)
y
o
u
A-160 Embedded Figure

rs
m
15.
18.

a
h
b
o
o
b
.w
(X) (X)

o
rd
p
re
s
s
.c
o
(a) (b) (c) (d)

m
(a) (b) (c) (d)
19.
16.

(X)
(X)

(a) (b) (c) (d)


(a) (b) (c) (d)
20.
17.

(X)
(X)

(a) (b) (c) (d)


(a) (b) (c) (d)

ANSWER KEY
1 (a) 5 (b) 9 (b) 13 (d) 17 (d)

2 (d) 6 (d) 10 (c) 14 (c) 18 (b)

3 (d) 7 (b) 11 (b) 15 (a) 19 (d)

4 (b) 8 (a) 12 (d) 16 (b) 20 (a)


y
o
u
Embedded Figure A-161

rs
m
a
h
b
o
o
b
.w
o
7. (b) 8. (a)

rd
1. (a) 2. (d)

p
re
s
s
.c
o
m
9. (b) 10. (c)
3. (d) 4. (b)

11. (b)

5. (b) 6. (d)
y
o
u
A-162 Non-Verbal Series

rs
m
a
CHAPTER

h
NON-VERBAL

b
24

o
o
b
.w
SERIES

o
rd
p
re
s
s
.c
The word “series” is defined as anything that follows or Answer Figure

o
forms a specific pattern or is in continuation of a given pattern

m
or sequence.
In this type of nonverbal test, two sets of figures pose the
problem. The sets are called problem Figures and Answer (a) (b) (c) (d)
Figures. Each problem figure changes in design from the Sol. (d) Two line segments are added in A to obtain B and one
preceding one. The answer figure set contains 4 figures line segment is added in B to obtain C. This process is
marked (a), (b), (c), (d). You are required to choose the correct repeated again to obtain D. Hence, answer figure d
answer figure which would best continue the series. continues the series.
TYPE I. TYPE III.
In these questions the items in the diagrams either increase
A definite relationship between elements in given figures. or decrease in number.
EXAMPLE 1.
EXAMPLE 3.
Study the problem figures marked (A), (B) and (C) carefully
Problem Figures
and try to establish the relationship between them. From the
answer figures marked a, b, c and d, pick out the figure
which most appropriately completes the series.
Problem Figures

(A) (B) (C)


Answer Figures

(A) (B) (C)


Answer Figures

(a) (b) (c) (d)


Sol. (c) The small circles are decreasing consecutively and the
black dots are increasing.
TYPE IV
Deletion of Elements : In these type of questions, each
(a) (b) (c) (d) figure is obtained by either sustaining the element of
Sol. (d) Note the direction of arrow which changes alternately. preceding figure as it is or deleting a part of an element or
The dots are also changing alternately. Hence we are one element or more than one element of the preceding
figure in a systematic way.
looking for a figure in which the arrow points down
and the dots and positioned as in figure (b). EXAMPLE 4.
TYPEII. Problem Figure
Additions of Elements : In these type of questions, each
figure is obtained by either sustaining the element of
preceding figure as it is or adding a part of element or one
element or more than one element of the preceding figure in
a systematic way.
(A) (B) (C) (D) (E)
EXAMPLE 2.
Answer Figure
Problem Figure

(A) 2
(B) (C) (D) (a) (b) (c) (d)
y
o
u
Non-Verbal Series A-163

rs
Sol. (a) TYPE VI.

m
a
TYPE V Replacement of Elements : In these type of questions, each

h
b
The qualitative characteristic of various elements in the figure is obtained by either sustaining the element of

o
diagrams change to complete the series. preceding figure as it is or replacing a part of element or one

o
b
element or more than one element by a new element of the

.w
Rotation Type : The various elements in the diagrams move
preceding figure in a systematic way.
in a specific manner. They may rotate in clockwise or anti-

o
rd
clockwise direction. EXAMPLE 6.

p
Problem Figure

re
EXAMPLE 5.

s
? * X

s
*

.c
Problem Figures
= X ?

o
*

m
? X = * ? C
+ +
2
(A) (B) (C) (D) (E)
Answer figure
+
* # C # * C
(A) (B) (C)
Answer Figures # *
(a) (b) ( c) (d)
+ + + Sol. (c) The elements positioned at north-east (NE) corners
disappear from the odd-numbered figures. The elements
+ positioned at the south-west (SW) corners disappear from
the even-numbered figures. Therefore * should not appear
in the answer figure. Hence (a), (b) and (d) cannot be the
(a) (b) (c) (d) answers. Also new elements are introduced at the NE corners
Sol. (d) The sign of plus is rotating clockwise. The pin changes in even-numbered figures. Therefore answer figure (c)
direction alternately. continues the given series.

DIRECTIONS (Qs.1-20) : Each of the following questions Answer Figures


consists of five problem figures marked A, B, C, D and E. From
the five answer figures marked (a), (b), (c) and (d) select a
figure which will continue the series established by the five
problem figures.
1. Problem Figures
(a) (b) (c) (d)
3. Problem Figures

(A) (B) (C) (D) (E)


Answer Figures

(A) (B) (C) (D) (E)


Answer Figures
(a) (b) (c) (d)
2. Problem Figures

(A) (B) (C) (D) (E) (a) (b) (c) (d)


y
o
u
A-164 Non-Verbal Series

rs
4. Problem Figures Answer Figures

m
a
h
b
o
o
b
.w
(A) (B) (C) (D) (E)

o
(a) (b) (c) (d)

rd
Answer Figures 9. Problem Figures

p
re
s
s
.c
o
m
(a) (b) (c) (d) (A) (B) (C) (D) (E)
5. Problem Figures Answer Figures

(A) (B) (C) (D) (E) (a) (b) (c) (d)


Answer Figures 10. Problem Figures

(a) (b) (c) (d)


(A) (B) (C) (D) (E)
6. Problem Figures
Answer Figures

(A) (B) (C) (D) (E)


Answer Figures (a) (b) (c) (d)
11. Question Figures :

(a) (b) (c) (d) Answer Figures :


7. Problem Figures

(a) (b) (c) (d)


(A) (B) (C) (D) (E) 12. Problem Figures
Answer Figures
x

xx x

(a) (b) (c) (d) (A) (B) (C) (D) (E)


8. Problem Figures Answer Figures
s

(A) (B) (C) (D) (E) (a) (b) (c) (d)


y
o
u
Non-Verbal Series A-165

rs
13. Problem Figures 17. Problem Figures

m
a
h
? = ? x = x

b
o
? = x

o
b
x = x ? = ?

.w
o
(A) (B) (C) (D) (E) (A) (B) (C) (D) (E)

rd
Answer Figures Answer Figures

p
re
= x

s
s
.c
? ? ? =

o
x = x = x ?

m
(a) (b) (c) (d) (a) (b) (c) (d)
14. Problem Figures 18. Problem Figures
=

(A) (B) (C) (D) (E) (A) (B) (C) (D) (E)
Answer Figures Answer Figures

+
(a) (b) (c) (d) (a) (b) (c) (d)
15. Problem Figures 19. Problem Figures

× ×
= = ×= = × × = × S S×
× × ×
= = =
(A) (B) (C) (D) (E) (A) (B) (C) (D) (E)
Answer Figures Answer Figures
× =
= =
× S C S× C S CS×
×
= ×
(a) (b) (c) (d) (a) (b) (c) (d)
16. Problem Figures 20. Problem Figures

(A) (B) (C) (D) (E) (A) (B) (C) (D)


Answer Figures Answer Figures

(a) (b) (c) (d) (a) (b) (c) (d)

ANSWER KEY
1 (d) 3 (c) 5 (b) 7 (d) 9 (c) 11 (c) 13 (b) 15 (c) 17 (a) 19 (a)
2 (d) 4 (d) 6 (d) 8 (c) 10 (c) 12 (c) 14 (a) 16 (d) 18 (d) 20 (d)
y
o
u
A-166 Non-Verbal Series

rs
m
a
h
b
o
o
b
.w
1. (d) Five line segments are added in each step to complete the 13. (b) In each step, the elements move in the sequence

o
squares in an ACW direction.

rd
2. (d) One of the pins gets inverted in each step. The pins gets

p
inverted sequentially from right to left.

re
s
3. (c) In one step, the existing element enlarges and a new element

s
14. (a) In each step, the upper elements is lost; the middle

.c
appears inside this element. In the next step, the outer element

o
is lost element reduces in size and becomes the upper

m
4. (d) In each step, both the elements move one space (each space element; the lower element enlarges and becomes the
is equal to half-a-side of the square boundary) downwards. middle element and a new element appears at the lower
Once any of the two elements reaches the lowermost position, position.
then in the next step, it reaches the uppermost position in 15. (c) All the elements together move one space to the right
the next column to the right. in each step and once they reach the rightmost
5. (b) Similar figure repeats in every third step and each time a position, then in the next step, they move to the leftmost
figure reappears it gets vertically inverted. position . Also, in the first step, the first (uppermost)
6. (d) In each step, all the existing elements move to the adjacent and the third elements interchange positions; in the
side (of the square boundary) in a CW direction. The number second step, the second and the fourth elements inter
of black circles decreases by one in first, third, fifth step and change positions and in the third step, none of the
the number of arrows increases by one in second, fourth, elements interchange positions. These three steps are
sixth steps. repeated to continue the series.
7. (d) In each step, the rectangle rotates through 90º; the square 16. (d) In the first step, the lowermost line segment is
rotates through 45º; the triangle gets vertically inverted and converted into a curve. In the second step, the second
all the elements move in the sequence given below. line segment also gets converted into a curve and the
existing curve is inverted. In each subsequent step, all
the elements (line segments and curve) move in the

8. (c) The total number of dots on the dice decreases by one in sequence
each subsequent figure.
9. (c) The ‘T’ and ‘ ’ gets inverted in each step while the symbol 17. (a) The central arrow rotates 90ºACW and 45ºCW
‘C’ gets laterally inverted in each step. The interchange of alternately. The half -arrow moves half- a - side of the
positions of symbol takes place in the following two square boundary in a CW direction and its head turns
sequences alternately. to the other side of the line in each step.

18. (d) The elements move in the sequences and

10. (c) Similar figure repeats in every second step and each time a
figure reappears, it gets rotated through 90ºACW.
alternately and in each step, the element at
11. (c) The series represents continous alphabets starting from
K. Hence, N is the right answer. the encircled position, gets vertically inverted.
12. (c) In the first step 1 to 2, the elements at the four ends 19. (a) In one step, the elements move in the sequence

move in the sequence while the circled


and the element at the encircled position,

element gets replaced by a new element. gets replaced by a new element. In the next step, the
In the next step (2 to 3), the elements move in the
elements move in the sequence . These two
sequence while the circled element gets
sequences get repeated alternately.
repalced by a new element. 20. (d) Similar figure repeats in every second step. Each time
the first figure reappears, it rotates 135ºACW and
Similar steps are followed in sequences 3 to 4 and 4 to moves along a diagonal (from lower-left to upper -right
5, so that we return to the first step in the sequence corner). Each time the second figure reappears, it
from 5 to the answer figure. rotates 135ºCW and moves along the other diagonal.
y
o
u
rs
SECTION - B : QUANTITATIVE APTITUDE

m
a
h
b
o
o
CHAPTER

b
.w
NUMBER SYSTEM &
1

o
rd
p
SIMPLIFICATION

re
s
s
.c
o
m
The ten symbols 0, 1, 2, 3, 4, 5, 6, 7, 8, 9 are called digits, which The number line : The number line is a straight line between
can represent any number. negative infinity on the left to positive infinity on the right.
Natural Numbers : These are the numbers (1, 2, 3, etc.) that are
used for counting. It is denoted by N. 4 3 2 1 0 1 2 3 4
There are infinite natural numbers and the smallest natural number Real Numbers : All numbers that can be represented on the
is one (1). number line are called real numbers.
Even numbers : Natural numbers which are divisible by 2 are It is denoted by R.
even numbers. It is denoted by E. R+ : Positive real numbers and
E = 2, 4, 6, 8,... R– : Negative real numbers.
Smallest even number is 2. There is no largest even number. Real numbers = Rational numbers + Irrational numbers.
Odd numbers : Natural numbers which are not divisible by 2 (a) Rational numbers : Any number that can be put in the form
are odd numbers.
p
It is denoted by O. of , where p and q are integers and q 0 , is called a
q
O = 1, 3, 5, 7, ...
Smallest odd number is 1. rational number.
There is no largest odd number. It is denoted by Q.
Based on divisibility, there could be two types of natural numbers : Every integer is a rational number.
Prime and Composite. Zero (0) is also a rational number. The smallest and largest
(a) Prime Numbers : Natural numbers which have exactly two rational numbers cannot be determined. Every fraction (and
factors, i.e., 1 and the number itself are called prime numbers. decimal fraction) is a rational number.
The lowest prime number is 2. p (Numerator)
2 is also the only even prime number. Q
q (Denominator)
(b) Composite Numbers : It is a natural number that has atleast
one divisor different from unity and itself. x y
If x and y are two rational numbers, then is also a
Every composite number can be factorised into its prime factors. 2
For Example : 24 = 2 × 2 × 2 × 3. Hence, 24 is a composite number. rational number and its value lies between the given two
The smallest composite number is 4. rational numbers x and y.
Whole Numbers : The natural numbers along with zero (0), form An infinite number of rational numbers can be determined
the system of whole numbers. between any two rational numbers.
It is denoted by W.
There is no largest whole number and Example 1 :
The smallest whole number is 0. Find three rational numbers between 3 and 5.
Solution :
Integers : The number system consisting of natural numbers,
their negative and zero is called integers. 3 5 8
1st rational number = 4
It is denoted by Z or I. 2 2
The smallest and the largest integers cannot be determined. 2nd rational number (i.e., between 3 and 4)
Remember 3 4 7
=
1 is neither prime nor composite. 2 2
1 is an odd integer. 3rd rational number (i.e., between 4 and 5)
0 is neither positive nor negative.
0 is an even integer. 4 5 9
2 is prime & even both. = .
All prime numbers (except 2) are odd. 2 2
y
o
u
B-2 Number System & Simplification

rs
(b) Improper fraction : If numerator is greater than or equal to

m
(b) Irrational numbers : The numbers which are not rational or

a
p its denominator, then it is a improper fraction.

h
which cannot be put in the form of , where p and q are

b
q 5 18 13

o
For example : , ,

o
2 7 13

b
integers and q 0, is called irrational number..

.w
NOTE :

o
It is denoted by Q ' or Qc .

rd
If in a fraction, its numerator and denominator are of equal

p
value then fraction is equal to unity i.e. 1.

re
2, 3, 5, 2 3,3 5, 3 3 are irrational numbers.

s
s
.c
(c) Mixed fraction : It consists of an integer and a proper
NOTE :

o
fraction.

m
(i) Every positive irrational number has a negative 1 2 5
For example : 1 , 3 , 7
irrational number corresponding to it. 2 3 9
(ii) 2 3 5 NOTE :
5 3 2 Mixed fraction can always be changed into improper fraction
and vice versa.
3 2 3 2 6
5 7 9 5 63 5 68
For example : 7 =
6 9 9 9 9
6 2 3
2 19 9 2 1 1 1
and 9 9
(iii) Some times, product of two irrational numbers is a 2 2 2 2
rational number.
(d) Equivalent fractions/Equal fractions : Fractions with same
For example : 2 2 2 2 2 value.

(2 3) (2 3) (2) 2 ( 3) 2 = 4 – 3 = 1 2 4 6 8 2
For example : , , , .
3 6 9 12 3
(e) Like fractions: Fractions with same denominators.
Both rational and irrational numbers can be represented on
number line. Thus real numbers is the set of the union of 2 3 9 11
For example : , , ,
rational and irrational numbers. 5 7 8 6
(f) Unlike fractions : Fractions with different denominators.
R Q Q'
Every real number is either rational or irrational. 2 4 9 9
For example : , , ,
5 7 8 2
Fraction : A fraction is a quantity which expresses a part of the NOTE :
whole.
Unlike fractions can be converted into like fractions.
Numerator 3
Fraction 4
Denominator For example : and
5 7
Example 2 :
3 7 21 4 5 20
Write a fraction whose numerator is 22 + 1 and denominator and
is 32 – 1. 5 7 35 7 5 35
Solution :
Numerator = 22 + 1 = 4 + 1 = 5 (g) Simple fraction : Numerator and denominator are integers.
Denominator = 32 – 1 = 9 – 1 = 8
3 2
Numerator 5 For example : and .
Fraction = 7 5
Denominator 8 (h) Complex fraction : Numerator or denominator or both are
fractional numbers.
TYPES OF FRACTIONS :
(a) Proper fraction : If numerator is less than its denominator, 2
1
then it is a proper fraction. 1 7
2 2
2 3, 3
For example : ,
2 6 5 2 2
For example : , 5
5 18 7 3
y
o
u
Number System & Simplification B-3

rs
(i) Decimal fraction : Denominator with the powers of 10.

m
NOTE :

a
h
2 9
For example :

b
(0.2), (0.09)
1, i 2 1, i3 i, i4 1

o
10 100 i

o
(j) Vulgar fraction : Denominators are not the power of 10.

b
.w
3 9 5 DIVISIBILITY RULES

o
For example : , , .

rd
7 2 193
Divisibility by 2 : A number is divisible by 2 if it’s unit digit is

p
Example 3 :

re
even or 0.

s
After doing 3/5 of the Biology homework on Monday night, Sanjay

s
Divisibility by 3 : A number is divisible by 3 if the sum of it’s

.c
did 1/3 of the remaining homework on Tuesday night. What
digits are divisible by 3.

o
fraction of the original homework would Sanjay have to do on

m
Wednesday night to complete the Biology assignment ? Divisibility by 4 : A number is divisible by 4 if the last 2 digits are
Solution : divisible by 4, or if the last two digits are 0’s.
Remaining homework on Monday night Divisibility by 5 : A number is divisible by 5 if it’s unit digit is
3 2 5 or 0.
=1– Divisibility by 6 : A number is divisible by 6 if it is simultaneously
5 5
divisible by 2 and 3.
1 2 2 Divisible by 7 : We use osculator (– 2) for divisibility test.
Work done on Tuesday night = of
3 5 15 99995 : 9999 – 2 × 5 = 9989
Remaining homework to complete the biology assignment 9989 : 998 – 2 × 9 = 980
2 2 6 2 4 980 : 98 – 2 × 0 = 98
=
5 15 15 15 Now 98 is divisible by 7, so 99995 is also divisible by 7.
Rounding off (Approximation) of Decimals : There are some Divisible by 11 : In a number, if difference of sum of digit at even
decimals in which numbers are found upto large number of decimal places and sum of digit at odd places is either 0 or multiple of 11,
places. then no. is divisible by 11.
For example : 3.4578, 21.358940789. For example, 12342 11
But many times we require decimal numbers upto a certain number Sum of even place digit = 2 + 4 = 6
of decimal places. Therefore, Sum of odd place digit = 1 + 3 + 2 = 6
If the digit of the decimal place is five or more than five, then the
Difference = 6 – 6 = 0
digit in the preceding decimal place is increased by one and if the
digit in the last place is less than five, then the digit in the 12342 is divisible by 11.
precedence place remains unchanged. Divisible by 13 : We use (+ 4) as osculator.
Example 4 : e.g., 876538 13
(a) Write 21.3751 upto two places of decimal. 876538: 8 × 4 + 3 = 35
(b) Write 3.27645 upto three places of decimal. 5 × 4 + 3 + 5 = 28
Solution :
8 × 4 + 2 + 6 = 40
(a) 21.3751 = 21.38
(b) 3.27645 = 3.276 0 × 4 + 4 + 7 = 11
Operations : The following operations of addition, subtraction, 1 × 4 + 1 + 8 = 13
multiplication and division are valid for real numbers. 13 is divisible by 13.
(a) Commutative property of addition : 876538 is also divisible by 13.
a+b=b+a Divisible by 17 : We use (– 5) as osculator.
(b) Associative property of addition : e.g., 294678: 29467 – 5 × 8 = 29427
(a + b) + c = a + (b + c) 27427: 2942 – 5 × 7 = 2907
(c) Commutative property of multiplication: 2907: 290 – 5 × 7 = 255
a*b=b*a 255: 25 – 5 × 5 = 0
(d) Associative property of multiplication : 294678 is completely divisible by 17.
(a * b) * c = a * (b * c) Divisible by 19 : We use (+ 2) as osculator.
(e) Distributive property of multiplication with respect to e.g: 149264: 4 × 2 + 6 = 14
addition : 4 × 2 + 1 + 2 = 11
(a + b) * c = a * c + b * c 1 × 2 + 1 + 9 = 12
Complex numbers : A number of the form a + bi, where a and b are real
2×2+1+4=9
number and i 1 (imaginary number) is called a complex number.. 9 × 2 + 1 = 19
It is denoted by C. 19 is divisible by 19
For Example : 5i (a = 0 and b = 5), 5 3i(a 5 and b = 3) 149264 is divisible by 19.
y
o
u
B-4 Number System & Simplification

rs
m
Divisibility by a Composite number: Two different numbers x and y when divided by a certain

a
A number is divisible by a given composite number if it is divisible divisor D leave remainder r 1 and r2 respectively. When the

h
b
by all factors of composite number. sum of them is divided by the same divisor, the remainder is

o
Example 5: r3. Then,

o
b
Is 2331024 divisible by 12

.w
Solution : divisor D r1 r2 r3

o
12 = 4 × 3 Method to find the number of different divisors (or factors)

rd
2331024 is divisible by 3 as (2+3+3+1+2+4) = 15 is divisible by 3 (including 1 and itself) of any composite number N :

p
re
2331024 is also divisible by 4 because last two digits (24) is STEP I : Express N as a product of prime numbers as

s
s
divisible by 4 N = xa × yb × zc .........

.c
STEP II : Number of different divisors (including 1 and itself)

o
Therefore 2331024 is divisible by 12

m
Example 6 : = (a + 1)(b + 1)(c + 1) ........
What is the value of M and N respectively if M39048458N is Example 8 :
divisible by 8 and 11, where M and N are single digit integers? Find the number of different divisors of 50, besides unity
and the number itself.
Solution :
Solution :
A number is divisible by 8 if the number formed by the last
If you solve this problem without knowing the rule, you will take
three digits is divisible by 8. the numbers in succession and check the divisibility. In doing so,
i.e., 58N is divisible by 8. you may miss some numbers. It will also take more time.
Clearly,N = 4 Different divisors of 50 are : 1, 2, 5, 10, 25, 50
Again, a number is divisible by 11 if the difference between If we exclude 1 and 50, the number of divisors will be 4.
the sum of digits at even places and sum of digits at the odd By rule : 50 = 2 × 5 × 5 = 21 × 52
places is either 0 or is divisible by 11. the number of total divisors = (1 + 1) × (2 + 1) = 2 × 3 = 6
or, the number of divisors excluding 1 and 50 = 6 – 2 = 4
i.e. (M 9 4 4 8) (3 0 8 5 N)
Example 9 :
= M 25 (16 N)
A certain number when divided by 899 leaves the remainder
= M N 9 must be zero or it must be divisible by 11
63. Find the remainder when the same number is divided
i.e. M N 2 by 29.
M 2 4 6 Solution :
Hence, M 6, N 4 Number = 899Q + 63, where Q is quotient
Example 7 : = 31 × 29 Q + (58 + 5) = 29 [ 31Q + 2] + 5
The highest power of 9 dividing 99! completely, is: Remainder = 5
Solution : Counting Number of Zeros
(c) 9 = 3 × 3 = 32 Sometimes we come across problems in which we have to
Highest power of 3 in 99! count number of zeros at the end of factorial of any num-
bers. for example -
99 99 99 99 Number of zeros at the end of 10!
3 32
33 34 10! = 10 × 9 × 8 × 7 × 6 × 5 × 4 × 3 × 2 ×1
Here basically we have to count number of fives, because
= 33 + 11 + 3 + 1 = 48
multiplication of five by any even number will result in 0 at
But we have 32
the end of final product. In 10! we have 2 fives thus total
48 number of zeros are 2.
highest power of 9 in 99! 24
2 Short cut :-
DIVISION ALGORITHM : Counting number of zeros at the end of n! value will be
Dividend = (Divisor × Quotient) + Remainder
where, Dividend = The number which is being divided n n n n
Divisor = The number which performs the division process 5 52 53 54
Quotient = Greatest possible integer as a result of division The integral value of this number will be the total number of
Remainder = Rest part of dividend which cannot be further divided zeros.
by the divisor. Example 10 :
Complete remainder : A complete remainder is the remainder Number of zeros at the end of 10!
obtained by a number by the method of successive division.
10 10
Complete remainder = [I divisor × II remainder] + I remainder Solution : Integral value
5 52
C.R. d1r2 r1 =2+0
C.R. d1d 2 r3 d1r2 r1 So, number of zeros in 10! = 2.
y
o
u
Number System & Simplification B-5

rs
m
greatest integers less than or equal to x and is a natural
10
number such that Pr < n.

a
Note:- Here 2 is less than 1 so will not count it.

h
5 Example 14 :

b
o
Example 11 : Find highest power of 7n in 50!

o
b
Number of zeros at the end of 100! Solution :

.w
100 100 100 The highest power 7 in 50!

o
Solution :

rd
5 52 53 50 50
= 7 1 8

p
integral value will be 72

re
7
= 20 + 4 = 24 zeros.

s
Example 15 :

s
.c
Example 12 : Find highest power 15 in 100!

o
m
Number of zeros at the end of 126! Solution :
Solution : Here given number 15 is not a prime number so first
126 126 126 convert 15 as product of Primes 15 = 3 × 5 therefore we
will find the highest power of 3 and 5 in 100!
5 52 53 Highest power of 3 in 100!
integral value will be
100 100 100 100
= 25 + 5 + 1 = 31 zeros. 2
3 3 33 34
Example 13 : = 33 + 11 + 3 + 1 = 48
Number of zeros at the end of 90! Highest power of 5 in 100!
Solution : 100 100
20 4 24
90 90 90 5 52
= 18 + 3 = 21 zeros
5 52 53 So 100! contains (3)48 × (5)24. Hence it contains 24
Power of a number contained in a factorial pairs of 3 and 5. Therefore, required power of 15 is 24, which
Highest power of a prime number P in N! is actually the power of the largest prime factor 5 of 15,
because power of largest prime factor is away equal to or
N N N N
= + 2 + 3 + - - - + r , where [x] denotes the less than the other prime factor of any number.
P P P P

TO FIND THE LAST DIGIT OR DIGITAT THE UNIT’S PLACE OF an.


(i) If the last digit or digit at the unit’s place of a is 1, 5 or 6, whatever be the value of n, it will have the same digit at unit’s place, i.e.,

(.....1)n (........1)
(.....5)n (........5)
(.....6) n (........6)

(ii) If the last digit or digit at the units place of a is 2, 3, 5, 7 or 8, then the last digit of a n depends upon the value of n and
follows a repeating pattern in terms of 4 as given below :
n last digit of (....2)n last digit of (....3)n last digit of (....7)n last digit of (....8)n
4x+1 2 3 7 8
4x+2 4 9 9 4
4x+3 8 7 3 2
4x 6 1 1 6

(iii)
If the last digit or digit at the unit’s place of a is either Solution : Here, 2, 4, 8, 6 will repeat after every four interval till
4 or 9, then the last digit of an depends upon the
value of n and follows repeating pattern in terms of 2 320 next digit will be 2, 4, 8 , so unit digit of 2323 will be 8.
as given below.
Example 17 :
n last digit of (....4)n last digit of (....9)n Find unit digit of 133133.
2x 6 1
Solution :
2x + 1 4 9
Cycle of 3 is 3, 9, 7, 1 which repeats after every fourth inter-
Example 16 : val will 133132, so next unit digit will be 3.
Find unit digit of 2323.
y
o
u
B-6 Number System & Simplification

rs
Example 18 : On dividing 15 by 7, we get 1 as remainder.

m
a
Find unit digit of 96363 ×7373. On dividing 17 by 7, we get 3 as remainder.

h
On dividing 19 by 7, we get 5 as remainder.

b
Solution : Unit digit of 96363 = 7

o
15

o
Unit digit of 7373 = 3

b
And combined remainder will be equal to remainder of i.e. 1.
So unit digit of 96363 × 7373 = 7 × 3 = 21. 7

.w
o
i.e. 1. Example 24 :

rd
Example 19 :

p
Find the remainder of expression 19 20 21

re
Find unit digit of 1717 × 2727 × 3737.

s
9

s
Solution : Unit digit of 1717 = 7

.c
Solution :
Unit digit of 2727 = 3

o
m
Unit digit of 3737 = 7 Remainder of given expression
1 2 3 6
which is
So unit digit of 1717 × 2727 × 3737 = 7 × 3 × 7 = 147 9 9
i.e., unit digit = 7 equal to 6.
Example 20 : POLYNOMIALTHEOREM
Find unit digit of 1818 × 2828 × 288288. This is very powerful theorem to find the remainder.
Solution : According to polynomial theorem.
Unit digit of 1818 is 4. (x + a)n = x n + n c 1 x n–1 . a 1 + n c 2 x n–2 a2 + nc 3 xn–3 a3
Unit digit of 2828 is 6.
Unit digit of 288288 is 6 + ·········· n cn 1 x1a n 1
an … (i)
So unit digit of 1818 × 2828 × 288288 . n
= 4 × 6 × 6 = 144 i.e., 4 x a
=
x
Example 21 :
Find unit digit of 1111 + 1212 + 1313 + 1414 + 1515. xn n
c1 x n 1 1
a n
c2 x n 2
a2 n
cn 1 x1 a n 1
an
… (ii)
Solution : x
Unit digit of 1111 = 1
Unit digit of 1212 = 6 an
remainder of expression (ii) will be equal to remainder of
Unit digit of 1313 = 3 x
Unit digit of 1414 = 6 because rest of the term contains x are completely divisible by x.
Unit digit of 1515 = 5 Example 25 :
So unit digit of given sum will be
1 + 6 + 3 + 6 + 5 = 21 i.e., 1 999
Find the remainder of .
Example 22 : 8
Solution :
Find unit digit of 2121 × 2222 × 2323 × 2424 × 2525.
Solution : 99
999 8 1
2525 will give 5 in unit place, when multiplied by an even
number i.e. 0, 2, 4, 6, 8. It will give zero at unit place. So, zero 8 8
will be at the unit digit of given question. According to polynomial theorem remainder will be equal
199 1
REMAINDERTHEOREM to remainder of the expression = ,1
8 8
a b c Example 26 :
Remainder of expression [i.e. a × b × c when di-
n
899
vided by n] is equal to the remainder of expression Find the remainder of .
7
ar br cr Solution :
[i.e. ar × br × cr when divided by n] , where
n
ar is remainder when a is divided by n. 899 7 1 199
i.e. 1
br is remainder when b is divided by n. and 7 7 7
cr is remainder when c is divided by n. Example 27 :
Example 23 : 11 13 17
Find the remainder of 15 × 17 × 19 when divided by 7. Find remainder of .
6
Solution : Solution :
15 17 19 11 13 17 5 1 5
Remainder of Expression will be equal to
7 6 6
1 3 5 15 1 1
2 i.e. 1 1
7 7 7 6
y
o
u
Number System & Simplification B-7

rs
Example 28 : Solution :

m
a
100
350 (32 )25 (7 2)25 2 25 (23 )8 2

h
9

b
Find remainder of .

o
7 7 7 7 7 7

o
Solution :

b
(7 1)8 2 1 2

.w
100 100 =
9 7 2 7 7

o
rd
7 7 Remainder is 2.

p
re
3 33
2100 299 2 2 2 833 2 Example 32 :

s
= =

s
7 7 7 7

.c
3250

o
33 Find remainder of

m
7 1 1 2 2 7
= 2= . i.e. 2 Solution :
7 7 7
Example 29 : (32 )125 (7 2)125 2125
950 7 7 7
Find remainder of .
7 (23 )41 22 1 4
Solution : =
7 7
50 16 Remainder is 4
950 7 2 250 23 22 816 4
= LAW OF SURDS
7 7 7 7 7 n
1
16 1 1 1
7 1 4 1 4 an a
i.e., 4 anbn (ab) n
7 7
Example 30 : 1
1 m 1
100
25 a n
a mn
Find remainder of .
7
Solution : LAW OF INDICES
50 am × an = am+n am ÷ an = am–n
25100 3 7 4 450
1
7 7 7 (am)n = amn am =
m
a
33
2 3
2 33 1
2100 7 1 1 2 a–m = am / n = n
am
= 2 am
7 7 7 7 a0 = 1
Remainder is 2.
Addition and subtraction of Surds
Example 31 : Example: 5 2 20 2 3 2 22 2
350 Example: 45 3 20 4 5 3 5 6 5 4 5 5
Find remainder of .
7
y
o
u
B-8 Number System & Simplification

rs
m
a
h
b
o
o
b
.w
1. Minimum difference between x and y such that 1x 71y 61 is (a) 38 (b) 31

o
rd
exactly divisible by 11 is, (c) 37 (d) 26

p
(a) 2 (b) 3 (c) 1 (d) 0 12. A boy wanted to write the numbers from the smallest number

re
2. The four integers next lower than 81, and the four next higher to the greatest number of three digits. How many times he

s
s
than 81, are written down and added together, this sum is

.c
needs to press the keys of the computer to do this job?
divisible by,

o
(a) 2708 (b) 2889

m
(a) 7 (b) 9 (c) 11 (d) 13
(c) 2644 (d) 2978
3. If ‘n’ is a natural number then the greatest integer less than
13. A number being successively divided by 3, 5 and 8 leaves
n
that or equal to 2 3 is 1,2 and 4 as remainders respectively. What are the
remainders if the order of divisors be reversed?
(a) odd
(a) 3, 3, 1 (b) 3, 1, 3
(b) even
(c) 1, 3, 3 (d) None of these
(c) even when ‘n’ is even and odd when ‘n’ is odd
(d) even when ‘n’ is odd and odd when n is even 14. The numbers 1 to 29 are written side by side as follows
4. How many numbers, between 1 and 300 are divisible by 3 1234567891011............................ 28 29
and 5 together? If the number is divided by 9, then what is the remainder?
(a) 16 (b) 18 (c) 20 (d) 100 (a) 3 (b) 1
(c) 0 (d) None of these
5. What is the remainder when 1! + 2! + 3! + ...... + 100! is
divided by 7 ? 15. If x 959 y is divisible by 44 and y >5, then what are values of
(a) 0 (b) 5 (c) 6 (d) 3 the digit x and y?
6. How many numbers, lying between 1 and 500, are divisible (a) x=7, y=6 (b) x=4, y=8
by 13? (c) x=6, y=7 (d) None of these
(a) 40 (b) 38 1 1 1 1 2 5 3 7
(c) 41 (d) 46 16. When is divided by ,
2 4 5 6 5 9 5 18
7. Two different numbers when divided by the same divisor,
left remainder 11 and 21 respectively, and when their sum the result is :
was divided by the same divisor, remainder was 4. What is 1 1
the divisor? (a) 2 (b) 3
18 6
(a) 36 (b) 28
(c) 14 (d) 9 3 1
(c) 3 (d) 5
8. A number when divided by a divisor, left remainder 23. When 10 10
twice of the number was divided by the same divisor, 17. A boy multiplied a certain number x by 13. He found that the
remainder was 11. Find the divisor. resulting product consisted of all nines entirely. Find the
(a) 12 (b) 34 smallest value of x.
(c) 35 (d) data inadequate (a) 76913 (b) 76933
9. A number when divided by 5 leaves a remainder 3. What is (c) 76923 (d) 75933
the remainder when the square of the same number is 18. A number is successively divided by 5,6,8; leaving
divided by 5? remainders 3, 4, 7 respectively. What will be the remainders
(a) 9 (b) 3 if the order of divisors be reversed?
(c) 0 (d) 4 (a) 7,4,3 (b) 5,3,4
10. The value of (c) 2,5,4 (d) 1,5,4
19. A certain number is divided by 385 by division by factors.
8 The quotient is 102, the first remainder is 4, the second is 6
3 (8 5) (4 2) 2 is
13 and the third is 10. Find the number.
(a) 39654 (b) 32754
15 13
(a) (b) (c) 38554 (d) None of these
17 17 20 Two numbers when divided by a certain divisor leave the
15 13 remainders 4375 and 2986 respectively; but when the sum
(c) (d) of the two numbers be divided by the same divisor, the
19 19
11. A number when successively divided by 7 and 8 leaves the remainder is 2361. The divisor is
remainders 3 and 5 respectively. What is the remainder when (a) 2014 (b) 5000
the same number is divided by 56? (c) 625 (d) 2639
y
o
u
Number System & Simplification B-9

rs
Find the unit digit in the product (2467)153 × (341)72.

m
21. 33. If the product of first sixty positive consecutive integers be
divisible by 8n, where n is an integer, then the largest

a
(a) 6 (b) 7

h
possible value of n is

b
(c) 8 (d) 9

o
22. Which digits should come in place of * and $ if the number (a) 18 (b) 19

o
b
62684*$ is divisible by both 8 and 5? (c) 17 (d) 16

.w
(a) 4, 0 (b) 0, 4 34. The digit in the unit’s place of the number represented by

o
(795 –358) is:

rd
(c) 0, 0 (d) 4, 4

p
23. A boy multiplies 987 by a certain number and obtains 559981 (a) 0 (b) 4

re
as his answer. If in the answer, both 9’s are wrong but the (c) 6 (d) 7

s
s
other digits are correct, then the correct answer will be: 35. In the product of first fourty positive consecutive integers

.c
be divisible by 5n, where n is an integer, then the largest

o
(a) 553681 (b) 555181

m
(c) 555681 (d) 556581 possible value of n is
24 There is one number which is formed by writing one digit 6 (a) 8 (b) 9
times (e.g. 111111, 444444 etc.). Such a number is always (c) 10 (d) 7
divisible by: 36. 55 3 17 3 72 3 is divisible by
(a) 7 and 11 (b) 11 and 13 (a) both 3 and 13 (b) both 7 and 17
(c) 7, 11 and 13 (d) None of these (c) both 3 and 17 (d) both 7 and 13
25. Find the value of * in the following. 37. After the division of a number successively by 3, 4 and 7,
the remainders obtained are 2, 1 and 4 respectively. What
2 2 * 1 2 1 will be the remainder if 84 divides the same number?
1 1
3 7 7 4 3 6 (a) 80 (b) 76
(c) 41 (d) 53
1 38. At a college football game, 4/5 of the seats in the lower deck
(a) 0.006 (b)
6 of the stadium were sold. If 1/4 of all the seating in the
(c) 0.6 (d) 6 stadium is located in the lower deck, and if 2/3 of all the
26. A number when divided by 296 gives a remainder 75. When seats in the stadium were sold, then what fraction of the
the same number is divided by 37, then the remainder will be: unsold seats in the stadium was in the lower deck ?
(a) 1 (b) 2 (a) 3/20 (b) 1/6
(c) 8 (d) 11 (c) 1/5 (d) 1/3
27. A number was divided successively in order by 4, 5 and 6. 39. A number A4571203B is divisible by 18. Find the value of A
The remainders were respectively 2, 3 and 4. The number is and B.
(a) 8, 4 (b) 6, 8
(a) 214 (b) 476
(c) 4, 6 (d) 6, 6
(c) 954 (d) 1908 40. What is the sum of all two-digit numbers that give a
28. The least number which must be subtracted from 6709 to remainder of 3 when they are divided by 7?
make it exactly divisible by 9 is: (a) 666 (b) 676
(a) 2 (b) 3 (c) 683 (d) 777
(c) 4 (d) 5 41. Let x and y be positive integers such that x is prime and y is
29. 2.002 + 7.9 {(2.8 – 6.3 (3.6 – 1.5) + 15.6)} = ? composite. Then
(a) 2.002 (b) 4.2845 (a) y – x cannot be an even integer
(c) 40.843 (d) 42.845 (b) xy cannot be an even integer.
(c) (x + y)/x cannot be an even integer
2 3 1 7 (d) None of the above statements is true.
30. 9 1 of 3 5 of ?
9 11 7 9
24 6
5 42. Evaluate
(a) (b) 8 24 6
4 (a) 2 (b) 3
32 (c) 4 (d) 5
(c) 8 (d) 9 43. Arranging the following in descending order
81
257, 438, 1519 we get
31. A number when divided successively by 4 and 5 leaves
remainder 1 and 4 respectively. When it is successively (a) 257 > 438 > 1519 (b) 438 > 1519 > 257
divided by 5 and 4, then the respective remainders will be: (c) 1519 > 257 > 438 (d) 257 > 1519 > 438
(a) 1, 2 (b) 2, 3 44. Arranging the following in ascending order
(c) 3, 2 (d) 4, 1 210000, 103000, 36000, 74000 we get
32. How many times must 79 be subtracted from 5 × 104 so as to (a) 36000 < 103000 < 210000 < 74000
obtain 43759? (b) 210000 < 74000 < 103000 < 36000
(a) 77 (b) 78 (c) 103000 < 36000 < 74000 < 210000
(c) 79 (d) 80 (d) 74000 < 36000 < 210000 < 10 3000
y
o
u
B-10 Number System & Simplification

rs
m
(a) 16 m (b) 18 m
3 1 8 4 2 5 5
If all the fractions , , , , , and (c) 20 m (d) 30 m

a
45. are arranged in the

h
5 8 11 9 7 12 12 53. The fluid contained in a bucket can fill four large bottles or seven

b
descending order of their values, which one will be the third?

o
small bottles. A full large bottle is used to fill an empty small

o
bottle. What fraction of the fluid is left over in the large bottle

b
1 4
when the small one is full?

.w
(a) (b)
8 9

o
2 3

rd
5 8 (a) (b)
(c) (d) 7 7

p
12 11

re
4 5

s
46. The smallest of 3, and (c) (d)

s
8 5, 7 6, 10 7 7

.c
o
11 2 is : 1

m
54. At an International Dinner, of the people attending were French
(a) 8 5 (b) 7 6 5
(c) 10 3 (d) 11 2 2
men. If the number of French women at the dinner was greater
47. Which one of the following is the least? 3
than the number of French men, and there were no other French
3, 3 2, 2 and 3 4 people at the dinner, then what fraction of the people at the
(a) (b) 3 dinner were not French?
2 4
3 1 2
(c) 3 (d) 2 (a) (b)
5 5
48. The smallest of 8 5, 7 6, 10 3 and
2 7
(c) (d)
11 2 is: 3 15
(a) 8 5 (b) 7 6 55. From a number of apples, a man sells half the number of existing

(c) (d) 1
10 3 11 2 apples plus 1 to the first customer, sells rd of the remaining
3
1 1
49. in simplified form equals to: 1
2 3 5 2 3 5 apples plus 1 to the second customer and th of the remaining
5
(a) 1 (b) 2 apples plus 1 to the third customer. He then finds that he has 3
apples left. How many apples did he have originally?
1 (a) 15 (b) 18
(c) (d) 0 (c) 20 (d) 25
2
56. The charges of hired car are ` 4 per km for the first 60 km, ` 5 per
50. The value of
km for the next 60 km and `8 for every 5 km for further journey.
If the balance amount left over with Rohit is one-fourth of what
2 3 1 2 3
be paid towards the charges of the hired car for travelling 320 km,
is
2 1 3 3 5 2 2 how much money did he have initially with him?
(a) `1075 (b) `1255
(a) 1 (b) 2 3 (c) `1540 (d) None of these
(c) 2 3 (d) 3 2 57. Arrange the following in ascending order
334, 251, 717, we get
3 (a) 334 > 251 > 717 (b) 717 > 251 > 334
51. When a ball bounces, it rises to of the height from which it
4 34
(c) 3 > 7 > 2 17 51 (d) 251 > 334> 717
fell. If the ball is dropped from a height of 32 m, how high will 58. If the product of first fifty positive consecutive integers be
it rise at the third bounce?
divisible by 7n, where n is an integer, then the largest
1 possible value of n is
(a) 13 m (b) 13 m
2 (a) 7 (b) 8
(c) 10 (d) 5
1 59. In an examination, a boy was asked to multiply a given
(c) 14 m (d) None of these
2
7
1 1 1 1 number by . By mistake, he divided the given number
52. of a pole is coloured red, white, blue, black,
19
10 20 30 40
7
1 1 by and got a result 624 more than the correct answer..
violet, yellow and the rest is green. If the length of the
19
50 60 The sum of digits of the given number is
green portion of the pole is 12.08 metres, then the length of the (a) 10 (b) 11
pole is : (c) 13 (d) 14
y
o
u
Number System & Simplification B-11

rs
m
ANSWER KEY

a
h
1 (a) 8 (d) 15 (a) 22 (a) 29 (d) 36 (c) 43 (b) 50 (c) 57 (b)

b
o
2 (b) 9 (d) 16 (d) 23 (c) 30 (b) 37 (d) 44 (a) 51 (b) 58 (b)

o
b
3 (a) 10 (b) 17 (c) 24 (c) 31 (b) 38 (a) 45 (b) 52 (a) 59 (d)

.w
4 (c) 11 (a) 18 (d) 25 (d) 32 (c) 39 (b) 46 (d) 53 (b)

o
rd
5 (b) 12 (b) 19 (a) 26 (a) 33 (a) 40 (b) 47 (d) 54 (d)

p
6 (b) 13 (a) 20 (b) 27 (a) 34 (b) 41 (d) 48 (d) 55 (c)

re
7 (b) 14 (a) 21 (b) 28 (c) 35 (b) 42 (b) 49 (c) 56 (a)

s
s
.c
o
m
1. (a) As 1x 71y 61 is exactly divisible by 11. 9. (d) Let the number be 5q + 3, where q is quotient
(1 + 7 + y + 1) – (x + 1 + 6) = 0 or multiple of 13 for
Now, (5q 3)2 25q 2 30q 9
minimum difference
9+y–7–x=0 = 25q2 + 30q + 5 + 4
= 5[5q2 + 6q + 1] + 4
x – y= 2
Hence, remainder is 4
2. (b) Four integers next lower than 81 is 80, 79, 78, 77 four
integers next higher than 81 is 82, 83, 84, 85 8
10. (b) 3 8 5 (4 2) 2
Sum = (80 + 82) + (79 + 83) + (78 + 84) + (77+ 85) 13
= 81 + 81 + 81 + 81 = 4 × 81
Sum is divisible by 9 as 81 is divisible by 9. 34
3 3 2
3. (a) putting n = 1, we get 2 3 = whose integral part is 3 13

putting n= 2, we get ( 2 2
3) =4+3+ 4 3 13
3 3 2
whose integral part is 11 34
which is again an odd number
Now, through the options it can be judged that the 3 34
3
greatest integer must always be an odd number. 13 2
4. (c) LCM of 3 and 5 = 15
3 13 2 13
Number divisible by 15 are 15, 30, 45 .....300.
3 34 17
Let total numbers are n
11. (a) 56 = d1 × d2
300 = 15 + (n – 1) × 15
required remainder = d1r2 + r1 where d1 = 7 and
300 = 15 + 15 n – 15
r1 = 3 and r2 = 5.
n = 20 i.e 7 × 5 + 3 = 38
5. (b) 7! + 8! + 9! + 10! + ....... + 100 = 7.6! + 8.7.6! + 9.8.7.6! + 12. (b) He wants to write from 1 to 999. He has to write 9
....... + 100! is completely divisible by 7 as each of the numbers of one digit, 90 numbers of two digits and 900
terms contain at least one 7 in it. numbers of three digits.
Now, 1! + 2! + 3! + 4! + 5! + 6! Total number of times = 1 × 9 + 2 × 90 + 3 × 900 = 2889
= 1 2 6 24 120 720 873 13. (a) Complete remainder = d1d2r3 + d1r2 + r1
which leaves a remainder of 5 when divided by 7. = 3 × 5 ×4 + 3 × 2 + 1 = 67
6. (b) Number divisible by 13, 26, 39, .... 494 Divided 67 by 8, 5 and 3, the remainders are 3, 3, 1.
14. (a) Sum of the digits of the ‘super’ number
Let n be the total numbers 494 = 13 + (n – 1) × 13
= 1 + 2 + 3 + ................... + 29
n = 38
7. (b) Divisor = [Sum of remainders] 29
.{2 1 (29 1).1}
– [ Remainder when sum is divided] 2
= 11 + 21 – 4 = 28
29 29 30
8. (d) Let number be N. .(2 28) = 29 15 435
2 2
Then, N = Divisor × Q1 + 23
435 when divided by 9 leave remainder 3.
2N = Divisor × Q2 + 11,
15. (a) x989y is divisible by 44 it means divisible by 4 and 11
where Q1 and Q2 are quotients respectively.
both.
Here, we have two equations and 3 variables. There x959y is divisible by 4. 9y is divisible by 4.
equations cannot be solved. Therefore y = 6 (given y > 5)
y
o
u
B-12 Number System & Simplification

rs
m
Now, x9596 is divisible by 11 26. (a) Number = (296 × Q) + 75 = (37 × 8Q) + (37 × 2) + 1

a
(x+5+6) – (9 + 9) = 0 = 37 × (8Q + 2) + 1.

h
(11 + x) – 18 = 0

b
Remainder = 1.

o
x= 7

o
27. (a) 4 x Remainders

b
x = 7, y = 6

.w
5 y 2

o
1 1 1 1 30 15 12 10 17

rd
6 z 3
2 4 5 6 60 60

p
16 . (d) 1 4

re
2 5 3 7 2 3 5 7 17
1

s
18

s
5 9 5 18 5 5 9 18 z = 6 × 1 + 4 = 10

.c
y = 5 × 10 + 3 = 53

o
m
17 51 1 x = 4 × 53 + 2 = 214
18 5
60 10 10 28. (c) On dividing 6709 by 9, we get remainder = 4.
17. (c) By actual division, we find that 999999 is exactly divisible Required number to be subtracted = 4.
by 13. The quotient 76923 is the required number. 29. (d) Given exp.= 2.002 + 7.9 (2.8 – 6.3 × 2.1 + 15.6)
18. (d) Complete remainder = d1d2r3 + d1r2 + r1 = 2.002 + 7.9 (2.8 – 13.23 + 15.6) = 2.002 + 7.9 × 15.17
= 5 × 6 × 7 + 5 ×4 + 3 = 233. = 2.002 + 40.843 = 42.845
Dividing 233, by reversing the divisors i.e. by 8, 6, 5;
11 36 36 7
respective remainders are 1, 5, 4. 30. (b) Given exp. 9 of of = 9 –4 4
19. (a) Let the number be z. Now 385 = 5 × 7 ×11 9 11 7 9
=9–1=8
5 z Remainders 31. (b) Complete remainder = d1r2 + r1
7 y 4 = 4 × 4 + 1 = 17
11 x 6 Now, 17 when divided successively by 5 and 4
102 10 The remainders are 2, 3.
32. (c) Let x be the number of times, then
x = 11 × 102 + 10 = 1132 79x + 43759 = 50,000
y = 7x + 6 = 7 × 1132 + 6 = 7930 6241
z = 5y + 4 = 5 × 7930 + 4 = 39654 x = (50000 – 43759) ÷ 79 = = 79
20. (b) Required Divisor = (sum of remainders) 79
– Remainder when sum is divided 33. (a) Product of first sixty consecutive integers = 601
= [ 4375 + 2986] – 2361 = 5000 8 = 2 × 2 × 2 = 23
21. (b) Clearly, unit’s digit in the given product = unit’s digit highest power of 2 is 60!
in 7153 × 172. 60 60 60 60 60
Now, 74 gives unit digit 1. 2 3
7153 gives unit digit (1 × 7) = 7. Also 172 gives unit digit 1.
2 2 2 24 25
Hence, unit’s digit in the product = (7 × 1) = 7. = 30 + 15 + 7 + 3 + 1 = 56
22. (a) Since the given number is divisible by 5, so 0 or 5 must 56
come in place of $. But, a number ending with 5 is highest power of 80 or (23) = 18
3
never divisible by 8. So, 0 will replace $.
Now, the number formed by the last three digits is 4*0, 34. (b) Unit digit in 74 is 1. So, unit digit in 792 is 1.
which becomes divisible by 8, if * is replaced by 4. Unit digit in 795 is 3.
Hence, digits in place of * and $ are 4 and 0 respectively. Unit digit in 34 is 1.
23. (c) 987 = 3 × 7 ×47 Unit digit in 356 is 1.
So, required number must be divisible by each one of Unit digit in 358 is 9.
3, 7, 47. Unit digit in (795 – 358) = (13 – 9) = 4.
None of the numbers in (a) and (b) are divisible by 3, 35. (b) Product of first fourty positive integers
while (d) is not divisible by 7. 1×2×3× ----×40= 40!
Correct answer is (c). 40 40
24. (c) Since 111111 is divisible by each one of 7, 11 and 13, Highset power of 5 8 1 9
so each one of given type of numbers is divisible by
5 52
each one of 7, 11, and 13. as we may write, 222222 largest possible value of n is 9
= 2 × 111111, 333333 = 3 × 111111, etc. (55)3 (17)3 (55 17) 3
36. (c) 55 3 17 3 72 3
5 2 x 5 2 1
25. (d) Let . Then, 553 173 [(55)3 (17)3 3 55 17 72]
3 7 7 4 3 6
3 55 17 72
5 7 x 5 2 5 5 6 37. (d) The required no. is 3[4 (7x + 4) + 1] + 2 = 84x + 53
6 x = 5’ x = 6.
3 2 7 4 3 6 5 So the remainder is 53, when divided by 84.
y
o
u
Number System & Simplification B-13

rs
m
38. (a) Let total number of seats in the stadium be p; Similarly,

a
number of seats in the lower deck be x and number of 2

h
7 6 7 6 2 7 6 13 2 42

b
seats in upper deck be y.

o
p = x + y, x = p/4, y = 3p/4 2

o
10 3 10 3 2 10 3 13 30

b
Now in the lower deck, 4x/5 seats were sold and x/5

.w
2
seats were unsold. 11 2 11 2 2 11 2 = 13 2 22

o
rd
No. of total seats sold in the stadium = 2p/3.

p
No. of unsold seats in the lower deck = x/5 = p/20 47. (d) The smallest number is 3
2

re
No. of unsold seats in the stadium = p/3

s
2

s
p / 20 3 48. (d) Here, 8 5

.c
Required fraction =

o
p /3 20

m
2 2
39. (b) The number is divisible by 18 i.e., it has to be divisible 8 5 2 8 5
by 2 and 9.
=8+5+2× 8 5 13 2 40
B may be 0, 2, 4, 6, 8.
A + 4 + 5 + 7 + 1 + 2 + 0 + 3 + B = A + B + 22. Similary,
A + B could be 5, 14 (as the sum can’t exceed 18, since 2
A and B are each less than 10). 7 6 7 6 2 7 6 = 13 2 42
So, A and B can take the values of 6, 8.
2
40. (b) Number is of the form = 7n + 3; n = 1 to 13 10 3
13
7n(n +1) 2
So, S (7n 3) = + 3n = 10 + 3 + 2 × 10 3 = 13 + 2 30, 11 2
n 1
2
Putting n = 13 we get 7 13 7 39 676 = 11 + 2 + 2 2 11 2 13 2 22
41. (d) x is prime say 7 Clearly, 13 + 2 22 is the smallest among these.
y is not prime but composite no. say 8, 9, 21
(a) 9 – 7 = 2 (b) 7 × 8 = 56 11 2 is the smallest.
21 7 1
(c) 4 49. (c)
7 2 3 5
Put x = 2 and y = 6 and check for the options.
By hit and trial all the 3 options can be proved wrong 2 3 5 2 3 5
24 6 2 6 6 3 6 2 3 5 2 3 2 6 5
42. (b) 3
24 6 2 6 6 6
43. (b) 257 = (23)19 = 819 2 3 5
438 = (42)19 = 1619 2 6
438 > 1519 > 257
44. (a) 210000 = (210)1000 = (1024)1000 1
Similary,
(10)3000 = (103)1000 = (1000)1000 2 3 5
36000 = (36)1000 = (729)1000
74000 = (74)1000 = (1029)1000 2 3 5
36000 < 103000 < 210000 < 74000 2 3 5 2 3 5
3 4
45. (b) 0.6, 0.44
5 9 2 3 5
1 2 2 6
0.0125, 0.28
8 7 Expression
8 5 2 3 5 2 3 5
0.727, 0.41
11 12
therefore, the descending order is 2 6 2 6
8 3 4 5 2 1 2 3 5 2 3 5 3 1
11 5 9 12 7 8 2 6 6 2
4 50. (c) Expression
So, the third fraction is =
9
2 2 2 2 3 1 2 3
46. (d) Here, 8 5 = 8 5 2 8 5
2 1 2 2 1 3 3 5
=8+5+2× 8 5 = 13 + 2 40
y
o
u
B-14 Number System & Simplification

rs
m
2 2 3 3 2 3 x x

a
Remaining apples x 1 1

h
2 2

b
2 2 1 3 3 5

o
o
1 x x 1

b
Apples sold to 2nd customer 1 1 1

.w
3 1 3 1 3 3 5 3 2 6 3

o
3 3 5 3 3 5 3 3 5 x 2

rd
p
6 3

re
9 3 3 5 3 5

s
x x 2

s
27 25 1

.c
Remaining apples
2 6 3

o
m
4 2 3
2 3 x x 2 x 5
2 1
51. (b) Height at the third bounce 2 6 3 3 3

3
3
27 27 1 1 x 5 x 2
32 m 32 m m 13 m . Apples sold to 3rd customer 1
4 64 2 2 5 3 3 15 3
x 5 x 2
1 1 1 1 1 1 Remaining apples
52. (a) Green portion 1 3 3 15 3
10 20 30 40 50 60
x x 5 2 4x
1 1 1 1 1 1 1 147 3 15 3 3 15 3
1 1 1
10 2 3 4 5 6 10 60
4x 7 4x 16 16 15
3 x 20
147 453 15 3 15 3 3 4
1
600 600
Let the length of the pole be x metres. 8
56. (a) Hire charges = ` 60 4 60 5 200 = `860
5
453 12.08 600
Then, x = 12.08 x 16
600 453 1
Suppose Rohit had ` x with him initially. Then, x – 860 860
53. (b) Let the capacity of the bucket be x litres. Then, 4
x = 1075.
x 57. (b) 334 = (32)17 = 917
Capacity of 1 large bottle ; Capacity of 1 small
4 251 = (23)17 = 817
x Clearly, 717 > 817 > 917
bottle . or 717 > 251 > 334
7 58. (b) Product of first fifty positive consecutive integers
x x 3x = 1 × 2 × .... × 50 = 50 !
Fluid left in large bottle
4 7 28 50 50
Largest possible value of n =
3x / 28 3x 4 3 7 72
Required fraction = =7+ 1=8
x/4 28 x 7
59. (d) Let the number be x
1
54. (d) French men = ; French women x 7
5 x = 624
7 19
1 2 1 5 1 19
5 3 5 15 3 19 7
x = 624
1 1 8 7 19
French people ,
5 3 15 361 49
x = 624
8 7 7 19
Not-French 1
15 15 624 133
55. (c) Let the total number of apples be x. Then x=
312
x x = 266
Apples sold to 1st customer 1 . Sum of digits (2 + 6 + 6) = 14
2
y
o
u
rs
m
a
CHAPTER

h
b
2

o
H.C.F & L.C.M

o
b
.w
o
rd
p
re
s
s
.c
FACTORS H.C.F. of two numbers by division method:

o
A number may be made by multiplying two or more other numbers Example 2 :

m
together. The numbers that are multiplied together are called Find the H.C.F. of 140 and 200 by division method.
factors of the final number. Solution :
Factors of 12 = 1, 2, 3, 4, 6, and 12. 140 200 1
All the numbers have a factor of one. 140
Common factor : A common factor of two or more given numbers 60 140 2
is a number which divides each given number completely. 120
Common factor of 12 and 18 are 1, 2, 3, 6. 20 60 3
Co-prime numbers : Two or more numbers that do not have a 60
common factor are known as co-prime or relatively prime. ×
For example : 4 and 15 are Co-prime numbers. H.C.F. of 140 and 200 = 20.
H.C.F. of three numbers by division method :
Highest common factor : The highest common factor (H.C.F.) of Example 3 :
two or more numbers is the greatest number which divides each
Find the H.C.F. of 324, 630 and 342 by division method.
of them exactly. It is also known as greatest common divisor Solution :
(G.C.D.).
H.C.F. can be calculated by : 324 630 1
324
(i) Prime factorisation method 306 324 1
(ii) Division method 306
(i) H.C.F. by prime factorisation method : 18 306 17
Example 1 : 306
Find the H.C.F. of 40 and 60 by prime factorisation method. ×
Solution : 18 342 19
342
Prime factors of 40. ×
2 40 H.C.F. of 324, 630 and 342 is 18.
2 20 H.C.F of polynomials :
2 10 40 2 2 2 5 When two or more polynomials are factorised, the product
55 of common factor is known as HCF of these polynomials.
1
e.g Lets find the HCF of 16x3 (x – 1)3 (x + 1)
Prime factors of 60
and 4xy (x + 1)2 (x – 1)
2 60
Now, 16x3 (x – 1)3 (x + 1)
2 30
3 15 60 2 2 3 5 = 2 × 2 × 2 × 2 × x × x × x × (x – 1) × (x – 1) × (x – 1) × (x + 1)
55 and 4xy (x + 1)2 (x – 1) = 2 × 2xy (x + 1) (x + 1) (x – 1)
1 H.C.F. = 2 × 2 × x × (x + 1) (x – 1) = 4x (x2 – 1)
Hence H.C.F. = 2 × 2 × 5 = 20.
Divide the greater number by the smaller number, divide the divisor MULTIPLES
by the remainder and so on, until no remainder is left. The last Multiples of a number are all those numbers which can be
divisor in the required H.C.F. divided completely by the given number.
Finding the H.C.F of more than two number: We find the H.C.F of For example, Multiples of 5 are 5, 10, 15, 20 etc.
any two say P. How find the H.C.F of P and the third number and Common multiples : Common multiples of two or more
so on. The last H.C.F will be the required H.C.F.
numbers are the numbers which can be exactly divided by each
(ii) H.C.F by Division method :
of the given number.
• Divide the greater number by the smaller number, divide the
For example, Multiples of 3 are 3, 6, 9, 12, 15, 18, 21, 24 etc. and
divisor by the remainder divide the remainder by the next remainder
Multiples of 4 are 4, 8, 12, 16, 20, 24, 28 etc.
and so on, until no remainder is left, the last divisor is the required
H.C.F. Common multiples of 3 and 4 are 12, 24 etc.
• Finding the H.C.F of more than two number : we find the H.C.F Least common multiple : The least common multiple (L.C.M.)
of any two say P. Now find the H.C.F of P and the third number of two or more numbers is the smallest number which is exactly
and so on. The last H.C.F will be the required H.C.F. divisible by each of them.
y
o
u
B-16 H.C.F & L.C.M

rs
L.C.M. can be calculated by : Example 7 :

m
a
(i) Prime factorisation method Find the H.C.F. and L.C.M. of 0.6, 9.6 and 0.36.

h
(ii) Division method

b
Solution :

o
(i) L.C.M. by prime factorisation method : H.C.F of 60, 96 and 36 = 12

o
b
Example 4 : Required HCF = 0.12

.w
Find the L.C.M. of 12 and 20 by prime factorization method.
L.C.M of 60, 96 and 36 = 1440

o
Solution :

rd
12 = 2 × 2 × 3 and 20 = 2 × 2 × 5 Required L.C.M = 144.0

p
The product of two numbers :

re
L.C.M = 2 × 2 × 3 × 5 = 60.

s
(ii) L.C.M. by division method : H.C.F. of numbers × L.C.M. of numbers = Product of numbers

s
.c
Example 5 :

o
Find the L.C.M. of 14, 56, 91 and 84.

m
Example 8 :
Solution :
If H.C.F. and L.C.M. of two numbers are 3 and 60
2 14, 56, 91, 84 respectively and one number is 12 then find the other
2 7, 28, 91, 42 number.
7 7, 14, 91, 21 Solution :
1, 2, 13, 3 Let the other number be x.
L.C.M = 2 × 2 × 7 × 2 × 13 × 3 = 2184. Product of numbers = H.C.F. × L.C.M.
L.C.M of polynomials : x × 12 = 3 × 60
When two or more polynomials are factorised, the product 3 60
of the factors with highest powers is the lowest common x = 15.
multiple (LCM) of the polynomials. 12
Eg. Consider the polynomials (x3 – 8) and (x2 – 4)
x3 – 8 = (x – 2) (x2 + 2x + 4)
The greatest number that will exactly divide x, y, z = H.C.F of
and x2 – 4 = (x + 2) (x – 2)
x, y and z.
L.C.M = (x – 2) (x + 2) (x2 + 2x + 4)
The greatest number that will divide x, y and z leaving
= (x + 2) (x3 – 8) remainders a, b and c respectively = H.C.F of (x – a), (y – b)
H.C.F. AND L.C.M. OF FRACTIONS : and (z – c ).
First express the given fractions in their lowest terms. The least number which is exactly divisible by x, y and z
Then, = L.C.M of x, y and z.
H.C.F. of numerators The least number which when divided by x, y and z leaves
H.C.F. the remainder a, b and c respectively= L.C.M of(x, y and z)–
L.C.M. of denominators R where R = (x – a) = (y – b) = (z – c)
L.C.M. of numerators The least number which when divided by x, y and z leaves
L.C.M. the same remainder r in each case = L.C.M of (x, y and z) + r
H.C.F. of denominators
The greatest number that will divide x, y and z leaving the same
Example 6 : remainder in each case = H.C.F of (x – y), (y – z) and
1 6 1 (z – x).
Find the H.C.F. and L.C.M. of 4 , ,10 . When two numbers P and Q are exactly divisible by a third
2 2 2
number r. Then p + q, p – q and pq is also divisible by r.
Solution :
1 9 6 1 21 Example 9 :
Here, 4 , 3 , 10 . The H.C.F of two numbers, each having three digits, is 17
2 2 2 2 2
and their L.C.M is 714.
H.C.F. of 9, 3, 21 3 1 Five sum of the numbers will be:
H.C.F. = 1 .
L.C.M. of 2,1, 2 2 2 Solution :
L.C.M. of 9, 3, 21 63 Let the number be 17x and 17y where x and y are co-prime.
L.C.M. = 63 .
H.C.F. of 2,1, 2 1 L.C.M of 17 x and 17 y = 17 xy
H.C.F. AND L.C.M. OF DECIMAL NUMBERS : According to question, 17xy = 714 xy = 42 = 6 × 7
H.C.F of decimal numbers :
STEP I : Find the H.C.F of given numbers without decimal. x = 6 and y = 7
STEP II : Put the decimal point from right to left according to the or x = 7 and y = 6
maximum decimal places among the given numbers.
L.C.M of decimal numbers : First numbers = 17x = 17 × 6 = 102
STEP I : Find the L.C.M of given numbers without decimal. Second numbers = 17y = 17 × 7 = 119
STEP II : Put the decimal point from right to left according to the
minimum decimal places among the given numbers. Required sum = 102 + 119 = 221
y
o
u
H.C.F & L.C.M B-17

rs
Example 10 :

m
Clearly, 0.375 < 0.5 < 0.583 < 0.64 < 0.666 < 0.736

a
Find the greatest number of six digits which on being 3 1 7 16 2 14

h
b
divided by 6,7, 8, 9 and 10 leaves 4, 5, 6, 7 and 8 as 8 2 12 25 3 19

o
remainders respectively.

o
Example 15 :

b
Solution : The H.C.F of two polynomials is x² – 1 and their L.C.M is

.w
The L.C.M. of 6, 7, 8, 9 and 10 = 2520 x4 – 10x2 + 9. If one of the polynomials is x³ – 3x² – x + 3, find

o
rd
The greatest number of 6 digits = 999999 the other.

p
Solution :

re
Dividing 999999 by 2520, we get 2079 as remainder.
Given that H.C.F of p (x) and q (x) = x² – 1 = (x + 1) (x – 1)

s
s
Hence the 6 digit number divisible by 2520 is Also, LCM of p (x) and q (x) = x4 – 10x² + 9 = x4 – 9x² – x² + 9

.c
999999 – 2079 = 997920

o
= x² (x² – 9) – (x² – 9) = (x² – 9) (x² – 1)

m
Since 6 – 4 = 2, 7 – 5 = 2, 8 – 6 = 2, 9 – 7 = 2, = (x + 3) (x – 3) (x + 1) (x – 1)
10 – 8 = 2, the remainder in each case is less than the and p (x) = x³ – 3x² – x + 3 = x² (x – 3) – (x – 3)
divisor by 2. = (x – 3) (x² – 1) = (x – 3) (x + 1) (x – 1)
p(x).q(x) = (H.C.F).(L.C.M)
Required number = 997920 – 2 = 997918.
Example 11 : (H.C.F) (L.C.M) (x 1)(x 1)(x 3)(x 3)(x 1)(x 1)
q(x)
p(x) (x 3)(x 1)(x 1)
What least number must be subtracted from 1936 so that
the remainder when divided by 9, 10, 15 will leave in each = (x + 3) (x + 1) (x – 1)
case the same remainder 7 ? = (x + 3) (x² – 1) = x³ + 3x² – x – 3
Solution : Example 16 :
The L.C.M. of 9, 10 and 15 is 90. Find the H.C.F and L.C.M of 6, 72 and 120, using the prime
On dividing 1936 by 90, the remainder = 46. factorisation method.
But 7 is also a part of this remainder. Solution :
Required number = 46 – 7 = 39. We have : 6 = 2 × 3, 72 = 23 × 32, 120 = 23 × 3 × 5
Example 12 : Here, 21 and 31 are the smallest powers of the common factors
Find the greatest number which will divide 410, 751 and 2 and 3 respectively.
1030 leaving a remainder 7 in each case. So, H.C.F (6, 72, 120) = 21 × 31 = 2 × 3 = 6
Solution : 23, 32 and 51 are the greatest powers of the prime factors 2,
Required number 3 and 5 respectively involved in the three numbers.
= H.C.F. of (410 – 7), (751 – 7) and (1030 – 7) So, L.C.M (6, 72, 120) = 23 × 32 × 51 = 360.
= 31. Example 17 :
Example 13 : Find the GCD of : 14x³ + 14, 42 (x² + 4x + 3) (x² – x + 1)
Find the H.C.F and L.C.M of 1.75, 5.6 and 7 Solution :
Solution : p (x) = 14x³ + 14 = 14 (x³ + 1) = 2 × 7 (x + 1) (x² – x + 1)
Making the same number of decimal places, the numbers q (x) = 42 (x² + 4x + 3) (x² – x + 1)
may be written as 1.75, 5.60 and 7.00. = 42 (x² + 3x + x + 3) (x² – x + 1)
Without decimal point, these numbers are 175, 560 and 700. = 42 [x (x + 3) + (x + 3)] (x² – x + 1)
Now, H.C.F of 175, 560 and 700 is 35. = 2 × 3 × 7 (x + 3) (x + 1) (x² – x + 1)
H.C.F of 1.75, 5.6 and 7 is 0.35. GCD of p (x) and q (x) = 14 (x + 1) (x² – x + 1) = 14 (x³ + 1)
L.C.M of 175, 560 and 700 is 2800. Example 18 :
L.C.M of 1.75, 5.6 and 7 is 28.00 i.e. 28. Two bills of Rs 6075 and Rs 8505 respectively are to be paid
COMPARISON OF FRACTIONS : separately by cheques of same amount. Find the largest
Rule : Convert each one of the given fractions in the decimal possible amount of each cheque.
form. Now, arrange them in ascending or descending order, Solution :
as per requirement. Largest possible amount of cheque will be the H.C.F (6075,
Example 14 : 8505).
We can write 8505 = 6075 × 1 + 2430
3 7 2 14 16 1
Arrange the fractions , , , , and in ascending Since, remainder 2430 0 again applying division concept
8 12 3 19 25 2 we can write
order of magnitude. 6075 = 2430 × 2 + 1215
Solution :
Again remainder 1215 0
Converting each of the given fractions into decimal form,
So, again applying the division concept we can write
3 7 2 14 2430 = 1215 × 2 + 0
we get : 0.375, 0.583, 0.666, 0.736 ,
8 12 3 19 Here the remainder is zero
So, H.C.F = 1215
16 1
0.64 and 0.5 . Therefore, the largest possible amount of each cheque will
25 2 be 1215.
y
o
u
B-18 H.C.F & L.C.M

rs
m
Example 19 : Solution :

a
A garden consists of 135 rose plants planted in certain

h
number of columns. There are another set of 225 marigold 95

b
The first watch ticks every seconds.

o
plants which is to be planted in the same number of columns. 90

o
b
What is the maximum number of columns in which they can 95 323

.w
be planted? They will tick together after (L.C.M of & ) seconds.
90 315

o
Solution :

rd
To find the maximum number of columns we need to find the 95 323 L.C.M of 95,323 19 5 17

p
Now, L.C.M of and = =

re
H.C.F(135, 225) 90 315 H.C.F of 90,315 45

s
We can write, 225 = 135 × 1 + 90

s
The number of times they will tick in the first 3600 seconds

.c
Since, remainder 90 0

o
19 5 17 3600 45 100

m
So, again applying division concept, we can write, = 3600 = = 100
135 = 90 × 1 + 45 45 19 5 17 323
Remainder 45 0 again using division concept, we have, Once they have already ticked in the beginning; so in 1 hour
90 = 45 × 2 + 0 they will tick 100 + 1 = 101 times.
Since, remainder is 0
So, H.C.F = 45 Example 21 :
Therefore, 45 is the maximum number of columns in which Find the H.C.F and L.C.M of 14xy3, 22x2y and 26x3y4.
the plants can be planted. Solution :
Example 20 : 14xy3 = 2 × 7 × x × y3
A watch ticks 90 times in 95 seconds and another watch 22x2y = 2 × 11 × x2 × y
ticks 315 times in 323 seconds. If both the watches are started 26x3y4 = 2 × 13 × x3 × y4
together, how many times will they tick together in the first H.C.F = 2 × x × y = 2xy
hour ? L.C.M = 2 × 7 × 11 × 13 × x3 × y4 = 2002x3y4.

1. The H.C.F and L.C.M of two numbers are 21 and 4641 (a) 21 (b) 48
respectively. If one of the numbers lies between 200 and (c) 84 (d) 108
300, then the two numbers are 7. If the L.C.M and H.C.F. of two numbers are 2400 and 16, one
(a) 273, 357 (b) 273, 361 number is 480; find the second number.
(c) 273, 359 (d) 273, 363 (a) 40 (b) 80
2. What is the smallest number which when increased by 5 is
(c) 60 (d) None of these
completely divisible by 8, 11 and 24?
8. The maximum number of students among them 1001 pens
(a) 264 (b) 259
and 910 pencils can be distributed in such a way that
(c) 269 (d) None of these each student gets the same number of pens and same
3. Which is the least number that must be subtracted from number of pencils is :
1856, so that the remainder when divided by 7, 12 and 16
(a) 91 (b) 910
will leave the same remainder 4?
(c) 1001 (d) 1011
(a) 137 (b) 1361
(c) 140 (d) 172 9. Find the maximum number of students among whom 429
4. Find the greatest number that will divide 148, 246 and 623 mangoes and 715 oranges can be equallly distributed.
leaving remainders 4, 6 and 11 respectively. (a) 100 (b) 120
(a) 11 (b) 12 (c) 160 (d) None of these
(c) 13 (d) 14 10. Two numbers both greater than 29, have H.C.F 29 and L.C.M
5. Three bells chime at an interval of 18, 24 and 32 minutes 4147. Five sum of the number is:
respectively. At a certain time they begin to chime together. What (a) 666 (b) 669
length of time will elapse before they chime together again. (c) 696 (d) 966
(a) 2 hours 24 minutes (b) 4 hours 48 minutes 11. Find the greatest number which when subtracted from 3000
(c) 1 hour 36 minutes (d) 5 hours is exactly divisible by 7, 11, 13.
6. The L.C.M and H.C.F of two numbers are 84 and 21, (a) 1799 (b) 2099
respectively. If the ratio of two numbers be 1 : 4, then the
(c) 1899 (d) 1999
larger of the two numbers is :
y
o
u
H.C.F & L.C.M B-19

rs
m
12. The L.C.M. of two number is 630 and their H.C.F. is 9. If the (a) 22 hrs (b) 12 hrs

a
sum of numbers is 153, their difference is (c) 11 hrs (d) 44 hrs

h
b
(a) 17 (b) 23 22. From 3 drums of milk, 279, 341 and 465 respectively are to be

o
o
(c) 27 (d) 33 drawn out. To do it in a minimum time, the capacity of the

b
measuring can be

.w
13. Suppose you have 108 green marbles and 144 red marbles.
(a) 271 (b) 61

o
You decide to separate them into packages of equal number

rd
of marbles. Find the maximum possible number of marbles (c) 111 (d) 31

p
in each package. 23. A man was engaged for a cetain number of days for `404.30

re
p but being absent for some days he was paid only

s
(a) 4 (b) 36

s
` 279.90p. His daily wages cannot exceed by:

.c
(c) 9 (d) 12

o
(a) ` 29.10 p (b) ` 31.30 p

m
14. Find the lowest number which when subtracted from 3000,
(c) ` 31.10 p (d) ` 31.41 p
is exactly divisible by 7, 11 and 13
24. Let N be the greatest number that will divide 1305, 4665 and
(a) 729 (b) 998 6905, leaving the same remainder in each case. Then sum of
(c) cannot be determined the digits is N is :
(d) none of these (a) 4 (b) 5
15. Find the greatest number that will divide 55, 127 and 175, so (c) 6 (d) 8
as to leave the same remainder in each case. 25. The L.C.M. of two numbers is 45 times their H.C.F. If one of
(a) 11 (b) 16 the numbers is 125 and the sum of H.C.F. and L.C.M. is
(c) 18 (d) 24 1150, the other number is:
16. One pendulum ticks 57 times in 58 seconds and another 608 (a) 215 (b) 220
times in 609 seconds. If they started simultaneously, find (c) 225 (d) 235
the time after which they will tick together. 26. The H.C.F. of two numbers is 23 and the other factors of
their L.C.M. are 13 and 14. The larger of the two numbers is:
211 1217 (a) 276 (b) 299
(a) s (b) s
19 19 (c) 322 (d) 345
27. An electric wire is sold only in multiples of 1 metre, and a
1218 1018 customer required several lengths of wire, each 85cm long.
(c) s (d) s
19 19 To avoid any wastage and to minimise labour, he should
17. Find the greatest possible rate at which a man should walk purchase minimum lengths of
to cover a distance of 70 km and 245 km in exact number of (a) 8.5 metres (b) 17 metres
days? (c) 1.7 metres (d) 1 metre
(a) 55 (b) 60 28. There are 4 numbers. The H.C.F. of each pair is 3 and the
(c) 35 (d) 45 L.C.M. of all the 4 numbers is 116. What is the product of 4
numbers?
18. The least number, which when divided by 2, 3, 4, 5 and 6,
(a) 9782 (b) 9396
leaves in each case, a remainder 1, but when divided by 7
(c) 9224 (d) 9100
leaves no remainder. The number is
29. Three wheels can complete respectively 60,36,24 revolutions
(a) 121 (b) 181
per minute. There is a red spot on each wheel that touches
(c) 241 (d) 301 the ground at time zero. After how much time, all these
19. I have a certain number of beads which lie between spots will simultaneously touch the ground again?
600 and 900. If 2 beads are taken away the remainder (a) 5/2 seconds (b) 5/3 seconds
can be equally divided among 3, 4, 5, 6, 7 or 12 boys. (c) 5 seconds (d) 7.5 seconds
The number of beads I have 30. Number of students who have opted the subjects A, B, C
(a) 729 (b) 842 are 60, 84, 108 respectively. The examination is to be
(c) 576 (d) 961 conducted for these students such that only the students
20. The greatest number which will divide 410, 751 and 1030 of the same subject are allowed in one room. Also the number
leaving a remainder 7 in each case is of students in each room must be same. What is the minimum
(a) 29 (b) 31 number of rooms that should be arranged to meet all these
conditions?
(c) 17 (d) 37
(a) 28 (b) 60
21. Three men start together to travel the same way around a
(c) 12 (d) 21
1 31. What will be the least number which when doubled will be
circular track of 11 kms. Their speeds are 4, 5 , and 8 kms
2 excactly divisible by 12,18,21 and 30?
per hour respectively. When will they meet at the starting (a) 196 (b) 630
point? (c) 1260 (d) 2520
y
o
u
B-20 H.C.F & L.C.M

rs
m
32. A hall is 13 metres 53 cm long and 8 metres 61cm broad is to (a) 54 (b) 64

a
be paved with minimum number of square tiles. The number (c) 71 (d) 81

h
b
of tiles required is: 39. The least number of five digits which is exactly divisible by

o
o
(a) 123 (b) 77 12, 15 and 18, is:

b
.w
(c) 99 (d) 57 (a) 10010 (b) 10051

o
33. In a school there are 391 boys and 323 girls. These are to be (c) 10020 (d) 10080

rd
divided into the largest possible equal classes, so that there 40. The sum of two numbers is 462 and their highest common

p
re
are equal number of boys and girls in each class. How many factor is 22. What is the maximum number of pairs that

s
classes are possible?

s
satisfy these conditions ?

.c
(a) 32 (b) 37 (a) 1 (b) 3

o
m
(c) 42 (d) 49 (c) 5 (d) 6
34. A shopkeeper has three kinds of sugar 184 kg; 230 kg and 41. Three numbers are in the ratio of 3: 4 : 5 and their L.C.M is
276kg. He wants to store it into minimum number of bags to
2400. Their H.C.F is:
equal size without mixing. Find the size of the bag and the
(a) 40 (b) 80
number of bags required to do the needful.
(c) 120 (d) 200
(a) 23 kg; 30 (b) 38 kg; 23
(c) 46 kg; 15 (d) 46 kg; 25 42. A number when divided by 49 leaves 32 as remainder. This
number when divided by 7 will have the remainder as
35. There are four prime numbers written in the ascending order
of magnitude. The product of first three numbers and that (a) 4 (b) 3
of the last three number is 385 and 1001 respectively . The (c) 2 (d) 5
fourth prime number is: 43. The traffic light at three different road crossing changes
(a) 11 (b) 13 after 24 second, 36 second, 54 second respectively. If they
(c) 17 (d) 19 all changes simultaneously. at 10 : 15 : 00 AM then at what
36. If the sum of two numbers is 55 and the H.C.F. and L.C.M. of time will they again changes, simultanously
these numbers are 5 and 120 respectively, then the sum of (a) 10 : 16 : 54 AM
the reciprocals of the numbers is equal to: (b) 10 : 18 : 36 AM
55 601 (c) 10 : 17 : 02 AM
(a) (b)
601 55 (d) 10 : 22 : 12 AM
11 120 44. When n is divisible by 5 the remainder is 2. What is the
(c) (d) remainder when n2 is divided by 5.
120 11
37. Product of two co-prime numbers is 117. Their L.C.M. should (a) 2 (b) 3
be: (c) 1 (d) 4
(a) 1 (b) 117 45. The least number which when divided by 48, 64, 90, 120 will
(c) equal to their H.C.F. (d) cannot be calculated leave the remainders 38, 54, 80, 110 respectively, is
38. The number of prime factors in the expression (a) 2870 (b) 2860
(6)10× (7)17× (11)27 is: (c) 2890 (d) 2880

ANSWER KEY
1 (a) 7 (b) 13 (b) 19 (b) 25 (c) 31 (b) 37 (b) 43 (b)
2 (b) 8 (a) 14 (b) 20 (b) 26 (c) 32 (b) 38 (b) 44 (d)
3 (d) 9 (d) 15 (d) 21 (a) 27 (b) 33 (c) 39 (d) 45 (a)
4 (b) 10 (c) 16 (c) 22 (d) 28 (b) 34 (c) 40 (d)
5 (b) 11 (d) 17 (c) 23 (c) 29 (c) 35 (b) 41 (a)
6 (c) 12 (c) 18 (d) 24 (a) 30 (d) 36 (c) 42 (a)
y
o
u
H.C.F & L.C.M B-21

rs
m
a
h
b
o
o
b
.w
1. (a) Product of the numbers
x y (153) 2 4(630 9)

o
= H.C.F × L.C.M= 21 × 4641

rd
= 21 × 3 × 7 × 13 × 17

p
23409 22680 729 27

re
= 3 × 7 × 3 × 7 × 13 × 17

s
The required numbers can be 13. (b) Required number = H.C.F of 108 and 144 = 36

s
.c
3 × 7 × 13 and 3 × 7 × 17 = 273 and 357 14. (b) Required number = 3000 – maximum possible multiple

o
of L.C.M of (7, 11 and 13)

m
2. (b) Required no. = L.C.M of ( 8, 11, 24 ) –5 = 264 – 5 = 259
= 3000 – 2(1001) = 998
3. (d) Suppose least no. be x
15. (d) Required number = H.C.F of (127 – 55), (175 – 127) and
1856 x n ( LCM of 7,12,16) 4 (175 – 55)
or 1856 -x = n (336) + 4 = H.C.F of 72, 48 and 120 = 24
we should take n = 5 so that n(336) is nearest to 1856 16. (c) Time gap between two consecutive ticks
and n (336)< 1856 58 609
1856 - x = 1680 + 4 = 1684 sec. and sec.
57 608
x = 1856 - 1684 = 172
4. (b) Required number 58 609
Required time = L.C.Mof and
= H.C.F of (148 – 4), (246 – 6) and (623 – 11) 57 608
= H.C.F of 144 , 240 and 612 = 12 L.C.M of 58 and 609 1218
5. (b) L.C.M of 18, 24 & 32 = 288 sec
H.C.F of 57 and 608 19
Hence they would chime after every 288 min. or 4 hrs 17. (c) Rate = H.C.F of 70 and 245 = 35
48 min 18. (d) L.C.M (2, 3, 4, 5, 6) = 60
6. (c) Let the numbers be x and 4x. Required number is of the form 60 k + 1
Then, 84 21 x 4x Least value of k for which 60k + 1 is divisible by 7 is
k=5
or 4x 2 1764
Required number = 60 × 5 + 1= 301
or x 2 441 or x = 21 19. (b) L.C.Mof the numbers = 420.
4x = 4 × 21 = 84 Hence there must be (420 × 2) + 2 = 842 beads.
Thus the larger number = 84 20. (b) Required number = H.C. F of (410 – 7), (751 – 7) and
7. (b) Product of numbers = (L.C.M× H.C.F) (1030 – 7)
480 × second number = 2400 × 16 = H. C. F of 403, 744 and 1023 = 31
second number = 80 21. (a) Time taken by them to complete the track
8. (a) Required number of students = H.C.F. of 1001 and 910 11 11 11 11 11
, , hrs , 2, hrs
= 91 4 11/ 2 8 4 8
9. (d) Required number = H.C.F of 429 and 715 = 143 11 11
10. (c) Product of numbers = 29 × 4147 Required time = L.C.Mof , 2,
4 8
let the number be 29a and 29 b.
Then, 29a × 29b = 29 × 4147 L.C.M of (11, 2, 11)
22hrs
ab = 143 H.C.F of (4,1,8)
Now co-primes with product 143 are (1, 143) and (11,13) 22. (d) To find the capacity we have to take the H.C.F of 279,
so the numbers are (29×1, 29×143) and (29×11, 29 × 13) 341 and 465.
Since both are greater than 29, so suitable pair is 279 = 31 × 3 × 3
(29 × 11, 29×13) i.e (319, 377) 341 = 31 × 11
Required sum = 319 + 377 = 696 465 = 31 × 3 × 5
11. (d) Required number = 3000 – L.C.M of 7, 11, 13 H.C.F (279, 341, 465) = 31
= 3000 – 1001 = 1999 Capacity of the measuring can be = 31 ml.
12. (c) Let numbers be x and y. 23 (c) His maximum daily wages must be the H.C.F. of 404.30
Product of two numbers = their (L.C.M× H.C.F) and 279.90 which is 31.10.
xy = 630 × 9 24. (a) N = H.C.F of (4665 – 1305), (6905 – 4665) and
Also, x + y = 153 (given) (6905 – 1305)
= H.C.F. of 3360, 2240 and 5600 = 1120
since x – y (x y) 2 4xy Sum of digits in N = (1 + 1 + 2 + 0) = 4
y
o
u
B-22 H.C.F & L.C.M

rs
m
25. (c) Let H.C.F. be h and L.C.M. be l. Then, l = 45h and l + h 35. (b) Let p, q, r and s be the four prime numbers, so that

a
= 1150. p × q × r = 385 and q × r × s =1001

h
b
45h + h = 1150 or h = 25. So, l = (1150 – 25) = 1125 q × r is the H.C.F. of 385 and 1001 which is 77.

o
o
25 1125 s = 1001 ÷ 77 = 13

b
Hence, other number 225

.w
125 36. (c) Let the numbers be a and b. Then, a + b = 55 and

o
26. (c) Clearly, the numbers are (23 × 13) and (23 × 14). ab = 5 × 120 = 600.

rd
Larger number = (23 × 14) = 322.

p
1 1 a b 55 11

re
27. (b) Required length = L.C.M of 100 cm and 85 cm Required sum .
a b ab 600 120

s
s
= 1700 cm = 17 m

.c
37. (b) H.C.F of co-prime numbers is 1.
28. (b) Product of ‘n’ numbers = (H.C.F for each pair)n ×

o
So, L.C.M. = 117/1 = 117.

m
L.C.M of ‘n’ numbers
=(3)4 × 116 = 9396 38. (b) Since, 2, 3, 7, 11 are prime numbers and the given
29. (c) A makes 1 rev. per sec expression is 210 × 310 × 317 × 1127
So the numbers of prime factors in the given expression
B makes 6 rev per sec is (10 + 10 + 17 + 27) = 64
10
39. (d) Least number of 5 digits is 10,000. L.C.M. of 12, 15 and
4
C makes rev. per sec 18 is 180.
10
On dividing 10000 by 180, the remainder is 100.
In other words A, B and C take 1 , 5 & 5 seconds to Required number = 10000 + (180 – 100) = 10080.
3 2 40. (d) There are 6 such pairs :
complete one revolution.
(22, 440), (44, 418), (88, 374), (110, 352)
5 5 L.C.M. of 1,5,5
L.C.M of 1, & 5 (176, 286), (220, 242)
3 2 H.C.F. of 1,3,2
Hence, after every 5 seconds the red spots on all the 41. (a) Let the numbers are 3x, 4x and 5x then their L.C.M = 60x
three wheels touch the ground So, 60x = 2400 (given)
30. (d) H.C.F of 60, 84 and 108 is 12 so each room contain 12 x = 40
students at minimum The number are (3×40), (4×40), (5×40) = 120, 160, 200
So that each room contains students of only 1 subject Hence required H.C.F. = 40
60 108 84 42. (a) Number = 49K + 32
Number of rooms = 21 rooms
12 12 12 = (7 × 7 K) + (7 × 4) + 4
31. (b) L.C.M. of 12, 18, 21, 30 = 2 × 3 × 2 × 3 × 7 × 5 = 1260 = 7 (7 K + 4) + 4
Required number = (1260 ÷ 2) = 630. So, the number when divided by 7 gives remainder = 4
32 (b) 13 m 53 cm = 1353 cm and 8 m 61 cm = 861 cm.
Short-method:- 32 7 = (7 × 4) + 4
H.C.F. of 1353 and 861 is 123.
43. (b) L.C.M of 24, 36 and 54 second.
Now since minimum tiles are required for having the
floor so area of the tile has to be the greatest so its size = 216 second
is greatest which is 123 cm. = 3 minute + 36 second.
the number of square tiles Required time = 10 : 15:00 + L.C.Mof 24, 36 and 54
Area of the floor 1353 861 = 10 : 15 : 00 + 3 min + 36 sec.
77 = 10 : 18 : 36 AM
Area of the tile 123 123
33. (c) The number of boys and girls in each class is the H.C.F. 44. (d) n = 5K + 2
of 391 and 323 which is 17. Squaring both side
391 323 n2 = (5K + 2)2
Number of classes 23 19 42
17 17 = 25K2 + 20K + 4
34. (c) Size of the bag is the H.C.F. of the numbers 184, 230, So, the required remainder is 4.
276 which is 46. 45. (a) Here, (48 – 38) = 10, (64 – 54) = 10, (90 – 80) = 10 and
The number of bags (120 – 110) = 10.
184 230 276 Required number = (L.C.M of 48, 64, 90 and 120) – 10
4 5 6 15
46 46 46 = 2870
y
o
u
rs
m
a
CHAPTER

h
ALGEBRAIC EXPRESSIONS

b
3

o
o
b
AND INEQUALITIES

.w
o
rd
p
re
s
s
.c
VARIABLE On reversing the digits, we have x at the tens place and

o
m
The unknown quantities used in any equation are known as (16 – x) at the unit place.
variables. Generally, they are denoted by the last English alphabet New number = 10x + (16 – x) = 9x + 16
x, y, z etc. Original number – New number = 18
An equation is a statement of equality of two algebraic
expressions, which involve one or more unknown quantities, called (160 – 9x) – (9x + 16) = 18
the variables. 160 – 18x – 16 = 18
LINEAR EQUATION – 18x + 144 = 18
An equation in which the highest power of variables is one, is – 18x = 18 – 144 18x = 126
called a linear equation. These equations are called linear be- x=7
cause the graph of such equations on the x–y cartesian plane is
a straight line. In the original number, we have unit digit = 7
Linear Equation in one variable : A linear equation which con- Tens digit = (16 – 7) = 9
tains only one variable is called linear equation in one variable. Thus, original number = 97
The general form of such equations is ax + b = c, where a, b Example 3 :
and c are constants and a 0 . The denominator of a rational number is greater than its
All the values of x which satisfy this equation are called its numerator by 4. If 4 is subtracted from the numerator and 2
solution(s). 1
is added to its denominator, the new number becomes .
6
NOTE : Find the original number.
Solution :
An equation satisfied by all values of the variable is called Let the numerator be x.
an identity. For example : 2x + x = 3x. Then, denominator = x + 4
x 4 1
4 3 x 4 2 6
Example 1 : Solve 5
x 2x x 4 1
Solution : x 6 6
4 3 8 3 6 (x – 4) = x + 6
5 5
x 2x 2x 6x – 24 = x + 6 5x = 30
x= 6
5 Thus, Numerator = 6, Denominator = 6 + 4 = 10.
5 10x = 5
2x
6
5 1 Hence the original number = .
x 10
10 2 Linear equation in two variables : General equation of a
APPLICATIONS OF LINEAR EQUATIONS WITH ONE linear equation in two variables is ax + by + c = 0, where a, b 0
VARIABLES and c is a constant, and x and y are the two variables.
Example 2 : The sets of values of x and y satisfying any equation are
The sum of the digits of a two digit number is 16. If the called its solution(s).
number formed by reversing the digits is less than the Consider the equation 2x + y = 4. Now, if we substi tut e x = –
original number by 18. Find the original number. 2 in the equat ion , we obta in 2.(–2) + y = 4 or – 4 + y = 4
Solution : or y = 8. Hence (–2, 8) is a solution. If we substitute x = 3 in
the equation, we obtain 2.3 + y = 4 or 6 + y = 4 or y = –2
Let unit digit be x. Hence (3, –2) is a solution. The following table lists six
Then tens digit = 16 – x possible values for x and the corresponding values for y, i.e.
Original number = 10 × (16 – x) + x = 160 – 9x. six solutions of the equation.
y
o
u
B-24 Algebraic Expressions and Inequalities

rs
The above equations are nothing else but equations of 2 lines.

m
x –2 –1 0 1 2 3

a
y 8 6 4 2 0 –2 Any pair (x, y) which satisfy both the equation is called a solution

h
b
If we plot the solutions of the equation 2x + y = 4 which appear to the above system of equations.

o
o
in the above table then we see that they all lie on the same line. S. No. Solutions Feature Example

b
We call this line the graph of the equation since it corresponds

.w
1. One Lines intersect (ii)
precisely to the solution set of the equation.

o
rd
p
(–2,8) (i)
y

re
at (1,–2) each other

s
(–1,6)

s
x + y = –1

.c
(0,4) x–y=3

o
m
(1,2) 2. No solution Lines are parallel
(ii) (i)
(2,0)
x

(3,–2) to each other


Example 4 : x + y = –1
Find the values of x and y which satisfy the equations : 2x + 2y = – 6
4x + 3y = 25 and x + 5y = 19. 3. Infinite Both the lines (i)
Solution :
Substitution Method : (ii)
4x + 3y = 25 ..... (i)
solutions coincide each
x + 5y = 19 ...... (ii)
– all points other
x = 19 – 5y lying on the x+y= 1
Substitute x = 19 – 5y in equation (i), we get given line 2x + 2y = 2
4 (19 – 5y) + 3y = 25
76 – 20y + 3y = 25 76 – 17y = 25 SYSTEMS OF LINEAR EQUATION
Consistent System : A system (of 2 or 3 or more equations taken
17y = 51 y= 3
together) of linear equations is said to be consistent, if it has at
Putting y = 3 in equation (ii), we obtain least one solution.
x + 5 × 3 = 19 Inconsistent System: A system of simultaneous linear equations
x + 15 = 19 x=4 is said to be inconsistent, if it has no solutions at all.
x = 4 and y = 3 is the solution. e.g. X + Y = 9; 3X + 3Y = 8
Elimination Method : Clearly there are no values of X & Y which simultaneously satisfy
4x + 3y = 25 .......(i) the given equations. So the system is inconsistent.
x + 5y = 19 .......(ii)
Multiply equation (ii) by 4 on both sides, we find
4x + 20y = 76 .......(iii)
Subtracting equation (i) from equation (iii), we have The system a1x + b1y = c1 and a2x + b2y = c2 has :
4x + 20y = 76 a1 b1
4x + 3y = 25 • a unique solution, if .
a2 b2
– – –
––––––––––––– a1 b1 c1
• Infinitely many solutions, if .
17 y = 51 a2 b2 c2
51 a1 b1 c1
y= =3 • No solution, if .
17 a2 b2 c2
Substituting value of y in equation (i), we get The homogeneous system a1x + b1y = 0 and
4x + 3 × 3 = 25 a1 b1
a2x + b2y = 0 has the only solution x = y = 0 when .
16 a2 b2
4x = 16 x= =4
4 The homogeneous system a1x + b1y = 0 and
x = 4 and y = 3 is the solution. a1 b1
a2x + b2y = 0 has a non-zero solution only when ,
a2 b2
Now consider two linear equations in two unknowns,
and in this case, the system has an infinite number of
a1x + b1y = c1 .......... (i)
solutions.
a2x + b2y = c2 .......... (ii)
y
o
u
Algebraic Expressions and Inequalities B-25

rs
Example 5 :

m
STEP I : Express the equation in the standard form,

a
Find k for which the system 6x – 2y = 3, kx – y = 2 has a i.e. ax2 + bx + c = 0.

h
b
unique solution. STEP II : Factorise the expression ax2 + bx + c.

o
o
Solution : STEP III : Put each of the factors equal to zero and find

b
.w
The given system will have a unique solution, the values of x.
These values of x are solutions or roots of the quadratic

o
a1 b1 6 2

rd
if i.e. or k 3. equation.

p
a2 b2 k 1

re
1 1

s
Example 8 : Solve x 1

s
x 2

.c
Example 6 :

o
Solution :

m
What is the value of k for which the system
x + 2y = 3, 5x + ky = –7 is inconsistent?
x
1
1
1 x2 1 3
Solution : x 2 x 2
a1 b1 c1 2(x 2 1) 3x 2x2 – 2 = 3x
The given system will be inconsistent if
a2 b2 c2
2x2 – 3x – 2 = 0
2x2 – 4x + x – 2 = 0
1 2 3
i.e. if or k = 10. 2x (x – 2) +1 (x – 2) = 0
5 k 7
(2x + 1) (x – 2) = 0
Example 7 :
Either 2x + 1 = 0 or x – 2 = 0
Find k such that the system 3x + 5y = 0, kx + 10y = 0 has a
2x = – 1 or x = 2
non-zero solution.
Solution : 1
x or x = 2
The given system has a non zero solution, 2

3 5 1
if or k = 6 x , 2 are solutions.
k 10 2
QUADRATIC EQUATION Nature of Roots : The nature of roots of the equation depends
upon the nature of its discriminant D.
An equation of the degree two of one variable is called
1. If D < 0, then the roots are non-real complex, Such roots are
quadratic equation.
always conjugate to one another. That is, if one root is p +
General form : ax2 + bx + c = 0...........(1) where a, b and c
iq then other is p – iq, q 0.
are all real number and a 0. 2. If D = 0, then the roots are real and equal. Each root of the
For Example :
b
2x2 – 5x + 3 = 0; 2x2 – 5 = 0; x2 + 3x = 0 equation becomes . Equal roots are referred as
2a
A quadratic equation gives two and only two values of the un-
known variable and both these values are called the roots of the repeated roots or double roots also.
equation. 3. If D > 0 then the roots are real and unequal.
4. In particular, if a, b, c are rational number, D > 0 and D is a
The roots of the quadratic equation (1) can be evaluated using
perfect square, then the roots of the equation are rational
the following formula.
number and unequal.
5. If a, b, c, are rational number, D > 0 but D is not a perfect
b b2 4ac square, then the roots of the equation are irrational (surd).
x ...(2)
2a Surd roots are always conjugate to one another, that is if
The above formula provides both the roots of the quadratic one root is p q . then the other is p q , q > 0.
equation, which are generally denoted by and ,
6. If a = 1, b and c are integers, D > 0 and perfect square, then
the roots of the equation are integers.
b b2 4ac b b 2 4ac
say and Sign of Roots : Let are real roots of the quadratic equation
2a 2a
ax2 + bx + c = 0 that is D = b 2 4 ac 0 . Then
2
The expression inside the square root b 4ac is called 1. Both the roots are positive if a and c have the same sign and
the DISCRIMINANT of the quadratic equation and denoted the sign of b is opposite.
by D. Thus, Discriminant (D) = b2 – 4ac. 2. Both the roots are negative if a, b and c all have the same
Solving a quadratic equation by Factorisation sign.
y
o
u
B-26 Algebraic Expressions and Inequalities

rs
m
3. The Roots have opposite sign if sign of a and c are opposite.
NOTE :

a
4. The Roots are equal in magnitude and opposite in sign if

h
b
b = 0 [that is its roots and – ] 1. Above relations hold for any quadratic equation whether

o
the coefficients are real or non-real complex.

o
5. The roots are reciprocal if a = c.

b
2. With the help of above relations many other symmetric

.w
1 functions of and can be expressed in terms of the

o
[that is the roots are and ] coefficients a, b and c.

rd
3. Recurrence Relation

p
re
6. If c = 0. then one root is zero.
n n n 2 n 2
) ( n 1 n 1) ( )

s
7. If b = c = 0, then both the roots are zero. (

s
.c
8. If a = 0, then one root is infinite. 4. Some symmetric functions of roots are

o
m
2 2
9. If a = b = 0, then both the roots are infinite. (i) ( )2 2
10. If a = b = c = 0, then the equation becomes an identity
(ii) ( )2 4
11. If a + b + c = 0 then one root is always unity and the other
2 2
c (iii) ( )( ) ( ) ( )2 4
root is, Hence the roots are rational provided a, b, c, are
a 3 3
(iv) ( )3 3 ( )
rational.
3 3
Example 9 : (v) ( – )3 3 ( )&

Find solutions of the equation 25 x 2 x 1 ( )2 4


Solution :
4 4
(vi) ( 2 2 2
) 2 2 2
25 x 2 x 1 2
( )2 2 2( )2
or 25 x 2 2 or 25 x 2 2
( x 1) x 1 2x
(vii) 4 4 2 2 2 2
( )( )
or 2 x 2 2 x 24 0 or x 2 x 12 0
or ( x 4)(x 3) 0 or x = 4, x = – 3 ( )2 2 ( )2 4
Example 10 :
If 2x 2 7xy 3y 2 0 , then what is the value of x : y ? FORMATION OF QUADRATIC EQUATION
Solution : WITH GIVEN ROOTS :
An equation whose roots are and can be written as
2x 2 7xy 3y 2 0 (x – ) (x – ) = 0 or x2 – ( )x+ = 0 or x2 – (sum of
the roots) x + product of the roots = 0.
2
x x Further if and are the roots of a quadratic equation
2 7 3 0
y y ax2 + bx + c = 0, then
ax2 + bx + c = a(x – ) (x – ) is an identity.
A number of relations between the roots can be derived
x b b2 4ac 7 49 24 7 5 1 using this identity by substituting suitable values of x real
3,
y 2a 2 2 4 2 or imaginary.
Condition of a Common Root between two quadratic equations :
x 3 x 1 Consider two quadratic equations
or
y 1 y 2
a 1x 2 b1 x c1 0 ...(i)
Symmetric Functions of Roots : An expression in , is called a
and a 2x 2 b2x c2 0 ...(ii)
symmetric function of , if the function is not affected by
interchanging and . If are the roots of the quadratic Let be a common root of the two equations

equation ax 2 bx c 0, a 0 then, Then a1 2 b1 c1 0 and a 2 2


b2 c2 0
On solving we get
b coefficien t of x
Sum of roots : 2
1
a coefficien t of x 2
b1c 2 b 2 c1 c1a 2 c 2a1 a1b 2 a 2 b1
c constant term
and Product of roots : 2
a coefficien t of x 2 2 b1c 2 b 2 c1 c1a 2 c 2 a1
or
a 1b 2 a 2 b1 a1b 2 a 2 b1
y
o
u
Algebraic Expressions and Inequalities B-27

rs
Solution :

m
Which gives the common root as well as the condition for common

a
root. a + b = 24 and a – b = 8

h
Condition that two quadratic equations have both the Roots

b
a = 16 and b = 8 ab = 16 × 8 = 128

o
Common :

o
A quadratic equation with roots a and b is

b
Suppose that the equations a 1x 2

.w
b1 x c1 0 and
x 2 – (a + b) x + ab = 0 or x 2 24 x 128 0

o
a 2x 2

rd
b2x c2 0 have both the roots common. Inequations : A statement or equation which states that

p
one thing is not equal to another, is called an inequation.

re
a1 b1 c1
then

s
Symbols :

s
a2 b2 c2

.c
‘<’ means “is less than”

o
If the coefficients of two quadratic equations are rational (real)

m
and they have one irrational (imaginary) root common then they ‘>’ means “is greater than”
must have both the roots common as such roots occur in ‘ ’ means “is less than or equal to”
conjugate pair.
Example 11 : ‘ ’ means “is greater than or equal to”
For example :
If a and b are the roots of the equation x 2 6 x 6 0, (a) x < 3 means x is less than 3.
then the value of a 2 b 2 is : (b) y 9 means y is greater than or equal to 9.
Solution :
The sum of roots = a + b = 6
Product of roots = ab = 6 PROPERTIES
1. Adding the same number to each side of an equation
Now, a 2 b2 (a b) 2 2ab = 36 – 12 = 24 does not effect the sign of inequality, it remains same, i.e.
Example 12 : if x > y then, x a y a .
If and are the roots of the quadratic equation 2. Subtracting the same number to each side of an inequation
2 2
does not effect the sign of inequaltiy, i.e., if x < y then,
ax 2 bx c 0 . Then find the value of in terms x – a < y – a.
3. Multiplying each side of an inequality with same number
of a, b and c. does not effect the sign of inequality, i.e., if x y then
Solution :
ax ay (where, a > 0).
b c
Here, and 4. Multiplying each side of an inequality with a negative
a a number effects the sign of inequality or sign of inequality
2 2 3 3 reverses, i.e., if x < y then ax > ay (where a < 0).
Thus, 5. Dividing each side of an inequation by a positive number
does not effect the sign of inequality, i.e., if x y then
2 2
( ) ( ) .......... (i) x y
a a (where a > 0).
2 2
Now, ( ) [( )2 2 ] 6. Dividing each side of an inequation by a negative
number reverses the sign of inequality, i.e., if x > y then
= [( )2 3 ]
x y
Hence (i) becomes (where a < 0).
a a
b b2 3c NOTE :
( )[( ) 2
3 )] a a2 a
= If a > b and a, b, n are positive, then an > bn but a–n < b–n.
c
For example 5 > 4; then 53 > 43 or 125 > 64, but
a
1
b b2 3ac 3abc b3 5–3 < 4–3 or .
125 64
=
c a2 a2c If a > b and c > d, then (a + c) > (b + d).
Example 13 : If a > b > 0 and c > d > 0, then ac > bd.
If a, b are the two roots of a quadratic equation such that If the signs of all the terms of an inequality are changed,
a + b = 24 and a – b = 8, then find a quadratic equation then the sign of the inequality will also be reversed.
having a and b as its roots.
y
o
u
B-28 Algebraic Expressions and Inequalities

rs
MODULUS :

m
Solution :

a
Let the present age of Kareem’s son be x years.

h
x, x 0 Then, Kareem’s age = 3x years

b
|x|

o
x, x 0 After 10 years, Kareem’s age = 3x + 10 years

o
b
and Kareem’s son’s age = x + 10 years

.w
1. If a is positive real number, x and y be the fixed real numbers,
then (3x + 10) + (x + 10) = 76

o
rd
(i) | x – y | < a y–a< x<y+a 4x = 56 x = 14

p
(ii) | x – y | a y– a x y+a Kareem’s present age = 3x = 3 × 14 = 42 years

re
Kareem’s son’s age = x = 14 years.

s
(iii) | x – y | > a x > y + a or x < y – a (iv) | x – y | a

s
.c
Example 18 :
x y + a or x y – a

o
The present ages of Vikas and Vishal are in the ratio 15 : 8.

m
After ten years, their ages will be in the ratio 5 : 3. Find their
2. Triangle inequality :
present ages.
(i) | x y | | x | | y |, x, y R Solution :
(ii) | x y | | x | | y |, x, y R Let the present ages of Vikas and Vishal be 15x years and 8x
years.
After 10 years,
Example 14 :
Vikas’s age = 15x + 10 and
If a – 8 = b, then determine the value of | a b | | b a | . Vishal’s age = 8x + 10
(a) 16 (b) 0 (c) 4 (d) 2 15x 10 5
Solution :
8x 10 3
(b) | a b | | 8 | 8
3(15x + 10) = 5(8x + 10)
|b a | | 8| 8 |a b| |b a | 8 8 0 45x + 30 = 40x + 50
20
Example 15 : Solve : 3x 4 19, x N 5x = 20 x= =4
5
Solution :
Present age of Vikas = 15x = 15 × 4 = 60 years
3x 4 19 Present age of Vishal = 8x = 8 × 4 = 32 years.
3x 4 4 19 4 [Subtracting 4 from both the sides]
3x 15 Example 19 :
Father’s age is 4 less than five times the age of his son and
3x 15 the product of their ages is 288. Find the father’s age.
[Dividing both the sides by 3]
3 3 Solution :
x 5; x N Let the son’s age be x years.
x = {1, 2, 3, 4, 5}. So, father’s age = 5x – 4 years.
x(5x – 4) = 288
Example 16 : Solve 5 2x 1 11 5x2 – 4x – 288 = 0 5x2 – 40x + 36x – 288 = 0
Solution : 5x (x – 8) + 36 (x – 8) = 0
5 2x 1 11 (5x + 36) (x – 8) = 0
5 + 1 2x – 1 + 1 11 + 1 36
[Adding 1 to each sides] Either x – 8 = 0 or 5x + 36 = 0 x = 8 or x =
5
6 2x 12
x cannot be negative; therefore, x = 8 is the solution.
6 2x 12 Son’s age = 8 years and Father’s age = 5x – 4 = 36 years.
[Dividing each side by 2]
2 2 2
If present age of the father is F times the age of his son. T
3 x 6
years later, the father’s age become Z times the age of son
x = {3, 4, 5, 6}.
APPLICATIONS (Z 1)T
then present age of his son is given by
Problems on Ages can be solved by linear equations in one (F Z)
variable, linear equations in two variables, and quadratic Example 20 :
equations.
Present age of the father is 9 times the age of his son. One
Example 17 :
year later, father’s age become 7 times the age of his son.
Kareem is three times as old as his son. After ten years, the
sum of their ages will be 76 years. Find their present ages. What are the present ages of the father and his son.
y
o
u
Algebraic Expressions and Inequalities B-29

rs
m
Solution : NOTE :

a
By the formula

h
b
IMPORTANTALGEBRAIC FORMULAE

o
(7 1) 6

o
Son’s age = 1= 1 = 3 years. 1 (a + b)2 = a2 + 2ab + b2

b
2

.w
(9 7)
2. (a – b)2 = a2 – 2ab + b2

o
So, father’s age = 9 × son’s age = 9 × 3 = 27 years.

rd
3. (a – b) (a + b) = a2 – b2

p
re
4. (a + b)2 + (a – b)2 = 2(a2 + b2)

s
If T1 years earlier the age of the father was n times the age of

s
5. (a + b)2 – (a – b)2 = 4ab

.c
his son, T2 years later, the age of the father becomes m

o
times the age of his son then his son’s age is given by (a + b)3 = a3 + b3 + 3ab (a + b)

m
6.

T (n 1) T1 (m 1) 7. (a – b)3 = a3 – b3 – 3ab (a – b)
Son’s age = 2
n m 8. a3 + b3 = (a + b) (a2 – ab + b2)
Example 21 : 9. a3 – b3 = (a – b) (a2 + ab + b2)
10 years ago, Shakti’s mother was 4 times older than her. 10. (a + b + c)2 = (a2 + b2 + c2) + 2 (ab + bc + ca)
After 10 years, the mother will be twice older than the
11. a3 + b3 + c3 – 3abc = (a + b + c) (a2 + b2 +c2 –ab –bc –ca)
daughter. What is the present age of Shakti?
12. a4 – b4 = (a2 – b2) (a2 + b2)
Solution :
By using formula, b e h k b e h k
13. a + d + g – j = (a + d + g – j) + + + –
10(4 1) 10(2 1) c f i 1 c f i 1
Shakti’s age = = 20 years.
4 2
IMPORTANT SERIES TYPE FORMULA

Present age of Father : Son = a : b 4P 1 1


After / Before T years = m : n (i) Value of P P p .....
2

Then son’s age = b × T(m n) 4P 1 1


an bm (ii) Value of P P p .....
2
T(m n)
and Father’s age = a × (iii) Value of
an bm P. P. p...... P
+ve for after & –ve for before
n 1) 2n
(iv) Value of P P P P P P (2
Example 22 :
The ratio of the ages of the father and the son at present is where n no. of times P repeated.
3 : 1. Four years earliar, the ratio was 4 : 1. What are the
SOME SPECIAL SERIES
present ages of the son and the father?
Solution : (n)( n 1)
Ratio of present age of Father and Son = 3 : 1 (i) Sum of first n natural numbers 1 + 2 + 3 + ... n
2
4 years before = 4 : 1 (ii) Sum of the squares of first n natural numbers
4(4 1) (n)( n 1)(2n 1)
Son’s age = 1 = 12 years. 12 + 22 + 32 + ... b2
4 1 3 1 6
(iii) Sum of the cubes of first n natural numbers
4(4 1) (n)(n 1)
2
Father’s age = 3 = 36 years. 13 + 23 + 33 + ... n3 =
4 1 3 1 2
y
o
u
B-30 Algebraic Expressions and Inequalities

rs
m
a
h
b
o
o
b
.w
o
1. Present ages of Sameer and Anand are in of the ratio of 5:4 of age when I was born. If my sister was 4 years of age

rd
respectively. Three years later, the ratio of their ages will when my brother was born, then, what was the age of my

p
become 11:9 respectively. What is Anand’s present age in

re
father and mother respectively when my brother was born?

s
years? (a) 32 yrs, 23 yrs (b) 32 yrs, 29 yrs

s
.c
(a) 24 (b) 27 (c) 35 yrs, 29 yrs (d) 35 yrs, 33 yrs

o
(c) 40 (d) cannot be determined

m
10. Father is aged three times more than his son Ronit. After 8
2. What fraction is that whose numerator being doubled and years, he would be two and a half times of Ronit’s age. After
2 further 8 years, how many times would he be of Ronit’s age?
denominator increased by 7, the value becomes , but when
3 1
the denominator being doubled, and the numerator increased (a) 2 times (b) 2 times
2
3
by 2, the value becomes ? 3
5 2
(c) (d) 3 times
(a) 3/5 (b) 4/5 (c) 5/7 (d) 7/11 4

6
11. An employer engaged a servant with free boarding and
3. Rajan got married 8 years ago. His present age is times lodging for one year with the condition that the servant will
5
be given ` 2500 and a uniform at the end of the year. The
his age at the time of his marriage. Rajan’s sister was 10 servant agreed but served the employer only for 10 months
years younger to him at the time of his marriage. The age of and thus received ` 2000 and a uniform. The price of the
Rajan’s sister is: uniform is:
(a) 32 years (b) 36 years (a) ` 250 (b) ` 350 (c) ` 400 (d) ` 500
(c) 38 years (d) 40 years 12. If x 0, y 0 and (x + y) 1, then the maximum value of
4. A father said to his son, “I was as old as you are at present (2x + 3y) is
at the time of your birth.” If the father’s age is 38 years now,
(a) 2 (b) 3 (c) 4 (d) 5
the son’s age five years back was:
(a) 14 years (b) 19 years 1
13. If x 2 x 2
0 , then which of the following is true?
(c) 33 years (d) 38 years x x
5. A person was asked to state his age in years. His reply was,
“Take my age three years later, multiply it by 3 and then 1 1
(a) x+ > –2 (b) x+ <–2
subtract three times my age three years ago and you will x x
know how old I am.” What was the age of the person?
1
(a) 18 years (b) 20 years (c) x+ <1 (d) Both (a) and (c)
x
(c) 24 years (d) 32 years
14. If the roots of the quadratic equation 3x2 – 5x + p = 0 are real
6. Tanya’s grandfather was 8 times older to her 16 years ago.
and unequal, then which one of the following is correct?
He would be 3 times of her age after 8 years from now. Eight
years ago, what was the ratio of Tanya’s age to that of her (a) p = 25/12 (b) p < 25/12
grandfather? (c) p > 25/12 (d) p 25/12
(a) 1 : 2 (b) 1 : 5 15. For what value of k, do the equations 3x – y + 8 = 0 and 6x –
ky = – 16 represent coincident lines?
(c) 3 : 8 (d) None of these
7. In a two-digit number, if it is known that its unit’s digit exceeds 1 1
its ten’s digit by 2 and that the product of the given number (a) (b)
2 2
and the sum of its digits is equal to 144, then the number is:
(c) 2 (d) –2
(a) 24 (b) 26 (c) 42 (d) 46
16. The pair of linear equations 13x + ky = k and 39x + 6y = k + 4
8. Ayesha’s father was 38 years of age when she was born
has infinitely many solutions, if k =
while her mother was 36 years old when her brother four
(a) 1 (b) 2
years younger to her was born. What is the difference
between the ages of her parents? (c) 4 (d) 6
(a) 2 years (b) 4 years 17. The value of k for which the system of equations 3x + 5y
(c) 6 years (d) 8 years = 0 and kx + 10y = 0 has a non-zero solution is
9 My brother is 3 years elder to me. My father was 28 years of (a) 0 (b) 2
age when my sister was born while my mother was 26 years (c) 6 (d) 8
y
o
u
Algebraic Expressions and Inequalities B-31

rs
18. If the system of equations

m
28. If a x b , b y c and cz = a; then the value of xyz =

a
3x + y = 1

h
(a) 0 (b) 1

b
(2k – 1) x + (k – 1) y = 2k + 1

o
(c) – 1 (d) 2

o
in inconsistent, then k =

b
(a) 1 (b) –1

.w
1 3 1
(c) –2 (d) 2 29. If x 5 , then the value of x is :

o
x x3

rd
19. The value of k for which

p
x + 2y + 7 = 0 (a) 125 (b) 110

re
(c) 45 (d) 75

s
2x + ky + 14 = 0

s
.c
and
x2 7 y 2 9z 2

o
represent coincident lines is

m
30. If x : y : z :: 1 : 3 : 5, then the value of is :
(a) 3 (b) 4 x
(c) –4 (d) –3 (a) 7 (b) 17
20. If a2 + b2 + c2 = 2a – 2b – 2, then the value of 3a – 2b + c is (c) 13 (d) 1
(a) 0 (b) 3
(c) 5 (d) 2 31. If (x + 1) is a factor of 2x 3 ax 2 (2a 3) x 2 , then the
1 1 1 value of ‘a’ is
21. If a + b + c = 3, a2+ b2 + c2 = 6 and = 1, where a,
a b c 3 1
b, c are all non-zero, then 'abc' is equal to (a) 3 (b) 2 (c) (d)
2 2
2 3
(a) (b) 4ab x 2a x 2b
3 2 32. If x , then the value of is equal to:
a b x 2a x 2b
1 1
(c) (d) (a) 0 (b) 1
2 3
(c) 2 (d) None of these
1 3
22. If a2 – 4a – 1 = 0, a 0, then the value of a2 + 3a + 2 – is x y z
a a 33. If a,
(2x y z ) ( x 2 y z) ( x y 2z )
(a) 24 (b) 26
(c) 28 (d) 30 then find ‘a’ if x y z 0
1
23. If x = 2 + 2 1 1
3 , then x + is equal to
x2 (a) (b)
3 4
(a) 10 (b) 12
(c) –12 (d) 14 1 1
24. If a = 4.965, b = 2.343 and c = 2.622, then the value of (c) (d)
8 2
a3 – b3 – c3 – 3 abc is
(a) –2 (b) –1
34. The least value of 3 x 3 x
is :
(c) 0 (d) 9.932
(a) 2 (b) 1
x2 y2 z2
25. If x + y + z = 0, then the value of is 2
x2 yz (c) 0 (d)
3
(a) –1 (b) 0
(c) 1 (d) 2 35. If a + b + c = 0, then find the value of
26. If a = 6 2 2
5 b= 6 5 then 2a – 5ab + 2b = a2 b2 c2
(a) 38 (b) 39 .
a2 bc b2 ca c2 ab
(c) 40 (d) 41
(a) 0 (b) 1
5
27. If p = (c) 6 (d) None of these
18
1 9 2 1 1
then 27 p3 – p p is equal to 36. If x = 2 then the value of x is
216 2 4 1
2
1
4 5 2
(a) (b) 2 ........
27 27
8 10 (a) 2 –1 (b) 2 +1
(c) (d)
27 27 (c) 3 (d) None
y
o
u
B-32 Algebraic Expressions and Inequalities

rs
m
47. If x = b + c – 2a, y = c + a – 2b, z = a + b – 2c, then the value
37.

a
6 6 6 .......... ? of x2 + y2 – z2 + 2xy is

h
b
(a) 2.3 (b) 3 (a) 0 (b) a + b + c

o
(c) 6 (d) 6.3

o
(c) a – b + c (d) a + b – c

b
.w
y2 x2 48. (y – z)3 + (z – x)3 + (x – y)3 is equal to
38. 1.5x = 0.04y, then value of is

o
y2 2 xy x2 (a) 3(y – z) (z + x) (y – x) (b) (x – y) (y + z) (x – z)

rd
(c) 3(y – z) (z – x) (x – y) (d) (y – z) (z – x) (x – y)

p
730 73

re
(a) (b) 1 1 1

s
77 77

s
49. If 2x = 3y = 6z then , is equal to

.c
x y z

o
73 74

m
(c) (d) (a) 0 (b) 1
770 77
39. If a2 = b + c, b2 = c + a, c2 = a + b , then the value of 3 1
(c) (d)
1 1 1 2 2
1 a b 1 1 c 50. If x2 + y2 + z2 + 2 = 2(y – x), then value of x3 + y3 + z3 is equal
(a) abc (b) a2 b2 c2 to
(c) 1 (d) 0 (a) 0 (b) 1
a b c (c) 2 (d) 3
40. 1 , then find the value of
1 a 1 b 1 c 51. If ax = (x + y + z)y , ay = (x + y + z)z and az = (x + y + z)x, then
the value of x + y + z (given a 0) is
1 1 1
(a) 0 (b) a3
1 a 1 b 1 c
(c) 1 (d) a
(a) 1 (b) 2
(c) 3 (d) 4 1
52. For what value (s) of a is x + x + a2 a perfect square?
1 4
41. If m 4 then, what is value of
m 2 1 1
(a) ± (b) ±
1 18 8
(m – 2)2 + ?
( m 2) 2 1 1
(c) – (d)
(a) – 2 (b) 0 5 4
(c) 2 (d) 4 53. If x2 + y2 + 2x + 1 = 0, then the value of x 31 + y35 is
1 1 (a) – 1 (b) 0
42. If x + = 1 and y + = 1, what is the value of xyz? (c) 1 (d) 2
y z
(a) 1 (b) – 1 54. The value of the expression x4 – 17x3 + 17x2 – 17x + 17 at
x = 16 is
1 (a) 0 (b) 1
(c) 0 (d)
2 (c) 2 (d) 3
43. x+y x–y
If 3 = 81 and 81 = 3, then the value of x is 55. If x + 2 = 2x, then the value of x4 – x3 + x2 + 2 is
2

15 (a) 1 (b) 0
(a) 42 (b) (c) – 1 (d)
8 2
17 1
(c) (d) 39 56. If nr tn be a perfect square, then the values of
8 4
t are :
44. If p = 999, then the value of 3 p(p 2 3p 3) 1 is
(a) ± 2 (b) 1, 2
(a) 1000 (b) 999 (c) 2, 3 (d) ± 1
(c) 998 (d) 1002 57. The values of a and b (a > 0, b < 0) for which 8x3 – ax2 + 54x
45. If px = ry = m and rw = pz = n, then which one of the following + b be a perfect cube are
is correct? (a) a = 12, b = – 9
(a) xw = yz (b) xz = yw (b) a = 36, b = – 27
(c) x + y = w + z (d) x – y = w – z (c) a = 18, b = – 27
46. If x2 + y2 – 4x + 4y + 8 = 0, then the value of x – y is (d) a = 16, b = – 9
(a) 4 (b) – 4 58. When x5 + 1 is divided by (x – 2), the remainder is:
(a) 15 (b) 17
(c) 0 (d) 8
(c) 31 (d) 33
y
o
u
Algebraic Expressions and Inequalities B-33

rs
m
1 1 69. If a + b + c = 2s then find the value of
(s – a)3 + (s – b)3 + 3 (s – a) (s – b)c is

a
59. If (3.7)x = (0.037)y = 10000, then what is the value of x ?

h
y (a) c (b) c2

b
o
(a) 1 (b) 2 (c) c3 (d) 2c2

o
The expression x4 – 2x2 + k will be a perfect square when the

b
70.
1 1

.w
(c) (d) value of k is
2 4

o
(a) 1 (b) 2

rd
60. If x2 + y2 + z2 = xy + yz + zx, (x 0), then the value of

p
1 1

re
4x 2y 3z (c) (d)
is 2 4

s
s
2x

.c
(a) 0 (b) 1 1 1

o
71. If 3x – = 6, then the value of 4x – is

m
3 1 4y 3y
(c) (d) (a) 2 (b) 4
2 2
(c) 6 (d) 8
a6 a4 a2 1
61. If a = 3 + 2 2 , then the value of is a b 2b b c
a3 72. If a + b + c = 0, find the value of .
c c a a
(a) 192 (b) 240
(a) 0 (b) l
(c) 204 (d) 212
(c) – 1 (d) 2
x z
62. If x + y = 2z then the value of is 4 4
x z y z 73. If x + = 4, find the value of x3 .
x x3
(a) 1 (b) 3
1 1
(c) (d) 2 (a) 8 (b) 8
2 2

1 1
x4 (c) 16 (d) 16
2
63. If x2 + 1 = 2x, then the value of x 2 is
2
x 3x 1 1
74. If x = 3 + 2 2 , then the value of x is
(a) 0 (b) 1 x
(c) 2 (d) – 2
(a) 1 (b) 2
x a x 3a
64. If Find the value of x. (c) 2 2 (D) 3 3
x b x a b
(a) a + 2b 1
75. If ‘a’ be a positive number, then the least value of a + is
(b) a – 2b a
(c) a + b (a) 1 (b) 0
(d) a – b
1
p q r (c) 2 (d)
65. If , then p + q + r = ? 2
b c c a a b 76. If a = 0, b 0, c 0, then the equation ax + by + c = 0
(a) 0 (b) 1 represents a line parallel to
(c) – 1 (d) – 2
(a) x + y = 0 (b) x-axis
1 (c) y-axis (d) none of these
66. If x + = 3 , then the value of x18 + x12 + x6 + 1 is
x 77. If x = 997, y = 998, z = 999, then the value of x2 + y2 + z2
(a) 0 (b) 1 – xy – yz – zx will be
(c) 2 (d) 3 (a) 3 (b) 9
1 (c) 16 (d) 4
1
67. If x4 + 4 = 119 amd x > 1, then the value of x3 – is 78. If a + b + c = 8, then the value of
x x3 (a – 4)3 + (b – 3)3 + (c – 1)3 – 3 (a – 4) (b – 3) (c – 1) is
(a) 54 (b) 18 (a) 2 (b) 4
(c) 72 (d) 36 (c) 1 (d) 0
1 3 3 1 1
68. If a 4 b 2 c and a, b, c are rational
3
4 32 1 79. If x a , y a , th en the value of
a a
numbers, then a + b + c is equal to 4 4 2 2
x + y – 2x y is
(a) 0 (b) 1 (a) 16 (b) 20
(c) 2 (d) 3 (c) 10 (d) 5
y
o
u
B-34 Algebraic Expressions and Inequalities

rs
m
(a) 0.2695 (b) 0.3695
1 2 1

a
80. If 5a 5 , then the value of 9a is (c) 2.695 (d) 3.695

h
3a 25a 2

b
1 1

o
If x = 2, then x2013 + 2014 = ?

o
51 29 85.
x

b
(a) (b) x

.w
5 5 (a) 0 (b) 1

o
(c) –1 (d) 2

rd
52 39
(c) (d) 86. If a = 331, b = 336 and c = – 667, then the value of a3 + b3 +

p
5 5

re
c3 – 3abc is

s
81. If a b c 0 , the value of (a) 1 (b) 6

s
.c
(c) 3 (d) 0

o
a2 b2 c2

m
bc ca ab is
87. The next term of the sequence,
(a) 2 (b) 3 1 1 1 1 1 1
1 , 1 1 , 1 1 1 , __ is
(c) 4 (d) 5 2 2 3 2 3 4
82. If a, b, c are real and a3 + b3 + c3 = 3abc and a b c 0,
then the relation between a, b, c will be 1
(a) 3 (b) 1
(a) a + b = c (b) a + c = b 5
(c) a = b = c (d) b + c = a
1 1
83. What would be the sum of (c) 5 (d) 1 1
1 + 3 + 5 + 7 + 9 + 11 + 13 + 15 + ....... up to 15th term? 2 5
(a) 250 (b) 240 88. The total area (in sq. unit) of the triangles formed by the
(c) 225 (d) 265 graph of 4x + 5y = 40, x-axis, y-axis and x = 5 and y = 4 is
84. If 13 + 23 + ....... + 93 = 2025, then the approx. value of (a) 10 (b) 20
(0.11)3 + (0.22)3 + ....... + (0.99)3 is (c) 30 (d) 40

ANSWER KEY
1 (a) 11 (d) 21 (b) 31 (a) 41 (c) 51 (d) 61 (c) 71 (d) 81 (b)
2 (b) 12 (b) 22 (d) 32 (c) 42 (b) 52 (b) 62 (a) 72 (a) 82 (c)
3 (c) 13 (d) 23 (d) 33 (b) 43 (c) 53 (a) 63 (d) 73 (b) 83 (c)
4 (a) 14 (b) 24 (c) 34 (a) 44 (a) 54 (b) 64 (b) 74 (b) 84 (c)
5 (a) 15 (c) 25 (d) 35 (d) 45 (a) 55 (b) 65 (a) 75 (c) 85 (d)
6 (d) 16 (b) 26 (b) 36 (b) 46 (c) 56 (d) 66 (a) 76 (b) 86 (d)
7 (a) 17 (c) 27 (c) 37 (b) 47 (a) 57 (b) 67 (d) 77 (a) 87 (a)
8 (c) 18 (d) 28 (b) 38 (b) 48 (c) 58 (d) 68 (a) 78 (d) 88 (a)
9 (a) 19 (b) 29 (b) 39 (c) 49 (a) 59 (c) 69 (c) 79 (a)
10 (a) 20 (c) 30 (b) 40 (d) 50 (a) 60 (c) 70 (a) 80 (d)
y
o
u
Algebraic Expressions and Inequalities B-35

rs
m
a
h
b
o
o
b
.w
1. (a) Let the present ages of Sameer and Anand be 5x years 10. (a) Let Ronit’s present age be x years. Then father’s

o
rd
and 4x years respectively. present age = (x + 3x) years = 4x years.

p
5x 3 11

re
Then 9(5x 3) 11(4x 3) x 6 5
and 4x 8 (x 8) 8x 16 5x 40

s
4x 3 9

s
2

.c
Anand’s present age = 4x = 24 years.

o
x 2x 2 x+2 3 3x 24 x 8.

m
2. (b) If be a fraction, then and
y y 7 3 2y 5 (4x 16) 48
Solving for x and y, we have x = 4 and y = 5 Hence, required ratio 2.
(x 16) 24
4 11. (d) 12 months wages = ` 2500 + Uniform; and
Thus the fraction is .
5 10 months wages = ` 2000 + Uniform
3. (c) Let Rajan’s present age be x years. Then, his age at the 2 months wages = ` 500
time of marriage = (x – 8) years.
12 months wages = ` 3000.
6 cost of the uniform = ` 500.
x (x 8) 5x 6x 48 x 48.
5 12. (b) It is given that x 0, y 0 and x + y 1.
Rajan’s sister’s age at the time of his marriage x+y 1 2(x + y) 2 2x + 2y 2.
= (x – 8) – 10 = (x – 18) = 30 years. 2x + 2y + y 2 + y
Rajan’s sister’s present age = (30 + 8) years 2x + 3y 2 + 1 = 3. (since y 1).
= 38 years.
1 1
4. (a) Let the son’s present age be x years. Then, (38 – x) = x 13. (d) Given that x 2 x 0
2 x
2x = 38 x = 19. x
son’s age 5 years back = (19 – 5)years = 14 years.
2
5. (a) Let the present age of the person be x years. 1 1
Then, 3(x + 3) – 3(x – 3) = x (3x + 9) – (3x – 9) = x x x 2 0
x x
x = 18.
6. (d) 16 years ago, let T = x years and G = 8x years. 1
After 8 years from now, T = (x + 16 + 8) years and Substituting x = y, we get
x
G = (8x + 16 + 8) years.
y2 + y – 2 < 0
8x + 24 = 3(x + 24) 5x = 48
(y – 1) (y + 2) < 0
48 either y – 1 <0; y + 2 >0
8
T x 8 5 88 11 or y + 2 < 0; y – 1 > 0.
8 years ago,
G 8x 8 48
8 8 424 53 i.e., y < 1, y > – 2 or y < –2; y 1
5
7. (a) Let the ten digit be x. Then, unit’s digit = x + 2
Number = 10x + (x + 2) = 11x + 2 not possible
Sum of digits = x + (x + 2) = 2x + 2
Therefore, –2 < y < 1
(11x + 2) (2x + 2) = 144 22x2 + 26x – 140 = 0
(11x + 35) (x – 2) = 0 x = 2
i.e. –2< x
FG 1IJ
<1.
Hence required number = 24 H x K
8. (c) Mother’s age when Ayesha’s brother was born = 36 yrs.
14. (b) The given equation is,
Father’s age when Ayesha’s brother was born
3x2 – 5x + p = 0
= (38 + 4) years = 42 years.
Required difference = (42 – 36) years = 6 years. We have, a = 3, b = – 5, c = p
9. (a) Clearly, my brother was born 3 years before I was born D = b2 – 4ac = 25 – 12 p
and 4 years after my sister was born. For Real and unequal, D > 0
So, father’s age when brother was born = (28 + 4) years 25 – 12 p > 0
= 32 years; 25
mother’s age when brother was born = (26 – 3) years 25 > 12 p p
12
= 23 years.
y
o
u
B-36 Algebraic Expressions and Inequalities

rs
15. (c) For coincident lines

m
1 3

a
We have a2 + 3a + 2

h
3 1 8 a a

b
[since 6x – ky = – 16 6x – ky + 16 = 0]

o
6 k 16

o
1 1

b
a2 3 a

.w
1 1 2
k 2 (c) holds. a a

o
2 k

rd
16. (b) 2

p
1 1
a 3 a 2

re
17. (c) a a

s
s
18. (d) The system of equations is inconsistent

.c
42 + 3 × 4 + 2 = 30

o
3 1 1

m
if 3 k – 3 = 2 k – 1 and 23. (d) x= 2 3
2k 1 k 1 2k 1
1 1 2 3
2k + 1 1 = = 2 3
x 2 3 2 3
k=2 (d) is true and k 2 (c) is false
2
1 1
Also
3 1
6k + 3 2k – 1 4k – 4 x2 = x –2
2k 1 2k 1 x2 x
k –1 (b) is false 2
= 2 3 2 3 2
For k = 1, x = 3, y = 1 – 9 = – 8 i.e. x = 3, y = – 8
This means, equations are consistent. Thus (d) holds. = 16 – 2 = 14
24. (c) a = 4.965 5, b = 2.343 2
19. (b) The given lines represent coincident lines if
c = 2.622
1 2 7 1 2 1 a–b=c
2 k 14 2 k 2 taking cube both sides
k=4 (a) is false; (c) is false; (d) is false. (b) holds a3 – b3 – 3a2b + 3ab2 = c3
a3 – b3 – c3 – 3ab(a – b) = 0
a3 – b3 – c3 – 3abc = 0
20. (c) a2 + b2 + c2 = 2a – 2b – 2
25. (d) x + y+ z = 0
(a2 – 2a + 1) + (b2 + 2b + 1) + c2 = 0
y + z = –x
(a – 1)2 + (b + 1)2 + c2 = 0
y2 + z2 + 2yz = x2
This equation is possible if
y2 + z2 = x2 – 2yz ...(1)
a – 1 = 0, b + 1 = 0 and c = 0
a = 1, b = –1, c = 0 x2 y2 z2 x2 2yz x 2 2(x 2 yz)
3a – 2b + c = 3 × 1 – 2 × (–1) + 0 2 = 2 2
=3+ 2=5 x yz x yz x yz
21. (b) a+ b+c=3 = 2]
Squaring both sides
a2 + b2 + c2 + 2 (ab + bc + ac) = 9 26. (b) 2a2 – 5ab + 2b2 2(a2 – 2ab + b2) – ab
6 + 2 (ab + bc + ca) = 9 2 (a –b)2 – ab
3 2
ab + bc + ca = ...(1) 2 6 5 6 5 6 5 6 5
2
2 × 4 × 5 – 1 = 39
1 1 1
given =1 1 9 2 1
a b c 27. (c) 27p3 – p p
216 2 4
3
ab + bc + ac = abc = [from (1)] 3 2
2 1 1 1
= (3p)3 – – 3 (3p)2. + 3 (3p)
22. (d) a2 – 4a – 1 = 0 6 6 6
a2 – 4a = 1
a (a – 4) = 1 3
1
3P
1 6
a–4=
a
3
5 1 8
1 3
a =4 ...(1) 18 6 27
a
y
o
u
Algebraic Expressions and Inequalities B-37

rs
33. (b) Given,

m
28. (b) ax b, b y c and c z a

a
h
x y z

b
a x b y c z abc a

o
2x y z x 2y z x y 2z

o
On comparing the powers of a, b, c we get

b
.w
x = 1, y = 1 and z = 1 xyz = 1 x = a (2x + y + z), y = a (x + 2y + z) and

o
29. (b) Using (a + b )3 = a3 + b3 + 3ab(a + b) z = a (x + y + 2z)

rd
x + y + z = a (4x + 4y + 4z) = 4a(x + y + z)

p
3

re
1 1 1
x x3 3 x 1

s
x 3 x 4a = 1 or a.

s
x

.c
4

o
1 34. (a) Working with the options, for x = 0, the least value of

m
(5) 3 x3 15
3
x f (x) 3x 3 x
2
1 Alternate :
or x 3 125 15 110
x3 1 x
Let A = 3x and 3
x y z A
30. (b) Let k
1 3 5 Both are positive.
x = k, y = 3 k, z = 5k 1
Now, A.M. of A and is greater than or equal to
x 2
7y 2
9z 2 A
Now, their G.M.
x
1
k 2 7 9k 2 9 25k 2 A
A 1
i.e. A.
k 2 A
= 1 63 225 = 289 17 1
or A 2
31. (a) Let f ( x ) 2 x 3 ax 2 (2a 3) x 2 A
If (x + 1) is a factor of the above expression, then f(–1) = 0 or 3 x 3 x 2.
We have, 35. (d) Given, a + b + c = 0
3 2
f ( 1) 2( 1) a( 1) (2a 3) 1 2 0
a2 ( b c) 2
2 a 2a 3 2 0
or a–3=0 or a = 3 a2 b2 c2
Now, consider
4ab a2 bc b2 ca c 2 ab
32. (c) Given, x
a b
(b c) 2 b2 c2
x 2b =
a2 bc b2 ca c2 ab
2a a b
Applying componendo and dividendo, we get (b c) 2 b2 c2
x 2a 2b a b a 3b (b c)2 bc b2 c(b c) c2 b(b c)
…(i)
x 2a 2b a b b a
( b c) 2 b2 c2
x 2a
Also, b2 c2 bc b2 c2 bc c2 b2 bc
2b a b
Applying componendo and dividendo, we get b2 c2 2 bc b 2 c2 2( b 2 c2 bc)
2
2 2 2 2
x 2b 2a a b 3a b b c bc b c bc
…(ii)
x 2b 2a a b a b
1
Add (i) & (ii), 36. (b) We can write given expression as x = 2
x
x 2a x 2b a 3b 3a b
x 2a x 2b b a a b x2 – 2x – 1 = 0 x=1± 2

1 2(b a ) x>2 x=1+ 2


= [a 3b 3a b] 2
b a (b a )
y
o
u
B-38 Algebraic Expressions and Inequalities

rs
m
37. (b) a b c a b c

a
6 6 6 ............ = 1

h
a b c a b c a b c a b c

b
o
6 x x

o
6 + x = x2

b
40. (d) Given that,

.w
x2 – x – 6 = 0
a b c

o
x2 – 3x + 2x – 6 = 0 1

rd
(x – 3) (x + 2) = 0 1 a 1 b 1 c

p
re
x=3 a b c
1 1 1 4

s
By direct formula

s
1 a 1 b 1 c

.c
o
4 6 1 1 5 1 1 1 1

m
Here P = 6, S 3 4
2 2 1 a 1 b 1 c
x 0.04 4 2
38. (b) y 1.5 150 75 1
41. (c) m 4
m 2
y 75
m2 – 6m + 9 = 0
x 2 (m – 3) (m – 3) = 0
m=3
y2 x2
Now, m–2=1
y2 2 xy x 2
1
(y x)( y x) Now (m – 2)2 + 2
m 2
= 2
(y x)
1
= 12 + 2
y 12
1
y x x
= y x 1
y 42. (b) Given that, x+ = 1
1 y
x
xy + 1 = y ...(i)
75 1
1 and y+ = 1
2 73 z
= 75 77
1 1 z –1
2 1– =y =y ...(ii)
z z
From eq. (ii),
1 1 1 z 1
39. (c) .....(i)
1 a 1 b 1 c y =
z
Given that, Comparing eqn. (i) with (ii)
a2 = b + c
z 1
a + a2 = a + b + c xy + 1 =
z
a (a + 1) = a + b + c
xyz + z = z – 1
a b c xyz = – 1
a+1=
a
43. (c) 3x+y = 81
3x+y = 3 4
1 a
x+y = 4
a 1 a b c and 81 x–y = 3
Similarly, (3 4)x–y = 3
3 4x–4y = 31 4x – 4y = 1
1 b By Eq. (i) × 4 + Eq. (ii), we have
b 1 a b c 4x + 4y = 16
4x 4y 1
1 c
c 1 a b c 17
8x = 17 x=
Put in eq. (i) 8
y
o
u
Algebraic Expressions and Inequalities B-39

rs
m
44. (a) p = 999 (Given) 1 1
x + a2 = ( x )2 + 2 x . + (a)2

a
52. (b) x+
4 8

h
3 3
Expression p 3p2 3p 1

b
o
1

o
Clearly a=
3
(p 1)3

b
8

.w
p + 1 = 999 + 1 = 1000 2

o
45. (a) Given Px = ry 1

rd
Then, expression = x
r = p x/y 8

p
re
...(i) 53. (a) x2 + y2 + 2x + 1 = 0

s
and p z = rw x2 + 2x + 1 + y2 = 0

s
.c
r = p z/ w (x + 1)2 + y2 = 0

o
...(ii) x+1=0 – 1 and y = 0

m
From Eqs. (i) and (ii) x31 + y35 = – 1
x z 54. (b) x4 – 17x3 + 17x2 – 17x + 17
px/y = pz/w xw = yz = x4 – 16x3 + 16x2 – 16x – x3 + x2 – x + 17
y w
When x = 16,
46. (c) x2 + y2 – 4x – 4y + 8 = 0 Expression = 164 – 164 + 163 – 162 – 163 + 162 – 16 + 17 = 1
x2 – 4x + 4 + y2 – 4y + 4 = 0 55. (b) x2 + 2 = 2x x2 – 2x + 2 = 0
(x – 2)2 + (y – 2)2 = 0 x2 – 2x + 2 ) x4 – x3 + x2 + 2 ( x2 + x + 1
x = 2 and y = 2 x4 – 2x3 + 2x2
– + –
x–y=2–2=0
x3 – x2 + 2
47. (a) x + y – z2 + 2xy
2 2
x3 – 2x2 + 2x
= x2 + y2 + 2xy – z2 – + –
= (x + y)2 – z2 = (x + y + z) (x + y – z)
x2 – 2x + 2
= (b + c – 2a + c + a – 2b + a + b – 2c) (x + y – z) x2 – 2x + 2
=0
x
48. (c) If a + b + c = 0,
x4 – + x2 + 2
x3
then, a3 + b3 + c3 = 3 abc
= (x2 – 2x + 2) (x2 + x + 1) = 0
Here, y – z + z – x + x – y = 0
(y – z)3 + (z – x)3 + (x – y)3 1
56. (d) For nr – tn + to be a perfect square,
= 3(y – z) (z – x) (x – y) 4
49. (a) 2x = 3y = 6–z = k r = 2 and t = ± 1
Check:
1 1 1
y
1 1 1
2 = kx ; 3 = k ; 6 = kz n2 – n + = n2 – 2.n.
4 2 4
2×3=6
2
1 1
1 1 1 1 = n
y 2
kx × k = kz kx y = kz
1 1 1 1 1 1
1 1 1 n2 + n + = n2 + 2.n.
=–
x y z
=0 4 2 4
x y z
50. (a) x2 + y2 + z2 + 2 = 2(y – x)
2
x2 + 2x + y2 – 2y + z2 + 2 = 0 1
= n
(x2 + 2x + 1) + (y2 – 2y + 1) + z2 = 0 2
(x + 1)2 + (y – 1)2 + z2 = 0 t=±1
x+1=0 x=–1 57. (b) 8x3 – ax2 + 54x + b
y–1=0 y=1 = (2x)3 – 3.(2x)2 . y + 3.(2x) . y2 + y3
3 . 2x . y2 = 54x
z=0
y=–3 b = y3 = – 27
x3 + y3 + z3 = – 1 + 1 + 0 = 0 ax = 3. (2x)2 . (– y)
2
51. (d) ax = (x + y + z)y = 12x2 × 3 = 36x2
ay = (x + y + z)z a = 36, b = – 27
az = (x + y + z)x 58. (d) Let f (x) x5 1
Multiplying corresponding terms, Since (x – 2) is the factor of (x5 +1), hence from Remainder
ax . ay. az = (x + y + z)x+y+z
Theorem, we have, f (2) (2)5 1 33
ax+y+z = (x + y + z)x+y+z
a= x+ y+ z Hence, the remainder = 33
y
o
u
B-40 Algebraic Expressions and Inequalities

rs
(3.7)x (0.037)y

m
59. (c) Given, = = 10000
(3.7)x = 104 and (0.037)y = 104 x6 1

a
h
=
(x 2 1 3x).x 2

b
4 4
1 3

o
37 = 10 x and 37 = 10 x

o
b
4 4 x6 1 x6 1

.w
4 4
1 = 3 1 3 =
10 x y x y (2x 3x)x 2 x3

o
10

rd
4 4 1 1 1

p
x6 1 x6

re
x y
=3–1
x y 2 1
=–

s
3 3
x x x3

s
60. (c) x2 + y2 + z2 = xy + yz + zx

.c
Multiply both sieds by 2.

o
m
2(x2 + y2 + z2) 1
3
1
3 1
= 2(xy + yz + zx) =– x 3 = – x
x
3 x
x
2x2 + 2y2 + 2z2 – 2xy – 2yz – 2zx = 0 x
(x – y)2 + (y – z)2 + (z – x)2 = 0
= – [23 – 3 × 2] =–2
x–y=0 x=y
y– z=0 y=z x a x 3a x a b x 3a
z–x=0 z=x 64. (b)
x b x a b x b x a
4x 2y 3z 4 2 3 3
= x b a x a 2a
2x 2 2
x b x a
61. (c) a=3+ 2 2
x b a x a 2a
1 1 1 3 2 2 x b x b x a x a
=
a 3 2 2 3 2 2 3 2 2
a 2a a 2a
1+ 1
3 2 2 x b a x x b x a
= 3 2 2
9 8
1 2
x + a = 2 (x + b) = 2x + 2b
1 x b x a
a+ =6
a 2x – x = a – 2b
6 4 2 6 4 2 x = a – 2b
a a a 1 a a a 1
Now, 3 = 3 3 3 p q r
a a a a a3 65. (a) = k (Let)
b c c a a b
1 1
= a3 3
a p = k(b – c), q = k(c – a), r = k(a – b)
a a Expression = p + q + r
= k(b – c) + k (c – a) + k (a – b)
3
1 1 1 1 = k(b – c + c – a + a – b)
= a 3a a a
a a a a =k×0=0
= (6)3 – 3(6) + 6 = 216 – 18 + 6 = 204
62. (a) x + y = 2z x = 2z – y 1
Subtract ‘z’ from both sides x – z = 2z – y – z = z – y 66. (a) x+ 3
x
x z Cubing both sides,
x z y z
1 1 3 1
x3 + 3 x = 3 x3 + + 3 3 =
x z x z x z x 3 x x3
= = 1
x z z y x z x z x z
3 3
63. (d) x2 + 1 = 2x (Given)
1
1 x3 + =0
x+
x
=2 x3
...(i) Now,
Expression x18 + x12 + y6 + 1= x12(x6 + 1) + 1(x6 + 1)
= (x12 + 1) (x6 + 1)
1 x6 1
x4 1
x2 x2 = (x12 + 1). x3 x3 =0
= 2 2 x3
x 3x 1 (x 3x 1)
y
o
u
Algebraic Expressions and Inequalities B-41

rs
m
1 Multiplying equation by 4 on both sides

a
67. (d) x4 + 4 = 119

h
x 1 1
4x 8 4x – 8

b
o
2 2 3y 3y

o
1 1
x2 x2

b
– 2 = 119 = 121 72. (a) a+b+c=0

.w
2
x x2 i.e. a = – (b + c); b = – (c + a); c = – (a + b)

o
2

rd
1 1 a b 2b b c
x2 x Now, –

p
= 11 + 2 = 11
x2 x c c a a

re
s
a b 2[–(c a)] b c

s
2
1 1 –

.c
x =9 x– =3 – ( a b) c a – (b c )

o
x x

m
Cubing both sides, –1+2–1=0
3 4
1 73. (b) x+ 4
x = 27 x
x
x2 + 4 = 4x x2 – 4x + 4 = 0 (x – 2)2 = 0
1 1 1 x=2
x3 – 3 –3
x = 27 x3 – – 3 × 3 = 27
x x x3 4 4 4 1 1
x3 (2)3 8 8 8
1 x 3
(2)3 8 2 2
x3 – = 27 + 9 = 36
x3
74. (b) x=3+2 2
1
68. (a) 3 3 = a3 4 b3 2 c x=2+1+2 2
4 2 1
2 1 x = ( 2)2 (1) 2 2.1. 2
1
= a.2 3 b.2 3 c
2 1 x = ( 2 1)2
23 23 1
x ( 2 1) ...(1)
1
1 1 2 –1 2 –1
23 1 2 1 2 –1
x 2 1 2 –1 2 –1
= a.2 3 b.2 3 c
1 2 1 1
23 1 23 23 1 Now, x– 2 1 – ( 2 –1) 2 1– 2 1
x
1
1 x– 2
2 1 x
23 1
a.2 3 b.2 3 c
2 1 1
75. (c) The least value of a is 2 where a = 1.
[ (a – b) (a2 + ab + b2) = a3 – b3] a
a = 0, b = 1, c = – 1 76. (b) It a = 0, b 0, c 0, then equation ax + by + c = 0
a+b+c=0+1–1=0 represents a line parallel to x-axis.
69. (c) a + b + c = 2s 77. (a) x2 + y2 + z2 – xy – yz – zx
c = 2s – a – b = (s – a) – (s – b)
(s – a)3 + (s – b)3 + 3 (s – a) (s – b)c 2 2
(x y2 z2 xy yz zx)
= (s – a)3 + (s – b)3+ 3 (s – a) (s – b) [(s – a) + (s – b)] 2
(Put the value of c)
= [(s – a) + (s – b)]3 = (2s – a – b)3 1
(2 x 2 2 y2 2 z 2 2 xy 2 yz 2 zx )
(a + b + c – a – b)3 = c3 (Put the value of 2s) 2
70. (a) x4 – 2x2 + k
(x2)2 – 2x2 + k (x2)2 – 2.1. x2 + k 1 2
(x y 2 2 xy y2 z 2 2 yz x 2 z 2 2 zx)
For above expression to make a perfect square, the k 2
value is equal to 1.
1
1 [( x y)2 ( y z )2 ( z x)2 ]
1 3x 6 2
71. (d) 3x – 6
4y 4y
1
Taking 3 common on both sides [(997 998)2 (998 999) 2 (999 997) 2 ]
2
6 1 1
x x 2 1 2 2 1
3 4.3 y 12 y [1 1 22 ] 6 3
2 2
y
o
u
B-42 Algebraic Expressions and Inequalities

rs
We have x3 + y3 + z3 – 3xyz = (x + y + z)

m
78. (d) 83. (c) The sum forms A.P.

a
(x2 + y2 + z2–xy – yz - zx) First term (a) = 1

h
b
Here x = a – 4, y = b – 3, z = c –1 Common difference (d) = 2

o
So, given expression is (x + y + z)

o
b
(x2 + y2 + z2 – xy – yz – zx) n

.w
Sum of 15 term = (2a (n 1)d )
= (a – 4 + b – 3 + c – 1) (x + y2 + z2 – xy – yz – zx)
2 2

o
= (a + b + c – 8) (x2 + y2 + z2 – xy – yz – zx)

rd
= (8 – 8) (x2 + y2 + z2 – xy – yz – zx) 15 15

p
Sum = (2 1 (15 1)2) = 30 225

re
=0 2 2

s
(a) x4 + y4 – 2x2y2

s
79.

.c
1 1
(x2 – y2)2 [(x + y) (x – y)]2 x 2 x 2 0

o
85. (d)

m
x x
2
1 1 1 1 x2 – 2x + 1 = 0 ; (x – 1)2 = 0 ; x = 1
a a a a
a a a a 1
2013
Now, x =1+1=2
2014
2 x
2 86. (d) Here, a + b + c = 0
2 a 16
a 331 + 336 – 667 = 0
a3 + b3 + c3 – 3abc = 0
1
80. (d) 5a 5
3a 1 1 1 1
87. (a) Next term will be 1 1 1 1
3 2 3 4 5
Multiply by on both sides
5 3 4 5 6
3
3 1 3 2 3 4 5
5a 5
5 3a 5
88 (a)
1
3a 3
5a
4x + 5y = 40
Squaring on both sides
1 1 A (5, 4)
9a 2 2
2 3a 9
25a 5a

1 6 39
9a 2 2
9 B C (10, 0)
25a 5 5 (5, 0)
81. (b) If a + b + c = 0
then a3 + b3 + c3 = 3abc
Dividing both sides by abc 1
Area of ABC = × BC × AC
3 3 3 2
a b c 3abc
abc abc abc abc 1 1
= × (10 – 5) × 4 = × 5 × 4
2 2
a2 b2 c2 Area = 10 sq unit.
3
bc ac ab
y
o
u
rs
m
a
CHAPTER

h
b
4

o
AVERAGE

o
b
.w
o
rd
p
re
s
s
.c
o
The average or mean or arithmetic mean of a number of quantities Example 3 :

m
of the same kind is equal to their sum divided by the number of The average of five consecutive odd numbers is 61. What
those quantities. is the difference between the highest and lowest numbers?
Arithmetic average is used for all averages like: Solution :
averge income, average profit, average age, average marks Let the numbers be x, x + 2, x + 4, x + 6 and x + 8.
etc.
Then, x (x 2) (x 4) (x 6) (x 8) 61
It is defined as the sum total of all volumes of items divided 5
by the total number of items. or 5x + 20 = 305 or x = 57.
In individual series. So, required difference = (57 + 8) – 57 = 8.
Average of a group consisting two different groups when
sum of observations
Average = their averages are known :
Number of observations (a) Let Group A contains m quantities and their average is
a and Group B contains n quantities and their average
x1 x2 x3 ...... xn
or x = is b, then average of group C containing a + b quantities
n
ma mb
To calculate the sum of observations, they should be in the = .
m n
same unit.
Example 4 :
Example 1 :
There are 30 student in a class. The average age of the first 10
A man purchased 5 toys at the rate of `200 each, 6 toys at students is 12.5 years. The average age of the next 20 students
the rate of `250 each and 9 toys at the rate of `300 each. is 13.1 years. The average age of the whole class is :
Calculate the average cost of one toy. Solution :
Solution : Total age of 10 students =12.5 × 10 = 125 years
Price of 5 toys = 200 × 5 = 1000 Total age of 20 students = 13.1 × 20 = 262 years
Price of 6 toys = 250 × 6 = 1500 125 262
Average age of 30 students 12.9 years
Price of 9 toys = 300 × 9 = 2700 30
1000 1500 2700 5200
Example 5 :
Aveage cost of one toy = The average age of students of a class is 15.8 years. The
20 20 average age of boys in the class is 16.4 years and that of the
= ` 260\- girls is 15.4 years. The ratio of the number of boys to the
Example 2 : number of girls in the class is
In three numbers, the first is twice the second and thrice the Solution :
third. If the average of these three numbers is 44, then the Let the number of boys in a class be x.
first number is : Let the number of girls in a class be y.
Sum of the ages of the boys = 16.4 x
Solution :
Sum of the ages of the girls = 15.4 y
Let the three numbers be x, y and z 15.8 (x + y) = 16.4 x + 15.4 y
Therefore, x = 2y = 3z,
x 2
0.6 x = 0.4 y
x x y 3
y and z
2 3 Required ratio = 2 : 3
Shortcut method :
x x
x 16.4 15.4
Now, 2 3 44
3
15.8
11x
or 44 or x 72 15.8 – 15.4 = 0.4 16.4 – 15.8 = 0.6
18
y
o
u
B-44 Average

rs
m
0.4 2 If, in a group, one or more new quantities are added or

a
required ratio excluded, then the new quantity or sum of added or excluded

h
0.6 3

b
quantities = [Change in no. of quantities × original average]
(b) It average of m quantities is a and the average of n

o
± [change in average × final no. of quantities]

o
quantities out of them is b then the average of remaining

b
Take +ve sign if quantities added and

.w
ma nb take –ve sign if quantities removed.

o
group rest of quantities is

rd
m n Example 8 :

p
Example 6 : The average weight of 29 students in a class is 48 kg. If the

re
Average salary of all the 50 employees increasing 5 officers weight of the teacher is included, the average weight rises

s
s
of a company is `850. If the average salary of the officers is

.c
by 500 g. Find the weight of the teacher.

o
`2500. Find the average salary of the remaining staff of the Solution :

m
company. Here, weight of the teacher is added and final average of
(a) 560 (b) 660 the group increases.
(c) 667 (d) 670 Change in average is (+)ve, using the formula
Solution : Sum of the quantities added
Here, m = 50, n = 5, a = 850, b = 2500
Change in no. of quantities Change in average
ma nb × ×
Average salary of remaining staff
m n Original average Final no. of quantities
50 850 5 2500 weight of teacher = (1 × 48) + (0.5 × 30) = 63 kg.
50 5 weight of teacher is 63 kg.
Example 9 :
42500 12500
= The average age of 40 students in a class is 15 years. When
45
10 new students are admitted, the average is increased by
= 667 (approx.) 0.2 year. Find the average age of the new students.
WEIGHTED AVERAGE Solution :
If we have two or more groups of members whose individual
Here, 10 new students are admitted.
averages are known, then combined average of all the
change in average is +ve. Using the formula
members of all the groups is known as weighted average.
Thus if there are k groups having member of number n1, n2, Sum of the quantities added
n3,.......nk with averages A1, A2, A3........Ak respectively then Change in no. of quantities Change in average
weighted average. × ×
n1 A1 n2 A2 n3 A3 .... nk Ak Original average Final no. of quantities
Aw =
n1 n2 n3 ... nk Sum of the weight of 10 new students admitted
Example 7 : = (10 × 15) + (0.2 × 50) = 160 kg.
The average monthly expenditure of a family was `2200 s 160
during the first 3 months; `2250 during the next 4 months Average age of 10 new students = a 16
na 10
and `3120 during the last 5 months of a year. If the total
saving during the year were `1260, then the average monthly Average age of 10 new students is 16 years.
income was
Solution :
n 1
Total annual income The average of first n natural numbers is
.
= 3 × 2200 + 4 × 2250 + 5 × 3120 + 1260 2
= 6600 + 9000 + 15600 + 1260 = 32460 The average of square of natural numbers till n is
n 1 2n 1
32460 .
Average monthly income = = ` 2705 6
12
n( n 1) 2
If X is the average of x1, x2, x3 ....xn then The average of cubes of natural numbers till n is
.
4
(a) Theaverage of x1 + a, x2 + a, x3 + a,.... xn + a is X + a.
The average of odd number s from 1 to n is
(b) The average of x1 – a, x2 – a, x3 – a .... xn – a is X – a
last odd number +1
(c) The average of ax1, ax2, ....axn is aX, provied a 0 .
2
x1 x2 x3 x x The average of even number s from 1 to n is
(d) The average of , , ,... n is , provided
a a a a a last even number + 2
a 0 2
y
o
u
Average B-45

rs
m
If n is odd: The average of n consecutive numbers,
9 25 30

a
consecutive even numbers or consecutive odd numbers is

h
always the middle number. 9 25 30

b
o
If n is even: The average of n consecutive numbers, 3 5 10

o
b
consecutive even numbers or consecutive odd numbers is 64 9

.w
always the average of the middle two numbers. 5 km/h

o
11 11
The average of first n consecutive even number is

rd
(n + 1). If a person covers Ath part of the distance at x km/h. Bth

p
re
The average of first n consecutive odd numbers is n. part of the distance at y km/h and the remaining Cth part at

s
z km/h, then the average speed during the entire journey is

s
The average of squares of first n consecutive even number

.c
o
2( n 1)(2n 1) 1

m
is . km / h
3 A B C
The average of squares of consecutive even numbers till n x y z
is ( n 1)( n 2) . Example 12 :
3
A train covers 50% of the jouney at 30 km/h. 25% of the
The average of square of consecutive odd numbers till n is journey at 25 km/h and the remaining at 20 km/h. Find the
n(n 2) average speed of the train during the entire journey.
.
3 Solution :
If the average of n consecutive numbers is m, then the The average speed
difference between the smallest and the largest number is
2(n – 1). 100 100
If a person or a motor car covers three equal distances at A B C 50 25 25
the speed of x km/h, y km/h and z km/h, respectively, then x y z 30 25 20
for the entire journey average speed of the person or motor
[Here, A = 50, B = 25 and C = 25]
3xzy
car is km/h. 100 1200
xy yz zx
47 / 12 47
Example 10 :
A train covers the first 160 km at a speed of 120 km/h, another 25
160 km at 140 km/h and the last 160 km at 80 km/h. Find the 25 km / h
47
average speed of the train for the entire journey.
If a certain distance is covered at a kmph and an equal
Solution :
distance at b kmph, then the average speed during whole
3xyz
Average speed 2ab
xy yz zx journey = kmph.
a b
3 120 140 80
Example 13 :
120 140 140 80 80 120
A motorist travels to a place 150 km away at an average
360 140 80 4032000 speed of 50 km/hr and returns at 30 km/hr. His average speed
16800 11200 9600 37600 for the whole journey in km/hr is :
11 Solution :
107 km/h
47 2ab 2 50 30
If a person cover A km at a speed of x km/h, B km at a speed Average speed km / hr km / hr
a b 50 30
of y km/h and C km at a speed of z km/h, the average speed
= 37.5 km/hr.
A B C
during the entire journey is km / h
A B C GEOMETRIC MEAN OR GEOMETRIC AVERAGE
x Y z Geometric mean of x1, x2, .... xm is denoted by
Example 11 :
G. M = n x1 x2 .... xn
A person cover 9 km at a speed of 3 km/h, 25 km at a speed
of 5 km/h and 30 km at a speed of 10 km/h. Find the average Example 14 :
speed for the entire journey. The production of a company for three successive years
Solution : has increased by 10%, 20% and 40% respectively what is
A B C the average increase of production.
The average speed Solution :
A B C
x y z G.M. = (10 × 20 × 40)1\3 = 20%
y
o
u
B-46 Average

rs
m
Example 15 : Solution :

a
The mean of the marks secured by 25 students of section A Let the seven consecutive integers be

h
b
of class X is 47, that of 35 students of section B is 51 and x, x + 1, x + 2, ......., x + 6

o
o
that of 30 students of section C is 53. Find the combined The sum of the first five is

b
mean of the marks of students of three sections of class X.

.w
x + x + 1 + x + 2 + x + 3 + x + 4 = 5x + 10
Solution :

o
rd
Mean of the marks of 25 students of XA = 47 5x 10
The average of these five is x 2 n

p
5

re
Sum of the marks of 25 students = 25 × 47 = 1175 ........(i)
The average of the seven will be

s
Mean of the marks of 35 students of XB = 51

s
.c
Sum of the marks of 35 students = 35 × 51 = 1785 .......(ii) 5x 10 x 5 x 6 7x 21

o
= x 3

m
Mean of the marks of 30 students of XC = 53 7 7
Sum of the marks of 30 students = 30 × 53 = 1590 .......(iii) As x + 2 = n, so x + 3 = x + 2 + 1 = n + 1
Adding (i), (ii) and (iii) Example 18:
Sum of the marks of (25 + 35 + 30) i.e., 90 students The average of 11 results is 50. If the average of first six result is 49
= 1175 + 1785 + 1590 = 4550 and that of last six results is 52, find the sixth result.
Thus the combined mean of the marks of students of three Solution :
4550 Average of 11 results
sections = = 50.56 1 2 3 4 5 6 7 8 9 10 11
90
Average of last 6 results = 52
Example 16 :
Average of first 6 results = 49
Find the A.M. of the sequence 1, 2, 3, ....., 100.
It is quite obvious that the sixth result is included twice,
Solution : once in the first six results and second in the last six results.
n Value of the sixth result = (Sum of first six results)
We have sum of first n natural numbers = (n 1) , + (Sum of last six results) – Sum of 11 results
2
= 6 × 49 + 6 × 52 – 11 × 50 = 56
here n = 100
Example 19 :
100 Typist A can type a sheet in 5 minutes, typist B in 6 minutes
Sum = 101 101 50 and typist C in 8 minutes. The average number of sheets
2
typed per hour per typist is ......... sheets.
Sum 101 50 Solution :
AM = 50.5
100 100 A types 12 sheets in 1 hour
Example 17 : B types 10 sheets in 1 hour
C types 7.5 sheets in 1 hour
A sequence of seven consecutive integers is given. The
average of the first five given integers is n. Find the average Average number of sheets types per hour per typist
of all the seven integers. 12 10 7.5 29.5
= = = 9.83
3 3
y
o
u
Average B-47

rs
m
a
h
b
o
o
b
.w
1. In a coconut grove, (x + 2) trees yield 60 nuts per year, x 10. The average age of 36 students in a group is 14 years.

o
rd
trees yield 120 nuts per year and (x – 2) trees yield 180 nuts When teacher’s age is included to it, the average increases

p
per year. If the average yield per year per tree be 100, then x by one. What is the teacher’s age in years?

re
is (a) 31 (b) 36

s
s
(a) 3 (b) 4 (c) 51 (d) cannot be determined

.c
(c) 5 (d) 6

o
11. A batsman in his 12th innings makes a score of 65 and

m
2. Out of 40 boys in a class, average weight of 30 is 60 kg and thereby increases his average by 2 runs. What is his average
the average weight of the remaining is 56 kg. The average after the 12th innings if he had never been ‘not out’?
weight (in kilogram) of the whole class is (a) 42 (b) 43
(a) 58.5 (b) 58 (c) 44 (d) 45
(c) 57 (d) 59 12. A pupil’s marks were wrongly entered as 83 instead of 63.
3. A school has 4 section of Chemistry in Class X having 40, Due to that the average marks for the class got increased
35, 45 and 42 students. The mean marks obtained in by half. The number of pupils in the class is:
Chemistry test are 50, 60, 55 and 45 respectively for the 4 (a) 10 (b) 20
sections. Determine the overall average of marks per student (c) 40 (d) 73
(a) 50.25 (b) 52.25 13. In the first 10 overs of a cricket game, the run rate was only
(c) 51.25 (d) 53.25 3.2. What should be the run rate in the remaining 40 overs
4. The average of 20 numbers is zero. Of them, at the most, to reach a target of 282 runs ?
how many may be greater than zero? (a) 6.25 (b) 6.50
(a) 0 (b) 1 (c) 6.75 (d) 7.00
(c) 10 (d) 19 14. The average number of printing error per page in a book of
5. The average of six numbers is 3.95. The average of two of 512 pages is 4. If the total number of printing error in the
them is 3.4, while the average of the other two is 3.85. What first 302 pages is 1,208, the average number of printing errors
is the average of the remaining two numbers? per page in the remaining pages is
(a) 4.5 (b) 4.6 (a) 0 (b) 4
(c) 4.7 (d) 4.8 (c) 840 (d) 90
6. Nine persons went to a hotel for taking their meals. Eight of 15. The average attendance in a school for the first 4 days of
them spent ` 12 each on their meals and the ninth spend the week is 30 and for the first 5 days of the week is 32. The
` 8 more than the average expenditure of all the nine. What attendance on the fifth day is
was the total money spent by them? (a) 32 (b) 40
(a) ` 115 (b) ` 117 (c) 38 (d) 36
(c) ` 119 (d) ` 122 16. The average expenditure of a labourer for 6 months was `
7. The average age of A and B is 20 years. If C were to replace 85 and he fell into debt. In the next 4 months by reducing
A, the average would be 19 and if C were to replace B, the his monthly expenses to ` 60 he not only cleared off his
averge would be 21. What are the age of A, B and C? debt but also saved ` 30. His monthly income is
(a) 22, 18, 20 (b) 20, 20, 18 (a) `70 (b) ` 72
(c) 18, 22, 20 (d) None of these (c) ` 75 (d) ` 78
8. 3 years ago the average age of a family of 5 members was 17 17. The average of a batsman for 40 innings is 50 runs. His
years. With the birth of a new baby, the average age of six highest score exceeds his lowest score by 172 runs. If these
members remains the same even today. Find the age of the two innings are excluded, his average drops by 2 runs. Find
new baby. his highest score.
(a) 1 year (b) 2 years (a) 172 (b) 173
1 (c) 174 (d) 175
(c) 1 years (d) cannot be determined 18. Last year, a Home Appliance Store sold an average
2
9. The average age of a group of person going for picnic is 16 (arithmetic mean) of 42 microwave ovens per month. In the
years. Twenty new persons with an average age of 15 years first 10 months of this year, the store has sold an average
join the group on the spot due to which their average (arithmetic mean) of only 20 microwave ovens per month.
becomes 15.5 years. Find the number of persons initially What was the average number of microwave ovens sold
going for picnic. per month during the entire 22 months period ?
(a) 20 (b) 18 (a) 21 (b) 30
(c) 22 (d) None of these (c) 31 (d) 32
y
o
u
B-48 Average

rs
m
19. The captain of a cricket team of 11 players is 25 years old 4 5
(a) 28 years (b) 31 years

a
and the wicket-keeper is 3 years older. If the age of these 7

h
7
two players are replaced by that of another two players, the

b
1

o
average of the cricket team drops by 2 years. Find the (c) 32 years (d) None of these

o
7

b
average age of these two players.

.w
(a) 15 years (b) 15.5 years 28. The average of two numbers is XY. If one number is X, then

o
the other number is
(c) 17 years (d) 16.5 years

rd
Y

p
20. A batsman makes a score of 87 runs in the 17th inning and
(a) Y (b)

re
thus increases his average by 3. Find his average after 17th 2

s
s
inning. (c) 2XY – X (d) X (Y – 1)

.c
(a) 36 (b) 39 29. In Arun’s opinion, his weight is greater than 65 kg but less

o
m
(c) 42 (d) 45 than 72 kg. His brother does not agree with Arun and he
21. Nine men went to a hotel. 8 of them spent ` 3 each over their thinks that Arun’s weight is greater than 60 kg but less than
meals and the ninth spent Rs 2 more than the average 70 kg. His mother’s view is that his weight cannot be greater
expenditure of all the nine. The total money spent by all of than 68 kg. If all of them are correct in their estimation, what
them was is the average of different probable weights of Arun?
(a) ` 26 (b) ` 40 (a) 67 kg (b) 68 kg
(c) ` 29.25 (d) ` 27 (c) 69 kg (d) None of these
22. A cricketer whose bowling average is 12.4 runs per wicket 30. The average age of a board of 8 functional directors in a
takes 5 wickets for 26 runs and thereby decreases his company is the same as it was 3 years ago, a younger man
average by 0.4. The number of wickets taken by him till the having been substituted for one of the directors. How much
last match was: younger was the new man than the director whose place he
(a) 64 (b) 72 took.
(c) 80 (d) 85 (a) 24 years (b) 26 years
23. In an examination, a pupil’s average marks were 63 per paper. (c) 28 years (d) None of these
If he had obtained 20 more marks for his Geography paper 31. The average weight of 45 students in a class is 52 kg. 5 of
and 2 more marks for his Histroy paper, his average per them whose average weight is 48 kg leave the class and
paper would have been 65. How many papers were there in other 5 students whose average weight is 54 kg join the
the examination? class. What is the new average weight (in kg) of the class ?
(a) 8 (b) 9 2
(a) 52.6 (b) 52
(c) 10 (d) 11 3
24. A car owner buys petrol at ` 7.50, ` 8.00 and ` 8.50 per litre 1
(c) 52 (d) None of these
for three successive years. What approximately is 3
his average cost per litre of petrol if he spends ` 4000 each 32. The average of 10 numbers is 40.2. Later it is found that two
year ? numbers have been wrongly copied. The first is 18 greater
(a) ` 8 (b) ` 9 than the actual number and the second number added is 13
(c) ` 7.98 (d) ` 8.50 instead of 31. Find the correct average.
(a) 40.2 (b) 40.4
25 A batsman has scored an average of 46 runs for a certain
(c) 40.6 (d) 40.8
number of innings played in England. When he came back
to India, he played another two test matches of two innings 33. There are 50 boys in a class. Their average weight is 45 kg.
each and scored at an average of 55 runs. For the innings in When one boy leaves the class, the average reduces
England and in India taken together, he has improved his by 100 g. Find the weight of the boy who left the class.
average by 2 runs over the matches played in England. (a) 40.9 kg (b) 42.9 kg
Find the number of innings played in England. (c) 49.9 kg (d) 39.9 kg
34. The average monthly sales for the first eleven months of
(a) 12 (b) 13
the year of a certain salesman were ` 12000, but due to his
(c) 14 (d) 15
illness during the last month, the average monthly sales for
26 There were 35 students in a hostel. Due to the admission of
the whole year came down to ` 11375. What was the value
7 new students, the expenses of mess were increased by
of sales during the last month ?
` 42 per day while the average expenditure per head
(a) 2100 (b) 875
diminished by ` 1. What was the original expenditure of the
(c) 700 (d) 4500
mess?
35. The average mark of a class of n students is 64. When eight
(a) ` 400 (b) ` 420
new students with an average mark of 73 join the class, the
(c) ` 445 (d) ` 465 new average of the entire class is a whole number. Find the
27. A family consists of grandparents, parents and three number of students now in the class, given that n lies
grandchildren. The average age of the grandparents is 67 years, between 25 and 60.
that of the parents is 35 years and that of the grandchildren is (a) 36 (b) 28
6 years. What is the average age of the family? (c) 54 (d) 72
y
o
u
Average B-49

rs
m
36. The average temperature for Monday, Tuesday and (a) 35 km/h (b) 40 km/h

a
Wednesday was 55°, the average for Tuesday, Wednesday (c) 30 km/h (d) 17.14 km/h

h
and Thursday was 60°, that for Thursday being 56°, what 45. The respective ratio between the speed of a car, a train and

b
o
was the temperature on Monday ? a bus is 5 : 9 : 4. The average speed of the car the bus and

o
b
(a) 39° (b) 41° the train is 72 km/h together. What is the average speed of

.w
(c) 45° (d) None of these the car and the train together?

o
37. If a, b, c, d, e are five consecutive odd numbers, then the (a) 82 km/h (b) 78 km/h

rd
average in terms of ‘a’ will be – (c) 84 km/h (d) cannot be determined

p
re
(a) a + 2 (b) a + 3 (c) a + 4 (d) a 46. The table given below has question-wise data on the

s
38. In preparing a meal for 4 people, a housewife uses 600 grams performance of students in an examination. The marks for

s
.c
of sprouts, 1 kg of potatoes, 1 cauliflower weighing each question are also listed. There is no negative or partial

o
1/2 kg each and 700 grams of meat. If one quarter of the

m
marking in the examination
weight is lost in preparation and cooking, then what is the
average weight in grams, of each person’s meal ?
(a) 2100 (b) 875 (c) 700 (d) 525 Ans wered Answered
Q.No Mark s Not attempled
39. Visitor to show were charged ` 15.00 each on the first day, correctly wrongly
` 7.50 on the second, ` 2.50 on the third day and total 1 2 21 17 6
attendance on the three days were in the ratio of 2:5:13,
respectively. Find the average charge per person for the 2 3 15 27 2
whole show. 3 2 23 18 3
(a) ` 5 (b) ` 7 (c) ` 9 (d) ` 11
40. The average weight of a group of 75 girls was calculated as what is the average of the marks obtained by the class in
47 kgs. It was later discovered that the weight of one of the the examination?
girls was read as 45 kg. Whereas her actual weight was 25 (a) 1.34 (b) 1.74
kg. What is the actual average weight of the group of 75 (c) 3.02 (d) 3.91
girls? 47. The average age of boys in the class is twice the number of
(a) 4.6 kg (b) 46.73 kg girls in the class. The ratio of boys and girls in the class of
(c) 47.83 kg (d) 45.93 kg 50 is 4 : 1. The total of the ages (in years) of the boys in the
41. The mean grade of a section of 20 students is 66% and that class is
of another section of 15 student is 70% what is combined (a) 2000 (b) 2500
mean grade. (c) 800 (d) 400
(a) 66.7% (b) 67.7% (c) 68.7% (d) 69.7% 48. There are 100 students in 3 sections A, B and C of a class.
42. The average age of a team of 15 employees is 36. The The average marks of all the 3 sections was 84. The average
youngest of them is 20 years old and the eldest is 56 years of B and C was 87.5 and the average marks of A is 70. The
old. Two of them with average age 28 leave the team. If one number of students in A was
of the two comes back on the condition that he will be made (a) 30 (b) 35
the team leader then which of the following can possible by (c) 20 (d) 25
the average age of the new team so formed ? 49. The average of 5 consecutive numbers is n. If the next two
(a) 35 (b) 36 (c) 38 (d) 39 numbers are also included, the average of the 7 numbers
43. On a journey across Kolkata, a taxi averages speed is will
40 kmph for 60% of distance, 30 kmph for 20% of the (a) increase by 2 (b) increase by 1
distance, and 10 kmph for the remainder. The average speed (c) remain the same (d) increase by 1.4
of the whole journey is- 50. On a journey across Kolkata, a taxi averages 40 kmph for
(a) 25 km/h (b) 26 km/h 60% of distance, 30 kmph for 20% of the distance, and 10
(c) 24 km/h (d) 30 km/h kmph for the remainder. The average speed of the whole
44. A man travels a journey with average speed of 40 km/h and journey is
return back with average speed of 30 km/h. Find his over all (a) 25 kmph (b) 26 kmph
average speed. (c) 24 kmph (d) 30 kmph

AN SWER KEY
1 (b) 11 (b) 21 (c) 31 (b) 41 (b)
2 (d) 12 (c) 22 (d) 32 (a) 42 (b)
3 (b) 13 (a) 23 (d) 33 (c) 43 (c)
4 (d) 14 (b) 24 (c) 34 (d) 44 (d)
5 (b) 15 (b) 25 (c) 35 (a) 45 (c)
6 (b) 16 (d) 26 (b) 36 (b) 46 (c)
7 (a) 17 (c) 27 (b) 37 (c) 47 (c)
8 (b) 18 (d) 28 (c) 38 (d) 48 (c)
9 (a) 19 (b) 29 (d) 39 (a) 49 (b)
10 (c) 20 (b) 30 (a) 40 (b) 50 (c)
y
o
u
B-50 Average

rs
m
a
h
b
o
o
b
.w
(x 2) 60 x 120 (x 2) 180 9. (a) Let the number of persons, initially going for

o
1. (b) 100

rd
(x 2) x (x 2) Picnic = x

p
Sum of their ages = 16x

re
360x 240
100 16x 15 20

s
3x Also, 15.5

s
.c
x 20
60x 240 x 4

o
0.5x 10 x 20

m
2. (d) Average weight of 30 = 60 kg
10. (c) Age of the teacher = (37 × 15 – 36 × 14) years = 51
Sum of weight of 30 boys = 1800
years.
Average weight of 10 =56 kg
11. (b) Let ‘x’ be the average score after 12 th innings
Sum of weight of 10 boys = 560
12 x = 11 × (x – 2) + 65
Average weight of the whole class
x = 43
Sum of weight of all boys 12. (c) Let there be x pupils in the class.
=
40 1 x
Total increase in marks = x .
sum of weight of 30 boys + sum of weight of 10 boys 2 2
= x x
40 (83 63) 20 x 40.
2 2
60×30+56×10 13. (a) Total runs in the first 10 overs
= 59kg
40 = 10 × 3.2 = 32
3. (b) Required average marks Run rate required in the remaining 40 overs
282 – 32 250
40 50 35 60 45 55 42 45 6.25 runs per over
40 40
40 35 45 42 14. (b) Remaining pages = 512 – 302 = 210
Let average printing error in remaining pages = x
2000 2100 2475 1890
162 1208 210 x
Then, 4
512
8465 210x = 840 x = 4
52.25
162 15. (b) Attendance on the fifth day = 32 × 5 – 30 × 4
4. (d) Average of 20 numbers = 0. = 160 – 120 = 40
Sum of 20 numbers = (0 × 20) = 0. 16. (d) Income of 6 months = ` (6 × 85) – debt
It is quite possible that 19 of these numbers may be = ` 510 – debt
positive and if their sum is a, then 20th number is (– a). Income of the man for next 4 months
5. (b) Sum of the remaining two numbers = ` 4 × 60 + debt + ` 30
= (3.95 × 6) – [(3.4 × 2) + (3.85 × 2)] = ` 270 + debt
= 23.70 – (6.8 + 7.7) = 23.70 – 14.5 = 9.20 Income of 10 months = ` 780
Average monthly income = ` 780 ÷ 10 = ` 78
9.2
Required average = 4.6. 17. (c) Total runs = 40 × 50 = 2000
2 Let his highest score be =x
6. (b) Let the average expenditure of all the nine be Rs. x. Then his lowest score = x – 172
Then, 12 × 8 + (x + 8) = 9x or 8x = 104 or x = 13. 2000 – x (x 172)
Total money spent = 9x = Rs. (9 × 13) = Rs. 117. Now 48
38
7. (a) Given A + B = 40 …(i) 2x = 2172 – 1824
C + B = 38 … (ii) x = 174
A + C = 42 … (iii)
(i) + (ii) + (iii) A + B + C = 60 …(iv) 42 12 20 10 504 200 704
18. (d) = = 32
from (i) and (iv), we get 12 10 22 22
C = 20 years 19. (b) Let average of team = x years
B = 18 years and A = 22 years Then, 25 + 28 + S9 = 11x … (i)
8. (b) Sum of present ages of the six members where S9 is the sum of ages of remaining players
= (17 × 6) years = 102 years. Also, Np + S9 = 11(x – 2) , … (ii)
Sum of present ages of the 5 members (excluding baby) where Np is the sum of ages of new players
= 5 × (17 + 3) years = 100 years. (i) – (ii) 53 – Np = 22
Age of the baby = 102 – 100 = 2 years Np = 31
y
o
u
Average B-51

rs
m
31 30. (a) Let the new man was younger than the director = x years
Average age of new two players 15.5 years

a
2 and 3 years ago, the sum of ages of board of directors

h
= S – 8 × 3 = S – 24

b
20. (b) Let the average after 17th inning = x.

o
Then, 3 years ago, average age of board of directors

o
Then, average after 16th inning = (x – 3).

b
S 24

.w
16(x – 3) + 87 = 17x or x = (87 – 48) = 39.
8

o
21. (c) Let the average expenditure of all the nine be Rs x

rd
Then, 3 × 8 + x +2 = 9x x = 3.25 S 24 S x
Now,

p
Total money spent = 9x = 9 × 3.25 = Rs. 29.25 8 8

re
x = 24 years

s
22. (d) Let the number of wickets taken till the last match be x.

s
Shortcut Method : If the new young director would

.c
Then,

o
have been not substituted, then total age would have
12.4x 26

m
12 12.4x 26 12x 60 increased at present by 8 × 3 = 24 years.
x 5 Therefore, the new man is 24 years younger keeping
34 340 the average at present same as 3 years ago.
0.4x 34 x 85.
0.4 4 31. (b) Total weight of 45 students
23. (d) Let the number of papers be x. Then, 63x + 20 + 2 = 65x = 45 × 52 = 2340 kg
or 2x = 22 or x = 11. Total weight of 5 students who leave
24. (c) Let average cost of petrol per litre be Rs x = 5 × 48 = 240 kg
Total weight of 5 students who join
12000
Then x = 5 × 54 = 270 kg
4000 4000 4000
Therefore, new total weight of 45 students
7.5 8 8.5
= 2340 – 240 + 270 = 2370
3 6120
Rs 7.98 per litre New average weight
2370 2
52 kg
2 1 2 767 45 3
15 8 17 32. (a) Sum of 10 numbers = 402
25. (c) Let the number of innings played in England be x. Corrected sum of 10 numbers
Total runs scored in England = 46 x = 402 – 13 + 31 – 18 = 402
Total runs scored for innings played in India 402
= 55 × 4 = 220 Hence, new average 40.2
10
( the number of innings played in India = 4) 33. (c) Here one boy is excluded and final average of the group
46x 220 decreases.
Also, 48
x 4 change in average is (–)ve = – 0.1 kg.
46x + 220 = 48 x + 192 Using the formula
2x = 28 x = 14 Sum of the quantities excluded
26. (b) Let the original average expenditure be ` x. Then, Change in no. of quantities Change in average
42(x – 1) – 35x = 42 7x = 84 x = 12. × ×
Original average Final no. of quantities
Original expenditure = ` (35 × 12) = ` 420.
67 2 35 2 6 3 weight of the boy who left = (1 × 45) – (– 0.1 × 49)
27. (b) Required average = = 49.9 kg
2 2 3 weight of the boy who left the class is 49.9 kg.
134 70 18 222 34. (d) Total sales for the first eleven months
7 7 = 12,000 × 11 = ` 132000
Total sales for the whole year = ` 11375 × 12
5 = ` 136500
31 years.
7 Value of sales during the last month
28. (c) Let the other number be N. = 136500 – 132000 = `4500.
X N 35. (a) Let ‘x’ be the increase in the average
Then, XY N = 2XY – X
2 64n 8 73
29 (d) Let Arun’s weight be X kg. 64 x
n 8
According to Arun, 65 < X < 72.
According to Arun’s brother, 60 < X < 70. (73 64)8 9 8
x= x=
According to Arun’s mother, X < 68. n 8 n 8
The values satisfying all the above conditions are 66 For ‘x’ to be a whole number 72 (= 9 × 8) should be
and 67. divisible by (n + 8)
From the choices it can be said that 36 and 72 are two
66 67 133
Required average 66.5 kg. such factors. But 72 does not lie within the range.
2 2 number of students in class are 36.
y
o
u
B-52 Average

rs
m
36. (b) Sum of temp. for Monday, Tuesday and Wednesday total distance travelled
44. (d) Average speed =

a
= 55 × 3 = 165°

h
total time taken
Sum of temp. for Tuesday, Wednesday, an Thursday

b
o
= 60 × 3 = 180° D D

o
= 17.14 km/h

b
Since temp. on Thursday = 56° D D

.w
Temp for Tue and Wed = 180 – 56 = 124° 40 30

o
Temp of Monday = 165 – 124 = 41°

rd
45. (c) Total speed of car, bus and train = 72 × 3 = 216 km
37. (c) If you assume the first number to be a, naturally next

p
re
number would be 2 more than a and so on 5 9
Speed of car & train 216 168 km/h

s
The numbers are : a, a + 2, a + 4, a + 6, a + 8. 5 9 4

s
.c
Hence, their average = Sum/5 = (a + 4) 168

o
Average 84 km/h

m
38. (d) Sprouts 600 gms + Potatoes 1000 gms + Cauliflower 2
500 gms + Meat 700 gms = 2800 gms. 46. (c) Total marks obtained = (21×2) + (15×3) + (23×2) = 133
Hence total cooked food = 2100 gms and is served Total number of students = 21 + 17 + 6 = 44
among 4 people.
133
Thus average weight is 525 gms. Average 3.02
39. (a) Let attendance on first, second and third day be 2, 5 44
and 13 respectively. 4
47. (c) Number of boys = 50 = 40
Total attendance of three days = 2 + 5 + 13 = 20 5
Total money collected = 2×15+5×7.50+13+2.50 1
= 30 + 37.5 + 32.5 = 100 Number of girls = 50 = 10
5
100 Average age of boys = 2 × 20 = 40
Average per person 5
20 Total ages of the boys = 40 × 20 = 800
40. (b) Total actual weight of all girls = 47 × 75 – 45 + 25 48. (c) Total marks of all three sections = 84 × 100 = 8400
= 3505 kg total marks of (B + C) = 87.5 (n 2 + n3)
3505 total marks of A = 70 × n 1
Avg weight = 46.73 kg. n1 + n2 + n3 = 100 ...(1)
75
70n1 + 87.5 n2 + 87.5 n3 = 8400 ...(2)
20 66 15 70
41. (b) 67.7% Multiplying equation (1) by 87.5 and subtract from
20 15 equation (2)
42. (b) Total age = 15 × 36 We get 17.5 n1 = 350
After the two left, total age = 15 × 36 – 2 × 28 n1 = 20
= 540 – 56 = 484 years. 49. (b) Let the numbers be n – 2, n – 1, n, n + 1 and n + 2.
Let the age of the person who returns be x years. Their average = n.
484 x Next two consecutive numbers are n + 3 and n + 4.
Then new average =
14 Therefore the average of 7 consecutive numbers
Now x lies between 20 and 36 (both inclusive) (n 2) (n 1) n (n 1) (n 2) (n 3) (n 4)
484 20 520 7
New average min = = 36 years.
14 14 5n 2 n 7
n 1
484 36 520 7
And New average max = = = 37.14 Hence, the average of 7 numbers will increase by 1.
14 14
50. (c) Let total distance be d.
years.
0.6d 3d
Total Distance time taken for 60% distance = h
43. (c) Average speed = 40 200
Total time
0.2d d
Let total distance is D km time taken for 20% distance = h
30 150
Time taken for 60% of distance = D 0.6 hour time taken for remaining 20% distance
40
0.2d d
= h
Time taken for 20% of distance = D 0.2 hour 10 50
30 d
Average speed = 3d d d
Time taken for 20% of distance = D 0.2
10 200 150 50
D 200 150 50 200 150 50
Average speed = 24 km/h =
D 0.6 D 0.6 D 0.2 22500 10000 30000 62500
40 30 10 = 24 kmph
y
o
u
rs
m
a
CHAPTER

h
b
PERCENTAGE 5

o
o
b
.w
o
rd
p
re
s
s
.c
o
PERCENTAGE : Fractional Equivalents of %

m
The word “per cent” is derived from the latin words “per
1 1 1
centum”, which means “per hundred”. 1% 33 %
100 3 3
A percentage is a fraction with denominator hundred.
1 2
It is denoted by the symbol %. 2% 40%
Numerator of the fraction is called the rate per cent. 50 5
VALUE OF PERCENTAGE : 1 1
4% 50%
Value of percentage always depends on the quantity to which it 25 2
refers. 1 2 2
5% 66 %
Consider the statement, “65% of the students in this class are 20 3 3
boys”. From the context, it is understood that boys form 65% of 1 1 3
the total number of students in the class. To know the value of 6 % 60%
4 16 5
65% , the value of the total number of student should be known.
1 3
If the total number of students is 200, then, 10% 75%
10 4
200 65 1 17 4
the number of boys = = 130; 11 % 80%
100 3 150 5
It can also be written as (200) × (0.65) = 130. 1 1 24
12 % 96%
Note that the expressions 6%, 63%, 72%, 155% etc. do not have 2 8 25
any value intrinsic to themselves. Their values depend on the 4
quantities to which they refer. 16% 100% = 1
25
To express the fraction equivalent to % :
2 1 23
Express the fraction with the denominator 100, then the numerator 16 % 115%
3 6 20
is the answer.
Example 1 : 1 1 4
20% 133 %
5 3 3
11
Express the fraction into the per cent. 1
12 25%
4
Solution :
Increase value
11 2 Increase % 100
100 91 Original value
11 12 3 2
= 91 %
12 100 100 3 Example 3 :
To express % equivalent to fraction : Rent of the house is increased from ` 7000 to ` 7700. Express
a the increase in price as a percentage of the original rent.
a% =
100 Solution :
Example 2 : Increase value = ` 7700 – ` 7000 = `700
5 Increase value 700
Express 45 % into fraction. Increase % = 100 = × 100= 10
6 Original value 7000
Solution :
Percentage rise = 10 %.
5 Decrease value
45
5 6 = 275 11 Decrease % 100
45 % = . Original value
6 100 6 100 24
y
o
u
B-54 Percentage

rs
Example 4 :

m
If the price of a commodity increases by r %, then reduction
in consumption, so as not to increase the expenditure is

a
The cost of a bike last year was ` 19000. Its cost this year is

h
` 17000. Find the per cent decrease in its cost.

b
r

o
Solution : 100 % .

o
100 r

b
.w
19000 17000 If the price of a commodity decreases by r %, then the increase
% decrease = 100
in consumption so as not to decrease the expenditure is

o
19000

rd
r

p
2000 100 % .

re
100 = 10.5 %. 100 r
19000

s
s
Example 7 :

.c
Per cent decrease = 10.5 %.
If the price of coal be raised by 20%, then find by how much

o
x

m
If A is x % of C and B is y % of C, then A is × 100 % of B. a householder must reduce his consumption of this
y commodity so as not to increase his expenditure ?
Example 5 :
A positive number is divided by 5 instead of being multiplied Solution :
by 5. By what per cent is the result of the required correct 20
value ? Reduction in consumption = 100 %
Solution : 100 20
Let the number be 1, then the correct answer = 5 20
= 100 % = 16.67 %
1 120
The incorrect answer that was obtained = .
5 POPULATION FORMULA
1 If the original population of a town is P, and the annual
The required % = 100 = 4 %. increase is r %, then the population after n years is
5 5
n
If two numbers are respectively x% and y% more than a r P
P 1 and population before n years = n
100 x 100 r
1
third number, then the first number is 100 y 100 % of 100
If the annual decrease be r %, then the population after n
100 y n
the second and the second is 100 % of the r
100 x years is P 1 and
first. 100
If two number are respectivley x% and y% less than a third P
population before n years = n
100 x r
number, then the first numbe if 100 y 100 % of the 1
100

100 y Example 8 :
second and the second is 100 % of the first.
100 x The population of a certain town increased at a certain rate
per cent per annum. Now it is 456976. Four years ago, it was
x % of a quantity is taken by the first, y % of the remaining 390625. What will it be 2 years hence ?
is taken by the second and z % of the remaining is taken by
Solution :
third person. Now, if A is left in the fund, then the initial
amount Suppose the population increases at r% per annum. Then,
4
A 100 100 100 r
390625 1 = 456976
= in the begining. 100
(100 x)(100 y)(100 z)
2
x % of a quantity is added. Again, y % of the increased r 456976 676
quantity is added. Again z % of the increased quantity is 1
100 390625 625
added. Now it becomes A, then the initial amount
2
A 100 100 100 r
Population 2 years hence = 456976 1
= 100
(100 x)(100 y)(100 z)
Example 6 : 676
= 456976 × = 494265 approximately.
3.5 % income is taken as tax and 12.5 % of the remaining is 625
saved. This leaves Rs. 4,053 to spend. What is the income ? Example 9 :
Solution : The population of a city increases at the rate of 4% per
By direct method, annum. There is an additional annual increase of 1 % in the
4053 100 100 population due to the influx of job seekers. Find, percentage
Income = = ` 4800. increase in the population after 2 years.
(100 3.5)(100 12.5)
y
o
u
Percentage B-55

rs
Solution : Solution :

m
a
The net annual increase = 5%. Put x = – 10 and y = – 20, then

h
b
Let the initial population be 100.

o
( 10)( 20)

o
Then, population after 2 years = 100 × 1.05 × 1.05 10 20 = – 28%

b
100

.w
= 110.25
The price of the car decreases by 28%.

o
Therefore, % increase in population

rd
STUDENT AND MARKS
= (110.25 – 100) = 10.25%

p
The percentage of passing marks in an examination is x%. If

re
If a number A is increased successively by x% followed by
a candidate who scores y marks fails by z marks, then the

s
y% and then z%, then the final value of A will be

s
.c
100(y z)

o
x y z maximum marks M =

m
A 1 1 1 x
100 100 100
A candidate scoring x % in an examination fails by ‘a’ marks,
In case a given value decreases by an percentage then we while another candidate who scores y% marks gets ‘b’ marks
will use negative sign before that. more than the minimum required passing marks. Then the
First Increase and then decrease :
If the value is first increased by x % and then decreased by 100(a b)
maximum marks M .
y x
xy
y % then there is x y % increase or decrease, In an examination x % and y % students respectively fail in
100
two different subjects while z % students fail in both
according to the +ve or –ve sign respectively.
subjects. then the % age of student who pass in both the
If the value is first increased by x% and then decreased by subjects will be {100 – (x + y – z)}%
x2 Example 12 :
x % then there is only decrease which is equal to 100 . Vishal requires 40% to pass. If he gets 185 marks, falls short
by 15 marks, what was the maximum he could have got ?
Example 10 : Solution :
A number is increased by 10%. and then it is decreased by If Vishal has 15 marks more, he could have scored 40%
10%. Find the net increase or decrease per cent. marks.
Solution : Now, 15 marks more than 185 is 185 + 15 = 200
10 10 Let the maximum marks be x, then
% change = 1% 40% of x = 200
100
i.e 1% decrease. 40 200 100
x 200 x 500
Average percentage rate of change over a period. 100 40
(New Value Old Value) 100 Thus, maximum marks = 500
= × % where n = period. Alternate method :
Old Value n
100(185 15) 100 200
The Error Maximum marks = = 500 .
The percentage error = × 100% 40 40
True Value
Example 13 :
SUCCESSIVE INCREASE OR DECREASE
A candidate scores 15% and fails by 30 marks, while another
If the value is increased successively by x % and y % then candidate who scores 40% marks, gets 20 marks more than
the final increase is given by the minimum required marks to pass the examination. Find
xy the maximum marks of the examination.
x y % Solution :
100
By short cut method :
If the value is decreased successively by x % and y % then
the final decrease is given by 100(30 20)
Maximum marks = 200
xy 40 15
x y %
100 2-DIMENSIONAL FIGURE AND AREA
Example 11 : If the sides of a triangle, square, rectangle, rhombus or radius
of a circle are increased by a%, its area is increased by
The price of a car is decreased by 10 % and 20 % in two
successive years. What per cent of price of a car is decreased a(a 200)
%.
after two years ? 100
y
o
u
B-56 Percentage

rs
m
If the sides of a triangle, square, rectangle, rhombus or radius
50

a
of a circle are decreased by a % then its area is decreased by = 100 % = 33.33 %.

h
150

b
a(200 a) .

o
% Example 17 :

o
100

b
Ravi’s weight is 25% that of Meena’s and 40% that of Tara’s.

.w
Example 14 :
What percentage of Tara’s weight is Meena’s weight.

o
If the radius of a circle is increased by 10 %, what is the

rd
percentage increase in its area ? Solution :

p
re
Solution : Let Meena’s weight be x kg and Tara’s weight be y kg.

s
s
Let R be the radius of circle. Then Ravi’s weight = 25% of Meena’s weight

.c
Area of Circle, A = R2

o
25

m
Now, radius is increased by 10% =×x .... (i)
100
New radius, R’ = R + 10% of R = 1.1 R Also, Ravi’s weight = 40% of Tara’s weight
New Area, A’ = (1.1R)2 = 1.21 R2
40
2 2 = ×y .... (ii)
1.21 R R 100
% increase in area = 100 = 21%
R2 From (i) and (ii), we get
Shortcut Method: 25 40
×x= ×y
Radius is increased by 10%. 100 100
10(10 200) 25x = 40y
So, Area is increased by = 21 %.
100 8
5x = 8y yx
If the both sides of rectangle are changed by x% and y% 5
xy Meena’s weight as the percentage of Tara’s weight
respectively, then % effect on area = x + y +
100 8
(+/– according to increase or decrease) y
x 5 100
Example 15 : = 100
y y
If the length and width of a rectangular garden were each
increased by 20%, then what would be the per cent increase 8
in the area of the garden ? = 100 160
5
Solution : Hence, Meena’s weight is 160% of Tara’s weight.
By direct formula Example 18 :
20 20 200 The monthly salaries of A and B together amount to `50,000.
% increase in area 44% A spends 80% of his salary and B spends 70% of his salary.
100
If now their saving are the same, then find the salaries of A
If A’s income is r % more than that of B, then B’s income is
less than that of A by and B.
r Solution :
100 %
100 r Let A’s salary by x, then B’s salary (50,000 – x)
If A’s income is r % less than that of B, then B’s income is A spends 80% of his salary and saves 20%
more than that of A by
B spends 70% of his salary and saves 30%
r
100 % Given that
100 r
20% of x = 30% of (50,000 – x)
Example 16 :
If A’s salary is 50 % more than B’s, then by what percent 20 30
B’s salary is less than A’s salary ? x 50, 000 x
Solution : 100 100
Let B’s salary be `x 50 x 30 50, 000
Then, A’s salary = x + 50% of x = 1.5x
B’s salary is less than A’s salary by 100 100
1.5 x x 100 30 50, 000
100 % 33.33% x = 30,000
1.5x 3 100
Shortcut method,
A’s salary `30,000
50
B’s salary is less than A’s salary by 100 % B’s salary = `50,000 – `30,000 = `20,000
100 50
y
o
u
Percentage B-57

rs
m
a
h
b
o
o
b
.w
1. If 3x + 7 = x2 + M = 7x + 5, what is the value of 120% of M? 10. When the price of a pressure cooker was increased by 15%,

o
the sale of pressure cookers decreased by 15%. What was

rd
(a) 8.90 (b) 9.90
the net effect on the sales?

p
(c) 9.98 (d) None of these

re
2. p is six times as large as q. The percent that q is less than (a) 15% decrease (b) no effect

s
s
p, is : (c) 2.25% increase (d) 2.25% decrease

.c
o
11. If 12% of 75% is greater than 5% of a number by 75, the

m
2 number is
(a) 16 (b) 60
3 (a) 1875 (b) 1890
(c) 1845 (d) 1860
1
(c) 83 (d) 90 12. When the price of sugar was increased by 32%, a family
3 reduced its consumption in such a way that the expenditure
3. If two numbers are respectively 20% and 50% of a third on sugar was only 10% more than before. If 30 kg were
number, what is the percentage of the first number to the consumed per month before, find the new monthly
second ? consumption.
(a) 10 (b) 20 (c) 30 (d) 40 (a) 20 kg (b) 25 kg
4. A sum of ` 4558 is divided among A, B and C such that (c) 30 kg (d) None of these
A receives 20% more than C, and C receives 25% less 13. A’s income is 60% of B’s income, and A’s expenditure is
than B. What is A's share in the amount ? 70% of B’s expenditure. If A’s income is 75% of B’s
(a) ` 1548 (b) ` 1720 expenditure, find the ratio of A’s savings to B’s savings.
(c) ` 1290 (d) ` 1345 (a) 5 : 1 (b) 1 : 5
5. The digit at unit place of a two-digit number is increased by (c) 3.5 : 1 (d) 2 : 7
100% and the digit at ten places of the same number is 14. The ratio of salary of a worker in July to that in June was
increased by 50%. The new number thus formed is 19 more
1 1
than the original number. What is the original number? 2 : 2 , by what % the salary of July more than salary of
2 4
(a) 22 (b) 63
June. Also find by what %, salary of June was less than that
(c) 24 (d) None of these
of July.
6. The owner of a boutique decides to calculate the per centage
of customers who purchase hats. If 40 per cent of the store’s 1 1
(a) 11 % and 10% (b) 10% and 11 %
customers decide to purchase items, and of those customers 9 9
15 percent purchase hats, then what per cent of the store’s
1
customers purchase hats ? (c) Both 10% (d) Both 11 %
(a) 4% (b) 6% (c) 15% (d) 24% 9
7. Groundnut oil is now being sold at ` 27 per kg. During last 15. In a housing society, 30 per cent of the residents are men
month its cost was ` 24 per kg. Find by how much % a over the age of 18 and 40 per cent are women over the age of
family should reduce its consumption, so as to keep the 18. If there are 24 children living in the housing society, then
expenditure same. how many total residents live ?
1 1 (a) 32 (b) 80 (c) 94 (d) 112
(a) 11 % (b) 11 %
9 11 16. There is an increase of 30% in the production of milk
9 1 chocolates in Amul Dairy in one month. If now it is 9,100
(c) 11 % (d) 9 %
10 10 milk chocolates per month, what was it one month ago?
8. 10% of the inhabitants of a village having died of cholera, a (a) 10,000 chocolates (b) 9000 chocolates
panic set in, during which 25% of the remaining inhabitants
(c) 8000 chocolates (d) 7000 chocolates
left the village. The population is then reduced to 4050. Find
the number of original inhabitants. 17. In a college election between two rivals, a candidate who
(a) 5000 (b) 6000 (c) 7000 (d) 8000 got 40% of the total votes polled, was defeated by his rival
9. Chunilal invests 65% in machinery, 20% in raw material and by 160 votes. The total number of votes polled was
still has ` 1,305 cash with him. Find his total investment. (a) 900 (b) 800 (c) 700 (d) 600
(a) ` 6,500 (b) ` 7, 225 18. A scooter costs ` 25, 000 when it is brand new. At the end
(c) ` 8,500 (d) None of these of each year, its value is only 80% of what it was at the
y
o
u
B-58 Percentage

rs
beginning of the year. What is the value of the scooter at

m
27. By reduction of 20% in the price of oranges, one can

a
the end of 3 years? purchase5 oranges morefor ` 2.50. Find the reduced price

h
of the oranges per dozen and also the original price.

b
(a) ` 10,000 (b) `12,500

o
(a) 120 paise, 140 paise (b) ` 0.8, `1.5

o
(c) ` 12,800 (d) ` 12,000

b
(c) ` 1.0, ` 1.5 (d) ` 1.2., ` 1.5

.w
19. The income of A is 150% of the income of B and the income 28. An inspector rejects 0.08% of the metres as defective. How

o
many metres will he examine to reject 2 metres?

rd
of C is 120% of the income of A. If the total income of A, B
(a) 200 m (b) 250 m

p
and C together is ` 86000, what is C’s income ?

re
(c) 2500 m (d) 3000 m
(a) ` 30000 (b) ` 32000

s
29. In a certain school, 20% of students are below 8 years of

s
.c
(c) ` 20000 (d) ` 36000 2

o
age. The number of students above 8 years of age is of

m
20. If the price of sugar is increased by 7%, then by how much 3
per cent should a housewife reduce her consumption of the number of students of 8 years age which is 48. What is
sugar, to have no extra expenditure? the total number of students in the school?
(a) 72 (b) 80
(a) 7 over 107% (b) 107 over 100%
(c) 120 (d) None of these
(c) 100 over 107% (d) 7% 30. A positive number is by mistake divided by 6 instead of
21. A student X passes his examination with 515 marks, having being multiplied by 6. What is the % error on the basis of
scored 3% above the minimum.If Y had obtained 710 marks, correct answer?
(a) 3 (b) 97 (c) 17 (d) 83
what % would have been above the minimum?
31. From the salary of an officer, 10% is deducted as house rent,
(a) 40% (b) 42% 20% of the rest, he spends on conveyance, 20% of the rest
(c) 50% (d) Cannot be determined he pays as income tax and 10% of the balance, he spends on
clothes. Then , he is left with ` 15,552. Find his total salary.
22. Ravi’s salary is 150% of Amit’s salary. Amit’s salary is 80%
(a) ` 25,000 (b) ` 30,000
of Ram’s salary. What is the ratio of Ram’s salary to Ravi’s
(c) ` 35,000 (d) ` 40,000
salary ? 32. If the radius of a circle is diminished by 10%, the area is
(a) 1 : 2 (b) 2 : 3 (c) 5 : 6 (d) 6 : 5 diminished by
23. In a shipment of 120 machine parts, 5 per cent were defective. (a) 36% (b) 20% (c) 19% (d) 10%
33. Anthony got 30% of the maximum marks in an examination
In an another shipment of 80 machine parts, 10 per cent
and failed by 10 marks. However, Amar who took the same
were also defective. For the two shipments combined, what examination, got 40% of the total marks and got 15 more
per cent of the machine parts were defective ? than the passing marks in the examination. What were the
(a) 6.5 % (b) 7.0% (c) 7.5% (d) 8.0% passing marks in the examination ?
(a) 35 (b) 250 (c) 75 (d) 85
28 34. In an election between two candidates, 75% of the voters
24. The sum of two numbers is of the first number. The cast their votes, out of which 2% of the votes were declared
25
invalid. A candidate got 9261 votes which were 75% of total
second number is what percent of the first? valid votes. Find the total number of votes enrolled in that
(a) 12% (b) 14% (c) 16% (d) 18% election.
25. In a class, 65% of the students are boys. On a particular day (a) 16080 (b) 16800 (c) 18600 (d) 16008
80% of girl students were present. What was the fraction of 35. Peter could save 10% of his income. But two years later
boys who were present that day if the total number of when his income is increased by 20%, he could save the
students present that day was 70%? same amount only as before. By how much percent has his
expenditure increased?
2 28
(a) (b) 2 1
3 65 (a) 22% (b) 22 % (c) 23 % (d) 24%
9 3
5 42
(c) (d) 36. A screw driver and a hammer currently have the same price.
6 65 If the price of a screw driver rises by 5% and the price of
26. In a competitive examination in State A, 6% candidates got hammer goes up by 3%, then how much more will it cost to
selected from the total appeared candidates. State B had an buy 3 screw drivers and 3 hammers ?
equal number of candidates appeared and 7% candidates (a) 3% (b) 4% (c) 5% (d) 8%
got selected with 80 more candidates got selected than A. 37. A company bought a total of 60 computers and 20 printers
What was the number of candidates appeared from each to modernise billing operations. If the price of each computer
State? was three times the price of each printer then what per cent
of the total cost of the purchase was the total cost of the
(a) 7600 (b) 8000 printers ?
(c) 8400 (d) Data inadequate (a) 10% (b) 11% (c) 15% (d) 20%
y
o
u
Percentage B-59

rs
both the games is 234. The number of students who neither

m
38. What is the total number of candidates at an examination, if

a
31% fail, and the number of those who pass exceeds the play basketball nor tennis is 52%. Determine the total number

h
b
number of those who fail by 247? of students in the college.

o
o
(a) 605 (b) 560 (c) 650 (d) 1,650 (a) 750 (b) 960 (c) 900 (d) 850

b
.w
39. In an election between two candidates, the candidate who 48. The length of a rectangular plot is increased by 25%. To

o
gets 30% of the votes polled is defeated by 15,000 votes. keep its area unchanged, the width of the plot should be :

rd
What is the number of votes polled by the winning

p
(a) kept unchanged (b) increased by 25%

re
candidate?
(c) increased by 20% (d) reduced by 20%

s
(a) 11,250 (b) 15,000 (c) 26,250 (d) 37,500

s
.c
40. In measuring the side of a square, an error of 5% in excess is 49. A store raised the price of an item by exactly 10 per cent.

o
m
made. The error % in the calculated area is, Which of the following could not be the resulting price of
the item ?
1 3 3
(a) 10 % (b) 10 % (c) 1 % (d) 25% (a) ` 5.50 (b) ` 7.60
4 4 4
(c) ` 11.00 (d) ` 12.10
41. If A’s salary is 25% higher than B’s salary, then how much
per cent is B’s salary lower than A’s ? 50. When the cost of petroleum increases by 40%, a man reduces
his annual consumption by 20%. Find the percentage change
1 1 in his annual expenditure on petroleum.
(a) 16 % (b) 20% (c) 25% (d) 33 %
3 3 (a) 20% (b) 16%
42. In the month of January, the Railway Police caught 4000 (c) 12% (d) 40%
ticketless travellers. In February, the number rise by 5%. 51. A reduction of 20% in the price of an apple enable a man to
However, due to constant vigil by the Police and the Railway buy 10 apple more for ` 54. The reduced price of apple per
staff, the number reduced by 5% and in April it further dozen is
reduced by 10%. The total number of ticketless travellers (a) ` 4.32 (b) ` 12.96
caught in the month of April was: (c) ` 10.80 (d) ` 14.40
(a) 3125 (b) 3255 (c) 3575 (d) 3591 52. After three successive equal percentage rise in the salary
43. The total population of a village is 5000. The number of the sum of 100 rupees turned into 133 rupees and 10 paise.
males and females increases by 10% and 15% respectively Find the percentage rise in the salary.
and consequently the population of the village becomes
(a) 13% (b) 10%
5600. What was the number of males in the village?
(a) 2000 (b) 2500 (c) 3000 (d) 4000 (c) 15% (d) 14%
44. An empty fuel tank of a car was filled with A type petrol. 53. In an examination in which full mark were 500, A got 10%
When the tank was half-empty, it was filled with B type less than B. B got 25% more than C. C got 20% less than D.
petrol. Again when the tank was half-empty, it was filled If a got 360 marks what % of full mark was obtained by D.
(a) 90% (b) 80%
with A type petrol. When the tank was half-empty again, it
(c) 50% (d) 60%
was filled with B type petrol. What is the percentage of A
54. In an examination 35% of total student failed in Hindi 45%
type petrol at present in the tank? failed in English and 20% in both. Find the percentage of
(a) 33.5% (b) 37.5% (c) 40% (d) 50% those who passed in both the subjects.
45. In an examination, 65% students passed in Civics and 60% (a) 40% (b) 60%
in Histroy, 40% passed in both of these subjects. If 90 (c) 50% (d) 30%
students failed in History and Civics both, then what is the 55. In an examination 80% of student passed in English 85% in
total number of students? mathematics and 75% in both English and mathematics. If
40 student failed in both the subject find total number of
(a) 600 (b) 650 (c) 700 (d) 750
students.
46. 40% of the people read newspaper X, 50% read newspaper (a) 350 (b) 400
Y and 10% read both the papers. What percentage of the (c) 450 (d) 600
people read neither newspaper? 56. The length of a rectangle is increased by 15% and breadth
(a) 10% (b) 15% (c) 20% (d) 25% decreased by 15%. Then the area of the new rectangle is
47. 40% of the students in a college play basketball, 34% of the (a) unchanged (b) increased by 2.25%
students play tennis and the number of students who play (c) decreased by 2.25% (d) increased by 15%
y
o
u
B-60 Percentage

rs
m
ANSWER KEY

a
h
1 (b) 8 (b) 15 (b) 22 (c) 29 (d) 36 (b) 43 (c) 50 (c)

b
o
2 (c) 9 (d) 16 (d) 23 (b) 30 (b) 37 (a) 44 (b) 51 (b)

o
b
3 (d) 10 (d) 17 (b) 24 (a) 31 (b) 38 (c) 45 (a) 52 (b)

.w
4 (a) 11 (a) 18 (c) 25 (d) 32 (c) 39 (c) 46 (c) 53 (b)

o
rd
5 (d) 12 (b) 19 (d) 26 (b) 33 (d) 40 (a) 47 (c) 54 (a)

p
6 (b) 13 (b) 20 (a) 27 (d) 34 (b) 41 (b) 48 (d) 55 (b)

re
7 (a) 14 (a) 21 (b) 28 (c) 35 (b) 42 (d) 49 (b) 56 (c)

s
s
.c
o
m
1. (b) If 3x + 7 = x2 + M = 7x + 5 (a) 22 34 12
ie, 3x + 7 = 7x + 5 (b) 63 96 33
(c) 24 38 14
1
or, 4x = 2, x Obviously, (d) is the correct option.
2 6. (b) Let total customers be 100
and 3x + 7 = x2 + M 40 of th em purchase item and 15% of 40 =
1 3 1 1 15
or, M 7 M 8 40 6 customers purchase hats which is only
4 2 4 2 100
1 6% of total customers.
M8 , 120% of M = 9.90
4 27 – 24 100
7. (a) % change in rate 100 %
2. (c) p = 6q. So, q is less than p by 5q. 24 8
For fixed expenditure, % change in consumption
5q
Required percentage = 100 % % change in rate
p 100
100 + % change in rate
5q 1 100 / 8 100 1
= 100 % 83 % 100 % 11 %
6q 3 1 9 9
100 1+
3. (d) Let the third number be 100. Then, the first and second 8
numbers will be 20 and 50, respectively. 8. (b) Let the total number of original inhabitants be x. Then,
20 (100 – 25)% of (100 –10)% of x = 4050
Required % = 100 40%
50 75 90 27
x 4050 x 4050
3x 100 100 40
4. (a) Let B get ` x. Then C gets = 75% of x
4 4050 40
x 6000.
3x 120 3 x 9x 27
and A gets = 120% of
4 100 4 10 Number of original inhabitants = 6000.
9x 3x 9. (d) Let he had originally ` x. Then
Now, + x = 4558 65% of x + 20 % of x + 1305 = x
10 4
0.65x + 0.2 x + 1305 = x
53 x 4558 20 0.15 x = 1305 x = ` 8700
= 4558 x= = 1720
20 53 His total investment = 65% of 8700 + 20% of 8700
= 85% of 8700 = ` 7395
9x 9 1720
Hence, A's share = =` = ` 1548
10 10 (common % change)2
10. (d) Net effect on sale = –
5. (d) Working with options, we have 100
Original New Difference
(15) 2
number number 2.25% decrease
100
y
o
u
Percentage B-61

rs
m
11. (a) Let the number be x, 17. (b) Let total number of votes polled be x.

a
12 75 5 Then, votes polled by other candidate

h
Then, x– x 75

b
100 100 100 = (100 – 40)% of x = 60% of x

o
Now 60% of x – 40% of x = 160

o
b
9x 5x 4x

.w
– 75 75 20x
100 100 100 160 x 800 votes

o
100

rd
75 100 18. (c) Cost of scooter = `25,000

p
x= 1875

re
4 Cost of scooter decrease 20% each year with respect

s
12. (b) Since, expenditure = price × consumption to the cost of scooter at the end of 3 years

s
.c
o
132 20
3

m
110% of 30 new consumption
100 = 25, 000 1
100
110 132 4 4 4
30 new consumption 25,000
100 100 = `12,800
5 5 5
New consumption = 25 kg 19. (d) Suppose Income of B = ` x
13. (b) Let B’s Income = ` x
150 3x
3 Income of A = x `
A’s Income = ` x 100 2
5
And B’s expenditure = ` y 120 3 x
Income of C =
100 2
7
A’s expenditure = ` y 6 3x 9x
10
5 2 5
3 3 7
Also, x y 3x 9x
5 4 10 x 86000
7 2 5
y y
A savings x y 8 y/8 10 x 15 x 18 x
86000
B savings 3 7 3 7 7 21y 7 10
x y y y y
5 10 5 8 10 40 10 43x = 860000
5 x = 20000
1: 5
25 9
20000 = ` 36000
So, income of C =
5
5
14. (a) Let the salary of July be ` x 20. (a) % reduction in consumption
2
% change in price
9 100
and the salary of June be ` x. 100 +% change in price
4
Required percentages 7 7
% %
100 7 107
5 9 5 9
x x x x
2 4 100 and 2 4 100 21. (b) Marks of y 100 % above minimum of y
9 5 Marks of x 100 + % above minimum of x
x x
4 2
710 100 y 710 103
100 + y = 142
100 100 1 515 103 515
% and % 11 % and 10%
9 10 9 y = 42%
15. (b) 30% of the residents are children. 22. (c) Let the salary of Ram be ` 100.
30% of the total residents = 24 Then, salary of Amit = ` 80 and salary of Ravi = ` 120
Total number of residents in the society Ratio of Ram’s salary to Ravi’s salary = 100 : 120
24 =5:6
100 80 23. (b) Total no. of machine parts in both the shipments
30
= (120 + 80) = 200
16. (d) Let one month ago, production be x chocolates.
Then, 130 % of x = 9100 Total defective machine parts in both the shipments
= 120 × 5% + 80 × 10% = 6 + 8 = 14
9100 100
x 7000 chocolates 14
130 Therefore, required % = 100 7%
200
y
o
u
B-62 Percentage

rs
33. (d) Let the maximum marks be x.

m
24. (a) Let the numbers be x and y. Then,

a
Then, x × 30% + 10 = x × 40% – 15

h
28 28 3

b
x y x y x x y x x × 10% = 25 or x = 250

o
25 25 25

o
b
30

.w
y 3 Therefore, passing marks = 250 10 85
100 % 12%. 100

o
rd
x 25 34. (b) Let the total number of votes enrolled be x. Then,

p
25. (d) Let the class has 100 students. Number of votes cast = 75% of x. Valid votes = 98% of

re
s
Number of girls = 35 and number of boys = 65. (75% of x).

s
.c
Since total number of present students = 70 and 75% of [98% of (75% of x)] = 9261

o
number of girls present = 80% of 35 = 28, so number of

m
boys present = 70 – 28 = 42. 75 98 75
x 9261
Required fraction = 42/65. 100 100 100
26. (b) Let the number of candidates appeared from each state
be x. 9261 100 100 100
x 16800.
Then, 7% of x – 6% of x = 80 1% of x = 80 75 98 75
x = 80 × 100 = 8000. 35. (b) Let original income = Rs. 100. Then, saving = ` 10 and
27. (d) Let original price be ` x per orange. Then, expenditure = ` 90.
Reduced rate = (1 – 0.2)x = ` 0.8 x New income = ` 120, New saving = ` 10.
2.50 2.50 25 2.5 1 New expenditure = ` (120 – 10) = ` 110.
5 5 x Increase in expenditure = ` (110 – 90) = ` 20.
0.8x x 8x x 8

1 20 2
Original price of oranges per dozen 12 ` 1.5 Increase % 100 % 22 %.
8 90 9

and Reduced price = ` (0.8)(1.5) = ` 1.2 36. (b) Let the original price of a screw driver and a hammer be
28. (c) Let the inspector examined x metres, ` 100 each.
then 0.08% of x = 2 Then, price of 3 screw drivers and 3 hammers = ` 600
Now, after increase of 5%, the price of 3 screw drivers
x 0.08 200 = ` 315
2 or x 2500 metres
100 0.08 And after 3% increase the price of 3 hammers = ` 309
29. (d) Let the number of students be x. Then, Increased price of 3 hammers and 3 screw drivers
Total number of students of 8 years and above 8 years = ` 624
= (100 – 20)% of x = 80% of x. 24
Therefore, % increase in price = 100 4%
80 600
80% of x = 48 + 2/3 of 48 x = 80 x = 100. 37. (a) Suppose price of the printer = P
100
Price of a computer = 3P
30. (b) Let the number be x. Then, Total cost of 60 computers = 180 P
6x x / 6 35 Total cost of 20 printers = 20 P
% error 100 100 97.2%
6x 36 Total cost of the purchase = 200 P
31. (b) Let the total salary be ` x. Thus total cost of the printers is 10% of the total cost.
Then, (100 – 10)% of (100 – 20)% of (100 – 20)% of 38. (c) Let the total number of candidates = x
(100 – 10)% of x = 15552 Then, number of passed candidates
= (100 – 31)% of x = 69% of x
90 80 80 90 Now, 69 % of x – 31% of x = 247
x 15552
100 100 100 100 38% of x = 247

15552 10000 38 247 100


x 30,000. x 247 x 650
64 81 100 38
32. (c) If the radius is diminised by r%, then 39. (c) Let the total number of votes be x

r2 102 votes polled by winning candidate


Area is diminished by 2r % 2 10 19%
100 100 = (100 – 30)% of x = 70% of x
y
o
u
Percentage B-63

rs
m
Now, 70% of x – 30% of x = 15,000 44. (b) Let the capacity of the tank be 100 litres. Then,

a
h
40% of x = 15,000 Initially : A type petrol = 100 litres.

b
o
After first operation :

o
15000 100

b
x 37,500

.w
40 100
A type petrol 50 litres;

o
2

rd
number of votes polled by winning candidate

p
re
B type petrol = 50 litres.
= 70% of 37500

s
s
After second operation :

.c
70 37500

o
26, 250

m
100 50
A type petrol 50 75 litres;
2
40. (a) If side is increased by a%, area increased by
B type petrol = (50/2) = 25 litres
a2
2a % After third operation :
100
75
A type petrol 37.5 liters;
5 2
1 2
2 5 10 %
100 4
25
41. (b) Let B’s salary be ` 100, B type petrol 50 62.5 litres.
2
then A’s salary = ` 125
Required percentage = 37.5%.
125 100 25
% lesser = 100 100 45. (a) Let the total number of students be x.
125 125
Number of students passed in one or both is given by :
1
= 100 20% n(A B) n(A) n(B) n(A B)
5
= 65% of x + 60% of x – 40% of x
Short cut method:
B’s salary is lower than A’s salary by 65 60 40 85 17
x x x x x.
100 100 100 100 20
25
100 % = 20%
100 25 17 3x
Failed in both x x .
20 20
42. (d) Number of ticketless travellers in April

5 5 10 3x 90 20
4000 1 1 1 90 x 600.
100 100 100 20 3

46. (c) n(A) = 40, n(B) = 50, n(A B) = 10.


21 19 9
4000 3591. n(A B) = n (A) + n (B) – n(A B)
20 20 10
= 40 + 50 – 10 = 80.
43. (c) Let the number of males be x. Then, number of females Percentage reading either or both newspapers
= (5000 – x). = 80%.
10% of x + 15% of (5000 – x) = (5600 –5000) Hence, percentage reading neither newspaper
= (100 – 80)% = 20%
10 15
x (5000 x) 600 47. (c) Let the numnber of students be 100.
100 100
Then number of students who play both the games
10x 75000 15x 60000. = (34 + 40) – (48) = 26

5x 15000 x 3000. If 26 students play both the games, then the total number
of students = 100
y
o
u
B-64 Percentage

rs
Therefore, if 234 students play both the games, then

m
x

a
the total number of students 1 1.1

h
100

b
o
100

o
= 234 900 x

b
26 0.1 x = 10

.w
100
48. (d) Let the original length and breadth be both 10 cm each.

o
Rise in salary is 10%

rd
Then original area = 100 cm2

p
re
New length = 10 × 1.25 = 12.5 cm x 10 9 x
53. (b) 360 = x – x 400

s
Let new breadth be x. Then, 12.5x = 100

s
100 10

.c
when x is mark obtained by B mark obtained by C = y

o
100

m
x 8cm
12.5 25 5y
400 = y + y ×
100 4
2
Hence, % reduction in breadth = 100 20% y = 320
10
mark obtained by D = z
49. (b) 5 + 10% = 5.50
20 4z
10 + 10% = 11 320 = z – z ×
100 5
11 + 10% = 12.10
z = 400
50. (c) First expenditure: Suppose 100 litres of petroleum at
100 units of money per litre, then total expenditure = 400
% of mark obtained of D = 100 80%
100 × 100 units of money = 10000 units of money. 500
Second expenditure: Now 80 litres of petroleum at 140 54. (a) Let A and B be the sets of student who failed in
units of money per litre, total expenditure = 80 × 140 Hindi and English respectively
units of money = 11200 units.
Then n (A) = 35
Expenditure increases by
n (B) = 45
11200 10000
10 12% n A B = 20
10000
Short-cut: Exp1 = PX, Exp2 = 1.4P (0.8X) = 1.12 PX. n A B = n (A) + n (B) – n A B
Directly we see, answer = 12%. = 35 + 45 – 20 = 60
51. (b) Let the original price of apple be ` x /dozen % of student failed in Hindi or English or both = 60%
4x % percentage passed = 100 – 60 = 40%
New price ` = /dozen. 55. (b) Let total no. of student = x
5
Let A and B represent the sets of students who passed
54 54 10 in English and mathematics respectively
= 4x x 12
5 n A B = n (A) + n (B) – n A B

5 1 5 = 80% of x + 85% of x – 75% of x


54
4x x 6 80 85 75 x 90 x 9x
= x x =
100 100 100 100 10
1 5
54 Students failed in both subjects
4x 6
9x x
54 6 =x– =
4x = 10 10
5
4x x
12.96 So, = 40 x = 400
5 10
52. (b) Let rise in salray be x% 56. (c) A=l×b
15 15
100 1
x
1
x
1
x
133.1 A= l l b b = 1.15 l × 0.85 b
100 100
100 100 100
A = 0.9775 A
3 A 0.9775A
x 133.1 100 = 2.25%
1 = 1.331 % change =
100 100 A
y
o
u
rs
m
a
CHAPTER

h
b
6

o
PROFIT & LOSS

o
b
.w
o
rd
p
re
s
s
.c
o
Cost Price : The amount paid to purchase an article or the

m
100 + Profit%
price at which an article is made, is known as its cost price. S.P. = × C.P.
The cost price is abbreviated as C.P. 100
Selling Price : The price at which an article is sold, is known 100 Loss%
as its selling price. S.P. = × C.P.
The selling price is abbreviated as S.P. 100
Profit : If the selling price (S.P.) of an article is greater than the 100 S.P.
cost price (C.P.), then the difference between the selling price C.P. =
100 Profit %
and cost price is called profit.
Thus, If S.P. > C.P., then 100 S.P.
C.P. =
Profit = S.P. – C.P. 100 Loss %
S.P. = C.P. + Profit
C.P. = S.P. – Profit. Example 2 :
Loss : If the selling price (S.P.) of an article is less than the cost A cycle was purchased for ` 1600 and sold for ` 1400. Find
price (C.P.), then the difference between the cost price (C.P.) the loss and loss %.
and the selling price (S.P.) is called loss. Solution :
Thus, if S.P. < C.P., then C.P. of the cycle = ` 1600
Loss = C.P. – S.P. S.P. of the cycle = ` 1400
C.P. = S.P. + Loss Since S.P < C.P, so there is a loss.
S.P. = C.P. – Loss Loss = C.P. – S.P.
= ` 1600 – ` 1400 = ` 200.
Example 1 :
An article was bought for Rs 2000 and sold for Rs 2200. Loss 200 1
Loss % = 100 100 12 %
Find the gain or loss. C.P. 1600 2
Solution : Example 3 :
C.P. of the article = ` 2000 By selling a table for ` 330, a trader gains 10%. Find the cost
S.P. of the article = ` 2200 price of the table.
Since S.P. > C.P. So there is gain. Solution :
Gain (profit) = S.P. – C.P. S.P. = ` 330, Gain = 10%
= ` 2200 – ` 2000 = ` 200
Profit and Loss percentage 100
C.P. = S.P .
The profit per cent is the profit that would be obtained for a 100 Gain %
C.P. of ` 100.
100
Similarly, the loss per cent is the loss that would be made =` 330
100 10
for a C.P. of ` 100.
100
Profit = 330 = ` 300.
Profit per cent = 100 110
C.P. Example 4 :
Loss A sells a bicycle to B at a profit of 20% and B sells it to C at
Loss per cent = 100
C.P. a profit of 25%. If C pays ` 225 for it, what did A pay for it.
Solution :

100 100
C.P. × Profit % C.P. of A = 225
Profit = 100 20 100 25
100
100 100
C.P. × Loss % 225 = ` 150.
Loss = 120 125
100
y
o
u
B-66 Profit & Loss

rs
Example 5 : Example 7 :

m
a
A mobile phone is sold for ` 5060 at a gain of 10%. What A shopkeeper sells the goods at 44% loss on cost price but

h
would have been the gain or loss per cent if it had been sold uses 30% less weight. What is his percentage profit or loss?

b
o
for ` 4370 ? Solution :

o
b
Solution : Given, x = – 44, y = 30

.w
S.P. = ` 5060, gain = 10% 30 44

o
The overall % gain/loss = 100 % = – 20%

rd
5060 100 100 30

p
C.P. = = ` 4600. Which represents loss being a negative expression.

re
100 10
Example 8 :

s
2nd S.P. = ` 4370

s
A cloth merchant says that due to slump in the market, he

.c
Since, S.P. < C.P., so there is loss.

o
sells the cloth at 10% loss, but he uses a false metre-scale

m
(4600 4370) 100 and actually gain 15%. Find the actual length of the scale.
Loss % = = 5%
4600 Solution :
If a man buys x items for `y and sells z items for `w, then True scale 100 gain%
False scale 100 loss%
xw
the gain or loss percent made by him is 1 100% 100 100 15
zy
False scale 100 10
Example 6 :
100 90
Some articles were bought at 6 for ` 5 and sold at 5 for False scale 78.26 cm
` 6. Gain percent is : 115
Solution : Example 9 :
Suppose, number of articles bought = L.C.M of 6 and 5 = 30 A dishonest dealer professes to sell his goods at cost price,
but he uses a weight of 960 g for the kg weight. Find his
5 gain per cent.
C.P of 30 articles = ` 30 25
6 Solution :
Error = 1 kg – 960 g
5 = 1000 g – 960 g = 40 g.
S.P. of 30 articles = ` 30 = ` 36
6
40
Gain % = 100
11 1000 40
Gain % = 100 % 44%
25 40 1
= 100 4 %
Shortcut method : 960 6
Quantity Price GOODS PASSING THROUGH SUCCESSIVE HANDS
When there are two successive profits of a% and b%, then
6 5 the resultant profit per cent is given by

5 6 ab
a b %
100
xw When there is a profit of a% and loss by b% in a transaction,
% profit 1 100%
yz then the resultant profit or loss per cent is given by

6 6 ab
1 100% a b % , according to the + ve or – ve sign
5 5 100
respectively.
11
100% 44% When cost price and selling price are reduced by the same
25 amount (A) and profit increases then cost price (C.P.)
DISHONEST DEALING
[Initial profit % Increase in profit %] A
=
Error Increase in profit %
Gain % 100
True value Error Example 10 :
A table is sold at a profit of 20%. If the cost price and selling
True Scale 100 gain% price are ` 200 less, the profit would be 8% more. Find the
False Scale 100 loss% cost price.
Solution :
A merchant uses y% less weight/length and sells his By direct method,
good at gain/loss of x%. The overall % gain/loss is given
(20 8) 200
y x C.P. = ` = ` 28 × 25 = ` 700.
100 % 8
by- 100 y
y
o
u
Profit & Loss B-67

rs
m
If cost price of x articles is equal to the selling price of y
NOTE :

a
h
x y

b
articles, then profit/loss percentage = 100 %, When no discount is given, ‘selling price’ is the same as ‘marked

o
y price’.

o
b
according to +ve or –ve sign respectively. • Discount = Marked price × Rate of discount.

.w
• S.P. = M.P. – Discount.

o
rd
Example 11 : Discount
• Discount % = 100 .

p
If the C.P. of 15 tables be equal to the S.P. of 20 tables, find M.P.

re
the loss per cent. • Buy x get y free i.e., if x + y articles are sold at cost price of

s
s
Solution : y

.c
x articles, then the percentage discount = x y × 100.

o
By direct method,

m
5
Profit/Loss% = 100 = 25% loss, since it is –ve. Example 14 :
20 How much % must be added to the cost price of goods so
that a profit of 20% must be made after throwing off a
Example 12 : discount of 10% from the marked price?
If the C.P. of 6 articles is equal to the S.P. of 4 articles. Find Solution :
the gain per cent. Let C.P. = ` 100, then S.P. = Rs 120
Solution : Also, Let marked price be ` x. Then
Let C.P. of an article be ` x; then, 120 100 1
90% of x = 120 x 133
C.P. of 6 articles = ` 6x 90 3
C.P. of 4 articles = ` 4x 1
M.P. should be ` 133
But S.P. of 4 articles = C.P. of 6 articles 3
S.P. of 4 articles = 6x 1
or M.P. = 33 % above C.P..
Thus, gain = S.P – C.P. = ` (6x – 4x) = ` 2x 3
Example 15 :
2x At a clearance sale, all goods are on sale at 45% discount. If
Gain % = 100 50
4x I buy a skirt marked ` 600, how much would I need to pay?
Thus, gain in the transaction = 50% Solution :
M.P. = ` 600, Discount = 45%
Example 13 : M.P. Discount% 600 45
By selling 33 metres of cloth, a man gains the sale price of Discount = = = ` 270.
100 100
11 metres. The gain % is
Solution : S.P. = M.P. – Discount
Gain = S.P. of 33 metres – C.P. of 33 metres = ` 600 – ` 270 = ` 330.
= S.P. of 11 metres Hence, the amount I need to pay is ` 330.
S.P. of 22 metres = C.P. of 33 metres Alternate method :
S.P = M.P × 0.55
gain = 600 × 0.55 = `330
% gain 100
C.P.of metres • A man purchases a certain number of articles at x a rupee
and the same number at y a rupee. He mixes them together
S.P.of 11 metres and sells them at z a rupee. Then his gain or loss %
100
C.P. of 33 metres
2xy
S.P.of 11 metres 11 = 1 ×100 according as the sign is +ve or –ve.
100 100 = 50% z(x y)
S.P. of 22 metres 22
• If two items are sold, each at ` x, one at a gain of p% and the
Shortcut method :
If on selling ‘x’ articles a man gains equal to the S.P. of y articles. p2
Then, other at a loss of p%, there is an overall loss given by %.
100
y 11 11 Note that in such cases there is always a loss.
% gain 100 100 100 50%
x y 33 11 22
2p2 x
Marked Price : The price on the lable is called the marked price The absolute value of the loss is given by .
1002 p2
or list price.
The marked price is abbreviated as M.P. • If CP of two items is the same and % Loss and % Gain on the
Discount : The reduction made on the ‘marked price’ of an two items are equal, then net loss or net profit is zero.
article is called the discount.
y
o
u
B-68 Profit & Loss

rs
Example 16 : Solution :

m
a
A shopkeeper sold two radio sets for ` 792 each, gaining Given, m = 10, n = 5, z = ` 462

h
b
10% on one, and losing 10% on the other. Then he

o
Solution : 100 2 z

o
Using the formula, C .P.

b
When selling price of two articles is same and 100 m 100 n

.w
% gain = % loss

o
100 2 462

rd
then there will be always loss.
we get C. P. for Mohit =

p
100 10 100 5

re
(10)2
% 1%

s
and overall % loss When two different articles sold at same S.P. x and y are %

s
100

.c
gain (or loss) on them. Then overall % gain or loss

o
Example 17 :

m
A man bought two housing apartments for ` 2 lakhs each. 100(x y) 2 xy
He sold one at 20% loss and the other at 20% gain. Find his (100 x) (100 y )
gain or loss. The above expression represents overall gain or loss
Solution : according to its sign.
When C.P. of two articles is same and Example 20 :
% gain = % loss A man sold two watches for ` 1000 each. On one he gains
Then, on net, there is no loss, no gain 25% and on the other 20% loss. Find how much % does he
If an article is sold at a price S.P1.., then %gain or % loss is gain or loss in the whole transaction?
x and if it is sold at a price S.P2... then % gain or % loss is y. Solution :
If the cost price of the article is C.P., then When S1 = S2, then
overall % gain or % loss
S .P1 S .P2 C.P. S .P1 S .P2
100 x 100 y 100 x y 100 x y 2 xy
= 100 x 100 y
Where x or y is negative, if in indicates a loss, otherwise it
is positive.
Example 18 : 100 25 20 2 25 20
By selling a radio for ` 1536, Ramesh lost 20%. What per 100 25 100 20
cent shall he gain or lose by selling it for `2000?
Solution : 100
% loss ( it is –ve)
Given, S.P1 = 1536, x = – 20 (–ve sign indicates loss) 41
S.P2 = ` 2000, y = ?
Using the formula, Example 21 :
S .P1 S.P2 After allowing a discount of 12% on the marked price of an
article, it is sold for ` 880. Find its marked price.
100 x 100 y Solution :
1536 2000 S.P. = ` 880 and Discount % = 12
we get Let M.P. = x
100 20 100 y
M.P. Discount % x 12 3
2000 80 1 Discount = = = x
100 y 104 100 100 25
1536 6
Now, M.P. = S.P. + Discount
1
y 4 % 3
6 x = 880 + x
25
1
Thus, Ramesh has a gain of 4 % by selling it for ` 2,000
6 3 22x
x x 880 880
It A sells an article to B at a gain/loss of m% and B sells it to 25 25
C at a gain/loss of n%. If C pays ` z for it to B then the cost
880 25
2 x= 40 25 = ` 1000
100 z 22
price for A is .
(100 m) 100 n Marked price of the article is ` 1000.
Example 22 :
Where m or n is negative, of it indicates a loss, otherwise it A shopkeeper offers his customers 10% discount and still
positive. makes a profit of 26%. What is the actual cost to him of an
Example 19 : article marked ` 280?
Mohit sells a bicycle to Rohit at a gain of 10% and Rohit Solution :
again sells it to Jyoti at a profit of 5%. If Jyoti pays `462 to M.P. = ` 280 and Discount % = 10
Rohit. What is the cost price of the bicycle for Mohit.
y
o
u
Profit & Loss B-69

rs
Example 26 :

m
M.P. Discount % 280 10 ` 28

a
Discount = = If an article is marked at `1000000, and on purchasing a

h
100 100 person gets discount of 30%, 30%, 20%, 10% and 10%

b
o
S.P. = M.P. – Discount = ` 280 – ` 28 = ` 252 successively. Find his over all discount percentage.

o
Now, S.P. = ` 252 and profit = 26%

b
Solution :

.w
100 [1–(.70×.70×.80×.90×.90)] × 100 = 68.24%

o
C.P. = 100 Profit % S.P.

rd
Example 27 :

p
re
100 An article is listed at ` 65. A customer bought this article

s
252 = ` 200 for ` 56.16 and got two successive discounts of which the

s
100 26

.c
Hence, the actual cost of the article is ` 200. first one is 10%. The other rate of discount of this scheme

o
m
that was allowed by the shopkeeper was :
SUCCESSIVE DISCOUNT: Solution :
In successive discounts, first discount is subtracted from the Price of the article after first discount
marked price to get net price after the first discount. Taking this 65 – 6.5 = ` 58.5
price as the new marked price, the second discount is Therefore, the second discount
calculated and it is subtracted from it to get net price after the
58.5 56.16
second discount. Continuing in this manner, we finally obtain = 100 4%
the net selling price. 58.5
In case of successive discounts a% and b%, the effective discount
Example 28 :
ab A shopkeeper offers 5% discount on all his goods to all his
is a b %
100 customers. He offers a further discount of 2% on the reduced
price to those customers who pay cash. What will you
ay by actually have to pay for an article in cash if its M.P. is
or Single discount = 1 1
100
1
100
100
` 4800?
Solution :
Example 23 : M.P. = ` 4800
Find the single discount equivalent to successive discounts First discount = 5% of M.P.
of 15% and 20%.
5
Solution : = 4800 = ` 240
By direct formula, 100
Net price after discount = ` 4800 – ` 240
ab = ` 4560
Single discount = a b %
100 Second discount = 2% of ` 4560
15 20 2
= 15 20 % = 32% = 4560 = ` 91.20
100 100
If three or more successive discounts on an article are a%, Net price after discount = ` 4560 – ` 91.20
b%, c% and respectively then a single discount to the = ` 4468.80
successive discounts will be By Direct Method :
a% b% c% d% 5 2
1 1 1 1 1 % S.P. 4800 1 1 ` 4468.80
100 100 100 100 100 100
A person buys two items for ` A and sells one at a loss of
Example 24 :
l % and other at a gain of g%. If each item was sold at the
Find single equivalent discount of successive discount of same price, then
30%, 20%, 40% and 10%. (a) The cost price of the items sold at loss
Solution :
Single discount = [1– (1–.30) × (1–.20) × (1–.40) × A(100+% gain)
=
(1–.10)×100] = 69.76% 100–% loss + 100+% gain
(b) The cost price of the item sold at gain
Example 25 :
Find the S.P of an article whose M.P is ` 9988467 giving A(100 % loss)
=
successive discount of 50%, 40% and 10%. 100–% loss + 100+%gain
Solution :
Example 29 :
Equivalent discount = [1– (.50×.60×.90)× 100] = 73%
Ramesh buys two books for `410 and sells one at a loss of
100 73 20% and the other at a gain of 25%. If both the books are
S.P = 9988467 = 2696886.09
100 sold at the same price. Find the cost price of two books.
y
o
u
B-70 Profit & Loss

rs
Solution : Example 31 :

m
The sale price of an article including the sales tax is ` 616.

a
Given, A = 410

h
The rate of sales tax is 10%. If the shopkeeper has made a

b
410 100 25

o
Cost price of the book sold at loss = profit of 12%, then the cost price of the article is :

o
100 20 100 25 Solution :

b
.w
410 125 Let the CP of the article be ` x

o
= `250/-
Then, SP = x 1.12 1.1

rd
80 125
Cost price of the book sold at profit

p
Now, x 1.12 1.1 616

re
410 100 20

s
616

s
100 20 100 25 x ` 500

.c
1.232

o
410 80 Example 32 :

m
= `160/- Pure ghee costs ` 100 per kg. After adulterating it with
80 125
SALES TAX vegetable oil costing ` 50 per kg, a shopkeeper sells the
To meet government’s expenditures like construction of roads, mixture at the rate of ` 96 per kg, thereby making a profit of
railway, hospitals, schools etc. the government imposes different 20%. In what ratio does he mix the two?
types of taxes. Sales tax (S.T.) is one of these tax.
Solution :
Sales tax is calculated on selling price (S.P.)
100
Mean cost price = ` 96 = ` 80 per kg.
NOTE : 120
If discount is given, selling price is calculated first and then By the rule of alligation:
sales tax is calculated on the selling price of the article.
C.P. of 1 kg ghee C.P. of 1 kg oil
100 50
Example 30 :
Sonika bought a V.C.R. at the list price of ` 18,500. If the rate of
sales tax was 8%, find the amount she had to pay for
purchasing the V.C.R.
Solution :
List price of V.C.R. = ` 18,500 Mean price
Rate of sales tax = 8% 80
Sales tax = 8% of ` 18,500
8
= 18500 = ` 1480 30 20
100
So, total amount which Sonika had to pay for purchasing Required ratio = 30 : 20 = 3 : 2.
the V.C.R. = ` 18,500 + ` 1480
= ` 19,980.

1. If the manufacturer gains 10%, the wholesale dealer 15% 4. A trader wants 10% profit on the selling price of a product
and the retailer 25%, then find the cost of production of a whereas his expenses amount to 15% on sales. What
table, the retail price of which is ` 1265? should be his rate of mark up on an article costing ` 9?
(a) ` 800 (b) ` 1000 (c) ` 900 (d) ` 600 2
2. The price of a jewel, passing through three hands, rises on (a) 20% (b) 66 %
3
the whole by 65%. If the first and the second sellers earned 100
20% and 25% profit respectively, find the percentage profit (c) 30% (d) %
earned by the third seller. 3
(a) 20% (b) 10% 5. If 11 lichchus are bought for 10 paise and 10 lichchus are
(c) 25% (d) No gain or loss sold for 11 paise, the gain % is
(a) 10% (b) 11% (c) 20% (d) 21%
1 6. A man sold 10 eggs for 5 rupees and gained 20%.How many
3. A man sold his book for ` 891, thereby gaining of its
10 eggs did he buy for 5 rupees?
cost price. Find his cost price. (a) 10 eggs (b) 12 eggs
(a) ` 850 (b) ` 810 (c) ` 851 (d) ` 840 (c) 14 eggs (d) 16 eggs
y
o
u
Profit & Loss B-71

rs
A man sold two steel chairs for ` 500 each. On one he gains

m
7. A person sells 36 oranges per rupee and suffers a loss of 19.

a
4%. Find how many oranges per rupee to be sold to have a 20% and on other, he loses 12%. How much does he gain or

h
gain of 8%? lose in the whole transaction?

b
o
(a) 30 (b) 31 (c) 32 (d) 33 (a) 1.5% gain (b) 2% gain

o
b
8. Coconuts were purchased at ` 150 per hundred and sold at (c) 1.5% loss (d) 2% loss

.w
` 2 per coconut. If 2000 coconuts were sold, what was the 20. A firm of readymade garments makes both men’s and

o
women’s shirts. Its average profit is 6% of the sales. Its

rd
total profit made?
profit in men’s shirts average 8% of the sales and women’s

p
(a) ` 500 (b) ` 1000 (c) ` 1500 (d) ` 2000

re
shirts comprise 60% of the output. The average profit per
9. A shopkeeper’s price is 50% above the cost price. If he

s
sale rupee in women shirts is

s
allows his customer a discount of 30% what profit does he

.c
(a) 0.0466 (b) 0.0666
make?

o
(c) 0.0166 (d) None of these

m
(a) 5% (b) 10% (c) 15% (d) 20% 21. A man purchases two watches at ` 560. He sells one at 15%
10. A shopkeeper purchases 10 kg of rice at ` 600 and sells at a profit and other at 10% loss. Then he neither gains nor loss.
loss as much the selling price of 2 kg of rice . Find the sale Find the cost price of each watch.
rate of rice/ kg. (a) ` 224, ` 300 (b) ` 200, ` 300
(a) ` 60 per kg (b) ` 50 per kg (c) ` 224, ` 336 (d) ` 200, ` 336
(c) ` 80 per kg (d) ` 70 per kg 22. A man bought a horse and a carriage for ` 3000. He sold the
11. A businessman allows a discount of 10% on the written horse at a gain of 20% and the carriage at a loss 10%, thereby
price. How much above the cost price must he mark his gaining 2% on the whole. Find the cost of the horse.
goods to make a profit of 17%? (a) `1000 (b) ` 1200 (c) `1500 (d) ` 1700
(a) 30% (b) 20% (c) 27% (d) 18% 23. Two electronic musical instruments were purchased for
12. A man sold an article at a loss of 20%. If he sells the article ` 8000. The first was sold at a profit of 40% and the second
for ` 12 more, he would have gained 10%. The cost price of at loss of 40%. If the sale price was the same in both the
the article is cases, what was the cost price of two electronic musical
instruments?
(a) ` 60 (b) ` 40 (c) ` 30 (d) ` 22
(a) ` 2000, ` 5000 (b) ` 2200, ` 5500
13. A milk man makes a profit of 20% on the sale of milk. If he (c) ` 2400, ` 5000 (d) ` 2400, ` 5600
were to add 10% water to the milk, by what %would his 24. A man sells an article at a gain 15%. If he had bought it at
profit increase? 10% less and sold it for ` 4 less, he would have gained
40 25%. Find the cost price of the article.
(a) 30 (b) (c) 22 (d) 10 (a) ` 150 (b) ` 160 (c) ` 170 (d) ` 180
3
25. A businessman, while selling 20 articles. loses the cost price
14. A grocer purchased 80 kg of sugar at ` 13.50 per kg and of 5 articles. Had he purchased the 20 articles for 25% less
mixed it with 120 kg sugar at ` 16 per kg. At what rate should
1
he sell the mixture to gain 16%? and sold them for 33 % more than the original selling price,
(a) ` 17 per kg (b) ` 17.40 per kg 3
what is his gain?
(c) ` 16.5 per kg (d) ` 16 per kg
1
15. A dishonest fruit seller professes to sell his goods at the (a) 5% (b) 75% (c) 33 % (d) 45%
cost price but weighs 800 grams for a kg weight. Find his 3
26. Five kg of butter was bought by a shopkeeper for ` 300.
gain percent.
One kg becomes unsaleable. He sells the remaining in such
(a) 100% (b) 150% (c) 50% (d) 200%
a way that on the whole he incurs a loss of 10%. At what
16. A shopkeeper purchased 150 identical pieces of calculators price per kg was the butter sold?
at the rate of ` 250 each. He spent an amount of ` 2500 on (a) ` 67.50 (b) ` 52.50 (c) ` 60 (d) ` 72.50
transport and packing. He fixed the labelled price of each 27. A manufacturer sells a pair of glasses to a wholesale dealer
calculator at ` 320. However, he decided to give a discount at a profit of 18%. The wholesaler sells the same to a retailer
of 5% on the labelled price. What is the percentage profit at a profit of 20%. The retailer in turn sells them to a customer
earned by him ? for ` 30.09, thereby earning a profit of 25%. The cost price
(a) 14 % (b) 15 % (c) 16 % (d) 20 % for the manufacturer is
17. A dishonest dealer sells his goods at the cost price but still (a) ` 15 (b) ` 16 (c) ` 17 (d) ` 18
earns a profit of 25% by underweighing. What weight does 28. By selling 66 metres of cloth a person gains the cost price
he use for a kg? of 22 metres. Find the gain per cent.
(a) 750 g (b) 800 g (c) 825 g (d) 850 g 1 1
18. A shopkeeper marks up his goods to gain 35%. But he (a) 22% (b) 22 % (c) 33% (d) 33 %
2 3
allows 10% discount for cash payment. His profit on the 29. A dairy man pays ` 6.40 per litres of milk. He adds water and
cash transaction therefore, in percentage, is sells the mixture at ` 8 per litres, there by making 37.5%
1 1 profit. The proportion of water to milk received by the
(a) 1 (b) 25 (c) (d) 31
13 21 customers is:
2 2 2 (a) 1: 10 (b) 1:12 (c) 1:15 (d) 1:20
y
o
u
B-72 Profit & Loss

rs
A man sells his car for ` 5000 and loses something. Had he

m
30. A single discount equal to a discount series of 10% and 42.

a
20% is sold it for ` 5600, his gain would have been double the

h
former loss. Find the cost price.

b
(a) 25% (b) 28% (c) 30% (d) 35%

o
31. The list price of a watch is ` 160. A retailer bought the same (a) ` 5500 (b) ` 5100 (c) ` 5400 (d) ` 5200

o
b
watch ` 122.40. He got two successive discounts one at 43. A manufacturer sells goods to an agent at a profit of 20%.

.w
10% and the other at a rate which was not legible. What is The agent’s wholesale price to a shopkeeper is at a profit of

o
10% and the shopkeeper retails his goods at a profit of

rd
the second discount rate?
12%. Find the retailer’s price of an article which had cost

p
(a) 12% (b) 14% (c) 15% (d) 18%

re
32. Instead of a metre scale cloth merchant uses a 120 cm scale the manufacturer ` 25.

s
(a) ` 37 (b) ` 40 (c) ` 44 (d) ` 46

s
while buying but uses an 80 cm scale while selling the same

.c
cloth. If he offers a discount of 20 per cent of cash payment, 1

o
44. A business man sells lot of shirts at a profit of 12 % and

m
what is his overall per cent profit? 2
(a) 20% (b) 25% (c) 40% (d) 15% invests the proceeds to buy lot of pants, which he sells at a
33. A trader marks his goods at such a price that he can deduct profit of 20%. If he makes a net profit of ` 2700, the cost of
15% for cash and yet make 20% profit. Find the marked the shirts was
price of an item which costs him ` 90 : (a) ` 2450 (b) ` 2225 (c) ` 2000 (d) ` 1880
11 3 45. A sells an article which costs him ` 400 to B at a profit of
(a) ` 135 (b) ` 105 20%. B then sells it to C, making a profit of 10% on the price
13 21
he paid to A. How much does C pay to B?
1 1 (a) ` 472 (b) ` 476 (c) ` 528 (d) ` 532
(c) ` 127 (d) ` 95
17 21 46. A shopkeeper buys 50 dozen eggs at ` 4 per dozen. Out of
34. A trader wants 10% profit on the selling price of a product them, 40 eggs were found broken. At what rate should he
whereas his expenses amount to 15% on sales. What should sell the remaining eggs per dozen so as to gain 5% on the
be his rate of mark up on an article costing ` 9? whole?
(a) ` 4 (b) ` 4.25 (c) ` 4.50 (d) ` 5.25
2 100
(a) 20% (b) 66 % (c) 30% (d) % 1
3 3 47. A person sells his table at a profit of 12 % and the other
2
35. A wholesaler sells 30 pens at the price of 27 pens to a retailer. 1
The retailer sells the pens at their market price. The profit hand if he sells the table at a loss of 8 % but on the whole
3
for the retailer is he gains ` 25. On the other hand if he sells the table at a loss
1 1 1 1
(a) 11% (b) 10% (c) 11 % (d) 9 % of 8 % and the chair at a profit of 12 % then he neither
9 11 3 2
36. A discount of 16% on the marked price of a book enables a gains nor loses. Find the cost price of the table.
man to buy a pen which costs ` 80. How much did he pay (a) ` 120 (b) ` 360 (c) ` 240 (d) ` 230
for the book? 48. Kabir buys an article with 25% discount on its marked price.
(a) ` 420 (b) ` 450 (c) ` 480 (d) ` 495 He makes a profit of 10% by selling it at ` 660. The marked
37. A shopkeeper fixes the marked price of an item 20% above price is
the cost price. He allows his customers a discount and makes (a) ` 600 (b) ` 685 (c) ` 700 (d) ` 800
a profit of 8%. Find the rate of discount. 49. On the eve of Gandhi Jayanti, Gandhi Ashram declared a
(a) 8% (b) 9% (c) 10% (d) 11% 25% discount on silk. If selling price of a silk saree is ` 525,
38. A chair originally costs ` 50. It was offered for sale at 108% what is its marked price ?
of its cost. After a week, the price was 10% discounted and (a) ` 700 (b) ` 725 (c) ` 750 (d) ` 775
was sold. Find the sale price. 50. A shopkeeper marks an article at a price which gives a profit
(a) ` 46.80 (b) ` 48.60 (c) ` 50 (d) ` 52.40 of 25%. After allowing certain discount, the profit reduces
39. By selling an umbrella for ` 30, a merchant gains 20%. During 1
a clearance sale, the merchant allows a discount of 10% off to 12 % . The discount percent is
2
the marked price (the price at which he used to sell). Find (a) 12% (b) 12.5% (c) 10% (d) 20%
his gain per cent.
(a) 6% (b) 7% (c) 8% (d) 9% 1
51. A CD was sold at a profit of 12 % . If it had been sold at a
40. By what % must the cost of goods be marked up so that 2
even after a discount of 20% the same amount is realised as profit of 15%, it would have gained him ` 10 more. The cost
before the discount? prices of CD is (in `)
(a) 20 (b) 25 (c) 40 (d) 10 (a) 450 (b) 500 (c) 400 (d) 550
41. Goods are sold so that when a discount of 4 percent is 52. A trader has a weighing balance that shows 1,200 gm for a
given on the sale price, a profit of 20 percent is made. How kilogram. He further marks up his cost price by 10%. Then
much percent, is the sale price higher than the cost price? the net profit percentage is
(a) 20% (b) 24% (c) 25% (d) 27% (a) 32% (b) 23% (c) 31.75% (d) 23.5%
y
o
u
Profit & Loss B-73

rs
m
53. A shopkeeper allows 10% discount on goods when he sells 57. A trader who marks his goods up to 50% offered a discount

a
without credit. Cost price of his goods is 80% of his selling of 20%. What % profit the trader makes after offering the

h
payment?

b
price. If he sells his goods by cash, then his profit is

o
(a) 30% (b) 70%

o
(a) 50% (b) 70%

b
(c) 20% (d) 50%

.w
(c) 25% (d) 40%
58. A retailer buys a sewing machine at a discount of 15% and

o
54. A dealer of scientific instruments allows 20% discount on sells it for ` 1955. Thus he makes a profit of 15%. The

rd
the marked price of the instruments and still makes a profit discount is

p
re
of 25%. If his gain over the sale of an instrument is ` 150, (a) ` 270 (b) ` 290

s
find the marked price of the instrument. (c) ` 300 (d) ` 310

s
.c
(a) ` 938.50 (b) ` 940 59. A tea-merchant professes to sell tea at cost price but uses a

o
m
(c) ` 938 (d) ` 937.50 false weight of 900 gram for a kilogram. The profit percent
55. Ram bought a T.V. with 20% discount on the labelled price. in his transaction is
Had he bought it with 30% discount he would have saved 1
(a) 11 % (b) 10%
` 800. The value of the T.V. set that he bought is 9
(a) ` 5,000 (b) ` 8,000 1
(c) ` 9,000 (d) ` 10,000 (c) 9 % (d) 15%
11
56. A sold an article to B at 20% profit and B sold it to C at 15% 60. Mahesh earned a profit of 20% by selling 60 apples at the
loss. If A sold it to C at the selling price of B, then A would rate of ` 42.50 for 5 apples. Then the total cost, at which the
make apples were bought is
(a) 5% profit (b) 2% profit (a) ` 452 (b) ` 425
(c) 2% profit (d) 5% loss (c) ` 450 (d) ` 485

ANSWER KEY
1 (a) 9 (a) 17 (b) 25 (c) 33 (c) 41 (c) 49 (a) 57 (c)
2 (b) 10 (b) 18 (c) 26 (a) 34 (d) 42 (d) 50 (c) 58 (c)
3 (d) 11 (a) 19 (a) 27 (c) 35 (c) 43 (a) 51 (c) 59 (a)
4 (d) 12 (b) 20 (a) 28 (d) 36 (a) 44 (c) 52 (a) 60 (b)
5 (d) 13 (b) 21 (c) 29 (a) 37 (c) 45 (c) 53 (c)
6 (b) 14 (b) 22 (b) 30 (b) 38 (b) 46 (c) 54 (a)
7 (c) 15 (a) 23 (d) 31 (c) 39 (c) 47 (b) 55 (b)
8 (b) 16 (a) 24 (b) 32 (a) 40 (b) 48 (d) 56 (b)

1. (a) Let the cost of production of the table be ` x. 3. (b) Let C. P. = ` x then profit = S.P. – C. P.
Then, 125% of 115% of 110% of x = 1265
1 11x
x 891 – x 891
125 115 110 10 10
x 1265
100 100 100
891 10
x ` 810
253 1265 160 11
x 1265 x ` 800
160 253 4. (d) Let the SP of the article be ` x
2. (b) Let the original price of the jewel be ` P and let the Expenses = 15% of x = ` 0.15x
profit earned by the third seller be x%. Profit = 10% of x = ` 0.10x
Then, (100 + x)% of 125% of 120% of P = 165% of P CP = ` 9 (given)
Therefore, 9 + 0.15x + 0.1x = x x = 12
(100 x) 125 120 165
P P 12 9
100 100 100 100 % increase for marked price = 100
9
165 100 100 100
(100 x) 110 x 10%. = %
125 120 3
y
o
u
B-74 Profit & Loss

rs
m
10 117 100

a
5. (d) C.P. for 1 lichchus = paise Then, 90% of x = 117 x 130

h
11 90

b
o
11 Marked price = 30% above C.P.

o
paise

b
S. P. for 1 lichchus =
10

.w
80 4
12. (b) S.P = C.P S.P C.P ...(1)

o
11 10 100 5

rd
p
gain % 10 11 100 21% 110 11

re
10 S.P 12 C.P S.P C.P 12 ...(2)

s
100 10

s
11

.c
From eqn. (1) and (2)

o
Short cut:

m
Quanlity Price 4 11
C.P C.P 12
11 10 5 10
10 11 11 4
C.P C.P 12 C.P ` 40
11 11 10 5
gain % 1 100%
10 10 13. (b) Let profit per litre = ` 20
So, C.P. / litre = ` 100
21 S.P. / litre = ` 120
100 %
100 On adding 10% water to the milk
= 21% 9
C.P. per litre ` 100
5 1 10
6. (b) S.P. for 1 egg = ` Rs
10 2 9
S.P. per litre ` 120
100 1 5 10
C. P. for 1 egg `
(100 20) 2 12 120 10 400
S.P. per litre ` `
He bought 12 eggs for 5 rupees. 9 3
7. (c) Let he sells x oranges per rupee.
400 100
1 Profit / litre 100
: (100 4) :: x : (100 8) 3 3
36
100 40
108 1 % by which profit increases = – 20 =
x 3 3
96 36 32
14. (b) C.P. of 200 kg of mixture = ` (80 × 13.50 + 120 × 16)
He sells 32 oranges per rupee.
= ` 3000.
150 3
8. (b) C.P. for one coconut Rs Rs 116
100 2 S.P. = 116% of Rs 3000 = ` 3000 ` 3480.
S.P. for one coconut = ` 2 100
3 1 3480
Profit on one coconut 2 ` Rate of S.P. of the mixture = Rs. per kg
2 2 200
1 = ` 17.40 per kg.
Profit on 2000 coconut 2000 ` 1000
2 15. (a) He gives 800 grams but charges the price of 1000 grams
9. (a) Let C.P. = Rs 100, then M. P. = ` 150 (1 kg)
S.P. = 70% of 150 = ` 105 on every 800 grams, he gains (1000 – 800) grams i.e.
105 100 200 grams.
% profit 100 5%
100 200
His gain % 100% 25%
10. (b) Let S. P. = ` x per kg 800
S.P. of 2 kg of rice = ` 2x = Loss Short cut :
Now, Loss = C.P. – S.P.
2x = 600 – 10 x error
Gain %
x = ` 50 per kg true weight–error
11. (a) Let CP = `100
Then, S.P = `117 200
100 = 25%
Let marked price be Rs x. 1000 200
y
o
u
Profit & Loss B-75

rs
m
16. (a) C.P. of 150 calculators 20. (a) Women's shirt comprise 60% of the output.

a
= 150 × 250 + 2500 = 37500 + 2500 = ` 40000 Men's shirts comprise (100 – 60) = 40% of the out put.

h
b
Labelled price of 150 calculators Average profit from men's shirt = 8% of 40

o
o
= 150 × 320 = ` 48000 = 3.2 out of 40

b
.w
Discount allowed = 5% Overall average profit = 6 out of 100
S.P. of 150 calculators

o
Average profit from women's shirts = 2.8 out of 60

rd
= 48000 – 5% of 48000 = ` 45600 i.e. 0.0466 out of each shirt.

p
re
5600 21. (c) Here, in whole transaction, there is neither gains nor
100 14

s
Profit % = loss, therefore,

s
40000

.c
Amount of gain in one watch

o
True weight 100 gain%

m
= Amount of loss in other watch
17. (b) False weight 100 x
0.15 CP1 0.10 CP2
Here S.P. = C. P. x=0
CP1 0.10 2
1000×100
False weight= 800 gm CP2 0.15 3
125
18. (c) Let cost Price = ` 100 Also CP1 + CP2 = 560
Marked price = ` 135 2
CP1 560 ` 224
After discount, selling price = 135 – 13.5 = 121.5 (2 3)
Profit% = 121.5 – 100 = 21.5%
19. (a) S.P. of the 1st chair = ` 500 and CP2 = 560 – 224 = ` 336
Gain = 20% 22. (b) Let the C.P. of horse = ` x
500 100 500 100 Then the C.P. of carriage = Rs (3000 – x)
C.P. of the 1st chair = 20% of x – 10% of (3000 – x) = 2% of 3000
100 20 120
1250 x (3000 – x)
3 60
5 10
S.P. of the 2nd chair = ` 500
Loss = 12% 2x 3000 x 600

500 100 500 100 3x 3600 x 1200


C.P. of the 2nd chair = 23. (d) Here, SP1 = SP2
100 12 88
500 25 250 25 CP1 6 3
= 140 CP1 = 60CP2
22 11 CP2 14 7
6250 3
CP1 8000 ` 2400
11 (3 7)
Now S.P. of both the chairs = ` 1000 and CP2 = 8000 – 2400 = ` 5600
C.P. of both the chairs 24. (b) Let the C.P. be Rs 100
1250 6250 13750 18750 32500 First S.P. = ` 115
3 11 33 33
Second C.P. = ` 90
32500 500 Second S.P = 125% of ` 90 = ` 112.50
Net gain = 1000 –
33 33
Difference of two selling prices is ` 115 – ` 112.50
500 33 500
Gain % = 100 = 100 = ` 2.50 and C.P. of the article is ` 100
32500 33 32500
But actual difference is ` 4.
100 20
= 1.5% (To one place of decimal) 100
65 13 C.P. ` 4 ` 160.
2.50
Shortcut method :
25. (c) Let the price of 1 article = ` 1
100 x y 2 xy 100 20 12 2 20 ( 12) Loss = 20 C.P – 20 S.P.
100 x 100 y 100 20 100 12 5C.P. = 20 C.P. – 20 S.P. 20 S.P = 15 C.P
100 8 480 320 CP1 of 20 articles = ` 20
= 1.5% gain SP1 of 20 articles = ` 15
208 208
y
o
u
B-76 Profit & Loss

rs
m
Also given that , had he purchased the 20 articles for 100 64
8

a
29. (a) Mean cost price = ` = ` using

h
1 137.5 11

b
25% less and sold them for 33 % more, then alligation rule.

o
3

o
b
CP2 of 20 articles = ` 15 c.p of 1 litre water c.p of 1 litre milk

.w
CP2 of 20 articles = ` 20 `0 6.40

o
rd
20 15 1

p
re
Gain percentage = 100 = 33 %
15 3

s
s
.c
26. (a) Let S.P. = ` x per kg 64

o
11

m
S.P. of 4 kg = ` 4x

100 –10
4x 300
100

270 64 64
x ` 67.50
4 110 11
27. (c) Let the cost price of manufactures is = P
64 64
18 59P Required ratio = =1 : 10
Selling price of manufacturer = P + P × 110 11
100 50
10 20
30. (b) Equivalent discount = 10 + 20 –
59 P 59 P 20 100
Wholesaler selling price = = 30 – 2 = 28%
50 50 100
d1 d2
59 P 59 P 354 P 31. (c) Retailer price = list price 1 1
= 100 100
50 250 250
10 d2
354 P 354 P 25 122.40 160 1 1
Retailer selling price = 100 100
250 250 100
d2 122.40 100
354 P 177 P 805 P 1 0.85
= 100 160 90
250 500 500
d 2 (1 0.85) 100 15%
805 P 32. (a) Let the cost of cloth per cm be ` x
Now, 30.09
500 As he uses 120 cm scale, so, he has 120 cm cloth cost
P = 17 incurred = 100x. While selling he uses 80 cm scale, so
100
Short actually he charges for 120 150 cm of cloth
80
100 100 100 Amount obtained after 20% discount
P 30.09 17
118 120 125 = 0.8 x 150 120 x
28. (d) Let C.P. of one metre of cloth = ` 1 20 x
Profit = 100 20%
100 x
then C.P. of 66 metres of cloth = ` 66
33. (c) SP = 90 × 1.2 = Rs 108
Gain = C.P. of 22 metres = ` 22
108
Marked price = = 127.05
22 1 0.85 `
% gain 100 33 %
66 3 34. (d) Let the SP of the article be ` x
Shortcut method : Expenses = 15% of x = ` 0.15x
If on selling ‘x’ articles, a man gains equal to the C.P. of Profit = 10% of x = Rs 0.10x
CP = ` 9 (given)
y
‘y’ articles, then % gain 100 Therefore, 9 + 0.15x + 0.1x = x x = 12
x
12 9
% increase for marked price = 100
22 1 9
% gain 100 33 %
66 3 100
= %
3
y
o
u
Profit & Loss B-77

rs
m
35. (c) Retailer’s S.P. = M.P. 44. (c) Let C.P. = ` x

a
Retailer’s C.P. for 30 Pens = M.P. of 27 pens

h
225
Retailer’s S.P. for 30 pens = M.P of 30 pens

b
120% of % of x 2700

o
2

o
30 27 100 1
100 11 %

b
% gain 120 225

.w
27 9 9 x 2700 x = 2000
36. (a) Let M.P. = ` 100 100 2 100

o
rd
then discount = ` 16 120

p
45. (c) C.P. for B = 120% of ` 400 = ` 400 = ` 480

re
when discount = ` 80, then M.P = ` x 100

s
s
100 x

.c
110
Now, it’s direct proportion C.P. for C = 110% of ` 480 = ` 480 = ` 528.

o
16 80 100

m
100 : x : : 16 : 80 46. (c) C.P. = 50 × 4 = ` 200
16x = 100 × 80 x = ` 500 Remaining eggs = 600 – 40 = 560
Now, since M. P. = ` 500, therefore, after 16% discount Let S.P. of eggs = ` x per dozen
16 560
man paid = 500 1 ` 420 Total S.P. = ` x
100 12
37. (c) Let C.P. = ` 100. Then M.P. = ` 120 and S.P. = ` 108 560 (100 5)%
x 200
12 12 100
% discount 100 % 10%
120 105 2400
x ` 4.5per dozen
50 108 100 560
38. (b) Offering price ` 54 47. (b) Suppose the cost price of table = ` T and cost price of
100
After 10% discount, S.P. = 90% of 54 a chair = ` C.
90 54 1 1
` 48.60 Then; 12 % of T + 8 % of C = 25
100 2 3
39. (c) (100 + g1) : S1 : : (100 + g2) : S2
1 1
10 and 8 % or T + 12 % of C = 0
(100 + 20) : 30 : : (100 + g2) : 30 1 3 2
100
[ 10% discount is allowed on S.P.] 25 25
or, T– C = 2500 ……(1)
120 : 30 : : (100 + g2) : 27 2 3
25 25
120 27 – T+ C=0 ……(2)
100 g 2 108 3 2
30
25 25
g2 = 8% (1) 2 + (2) 3 gives T– T = 1250
40. (b) Let C.P. = Rs 100, Also, let M.P. = ` x 4 9
Given, C.P. after 20% discount on M.P. = C.P. 225 100
80 % of x = 100 or, T = 1250
36
100 100 T = 360 Price of a table = ` 360
x ` 125
80 48. (d) Let the marked price be `x.
M.P. = 25 % above C.P.
41. (c) Let the C.P. be Rs 100 3
C.P. = (x – 25% of x) = x
S.P. = Rs 120 4
Discount being 4%, S.P. is 96% of sale price.
3x 3x 33
96% of sale price = ` 120 S.P. 10% of x
4 4 40
100
sale price = ` 120 × ` 125 33
96 But, x 660 x = 800.
Sale price is 25% higher than the C.P. 40
42. (d) Let his loss = ` x. Then, 49. (a) Let the marked price be ` x.
C.P. = 5000 + x = 5600 – 2x 3
3x = 600 x = 200 S.P. = (x – 25% of x) = x
4
C.P. =5000 + 200 = Rs 5200
43. (a) Retailer’s price = 112 % of 110% of (120 % of 25) But, S.P = ` 525
112 110 120 3
25 ` 36.96 ` 37 x 525 x 700
100 100 100 4
y
o
u
B-78 Profit & Loss

rs
m
50. (c) Short cut method : 25

a
Cost price, C.P. = C.P. + C.P. = 0.8x

h
Profit Discount 100

b
Net profit = Profit +Discount +

o
100 0.8 100 16

o
C.P. = x

b
25 25×Discount 125 25

.w
25 – Discount–
2 100 25

o
x= C.P..

rd
(‘–’ to represent discount) 16

p
re
25 –5 25
– 25 Given that C.P = 150

s
Discount 100

s
2 4

.c
Discount % = 10% 150 100

o
C.P. = = 600

m
25
51. (c) Ist case :
25
Marked price x = 6,000 = ` 938.50
25 16
100 C.P
100 Profit% 2 55. (b) Let labelled price of T.V. be ` x
S.P C.P. S.P.
100 100 20
112.5 Price after 20% discount, x – x = 0.8x
S.P CP ...(1) 100
100
30
IInd case : Price after 30% discount, x – x = 0.7x
100
100 Profit % 100 15 According to question
S.P C.P. (S.P 10) C.P. 0.8x – 0.7x = 800
100 100
x = 800 × 10 = 8000
115 56. (b) Let ` 100 be the cost price for A.
(S.P+10)= C.P ...(2) S.P. for A = 100 + 20% of 100 = 120
100
S.P. for B = 120 – 15% of 120 = 102
Dividing equation (1) by (2)
102 100
S.P 112.5 100 Profit % = 100 = 2%
(C.P) 100
S.P+10 100 115(C.P) 57. (c) Let cost price of good be 100
Trades mark up at 50% more i.e. 150
112.5
S.P (S.P + 10) 20
115 Selling price of goods 150 – 150 120
100
115 S.P = 112.5 SP + 1125
120 –100
S.P = 450 Profit % 100 20
100
S.P 100 450 100 58. (c) Let original price of sewing machine be ` x
C.P 400
112.5 112.5 15
52. (a) The trader professes to sell 1200 kg but sells only Retailer sought it at x – x 0.85x
100
1000 kg. 15
So profit = 20% 0.85x 0.85x 1955
Markup = 10% 100
1.15 × 0.85x = 1955
10 20 1955 10000
Total profit 10 20 32% x 2000
100 115 85
53. (c) Let marked price of goods be ` 100.
15
10 Discount is 200 ` 300
Selling price of goods = 100 100 = ` 90 100
100 1000 – 900 1
Cost price of goods is 80% of its selling price 59. (a) Profit % 100 11 %
900 9
80 60. (b) Selling price of 5 apples = ` 42.50
C.P. = 90 = 72
100 42.5
Selling price of 60 apples 60 510
Profit on goods = (90 – 72) = ` 18 5
C.P + Profit = S.P
18
Profit % = 100 = 25% 20
72 C.P + C.P 510
54. (a) Let marked price of the instrument be ` x 100
510
20 C.P 100 ` 425
Selling price, S.P. = x – x = 0.8x 120
100
y
o
u
Simple and Compound Interest B-79

rs
m
a
CHAPTER

h
SIMPLE AND

b
7

o
o
b
COMPOUND INTEREST

.w
o
rd
p
re
s
s
.c
o
INTEREST Example 3 :

m
Interest is the fixed amount paid on borrowed money. What rate per cent per annum will produce `250 as simple
The sum lent is called the Principal. interest on `6000 in 2.5 years
The sum of the principal and interest is called the Amount. Solution :
Interest is of two kinds : P = `6000; Time (T) = 2.5 years; S.I. = `250
(i) Simple interest (ii) Compound interest
(i) Simple Interest : When interest is calculated on the original S.I. 100 250 100 10 5 2
Rate 1 %
principal for any length of time, it is called simple interest. P T 6000 2.5 6 3 3
Principal Time Rate Example 4 :
Simple interest To buy furniture for a new apartment, Sylvia Chang bor-
100
rowed `5000 at 11% simple interest for 11 months. How much
P R T interest will she pay?
i.e. S.I.
100 Solution :
Amount = Principal + Interest From the formula, I = Prt, with P = 5000, r = .11, and t = 11/12
PRT RT (in years). The total interest she will pay is
i.e. A= P+ I P P 1
100 100 I = 5000 (.11) (11/12) = 504.17
or ` 504.17
100 S.I. (ii) Compound Interest : Money is said to be lent at compound
Principal (P)
R T interest when at the end of a year or other fixed period, the
100 S.I. interest that has become due is not paid to the lender, but is
Rate (R) added to the sum lent, and the amount thus obtained becomes
T P
the principal in the next year or period. The process is repeated
100 S.I. until the amount for the last period has been found. Hence,
Time (T)
P R When the interest charged after a certain specified time
If rate of simple interest differs from year to year, then period is added to form new principal for the next time period,
(R1 R 2 R 3 .....) the interest is said to be compounded and the total interest
S.I. P accrued is compound interest.
100
Example 1 : n
r
Find the amount to be paid back on a loan of `18,000 at C.I. P 1 –1 ;
100
5.5% per annum for 3 years
Solution : n
r
P = `18000, R = 5.5%, T = 3 years Amount (A) P 1
100
P R T 18000 5.5 3 where n is number of time period.
S.I. = `2970
100 100 If rate of compound interest differs from year to year, then
Amount = P + I = 18000 + 2970 = `20970
r1 r2 r3
Example 2 : Amount P 1 1 1 .......
In how many years will a sum of money triple itself, at 25% 100 100 100
per annum simple interest. Example 5 :
Solution : If `60000 amounts to `68694 in 2 years then find the rate of
Let the sum of money be ` P. So, A = 3P interest.
and S.I. = A – P = 3P – P = 2P Solution :
R = 25% Given : A = `68694
100 S.I. 100 2P P = `60000
T = 8 years n = 2 years
P R P 25
r=?
y
o
u
B-80 Simple and Compound Interest

rs
m
n
r

a
A P 1 1

h
2 15
100 15

b
Amount ` 8000 1 1 3

o
100 100

o
2

b
r

.w
68694 60000 1
100

o
rd
23 23 21
2 2 ` 8000 Rs.11109

p
68694 r 11449 r 20 20 20

re
1 1
60000 100 10000 100

s
C.I. = `(11109 – 8000) = `3109.

s
.c
Compound interest – when interest is calculated half-yearly

o
r 11449

m
1 1.1449 Since r is calculated half-yearly therefore the rate per cent
100 10000
will become half and the time period will become twice, i.e.,
r
1 1.07 r
100 Rate per cent when interest is paid half-yearly %
2
r
1.07 – 1 0.07 and time = 2 × time given in years
100
Hence,
r = 0.07 × 100 = 7%
2n
r
Example 6 : A P 1
In how many years, the sum of `10000 will become `10920.25 2 100
if the rate of compound interest is 4.5% per annum?
Example 8 :
Solution :
What will be the compound interest on `4000 in 4 years at
A = `10920.25
P = `10000 8 per cent annum. If the interest is calculated half-yearly.
Rate of interest = 4.5% Solution :
Time (n) = ? Given : P = `4000, r = 8%, n = 4 years
Since interest is calculated half-yearly, therefore,
n
r
A P 1 8
100 r % 4% and n = 4 × 2 = 8 half years
2
n
4.5 8 8
10920.25 10000 1 4 26
100 A 4000 1 4000
100 25
n n
10920.25 0.9 20.9 = 4000 × 1.3685 = 5474.2762
1
10000 20 20
Amount = `5474.28
436.81 20.9
n
20.9
2
20.9
n Interest = Amount – Principal
400 20 20 20 = `5474.28 – `4000 = `1474.28
Compound interest – when interest is calculated quarterly
Hence `10000 will become `10920.25 in 2 years at 4.5%. Since 1 year has 4 quarters, therefore rate of interest will
Compound interest – when interest is compounded annually
1
but time is in fraction become th of the rate of interest per annum, and the time
4
p period will be 4 times the time given in years
If time = t years, then
q Hence, for quarterly interest
4 n 4n
p r/4 r
r
t
qr
A P 1 P 1
A P 1 1 100 400
100 100
Example 9 :
Example 7 : Find the compound interest on `25625 for 12 months at
Find the compound interest on `8000 at 15% per annum for 16% per annum, compounded quarterly.
2 years 4 months, compound annually. Solution :
Solution : Principal (P) = `25625
4 1 16
Time = 2 years 4 months = 2 years = 2 years Rate (r) = 16% % 4%
12 3 4
y
o
u
Simple and Compound Interest B-81

rs
m
Time = 12 months = 4 quarters n
R

a
P 1

h
4 4 and if the annual decrease is R%, then the
4 26

b
A 25625 1 25625 100

o
100 25

o
population in a years is given by a change of sign in the

b
.w
26 26 26 26 n
25625 = `29977.62 R

o
25 25 25 25 formula i.e P 1

rd
100
C.I. = A – P = 29977.62 – 25625 = `4352.62

p
re
Difference between Compound Interest and Simple Interest Example 11 :

s
s
When T = 2 If the annual increase in the population of a town is 4% and

.c
the present population is 15625 what will be the population

o
2
R

m
(i) C.I. – S.I. = P in 3 years.
100 Solution :
R S.I. 3
(ii) C.I. – S.I. = 4
2 100 15625 1
100
When T = 3
Required population: 15625 (1.04)3 = 17576
PR 2 300 R
(i) C.I. – S.I. = NOTE :
104 100
2
• A certain sum is lent out on a certain rate of interest for a
S.I. R R certain period. Again the same sum is out on x% higher rate
(ii) C.I. – S.I. = 3 3
100 100 of interest for y% higher period. Then the % increase in S.I
When C.I. is compounded annually, the ratio of S.I. to C.I. xy
at the same rate per annum and for the same period is given is given by x y %
100
S .I rt • P is lent out at the rate of R1% and P2 is lent out at the rate
by
C. I r
t
of R2%. Then over all rate of interest will be
100 1 1
100 P1 R1 P2 P2
R
Example 10 : P1 P2
The difference between compound interest and simple
interest on a certain amount of money at 5% per annum 1
for 2 years is `15. Find the sum : • x1 part of the principal is lent out on R1% rate of interest,
(a) `4500 (b) `7500
(c) `5000 (d) `6000 1
Solution : x2 part of the principal is lend out on R2% rate of interest,....,
(d) Let the sum be `100.
100 5 2 1
Therefore, SI Rs 10
100 xn part on Rn% rate of interest. The over all rate of interest
2 on whole sum is equal to
5
and CI 100 1 100
100 1 1 1
R1 R2 .... Rn
x1 x2 xn
21 21 41
100 100 Rs
20 20 4 EFFECTIVE RATE
41 1 If ` 1 is deposited at 4% compounded quarterly, a calculator
Difference of CI and SI 10
4 4 can be used to find that at the end of one year, the compound
amount is `1.0406, an increase of 4.06% over the original `1.
1
If the difference is , the sum = 100 The actual increase of 4.06% in the money is somewhat
4
higher than the stated increase of 4%. To differentiate
If the difference is `15, the sum between these two numbers, 4% is called the nominal or
= 400 × 15 = `6000
stated rate of interest, while 4.06% is called the effective
POPULATION FORMULA
rate. To avoid confusion between stated rates and effective
The original population of a town is P and the annual
rates, we shall continue to use r for the stated rate and we
increase is R%, then the population in years is
will use re for the effective rate.
y
o
u
B-82 Simple and Compound Interest

rs
m
Example 12: Example 14 :

a
Find the effective rate corresponding to a stated rate of 6% Subash purchased a refrigerator on the terms that he is

h
required to pay `1,500 cash down payment followed by

b
compounded semiannually.

o
Solution : `1,020 at the end of first year, `1,003 at the end of second

o
b
A calculator shows that `100 at 6% compounded year and ` 990 at the end of third year. Interest is charged

.w
semiannually will grow to at the rate of 10% per annum. Calculate the cash price :

o
Solution :

rd
2
.06 Cash down payment = `1500

p
A 100 1 100 (1.03)2 $ 106.09

re
2 Let `x becomes `1020 at the end of first year.

s
s
.c
Thus, the actual amount of compound interest is 10
Then, 1020 x 1

o
`106.09 – `100 = ` 6.09. Now if you earn ` 6.09 interest on 100

m
` 100 in 1 year with annual compounding, your rate is
6.09/100 = .0609 = 6.09%. 1020 100
or x ` 927.27
Thus, the effective rate is re = 6.09%. 110
NOTE : 10
2
Similarly, 1003 y1
In the preceding example we found the effective rate by 100
dividing compound interest for 1 year by the original principal.
The same thing can be done with any principal P and rate r 1003 20 20
or y ` 828.92
compounded m times per year. 22 22

compound interest 990 20 20 20


Effective rate and z ` 743.80
principal 22 22 22
Hence, CP = 1500 + 927.27 + 828.92 + 743.80
compound amount principal = 3999.99 or `4000.
re Example 15 :
principal
The difference between the interest received from two
m different banks on `500 for 2 yrs is `2.5. Find the difference
r
m r
P 1 P P 1 1 between their rates.
m m
= Solution :
P P
500 2 r1
I1 = = 10 r 1
r
m 100
= re 1 1]
m 500 2 r2
I2 = = 10 r2
100
Example 13 : I1 – I2 = 10r1 – 10r2=2.5
A bank pays interest of 4.9% compounded monthly. Find
2.5
the effective rate. Or, r1 – r2= = 0.25%
Solution : 10
Use the formula given above with r = .049 and m = 12. Short-cut method :
When t1= t2,
12
.049 Id 100 2.5 100
The effective rate is re 1 1 (r1 –r2) = = 0.25%
12 sum t 500 2
= 1.050115575 – 1 .0501 or 5.01% Example 16 :
At what rate per cent compound interest does a sum of
Present worth of `P due n years hence
money becomes nine - fold in 2 years?
P Solution :
Present worth = n Let the sum be `x and the of compound interest be r% per
r annum; then
1
100
2 2
r r
Equal annual instalment to pay the borrowed amount 9x = x 1 or, 9 = 1
Let the value of each instalment = `x 100 100
Rate = r% and time = n years r r
Then, Borrowed Amount or, 3 = 1+ ; or, =2 r = 200%
100 100
x x x Short cut method :
.....
r 2 n The general formula of compound interest can be changed
1 r r
100 1 1 to the following form :
100 100
y
o
u
Simple and Compound Interest B-83

rs
m
If a certain sum becomes ‘m’ times in ‘t’ years, the rate of ratio of two amounts = 2 : 5

a
1/ t

h
compound interest r is equal to 100 (m) 1

b
7000

o
amount lent at 6 % = × 2 = `2000

o
1/ t 7
In this case , r = 100 (9) 1

b
.w
= 100 (3 – 1) = 200% Example 19 :

o
rd
Example 17 : As n amount of money grows upto `4840 in 2 yrs and upto

p
The simple interest on a certain sum of money at 4% per `5324 in 3 yrs on compound interest. Find the rate percent

re
annum for 4 yrs is `80 more than the interest on the same

s
Solution :

s
sum for 3 yrs at 5% per annum. Find the sum.

.c
Solution :

o
We have,

m
Let the sum be `x, then at 4% rate for 4 yrs the simple interest
P + CI of 3 yrs = `5324.......(i)
x 4 4 4x
= `
P + CI of 2yrs = `4840........(ii)
100 25
At 5%rate for 3 yrs the simple interest Subtracting (ii) from (i), we get
x 5 3 3x
= =` CI of 3rd year = 5324 – 4840 = `484.
100 20
Thus, the CI calculated in the third year which is `484 is
Now, we have , 4x 3x
80 basically the amount of interest on the amount generated
25 20
after 2 years which is `4840.
16x 15x
or = 80 x = `8000 484 100
100
r= = 10%
Alternate Method : 4840 1
For this type of question
Alternate method :
Difference 100 80 100
Sum = = ` 8000
[r2 t1 r2 t 2 ] 4 4 3 5 Difference of amount after n yrs and( n 1) yrs 100
Example 18 : Amount after n yrs
Some amount out of `7000 was lent at 6% per annum and
the remaining at 4% per annum. If the total simple interest In this, n = 2.
from both the fractions in 5 yrs was `, 1600, find the sum lent
at 6% per annum. Difference of amount after 2 yrs and 3yrs 100
rate =
Solution : Amount after 2 yrs
Suppose ` x was lent at 6 % per annum.
x 6 5 (7000 x) 4 5 (5324 4840) 484 100
Thus, + = 1600 = × 100 = = 10%
100 100 4840 4840
3x 7000 x Example 20 :
or, + = 1600
10 5 Find the compound interest on `18,750 in 2 yrs the rate of
or, 3x + 14,000 – 2x = 16000
interest being 4% for the first year and 8% for the second
x = 16000 – 14000 = ` 2000
By Method of Alligation : Overall rate of interest year.

1600 100 32 Solution :


= = %
5 7000 7 After first year the amount
6% 4%
4 104
= 18750 1 = 18750
100 100

32 104 108
%
7 After 2nd year the amount = 18750
100 100

26 27
= 18750 = 21060
25 25
4 10
% %
7 7 CI = 21060 –18,750 = ` 2310.
y
o
u
B-84 Simple and Compound Interest

rs
m
a
h
b
o
o
b
.w
What annual instalment will discharge a debt of ` 4,200 due

o
1 1 10.

rd
1. Out of a certain sum, rd is invested at 3%, th at 6% and in 5 years at 10% simple interest?
3 6

p
re
the rest at 8%. If the simple interest for 2 years from all these (a) ` 500 per year (b) ` 600 per year

s
investments amounts to ` 600, find the original sum. (c) ` 700 per year (d) ` 800 per year

s
.c
(a) ` 4000 (b) ` 5000 11. Adam borrowed some money at the rate of 6% p.a. for the

o
m
(c) ` 6000 (d) ` 7000 first two years, at the rate of 9% p.a. for the next three years,
2. A sum of money lent out at simple interest amounts to and at the rate of 14% p.a. for the period beyond five years.
` 1008 in 2 years and ` 1164 in 3½ years. Find the rate % p.a. If he pays a total interest of ` 11,400 at the end of nine
(a) 13% (b) 14%
years, how much money did he borrow?
1
(c) 12 % (d) 15% (a) ` 10,000 (b) ` 12,000
2 (c) ` 14,000 (d) ` 16,000
3. A sum of money lent out at simple interest amounts to ` 720
12. A person borrows ` 5000 for 2 years at 4% p.a. simple
after 2 years and to ` 1,020 after a further period of 5 years.
Find the sum and the rate %. 1
(a) ` 500, 10% (b) ` 600, 10% interest. He immediately lends it to another person at 6 %
4
(c) ` 500, 12% (d) ` 600, 12%
4. A sum was put at simple interest at a certain rate for 4 years p.a. for 2 years. Find his gain in the transaction per year.
Had it been put at 2% higher rate, it would have fetched ` (a) ` 112.50 (b) ` 125
56 more. Find the sum. (c) ` 150 (d) ` 167.50
(a) ` 500 (b) ` 600 13. A certain amount earns simple interest of ` 1750 after 7
(c) ` 700 (d) ` 800 years Had the interest been 2% more, how much more
5. Simple interest on a certain sum is 16 over 25 of the sum. interest would it have earned?
Find the rate per cent and time, if both are equal. (a) ` 35 (b) ` 245
(a) 8% and 8 years (b) 6% and 6 years
(c) ` 350 (d) Cannot be determined
(c) 10% and 10 years (d) 12 % and 12 years
6. A father left a will of ` 68,000 to be divided between his two 14. What will be the ratio of simple interest earned by certain
sons aged 10 years and 12 years such that they may get amount at the same rate of interest for 6 years and that for
equal amount when each attains the age of 18 years If the 9 years?
money is reckoned at 10% p.a., find how much each gets at (a) 1 : 3 (b) 1 : 4
the time of the will. (c) 2 : 3 (d) Data inadequate
(a) ` 30,000, ` 38,000 15. Two equal sums of money were invested, one at 4% and
(b) ` 28,000, ` 40,000 the other at 4.5%. At the end of 7 years, the simple
(c) ` 32,000, ` 36,000 interest received from the latter exceeded to that received
(d) cannot be determined.
from the former by ` 31.50. Each sum was :
7. If there are three sum of money P,Q and R so that P is the
simple interest of Q and Q is the simple interest of R, rate (a) ` 1,200 (b) ` 600
r % and time are same in each case, then the relation of P, Q (c) ` 750 (d) ` 900
and R is given by 16. Nitin borrowed some money at the rate of 6% p.a. for the
(a) P2 = QR (b) Q2 = PR first three years, 9% p.a. for the next five years and 13% p.a.
2
(c) R = PQ (d) PQR = 100 for the period beyond eight years If the total interest paid
8. In how many minimum number of complete years, the by him at the end of eleven years is ` 8160, how much
interest on ` 212.50 P at 3% per annum will be in exact
money did he borrow?
number of rupees?
(a) 6 (b) 8 (a) ` 8000 (b) ` 10,000
(c) 9 (d) 7 (c) ` 12,000 (d) Data inadequate
9. A milk man borrowed ` 2,500 from two money lende For 17. An automobile financier claims to be lending money at simple
one loan, he paid 5% p.a. and for the other, he paid 7% p.a. interest, but he includes the interest every six months for
The total interest paid for two years was ` 275. How much calculating the principal. If he is charging an interest of
did he borrow at 7% rate? 10%, the effective rate of interest becomes :
(a) ` 600 (b) ` 625 (a) 10% (b) 10.25%
(c) ` 650 (d) ` 675
(c) 10.5% (d) None of these
y
o
u
Simple and Compound Interest B-85

rs
A lent ` 5000 to B for 2 years and ` 3000 to C for 4 years on

m
18. 26. A person invested in all ` 2600 at 4%, 6% and 8% per annum

a
simple interest at the same rate of interest and received simple interest. At the end of the year, he got the same

h
` 2200 in all from both of them as interest. The rate of interest

b
interest in all the three cases. The money invested at 4% is :

o
per annum is: (a) ` 200 (b) ` 600

o
b
(a) 5% (b) 7% (c) ` 800 (d) ` 1200

.w
1 27. Divide ` 2379 into 3 parts so that their amounts after 2, 3

o
(c) 7 % (d) 10%

rd
8 and 4 years respectively may be equal, the rate of interest

p
19. A sum of ` 725 is lent in the beginning of a year at a certain being 5% per annum at simple interest. The first part is:

re
rate of interest. After 8 months, a sum of ` 362.50 more is (a) 759 (b) 792

s
s
lent but at the rate twice the former. At the end of the year, (c) 818 (d) 828

.c
28. A man invests ` 3000 at a rate of 5% per annum. How much

o
` 33.50 is earned as interest from both the loans. What was

m
the original rate of interest? more should he invest at a rate of 8%, so that he can earn a
(a) 3.6% (b) 4.5% total of 6% per annum ?
(c) 5% (d) None of these (a) ` 1200 (b) ` 1300
20. The difference between the simple interest received from (c) ` 1500 (d) ` 2000
two different sources on ` 1500 for 3 years is ` 13.50. The 29. Two equal sums of money are lent at the same time at 8%
difference between their rates of interest is: and 7% per annum simple interest. The former is recovered
(a) 0.1% (b) 0.2% 6 months earlier than the later and the amount in each case
(c) 0.3% (d) 0.4% is ` 2560. The sum and the time for which the sum of money
21. The rates of simple interest in two banks A and B are in the were lent out, are:
ratio 5 : 4. A person wants to deposit his total savings in (a) ` 2500, 3.5 years and 4 years
two banks in such a way that he received equal half-yearly (b) ` 2000, 3.5 years and 4 years
interest from both. He should deposit the savings in banks (c) ` 2000, 4 years and 5.5 years
A and B in the ratio. (d) ` 3000, 4 years and 4.5 years
(a) 2 : 5 (b) 4 : 5 30. Peter invested an amount of ` 12,000 at the rate of
(c) 5 : 2 (d) 5 : 4 10% p.a simple interest and another amount at the rate of
22. The price of a T.V. set worth ` 20,000 is to paid in 20 20% p.a. simple interest. The total interest earned at the end
instalments of ` 1000 each. If the rate of interest be 6% per of one year on the total amount invested became 14% p.a.
annum, and the first instalment be paid at the time of Find the total amount invested?
purchase, then the value of the last instalment covering the (a) ` 20,000 (b) ` 22,000
interest as well will be : (c) ` 24,000 (d) ` 25,000
(a) ` 1050 (b) ` 2050 31. Sumit lent some money to Mohit at 5% per annum simple
(c) ` 3000 (d) None of these interest. Mohit lent the entire amount to Birju on the same
23. Mr. Thomas invested an amount of ` 13,900 divided in two
different schemes A and B at the simple interest rate of 14% 1
day at 8 % per annum. In this transaction, after a year,
p.a. and 11% p.a. respectively. If the total amount of simple 2
interest earned in 2 years be ` 3508, what was the amount Mohit earned a profit of ` 350. Find the sum of money lent
invested in Scheme B? by Sumit to Mohit.
(a) ` 6400 (b) ` 6500 (a) ` 10,000 (b) ` 9,000
(c) ` 7200 (d) ` 7500 (c) ` 10,200 (d) None of these
24. An amount of ` 1,00,000 is invested in two types of shares. 32. Rajesh gave ` 1200 on loan. Some amount he gave at 4%
The first yields an interest of 9% p.a. and the second, 11% per annum simple interest and remaining at 5% per annum
3 simple interest. After two years, he got ` 110 as interest.
p.a. If the total interest at the end of one year is 9 % , then
4 Then the amounts given at 4% and 5% per annum simple
the amount invested in each share was: interest are, respectively :
(a) ` 52,500; ` 47,500 (a) ` 500, ` 700 (b) ` 400, ` 800
(b) ` 62, 500; ` 37,500 (c) ` 900, ` 300 (d) ` 1100, ` 1100
(c) ` 72,500: ` 27,500 33. If the difference between S.I and C.I for 2 years on a sum of
(d) ` 82, 500; ` 17,500 money lent at 5% is ` 6, then the sum B.
25. David invested certain amount in three different Schemes (a) ` 2200 (b) ` 2400
A, B and C with the rate of interest 10% p.a., 12% p.a. and (c) ` 2600 (d) ` 2000
15% p.a. respectively. If the total interest accrued in one 34. Zovo got a loan of ` 8,000 against his fixed deposits to
year was ` 3200 and the amount invested in Scheme C was purchase a scooter. If the rate of interest is 10% p.a.
150 % of the amount invested in Scheme A and 240% of the compounded half yearly, find the amount that he pays after
amount invested in Scheme B, what was the amount invested
one and half year.
in Scheme B?
(a) ` 9000 (b) ` 9200
(a) ` 5000 (b) ` 6500
(c) ` 9350 (d) ` 9500
(c) ` 8000 (d) cannot be determined
y
o
u
B-86 Simple and Compound Interest

rs
m
35. A sum of money placed at compound interest doubles itself money to another person at 10% compound interest per

a
in 5 years Find in how many years it will become 8 times. annum and thereby the man made a profit of ` 13,205 in 4

h
(a) 15 years (b) 20 years years. The man borrowed

b
o
(c) 25 years (d) 30 years (a) ` 80,000 (b) ` 1,00,000

o
b
36. The C.I. on a certain sum of money for the 4th year at 8% (c) ` 1,20,000 (d) ` 1,50,000

.w
p.a. is ` 486. What was the compound interest for the third 39. Two equal sums were borrowed at 8% simple interest per

o
year on the same sum at the same rate? annum for 2 years and 3 years respectively. The difference

rd
(a) ` 450 (b) ` 475 in the interest was ` 56. The sums borrowed were

p
re
(c) ` 456 (d) None of these (a) ` 800 (b) ` 700

s
37. A sum amounts double in 8 years by simple interest. Then (c) ` 560 (d) ` 350

s
.c
the rate of simple interest p.a. is 40. The difference between C. I. (Compound Interest) and S.I.

o
m
(a) 10% (b) 12.5% (Simple Interest) on a sum of ` 4,000 for 2 years at 5% p.a.
(c) 15% (d) 20% payable yearly is
38. A man borrowed some money from a private organisation (a) ` 20 (b) ` 10
at 5% simple interest per annum. He lended 50% of this (c) ` 50 (d) ` 60

ANSWER KEY
1 (b) 6 (c) 11 (b) 16 (a) 21 (b) 26 (d) 31 (a) 36 (a)
2 (a) 7 (b) 12 (a) 17 (b) 22 (d) 27 (d) 32 (a) 37 (b)
3 (b) 8 (b) 13 (d) 18 (d) 23 (a) 28 (c) 33 (b) 38 (b)
4 (c) 9 (b) 14 (c) 19 (d) 24 (b) 29 (b) 34 (b) 39 (b)
5 (a) 10 (c) 15 (d) 20 (c) 25 (a) 30 (a) 35 (a) 40 (b)

1 1 1 P T
1. (b) Rest part 1 4. (c) Difference in S.I. (R1 R 2 )
3 6 2 100
Rate % per annum on total sum
P 4 2
56 ( R1 – R2 = 2)
1 1 1 100
3 6 8 6%
3 6 2
56 100
P ` 700
600 100 4 2
P ` 5, 000
6 2 16 P R R
2. (a) S.I. for 1½ years = ` (1164 – 1008) = ` 156 5. (a) P
25 100
156 2 2
S. I. for 2 years = ` Rs 208 1600 40
3 R2 R 8%
25 5
Principal = ` (1008 – 208) = ` 800 Also, time = 8 years
Now , P = 800, T= 2, S.I. = 208 6. (c) Let one gets = ` x
100 208 then, second gets = ` (68,000 – x)
Rate % 13% Given : A1 = A2
800 2
3. (b) S.I. for 5 years = ` (1020 –720) = ` 300 x 10 8 (68000 x) 10 6
x (68, 000 x)
300 100 100
SI. for 2 years = Rs 2 Rs 120
5 x[100 80] (68, 000 x)[100 60]
Principal = ` (720 – 120) = ` 600
180x
Now, P = 600, T = 2, S.I. = 120 68, 000 – x
160
120 100
R 10% 34x 68000 16 x Rs32,000
600 2
second gets = ` 36,000
y
o
u
Simple and Compound Interest B-87

rs
m
Altornatively: 12. (a) Gain in 2 years

a
h
1 25 2 5000 4 2

b
1 =` 5000

o
S1 100 r1t1 8 4 100 100

o
180

b
S2 1 9 = ` (625 – 400) = ` 225.

.w
1
160

o
100 r2 t2 225

rd
Gain in 1 year = ` = ` 112.50
2

p
re
8
S1 68000 32000 13. (d) We need to know the S.I., principal and time to find the

s
17

s
rate. Since the principal is not given, so data is

.c
S2 = 68000 – 32000 = 36000 inadequate.

o
m
Q r t R r t 14. (c) Let the principal be P and rate of interest be R%.
7. (b) P and Q
100 100 P R 6
P Q r t 100 6PR 6
Required ratio = 2 : 3.
Q R 100 P R 9 9PR 9
Q2 = PR. 100

3 51 15. (d) Difference of S.I. = ` 31.50


8. (b) Interest for one year ` 212.50
1 `
100 8 Let each sum be ` x. Then
Thus in 8 years, the interest is ` 51.
9. (b) Let he borrowed at 5% = ` x 1
x 4 7
2 x 4 7
He borrowed at 7% = ` (2500 – x) 31.50
Now I1 + I2 = 275 100 100

x 5 2 (2500 x) 7 2 7 x 1 63
275 or
100 100 100 2 2
10x + 14 (2500 – x) = 27500 or x = ` 900
4x = 35000 – 27500 = 7500 16. (a) Let the sum be ` x. Then,
x = ` 1875 x 6 3 x 9 5 x 13 3
Sum borrowed at 7% rate = 2500 – 1875 = ` 625 8160
100 100 100
10. (c) Shortcut method :
18 x + 45x + 39x = (8160 × 100) 102x = 816000
If borrowed amount be ` M and it is to be paid in equal
x = 8000.
instalments, then
17. (b) Let the sum be ` 100. Then,
ra n(n 1)
M na 100 10 1
100 Y 2 S.I. for first 6 months = ` Rs. `
100 2
where Y = no. of instalments per annum
a = annual instalment 105 10 1
S.I. for last 6 months = ` ` 5.25.
Here, M = 4200, y = 1, r = 10, n = 5, a = ? 100 2
10a 5(5 1) So, amount at the end of 1 year = ` (100 + 5 + 5.25)
4200 5a = ` 110.25.
100 2
Effective rate = ` (110.25 – 100) = 10.25%.
4200 a 5 1 6a 4200 18. (d) Let the rate be R% p.a. Then,
a = ` 700 5000 R 2 3000 R 4
2200
11. (b) Let the sum borrowed be x. Then, 100 100
x 6 2 x 9 3 x 14 4 2200
11400 100R 120R 2200 R 10.
100 100 100 220
19. (d) Let the original rate be R%. Then, new rate = (2R)%.
3x 27x 14x 95x
11400 11400 725 R 1 362.50 2R 1
25 100 25 100 33.50
100 100 3
11400 100 (2175 725)R 33.50 100 3 10050
x 12000.
95 10050
R 3.46%
Hence, sum borrowed = ` 12,000. 2900
y
o
u
B-88 Simple and Compound Interest

rs
m
1500 R1 3 1500 R 2 3 3

a
20. (c) 13.50 And, z = 150% of x = x

h
100 100 2

b
o
o
1350 2 2 12 8

b
4500(R 1 R 2 ) 1350 R1 R 2 0.3% x z y y ..... (iii)

.w
4500 3 3 5 5

o
21. (b) Let the savings be X and Y and the rates of simple From (i), (ii) and (iii), we have :

rd
interest be 5x and 4x respectively. 16y + 12y + 36y = 320000 64y = 320000 y = 5000.

p
re
Sum invested in Scheme B = ` 5000.
1 1 1 1 X 4

s
Then, X 5x Y 4x or , 26. (d) Let the parts be x, y and [2600 – (x + y)]. Then,

s
.c
2 100 2 100 Y 5

o
i.e., X : Y = 4 : 5. x 4 1 y 6 1 [2600 (x y)] 8 1

m
22. (d) Money paid in cash = ` 1000. 100 100 100
Balance payment = ` (20000 – 1000) = ` 19000. y 4 2 2
23. (a) Let the sum invested in Scheme A be ` x and that in or y x.
x 6 3 3
Scheme B be ` (13900 – x).
5
x 14 2 (13900 x) 11 2 2600 x 8
Then,
100 100
3508 So, x 4 1 3
100 100
28x – 22x = 350800 – (13900 × 22) 6x = 45000
(7800 5x) 8
x = 7500. 4x 52x (7800 8)
3
So, sum invested in Scheme B = ` (13900 – 7500)
= ` 6400. 7800 8
24. (b) Let the sum invested at 9% be ` x and that invested at x 1200.
52
11% be ` (100000 – x).
Money invested at 4% = ` 1200.
Then,
27. (d) Let the parts be x, y and [2379 – (x + y)].
x 9 1 (100000 x) 11 1
5 5
100 100 x x 2 y y 3
100 100
39 1
100000 5
4 100 z z 4
100
9x 1100000 11x 39000
9750 11x 23y 6z 10k 20k 5k
100 4 k x ,y ,z
10 20 5 11 23 6
2x = (1100000 – 975000) = 125000 x = 62500.
Sum invested at 9% = ` 62500. But x + y +z = 2379.
Sum invested at 11% = ` (100000 – 62500) = ` 37500. 10k 20k 5k
Alternatively: 2379
11 23 6
r r2 r r1 1380 k + 1320 k + 1256 k = 2379 × 11 × 23 × 6
P1= p r r & P2 p
r2 r1
1 2 2379 11 23 6 3 11 23 6
k
3965 5
3 3
9 –11 9 –9 10 3 11 23 6
P1 1,00,000 4 , P2 1,00,000 4 x 828.
9 –11 11 – 9 11 5
Hence, the first part is ` 828.
P1 = 62500, P2 = 37500 28. (c) 3000(1 0.05) x (1 0.08) (3000 x )(1 0.06)
25. (a) Let x, y and z be the amounts invested in schemes A, B or 3150 1.08x 3180 1.06x
and C respectively. Then,
30
x 10 1 y 12 1 z 15 1 or 0.02x 30 or x 1500
3200 0.02
100 100 100 29. (b) Let the sum be ` x.
10x + 12y + 15z = 320000 ..... (i) ` x is lent at 8%for t years and ` x is also lent at 7%
12 1
Now, z = 240% of y = y ..... (ii) for t years
5 2
y
o
u
Simple and Compound Interest B-89

rs
m
x t 8 5 5
R R

a
x 2560 …(1) P 1 2P 1 2 … (i)

h
100 35. (a)
100 100

b
o
o
x (2 t 1) 7 n

b
and x 2560 …(2) R

.w
2 100 Let P 1 8P
100

o
Solving (1) and (2),

rd
p
x = ` 2,000, t = 3.5 years n 5 3

re
R 3 R
30. (a) Let the second amount be ` x. Then, 1 2 1 [By (i)]

s
100 100

s
.c
12, 000 10 1 x 20 1 (12000 x) 14 1

o
m
n 15
100 100 100 R R
1 1
120000 + 20 x = (12000 + x) 14 100 100
6x = 168000 – 120000 n = 15 years
x = ` 8000
Total investment = 12,000 + 8000 = ` 20,000 36. (a) If ‘x’ be the interest of third year, then 108% of x = 486
31. (a) We have, 100
x 486 450
108
1
8 5 % of x ` 350 37. (b) Let P be the principle amount and R be rate of interest.
2
P R 8
2P P
350 100
100% of x 100 ` 10, 000
3.5 100
R 12.5%
32. (a) Let the amount of the loss at 4% per annum be ` x. 8
Amount given at 5% per annum = ` (1200 – x) 39. (b) Let principal be represented by P.
Ist Case :
x 4 2 (1200 x ) 5 2
Now, 110
100 100 P R T P 8 3
S.I.
x = ` 500 100 100
And, (1200 – x) = 1200 – 500 = ` 700 IInd Case :
P R T P 8 3
PR 2 S.I
33. (b) Difference = 100 100
10000 According to question
P×5×5 P 8 3 P 8 2
6= – 56
10000 100 100
6 × 400 = ` 2400.
P 8 56 100
34. (b) Here, n = 3 for interest is reckoned half-yearly. 56 P 700
100 8
3
10
A 8, 000 1 PR 2
2 100 40. (b) Required difference =
(100)2
3
21 4000 5 5
A 8,000 ` 9261 ` 9200 =`10
20 100 100
y
o
u
B-90 Ratio, Proportion and Partnership

rs
m
CHAPTER

a
h
RATIO, PROPORTION
8

b
o
o
b
AND PARTNERSHIP

.w
o
rd
p
re
s
s
.c
RATIO

o
m
Ratio is strictly a mathematical term to compare two similar
quantities expressed in the same units. The two quantities must be of the same kind and in same
unit.
The ratio of two terms ‘x’ and ‘y’ is denoted by x : y.
The ratio is a pure number, i.e., without any unit of
In general, the ratio of a number x to a number y is
measurement.
defined as the quotient of the numbers x and y.
The ratio would stay unaltered even if both the antecedent
The numerator of the ratio is called the antecedent (x) and the consequent are multiplied or divided by the same
and the denominator is called the consequent (y) of the ratio. number.
COMPARISON OF TWO OR MORE RATIOS
Compound ratio : Ratios are compounded by multiplying
Two or more ratios may be compared by reducing the equivalent together the antecedents for a new antecedent and the
fractions to a common denominator and then comparing the consequents for a new consequent.
magnitudes of their numerator. Thus, suppose 2 : 5, 4 : 3 and 4 : 5
a c
2 4 4 The compound of a : b and c : d is , i.e., ac : bd.
are three ratios to be compared then the fractions , and b d
5 3 5 Example 3 :
are reduced to equivalent fractions with a common denominator. Find the compound ratio of the four ratios :
For this, the denominator of each is changed to 15 equal to the 4 : 5, 15 : 13, 26 : 3 and 6 : 17
L.C.M. their denominators. Hence the given ratios are expressed Solution :

6 20 12 4 15 26 6 48
, and or The required ratio =
15 15 15 5 13 3 17 17
2 : 5, 4 : 3, 4 : 5 according to magnitude. or 48 : 17
The duplicate ratio of x : y is x2 : y2.
Example 1 : The triplicate ratio of x : y is x3 : y3.
Which of the ratios 2 : 3 and 5 : 9 is greater ? The subduplicate ratio of x : y is x : y.
Solution :
The subtriplicate ratio of x : y is 3 x : 3 y .
2 5
In the form of fractions, the given ratios are and , 1 1
3 9 Reciprocal ratio of a : b is : or b : a
Reducing them to fractions with a common denominator a b
Inverse ratio
6 5 Inverse ratio of x : y is y : x.
they are written as and .
9 9
PROPERTIES
6
Hence the greater ratio is or 2 : 3. a c
9 b d
1. If then , i.e., the inverse ratios of two
b d a c
Example 2 : equal ratios are equal. The property is called Invertendo.
Are the ratios 3 to 4 and 6:8 equal ?
a c a b
Solution : 2. If then , i.e., the ratio of antecedents and
b d c d
The ratios are equal if 3/4 = 6/8.
These are equal if their cross products are equal; that is, consequents of two equal ratios are equal. This property
if 3 × 8 = 4 × 6. Since both of these products equal 24, the is called Alternendo.
answer is yes, the ratios are equal. a c a b c d
3. If , th en . This property is
Remember to be careful! Order matters! b d b d
A ratio of 1 : 7 is not the same as a ratio of 7:1. called Componendo.
y
o
u
Ratio, Proportion and Partnership B-91

rs
a : b

m
a c a b c d

a
4. If , th en . This property is

h
b d b d

b
o
called Dividendo.

o
b
c : d

.w
a c a b c d
5. If , th en . This property is

o
b d a b c d

rd
p
called Componendo – Dividendo.

re
e : f

s
s
a c e ace : bce bde : bdf

.c
6. If .............. . Then,
b d f

o
To divide a given quantity into a given ratio.

m
Suppose any given quantity a, is to be divided in the ratio
sum of Numerators m : n.
Each ratio = sum of denominators
Let one part of the given quantity be x then the other part
will be a – x.
a c a c e ...
i.e. x m
b d b d f ... or nx = ma – mx or (m + n) x = ma
a x n
7. If we have two equations containing three unknowns as
a1x + b1y + c1z = 0 and ... (i) ma
one part is and the other part will be
a2x + b2y + c2z = 0 ... (ii) m n
then, the values of x, y and z cannot be resolved without ma na
having a third equation. a
m n m n
However, in the absence of a third equation, we can find
the proportion x : y : z. Example 4 :
This will be given by Divide 70 in the ratio 3 : 7.
b1c2 – b2c1 : c1a2 – c2a1 : a1b2 – a2b1. Solution :
8. To find the ratio of the two variables of a homogeneous Let one part be x
equation of second degree. then the other part = 70 – x
For this all the terms of the homogeneous equation are x 3
taken on one side and factorized into linear factors. A linear or 7x = 210 – 3x
70 x 7
equation is formed from each of the factors and the ratio
of the variables obtained or x = 21 and 70 – x = 49
Hence the two required parts of 70 are 21 and 49.
9. A number which when subtracted from the terms of the
Short cut method:
bc ad
ratio a:b make it equal to the ratio c:d is 3
c d First part 70 21
10

If the ratio between the first and the second quantities is 7


Second part 70 49
a : b and the ratio between the second and third quantities 10
is c : d, then the ratio among first, second and third quanti- Example 5 :
ties is given by ac : bc : bd
What is the least integer which when subtracted from both
The above ratio can be represented diagrammatically as
a : b 60
the numerator and denominator of will give a ratio equal
70
16
to ?
c : b 21
Solution :
ac : bc : bd Let x be the required integer. Then,
If the ratio between the first and the second quantities is 60 x 16
a : b and the ratio between the second and third quanti-
70 x 21
ties is c : d and the ratio between the third and fourth
quantities is e : f, then the ratio among first, second, third 1260 – 21x = 1120 – 16x
and fourth quantities is given by ace : bce : bde : bdf 5x = 140 x = 28.
y
o
u
B-92 Ratio, Proportion and Partnership

rs
Example 6 : Solution :

m
Ratio of their areas = 22 : 52 = 4 : 25

a
x 4 3x 4y

h
If , find the value of .

b
y 5 4x 3y In any two 3-dimensional figures, if the corresponding

o
o
Solution : sides are in the ratio x : y, then their volumes are in the

b
ratio x3 : y3.

.w
3x

o
4 4 If the ratio between two numbers is a : b and

rd
3x 4y 3 4
y 5 32 if each number is increased by x, the ratio becomes c : d.
.

p
4x 4

re
4x 3y 31 Then
3 4 3

s
y 5 x (a b) (c d)

s
Sum of the two numbers =

.c
Example 7 : ad bc

o
m
x (a b) (c d)
Find the value of x a x b , if x = 2ab . Difference of the two numbers =
ad bc
x a x b a b
Solution : xa (c d) xb (c d)
Two numbers are given as and
2ab x 2b ad bc ad bc
x= Example 10 :
a b a a b
The ratio between two numbers is 3 : 4. If each number
By componendo – dividendo,
be increased by 2, the ratio becomes 7 : 9. Find the
x a 3b a numbers.
x a b a Solution :

x 2a 2 3 (7 9) 2 4 (7 9)
Similarly, Numbers are and or 12 and 16.
b a b 3 9 4 7 3 9 4 7

x b 3a b If the sum of two numbers is A and their difference is


x b a b a, then the ratio of numbers is given by A + a : A – a.

x a x b 3b a 3a b Example 11 :
x a x b b a a b The sum of two numbers is 60 and their difference is 6.
What is the ratio of the two numbers ?
(3b a) 3a b 2a 2b Solution :
2.
a b a b a b The required ratio of the numbers
Example 8 :
60 6 66 11
Divide ` 581 among A, B and C such that four time A’s or 11 : 9
60 6 54 9
share is equal to 5 times B’s share which is equal to
seven times C’s share. Example 12 :
Solution : Three persons A, B, C whose salaries together amount to
4 times A’s share = 5 times B’s share ` 14400, spend 80, 85 and 75 per cent of their salaries
= 7 times C’s share. respectively. If their savings are in the ratio 8 : 9 : 20, find
A 's share B 's share C 's share their respective salaries.
35 28 20 Solution :
[Dividing by L.C.M. of 4, 5 and 7 i.e. 140] A, B and C spend 80%, 85% and 75% respectively of their
A : B : C = 35 : 28 : 20 salaries.
35 A, B and C save 20%, 15% and 25% respectively of
Share of A = 581 = ` 245.
35 28 20 their salaries.
28 So, 20 % of A’s salary : 15% of B’s salary :
Share of B = 581 = ` 196. 25 % of C’s salary = 8 : 9 : 20
83
20 1 3
Share of C = 581 = ` 140. of A’s salary : of B’s salary :
83 5 20
In any 2-dimensional figures, if the corresponding sides
are in the ratio x : y, then their areas are in the ratio 1
of C’s salary = 8 : 9 : 20 ....(i)
x2 : y2. 4
Example 9 : 1
of A 's salary
The ratio of the radius of two circles is 2 : 5. Find the ratio 8
Now 5
of their areas. 3 9
of B's salary
20
y
o
u
Ratio, Proportion and Partnership B-93

rs
If three quantities a, b and c are in continued proportion,

m
A 's salary 3 5 2

a
8 then a : b = b : c

h
B 's salary 20 9 3 ac = b2

b
o
b is called mean proportional.

o
A’s salary : B’s salary = 2 : 3 ...(ii)

b
If three quantities are proportionals, then first is to the third

.w
Similarly, B’s salary : C’s salary = 3 : 4 ...(iii) is the duplicate ratio of the first is to the second.

o
If a : b :: b : c then a : c = a2 : b2

rd
From (ii) and (iii)

p
A’s salary : B’s salary : C’s salary = 2 : 3 : 4.

re
TO FIND THE MEAN PROPORTIONAL

s
s
2 Example 13 :

.c
A’s salary = 14400 = ` 3200

o
2 3 4 Find the mean proportional between 3 and 75.

m
Solution :
3 Let x be the required mean proportional. Then,
B’s salary = × 14400 = ` 4800 3 : x :: x : 75
2 3 4
x = 3 75 15
4
C’s salary = × 14400 = ` 6400. Example 14 :
2 3 4
A courier charge to a place is proportional to the square
a k a root of the weight of the consignment. It costs ` 54 to
a
(viii) If the ratio 1 and k is a positive number, then courier a consignment weighing 25 kilos. How much more
b b k b
will it cost (in rupees) to courier the same consignment as
two parcels weighing 16 kilos and 9 kilos respectively ?
a k a
and b Solution :
k b
Courier charges weight of the consignment
a
Similarly, if 1 , then
b or Courier charges = k weight of the consignment .
a k a a k a For weight = 25 kilos, courier charges is given to ` 54.
and
b k b b k b 54 = k 25 or k = 10.80
Cost of courier for 16 kilos,
c a a c a
(ix) If , then b d b C16 k 16 10.8 16
d b
C16 = ` 43.20
c a a c a Cost of courier for 9 kilos,
and if , then b d b
d b
C9 k 9 = 10.8 9 = ` 32.4
PROPORTION Total cost of courier for two parcels
When two ratios are equal, the four quantities composing them C16 + C9 = 43.20 + 32.40 = 75.60
are said to be in proportion. Difference to be paid = 75.60 – 54 = ` 21.60.

a c TO FIND THE VALUES OF AN UNKNOWN WHEN FOUR


If , then a, b, c, d are in proportions.
b d NUMBERS ARE IN PROPORTION
Example 15 :
This is expressed by saying that ‘a’ is to ‘b’ as ‘c’ is to ‘d’
What must be added to each of the four numbers 10, 18, 22,
and the proportion is written as
38 so that they become in proportion ?
a : b :: c : d or a:b=c: d Solution :
The terms a and d are called the extremes while the terms Let the number to be added to each of the four numbers
b and c are called the means. be x.
By the given condition, we get
(10 + x) : (18 + x) : : (22 + x) : (38 + x)
If four quantities are in proportion, the product of the
(10 + x) (38 + x) = (18 + x) (22 + x)
extremes is equal to the product of the means.
380 + 48x + x2 = 396 + 40x + x2
Let a, b, c, d be in proportion, then
a c Cancelling x2 from both sides, we get
ad = bc. 380 + 48x = 396 + 40x
b d
48x – 40x = 396 – 380
y
o
u
B-94 Ratio, Proportion and Partnership

rs
m
16 DIRECT PROPORTION
8x = 16 2x

a
If on the increase of one quantity, the other quantity

h
8

b
increases to the same extent or on the decrease of one, the

o
Therefore, 2 should be added to each of the four given
other decreases to the same extent, then we say that the given

o
numbers.

b
two quantities are directly proportional. If A and B are directly

.w
TO FIND THE FOURTH PROPORTIONAL
proportional then we denote it by A B.

o
Example 16 :

rd
Also, A = kB, k is constant
Find the fourth proportional to p² – pq + q2, p3 + q3, p – q

p
re
A
Solution : k

s
B

s
Let x be the fourth proportional

.c
If b1 and b2 are the values of B corresponding to the

o
(p2– pq + q2) : (p3 + q3) = (p – q) : x

m
values a1, a2 of A respectively, then
(p2 – pq + q2) × x = (p3 + q3) (p – q)
a1 a2
(p3 q3 ) (p q) b1 b2
x
(p 2 pq q 2 ) Some Examples :
1. Work done number of men
(p q) (p2 pq q 2 ) (p q) 2. Cost number of Articles
x
(p2 pq q 2 ) 3. Work wages
4. Working hour of a machine fuel consumed
x (p q) (p q) p2 q2 5. Speed distance to be covered
INDIRECT PROPORTION (OR INVERSE PROPORTION)
The required fourth proportional is p2 – q2
If on the increase of one quantity, the other quantity
TO FIND THE THIRD PROPORTIONAL decreases to the same extent or vice versa, then we say that the
Example 17 : given two quantities are indirectly proportional. If A and B are
Find third proportional to a2 – b2 and a + b.
1
Solution : indirectly proportional then we denote it by A .
B
Let x be the required third proportional
Then a2 – b2 : a + b = a + b : x k
Also, A (k is a constant)
(a2 – b2) x = (a + b) (a + b) B
AB = k
a b) (a b) a b
x If b1, b2 are the values of B corresponding to the values
2 2 a b
a b a1, a2 of A respectively, then
USING THEOREM ON EQUAL PROPORTION a1b1 = a2b2
Example 18 : Some Examples :
1. More men, less time
a b c 1
If , prove that each is equal to or –1. 2. Less men, more hours
b c c a a b 2 3. More speed, less taken time to be covered distance
Solution : Example 19 :
a b c A garrison of 3300 men had provision for 32 days, when
We have given at the rate of 850 gm per head. At the end of 7 days, a
b c c a a b
reinforcement arrived and it was found that the provision
Sum of antecedents would last 17 days more, when given at the rate of 825 gm
Each ratio = per head. What was the strength of the reinforcement ?
Sum of consequents
Solution :
[By theorem on equal ratios]
There is a provision for 2805 × 32 kg for 3300 men for 32
a b c a b c 1 days @ 850 gm per head per day.
= = if a b c 0
b c c a a b 2(a b c) 2 In 7 days, 3300 men consumed
If a + b + c = 0, then b + c = –a 2805 32
a a 7 = 2805 × 7 kg.
1 32
b c a
Let the strength of the reinforcement arrived after
b b c c
Similarly, 1, 1 7 days be x.
c a b a b c (3300 + x) men had provision of 2805 × 25 kg for
1 17 days @ 825 gm per head per day, i.e.
Hence each ratio = if a b c 0 = –1 if a + b + c = 0
2
y
o
u
Ratio, Proportion and Partnership B-95

rs
m
(3300 x) 825 17 5 3 5 3
: ::10:x x 10

a
= 2805 × 25 Then,

h
1000 8 8 8 8

b
o
3 8
6.

o
1000 2805 25 x 10

b
(3300 + x) = = 5000 8 5

.w
825 17
Example 22 :

o
x = 1700

rd
A fort had provision of food for 150 men for 45 days. After
Strength of the reinforcement arrived after 7 days

p
10 days, 25 men left the fort. The number of days for which

re
= 1700. the remaining food will last, is :

s
On k-method

s
Solution :

.c
Example 20 :

o
After 10 days : 150 men had food for 35 days.

m
Suppose 125 men had food for x days. Now,
(a b)2 a
If a, b, c be in continued proportion, prove that 2
. Less men, More days (Indirect Proportion)
(b c) c Then,
Solution : men days

a b 150 35
Let k , then a = bk and b = ck
b c 125 x
Hence a = bk = ck . k = ck2 and b = ck 125 :150 ::35:x 125 x 150 35
Substituting these values of a and b, we get
150 35
2 2 2 2 2 2
x x 42.
(ck ck) [ck(k 1) c k (k 1) 125
L.H.S. = 2 2 2 2
k2 Hence, the remaining food will last for 42 days.
(ck c) [c(k 1)] c (k 1)
Compound Proportion or Double Rule of Three
In the compound proportion, number of ratios are more than two.
ck 2
and R.H.S. = k2 Example 23 :
c If the cost of printing a book of 320 leaves with 21 lines on
each page and on an average 11 words in each line is ` 19,
a b) 2 a find the cost of printing a book with 297 leaves, 28 lines on
L.H.S. = R.H.S. Hence 2
(b c) c each page and 10 words in each line.
Solution :
RULE OF THREE Less leaves, less cost (Direct Proportion)
In a problem on simple proportion, usually three terms are More lines, more cost (Direct Proportion)
given and we have to find the fourth term, which we can solve by Less words, less cost (Direct Proportion)
using Rule of three. In such problems, two of given terms are of
same kind and the third term is of same kind as the required fourth leaves 320 : 297
term. First of all we have to find whether given problem is a case lines 21: 28 ::19 : x
of direct proportion or indirect proportion. For this, write the words 11:10
given quantities under their respective headings and then mark
the arrow in increasing direction. If both arrows are in same direc- 320 × 21 ×11 ×x = 297 × 28 × 10 × 19
tion then the relation between them is direct otherwise it is indi- 171 3
x 21
rect or inverse proportion. Proportion will be made by either head 8 8
to tail or tail to head. Example 24 :
The complete procedure can be understand by the
If 80 lamps can be lighted, 5 hours per day for 10 days for `
examples. 21.25, then the number of lamps, which can be lighted 4
Example 21 : hours daily for 30 days, for ` 76.50, is :
A man completes 5/8 of a job in 10 days. At this rate, how Solution :
many more days will it take him to finish the job ?
Solution : Let the required number of lamps be x.
Less hours per day, More lamps
5 5 3 (Indirect Proportion)
Work done = . Balance work = 1 .
8 8 8 More money, More lamps (Direct Proportion)
Less work, Less days (Direct Proportion) More days, Less lamps (Indirect Proportion)
Let the required number of days be x. Then,
Work days Hours per day 4:5
5/8 10 Money 21.25 : 76.50 : : 80 : x
Number of days 30 :10
3/8 x
y
o
u
B-96 Ratio, Proportion and Partnership

rs
m
4 21.25 30 x 5 76.50 10 80 1 27x

a
x 369

h
10 140

b
5 76.50 10 80

o
x x 120. 14x + 27x = 369 × 140

o
4 21.25 30

b
.w
PARTNERSHIP 369 140
x 9 140 1260

o
A partnership is an association of two or more persons 41

rd
who invest their money in order to carry on a certain business.

p
5 9
1260 = ` 405

re
A partner who manages the business is called the B’s share
14 10

s
working partner and the one who simply invests the money is

s
.c
called the sleeping partner. 6 9

o
C’s share 1260 = ` 486

m
Partnership is of two kinds : 14 10
(i) Simple (ii) Compound. Example 27 :
Simple partnership : If the capitals is of the partners are A and B invested in the ratio 3 : 2 in a business. If 5% of the
invested for the same period, the partnership is called simple. total profit goes to charity and A’s share is ` 855, find the
Compound partnership : If the capitals of the partners total profit.
are invested for different lengths of time, the partnership is Solution :
called compound. Let the total profit be ` 100.
Then, ` 5 goes to charity.
If the period of investment is the same for each partner, Now, ` 95 is divided in the ratio 3 : 2.
then the profit or loss is divided in the ratio of their
95
investments. A’s share 3 ` 57
3 2
If A and B are partners in a business, then
But A’s actual share is ` 855.
Investment of A Pr ofit of A Loss of A
or 100
Investment of B Pr ofit of B Loss of B Actual total profit 855 ` 1500
57
If A, B and C are partners in a business, then
Investment of A : Investment of B : Investment of C MONTHLY EQUIVALENT INVESTMENT
= Profit of A : Profit of B : Profit of C, or It is the product of the capital invested and the period for
= Loss of A : Loss of B : Loss of C which it is invested.
Example 25 : If the period of investment is different, then the profit or
Three partner Rahul, Puneet and Chandan invest ` 1600, loss is divided in the ratio of their Monthly Equivalent Invest-
` 1800 and ` 2300 respectively in a business. How ment.
should they divide a profit of ` 399 ?
Monthly Equivalent Investment of A
Solution :
Monthly Equivalent Investment of B
Profit is to be divided in the ratio 16 : 18 : 23
16 Pr ofit of A Loss ofA
Rahul’s share of profit 399 = or
16 18 23 Pr ofit of B Loss of B
16
399 ` 112 Investment of A Period of Investment of A
57 i.e.,
Investment of B Period of Investment of B
18
Puneet’s share of profit 399 ` 126
57 Pr ofit of A Loss ofA
23 or
Chandan’s share of profit 399 ` 161 Pr ofit of B Loss of B
57
Example 26 : If A, B and C are partners in a business, then
A, B and C enter into a partnership by investing 1500, Monthly Equivalent Investment of A : Monthly Equivalent
2500 and 3000 rupees, respectively. A as manager gets Investment of B : Monthly Equivalent Investment of C
one-tenth of the total profit and the remaining profit is = Profit of A : Profit of B : Profit of C.
divided among the three in the ratio of their investment. = Loss of A : Loss of B : Loss of C.
If A’s total share is ` 369, find the shares of B and C.
Example 28 :
Solution :
If total profit is x, then A and B start a business. A invests ` 600 more than B for 4
months and B for 5 months. A’s share is ` 48 more than that
1 15 9
A’s share = x of the balance x of B, out of a total profit of ` 528. Find the capital contributed
10 15 25 30 10 by each.
y
o
u
Ratio, Proportion and Partnership B-97

rs
m
Solution : = 25 × 36 + 10 × 24 + 10 × 12 : 35 × 24 : 35 × 12

a
= 25 × 3 + 10 × 2 + 10 × 1 : 35 × 2 : 35 × 1

h
528 – 48

b
B’s profit ` 240 = 75 + 20 + 10 : 70 : 35

o
2

o
= 105 : 70 : 35, i.e. 3 : 2 : 1

b
A’s profit = 528 – 240 = ` 288

.w
2

o
A 's capital 4 288 6 Rajeev’s share in the profit = 150000 =` 50000

rd
B's capital 5 240 5 6

p
re
Example 31 :

s
A 's capital 6 5 3

s
A began a business with ` 4500 and was joined afterwards

.c
B's capital 5 4 2 by B with ` 5400. If the profits at the end of year was divided

o
m
B's capital 600 3 in the ratio 2 : 1 then B joined the business after :
B's capital 2 (a) 5 month (b) 4 months
B’s capital = ` 1200 and A’s capital = ` 1800 (c) 6 months (d) 7 months
Example 29 : Solution :
Three persons A, B, C rent the grazing of a park for ` 570. A (d) Let B joined after x months.
puts in 126 oxen in the park for 3 months, B puts in 162 oxen Then, 4500 × 12 : 5400 × (12 – x) = 2 : 1
for 5 months and C puts in 216 oxen for 4 months. What part
of the rent should each person pay ? 4500 12 2
or
Solution : 5400 (12 x ) 1
Monthly equivalent rent of A = 126 × 3 = 378
Monthly equivalent rent of B = 162 × 5 = 810 45 12 2
Monthly equivalent rent of C = 216 × 4 = 864 54 12 x 1
Rent is to be divided in the ratio 378 : 810 : 864, i.e. 7 : 15 : 16
5
or 1
7 12 x
A would have to pay of the rent
7 15 16 or x = 7 months
7 7 Example 32 :
of the rent 570 = ` 105 A sum of ` 3115 is divided among A, B and C so that if ` 25,
38 38
` 28 and ` 52 be diminished from their shares respectively,
15 15 the remainders shall be in the ratio 8 : 15 : 20. Find the share
B would have to pay of the rent 570
38 38 of each.
= ` 225 Solution :
(A’s share – 25) : (B’s share – 28) : (C’s share – 52) = 8 : 15 :
16 8 20.
and C would have to pay , i.e. of the rent
38 19
A’s share 25 B’s share 28
=
8 8 15
570 ` 240
19 C’s share 52
Example 30 : = k (say)
20
Shekhar started a business investing ` 25,000 in 2010. In A’s share – 25 = 8k
2011, he invested an additional amount of ` 10,000 and A’s share = 8k + 25
Rajeev joined him with an amount of ` 35,000. In 2013, Similarly, B’s share = 15k + 28 and C’s share = 20k + 52
Shekhar invested another additional amount of ` 10,000 8k + 25 + 15k + 28k + 20k + 52 = 3115
and Jatin joined them with an amount of ` 35,000. What will k = 70
be Rajeev’s share in the profit of ` 1,50,000 earned at the A gets ` 585, B gets ` 1078 and C gets ` 1452.
end of 3 years from the start of the business in 2010? Example 33 :
(a) ` 45,000 (b) ` 50,000 A, B and C enter into a partnership. A advances ` 1200 for 4
(c) ` 70,000 (d) ` 75,000 months, B gives ` 1400 for 8 months and C ` 1000 for 10
months. They gain ` 585 altogether. Find the share of profit
Solution : each.
(b) Ratio of Shekhar, Rajeev and Jatin’s investments Solution :
= 25000 × 36 + 10000 × 24 + 10000 × Monthly Equivalent Investment of A = 1200 × 4 = 4800
12 : 35000 × 24 : 35000 × 12, Monthly Equivalent Investment of B = 1400 × 8 = 11200
y
o
u
B-98 Ratio, Proportion and Partnership

rs
m
Monthly Equivalent Investment of C = 1000 × 10 = 1000 Solution :

a
Profit is divided in the ratio 48 : 112 : 100, i.e., 12 : 28 : 25 If the total profit is 100x, each gets 30x as equal distribution.

h
b
Balance profit of 40x is divided in the ratio of capital
12

o
A’s share of profit is × 585 = ` 108 = 1250 : 850 = 25 : 17

o
65

b
.w
25 17
28 One partner gets × 40x and the other gets × 40x

o
B’s share of profit is × 585 = ` 252 42 42

rd
65

p
25 17

re
25 This difference × 40x – × 40x = 320,
42 42

s
C’s share of profit is × 585 = ` 225

s
65

.c
8

o
Example 34 : i.e., × 40x = 320

m
Three man A, B and C subscribe ` 4700 for a business. A 42
subscribes ` 700 more than B and B ` 500 more than C. How x = 42 Total profit = ` 4200.
much will each receive out of a profit of ` 423 ? Example 36 :
Solution :
Divide ` 581 among A, B and C such that four times A’s
If C subscribes ` x, then,
share is equal to 5 times B’s share which is equal to seven
B subscribes ` (x + 500) and A subscribes ` (x + 1200)
3x + 1700 = 4700; x = ` 1000 times C’s share.
Ratio of profits of C, B and A = 1000 : 1500 : 2200 Solution :
i.e. 10 : 15 : 22 4 times A’s share = 5 times B’s share = 7 times C’s share

10 A 's share B 's share C 's share


C’s share of profit = × 423 = ` 99 = =
47 35 28 20
.... (dividing by LCM of 4, 5 and 7, i.e., 140)
15 A : B : C = 35 : 28 : 20
B’s share of profit = × 423 = ` 135
47
35
22 Share of A = × 581 = ` 245
A’s share of profit = × 423 = ` 198 35 28 20
47
Example 35 : 28
Share of B = × 581 = ` 196
Two partners invested ` 1250 and ` 850 respectively in a 83
business, they decided to distribute equally 60% of the
20
profit, and the remaining as the interest on their capital. If Share of C = × 581 = ` 140
one receives ` 320 more than the other, find the total profit. 83
y
o
u
Ratio, Proportion and Partnership B-99

rs
m
a
h
b
o
o
b
.w
1. 465 coins consists of 1 rupee, 50 paise and 25 paise coins. 12. The ratio of number of ladies to gents at a party was 1 : 2,

o
rd
Their values are in the ratio 5: 3 : 1. The number of each type but when 2 ladies and 2 gents left, the ratio became 1 : 3.

p
of coins respectively is How many people were originally present at the party?

re
(a) 155, 186, 124 (b) 154, 187, 124 (a) 6 (b) 9

s
s
(c) 154, 185, 126 (d) 150, 140, 175 (c) 12 (d) None of these

.c
2. 110. If x : y ::2 : 3 and 2 : x :: 4 : 8 the value of y is 13. A man divides his property so that his son’s share to his

o
m
(a) 6 (b) 8 wife’s and the wife’s share to his daughter are both in the
(c) 4 (d) 12 ratio 3 : 1. If the daughter gets ` 10,000 less than the son,
1 2 3 find the total worth of the property.
3. If ` 782 be divided into three part, proportional to : : , (a) ` 16,200 (b) ` 16,250
2 3 4
(c) ` 16,500 (d) None of these
then the first part is :
14. A, B and C are partners. A receives 9/10 of the profit and B
(a) ` 182 (b) ` 190
and C share the remaining profit equally. A's income is
(c) ` 196 (d) ` 204
4. The dimensions of a rectangular room when increased by increased by ` 270 when the profit rises from 12 to 15%.
4 metres are in the ratio of 4 : 3 and when decreased by Find the capital invested by B and C each
4 metres, are in the ratio of 2 : 1. The dimensions of the (a) ` 5000 (b) ` 1000
room are (c) ` 500 (d) ` 1500
(a) 6 m and 4 m (b) 12 m and 8 m 15. The salaries of A,B,C are in the ratio 2 : 3 : 5. If the increments
(c) 16 m and 12 m (d) 24 m and 16 m of 15%, 10% and 20% are allowed respectively in their
5. What is the least integer which when added to both terms salaries, then what will be the new ratio of their salaries?
of the ratio 5 : 9 will make a ratio greater than 7 : 10? (a) 3 : 3 : 10 (b) 10 : 11 : 20
(a) 6 (b) 8 (c) 23 : 33 : 60 (d) Cannot be determined
(c) 5 (d) 7 16. In an express train, the passengers travelling in A.C. sleeper
6. Two numbers are such as that square of one is 224 less class, First class and Sleeper class are in the ratio 1:2:7, and
than 8 times the square of the other. If the numbers are rate for each class is in the ratio 5 : 4 : 2. If the total income
in the ratio of 3 : 4, they are from this train is ` 54, 000, find the income of Indian Railways
(a) 12, 16 (b) 6, 8 from A.C. sleeper class.
(c) 9, 12 (d) None of these (a) ` 12,000 (b) ` 20,000
(c) ` 22,000 (d) ` 10,000
3
7. Given that 24 carat gold is pure gold, 18 carat gold is gold 1
4 17. A, B and C started a business. A invests capital for
2
5
and 20 carat gold is gold, the ratio of the pure gold in 18 1 1 1
6 time, B invests capital for time and C invests the
4 8 2
carat gold to the pure gold in 20 carat gold is :
(a) 3 : 8 (b) 9 : 10 (c) 15 : 24 (d) 8 : 5 remaining capital for whole time. Find the share of B in
8. A bag contains ` 216 in the form of one rupee, 50 paise and the total profit of ` 9900.
25 paise coins in the ratio of 2 : 3 : 4. The number of 50 paise (a) ` 2200 (b) ` 1100
coins is : (c) ` 6600 (d) ` 4400
(a) 96 (b) 144 (c) 114 (d) 141 a 2 b2
18. If a : b = c : d then the value of is
A B C c2 d2
9. If A : B : C = 2 : 3 : 4, then : : is equal to :
B C A
a b
(a) 4 : 9 : 16 (b) 8 : 9 : 12 (a) 1/2 (b)
c d
(c) 8 : 9 : 16 (d) 8 : 9 : 24
10. Tea worth ` 126 per kg and ` 135 per kg are mixed with a b ab
a third variety in the ratio 1 : 1 : 2. If the mixture is worth (c) (d)
c d cd
` 153 per kg, then the price of the third variety per kg is
1" 7"
(a) ` 169.50 (b) ` 170 19. A photograph measuring 2 1 is to be enlarged so
(c) ` 175.50 (d) ` 180 2 8
11. The sum of three numbers is 98. If the ratio of the first to the that the length will be 4". How many inches will the enlarged
second is 2 : 3 and that of the second to the third is 5 : 8, breadth be?
then the second number is : 1 1 3
(a) 1 (b) 2 (c) 3 (d) 3
(a) 20 (b) 30 (c) 38 (d) 48 2 8 8
y
o
u
B-100 Ratio, Proportion and Partnership

rs
m
20. The ratio of the number of boys and girls in a college is 7: 8. in the fourth container is:

a
If the percentage increase in the number of boys and girls (a) 4 : 1 (b) 151 : 48

h
be 20% and 10 % respectively, what will be the new ratio?

b
(c) 157 : 53 (d) 5 : 2

o
(a) 8 : 9 (b) 17 : 18 32. Two casks of 48 L and 42 L are filled with mixtures of wine

o
b
(c) 21 : 22 (d) Cannot be determined and water, the proportions in the two casks being

.w
21. In a mixture of 45 litres, the ratio of milk and water is 4 : 1. respectively 13 : 7 and 18 : 17. If the contents of the two

o
How much water must be added to make the mixture ratio

rd
casks be mixed and 20 L of water is added to the whole,
3:2?

p
what will be the proportion of wine to water in the resultant

re
(a) 72 litres (b) 24 litres solution?

s
(c) 15 litres (d) 1.5 litres

s
(a) 21 : 31 (b) 12 : 13

.c
22. If a : b = 2 : 3, b : c = 3 : 4, c : d = 4 : 5, find a : b : c : d. (c) 13 : 12 (d) None of these

o
m
(a) 5 : 4 : 3 : 2 (b) 30 : 20 : 15 : 12 33. A sum of money is to be divided among A, B and C in the
(c) 2 : 3 : 4 : 6 (d) 2 : 3 : 4 : 5 ratio 2 : 3 : 7. If the total share of A and B together is ` 1,500
1 less than C, What is A’s share in it?
23. In what proportion must a number be divided so that of (a) ` 1,000 (b) ` 1,500
4
(c) ` 2,000 (d) Data insufficient
1 34. The Binary Ice-cream Shopper sells two flavours : Vanilla
the first part and of the second part are together equal to
3 and Chocolate. On Friday, the ratio of Vanilla cones sold to
1 Chocolate cones sold was 2 : 3. If the store had sold 4 more
of the original number? Vanilla cones, then, the ratio of Vanilla cones sold to the
2
(a) 1 : 2 (b) 5 : 4 (c) 2 : 3 (d) 4 : 5 Chocolate cones sold would have been 3 : 4. How many
24. Divide ` 671 among A, B, C such that if their shares be Vanilla cones did the store sell on Friday ?
increased by ` 3, ` 7 and ` 9 respectively, the remainder (a) 32 (b) 35 (c) 42 (d) 48
shall be in the ratio 1 : 2 : 3. y y x x
(a) ` 112, ` 223, ` 336 (b) ` 114, ` 221, ` 336 35. If , then find x : y : z.
x z z y
(c) ` 112, ` 227, ` 332 (d) ` 114, ` 223, ` 334
25. If ` 1066 is divided among A, B, C and D such that (a) 1 : 2 : 3 (b) 3 : 2 : 1
A : B = 3 : 4, B : C = 5 : 6 and C : D = 7 : 5, who will get the (c) 4 : 2 : 3 (d) 2 : 4 : 7
maximum? 36. At a start of a seminar, the ratio of the number of male
(a) B (b) A participants to the number of female participants was 3 : 1.
(c) C (d) D During the tea break 16 participants left and 6 more female
26. Zinc and copper are melted together in the ratio 9 : 11. What participants registered. The ratio of the male to the female
is the weight of melted mixture, if 28.8 kg of zinc has been participants now became 2 : 1. What was the total number
consumed in it? of participants at the start of the seminar ?
(a) 58 kg (b) 60 kg (c) 64 kg (d) 70 kg (a) 64 (b) 48 (c) 54 (d) 72
27. If a/b = 1/3, b/c = 2, c/d = 1/2, d/e = 3 and e/f = 1/4, then what 37. A contractor employed 25 labourers on a job. He was paid
is the value of abc/def ? ` 275 for the work. After retaining 20% of this sum, he
(a) 3/8 (b) 27/8 distributed the remaining amount amongst the labourers. If
(c) 3/4 (d) 27/4 the number of male to female labourers was in the ratio 2 : 3
28. The income of A and B are in the ratio 3 : 2 and expenses are and their wages in the ratio 5 : 4, what wages did a female
in the ratio 5 : 3. If both save ` 200, what is the income of A? labourer get ?
(a) ` 1000 (b) ` 1200 (a) ` 15 (b) ` 8 (c) ` 14 (d) ` 10
(c) ` 1500 (d) ` 1800 38. A man ordered 4 pairs of black socks and some pairs of
29. A sum of money is divided among A, B and C in the ratio of brown socks. The price of a black pair is double that of a
3 brown pair. While preparing the bill, the clerk interchanged
3 : 4 : 5. 5. If the lowest share is ` 30, then the total amount the number of black and brown pairs by mistake which
4
of money is increased the bill by 50%. The ratio of the number of black
(a) ` 212 (b) ` 106 (c) ` 53 (d) ` 159 and brown pairs of sock in the original order was :
30. A and B are two alloys of gold and copper prepared by (a) 4 : 1 (b) 2 : 1 (c) 1 : 4 (d) 1 : 2
mixing metals in the ratio 7 : 2 and 7 : 11 respectively. If equal 39. A certain number of persons can dig a trench 100 m long,
quantities of the alloys are melted to form a third alloy C, 50 m broad and 10 m deep in 10 days. The same number of
the ratio of gold and copper in C will be: persons can dig another trench 20 m broad and 15 m deep in
(a) 5 : 7 (b) 5 : 9 (c) 7 : 5 (d) 9 : 5 30 days. The length of the second trench is :
31. Three containers have their volumes in the ratio 3 : 4 : 5. (a) 400 m (b) 500 m (c) 800 m (d) 900 m
They are full of mixtures of milk and water. The mixtures 40. In a dairy farm, 40 cows eat 40 bags of husk in 40 days. In
contain milk and water in the ratio of (4: 1), ( 3 : 1) and (5 : 2) how many days one cow will eat one bag of husk ?
respectively. The contents of all these three containers are 1
poured into a fourth container. The ratio of milk and water (a) 1 (b) (c) 40 (d) 80
40
y
o
u
Ratio, Proportion and Partnership B-101

rs
m
41. The resistance of a wire is proportional to its length and entitled to half the total profit. When did B join as a partner ?

a
inversely proportional to the square of its radius. Two wires (a) 10 months after A (b) 6 months after A

h
b
of the same material have the same resistance and their radii (c) 1 months after A (d) 8 months after A

o
o
are in the ratio 9 : 8. If the length of the first wire is 162 cms., 46. A and B enter into a partnership. A puts in ` 2000 but at the

b
.w
end of 3 months, withdraws ` 500 and again at the end of 8
find the length of the other.
months withdraws ` 300. Out of a total profit of ` 900 at the

o
(a) 64 cm. (b) 120 cm.

rd
end of the year, B’s share was ` 400. Find B’s capital.

p
(c) 128 cm. (d) 132 cm. (a) ` 1000 (b) ` 1220

re
42. The prize money of ` 1,800 is divided among 3 students A,

s
(c) ` 1340 (d) ` 1500

s
.c
B and C in such a way that 4 times the share of A is equal to 47. A and B continued in a joint business for 36 months. A

o
6 times the share of B, which is equal to 3 times the share of contributes ` 300 for a certain time and B invests ` 500 for

m
C. Then A’s share is the remaining period. If out of a total profit of ` 1,020 A gets
(a) ` 400 (b) ` 600 ` 495 for how long did B keep his money.
(c) ` 700 (d) ` 800 (a) 1 year (b) 14 months
(c) 15 months (d) 18 months
1 1 1
43. Divide 81 into three parts so that of 1st, of 2nd and 48. A, B and C start a business by investing ` 2000, ` 3000 and
2 3 4
` 4000 respectively. But B increases his investment to `
of 3rd are equal. 4000 after 4 months and C withdraws ` 1000 at the end of 9
(a) 36, 27, 18 (b) 27, 18, 36 months. What is A’s share out of a total profit of ` 8475
(c) 18, 27, 36 (d) 30, 27, 24 earned in a year ?
44. A, B and C entered into partnership, and provided capitals (a) ` 1800 (b) ` 1600
of ` 22,000, ` 26,000 and ` 34,000 respectively. Some months (c) ` 1500 (d) ` 1700
later ` 10,000 extra capital being needed, it was supplied by 49. A, B and C enter into a partnership with their capitals in the
B. At the end of 12 months the total profit was ` 50,274, and
A’s share was ` 12,474. When did B supply the extra capital ? 7 4 6
: : . After 4 months, A increases his share 50%. If the
(a) After 6 months (b) After 5 months 2 3 5
(c) After 4 moths (d) After 8 months total profit at the end of the year was ` 2,16,000, then B's
45. A started a business with a certain amount of money. After share in the profit was
a few months B became his partner, contributing three times (a) ` 22,000 (b) ` 24,000
what A had contributed. At the end of the year, each was (c) ` 30,000 (d) ` 40,000

AN SW ER K EY
1 (a) 8 (b ) 15 (c) 22 (d) 29 (b) 36 (a) 43 (c)
2 (a) 9 (d ) 16 (d ) 23 (a) 30 (c) 37 (b) 44 (c)
3 (d) 10 (c) 17 (b ) 24 (a) 31 (c) 38 (c) 45 (b )
4 (b) 11 (b ) 18 (d ) 25 (c) 32 (b) 39 (b) 46 (b)
5 (c) 12 (c) 19 (c) 26 (c) 33 (b) 40 (c) 47 (b)
6 (b) 13 (b ) 20 (c) 27 (a) 34 (a) 41 (c) 48 (a)
7 (b) 14 (c) 21 (c) 28 (b) 35 (c) 42 (b) 49 (d )
y
o
u
B-102 Ratio, Proportion and Partnership

rs
m
a
h
b
o
o
b
.w
o
1. (a) The ratio of number of coins = 5 : 6 : 4 According to question,

rd
465 3x 4x 8x 6x 4x

p
The number of one rupee coins 5 155 2x 216 216

re
5 6 4 2 4 4

s
465 x = 48

s
The number of 50 paise coins 6 186

.c
5 6 4 Number of 50 paise coins = 48 × 3 = 144

o
m
465 9. (d) Let A = 2x, B = 3x and C = 4x. Then,
The number of 25 paise coins 4 124
5 6 4 A 2x 2 B 3x 3 C 4x 2
x 2 , and
2 4 B 3x 3 C 4x 4 A 2x 1
2. (a) ;
y 3 x 8 A B C 2 3 2
: : : : 8 : 9 : 24.
3 3 B C A 3 4 1
y x 4 6
2 2 10. (c) Let the third type of tea is priced at ` x per kg. Also
1 2 3 suppose that the three types of tea mixed together
3. (d) Given the ratio = : : 6 : 8 : 9.
2 3 4 are l, l and 2 kg,respectively.
6 126 1 135 1 2x
1st part = ` 782 = ` 204. Now, 153
23 1 1 2
4. (b) Let the length and breadth of the rectangular room 261 2 x
be and b. 153 261 + 2x = 612
4
4 4
We have, 351
b 4 3 x ` 175.5 per kg.
2
3 + 12 = 4b + 16
11. (b) A : B = 2 : 3 = 2 × 5 : 3 × 5 = 10 : 15
3 – 4b = 4 ...(1) and B : C = 5 : 8 = 5 × 3 : 8 × 3 = 15 : 24
4 2 Therefore, A : B : C = 10 : 15 : 24
Again, we have 4 2b 8
b 4 1 Let the numbers be 10x, 15x and 24x.
2b 4 ...(2) Then, 10x + 15x + 24x = 98
Solving (1) and (2), we get = 12 and b = 8. or 49x = 98 or x = 2
Second number = 15x = 15 × 2 = 30
5 x 7
5. (c) If x is the integer, 12. (c) Let number of ladies = x
9 x 10 and, number of gents = 2x
50 + 10x > 63 + 7x
3x > 13 x 2 1
Now, 3x 6 2x 2
2x 2 3
13
x x=4
3 Total number of people originally present
13 = 4 + 8 = 12
The least integer greater than is 5.
3 Short-cut Method
6. (b) Given, ratio of numbers is 3 : 4 1 1
The numbers are 3x and 4x. As,
2 3
Now, according to the question
2 1 2 1 3
16 x 2 8(3 x) 2 224 Total number of peoples = 12
1 3 2 1
56 x2
2 2
16 x 72 x 224 224 13. (b) Let Son’s share = ` S;
x=2 Daughter’s share = ` D;
Required numbers = 6, 8 and Wife’s share = ` W.
7. (b) 18 carat gold Also, S : W = W : D = 3 : 1
3 3 S:W: D =9:3 :1
= pure gold = 24 18 carat gold then S = 9x , D = x
4 4
and 9x – x = 10,000 x = ` 1250
5 5 Total worth of the property = (9 + 3 + 1) x = 13x
20 carat gold = pure gold = 24 20 carot gold
6 6 = 13 × 1250 = ` 16,250
Required ratio = 18 : 20 = 9 : 10 14. (c) Let the profit = x
8. (b) Let the no. of one rupee, 50 paise and 25 paise coins be 9x x
2x, 3x and 4x respectively. Profit of A , Remaining profit
10 10
y
o
u
Ratio, Proportion and Partnership B-103

rs
m
x 42x 44x
i.e. and

a
Profit of B 5 5

h
20

b
42x 44x

o
x Required ratio = : 21: 22.

o
Profit of C 5 5

b
20

.w
4
21. (c) Quantity of milk = 45 = 36 litres

o
9 1 1 5
Ratio of profit

rd
: :
10 20 20

p
1

re
= 18 : 1 : 1 Quantity of water = 45 = 9 litres
5

s
A’s income is increased by ` 270 . When profit rises 3%

s
Let x litres of water be added.

.c
270

o
Investment of A 100 ` 9000. 36 3

m
3 Then,
If investment of A, B and C = 18x, x and x 9 x 2
18x = 9000 72 = 27 + 3x or 3x = 45
x = 500 or x = 15 litres
B investment = ` 500. 22. (d) Obviously the ratio is 2 : 3 : 4 : 5
C investment = ` 500. 23. (a) Let number be divided in ratio x : y. Then
15. (c) Let A = 2k, B = 3k and C = 5k.
x y
115 115 23 First part , second part
A’s new salary of 2k 2k k x y x y
100 100 10
110 110 33 1 x 1 y 1
B’s new salary of 3k 3k k Now,
100 100 10 4 x y 3 x y 2
24. (a) Let A’s share be ` x,
120 120 B’s share be ` y. Then,
C’s new salary of 5k 5k 6k
100 100 C’s share = ` [671 – (x + y)]
23k 33k Now, x 3 : y 7 : 671 (x y) 9 1: 2 : 3
New ratio : : 6k 23: 33: 60.
10 10 x 3 : y 7 : 680 (x y) 1: 2 : 3
16. (d) Let number of passengers = x, 2x, 7x
and Rate = 5y, 4y, 2y 1
x 3 690 115
Now, since income = Rate × Number of passengers 6
Income of passengers = 5xy, 8xy, 14 xy x = ` 112
Income in ratio = 5 : 8 : 14 2
Also y 7 690 230
5 6
Income from A.C. sleeper class = 54, 000
5 8 14 y ` 223
= ` 10, 000 C’s share Rs[671 (112 223)] Rs336
1 1 5 3 25. (c) Since A : B = 3 : 4 …(1)
17. (b) C's capital = 1 1
2 8 8 8 B: C= 5 :6 …(2)
Ratio of capitals of A, B and C and C : D = 7 : 5 …(3)
Therefore, by, proportionating, (1) and (2)
1 1 1 1 3 A : B = 3 × 5 : 4 × 5 = 15 : 20
= : : 1
2 4 8 2 8 B : C = 20 : 24 and C : D = 7 : 5
1 1 3 Hence, A : B : C = 15 : 20 : 24 …(4)
= : : =2:1:6 Now, A : B : C = 15 × 7 : 20 × 7 : 24 × 7
8 16 8
= 105 : 140 : 168
1 C : D = 24 × 7 : 24 × 5 = 168 : 120
B's share = ` 9900 = ` 1100
9 [By proportionating (3) and (4)]
18. (d) 1 : 2 = 3 : 6, so (a2 + b2)/(c2 + d2) = 5/45 = 1/9 Hence, A : B : C : D = 105 : 140 : 168 : 120
From the given options, only ab/cd gives us this value. Hence, C gets the maximum share.
19. (c) Let enlarged breadth be x inches. Then, 26. (c) For 9 kg zinc, mixture melted = (9 + 11) kg.
For 28.8 kg zinc, mixture melted
5 15
: 4 :: : x
2 8 20
= 28.8 kg 64 kg.
5 15 9
x 4 x 3 inches
2 8 27. (a) a : b : c = 2 : 6 : 3
20. (c) Originally, let the number of boys and girls in the college a : b : c : d : e : f = 6 : 18 : 9 : 18 : 6 : 24
be 7x and 8x respectively. abc/def = 3/8
Their increased number is (120% of 7x) and (110% of 8x) 28. (b) Let income of A = ` 3x, income of B = ` 2x
120 110 and expenditure of A = ` 5y,
i.e. 7x and 8x expenditure of B = ` 3y
100 100
y
o
u
B-104 Ratio, Proportion and Partnership

rs
Total quantity of wine = 52.8 L

m
Now, saving = income – expenditure

a
3x – 5y = 2x – 3y = 200 Quantity of water = 57.2 L

h
x = 2y and y = 200

b
52.8 528 12

o
x = 400 Ratio after mixing =

o
A’s income = ` 1200 57.2 572 13

b
.w
= 12 : 13.
15
29. (b) Let A’s share ` x , B’s share = ` 4x and 33. (b) Let A’s share = ` 2x, B’s share = ` 3x and C’s share

o
4

rd
= ` 7x
C’s share = ` 5.5x

p
Now, 7x (2x 3x) 1500 x 750

re
15
Given x 30 x 8 A’s share = ` 2x = ` 1500

s
s
4

.c
Total amount = 30 + 32 + 44 = ` 106 V 2 V 4 3

o
34. (a) and …(i)

m
7 7 7 C 3 C 4
30. (c) Gold in C = units units.
9 18 6 3V V 4 3
C= [From (i)]
2 3V / 2 4
2 11 5
Copper in C = units units. where V denoted for vanilla and C for chocolate.
9 18 6
9V
7 5 4V + 16 = 8V + 32 = 9V V = 32
Gold : Copper = : 7 : 5. 2
6 6
y y x
31. (c) Let the three containers contain 3x, 4x and 5x litres of 35. (c) We have,
mixtures, respectively. x z z
4 12x yz xy x2 yz xz ....(1)
Milk in 1st mix. = 3x litres litres.
5 5 x y
Also, x 2 xz y2 ...(2)
12x 3x y x z
Water in 1st mix. = 3x litres litres.
5 5 From (1) and (2), we have
3 yz xy yz y2
Milk in 2nd mix. = 4x litres 3x litres.
4 2 yz xy y 2
Water in 2nd mix. = (4x – 3x) litres = x litres. 2z = x + y ...(3)
5 25x Checking with the options, we find that the values
Milk in 3rd mix. = 5x litres litres. given in option c satisfies the equation (3)
7 7
36. (a) Let the number of male and female participants be 3x
25x 10x and x respectively.
Water in 3rd mix. = 5x litres litres.
7 7 Therefore total no. of participants are 4x.
Total milk in final mix. During the tea break, the number of male participants are
3
12x
3x
25x
litres
314x
litres. (4x 16) 3x 12 ....... (i)
= 4
5 7 35
and the number of female participants are
Total water in final mix.
1
3x 10x 106x (4x 16) 6 x 2 ....... (ii)
= x litres litres. 4
5 7 35
Required ratio of milk and water 3x 12 2
Now,
314x 106x x 2 1
= : 157 : 53.
35 35 3x 12 2x 4 x 16.
32. (b) In first cask, Therefore, the total number of participants are
= 4 × 16 = 64.
7
Quantity of water = 48 16.8L 2
20 37. (b) Number of males = 25 10
5
13
Quantity of wine = 48 31.2L 3
20 Number of females = 25 15
In second cask, 5
Amount distributed among males and females
17
Quantity of water = 42 20.6L = 275 × 80% = ` 220
35 Let the wage paid to a male be ` 5x and that to a female
18 be ` 4x. Therefore,
Quantity of wine = 42 21.6L 10 × 5x + 15 × 4x = 220 50x + 60x = 220 x = 2
35
Now after mixing: Wage received by a feamale labourer = 2 × 4 = ` 8
y
o
u
Ratio, Proportion and Partnership B-105

rs
38. (c) Let x pairs of brown socks were ordered.

m
x = 2k, y = 3k, = 4k

a
Let P be the price of a brown pair. According to question

h
Therefore, price of the black pair of sock = 2P x + y + z = 81

b
o
Now, 4P + 2Px = 1.5 (Px + 8P) 2k + 3k + 4k = 81 9k = 81 k = 9

o
b
Hence, parts are 18, 27, 36.

.w
3
4P 2Px (Px 8P) 8P 4Px 3Px 24P 44. (c) A’s total capital in partnership is 22000 × 12 = 264000.

o
2
B’s total is 26000 × 12 = 312000

rd
Px = 16P x = 16

p
C’s total is 34000 × 12 = 408000

re
4 Let B invested ` 10000 for x months then this amount

s
Required ratio = 1: 4

s
16 will be 10000x.

.c
Total amount is 264000 + 312000 + 408000 + 10000x

o
39. (b) Let the required length be x metres.

m
More breadth, Less length (Indirect Proportion) = 984000 + 10000x
More depth, Less length (Indirect Proportion)
264000 12474
More days, More length (Direct Proportion) Then, x 8
984000 10000x 50274
Breadth 20:50 So B supply the extra capital after 4 months.
Depth 15:10 : : 100 : x 45. (d) If A’s share of capital was ` x, B’s share was ` 3x. A’s
Days 10:30 capital was in the business for 12 months, and let us
20 15 10 x 50 10 30 100
assume that B’s capital was in it for n months.
x × 12 = 3x (n)
50 10 30 100
x x 500. n=4
20 15 10 B joined 8 months after A started.
40. (c) Let the required number of days be x.
46. (b) Ratio of profits = (A’s ` 2000 for 3 months) + (A’s `
Less cows, More days (Indirect Proportion)
Less bags, Less days (Direct Proportion) 1500 for 5 months) + (A’s ` 1200 for 4 months) : (B’s
capital x for 12 months)
Cows 1:40
:: 40 : x 18300 500
Bags 40:1 = (6000 + 7500 + 4800) : 12x
12x 400
1 40 x 40 1 40 x 40.
41. (c) If R is the resistance, l is the length and r is the radius. 1525 5
x = ` 1220
x 4
R So B’s capital = ` 1220
r2
47. (b) Let A contributes for x months than B contributes for
k (36 – x) months.
R= 2 (where k is a constant)
r
x 300 3x
Ratio of A’s part to B’s part =
k 162 (36 x) 500 180 5x
R1 81
= ; But R1 = R2. Then part of A in the profit of
R2 k
64 3x
` 1020 = 1020 × 495
3x 180 5x
k 162 162
=k× =
81 64 81 64 1020 3x
495 3060x 495 (180 2x)
= 128 cms. 180 2x
3060x = 89100 – 990x
42. (b) 4A = 6B 2A = 3B A : B = 3 : 2
4050x = 89100
B = 3C 2 B = C B : C = 1 : 2
A : B : C 89100
x 22
3 : 2 4050
1 : 2 So, B contributes for (36 – 22) = 14 months
3 : 2 : 4 48. (a) A’s Monthly Equivalent Investment = (2000 × 12)
B’s Monthly Equivalent Investment = (3000 × 4 + 4000 × 8)
3 3
A’s share 1800 1800 600 C’s Monthly Equivalent Investment = (4000 × 9 + 3000 × 3)
(3 2 4) 9 Profit sharing ratio = 24000 : 44000 : 45000 = 24 : 44 : 45
43. (c) Let lst, 2nd and 3rd part represented by x, y, z
24
1 1 1 A’s share = × 8475 = 24 × 75 = ` 1800.
Let x y z k 113
2 3 4
y
o
u
B-106 Mixture and Alligations

rs
m
a
CHAPTER
MIXTURE AND

h
b
9

o
o
b
.w
ALLIGATIONS

o
rd
p
re
s
s
.c
Simple Mixture : When two different ingredients are mixed Example 2 :

o
together, it is known as a simple mixture. A mixture of a certain quantity of milk with 16 litres of water

m
Compound Mixture : When two or more simple mixtures are is worth 90 P per litre. If pure milk be worth ` 1.08 per litre,
mixed together to form another mixture, it is known as a how much milk is there in the mixture ?
compound mixture. Solution :
Alligation : Alligation is nothing but a faster technique of
The mean value is 90P and the price of water is 0 P.
solving problems based on the weighted average situation as
Milk Water
applied to the case of two groups being mixed together. 0
108
The word ‘Alligation’ literally means ‘linking’.
Alligation rule : It states that when different quantities
of the same or different ingredients of different costs are 90
mixed together to produce a mixture of a mean cost, the
ratio of their quantities is inversely proportional to the
difference in their cost from the mean cost. 90 0 108 90
Quantity of Cheaper Price of Dearer Mean Price By the Alligation Rule, milk and water are in the ratio of 5 : 1.
Quantity of Dearer Mean Price Price of Cheaper Quantity of milk in the mixture = 5 × 16 = 80 litres.
Graphical representation of Alligation Rule :
Quantity Quantity Price of the Mixture :
a b When quantities Qi of ingredients Mi’s with the cost Ci’s
Mean are mixed then cost of the mixture Cm is given by
Average (d)
Ci Q i
Cm
Qi
b–d d–a Example 3 :
Quantity of a b d 5 kg of rice of ` 6 per kg is mixed with 4 kg of rice to get a
mixture costing ` 7 ker kg. Find the price of the costlier rice.
Quantity of b d a
Solution :
Applications of Alligation Rule :
Let the price of the costlier rice be ` x.
(i) To find the mean value of a mixture when the prices of two
or more ingredients, which are mixed together and the By direct formula,
proportion in which they are mixed are given. 6 5 4 x
(ii) To find the proportion in which the ingredients at given 7
9
prices must be mixed to produce a mixture at a given price.
Example 1 : 63 – 30 = 4x 4x = 33
In what proportion must sugar at ` 13.40 per kg be mixed
with sugar at ` 13.65 per kg, so that the mixture be worth ` 33
x= 8.25
13.20 a kg ? 4
Solution : Straight line approach of Alligation
C.P. of 1 kg C.P. of 1 kg Let Q1 and Q2 be the two quantities, and n 1 and n2 are the num-
Cheaper sugar Dearer sugar ber of elements present in the two quantities respectively,
(1340 paise) (1365 paise) Q1 Q2
Av
Average price
(1320 paise)
n1 n2
45 20 where Av is the average of the new group formed then n 1 corre-
Quantity of cheaper sugar 45 9 sponds to Q 2 – Av, n 2 corresponds to Av – Q 1 and
(n1 + n2) corresponds to Q2 – Q1.
Quantity of dearer sugar 20 4
Let us consider the previous example.
They must be mixed in the ratio 9 : 4.
y
o
u
Mixture and Alligations B-107

rs
Example 4 : And,

m
a
5 kg of rice at ` 6 per kg is mixed with 4 kg of rice to get a

h
qA
mixture costing ` 7 per kg. Find the price of the costlier rice. Quantity of A in resultant mixture

b
(M N)

o
Solution : qA q B

o
b
Using straight line method,

.w
qB
6 Quantity of B in resultant mixture (M N)
x

o
qA q B

rd
7
(ii) When qA and qB are known and M and N have to be found

p
re
out

s
5 4

s
.c
4 corresponds to 7 – 6 and 5 corresponds to x – 7. x qA

o
i.e. 4 1

m
Quantity of mixture 1 Q1 x y qA qB
5 1.25
Quantity of mixture 2 Q2 qA a
Hence, x – 7 = 1.25
x = 8.25 qA qB a b
Example 5 : And,
A jar contains a mixture of two liquids P and Q in the ratio Quantity of mixture 1
4 : 1. When 15 litres of the mixture is taken out and 15 litres
of liquid Q is poured into the jar, the ratio becomes 2 : 3. Q1
× Quantity of resultant mixture
How many litres of liquid P was contained in the jar. Q1 Q 2
Solution :
Quantity of mixture 2
1 1
Fraction of Q in original mixture = Q2
1 4 5 × Quantity of resultant mixture
Q1 Q 2
3 3
Fraction of Q in resulting mixture =
2 3 5 REMOVAL AND REPLACEMENT
Q in mixture Pure Q (i) Let a vessel contains Q unit of mixture of ingredients A and
1 1 B. From this, R unit of mixture is taken out and replaced by
5
an equal amount of ingredient B only.
3 If this process is repeated n times, then after n operations
5
n
2 2 Quantity of A left R
5
1
5 Quantity of A originally present Q
Thus, the original mixture and liquid Q are mixed in the same
and Quantity of B left = Q – Quantity of A Left
ratio.
(ii) Let a vessel contains Q unit of ingredient A only. From this
If 15 litres of liquid Q is added, then after taking out 15
R unit of ingredient A is taken out and replaced by an equal
litres of mixture from the jar, there should have 15 litres of
amount of ingredient B.
mixture left.
So, the quantity of mixture in the jar If this process is repeated n times, then after n operations,
= 15 + 15 = 30 litres n
R
Quantity of A left = Q 1
30 Q
and quantity of P in the jar = 4 = 24 litres.
5 Quantity of B = 1 – Quantity of A left
Alligation Rule for Compound Mixture : Remember that in
compound mixture, same mixtures i.e. mixtures of same Example 6 :
ingredients are mixed together in different proportion to A container contains 40 litres of milk. From this container, 4
make a new mixture. litres of milk was taken out and replaced by water. This
Let Mixture 1 has ingredients A and B in ratio a : b process was repeated further two times. How much milk is
and Mixture 2 has ingredients A and B in ratio x : y. now contained by the container ?
Now, M unit of mixture 1 and N unit of mixture 2 are mixed to Solution :
form compound mixture. Then, in the resultant mixture, the Milk Water
ratio of A and B is To start with 40 litres
After 1st operation 36 litres 4 litres
a x
M N 4 4
Quantity of ingredient A qA a b x y After 2nd operation 36 – ×36 4– ×4+4
(i) 40 40
Quantity of ingredient B qB b y
M N = 32.4 litres = 4 – 0.4 + 4
a b x y = 7.6 litres
y
o
u
B-108 Mixture and Alligations

rs
Solution :

m
4 4
After 3rd operation 32.4 –
×32.4 7.6 – × 7.6 + 4

a
2 3 3 1 1 1 2 1 151 89

h
40 40 : :

b
= 151 : 89
= 32.4 – 3.24 =7.6 – 0.76 + 4 3 4 5 2 3 4 5 2 60 60

o
o
= 29.16 = 10.84 Example 9 :

b
The quantity of milk in the container is 29.16 litres.

.w
The ratio of water and milk in a 30 litres mixture is 7 : 3. Find
Short-cut Method: the quantity of water to be added to the mixture in order to

o
rd
3 make this ratio 6 : 1.
4

p
Quantity of milk in container : 40 1 29.16 litres Solution :

re
40 In this example the ratio of water : milk is given and water is

s
s
Examples 7 : further added. But in the above formula ratio of A : B is

.c
A dishonest hair dresser uses a mixture having 5 parts pure given and quantity B is added. So the formula in this

o
changed scenario becomes :

m
After shave lotion and 3 parts of pure water. After taking
out some portion of the mixture, he adds equal amount of x(bc – ad)
pure water to the remaining portion of the mixture such that Quantity of B added =
d(a+b)
the amount of Aftershave lotion and water become equal.
The part of the mixture taken out is 30 (3 6 7 1) 30 (18 7)
Required quantity =
Solution : 1(7 3) 1 10
Let quantity of pure After shave lotion = 5kg 30 11
and quantity of pure water = 3 kg = 33 litres.
10
Total quantity of the mixture = 8 kg A mixture contains A and B in the ratio a : b. If x litres of B
Again let x kg of mixture is taken out of 8kg of mixture. is added to the mixture, A and B become in the ratio
5x ax
Now, the amount of Aftershave lotion left = 5 kg a : c. Then the quantity of A in the mixture is given by
8 c b
bx
3x and that of B is given by .
and the amount of water left = 3 kg c b
8 Example 10 :
The amount of water after adding x kg of water becomes A mixture contains beer and soda in the ratio of 8 : 3. On
adding 3 litres of soda, the ratio of beer to soda becomes 2
3x 5x
3 x kg 3 kg : 1 (i.e., 8 : 4). Find the quantity of beer and soda in the
8 8 mixture.
According to question, Solution :
5x 5x 8 3
5 3 Quantity of beer in the mixture = = 24 litres
8 8 4 3
3 3
10x 8 and the quantity of soda in the mixture = = 9 litres.
2 x 4 3
8 5 Example 11 :
1 Mira’s expenditure and savings in the ratio 3 : 2 .Her income
of the 8 kg mixture is taken out. increase by 10%. Her expenditure also increases by 12%. By
5
If in x litres mixture of A and B, the ratio of A and B is a : b, how many %does her saving increase ?
the quantity of B to be added in order to make the ratio c : d Solution :
Expenditure Saving
x (ad bc)
is . 12 x
c (a b) (% increase in exp) (% increase in saving)
If x glasses of equal size are filled with a mixture of milk and
water. The ratio of milk and water in each glass are as follows:
a1 : b1, a2 : b2, a3 : b3...ax : bx
10
If the content of all the x glasses are emptied into a single (% increase in income)
large vessel, then proportion of milk and water in it is given
a1 a2 ax
by ... :
a1 b1 a2 b2 ax bx 3 2
(given)
b1 b2 bx We get two values of x, 7 and 13 .But to get a viable answer,
... we must keep in mind the central value (10) must lie between
a1 b1 a2 b2 ax bx
x and 12. Thus the value of x should be 7 and not 13.
Example 8 : required % increase = 7%
In four vessels each of 20 litres capacity mixture of milk and Example 12 :
water is filled. The ratio of milk and water are 2:1, 3:1, 3:2 and A vessel of 80 litre is filled with milk and water. 70% of milk
1:1 in the four respective vessels. If all the four vessels are and 30% of water is taken out of the vessel . It is found that
emptied into a single large vessel, find the proportion of the vessel is vacated by 55%. Find the initial quantity of
milk and water in the mixture. milk and water.
y
o
u
Mixture and Alligations B-109

rs
Solution :

m
Applying the alligation rule, we have

a
Here the % values of milk and water that is taken from the 6% 5%

h
vessel should be taken into consideration. ` 90 ` 75

b
o
milk water

o
70 % 30 %

b
.w
` 85

o
rd
p
55 %

re
` 10 `5

s
s
2 1

.c
1

o
25 % 15 % Money invested at 5% = × ` 1,500 = ` 500

m
5:3 3
Ratio of milk to water = 5 : 3 Example 15 :
Three vessels containing mixtures of milk and water are of
80
quantity of milk = × 5 = 50 litres capacities which are in the ratio 1 : 2 : 3. The ratios of milk
5 3 and water in the three vessels are 4 : 1, 3 : 2 and 2 : 3 respec-
80 tively. If one-fourth the contents of the first vessel, one-
and quantity of water = × 3 = 30 litres third of that of the second vessel and half of that of the third
5 3
Example 13 : vessel are mixed, what is the ratio of milk and water in the
Nine litres are from drawn from a case full of water and it is new mixture ?
then filled with milk .Nine litres of mixture are drawn and the Solution :
cask is again filled with milk .The quantity of water now left Part of milk in the resultant solution
in the cask is to that of the milk in it as 16 : 9. How much does 1 1 4 1 2 3 1 3 2 1
the cask hold ? = × × + × × + × × =
4 6 5 3 6 5 2 6 5 5
Solution : Part of water in the resultant solution
Let there be x litres in the cask .From the above formula we
have, after n operations : 1 1 1 1 2 2 1 3 3 73
= × × + × × + × × =
n
4 6 5 3 6 5 2 6 5 360
Water left in vessel after n operations x y 1 73
Whole quantity of milk in vessel = x Ratio of milk to water = : = 72 : 73.
5 360
2 Example 16 :
x 9 16 16
Thus in this case, = = Sea water contains 5 % salt by weight. How many kg of
x 16 9 25 fresh water must be added to 60 kg of sea water for the
x = 45 litres content of salt in solution to be made 3%.
Example 14 : Solution :
` 1500 in invested in two such part that if one invested at Let x kg of fresh water is added to sea water
16%, and the other at 5% the total interest in one year from q salt 5%of 60 3
both investments is ` 85. How much invested at 5%?
q salt q water = 60 x = 100
Solution :
If the whole money is invested at 6% the annual income is (given 3% salt in solution)
6% of ` 1,500 = ` 90. If the whole money is invested at 5%, 3 3
the annual income is 5% of ` 1,500 = ` 75. But real income = x = 40 kg.
60 x 100
= ` 85. 40 kg of fresh water must be added to sea water..
y
o
u
B-110 Mixture and Alligations

rs
m
a
h
b
o
o
b
.w
1. Gold is 19 times and copper is 9 times as heavy as water . In 10. Pure milk costs ` 3.60 per litre. A milkman adds water to 25

o
rd
what ratio must these metals be mixed so that the mixture litres of pure milk and sells the mixture at ` 3 per litre. How

p
may be 15 times as heavy as water? many litres of water does he add?

re
(a) 2 : 3 (b) 3 : 2 (a) 2 litres (b) 5 litres

s
s
(c) 1 : 3 (d) 2 : 1 (c) 7 litres (d) 11 litres

.c
o
2. Six litres of a 20% solution of alcohol in water are mixed with 11. In what ratio must water be mixed with milk to gain 20% by

m
4 litres of a 60% solution of alcohol in water. The % alcoholic selling the mixture at cost price?
strength of the mixture is (a) 1 : 3 (b) 1 : 5 (c) 1 : 7 (d) 1 : 10
(a) 80 (b) 40 12. Two liquids are mixed in the proportion of 3 : 2 and the
(c) 36 (d) 48 mixture is sold at ` 11 per kg at a 10% profit. If the first liquid
3. A merchant lent out `1,000 in two parts, one at 8% and the costs ` 2 more per kg than the second, what does it cost per
other at 10% interest. The yearly average comes out to be litre?
9.2%. Find the amount lent in two parts. (a) ` 11 (b) ` 10.50
(a) ` 400, ` 600 (b) ` 500, ` 500 (c) ` 11.50 (d) ` 10.80
(c) ` 300, ` 700 (d) cannot be determined 13. A chemist has 10 litres of a solution that is 10 per cent nitric
4. One litre of water was mixed to 3 litres of sugar. Solution acid by volume. He wants to dilute the solution to 4 per
containing 4% of sugar. What is the percentage of sugar in cent strength by adding water. How many litres of water
the solution? must he add ?
(a) 3 (b) 4 (a) 15 (b) 20 (c) 18 (d) 25
(c) 6 (d) Insuffficient data 14. How much water must be added to a cask which contains
5. How much water must be added to 60 litres of milk at 40 litres of milk at cost price ` 3.5/litres so that the cost of
1 2 milk reduces to ` 2/litre?
1 litres for ` 20 so as to have a mixture worth ` 10 a (a) 20 (b) 35
2 3
litre? (c) 45 (d) None of these
(a) 10 litres (b) 12 litres 15. The number of millilitres of water added to reduce 9 ml of
(c) 15 litres (d) 18 litres aftershave lotion, containing 50% alcohol, to a lotion
6. How many kg of salt at 42 paise per kg must a man mix with containing 30% alcohol is
25 kg of salt at 24 paise per kg so that he may, on selling the (a) 3 (b) 4 (c) 5 (d) 6
mixture at 40 paise per kg gain 25% on the outlay? 16. In my pocket there are `25 consisting of only the
(a) 15 kg (b) 18 kg (c) 20 kg (d) 24 kg denominations of 20 paise and 50 paise. Thus there are
7. A trader mixes 80 kg of tea at ` 15 per kg with 20 kg of tea at total 80 coins in my pocket. The no. of coins of the
cost price of ` 20 per kg. In order to earn a profit of 25%, denomination of 50 paise is
what should be the sale price of the mixed tea? (a) 30 (b) 70
(a) ` 23.75 (b) ` 22 (c) 50 (d) 25
(c) ` 20 (d) ` 19.20 17. There are 65 students in a class. 39 rupees are distributed
8. A company blends two varieties of tea from two different among them so that each boy gets 80 P and each girl gets
tea gardens, one variety costing ` 20 per kg and other 30 P. Find the number of boys and girls in that class.
` 25 per kg, in the ratio 5 : 4. He sells the blended tea at (a) 45, 20 (b) 40, 25
` 23 per kg. Find his profit percent : (c) 39, 26 (d) 29, 36
(a) 5% profit (b) 3.5% loss 18. The ratio of milk and water in 55 litres of adulterated milk is
(c) 3.5% profit (d) No profit, no loss 7 : 4. How much water must be added to make the mixture’s
9. Equal amounts of water were poured into two empty jars of ratio 7 : 6?
(a) 5 lt (b) 10 lt (c) 15 lt (d) 25 lt
1
different capacities, which made one jar full and the other 19. A and B are two alloys of gold and copper prepared by
4 mixing metals in the ratio 7 : 2 and 7 : 11 respectively. If equal
1 quantities of the alloys are melted to form a third alloy C,
jar full. If the water in the jar with lesser capacity is then
3 then the ratio of gold and copper in alloy C will be
poured into the jar with greater capacity, then the part of (a) 5 : 7 (b) 5 : 9 (c) 7 : 5 (d) 9 : 5
the larger jar filled with water is 20. A cane contains a mixture of two liquids A and B in the ratio
1 7 1 1 7 : 5. When 9 litres of mixture are drawn off and the cane is
(a) (b) (c) (d) filled with B, the ratio of A and B becomes 7 : 9. How many
2 12 4 3
litres of liquid A was contained by the cane initially?
(a) 10 (b) 20 (c) 21 (d) 25
y
o
u
Mixture and Alligations B-111

rs
m
strength only. How much of the butt had he stolen?
21. Given that 24 carat gold is pure gold, 18 carat gold is 3 gold

a
4 3 5 4 7

h
b
5 (a) (b) (c) (d)
8 7 7 11

o
and 20 carat gold is gold, the ratio of the pure gold in 18

o
6 32. Three containers A, B and C are having mixtures of milk and

b
carat gold to the pure gold in 20 carat gold is :

.w
water in the ratio 1 : 5, 3 : 5 and 5 : 7, respectively. If the
(a) 3 : 8 (b) 9 : 10

o
(c) 15 : 24 (d) 8 : 5 capacities of the containers are in the ratio 5 : 4 : 5, then find

rd
22. Five litres of water is added to a certain quantity of pure the ratio of the milk to the water if the mixtures of all the

p
three containers are mixed together.

re
milk costing ` 3 per litre. If by selling the mixture at the same

s
price as before, a profit of 20% is made, what is the amount (a) 51 : 115 (b) 52 : 115

s
.c
of pure milk in the mixture? (c) 53 : 115 (d) 54 : 115

o
(a) 22 litres (b) 25 litres 33. Tea worth ` 126 per kg and ` 135 per kg are mixed with a

m
(c) 27 litres (d) None of these third variety in the ratio 1 : 1 : 2. If the mixture is worth ` 153
23. A jar full of whisky contains 40% alcohol. A part of this per kg, then the price of the third variety per kg is:
whisky is replaced by another containing 19% alcohol and (a) ` 169.50 (b) `170
now the percentage of alcohol was found to be 26%. The (c) ` 175.50 (d) ` 180
quantity of whisky replaced is: 34. Several litres of acid were drawn off from a 54 litre vessel,
1 2 2 3 full of acid and and equal amount of water was added. Again
(a) (b) (c) (d) the same volume of the mixture was drawn off and replaced
3 3 5 5
24. A mixture consists of 15 parts of coffee, purchased at by water. As a result now, the vessel contained 24 litres of
` 2.10 per kg and 1 part of chicory, purchased at 98 paise per pure acid. How much of the acid was drawn off initially ?
kg. If it is sold at ` 2.25 per kg, what profit would be made on (a) 12 L (b) 16 L (c) 18 L (d) 24 L
the sale of 5 quintals? 35. An empty fuel tank of a car was filled with A type petrol.
(a) ` 100 (b) ` 105 (c) ` 110 (d) ` 115 When the tank was half-empty, it was filled with B type
25. How many litres of pure alcohol must be added to 10 litres petrol. Again when the tank was half-empty, it was filled
of mixture which is 15% alcohol to make a mixture which will with A type petrol. When the tank was half-empty again, it
be 25% alcohol? was filled with B type petrol. What is the percentage of A
5 5 3 4 type petrol at present in the tank ?
(a) (b) (c) (d) (a) 33.5% (b) 37.5% (c) 40% (d) 50%
4 2 4 3
26. How many kg of custard powder costing ` 40 kg must be 36. How many kilograms of tea powder costing ` 31 per kg be
mixed with 16 kg of custard powder costing ` 55 kg so that mixed with thirty six kilograms of tea powder costing ` 43
25% may be gained by selling the mixture at ` 60 kg? per kg, such that the mixture when sold at ` 44 per kg gives
(a) 11 kg (b) 14 kg (c) 12 kg (d) 20 kg profit of 10% ?
27. From a cask full of milk, 10 litres are taken out of 50 litres and (a) 12 kg (b) 14 kg
is filled with water. This was done twice. What is the quantity (c) 16 kg (d) 8 kg
of milk now left in the cask? 37. A mixture of Nitric acid and Sulfuric acid is taken in the ratio
(a) 20 litres (b) 32 litres of 1 : 2 and another mixture of the same is taken in the ratio
(c) 25 litres (d) 30 litres 2 : 3. How many parts of the two mixtures must be taken to
28. Two vessels A and B contain spirit and water mixed in the attain a new mixture consisting of Nitric acid and Sulfuric
ratio 5 : 2 and 7 : 6 respectively. Find the ratio in which acid in the ratio of 3 : 5 ?
these mixture be mixed to obtain a new mixture in vessel C (a) 3 : 5 (b) 5 : 3
containing spirit and water in the ratio 8 : 5 ?
(c) 2 : 3 (d) 3 : 2
(a) 4 : 3 (b) 3 : 4 (c) 5 : 6 (d) 7 : 9
38. In two alloys, the ratio of iron and copper is 4 : 3 and 6 : 1,
29. Two vessels A and B contain milk and water mixed in the respectively. If 14 kg of the first alloy and 42 kg of the
ratio 8 : 5 and 5 : 2 respectively. The ratio in which these two second alloy is mixed together to form a new alloy, then
mixtures be mixed to get a new mixture containing 69 3 % what will be the ratio of iron to copper in the new alloy ?
13 (a) 11 : 3 (b) 11 : 8
milk, is :
(a) 2 : 7 (b) 3 : 5 (c) 8 : 1 (d) None of these
(c) 5 : 2 (d) 5 : 7 39. In a mixture of 45 litres, the ratio of milk and water is 4 : 1.
How much water must be added to make the mixture ratio
1
30. Three bars of gold, weighing 6, 5, 4 and of 15, 14, 12 carats 3:2 ?
2 (a) 72 litres (b) 24 litres
fineness are mixed together. What is the fineness of the (c) 15 litres (d) 1.5 litres
compound?
40. Zinc and copper are melted together in the ratio 9 : 11. What
(a) 16 carats (b) 20 carats
is the weight of melted mixture, if 28.8 kg of zinc has been
(c) 11 carats (d) 14 carats
31. A butler stole wine from a butt of sherry which contained consumed in it ?
32% of spirit and then replaced what he stole, by wine (a) 58 kg (b) 60 kg
containing only 18% spirit. The butt was then of 24% (c) 64 kg (d) 70 kg
y
o
u
B-112 Mixture and Alligations

rs
m
ANSWER KEY

a
h
1 (b) 6 (c) 11 (b) 16 (a) 21 (b) 26 (b) 31 (c) 36 (a)

b
o
o
2 (c) 7 (c) 12 (d) 17 (c) 22 (b) 27 (b) 32 (c) 37 (a)

b
.w
3 (a) 8 (c) 13 (a) 18 (b 23 (b) 28 (d) 33 (c) 38 (a)

o
rd
4 (a) 9 (b) 14 (d) 19 (c) 24 (c) 29 (a) 34 (c) 39 (c)

p
re
5 (c) 10 (b) 15 (d) 20 (c) 25 (d) 30 (d) 35 (b) 40 (c)

s
s
.c
o
m
1. (b) By the rule of alligation, we have Quantity of money lent at 8%
Gold Copper 2
19 times 9 times 1000 ` 400
2 3
and quantity of money lent at 10%
15 times 3
1000 ` 600
2 3
15 – 9 = 6 19 – 15 = 4 4. (a) New % of sugar in (3 + 1) litre solution

Required ratio6 0.04 3


3:2 0.03 3%
4 (3 1)
2. (c) Quantity of alcohol in 6 litres of 20% solution
2 40
20 5. (c) C.P. of 1 litre of milk = ` 20 ` .
6l 1.2 3 3
100
C.P. of 1 litre C.P. of 1 litre
Quantity of alcohol in 4 litres of 60% solution of water of milk

60 40
0 Mean `
4l 2.4 price 3
100
32
`
total alc. 3
% alcoholic strength of the mixture = 100
total sol.
40 32 8 32 32
– = –0 =
1.2 2.4 3 3 3 3 3
100 36%
10 8 32
Ratio of water and milk = : = 8 : 32 = 1 : 4.
By direct formula : 3 3
6 20 4 60 Quantity of water to be added to 60 litres of milk
% alcoholic strength in mixture 36
6 4 1
= 60 litres = 15 litres.
3. (a) Here, the % value of first part and second part from 4
`1000 should be taken into consideration.
100
First Part Second Part 6. (c) Here, cost price of mixture 40 32 paise
100 25
8% 10%
q1 32 24 8 4
q2 42 32 10 5
4
9.2% and hence q1 25 20 kg
5
80 15 20 20
7. (c) C.P. of mixture ` 16
80 20
10% – 9.2% 9.2% – 8% (100 25)
Quantity lent at 8% 10 – 9.2 0.8 2 S.P. 16 ` 20
100
Quantity lent at 10% 9.2 8 1.2 3
y
o
u
Mixture and Alligations B-113

rs
8. (c) Let the quantity of two varieties of tea be 5x kg and 4x

m
13. (a) Out of 10 litre of solution, there is 1 litres of nitric acid

a
kg, respectively. and 9 litres of water.

h
Now, SP = 23 × 9x = 207x Let x litres of water be added to the solution so that the

b
o
and CP = 20 × 5x + 25 × 4x = 200x diluted solution is of 4% strength.

o
b
7x 4% of (10 + x) = 1 x = 15.

.w
Profit % = 100 3.5% 14. (d) This question can be solved in so many different ways.
200x

o
9. (a) Amounts of water in two jars are equal; the jar with the But the method of alligation method is the simplest of

rd
1 all the methods. We will apply the alligation on price of

p
re
greater capacity is full, and the Jar with lesser milk, water and mixture.
4

s
Milk Water

s
1

.c
capacity is full. 3.5 0
3

o
m
When the water in smaller jar is poured into the Mean
larger Jar, the addition of an equal amount of water will 2
2 1.5
double the amount in the larger jar, which will then be ratio of milk and water should be 2 : 15 = 4 : 3
1 1 40
2 full. added water = 3 = 30 litres
4 2 4
10. (b) C.P of 1 C.P of 1 15. (d) The given solution has 50% alcohol. Water which is to
of pure milk of water be added has 0% alcohol concentration.
`3.60 0 Alcohol concentration :

`3

3– 0 3.60 – 3
Quantity of pure milk 3 0 3 5 Water should be added in the ratio 2 : 3
Quantity of water 3.6 3 0.6 1 Quantity of water to be added 2 9 6 ml
Since in every 5 litres of milk, he adds 1 litre of water. 3
In every 25 litres of milk, he adds 5 litres of water. 16. (a) Go through options :
11. (b) Let C.P. of milk be ` 1 per litre. 30 × 50 + 50 × 20 = 2500 paise
Then, S.P. of 1 litre of mixture = ` 1. Alternatively : Since the average price of a coin
Gain = 20% 2500
31.25 paise
100 5 80
C.P. of 1 litre of mixture = ` 1 ` .
120 6 20 50
By the rule of alligation, we have :
31.25
C.P. of 1 litre C.P. of 1 litre 18.75 11.25
of water of milk
So the ratio of no. of 20 paise coins to the no. of 50
0 Mean `1 paise coins
price = 18.75 : 11.25
5 = 75 : 45 = 5 : 3
`
6 Therefore, the no. of coins of the denominations of 50
paise is 30.
5 1 5 5 17. (c) Here, alligation is applicable for ‘money per boy or
1– = –0 =
6 6 6 6 girl”.
3900
1 5 Mean value of money per student = 60 P
Ratio of water and milk = : 1: 5. 65
6 6 Boys Girls
100 80 30
12. (d) C.P. of mixture 11 ` 10
100 10 60
Let the cost of second liquid be ` x.
30 20
Then, cost of first liquid be ` (x + 2).
Boys : Girls = 3 : 2
(x 2) 3 2x 65
10 Number of boys = 3 39
5 3 2
5x + 6 = 50 x = ` 8.8 and number of girls = 65 – 39 = 26
cost of first liquid = ` (8.8 + 2) = ` 10.80
y
o
u
B-114 Mixture and Alligations

rs
(b) Here, S. P. of mixture = C. P. of pure milk = ` 3 per litre

m
18. (b) By the rule of alligation, 22.

a
water concentration,

h
100 20

b
Original solution Water Now, S. P. of mixture C.P. of mixture

o
4 1 100

o
11

b
3×100

.w
C.P. of mixture = = ` 2.5 per litre
120

o
rd
6 (mixture) Also, C. P of water = ` 0

p
13 By the rule of alligation :

re
s
s
.c
o
7 14

m
13 143
water must be added to the mixture in the ratio
14 7
: i.e. 2 : 11
143 13
Short cut Method :
55 7 6 7 4
Quantity of water added
7 7 4
2.5
Ratio of pure milk and water in mixture = 5 :1
55 14 0.5
= 10 litres
7 11 For five litres of water, quantity of pure milk
2 = 5 × 5 = 25 litres
Quantity of water to be added 55 10 litres 23. (b) By the rule of alligation, we have :
11
Strength of Strength of
19. (c) Short cut Method: first jar 2nd jar
40% 19%
7 7
Ratio of gold and copper in type C alloy :
9 18
Mean
2 11 21 15 strength
: 7 :5 26%
9 8 18 18

20. (c) Suppose the cane initially contains 7x and 5x litres of 7 14


So, ratio of 1st and 2nd quantities = 7 : 14 = 1 : 2.
mixtures A and B respectively.
Quantity of A in mixture left 2
Required quantity replaced = .
3
7 21
= 7x 9 litres 7x litres. 2.10 15 0.98 1
12 4 24. (c) C. P. of mixture
15 1
Quantity of B in mixture left
= ` 2.03 per kg
5 15 Profit on 1 kg of mixture = ` (2.25 – 2.03)
= 5x 9 litres 5x litres.
= ` 0.22
12 4
Profit on 5 quintals mixture = 0.22 × 500
21 ( 1 quintal = 100 kg)
7x = ` 110
4 7 28x 21 7
25. (d) By the rule of alligation,
15 9 20x 21 9
5x 9 Alcohol concentration :
4
252 x – 189 = 140 x + 147
112 x = 336 x = 3.
So, the cane contained 21 litres of A.
3 3
21. (b) 18 carat gold = pure gold = 24 18 carat gold
4 4
5 5
20 carat gold = pure gold = 24 20 carat gold
6 6
Required ratio = 18 : 20 = 9 : 10
y
o
u
Mixture and Alligations B-115

rs
m
Alcohol must be added in the ratio of 10 : 75 or 2 : 15 9
mean price = ` .

a
Quantity of alcohol to be added in 10 litres 13

h
b
2 4 By the rule of alligation, we have:

o
10

o
15 3 C.P. of 1 litre mixture in A C.P. of 1 litre mixture in B

b
.w
8 5
100 60 Mean price

o
26. (b) C. P. of mixture ` 48 13 7

rd
(100 25) 3

p
13

re
Let x kg be mixed. Then, 2 9

s
91 91

s
40 x 55 16

.c
48 2 1

o
16 x Required ratio : 2 : 7.

m
91 13
8x 16[55 48] x 14 kg 25
6 15 5 14 4
10 1 30. (d) Fineness 2 210
of the whole 14 carats
27. (b) 10 litres are withdrawn 6 5 4 15
50 5 31. (c) By the rule of alligation,
Quantity of milk after 2nd operation
2
1
50 1
5

16
50 32 litres
25
28. (d) Let the C.P. of spirit be ` 1 per litre.
5
Spirit in 1 litre mix. of A = litre; C.p. of a litre mix. in
7
5 Quantity of 32% spirit 6 3
A= ` .
7 Quantity of 18% spirit 8 4
7
Spirit in 1 litre mix. of B = litre; C.P. of 1 litre mix. in 3
13 of the butt
5
Now, wine of 32% spirit =
7
B=` .
13
3 4
8 8 The rest part i.e 1 of the butt has been
Spirit in 1 litre mix. of C = litre; Mean price = ` . 7 7
13 13
By the rule of alligation, we have : stolen.
32. (c) Ratio of milk in the containers are,
C.P. of 1 litre mixture in A C.P. of 1 litre mixture in B
1 3 5 5 3 25
5 7 5 :4 :5 : :
Mean price 6 8 12 6 2 12
7 13
8 and the ratio of water in the containers are,
13 5 5 7 25 5 35
1 9 5 :4 :5 : :
13 91 6 8 12 6 2 12
Ratio of mixture of milk and water in the containers
1 9
Required ratio : 7 : 9. 1 3 5 5 5 7
13 13 = 5 4 5 : 5 4 5
6 8 12 6 8 12
29. (a) Let cost of 1 litre milk be ` 1.
= 106 : 230 = 53 : 115
8
Mlk in 1 litre mix. in B litre, C.P. of a litre mix. in 33. (c) Let the third type of tea is priced at ` x per kg. Also
13 suppose that the three types of tea mixed together are
5
B=` . m, m and 2m kg,respectively.
7
5 126m 135m 2mx
Now, 153
Milk in 1 litre mix. in B = litre, C.P. of 1 litre mix. in m m 2m
7
5 261 2 x
B=` or 153
7 4
900 1 9 or 261 + 2x = 612
Milk in 1 litre of final mix. 1 litre;
13 100 13 351
or x = ` 175.5 per kg.
2
y
o
u
B-116 Mixture and Alligations

rs
34. (c) Let a container contains x units of liquid and y units of

m
Then 1 ?

a
liquid is taken out from it. If this operation is repeated

h
n times, then the final quantity of the liquid in the 1 36

b
= = 12 kg.

o
n 3

o
y

b
container is x 1 . 37. (a) By alligation rule

.w
x
CPcheaper CPdearer

o
Therefore, from this equation, we have

rd
2 1/3 2/5

p
y
24 54 1

re
3/8
54

s
s
(y = amount of acid initially drawn off) 1/40 1/24

.c
o
2 The ratio in which the two are to be mixed is
y 24 4

m
or 1
54 54 9 1 1
: =3:5
y 2 40 24
or 1 or x 1
54 3 54 3 38. (a) Iron Copper
or y = 18 litres Alloy I 8 : 6 14 kg.
35. (b) Let the capacity of the car be 100 litres. Alloy II 36 : 6 42 kg.
Type - A Type - B 44 : 12 56 kg
At the start 100 –
4
When the tank 39. (c) Quantity of milk = 45 = 36 litres
was half 50 50 5
When the tank was half 1
again 25 + 50 = 75 25 Quantity of water = 45 = 9 litres
When the tank 12.5 + 50 5
was half again 37.5 = 62.5 Let x litres of water be added.
Percentage of A type petrol now is 37.5 %
36 3
Then,
44 100 9 x 2
36. (a) C.P. of the mixture = = ` 40 per kg
110
72 = 27 + 3x or 3x = 45
Using alligation rule, the required ratio or x = 15 litres
31 43 40. (c) For 9 kg zinc, mixture melted = (9 + 11) kg.
40 For 28.8 kg zinc, mixture melted
= =1:3
3 9 20
= 28.8 kg 64 kg.
If 3 36 kg 9
y
o
u
Time and Work & Pipes and Cisterns B-117

rs
m
a
CHAPTER

h
TIME AND WORK &

b
10

o
o
b
PIPES AND CISTERNS

.w
o
rd
p
re
s
s
.c
o
TIME AND WORK 1 1 5

m
In most of the problems on time and work, either of the following In the first 2 hours, of the field is mown.
8 12 24
basic parameters are to be calculated :
5 5
TIME: In 8 hours, 4 of the field is mown.
24 6
If A can do a piece of work in X days, then A’s one day’s
5 1
1 Now, 1 – of the field remains to be mown.
work = th part of whole work. 6 6
X
1
1 In the 9th hour, Vikas mows of the field.
If A’s one day’s work = th part of whole work, then A can 8
X
finish the work in X days. 1 1 1
Remaining work = –
If A can do a piece of work in X days and B can do it in Y 6 8 24
days then A and B working together will do the same work
1 1
XY Vishal will finish the remaining work in
in days. 24 12
X Y
If A, B and C can do a work in X, Y and Z days respectively 1
or of an hour..
then all of them working together can finish the work in 2
XYZ 1 1
days. The total time required is 8 1 or 9 hours.
XY YZ XZ
2 2
Example 1 :
A can do a piece of work in 5 days, and B can do it in 6 days. 1 1
How long will they take if both work together ? Thus, the work will be finished at 8 9 17 or 5.30 pm.
2 2
Solution : Example 3 :
1 A can do a piece of work in 36 days, B in 54 days and C in 72
A’s 1 day’s work = th part of whole work and
5 days. All the three began the work together on the Dec. 15,
1 2014, but A left 8 days and B left 12 days before the
B’s 1 day’s work = th part of whole work completion of the work. If C took the rest for a week then in
6
how many days, the work was finished from the day it
1 1 11 th started ?
(A + B)’s one day’s work = part of whole work.
5 6 30 Solution :
So, both together will finish the work in Let the total time taken be x days.
30 8 According to the given condition
days 2 days.
11 11 x 8 x 12 x
1
By Direct Formula : 36 54 72
5 6 30 8 6 (x 8) 4 (x 12) 3x
A + B can do the work in days 2 days. 1
5 6 11 11 216
Example 2 :
Two men, Vikas and Vishal, working separately can mow a 6x 48 4x 48 3x 13x 96
1 1
field in 8 and 12 hours respectively. If they work in stretches 216 216
of one hour alternately, Vikas beginning at 8 a.m, when will 13x – 96 = 216 13x = 216 + 96 = 312
the mowing be finished?
Solution : 312
x 24
1 13
In the first hour, Vikas mows of the field. Since, C takes the rest for a week, so the number of days in
8
which the work was finished from one day it started = 31 i.e.
1
In the second hour, Vishal mows of the field. on 14.01.2015.
12
y
o
u
B-118 Time and Work & Pipes and Cisterns

rs
m
Example 4 : If A is ‘a’ times efficient than B and working together they

a
A and B can do a certain piece of work in 8 days, B and C

h
Z(a 1)
can do it in 12 days and C and A can do it in 24 days. How

b
finish a work in Z days then, time taken by A =

o
long would each take separately to do it ? a

o
b
Solution : days. and time taken by B = Z(a + 1) days.

.w
(A + B)’s one days’s work = 1/18, If A working alone takes ‘x’ days more than A and B together,

o
(A + C)’s one days’s work = 1/24, and B working along takes ‘y’ days more than A and B

rd
(B + C)’s one days’s work = 1/12, together then the number of days taken by A and B working

p
re
Now add up all three equations :
together is given by [ xy] days.

s
s
1 1 1 13

.c
2 (A + B + C)’s one days’s work =

o
18 24 12 72 Example 6 :

m
13 A and B can do alone a job in 6 days and 12 days. They began the
(A + B + C)’s one days’s work = work together but 3 days before the completion of job, A leaves
144 off. In how many days will the work be completed?
A’s one days’s work = (A + B + C)’s one days’s work Solution :
13 1 1 Let work will be completed in x days. Then,
– (B + C)’s one days’s work = work done by A in (x – 3) days + work done by B in
144 12 144
Since A completes of the work in 1 day, he will complete 1 x days = 1

144 i.e. x 3 x 1
work in = 144 days 6 12
1
By similar logic we can find that B needs days and C will 3x 6
1 x 6 days
144 12
require days. By Direct Formula:
5
If A and B together can do a piece of work in X days and A 12(6 3)
Required time = 6 days
alone can do it in Y days, then B alone can do the work in 12 6
XY Example 7 :
days.
Y–X A is half good a workman as B and together they finish a
Example 5 : job in 14 days. In how many days working alone will B
A and B together can do a piece of work in 6 days and A alone finish the job.
can do it in 9 days. In how many days can B alone do it? Solution :
Solution : Let B can do the work in x days
and A can do the work in 2x days
1
(A + B)’s 1 day’s work = th part of the whole work. 1 1 1
6 Then, (given)
x 2x 14
1
A’s 1 day’s work = th part of the whole work. 3
9 x 14 21 days
2
1 1 3 2 1 By Direct Formula :
B’s 1 day’s work = th
6 9 18 18
1
part of the whole work. Time taken by B = 14 1 21 days
B alone can do the work in 18 days. 2
By Direct Formula : If n men or m women can do a piece of work in X days, then
B alone can do the whole work in N men and M women together can finish the work in
6 9 54 nmX
18 days days.
9–6 3 nM mN
A and B can do a work in ‘X’ and ‘Y’ days respectively. They Example 8 :
started the work together but A left ‘a’ days before completion 10 men can finish a piece of work in 10 days, where as it
of the work. Then, time taken to finish the work is takes 12 women to finish it in 10 days. If 15 men and 6
Y(X a) women undertake to complete the work, how many days
they will take to complete it ?
X Y
Solution :
If ‘A’ is ‘a’ times efficient than B and A can finish a work in It is clear that 10 men = 12 women or 5 men = 6 women
X days, then working together, they can finish the work in 15 men + 6 women = (18 + 6) i.e. 24 women
aX Now 12 women can complete the work in 10 days
days. 24 women will do it in 5 days.
a 1
y
o
u
Time and Work & Pipes and Cisterns B-119

rs
m
By Direct Formula : Example 11 :

a
Two men and 7 boys can do a piece of work in 14 days. 3

h
10 12 10
5 days

b
Required time = men and 8 boys can do it in 11 days. In how many days can

o
10 6 12 15
8 men and 6 boys do a work 3 times as big as the first ?

o
Example 9 :

b
Solution :

.w
If 3 men or 4 women can reap a field in 43 days, how long 2 men + 7 boys in 14 days 28 men + 98 boys in 1 day

o
will 7 men and 5 women take to reap it ? 3 men + 8 boys in 11 days 33 men + 88 boys in 1 day

rd
Solution :

p
28 men + 98 boys = 33 men + 88 boys

re
1 2 boys 1 man
3 men reap of the field in 1 day..

s
s
43 Now, 2 men + 7 boys = 11 boys; 8 men + 6 boys = 22 boys

.c
More boys, fewer days; more work, more days

o
1

m
1 men reaps of the field in 1 day.. Boys Days Work
43 3
11 14 1
1
4 women reap of the field in 1 day..
43
1 22 x 3
1 woman reaps of the field in 1 day..
43 4
x 11 3
7 5 1 = × Number of days = 21 days.
14 12 1
7 men and 5 women reap = of the
43 3 43 4 12 Example 12 :
field in 1 day. Kaberi takes twice as much time as Kanti and thrice as much
7 men and 5 women will reap the whole field in 12 days. as Kalpana to finish a place of work. They together finish
Alternate method the work in one day. Find the time taken by each of them to
1 finish the work.
Required number of days = Solution :
7 5
43 3 43 4 Here, the alone time of kaberi is related to the alone times of
other two persons, so assume the alone time of kaberia = x,
43 3 4
= = 12 days. x x
4 4 5 3 Then, alone time of Kanti = and of Kalpana =
2 3
Example 10 :
If 12 men and 16 boys can do a piece of work in 5 days and Kaberi's 1 day work + Kanti’s 1 day work + Kalpana's 1 day
13 men and 24 boys can do it in 4 days, how long will 7 men work = combined 1 days work
and 10 boys take to do it ? 1 1 1 1
Solution : + + = x= 6
x x/2 x/3 1
12 men and 16 boys can do the work in 5 days .... (i)
13 men and 24 boys can do the work in 4 days .... (ii) Alone time for Kaberi = 6 days, for Kanti = 6/2 = 3 days,
Now it is easy to see that if the no. of workers be multiplied Kalpana = 6/3 = 2 days,
by any number, the time must be divided by the same number
(derived from : more worker less time). Example 13 :
Hence multiplying the no. of workers in (i) and (ii) by 5 and 1 man or 2 women or 3 boys can do a work in 44 days. Then
4 respectively, we get 5 (12 men + 16 boys) can do the work in how many days will 1 man, 1 woman and 1 boy do the
in 5/5 = 1 day work?
4 Solution :
4 (13 men + 24 boys) can do the work in = 1 day
4 Number of required days
or, 5(12m + 16b) = 4 (13m + 24b)
or, 60 m + 80 b = 52 m + 96 b 1 44 1 2 3
= = = 24 days
or, 60 m – 52 m = 96 b – 80 b 1 1 1 6 3 2
or, 8 m = 16 b 44 1 44 2 44 3
1 men = 2 boys.
If ‘M1’ persons can do ‘W1’ works in ‘D1’ days and ‘M2’
Thus, 12 men + 16 boys = 24 boys + 16 boys = 40 boys
and 7 men + 10 boys = 14 boys + 10 boys = 24 boys persons can do ‘W2’ works in ‘D2’ days then
The question now becomes : M1D1W2 = M2D2W1
“If 40 boys can do a piece of work in 6 days how long will 24 If T1 and T2 are the working hours for the two groups then
boys take to do it ?” M1D1W2T1 = M2D2W1T2
Using basic formula Similarly,
we have, M1D1W2T1E1 = M2D2W1T2E2, where E1 and E2 are the
40 × 5 = 24 × D2 efficiencies of the two groups.
40 5 1 If the number of men to do a job is changed in the ratio a : b,
or, D2 = = 8 days
24 3 then the time required to do the work will be in the ratio b :
y
o
u
B-120 Time and Work & Pipes and Cisterns

rs
m
a, assuming the amount of work done by each of them in
1 1 5

a
the given time is the same, or they are identical. The expert’s 4 day’s work = 1 – =

h
3 4 12

b
A is K times as good a worker as B and takes X days less

o
than B to finish the work. Then the amount of time required Now, total wages of ` 960 is to be distributed among two

o
b
friends and the expert in proportion to the amount of work

.w
K X done by each of them.
by A and B working together is days.

o
K2 1 So, 960 is to be divided in the proportion of

rd
1 1 5

p
If A is n times as efficient then B, i.e. A has n times as much
: : or 4 : 3 : 5

re
3 4 12
1

s
s
capacity to do work as B, A will take of the time taken by

.c
n 5
Share of expert = x 960 = ` 400

o
B to do the same amount of work. 12

m
Example 14 : Hence, the expert should get ` 400.
5 men prepare 10 toys in 6 days working 6 hrs a day. Then in Example 17 :
how many days can 12 men prepare 16 toys working 8 hrs a A certain number of men can do a work in 60 days. If there
day ? were 8 men more it could be finished in 10 days less. How
Solution : many men are there ?
This example has an extra variable ‘time’ (hrs a day), so the Solution :
‘basic-formula’ can’t work in this case. An extended formula Let there be x men originally.
is being given : (x + 8) men can finish the work in (60 – 10) = 50 days.
M1 D1 T1 W2 = M2 D2 T2 W1 Now, 8 men can do in 50 days what x men do in 10 days,
Here, 5 × 6 × 6 × 16 = 12 × D2 × 8 × 10 then by basic formula we have
5 6 6 16 8 50
D2 = = 3 days. x= = 40 men.
12 8 10 10
Example 15 : Alternate method :
A and B can do a work in 45 days and 40 days respectively. We have :
They began the work together, but A left after some time x men to the work in 60 days and (x + 8) men do th work in
and B finished the remaining work in 23 days. After how (60 – 10) = 50 days.
many days did A leave ? Then by “basic formula”, 60x = 50(x + 8)
Solution :
B finished the remaining work in 23 days. 50 8
x= = 40 men.
23 10
Work done by B in 23 days = work Example 18 :
40
Two coal loading machines each working 12 hours per day
23 17 for 8 days handles 9,000 tonnes of coal with an efficiency
A + B do together 1 – = work
40 40 of 90%. While 3 other coal loading machines at an efficiency
40 45 40 45 of 80 % set to handle 12,000 tonnes of coal in 6 days. Find
Now, A + B do 1 work in = days how many hours per day each should work.
40 45 85
Solution :
17 40 45 17
A + B do work in × = 9 days. N1 D1 R1 E1 N2 D2 R2 E2
40 85 40 Here =
Alternate method : W1 W2
If we ignore the intermediate steps, we can write a direct N1 = 2, R1 = 12h/ day : N2 = 3, R2 = ?
40 45 40 23 90
formula as : = 9 days. E1 = W1 = 9,000 ;
40 45 40 100
Example 16 :
80
Two friends take a piece of work for ` 960. One alone could E2 = W2 = 12, 000
do it in 12 days, the other in 16 days with the assistance of 100
an expert they finish it in 4 days. How much remuneration
2 8 12 90 3 6 R 2 80
the expert should get ? =
Solution : 9, 000 100 12, 000 100
4 1 R2 = 16 h / day.
First friend’s 4 day’s work = = (Since, the work is Each machine should work 16 h/ day.
12 3
finished in 4 days, when expert assists )
WORK AND WAGES
4 1
Second friends’s 4 day’s work = = Wages are distributed in proportion to the work done and
16 4 in indirect proportion to the time taken by the individual.
y
o
u
Time and Work & Pipes and Cisterns B-121

rs
m
Example 19 : Solution :

a
A, B and C can do a work in 6, 8 and 12 days respectively. This method is a substitute for the conventional method

h
Doing that work together they get an amount of Rs. 1350. and can be safely employed for most of the problems.

b
o
What is the share of B in that amount? Step 1 : Assume the thing to be found as ‘X’

o
b
Solution : Step 2 : In the first place look for X’s counterpart.

.w
1 e.g. in the above example, X = no. of men

o
A’s one day’s work So X’s counterpart = No. of men, given = 8.

rd
6
So write X = 8x.......

p
re
1 Now see the direct and indirect variation or simply see by
B’s one day’s work

s
8 which operation more men will be required & by which fewer:

s
.c
1 32 3 9 10 9

o
C’s one day’s work We have X 8 30 men

m
12 18 2 12 8 6
A’s share : B’s share : C’s share Example 23 :
1 1 1 If 5 engines consume 6 tonnes of coal when each runs 9 hrs
: :
6 8 12 per day, how much coal will be needed for 8 engines, each
Multiplying each ratio by the L.C.M. of their denominators, running 10 hrs. per day, it being given that 3 engines of the
the ratios become 4 : 3 : 2 former type consume as much as 4 engines of latter type ?
Solution :
1350 3
B’s share ` 450
9 8 10 3
We have X = 6 8 tons
Example 20 : 5 9 4
If 6 men working 8 hours a day earn ` 1680 per week, then Explanation :
how much will 9 men working 6 hours a day earn per week ? (1) More engines more coal ( > 1)
Solution : (2) More time, more coal ( > 1)
6m 8 hours ` 1680 (3) Latter consumes less coal than former ( < 1).
9m 6 hours ? In case of men working we have more time, less men (< 1)
but here we have more time, more coal ( > 1).
6 9
1680 = ` 1890 Here let W = 6 tonnes 5 × 9 × 4/3 engine hours
8 6 and let X 8 × 10 × 1 engine hours.
Alternate method :
As earnings are proportional to the work done, we have 8 10 1
or X = 6 tons × 8 tons
5 9 (4 / 3)
M1D1 M 2 D2 6 8 9 6
W2 = `Rs.1890
1890 Example 24 :
W1 W2 1680 W2
A garrison of 1500 men is provisioned for 60 days. After 25
Example 21 : days the garrison is reinforced by 500 men, how long will
A can do a piece of work in 15 days and B in 20 days. They the remaining provisions last ?
finished the work with the assistance of C in 5 days and got Solution :
` 45 as their wages, find the share for each in the wages. Since the garrison is reinforced by 500 men therefore then
Solution : are (1500 + 500) or 2000 men now,
A did in 5 days 1/3 of the work, since 60 – 25 = 35 days.
B did in 5 days 1/4 of the work. The provisions left would last 1500 men 35 days
1 1 5 Provisions left would last 1 man 35 × 1500 days
C did in 5 days 1 of the work Provisions left would last 2000 men
3 4 12
Since A, B, C did in 5 days 1/3, 1/4, 5/12 of the work 1500
35 × 26.25 days
respectively. 2000
1 Alternate method:
A’s share = ` 45 = Rs.15
` 15 1500 × 60 = (1500 × 25) + (2000 × X)
3
90000 – 37500 = 2000X
1 X = 26.25 days.
B’s share = ` 45 = `Rs.11.25
11.25
4 Example 25 :
40 men can cut 60 trees is 8 hrs. If 8 men leaves the job how
5
C’s share = ` 45 = Rs.18.75
` 18.75 many trees will be cut in 12 hours ?
12 Solution :
Example 22 : 40 men – working 8 hrs – cut 60 trees
If 8 men, working 9 hours per day can build a wall 18 meter
long, 2 meters wide and 12 meters high in 10 days, how 60
or, 1 man – working 1 hr – cuts tress
many men will be required to build a wall 32 meters long, 40 8
3 meters wide and 9 meters high by working 6 hours a day in 60 32 12
8 days ? Thus, 32 – working 12 hrs – cut = 73 trees.
40 8
y
o
u
B-122 Time and Work & Pipes and Cisterns

rs
m
Using basic concepts : Solution :

a
M1 = 40, D1 = 8 (As days and hrs both denote time) 6 8 24

h
W1 = 60 (cutting of trees is taken as work) (A + B) can do the work in = days.

b
6 8 7

o
M2 = 40 – 8 = 32, D2 = 12, W2 = ?

o
b
Putting the values in the formula 24

.w
C takes days to complete the work.
M1 D1 W2 = M2 D2 W1 7

o
We have , 40 × 8 × W2 = 32 × 12 × 60

rd
24

p
32 12 60 8
24 8 2

re
or, W2 = = 72 trees. 7
40 8 (B + C) takes 24 = = 2 days.

s
24 56 5

s
8

.c
Example 26 : 7

o
Two women, Ganga and Sarswati, working separately can

m
Example 29 :
mow a field in 8 and 12 hrs respectively, If they work in
A group of 20 cows can graze a field 3 acres in size in 10
stretches of one hour alternately, Ganga beginning at 9 a.m.,
days. How many cows can graze a field twice as large in 8
when will the mowing be finished ?
days ?
Solution :
Solution :
1
In the first hour Ganga mows of the field. Here, first of all, let us see how work can be defined. It is
8 obvious that work can be measured as “acres grazed”.
1 In the first case, there were 20 cows in the group.
In the second hour Saraswati mows of the fields.
12 They had to work for 10 days to do the work which we call
1 1 5 W (which = 3)
In the first 2 hrs of the field is mown. 20 × 10 = 3 ........... (i)
8 12 24
Do not be worried about the numerical values on either
5 5
In 8 hrs × 4 = of the field is mown...... (i) side. The point is that logically this equation is consistent
24 6 as the LHS indicates “Cowdays” and the RHS indicates
5 1 “Acres”, both of which are correct ways of measuring work
Now, 1 = of the remains to be mown. In the 9th done.
6 6
Now the field is twice as large. Hence the new equation is
1
hour Ganga mows of the field. C×8 =6 ........... (ii)
8
Just divide (ii) by (i) to get the answer.
1 1 1
Saraswati will finish the mowing of = of the 8C 6
6 8 24 =
200 3
1 1 1
field in or of an hours.
24 12 2 400
8C = 2 × 200 C= = 50 cows.
1 1 8
the total time required is 8 1 or 9 hrs. Hence, there were 50 cows in the second group.
2 2

Thus, the work will be finish at 9 + 9


1 1 1
= 18 or 6 p.m.
PIPE AND CISTERNS
2 2 2 The same principle of Time and Work is employed to solve the
Example 27 : problems on Pipes and Cisterns. The only difference is that in
I can finish a work in 15 days at 8 hrs a day. You can finish this case, the work done is in terms of filling or emptying a cistern
2 (tank) and the time taken is the time taken by a pipe or a leak
it in 6 days at 9 hrs a day. Find in how many days we can (crack) to fill or empty a cistern respectively.
3
Inlet : A pipe connected with a tank (or a cistern or a reservoir) is
finish it working together 10 hrs a day. called an inlet, if it fills it.
Solution : Outlet : A pipe connected with a tank is called an outlet, if it
First suppose each of us works for only one hr a day. empties it.
Then I can finish the work in 15 × 8 = 120 days and you can If a pipe can fill a tank in x hours, then the part filled in 1
20 1
finish the work in × 9 = 60 days. hour
3 x
But here we are given that we do the work 10 hrs a day. If a pipe can empty a tank in y hours, then the part of the
Then clearly we can finish the work in 4 days. full tank emptied in 1 hour
1
.
Example 28 : y
A can do a work in 6 days. B takes 8 days to complete it. C If a pipe can fill a tank in x hours and another pipe can
takes as long as A and B would take working together. How empty the full tank in y hours, then the net part filled in 1
long will it take B and C to complete the work together ? 1 1
hour, when both the pipes are opened – .
x y
y
o
u
Time and Work & Pipes and Cisterns B-123

rs
m
Time taken to fill the tank, when both the pipes are Example 31 :

a
xy Three pipes A, B and C can fill a cistern in 6 hrs. After

h
opened .

b
y–x working together for 2 hrs, C is closed and A and B fill the

o
cistern in 8 hrs. Then find the time in which the cistern can

o
If a pipe can fill a tank in x hours and another can fill the

b
be filled by pipe C.

.w
xy
same tank in y hours, then time taken to fill the tank , Solution :

o
y x

rd
when both the pipes are opened. 1

p
A + B + C can fill in 1 hr of cistern.

re
If a pipe fills a tank in x hours and another fills the same tank 6

s
is y hours, but a third one empties the full tank in z hours,

s
.c
and all of them are opened together, then net part filled in 1 2 1
A + B + C can fill in 2 hrs of cistern.

o
6 3

m
1 1 1
hr – 1 2
x y z Remaining part 1– is filled by A + B in 8 hrs.
3 3
xyz
Time taken to fill the tank hours. 8 3
yz xz – xy (A + B) can fill the cistern in = 12 hrs.
A pipe can fill a tank in x hrs. Due to a leak in the bottom it 2
is filled in y hrs. If the tank is full, the time taken by the leak Since (A + B + C) can fill the cistern in 6 hrs.
xy C = (A + B + C) – (A + B) can fill the cistern in
to empty the tank hrs.
y–x 12 6
A cistern has a leak which can empty it in X hours. A pipe hours = 12 hours.
12 – 6
which admits Y litres of water per hour into the cistern is Example 32 :
turned on and now the cistern is emptied in Z hours. Then Pipe A can fill a tank in 20 hours while pipe B alone can fill it
X Y Z in 30 hours and pipe C can empty the full tank in 40 hours.
the capacity of the cistern is litres.
Z X If all the pipes are opened together, how much time will be
A cistern is filled by three pipes whose diameters are X cm., needed to make the tank full ?
Y cm. and Z cm. respectively (where X < Y < Z). Three pipes Solution :
are running together. If the largest pipe alone will fill it in P By direct formula,
minutes and the amount of water flowing in by each pipe is
proportional to the square of its diameter, then the time in 20 30 40
The tank will be fill in
which the cistern will be filled by the three pipes is 30 40 20 40 – 20 30

PZ2 120 1
hrs.
minutes. 17
X 2
Y 2
Z 2 7 7
Example 33 :
If one filling pipe A is n times faster and takes X minutes less
Three pipes A, B and C can fill a tank in 6 minutes, 8 minutes
time than the other filling pipe B, then the time they will take
to fill a cistern, if both the pipes are opened together, is and 12 minutes, respectively. The pipe C is closed 6 minutes
before the tank is filled. In what time will the tank be full ?
nX X Solution :
minutes. A will fill the cistern in minutes Let it takes t minutes to completely fill the tank.
(n 2 1) n 1
nX t t t 6
and B will take to fill the cistern minutes. Now, 1
n 1 6 8 12
Here, A is the faster filling pipe and B is the slower one. 4t 3t 2 t 12
Two filling pipes A and B opened together can fill a cistern or 1
in t minutes. If the first filling pipe A alone takes X minutes 24
more or less than t and the second fill pipe B along takes Y or 9t – 12 = 24
minutes more or less than t minutes, then t is given by or 9t = 36 t = 4 min.
Example 34 :
[t xy] minutes. If three taps are opened together, a tank is filled in 12 hrs.
Example 30 : One of the taps can fill it in 10 hrs and another in 15 hrs.
A pipe can fill a cistern in 6 hours. Due to a leak in its bottom, How does the third tap work ?
it is filled in 7 hours. When the cistern is full, in how much Solution :
time will it be emptied by the leak? We have to find the nature of the third tap, whether it is a
Solution : filler or a waste pipe.
Part of the capacity of the cistern emptied by the leak in one Let it be a filler pipe which fills in x hrs.
1 1 1 10 15 x
hour = of the cistern. Then, 12
6 7 42 10 15 10x 15x
The whole cistern will be emptied in 42 hours. or, 150x = 150 × 12 + 25x × 12
y
o
u
B-124 Time and Work & Pipes and Cisterns

rs
m
or – 150x = 1800 x = – 12 Example 38 :

a
–ve sign shows that the third pipe is a waste pipe which A barrel full of beer has 2 taps one midway, which draw a

h
vacates the tank in 12 hrs. litre in 6 minutes and the other at the bottom, which draws

b
o
Example 35 : a litre in 4 minutes. The lower tap is lower normally used

o
b
4 pipes can fill a reservoir in 15, 20, 30 and 60 hours after the level of beer in the barrel is lower than midway. The

.w
respectively. The first was opened at 6 am, second at 7 am capacity of the barrel is 36 litres. A new assistant opens the

o
third at 8 am and fourth at 9 am. When will the reservoir be lower tap when the barrel is full and draws out some beer.

rd
full ? As a result the lower tap has been used 24 minutes before

p
re
Solution : the usual time. For how long was the beer drawn out by the

s
Let the time be t hours after 6 am. new assistant ?

s
.c
Solution :
1 (t 1) (t 2) (t 3)

o
t 1 The top tab is operational till 18 litres is drawn out.

m
15 20 30 60 Time after which the lower tap is usually open
4t + 3 (t – 1) + 2 (t – 2) + (t – 3) = 60 = 18 × 6 = 108 minutes
t = 7 hours Time after which it is open now = 108 – 24 = 84 minutes
It is filled at 1 pm Litres drawn = 84/6 = 14 litres
Example 36 : 18 – 14 = 4 litres were drawn by the new assistant.
A and B can fill a cistern in 7.5 minutes and 5 minutes re- Time = 4 × 4 = 16 minutes
spectively and C can carry off 14 litres per minute. If the Example 39 :
cistern is already full and all the three pipes are opened, A cistern can be filled by two pipes filling separately in 12
then it is emptied in 1 hour. How many litres can it hold ? and 16 min. respectively. Both pipes are opened together
Solution : for a certain time but being clogged, only 7/8 of the full
If the capacity is L litres, water filled in 1 hour = Water quantity of water flows through the former and only 5/6
removed in 1 hour. through the latter pipe. The obstructions, however, being
L L suddenly removed, the cistern is filled in 3 min. from that
L 60 60 14 60 moment. How long was it before the full flow began?
1 5
7 Solution :
2
1 1 7
2L Both the pipes A and B can fill of the
L 60 12L 14 60 L + 8L + 12L = 14 × 60 12 16 48
15 cistern in one minute, when their is no obstruction.
21L = 14 × 60 or L = 40 litres. With obstruction, both the pipes can fill
So the capacity of the cistern is 40 litres. 1 7 1 5 7 5 1
Example 37: of the cistern in one minute.
12 8 16 6 96 96 8
A cistern can be filled by two taps A and B in 25 minutes
and 30 minutes respectively can be emptied by a third in 15 Let the obstructions were suddenly removed after x min-
minutes. If all the taps are turned on at the same moment, utes.
x
what part of the cistern will remain unfilled at the end of 100 With obstruction, of the cistern could be filled in
minutes ? 8
Solution : x 8 x
x minutes and so the remaining 1 of the
1 1 1 1 8 8
We have part filled in 1 minute cistern was filled without obstruction in 3 minutes, i.e. In
25 30 15 150
1 8 x
Hence, 1 100 = 1/3rd of the tank is unfilled after one minute, of the cistern was filled.
150 24
100 minutes. 8 x 7
16 2x 7 x = 4.5
24 48
y
o
u
Time and Work & Pipes and Cisterns B-125

rs
m
a
h
b
o
o
b
.w
o
1. A does a work in 10 days and B does the same work in 15 9. A is 30% more efficient than B. How much time will they,

rd
days. In how many days they together will do the same working together, take to complete a job which A alone

p
work ? could have done in 23 days ?

re
s
(a) 5 days (b) 6 days (a) 11 days (b) 13 days

s
.c
(c) 8 days (d) 9 days 3

o
2. A man can do a piece of work in 10 days but with the (c) 20 days (d) None of these

m
17
assistance of his son, the work is done in 8 days. In how
10. A and B can finish a work in 10 days while B and C can do
many days, his son alone can do the same piece of work?
it in 18 days. A started the work, worked for 5 days, then B
(a) 15 days (b) 22 days
worked for 10 days and the remaining work was finished by
(c) 30 days (d) 40 days
C in 15 days. In how many days could C alone have finished
3. A can finish a work in 18 days and B can do the same work
the whole work ?
in half the time taken by A. Then, working together, what
(a) 30 days (b) 15 days
part of the same work they can finish in a day?
(c) 45 days (d) 24 days
1 1 11. 24 men working 8 hours a day can finish a work in 10 days.
(a) (b)
6 9 Working at the rate of 10 hours a day, the number of men
required to finish the same work in 6 days is :
2 2
(c) (d) (a) 30 (b) 32
5 7 (c) 34 (d) 36
4. George takes 8 hours to copy a 50 page manuscript while 12. 12 men complete a work in 18 days. Six days after they had
Sonia can copy the same manuscript in 6 hours. How many started working, 4 men joined them. How many days will all
hours would it take them to copy a 100 page manuscript, if of them take to complete the remaining work?
they work together ? (a) 10 days (b) 12 days
6 (c) 15 days (d) 9 days
(a) 6 (b) 9 13. A tyre has two punctures. The first puncture along would
7
have made the tyre flat in 9 minutes and the second alone
5 would have done it in 6 minutes. If air leaks out at a constant
(c) 9 (d) 14
7 rate, how long does it take both the punctures together to
5. A can do a piece of work in 25 days and B in 20 days. They make it flat ?
work together for 5 days and then A goes away. In how 1 1
many days will B finish the remaining work ? (a) 1 minutes (b) 3 minutes
2 2
(a) 17 days (b) 11 days
3 1
(c) 10 days (d) None of these (c) 3 minutes (d) 4 minutes
5 4
6. A man is twice as fast as a woman. Together the man and
14. A man, a woman or a boy can do a job in 20 days, 30 days or
the woman do the piece of work in 8 days. In how many
60 days respectively. How many boys must assist 2 men
days each will do the work if engaged alone?
and 8 women to do the work in 2 days ?
(a) man-14 days, woman-28 days
(a) 15 boys (b) 8 boys
(b) man-12 days, woman-24 days (c) 10 boys (d) None of these
(c) man-10 days, woman-20 days 15. A can do 50% more work as B can do in the same time. B
(d) None of these alone can do a piece of work in 20 hours. A, with help of B,
7. A and B can do a job in 16 days and 12 days respectively. 4 can finish the same work in how many hours ?
days before finishing the job, A joins B. B has started the (a) 12 (b) 8
work alone. Find how many days B has worked alone ? 1
(a) 6 days (b) 4 days (c) 13 1 (d) 5
3 2
(c) 5 days (d) 7 days 16. A machine P can print one lakh books in 8 hours, machine
8. A contractor undertakes to built a walls in 50 days. He Q can print the same number of books in 10 hours while
employs 50 peoples for the same. However after 25 days he machine R can print them in 12 hours. All the machines are
finds that only 40% of the work is complete. How many started at 9 a.m. while machine P is closed at 11 a.m. and the
more man need to be employed to complete the work in remaining two machines complete the work. Approximately
time? at what time will the work be finished?
(a) 25 (b) 30 (a) 11 : 30 am (b) 12 noon
(c) 35 (d) 20 (c) 12 : 30 pm (d) 1 pm
y
o
u
B-126 Time and Work & Pipes and Cisterns

rs
17. A can do a piece of work in 10 days, while B alone can do it

m
have completed in 3 days. Find the time for A to complete

a
in 15 days. They work together for 5 days and the rest of the job alone.

h
the work is done by C in 2 days. If they get ` 450 for the

b
1 3

o
whole work, how should they divide the money ? (a) 6 days (b) 5 days

o
4 4

b
(a) ` 225, ` 150, ` 75 (b) ` 250, ` 100, ` 100

.w
(c) ` 200, ` 150, ` 100 (d) ` 175, ` 175, ` 100 (c) 5 days (d) None of these

o
18. A alone would take 8 days more to complete the job than if 26. 12 men can complete a piece of work in 4 days, while 15

rd
both A and B would together. If B worked alone, he took women can complete the same work in 4 days. 6 men start

p
re
1 working on the job and after working for 2 days, all of them
4 days more to complete the job than A and B worked

s
stopped working. How many women should be put on the

s
2

.c
together. What time would they take if both A and B worked job to complete the remaining work, If it is to be completed

o
m
together ? in 3 days ?
(a) 7 days (b) 5 days (a) 15 (b) 18
(c) 4 days (d) 6 days (c) 22 (d) Data inadequate
19. 10 men and 15 women together can complete a work in 6 27. If 6 men and 8 boys can do a piece of work in 10 days while
days. It takes 100 days for one man alone to complete the 26 men and 48 boys can do the same in 2 days, the time
same work. How many days will be required for one woman taken by 15 men and 20 boys in doing the same type of
alone to complete the same work ? work will be:
(a) 90 (b) 125 (a) 4 days (b) 5 days
(c) 145 (d) None of these (c) 6 days (d) 7 days
1 28. A contract is to be completed in 46 days and 117 men were
20. After working for 8 days, Anil finds that only of the work
3 set to work, each working 8 hours a day. After 33 days, 4/7
has been done. He employs Rakesh who is 60% efficient as of the work is completed. How many additional men may be
Anil. How many more days will Anil take to complete the employed so that the work may be completed in time, each
job ? man now working 9 hours a day ?
(a) 15 days (b) 12 days (a) 80 (b) 81
(c) 10 days (d) 8 days (c) 82 (d) 83
21. A sum of ` 25 was paid for a work which A can do in 32 29. Ramesh is twice as good a workman as Sunil and finishes a
days, B in 20 days, B and C in 12 days and D in 24 days. piece of work in 3 hours less than Sunil. In how many hours
How much did C receive if all the four work together ? they together could finish the same piece of work ?
14 16 (a) 1 (b) 2
(a) ` (b) ` 2
3 3 3
15 17
(c) ` (d) ` (c) 1 2 (d) None of these
3 3 3
22. A and B can do a job in 15 days and 10 days, respectively. 30. One hundred men in 10 days do a third of a piece of work.
They began the work together but A leaves after some days The work is then required to be completed in another 13
and B finished the remaining job in 5 days. After how many days. On the next day (the eleventh day) 50 more men are
days did A leave ? employed, and on the day after that, another 50. How many
(a) 2 days (b) 3 days men must be discharged at the end of the 18th day so that
(c) 1 day (d) None of these the rest of the men, working for the remaining time, will just
23. Mr. Suresh is on tour and he has ` 360 for his expenses. If complete the work ?
he exceeds his tour by 4 days he must cut down daily (a) 100 (b) 105
expenses by ` 3. The number of days of Mr. Suresh’s tour (c) 110 (d) 115
programme is : 31. If 12 men or 15 women or 18 boys can do a piece of work in
(a) 20 days (b) 24 days 15 days of 8 hours each, find how many men assisted by 5
(c) 40 days (d) 42 days women and 6 boys will finish the same work in 16 days of 9
24. A can do a job in 3 days less time than B. A works at it alone hours each.
for 4 days and then B takes over and completes it. If (a) 6 men (b) 2 men
altogether 14 days were required to finish the job, then in (c) 8 men (d) 4 men
how many days would each of them take alone to finish it? 32. The work done by a man, a woman and a child is in the ratio
(a) 17 days, 20 days (b) 12 days, 15 days of 3 : 2 : 1. There are 20 men, 30 women and 36 children in a
(c) 13 days, 16 days (d) None of these factory. Their weekly wages amount to ` 780, which is
25. Two workers A and B working together completed a job in 5 divided in the ratio of work done by the men, women and
days. If A worked twice as efficiently as he actually did and children. What will be the wages of 15 men, 21 women and
30 children for 2 weeks ?
1 (a) ` 585 (b) ` 292.5
B worked as efficiently as he actually did, the work would
3 (c) ` 1170 (d) ` 900
y
o
u
Time and Work & Pipes and Cisterns B-127

rs
m
33. 2 men and 3 boys can do a piece of work in 10 days while due to leakage in the bottom, 32 minutes extra are taken for

a
3 men and 2 boys can do the same work in 8 days. In how the cistern to be filled up. If the cistern is full, in what time

h
many days can 2 men and 1 boy to the work ? would the leak empty it ?

b
o
(a) 110 hr (b) 112 hr

o
1 1

b
(a) 12 days (b) 11 days (c) 115 hr (d) 100 hr

.w
2 2 43. Two pipes A and B can fill a cistern in 10 and 15 minutes

o
1 1 respectively. Both fill pipes are opened together, but at the

rd
(c) 15 days (d) 13 days end of 3 minutes, ‘B’ is turned off. How much time will the

p
2 2

re
34. A can do a certain job in 12 days. B is 60% more efficient cistern take to fill ?

s
(a) 6 min (b) 8 min

s
than A. How many days B alone take to do the same job ?

.c
(c) 10 min (d) 12 min

o
1

m
(a) 7 (b) 11 44. A cistern has two taps which fill it in 12 minutes and 15
2 minutes respectively. There is also a waste pipe in the cistern.
1 When all the three are opened, the empty cistern is full in 20
(c) 8 (d) 8 minutes. How long will the waste pipe take to empty the full
2
cistern ?
35. 12 men and 16 boys can do a piece of work in 5 days, 13 men
(a) 10 min (b) 12 min
and 24 boys can do it in 4 days. Then the ratio of daily work
(c) 15 min (d) None of these
done by a man to that of a boy is
(a) 2 : 1 (b) 3 : 1 45. Two pipes A and B can fill a tank in 15 and 12 hours
(c) 3 : 2 (d) 5 : 4 3
respectively. Pipe B alone is kept open for of time and
36. x is 3 times as faster as y and is able to complete the work in 4
40 days less than y. Then the time in which they can complete both pipes are kept open for remaining time. In how many
the work together ? hours, the tank will be full ?
(a) 15 days (b) 10 days (a) 18 h (b) 20 h
1 (c) 10 h (d) 13.5 h
(c) 7 days (d) 5 days 46. A pipe can fill a tank in 15 minutes and another one in 10
2
minutes. A third pipe can empty the tank in 5 minutes. The
37. Pipe A can fill a tank in 5 hours, pipe B in 10 hours and pipe
first two pipes are kept open for 4 minutes in the beginning
C in 30 hours. If all the pipes are open, in how many hours
and then the third pipe is also opened. In what time will the
will the tank be filled ?
(a) 2 (b) 2.5 tank be empited ?
(c) 3 (d) 3.5 (a) 35 min (b) 15 min
38. Pipe A and B running together can fill a cistern in 6 minutes. (c) 20 min (d) Cannot be emptied
If B takes 5 minutes more than A to fill the cistern then the 47. Two fill pipes A and B can fill a cistern in 12 and 16 minutes
times in which A and B will fill the cistern separately will be, respectively. Both fill pipes are opened together, but 4
respectively: minutes before the cistern is full, one pipe A is closed. How
(a) 15 min, 20 min (b) 15 min, 10 min much time will the cistern take to fill ?
(c) 10 min, 15 min (d) 25 min, 20 min 1 1
39. Pipes A and B can fill a tank in 5 and 6 hours respectively. (a) 9 min . (b) 3 min .
7 3
Pipe C can empty it in 12 hours. If all the three pipes are
opened together, then the tank will be filled in : (c) 5 min. (d) 3 min.
48. Two fill taps A and B can separately fill a cistern in 45 and 40
13 8 minutes respectively. They started to fill a cistern together
(a) 1 hours (b) 2 hours
17 11 but tap A is turned off after few minutes and tap B fills the
9 1 rest part of cistern in 23 minutes. After how many minutes,
(c) 3 hours (d) 4 hours was tap A turned-off ?
17 2
(a) 9 min (b) 10 min
40. Two taps can fill a tank in 12 and 18 minutes respectively.
(c) 12 min (d) None of these
Both are kept open for 2 minutes and the first is turned off.
49. Three fill pipes A, B and C can fill separately a cistern in 3,
In how many minutes more will the tank be filled ?
4 and 6 minutes respectively. A was opened first. After 1
(a) 15 min. (b) 20 min.
minute, B was opened and after 2 minutes from the start of
(c) 11 min. (d) 13 min.
A, C was also opened. Find the time when the cistern will be
41. One fill pipe A is 3 times faster than second fill pipe B and
full ?
takes 10 minutes less time to fill a cistern than B takes. Find
when the cistern will be full if fill pipe B is only opened. 1 1
(a) 2 min (b) 4 min
(a) 20 min (b) 18 min 9 2
(c) 15 min (d) 10 min
42. Two pipes can fill a cistern in 14 and 16 hours respectively. 3
(c) 3 min (d) None of these
The pipes are opened simultaneously and it is found that 4
y
o
u
B-128 Time and Work & Pipes and Cisterns

rs
50. A tap can fill a tank in 16 minutes and another can empty it

m
59. 4 pipes each of 3 cm diameter are to be replaced by a single

a
in 8 minutes. If the tank is already 1/2 full and both the taps pipe discharging the same quantity of water. What should

h
are opened together, will the tank be filled or emptied? How

b
be the diameter of the single pipe, if the speed of water is

o
long will it take before the tank is either filled or emptied the same.

o
b
completely as the case may be ? (a) 2 cm (b) 4 cm

.w
(a) Emptied; 16 min (b) Filled; 8 min (c) 6 cm (d) 8 cm

o
(c) Emptied; 8 min (d) Filled; 12 min

rd
60. A ship 55 kms. from the shore springs a leak which admits 2
51. A pump can be operated both for filling a tank and for

p
tones of water in 6 min ; 80 tones would suffer to sink her,

re
emptying it. The capacity of tank is 2400 m3. The emptying but the pumps can throw out 12 tones an hour. Find the

s
capacity of the pump is 10 m3 per minute higher than its

s
average rate of sailing that she may just reach the shore as

.c
filling capacity. Consequently, the pump needs 8 minutes

o
she begins to sink.

m
less to empty the tank to fill it. Find the filling capacity of
(a) 5.5 km/h (b) 6.5 km/h
pump.
(c) 7.5 km/h (d) 8.5 km/h
(a) 50 m3/min (b) 60 m3/min 61. Two pipes A and B can fill a tank in 24 minutes and 32
(c) 58 m /min3 (d) None of these minutes respectively. If both the pipes are opened
52. A cistern has three pipes, A, B and C. The pipes A and B can
simultaneously, after how much time should B be closed
fill it in 4 and 5 hours respectively and C can empty it in 2
so that the tank is full in 18 minutes?
hours. If the pipes are opened in order at 1, 2 and 3 a.m.
(a) 6 min. (b) 8 min.
respectively, when will the cistern be empty ?
(c) 12 min. (d) 14 min.
(a) 3 p.m. (b) 4 p.m.
(c) 5 p.m. (d) 6 p.m. 62. A can build up a wall in 8 days while B can break it in 3 days.
53. A tank is filled in 5 hours by three pipes A, B and C. The A has worked for 4 days and then B joined to work with A
pipe C is twice as fast as B and B is twice as fast as A. How for another 2 days only. In how many days will A alone
much time will pipe A alone take to fill the tank ? build up the remaining part of wall?
(a) 20 hrs (b) 25 hrs 1 1
(c) 35 hrs (d) Cannot be determind (a) 13 days (b) 7 days
3 3
54. Two pipes A and B can fill a tank in 15 hours and 20 hours
respectively while a third pipe C can empty the full tank in 1
25 hours. All the three pipes are opened in the begining. (c) 6 days (d) 7 days
3
After 10 hours, C is closed. In how much time, will the tank
63. A group of men decided to do a job in 4 days. But since
be full ?
20 men dropped out every day, the job completed at the
(a) 12 hrs (b) 13 hrs
end of the 7th day. How many men were there at the
(c) 16 hrs (d) 18 hrs
beginning?
55. Three taps A, B and C can fill a tank in 12, 15 and 20 hours
respectively. If A is open all the time and B and C are open (a) 240 (b) 140
for one hour each alternately, then the tank will be full in : (c) 280 (d) 150
64. One man and six women working together can do a job
2
(a) 6 hrs. (b) 6 hrs. in 10 days. The same job is done by two men in 'p' days
3 and by eight women in p + 5 days. By what percentage
1 is the efficiency of a man greater than that of a woman?
(c) 7 hrs. (d) 7 hrs.
2 (a) 300% (b) 500%
56. Two pipes can fill a tank in 20 and 24 minutes respectively (c) 600% (d) 700%
and a waste pipe can empty 3 gallons per minute. All the 65. The total number of men, women and children working in
three pipes working together can fill the tank in 15 minutes. a factory is 18. They earn ` 4000 in a day. If the sum of
The capacity of the tank is : the wages of all men, all women and all children is in the
(a) 60 gallons (b) 100 gallons ratio of 18 : 10 : 12 and if the wages of an individual man,
(c) 120 gallons (d) 180 gallons woman and child is in the ratio 6 : 5 : 3, then how much
57. A hot pipe takes 3 minutes longer to fill a tank than the cold a woman earn in a day?
pipe. Together they take 6 minutes 40 seconds. Time taken (a) ` 400 (b) ` 250
by the cold pipe alone to fill the tank is : (c) ` 150 (d) ` 120
(a) 6 minutes (b) 18 minutes 66. A can do a job in 3 days less time than B. A works at it alone
(c) 9 minutes (d) 12 minutes for 4 days and then B takes over and completes it. If
58. Water flows at 3 metres per sec through a pipe of radius 4 altogether 14 days were required to finish the job, then in
cm. How many hours will it take to fill a tank 40 metres long,
how many days would each of them take alone to finish it?
30 metres broad and 8 metres deep, if the pipe remains full?
(a) 17 days, 20 days (b) 12 days, 15 days
(a) 176.6 hours (b) 120 hour
(c) 135.5 hours (d) None of these (c) 13 days, 16 days (d) None of these
y
o
u
Time and Work & Pipes and Cisterns B-129

rs
m
67. 3 small pumps and a large pump are filling a tank. Each of the 72. Pratibha is thrice as efficient as Sonia and is therefore able

a
three small pumps works at 2/3rd the rate of the large pump. to finish a piece of work in 60 days less than Sonia. Pratibha

h
b
If all 4 pumps work at the same time, they should fill the tank and Sonia can individually complete the work respectively

o
in what fraction of the time that it would have taken the large

o
in

b
pump alone? (a) 30, 60 days (b) 60, 90 days

.w
(a) 4/7 (b) 1/3 (c) 30, 90 days (d) 40, 120 days

o
rd
(c) 2/3 (d) 3/4 73. A can do a certain work in the same time in which B and C

p
68. The Bubna dam has four inlets. Through the first three together can do it. If A and B together could do it in 10 days

re
inlets, the dam can be filled in 12 minutes; through the

s
and C alone in 50 days, then B alone could do it in

s
second, the third and the fourth inlet, it can be filled in 15

.c
(a) 15 days (b) 20 days

o
minutes; and through the first and the fourth inlet, in 20 (c) 25 days (d) 30 days

m
minutes. How much time will it take all the four inlets to fill 74. A can do a piece of work in 6 days. B can do the same work
up the dam? in 15 days. How long would both of them take to do the
(a) 8 min (b) 10 min same work ?
(c) 12 min (d) None of these (a) 2 days (b) 4 days
69. Seventy-five men are employed to lay down a railway line
(c) 6 days (d) 8 days
in 3 months. Due to certain emergency conditions, the work
75. 12 men construct 1.5 km of road in 7 days. 28 men will
was to be finished in 18 days. How many more men should
construct 12 km of roads in
be employed to complete the work in the desired time ?
(a) 300 (b) 325 (a) 20 days (b) 24 days
(c) 350 (d) 375 (c) 28 days (d) 38 days
70. A, B and C together can do a piece of work in 40 days. After 76. X and Y can do a piece of work in 30 days. They work
working with B and C for 16 days, A leaves and then B and together for 6 days and then X quits and Y finishes the work
C complete the remaining work in 40 days more. A alone in 32 more days. In how many days can Y do the piece of
could do the work in work alone?
(a) 80 days (b) 90 days (a) 30 days (b) 32 days
(c) 100 days (d) 120 days (c) 34 days (d) 40 days
71. Three pipes A, B and C can fill a tank in 6 hours. After 77. 40 men can finish a piece of work in 60 days. After some
working it together for 2 hours, C is closed and A and B can days, 10 men leave the work so that the work is finished in
fill the remaining part in 7 hours. The number of hours taken 70 days. The number of days after which 10 men left the
by C alone to fill the tank is work is
(a) 10 (b) 12 (a) 20 days (b) 25 days
(c) 14 (d) 16
(c) 30 days (d) 40 days

ANSWER KEY
1 (b) 11 (b) 21 (b) 31 (b) 41 (c) 51 (a) 61 (b) 71 (c)
2 (d) 12 (d) 22 (b) 32 (c) 42 (b) 52 (c) 62 (b) 72 (c)
3 (a) 13 (c) 23 (a) 33 (a) 43 (b) 53 (c) 63 (b) 73 (c)
4 (a) 14 (b) 24 (b) 34 (a) 44 (a) 54 (a) 64 (b) 74 (c)
5 (b) 15 (b) 25 (a) 35 (a) 45 (c) 55 (c) 65 (b) 75 (b)
6 (b) 16 (d) 26 (a) 36 (a) 46 (c) 56 (c) 66 (b) 76 (d)
7 (c) 17 (a) 27 (a) 37 (c) 47 (a) 57 (d) 67 (b) 77 (c)
8 (a) 18 (d) 28 (b) 38 (c) 48 (a) 58 (a) 68 (b)
9 (b) 19 (d) 29 (a) 39 (c) 49 (a) 59 (c) 69 (a)
10 (c) 20 (c) 30 (c) 40 (d) 50 (c) 60 (a) 70 (c)
y
o
u
B-130 Time and Work & Pipes and Cisterns

rs
m
a
h
b
o
o
b
.w
1 1 1

o
1. (b) A’s 1 day’s work = and B’s 1 day’s work = B’s one day’s work th work

rd
10 15 12

p
1 1 1 Let B has worked alone = x days. Then,

re
(A + B)’s 1 day’s work = .

s
10 15 6 A’s amount of work + B’s amount of work = 1

s
.c
So, both together will finish the work in 6 days.
1 1

o
1 4 (x 4) 1

m
2. (d) (Man + Son)’s one day’s work = 16 12
8
1 1 x 4 3
Man’s one day’s work = 1 x 12 4
10 4 12 4
1 1 1 x 5 days
Son’s one day’s work =
8 10 40 8. (a) 50 men complete 0.4 work in 25 days.
Son can do it in 40 days.
1 1 Applying the work rule, m1 d1 w 2 m 2 d 2 w1
3. (a) A’s 1 day’s work = and B’s 1 day’s work = . we have,
18 9
1 1 1 50 25 0.6 m 2 25 0.4
(A + B)’s 1 day’s work = .
18 9 6 50 25 0.6
or m2 = 75 men
4. (a) In an hour, George and Sonia together can copy 25 0.4
1 1 7 Number of additional men required = (75 – 50) = 25
of a 50-page manuscript.
6 8 24 9. (b) Ratio of times taken by A and B = 100 : 130 = 10 : 13.
7 Suppose B takes x days to do the work.
i.e. In an hour they together can copy of the
48 23 13 299
Then, 10 : 13 : : 23 : x x x .
100-page manuscript. 10 10
i.e. They together can copy a 100-page manuscript in
48 6 1 10
hours, i.e. 6 hours. A’s 1 day’s work = ; B’s 1 days work = .
7 7 23 299
5. (b) (A + B)’s 5 days’ work
1 10 23 1
1 1 45 9 (A + B)’s 1 day’s work = .
23 299 299 13
=5
25 20 100 20 A and B together can complete the job in 13 days.
9 11 Alternate Method:
Remaining work = 1
20 20 1.3×23
A and B together complete work in =13 days
11 1.3+1
of the work would be finished by B in 10. (c) Let C completes the work in x days.
20
11 1
Work done by (A + B) in 1 day =
20 11days. 10
1
1
20 Work done by (B +C) in 1 day =
18
6. (b) Let the man alone do the work in x days.
Then, the woman alone do the work in 2x days. A’s 5 days’ work + B’s 10 days’ work + C’s 15 days’
work = 1
1 or (A + B)’s 5 days’ work + (B + C)’s 5 days’ work
Their one day’s work = th part of whole work
8 + C’s 10 days’ work = 1
1 1 1 5 5 10
i.e. or 1 or x = 45 days
x 2x 8 10 18 x
x 12 days 11. (b) m1 d1 t1 = m2 d2 t2
man takes 12 days and woman 2x = 24 days. 24 × 10 × 8 = m2 × 6 × 10
1
7. (c) A’s one day’s work = th work 24 10 8
16 m2 32 men
6 10
y
o
u
Time and Work & Pipes and Cisterns B-131

rs
m
1 23
12. (d) In 1 day, work done by 12 men = So, work will be done by Q and R in

a
18

h
60

b
6 1

o
In 6 days, work done by 12 men = 60 23 23

o
18 3 hours 2 hours.

b
11 60 11

.w
2
Remaining work = So, the work will be finished approximately 2 hours

o
3

rd
after 11 a.m., i.e., around 1 p.m.
Now, m1 d1 w 2 m 2 d 2 w1

p
re
1 1 5
2 17. (a) Work done by A and B in 5 days = 5

s
or 12 18 16 d 2 1 10 15 6

s
.c
3

o
5 1
4 18 2 Work remaining = 1

m
or d2 9 days 6 6
16
C alone can do the work in 6 × 2 = 12 days
1 1 5 5 5 2
13. (c) 1 minute’s work of both the punctures = . : : 3 : 2 :1
9 6 18 Ratio of their share work =
10 15 12
So, both the punctures will make the tyre flat in Share of wages = ` 225, ` 150, ` 75.
18 3 18. (d) Let if both A and B work together, they take x days.
3 min.
5 5 1
(A + B)’s 1 days’s work th work .
1 1 x
14. (b) Man’s two day’s work = 2 th work th work
20 10 1
A’s 1 day’s work th work .
Woman’s two days’s work x 8
1 1 1
2 th work th work B’s 1 day’s work th work .
30 15 x 9/2
1 1
Boy’s two day’s work th work =
2 th work 1 2 1
60 30 Now,
x 8 2x 9 x
Now, let 2 men, 8 women and x boys can complete
work in 2 days. Then , x(2x 9 2x 16) (x 8)(2x 9)
2 men’s work +8 women’s work + x boy’s work =1 4x2 + 25x = 2x2 + 25x + 72
1 1 1 x2 = 36 x = 6 days
2 8 x 1
10 15 30 Alternate Method:
1 8 9
x 1 30 x = 8 boys A and B together finish the work in 8× =6 days
5 15 2
15. (b) B alone can do a work in 20 hours. 1
3 19. (d) 1 man’s 1 day’s work = .
A alone can do of the work in 20 hours. 100
2
1
40 (10 men + 15 women)’s 1 day’s work = .
i.e., A alone can do the same work in hours 6
3 15 women’s 1 day’s work
3 1 5 1
(A + B)’s one hour’s work 1 10 1 1 1
40 20 40 8 = .
A and B together can finish the whole work in 6 100 6 10 15
8 hours.
1
1 1 1 37 1 woman’s 1 day’s work = .
16. (d) (P + Q + R)’s 1 hour’s work = . 225
8 10 12 120 1 woman alone can complete the work in 225 days.
37 37 1
Work done by P, Q and R in 2 hours = 2 . 20. (c) In 8 days, Anil does rd work .
120 60 3
37 23 1
Remaining work = 1 . in 1 day, he does th work.
60 60 24
1 1 11 1 1
(Q + R)’s 1 hour’s work = . Rakesh’s one day’s work = 60% of = th work .
10 12 60 24 40
11 1 2
Now, work is done by Q and R in 1 hour.. Remaining work 1
60 3 3
y
o
u
B-132 Time and Work & Pipes and Cisterns

rs
m
(Anil and Rakesh)’s one day’s work x 3 days

a
1 1 1

h
23. (a) Let Suresh undertakes a tour of x days.
th work

b
=
24 40 15

o
360

o
Then, expenses for each day =

b
1 x

.w
Now, th work is done by them in one day..
15

o
360 360

rd
Now, 3
2 2 x 4 x

p
rd work is done by them in 15 10 days

re
3 3
1 1

s
or 360 =3

s
1

.c
21. (b) A’s one day’s work = x x 4

o
32

m
or x 2 4 x 480 0 or x = – 24 or x = 20
1
B's one day’s work = Since, x 24 we have x = 20
20
24. (b) Let B can finish the work in x days.
1 Then A can finish the work in (x – 3) days.
(B + C)'s one day’s work =
12 1
B’s one day’s work th work
1 1 1 x
C's one day’s work =
12 20 30 1
A’s one day’s work th work
1 x 3
D's one day’s work =
24
4
(A + B + C + D)’s one day’s work A’s 4 days’ work = th work
x 3
1 1 1 1 75 120 80 100
= 4 x 7
32 20 30 24 2400 Remaining work 1 th work
x 3 x 3
375 15 5
= The remaining work done by B in 14 – 4 = 10 days.
2400 96 32
x 7
5 1 Now, in 10 days, work done by B th work
Out of of work done, of the work is done x 3
32 30
by C. 1 x 7
Out of ` 25 paid for the work, C will receive In 1 day, work done by B = th work
10 x 3
1/ 30 1 32 16
` 25, i.e. 25, i.e. ` 1 x 7 1
5 / 32 30 5 3 and
10 x 3 x
1
22. (b) A’s one day’s work = th work . x 15days
15
B will finish in 15 days and A will finish in 12 days
1
B’s one day’s work th work . 1
10 25. (a) (A + B)’s one day’s work th work
5
1 1 1 Let A can do job in x days. Then,
(A + B)’s one day’s work th work.
15 10 6
1
Let A left after x days. A’s one day’s work th work
x
x
(A + B)’s x days’ work th work . 1 1 x 5
6 and B’s one day’s work – th work
5 x 5x
x 6 x
Remaining work 1 th work.
6 6 1 1
Now , (2A)’s work + B’s work = rd work
3 3
6 x
Now, in 5 days, work done by B th work .
6 2 1 x 5 1 25 1
x 6 days
6 x x 3 5x 3 4 4
In 1 day work done by B th work
30 1
26. (a) 1 man’s 1 day’s work = ;
6 x 1 48
and
30 10 1
1 woman’s 1 day’s work = .
60
y
o
u
Time and Work & Pipes and Cisterns B-133

rs
m
100 × 10 + 150 × 1 + 200 × 7 + (200 – X) × 5 = 100 × 30
6 1

a
6 men’s 2 day’s work = 2 . 5X = 550 X = 110 men

h
48 4 31. (b) Given 12 men 15 women 18 boys

b
o
1 Man 1.5 boys, 1 woman = 6/5 boys.

o
1 3

b
Remaining work = 1 . Now, 5W + 6B = 12B.

.w
4 4
Required answer is calculated as follows :

o
1 Total no. of boys reqd. = 18 × [(15/16) × (8/9)]

rd
Now, work is done in 1 day by 1 woman.
= 15 boys

p
60

re
The number of boys already present = 12.
3

s
So, work will be done in 3 days by Hence, 3 boys more required.

s
.c
4 But 3 boys = 2 men.

o
m
3 1 So, 2 men are required.
60 = 15 women. 32. (c) Men Women Children
4 3
Work 3 2 1
27. (a) Let 1 man’s 1 day’s work = x and
1 boy’s 1 day’s work = y. Numbers 20 30 36
Ratio of wages = (3 × 20) : (2 × 20) : (1 × 36) = 5 : 5 : 3
1 1
Then, 6x + 8y = and 26x + 48y = . 5
10 2 Total wages of men = 780 `300
Solving these two equations, we get : 13
Wages of a man = ` 15
1 1
x and y . Similarly, wages of woman = ` 10
100 200 and wages of child = ` 5
(15 men + 20 boys)’s 1 day’s work Total waves of 15 men, 21 women and 30 children
15 20 1 = 15 × 15 + 21 × 10 + 30 × 5 = 585
= . Total wages for 2 weeks = ` 1170
100 200 4
15 men and 20 boys can do the work in 4 days. 33. (a) Let 1 man’s 1 days’ work= x & 1 boy’s 1 day’s work = y
28. (b) Let x additional men employed. 1 1
117 men were supposed to finish the whole work in Then, 2x + 3y = and 3x + 2y =
10 8
46 × 8 = 368 hours.
4 7 1
But 117 men completed of the work in 33 × 8 Solving, we get : x and y
7 200 100
= 264 hours (2 men + 1 boy)’s 1 day’s work
117 men could complete the work in 462 hours. 7 1 16 2
3 = 2 1
Now (117 + x) men are supposed to do of the work, 200 100 200 25
7 So, 2 men and 1 boy together can finish the work in
working 9 hours a day, in 13 × 9 = 117 hours, so as to
finish the work in time. 1
12 days.
i.e. (117 + x) men are supposed to complete the whole 2
7 34. (a) Ratio of time taken by A and B = 160 : 100
work in 117 = 273 hours. =8:5
3
Suppose, B alone takes x days to do the job
(117 + x) × 273 = 117 × 462 then, 8 : 5 : : 12 : x
(117 + x) × 7 = 3 × 462 8x = 5 × 12
x + 117 = 3 × 66 = 198 x = 81
Required number of additional men to finish the 5 12 1
x= 7 days.
work in time = 81. 8 2
29. (b) Let Sunil finishes the job in x hours. 35. (a) Let 1 man’s 1 days work = x
x 1 boy’s 1 day’s work = y
Then, Ramesh will finish the job in hours.
2
1
x 12x + 16y =
We have, x 3 x 6 5
2
Therefore, Sunil finishes the job in 6 hours and Ramesh 1
in 3 hours. 13x + 24y =
4
1 1 1 Solving these two equation we get,
Work done by both of them in 1 hour =
6 3 2
1 1
They together finish the piece of work in 2 hours. x ,y
30. (c) Suppose that X men must be discharged at the end of 100 200
the 18th day. Required ratio = 2 : 1
y
o
u
B-134 Time and Work & Pipes and Cisterns

rs
m
36. (a) If x complete a work in x days. y will do the same task
112 32

a
in 3x days. 8 hrs.

h
3x – x = 40 15 60

b
o
x = 20

o
1

b
y will finish the task in 60 days Due to leakage, filled part in one hour

.w
(x + y)’s 1 days work 8

o
part of cistern emptied, due to leakage in one hour
1 1 1

rd
= 15 1 1

p
20 60 15 th

re
Both of them will complete the work in 15 days. 112 8 112

s
s
In 112 hr, the leakage would empty the cistern.

.c
1 1 1 1

o
37. (c) Part filled by (A + B + C) in 1 hour = . 1 1 1

m
5 10 30 3 43. (b) In one min, (A + B) fill the cistern th
All the three pipes together will fill the tank in 10 15 6
3 hours.
38. (c) Let pipe A fills the cistern in x minutes. 3 1
In 3 min, (A + B) fill the cistern = th
Therefore, pipe B will fill the cistern in (x + 5) minutes. 6 2
1 1 1 1 1
Now, x = 10 Remaining part 1
x x 5 6 2 2
Thus, the pipes A and B can fill the cistern in
10 minutes and 15 minutes, respectively 1
th part filled by A in one min.
10
1 1 1 17
39. (c) Net part filled in 1 hour = – .
5 6 12 60 1 1
nd part filled by A in 10 5 min .
2 2
60 9
The tank will be full in hrs i.e., 3 hrs. Total time = 3 + 5 = 8 min.
17 17 44. (a) Work done by the waste pipe in 1 minutes
1 1 1 1 1
40. (d) Part filled by first tap in one min th
12 = [–ve sign means emptying]
20 12 15 10
1 Waste pipe will empty the full cistern in 10 minutes.
Part filled by second tap in one min th
18 45. (c) Let the required time be x hours, then
1 1 1 3 1 3 1 3
Now, 2 unfilled part = 1 x x x x x 1
12 18 12 4 15 4 12 4
13 x x x
unfilled part = th 1
18 16 60 48
1 x = 10 hours
th part of tank is filled by second tap in 1min. 46. (c) Proportion of the volume of the tank filled by both the
18
13 1 1 2
th part of tank is filled by second tap in 1 min. pipes in 4 min = 4 = rd of the tank.
18 15 10 3
Volume of the tank filled by all the pipes working
13
18 min = 13 min.
18 1 1 1 1
together =
41. (c) Let B can fill the cistern in x min. Then, 15 10 5 30
x 1
then A can fill the cistern in min i.e. tank is emptied in 1 min.
3 30
x 2 2 30
Given x 10 x 15 min rd of the tank can be emptied in = 20 min
3 3 3
42. (b) Cistern filled by both pipes in one hour 47. (a) Let cistern will be full in x min. Then,
1 1 15 part filled by B in x min + part filled by A in (x – 4)
= th min = 1
14 16 112
x x 4
112 1
Both pipes filled the cistern in hrs . 16 12
15
64 1
Now, due to leakage both pipes filled the cistern in x 9 hours.
7 7
y
o
u
Time and Work & Pipes and Cisterns B-135

rs
m
48. (a) Let A was turned off after x min. Then, 7 7 7 60
: ::1:x or x 1 2 hours.

a
cistern filled by A in x min + cistern filled by

h
60 30 30 7
B in (x + 23) min = 1

b
o
The tank will be full in (10 + 2) hrs = 12 hrs.

o
x x 23

b
1 1 1 9 3

.w
45 40 55. (c) (A + B)’s 1 hour’s work =
12 15 60 20

o
17x 207 360 x 9 min .

rd
49. (a) Let cistern will be full in x min. Then, 1 1 8 2

p
(A + C)'s 1 hour’s work =

re
part filled by A in x min + part filled by B in (x – 1) min 12 20 60 15

s
s
+ part filled by C in (x – 2)min = 1

.c
3 2 17

o
x x 1 x 2 Part filled in 2 hrs =
20 15 60

m
1
3 4 6
17 17
19 1 Part filled in 6 hrs = 3
9x 19 x 2 min 60 20
9 9
50. (c) If both the pumps are opened together, then the tank 17 3
Remaining part = 1
will be emptied because the working efficiency of pump 20 20
empting is more than that of the pump filling it. Thus in 3
1 min net proportion of the volume of tank filled Now, it is the turn of A and B and part is filled by
20
1 1 1 A and B in 1 hour.
= Total time taken to fill the tank = (6 + 1) hrs = 7 hrs.
8 16 16
56. (c) Work done by the waste pipe in 1 minute
or the tank will be emptied in 16 min
1 1 1 1 11 1
1 = .
tank will be emptied in 8 min. 15 20 24 15 120 40
2 [–ve sign means emptying]
51. (a) Let the filling capacity of pump be x m3/min.
Then, emptying capacity of pump = (x + 10) m3 /min. 1
Volume of part = 3 gallons.
40
2400 2400
8 Volume of whole = (3 × 40) gallons = 120 gallons.
x x 10
57. (d) Pipe 1 (Hot) 3 + X, X Pipe 2 (cold)
x2 + 10 x – 3000 = 0
(x – 50)(x + 60) = 0 x = 50 m3/min. X(X 3) 2
Together 6 min.
52. (c) Hint : Let the time be t hours after 1 a.m. 2X 3 3
t t 1 t 2 X(X 3) 2 20
0 = 6 min.
4 5 2 2X 3 3 3
40X + 60 = 3X (X + 3)
t t t 1
1 40X + 60 = 3X2 + 9X
4 5 4 5 3X2 – 31X – 60 = 0
t X = 12 minutes
16 hours from 1 a.m is 5 p.m 58. (a) Radius of the pipe (r) = 4 cm = 0.04 meter
53. (c) Suppose pipe A alone takes x hours to fill the tank. Volume of water flowing out per sec
x x = r2 × rate of flow
Then, pipes B and C will take and hours
2 4 22
= 0.042 3 cu meters 0.0151 cubic m
respectively to fill the tank. 7
1 2 4 1 7 1 8
x 35 hrs. Time taken to fill the tank = 40 × 30 × sec
x x x 5 x 5 0.0151
1 1 1 23 40 30 8 1
54. (a) Part filled in 10 hours = 10 . = hours 176.6 hours
15 20 25 30 0.01 3600
59. (c) Let h be the length of water column discharged in 1
23 7
Remaining part = 1 . hour or 1 minute.
30 30 Volume discharged by the 4 pipes = Volume discharged
1 1 7 by the single pipe.
(A + B)'s 1 hour’s work = . 4 × × (1.5)2 × h = × (r)2 × h
15 20 60
r2 = 9 r = 3, Diameter = 6 cm.
y
o
u
B-136 Time and Work & Pipes and Cisterns

rs
4n = 7n – 420

m
60. (a) Rate of admission of water

a
3n = 420
2 1

h
n = 140 workers

b
= tonnes / min = tonnes/ min
6 3

o
64. (b) Let the work (in units) done by a man and a woman

o
Rate of pumping out of water in one day be M and W respectively. Total work (in

b
.w
12 1 units) = 10(M + 6W) = 10M + 60W

o
= tonnes/min = tonnes/ min.
10 M 60W 10 M 60W

rd
60 5
=5

p
8W 2M

re
2 12
Rate of accumulation = tonnes / min.

s
6 60 5M 30W 5

s
=

.c
Time to accumulate 80 tonnes of water 4W M 2

o
m
Amount of water 80 M
= = = 600 min. = 10 hours On putting = x, we get
Accumulation rate 1 1 W
3 5 5x 30 5 M
x = 6 or =6
Average sailing rate so as avoid sinking 4 x 2 W
Distance 55 The efficiency of a man is greater than that of
= = km/ h = 5.5 km/h a woman by 500%.
Time 10
65. (b) Ratio of number of men, women and children
61. (b) Let B be closed after x minutes. Then, part filled by
(A + B) in x min. + part filled by A in (18 – x) min = 1. 18 10 12
= : : = 3x : 2x : 4x
6 5 3
1 1 1
x (18 x) =1 (3x + 2x + 4x) = 18
24 32 24 x=2
7 x 18 x Therefore, number of women = 4
or, 1 or, 7x + 4(18 – x) = 96
96 24 10
Share of all women = 4000 = ` 1000
or, 3x = 24 x = 8. 40
So, B should be closed after 8 min. ( 18 + 10 + 12 = 40)
Direct Formula:
1000
18 Share of each woman = ` 250
Pipe B should be closed after 1 32 = 8 min. 4
24 66. (b) Let B can finish the work in x days.
Then A can finish the work in (x – 3) days.
1
62. (b) A’s one day’s work th work 1
8 B’s one day’s work th work
x
1
B’s one day’s work rd work 1
3 A’s one day’s work th work
x 3
1 1
A’s 4 day’s work = 4 nd work 4
8 2 A’s 4 days’ work = th work
x 3
1 1 1
In next two days, total wall 2 2 4
x 7
2 8 3 Remaining work 1 th work
x 3 x 3
1 The remaining work done by B in 14 – 4 = 10 days.
th wall
12 x 7
Now, in 10 days, work done by B th work
1 11 x 3
Remaining wall 1 th
12 12 1 x 7
In 1 day, work done by B = th work
1 10 x 3
Now, th wall is built up by A in one day..
8
1 x 7 1
11 11 7 1 days and
th wall is built up by A in 8 = . 10 x 3 x
12 12 3
x 15days
63. (b) Go through option
140 × 4 = (140 + 120 + 100 + ... + 20) B will finish in 15 days and A will finish in 12 days
560 = 560 67. (b) Suppose large pump takes t hours to fill a tank
Alternatively: Let n be the initial number of worker 1
then 1 hour work of large pump fills part
t
n × 4 = n + (n – 20) + (n – 40) + ... + (n – 120)
y
o
u
Time and Work & Pipes and Cisterns B-137

rs
m
1 2 1 2

a
1 hour work of each small pump fills Remaining work = 1

h
t 3 3 3

b
o
1 2 3 2

o
1 hour work of all 4 pumps fill 3 In one hour (A + B) can do part of work

b
t 3t t 3 7

.w
o
3 1 1 1 1

rd
Therefore, part is filled by all 4 pumps in 1 hour
t C 6 B C

p
re
t t 1 1 2 3

s
Whole tank would be filled in 1 h this is

s
3 3 C 6 21 42

.c
o
1/3 of the time taken by large pump i.e., t hour C = 14 hours

m
68. (b) Let the inlets be A, B, C and D. 72. (c) Let Pratibha can finish the work in x days then, Sonia
A + B + C = 8.33 % can finish the same work in 3x days
B + C + D = 6.66% According to question
A + D = 5% 3x – x = 60
Thus 2A + 2B + 2C + 2D = 20% 2x = 60 x = 30
and A + B + C + D = 10% Pratibha and Sonia can individually complete the work
10 minutes would be required to fill the tank in 30 days and 90 days respectively.
completely. 1 1
69. (a) More the no. of men less time they take to complete 73. (c) (A + B)’s 1 day’s work ; C’s 1day’s work
10 50
work.
1 1 6 3
Let x men are added. (A + B + C)’s 1 day’s work ...(1)
10 50 10 25
75 18
(Inverse Proportion) Also, A’s 1 day’s work = (B + C)’s 1 day’s work ...(2)
75 x 90
3
75 1 From (1) and (2), we get : 2 × (A’s 1 day’s work)
25
75 x 5
3
375 – 75 = x A’s 1 day’s work
x = 300 50
th 1 3 2 1
1 B’s 1 day’s work –
70. (c) (A + B + C)'s 1 day's work = part of whole 10 50 50 25
40 So, B alone could do the work in 25 days.
work 1 1
16 2 74. (c) A’s 1 day’s work and B’s 1 day’s work
10 15
(A + B + C)'s 16 days work = of whole work
40 5 1 1 1
(B + C) completes remaining work in 40 days (B + C) (A + B)’s 1 day’s work
10 15 6
th
3 So both together will finish the work in 6 days.
completes part of work in 40 days. 75. (b) Let the required number of days be x.
5
Then, more men, more km (Direct proportion)
40 5 200 more days, more km (Direct proportion)
(B + C) completes whole work in days.
3 3 men
1 1 1 1 Men 12 : 28
::1.5 :12
A B C 40 Days 7 : x
1 3 1 12 × 7 × 12 = 28 × x × 1.5
A 200 40 12 7 12
x 24
1 1 1 4 28 1.5
A 40 200 200 1 1
76. (d) (x + y)’s 6 days’ work = 6 .
1 1 30 5
A 50 1 4
A alone can complete whole work in 50 days. Remaining work = 1
5 5
1 1 1 1
71. (c) 4
A B C 6 Now, work is done by y in 32 days.
5
2 1 5
(A + B + C) can do part of work in 2 days. Whole work will be done by y in 32 = 40 days.
6 3 4
y
o
u
B-138 Speed, Time & Distance (Trains, Boat & Stream)

rs
m
CHAPTER

a
h
11

b
SPEED, TIME & DISTANCE

o
o
b
.w
(TRAINS, BOAT & STREAM)

o
rd
p
re
s
s
.c
SPEED, TIME AND DISTANCE ab

o
Concept of speed, time and distance is based on the formula d = (t1 – t2)

m
b a
Speed × Time = Distance
Speed (s) : t1t 2
The rate at which any moving body covers a particular distance is d = (a – b)
t 2 t1
called its speed. If a body travels a distance ‘d’ from A to B with speed ‘a’ in
Distance time t1 and travels back from B to A i.e., the same distance
Speed ;
Time m
with of the usual speed ‘a’, then the change in time taken
Time (t) : n
It is the time duration over which the movement has occured. The to cover the same distance is given by :
unit used for measuring time is synchronous with denominator of m
the unit used for measuring speed. Thus, if the speed is measured Change in time = 1 × t ; for n > m
n 1
in terms of km/h then time is measured in hours.
Distance n
Time = ; = 1 × t1; for m > n
Speed m
Unit : SI unit of speed is metre per second (mps). It is also mea- If first part of the distance is covered at the rate of v1 in time
sured in kilometers per hour (kmph) or miles per hour (mph). t1 and the second part of the distance is covered at the rate of
Conversion of units : v1 t1 v2 t 2
(i) 1 hour = 60 minutes = 60 × 60 seconds. v2 in time t2, then the average speed is
t1 t 2
1 km = 1000 m
1 km = 0.625 mile Relative Speed : When two bodies are moving in same direction
1 mile = 1.60 km, i.e. 8 km = 5 miles with speeds S1 and S2 respectively, their relative speed is the
1 yard = 3 feet difference of their speeds.
1 foot = 12 inches i.e. Relative Speed = S1 – S2
When two bodies are moving in opposite direction with speeds S1
5
1 km/h m/sec, and S2 respectively, their relative speed is the sum of their speeds.
18
i.e. Relative Speed = S1 + S2
18
1 m/sec km/h
5 Example 1 :
22 The driver of a maruti car driving at the speed of 68 km/h
1 miles/hr ft/sec. locates a bus 40 metres ahead of him. After 10 seconds, the
15
bus is 60 metres behind. Find the speed of the bus.
Total Distance Solution :
Average speed
Total time Let speed of Bus = SB km/h.
While travelling a certain distance d, if a man changes his Now, in 10 sec., car covers the relative distance
speed in the ratio m : n, then the ratio of time taken becomes = (60 + 40) m = 100 m
n : m. 100
If a certain distance (d), say from A to B, is covered at ‘a’ Relative speed of car = = 10 m/s
10
km/hr and the same distance is covered again say from B to 18
A in ‘b’ km/hr, then the average speed during the whole = 10 36 km / h
journey is given by : 5
68 – SB 36
2ab
Average speed = km/h SB 32km / h
a b
... (which is the harmonic means of a and b Example 2 :
Also, if t1 and t2 is time taken to travel from A to B and B to If a person goes around an equilateral triangle shaped field
A, respectively, the distance ‘d’ from A to B is given by : at speed of 10, 20 and 40 kmph on the first, second and third
ab side respectively and reaches back to the starting point, then
d = (t1 + t2) find his average speed during the journey.
a b
y
o
u
Speed, Time & Distance (Trains, Boat & Stream) B-139

rs
m
Solution : Now the dog gains (16 – 12) or 4 km/h.

a
Let the measure of each side of triangle is D km. The person 60 300 1

h
The dog will gain 300 m in min or 4 min.

b
travelled the distance from A to B with 10 kmph, B to C 4 1000 2

o
with 20 kmph and C to A with 40 kmph.

o
1
Again, the distance gone by the hare in 4 min

b
.w
A 2
12 1000 1

o
4 900m

rd
60 2

p
Distance of the place where the hare is caught from the

re
D D
spot H where the hare took flight = 200 + 900 = 1100 m

s
s
If two persons (or vehicles or trains) start at the same time in

.c
opposite directions from two points A and B, and after

o
m
crossing each other they take x and y hours respectively to
B D C complete the journey, then
If TAB = Time taken by the person to travel from A to B,
Speed of first y
TBC = Time taken by the person to travel from B to C and
TCA = Time taken by the person to travel from C to A. Speed of second x
Then total time = TAB + TBC + TCA Example 5 :
D D D 8 4 2 7D A train starts from A to B and another from B to A at the
10 20 40
D same time. After crossing each other they complete their
80 40
Total distance travelled = D + D + D = 3D 1 4
journey in 3 and 2 hours respectively. If the speed of
Hence, average speed 2 7
3D 120 1 the first is 60 km/h, then find the speed of the second train.
17 kmph. Solution :
7D 7 7
40 4
Example 3 : 2
1st train 's speed y 7
Two guns were fired from the same place at an interval of 15 2nd train 's speed x 1
min, but a person in a bus approaching the place hears the 3
2
second report 14 min 30 sec after the first. Find the speed of
the bus, supposing that sound travels 330 m per sec. 18 2 6
Solution : 7 7 7
Distance travelled by the bus in 14 min 30 sec could be
60 6
travelled by sound in (15 min – 14 min 30 sec) = 30 second.
2nd train 's speed 7
1
Bus travels = 330 × 30 in 14 min. 2nd train’s speed = 70 km/h.
2
a
Speed of the bus per hour If new speed is of usual speed, then
b
330 30 2 60 99 12 1188 28
40 km / hr Change in time
29 1000 29 29 29 Usual time
Example 4 : b
1
A hare sees a dog 100 m away from her and scuds off in the a
opposite direction at a speed of 12 km/h. A minute later the
dog perceives her and gives chase at a speed of 16 km/h. Example 6 :
How soon will the dog overtake the hare and at what distance 3
from the spot where the hare took flight ? A boy walking at of his usual speed, reaches his school
5
Solution : 14 min late. Find his usual time to reach the school.
Suppose the hare at H sees the dog at D. Solution :
14 14 3
Usual time = 21 min
D K 5 2
H 1
DH = 100 m 3
Let K be the position of the hare where the dog sees her. Example 7 :
HK = the distance gone by the hare in 1 min A train after travelling 50 km, meets with an accident and
4
12 1000 then proceeds at of its former rate and arrives at the
1m = 200 m 5
60
terminal 45 minutes late. Had the accident happened 20 km
DK = 100 + 200 = 300 m further on, it would have arrived 12 minutes sooner. Find
The hare thus has a start of 300 m. the speed of the train and the distance.
y
o
u
B-140 Speed, Time & Distance (Trains, Boat & Stream)

rs
m
Solution : A man leaves a point A at t1 and reaches the point B at t2.

a
Let A be the starting place, B the terminal, C and D the Another man leaves the point B at t 3 and reaches the point

h
places where the accidents to be placed. A at t4, then they will meet at

b
o
o
t 2 – t1 t 4 – t1

b
A C D B t1

.w
4 t 2 – t1 t4 – t3
5

o
By travelling at of its original rate the train would take

rd
5 4 Example 10 :

p
A bus leaves Ludhiana at 5 am and reaches Delhi at 12

re
1
of its usual time, i.e., of its original time more. noon. Another bus leaves Delhi at 8 am and reaches Ludhiana

s
4

s
at 3 pm. At what time do the buses meet?

.c
1
Solution :

o
of the usual time taken to travel the distance

m
4 Converting all the times into 24 hour clock time, we get 5 am
CB = 45 min. ...(i) = 500, 12 noon = 1200, 8 am = 800 and 3 pm = 1500
1 (1200 – 500)(1500 – 500)
and of the usual time taken to travel the distance Required time 500
4 (1200 – 500) (1500 – 800)
DB = (45 – 12) min ...(ii)
Subtracting (ii) from (i), 700 1000
00 = 1000 = 10 am.
1 700 700
of the usual time taken to travel the distance CD = 12 min. Relation between time taken with two different modes of
4
Usual time taken on travel 20 km = 48 min. transport : t2x + t2y = 2(tx + ty)
20 where,
Speed of the train per hour 60 or 25 km/h. tx = time when mode of transport x is used single way.
48
ty = time when mode of transport y is used single way.
From (i), we have
t2x = time when mode of transport x is used both ways.
Time taken to travel CB = 45 × 4 min = 3 hrs.
t2y = time when mode of transport y is used both ways.
The distance CB = 25 × 3 or 75 km.
Hence the distance AB = the distance (AC + CB) Example 11 :
= 50 + 75 or 125 km. A man takes 6 hours 30 min. in going by a cycle and coming
A man covers a certain distance D. If he moves S1 speed back by scooter. He would have lost 2 hours 10 min by
faster, he would have taken t time less and if he moves S2 going on cycle both ways. How long would it take him to go
speed slower, he would have taken t time more. The original by scooter both ways ?
speed is given by Solution :
2 S1 S2 Clearly, time taken by him to go by scooter both way
S2 S1 1
Example 8 : = 6h.30m – 2h.10m = 4h.20m = 4 h
3
A man covers a certain distance on scooter. Had he moved
Example 12 :
3 km/h faster, he would have taken 20 min less. If he had
A man travels 120 km by ship, 450 km by rail and 60 km by
moved 2 km/h slower, he would have taken 20 min more.
horse taking altogether 13 hrs 30 min. The speed of the train
Find the original speed.
is 3 times that of the horse and 1½ times that of the ship.
Solution :
Find the speed of the train.
2 3 2 Solution :
Speed 12km / hr.
3–2 If the speed of the horse is x km/hr; that of the train is 3x and
If a person with two different speeds U & V cover the same 3x
distance, then required distance that of the ship is = 2x km/hr

U V
Difference between arrival time 120 450 60 27
U V + + =
U V 2x 3x x 2
Also, required distance otal time taken
U V 60 150 60 27 270 27
Example 9 : + + = =
x x x 2 x 2
A boy walking at a speed of 10 km/h reaches his school 12
x = 20 Speed of the train = 60 km/hr..
min late. Next time at a speed of 15 km/h reaches his school
7 min late. Find the distance of his school from his house ? Example 13 :
Solution : Rajesh travelled from the city A to city B covering as much
Difference between the time = 12 – 7 = 5 min
5 1
hr
distance in the second part as he did in the first part of his
60 12 journey. His speed during the second part was twice his
15 10 1 150 1 speed during the first part of the journey. What is his average
Required distance = 2.5 km
15 10 12 5 12 speed of journey during the entire travel ?
y
o
u
Speed, Time & Distance (Trains, Boat & Stream) B-141

rs
m
(1) His average speed is the harmonic mean of the individual Example 16 :

a
speed for the two parts. Two cyclists start from the same place to ride in the same

h
b
(2) His average speed is the arithmetic mean of the individual direction. A starts at noon at 8 kmph and B at 1.30 pm at

o
speed for the two parts. 10kmph. How far will A have ridden before he is overtaken

o
b
(3) His average speed is the geometric mean of the individual by B ? Find also at what times A and B will be 5 km apart.

.w
speeds for the two parts. Solution :

o
If A rides for X hours before he is overtaken, then B rides for

rd
(4) Cannot be determined.
(X – 15) hrs.

p
Solution :

re
The first part is ½ of the total distance & the second part is 8X = 10 (X – 1.5) X = 7.5

s
s
½ of the total distance. Suppose, he travels at a km/hr speed A will have ridden 8 × 7.5 km or 60 km.

.c
during the first half & b km/hr speed during the second half. For the second part, if Y = the required number of hours after

o
m
When distance travelled is the same in both parts of the noon, then
2ab 8X = 10 (X – 1.5) ± 5
journey, the average speed is gives by the formula i.e. X = 10 or 5 according as B is ahead or behind A.
a b
the harmonic mean of the two speeds. The required times are 5 p.m. and 10 p.m.
Example 14 : Example 17 :
Two friends X and Y walk from A to B at a distance of 39 km, Two men A and B start from a place P walking at 3 kmph and
3½ kmph respectively. How many km apart will they be at the
1 end of 2½ hours ?
at 3 km an hour and 3 km an hour respectively. Y reaches B,
2 (i) If they walk in opposite directions ?
returns immediately and meet x at C. Find the distance from A (ii) If they walk in the same direction ?
to C . (iii) What time will they take to be 16 km apart if.
Solution : (a) they walk in opposite directions ?
When Y meets X at C, Y has walked the distance AB + BC (b) in the same direction ?
and X has walked the distance AC. Solution :
Y
(i) When they walk in opposite directions, they will be
1 1
3 3 6 km apart in 1 hour..
2 2
A B
C
1 1 5 1
y ln 2 hours they will be 6 16 km apart.
2 2 2 4
X
(ii) If they walk in the same direction, they will be
So, both X and Y have walked together a distance 1 1
= 2 × AB = 2 × 39 = 78 km . 3 3 km apart in 1 hour..
2 2
1 6 1 1 5 1
The ratio of the speeds of X and Y is 3 : 3 i.e, ln 2 hours they will be 1 km apart.
2 7 2 2 2 4
6 (iii) Time to be 16 km apart while walking in opposite
Hence , the distance travelled by X = AC = × 78 = 36 km directions
6 7
Example 15 : 16 6
= 2 hours.
A man rides one-third of the distance from A to B at the rate 1 13
3 3
of ‘a’ kmph and the remainder at the rate of ‘2b’ kmph. If he 2
had travelled at the uniform rate of 3c kmph, he could have
rode from A to B and back again in the same time. Find a But if they walk in the same direction,
relationship between a, b and c.
16
Solution : time = 32 hours
1
Let the distance between A and B is X km and T 1 and T2 be 3 3
the time taken, then 3

X 2X X X a b TRAINS
T1 , T2 , T1 T2 A train is said to have crossed an object (stationary or moving)
3a 6b 3b 3 ab
only when the last coach of the train crosses the said object
2X completely. It implies that the total length of the train has crossed
Let T3 be the time taken in third case, then T3
3c the total length of the object.
2X X 2 a b 2ab Time taken by a train to cross a pole/a standing man
(a b) c
3c 3ab c ab a b Length of train
.
Speed of train
y
o
u
B-142 Speed, Time & Distance (Trains, Boat & Stream)

rs
m
Time taken by a train to cross platform/bridge etc. (i.e. a (d) another train 160 m long running at 36 km/h in same

a
stationary object with some length) direction ?

h
b
length of train + length of platform/bridge etc. (e) another train 160 m long running at 36 km/h in opposite

o
. direction ?

o
speed of train

b
(f) a man running at 6 km/h in same direction ?

.w
When two trains with lengths L1 and L2 and with speeds S1 (g) a man running at 6 km/h in opposite direction ?

o
and S2 respectively, then

rd
Solution :
(a) When they are moving in the same direction, time taken

p
(a) The statue is a stationary object, so time taken by train

re
by the faster train to cross the slower train
is same as time taken by train to cover a distance equal

s
L1 L2

s
. to its own length.

.c
difference of their speeds

o
5

m
Now, 54 km/h 54 15m / s
(b) When they are moving in the opposite direction, time 18
taken by the trains to cross each other 90
Required time = 6 sec.
L1 L 2 15
.
sum of their speeds (b) The platform is stationary of length = 120 m.
Suppose two trains or two bodies are moving in the same Length to be covered
direction at u km/hr and v km/hr respectively such that u > v, = Length of the train + Length of the platform
then = 90 + 120 = 210 m
their relative speed = (u – v) km/hr. 210
If their lengths be x km and y km respectively, then time Required time = 14 sec.
15
taken by the faster train to cross the slower train (moving in
(c) Length to be covered
x y = Length of the train + length of the other train
the same direction) = hrs.
u v = 90 + 150 = 240 m.
Suppose two trains or two bodies are moving in opposite 240
directions at u km/hr and v km/hr, then their relative speed Required time = 16 sec.
15
= (u + v) km/hr. (d) Another train is moving in same direction.
If their lengths be x km & y km, then : Length to be covered
x y = Length of the train + length of the other train
time taken to cross each other = hrs. = 90 + 160 = 250 m
u v
Relative speed = 54 – 36 = 18 kmph.
If a man is running at a speed of u m/sec in the same direction
250
in which a train of length L meters is running at a speed Required time 50 sec.
5
v m/sec, then (v – u) m/sec is called the speed of the train 18
18
relative to man. Then the time taken by the train to cross the
(e) Another train is moving in opposite direction.
1 Length to be covered
man = seconds
v u = Length of the train + length of the other train
If a man is running at a speed of u m/sec in a direction opposite = 90 + 160 = 250 m
to that in which a train of length L meters is running with a Relative speed = 54 + 36 = 90 kmph
speed v m/sec, then (u + v) is called the speed of the train 250
relative to man. Required speed 10 sec .
5
Then the time taken by the train to cross the man 90
18
1 (f) The man is moving in same direction,
= seconds. so Length to be covered = Length of the train,
v u
and relative speed = speed of train – speed of man
If two trains start at the same time from two points A and B
towards each other and after crossing, they take a and b 90
Required time
hours in reaching B and A respectively. Then, 5
(54 – 6)
18
A’s speed : B’s speed = b: a .
90 27 3
3 6 sec.
Example 18 : 40 4 4
How long does a train 90 m long running at the rate of (g) The man is moving in opposite direction, so
54 km/h take to cross – Length to be covered = Length of the train, and
(a) a Mahatma Gandhi’s statue ? relative speed = speed of train + speed of man
(b) a platform 120 m long ? Required time
90 27 2
5 sec.
(c) another train 150m long, standing on another parallel 5 5 5
(54 6)
track ? 18
y
o
u
Speed, Time & Distance (Trains, Boat & Stream) B-143

rs
m
Example 19 : Let the speed of a boat (or man) in still water be X m/sec

a
Two trains of equal lengths are running on parallel tracks in and the speed of the stream (or current) be Y m/sec. Then,

h
b
the same direction at 46 km/h and 36 km/h, respectively. The Speed of boat with the stream (or downstream or D/S)

o
faster train passes the slower train in 36 sec. Find the length

o
= (X + Y) m/sec.

b
of each train is ?

.w
Speed of boat against the stream (or upstream or U/S)
Solution :

o
= (X – Y) m/sec.

rd
Let the length of each train be x metres.
Speed of boat in still water is

p
Then, the total distance covered = (x + x) = 2x m

re
(X Y) (X – Y) Downstream Upstream

s
10 5 X

s
Relative speed = (46 – 36) = 10 km/h = m/s 2 2

.c
18

o
m
(X Y) – (X – Y)
2 x 18 Speed of the stream or current is Y
Now, 36 or x = 50 m 2
50
Downstream Upstream
Example 20 :
2
A train 110 m in length travels at 60 km/h. How much time
does the train take in passing a man walking at 6 km/h against Example 22 :
the train ? A boat is rowed down a river 28 km in 4 hours and up a river
12 km in 6 hours. Find the speed of the boat and the river.
Solution :
Solution :
Relative speeds of the train and the man
Downstream speed is 28 7 kmph
66 5 4
= (60 + 6) = 66 km/h = m/s
18 12
Upstream speed is = 2 kmph
Distance = 110 m 6
Therefore, time taken in passing the men 1
Speed of Boat (Downstream + Upstream Speed)
2
110 18
= 6s 1
66 5 [7 2] 4.5 kmph
2
Example 21 :
Two trains 137 metres and 163 metres in length are running 1
Speed of current (Downstream–Upstream speed)
towards each other on parallel lines, one at the rate of 42 2
kmph and another at 48 kmph. In what time will they be clear 1
of each other from the moment they meet? (7 – 2) 2.5 kmph
2
Solution : Example 23 :
Relative speed of the trains P, Q, and R are the three towns on a river which flows
= (42 + 48) kmph = 90 kmph uniformly. Q is equidistant from P and R. I row from P to Q
and back in 10 hours and I can row from P to R in 4 hours.
5 Compare the speed of my boat in still water with that of the
= 90 m/sec = 25 m/sec.
18 river.
Time taken by the trains to pass each other Solution :
= Time taken to cover (137 + 163) m at 25 m/sec Let the speed of the boat be v1 and the speed of the current
be v2 and d be the distance between the cities.
300
= sec = 12 seconds. d d
25 Now, 4 and 6
v1 v 2 v1 v 2
BOAT & STREAM
Stream : It implies that the water in the river is moving or flowing. v1 v 2 6
Upstream : Going against the flow of the river. v1 v 2 4
Downstream : Going with the flow of the river.
Still water : It implies that the speed of water is zero (generally, in 2v1 10 v
or or 1 5 :1
a lake). 2v2 2 v2
Flow of water Required ratio = (5 + 1) : 5 = 6 : 5
A man can row X km/h in still water. If in a stream which is
Boat flowing of Y km/h, it takes him Z hours to row to a place and
Up stream Down stream
Boat Z(X 2 – Y 2 )
back, the distance between the two places is
2X
y
o
u
B-144 Speed, Time & Distance (Trains, Boat & Stream)

rs
m
Example 24 : Solution :

a
A man can row 6 km/h in still water. When the river is running Let the width of the river be x.

h
b
at 1.2 km/h, it takes him 1 hour to row to a place and back. Let a, b be the speeds of the ferries.

o
How far is the place ?

o
720 (x 720)

b
Solution :

.w
a b ...(i)
Man’s rate downstream = (6 + 1.2) = 7.2 km/h.

o
(x 720) 400 720 (x 400)

rd
Man’s rate upstream = (6 – 1.2) km/h = 4.8 km/h. 10 10 ... (ii)

p
Let the required distance be x km. a a b b

re
(Time for ferry 1 to reach other shore + 10 minute wait + time

s
x x

s
Then = 1 or 4.8x + 7.2x = 7.2 × 4.8 to cover 400m)

.c
7.2 4.8
= Time for freely 2 to cover 720m to other shore + 10 minute

o
m
7.2 4.8 wait + Time to cover (x – 400m))
x 2.88km
12
a 720
By direct formula : Using (i), we get
b (x 720)
1 62 – (1.2) 2 (x 320) (x 320) a (x 320)
Required distance Using (ii),
2 6 a a b (x 320)

36 1.44 34.56 On, solving we get, x = 1760m


2.88 km Example 27 :
12 12
A man rows 27 km with the stream and 15 km against the
A man rows a certain distance downstream in X hours and stream taking 4 hours each time. Find this rate per hour in
returns the same distance in Y hours. If the stream flows at still water and the rate at which the stream flows.
the rate of Z km/h, then the speed of the man in still water is Solution :
given by
27 3
Z(X Y) Speed with the stream = 6 kmph
km / hr 4 4
Y–X
And if speed of man in still water is Z km/h then the speed of 15 3
Speed against the stream = 3 kmph.
stream is given by 4 4
Z(Y – X) 1 3 3 1
km / hr Speed of the man in still water = 6 3 5 kmph
X Y 2 4 4 4
Example 25 : 1 3 3
Vikas can row a certain distance downstream in 6 hours and Speed of the stream = 6 3 1.5 kmph
2 4 4
return the same distance in 9 hours. If the stream flows at
Example 28 :
the rate of 3 km/h, find the speed of Vikas in still water.
Solution : On a river, B is between A and C and is also equidistant from
A and C. A boat goes from A to B and back in 5 hours 15
By the formula,
minutes and from A to C in 7 hours. How long will it take to
3 9 6 go from C to A if the river flows from A to C ?
Vikas’s speed in still water = 15 km/h
9–6 Solution :
If the speed in still water is x kmph and speed of the river is
If a man capable of rowing at the speed u of m/sec in still y kmph, speed down the river = x + y and speed up the river
water, rows the same distance up and down a stream flowing x – y.
at a rate of v m/sec, then his average speed through the d d 1
5 ............. (1)
journey is x y x y 4
Upstream Downstream (u v) (u v) 2d
= = 7 ............. (2)
Man 's rate in still water u x y
Example 26 :
2d 2d 1
Two ferries start at the same time from opposite sides of a Multiplying (1) by 2, we get 10
river, travelling across the water on routes at right angles to x y x y 2
the shores. Each boat travels at a constant speed though
2d 21 2d
their speeds are different. They pass each other at a point 7 7
720m from the nearer shore. Both boats remain at their sides x y 2 x y
for 10 minutes before starting back. On the return trip they
2d 1
meet at 400m from the other shore. Find the width of the 3 hours = Time taken to travel from C to A.
x y 2
river.
y
o
u
Speed, Time & Distance (Trains, Boat & Stream) B-145

rs
m
a
h
b
o
o
b
.w
1. An aeroplane flies along the four sides of a square at the 9. A train does a journey without stoppage in 8 hours, if it had

o
speeds of 200, 400, 600 and 800 km/h. Find the average speed travelled 5 km/h faster, it would have done the journey in 6

rd
of the plane around the field. hours 40 minutes. Find its original speed.

p
re
(a) 384 km/h (b) 370 km/h (a) 25 km/h (b) 40 km/h

s
s
(c) 368 km/h (d) None of these (c) 45 km/h (d) 36.5 km/h

.c
2. A monkey ascends a greased pole 12 metres high. He

o
10. A train leaves station X at 5 a.m. and reaches station Y at 9

m
ascends 2 metres in first minute and slips down 1 metre in a.m. Another train leaves station Y at 7 a.m. and reaches
the alternate minute. In which minute, he reaches the top ? station X at 10: 30 a.m. At what time do the two trains cross
(a) 21st (b) 22nd each other ?
(c) 23rd (d) 24th (a) 7 : 36 am (b) 7 : 56 am
1 (c) 8 : 36 am (d) 8 : 56 am
3. A man walks a certain distance and rides back in 6 h. He
4 11. Cars C1 and C2 travel to a place at a speed of 30 km/h and 45
3 1
can walk both ways in 7 h. How long it would take to ride km/h respectively. If car C2 takes 2 hours less time than
4 2
both ways ? C1 for the journey, the distance of the place is
1 (a) 300 km (b) 400 km
(a) 5 hours (b) 4 hours (c) 350 km (d) 225 km
2
12. A man covers a certain distance on a toy train. If the train
3 moved 4 km/h faster, it would take 30 minutes less. If it
(c) 4 hours (d) 6 hours
4 moved 2 km/h slower, it would have taken 20 minutes more.
4. There are 20 poles with a constant distance between each Find the distance.
pole. A car takes 24 seconds to reach the 12th pole . How (a) 60 km (b) 58 km
much time will it take to reach the last pole? (c) 55 km (d) 50 km
(a) 25.25 s (b) 17.45 s 13. A goods train leaves a station at a certain time and at a fixed
(c) 35.75 s (d) 41.45 s speed. After 6 hours, an express train leaves the same station
5. A man is walking at a speed of 10 km per hour. After every and moves in the same direction at a uniform speed of 90
kilometre, he takes rest for 5 minutes. How much time will he kmph. This train catches up the goods train in 4 hours. Find
take to cover a distance of 5 kilometres? the speed of the goods train.
(a) 48 min. (b) 50 min. (a) 36 kmph (b) 40 kmph
(c) 45 min. (d) 55 min.
(c) 30 kmph (d) 42 kmph
6. On a journey across Bombay, a tourist bus averages
10 km/h for 20% of the distance, 30 km/h for 60% of it and 20 14. Without stoppages, a train travels certain distance with an
km/h for the remainder. The average speed for the whole average speed of 80 km/h, and with stoppages, it covers the
journey was same distance with an average speed of 60 km/h. How many
(a) 10 km/h (b) 30 km/h minutes per hour the train stops ?
(c) 5 km/h (d) 20 km/h (a) 15 (b) 18
7. In a 800 m race around a stadium having the circumference (c) 10 (d) None of these
of 200 m, the top runner meets the last runner on the 5th 15. If a man walks to his office at 3/4 of his usual rate, he reaches
minute of the race. If the top runner runs at twice the speed office 1/3 of an hour later than usual. What is his usual time
of the last runner, what is the time taken by the top runner to to reach office.
finish the race ? 1
(a) 20 min (b) 15 min (a) hr (b) 1 hr
2
(c) 10 min (d) 5 min
3
8. A man walks half of the journey at 4 km/h by cycle does one (c) hr (d) None of these
third of journey at 12 km/h and rides the remainder journey 4
in a horse cart at 9 km/h, thus completing the whole journey 16. A train running between two stations A and B arrives at
in 6 hours and 12 minutes. The length of the journey is its destination 10 minutes late when its speed is 50 km/h
and 50 minutes late when its speed is 30km/h. What is the
1332 distance between the stations A and B ?
(a) 36 km (b) km
67 (a) 40 km (b) 50 km
(c) 40 km (d) 28 km (c) 60 km (d) 70 km
y
o
u
B-146 Speed, Time & Distance (Trains, Boat & Stream)

rs
m
17. A thief goes away with a Maruti car at a speed of 40 km/h. 27. A person can swim in still water at 4 km/h. If the speed of

a
The theft has been discovered after half an hour and the water is 2 km/h, how many hours will the man take to swim

h
owner sets off in another car at 50 km/h. When will the back against the current for 6 km.

b
o
owner overtake the thief from the start. (a) 3 (b) 4

o
b
1 1

.w
(a) 2 hours (b) 2 hr 20 min (c) 4 (d) Insufficient data
2

o
2

rd
(c) 1 hr 45 min (d) cannot be determined 28. A boat running downstream covers a distance of 16 km in 2

p
18. A starts 3 min after B for a place 4.5 km away. B on reaching hours while for covering the same distance upstream, it takes

re
his destination, immediately returns back and after walking 4 hours. What is the speed of the boat in still water?

s
s
a km meets A. If A walks 1 km in 18 minutes then what is B’s (a) 4 km/h (b) 6 km/h

.c
(c) 8 km/h (d) Data inadequate

o
speed ?

m
(a) 5 km/h (b) 4 km/h 29. A boat goes 24 km upstream and 28 km downstream in 6
(c) 6 km/h (d) 3.5 km/h hours. It goes 30km upstream and 21 km downstream in 6
19. A long distance runner runs 9 laps of a 400 metres track hours and 30 minutes. The speed of the boat in still water is :
everyday. His timings (in minutes) for four consecutive days (a) 10 km/h (b) 4 km/h
are 88, 96, 89 and 87 resplectively. On an average, how many (c) 14 km/h (d) 6km/h
metres/minute does the runner cover ? 30. A man who can swim 48 m/min in still water swims 200 m
(a) 40 m/min (b) 45 m/min against the current and 200 m with the current. If the difference
(c) 38 m/min (d) 49 m/min between those two times is 10 minutes, find the speed of the
20. It takes eight hours for a 600 km journey, if 120 km is done current.
by train and the rest by car. It takes 20 minutes more, if 200 (a) 30 m/min (b) 29 m/min
km is done by train and the rest by car. The ratio of the (c) 31 m/min (d) 32 m/min
speed of the train to that of the speed of the car is : 31. A circular running path is 726 metres in circumference. Two
(a) 4 : 3 (b) 3 : 4 men start from the same point and walk in opposite directions
(c) 3 : 2 (d) 2 : 3 at 3.75 km/h and 4.5 km/h, respectively. When will they meet
21. In a flight of 600 km, an aircraft was slowed down due to bad for the first time ?
weather. Its average speed for the trip was reduced by 200 (a) After 5.5 min (b) After 6.0 min
km/ hr and the time of flight increased by 30 minutes. The (c) After 5.28 min (d) After 4.9 min
duration of the flight is: 32. A train after travelling 150 km meets with an accident and
(a) 1 hours (b) 2 hours then proceeds with 3/5 of its former speed and arrives at its
(c) 3 hours (d) 4 hours destination 8 h late. Had the accident occurred 360 km further,
22. Points A and B are 70 km apart on a highway. One car starts it would have reached the destination 4 h late. What is the
form A and the another one from B at the same time. If they total distance travelled by the train?
travel in the same direction, they meet in 7 hours. But if they (a) 840 km (b) 960 km
travel towards each other, they meet in one hour. The speeds (c) 870 km (d) 1100 km
of the two cars are, respectively. 1
(a) 45 and 25 km/h (b) 70 and 10 km/h 33. A man swimming in a steam which flows 1 km/hr., finds
2
(c) 40 and 30 km/h (d) 60 and 40 km/h that in a given time he can swim twice as far with the stream
23. Anil calculated that it will take 45 minutes to cover a distance as he can against it. At what rate does he swim?
of 60 km by his car. How long will it take to cover the same 1 1
distance if the speed of his car is reduced by 15 km/hr? (a) 5 km/hr (b) 4 km/hr
2 2
(a) 36 min (b) 55.38 min
(c) 48 min (d) 40 min 1
(c) 7 km/hr (d) None of these
24. The jogging track in a sports complex is 726 metres in 2
circumference. Pradeep and his wife start from the same 34. Two persons start from the opposite ends of a 90 km straight
point and walk in opposite directions at 4.5 km/h and 3.75 track and run to and fro between the two ends. The speed of
km/h, respectively. They will meet for the first time in : first person is 30 m/s and the speed of other is 125/6 m/s.
(a) 5.5 min (b) 6.0 min They continue their motion for 10 hours. How many times
(c) 5.28 min (d) 4.9 min they pass each other?
25. A river 3 m deep and 40 m wide is flowing at the rate of 2 km (a) 10 (b) 9
per hour. How much water (in litres) will fall into the sea in a (c) 12 (d) None of these
minute? 35. A man starts from B to K, another from K to B at the same
(a) 4,00,000 (b) 40,00,000 time. After passing each other they complete their journeys
(c) 40,000 (d) 4,000
1 4
26. The speed of a boat in still water is 15 km/h and the rate of in 3 and 4 hours, respectively. Find the speed of the
stream is 5 km/h. The distance travelled downstream in 24 3 5
minutes is second man if the speed of the first is 12 km/hr.
(a) 4 km (b) 8 km (a) 12.5 kmph (b) 10 kmph
(c) 6 km (d) 16 km (c) 12.66 kmph (d) 20 kmph
y
o
u
Speed, Time & Distance (Trains, Boat & Stream) B-147

rs
m
36. A train 100 metres long moving at a speed of 50 km/hr. crosses 42. If a train runs at 70 km/hour, it reaches its destination late by

a
a train 120 metres long coming from opposite direction in 6 12 minutes. But if it runs at 80 km/hour, it is late by 3 minutes.

h
sec. The speed of the second train is The correct time to cover the journey is

b
o
(a) 60 km/hr. (b) 82 km/hr. (a) 58 minutes (b) 2 hours

o
b
(c) 70 km/hr. (d) 74 km/hr. (c) 1 hour (d) 59 minutes

.w
37. A passenger sitting in a train of length 100 m, which is 43. A car covers four successive 6 km stretches at speeds of

o
running with speed of 60 km/h passing through two bridges,

rd
25 kmph, 50 kmph, 75 kmph and 150 kmph respectively. Its
notices that he crosses the first bridge and the second bridge

p
average speed over this distance is

re
in time intervals which are in the ratio of 7 : 4 respectively. If (a) 25 kmph (b) 50 kmph

s
the length of first bridge be 280 m, then the length of second

s
(c) 75 kmph (d) 150 kmph

.c
bridge is:

o
44. A is faster than B. A and B each walk 24 km. The sum of their

m
(a) 490 m (b) 220 m
speed is 7 km/hr and the sum of time taken is 14 hours. A’s
(c) 160 m (d) Can’t be determined
speed is
38. A man can row a certain distance against the stream in six
hours. However, he would take two hours less to cover the (a) 4 km/hr (b) 3 km/hr
same distance with the current. If the speed of the current is (c) 5 km/hr (d) 7 km/hr
2 kmph, then what is the rowing speed in still water? 45. In a 1 km race A beats B by 40 meters or 7 seconds. Find A’s
(a) 10 kmph (b) 12 kmph time over the course.
(c) 14 kmph (d) 8 kmph (a) 172 sec (b) 150 sec
39. A boat, while going downstream in a river covered a dis- (c) 160 sec (d) 168 sec
tance of 50 mile at an average speed of 60 miles per hour. 46. A hare sees a dog 100 meters away from her and scuds off in
While returning, because of the water resistance, it took the opposite direction at the speed of 12 km/hr. A minute
one hour fifteen minutes to cover the same distance . What later the dog sees her and chases her at a speed of 16 km/hr.
was the average speed of the boat during the whole jour- How soon will the dog overtake her ?
ney? (a) 240 sec (b) 360 sec
(a) 40 mph (b) 48 mph (c) 270 sec (d) 180 sec
(c) 50 mph (d) 55 mph
47. On reducing my speed to 3 km/hr. I reach office 10 minutes
40. Two trains, 130 m and 110 m long, are going in the same
late. I usually travel at 133% of this speed and reach early
direction. The faster train takes one minute to pass the other
by the same margin. How far is my office ?
completely. If they are moving in opposite directions, they
(a) 4 km (b) 6 km
pass each other completely in 3 seconds. Find the speed of
each train. (c) 3.5 km (d) 4.5 km
(a) 38 m/sec, 36 m/sec (b) 42 m/sec, 38 m/sec 48. A monkey climbs a slippery pole 12 m height rises 1 m in
(c) 36 m/sec, 42 m/sec (d) None of these every 1 min and slips ½ metre in every next minute. Find
41. A passenger sitting in a train of length 100 m, which is how soon it will reach the top.
running with speed of 60 km/h passing through two bridges, (a) 45 min (b) 40 min
notices that he crosses the first bridge and the second bridge (c) 35 min (d) 48 min
in time intervals which are in the ratio of 7 : 4 respectively. If 49. It takes 8 hours for a 600 km journey, if 120 km is done by
the length of first bridge be 280 m, then the length of second train and the rest by car. It takes 20 minutes more if 200 km is
bridge is: down by train and the rest by car. The ratio of the speed of
(a) 490 m (b) 220 m the train to that of the car is
(c) 160 m (d) Can’t be determined (a) 2 : 3 (b) 3 : 2
(c) 3 : 4 (d) 4 : 3

ANSWER KEY
1 (a) 11 (d) 21 (a) 31 (c) 41 (c)
2 (a) 12 (a) 22 (c) 32 (c) 42 (c)
3 (c) 13 (a) 23 (b) 33 (b) 43 (b)
4 (d) 14 (a) 24 (c) 34 (c) 44 (b)
5 (b) 15 (b) 25 (b) 35 (b) 45 (a)
6 (d) 16 (b) 26 (b) 36 (b) 46 (d)
7 (c) 17 (a) 27 (a) 37 (c) 47 (c)
8 (a) 18 (a) 28 (b) 38 (a) 48 (a)
9 (a) 19 (a) 29 (a) 39 (b) 49 (a)
10 (b) 20 (b) 30 (d) 40 (b)
y
o
u
B-148 Speed, Time & Distance (Trains, Boat & Stream)

rs
m
a
h
b
o
o
b
.w
1. (a) Let each side of the square be x km and let the average
31

o
speed of the plane around the field be y km/h. Then, total time taken to complete journey hr

rd
Then, 5

p
re
x x x x 4x 31
t1 t2 t3

s
s
200 400 600 800 y 5

.c
o
x 1 x 1 x 31

m
25x 4x 2400 4
y 384. 2 4 3 12 6 9 5
2400 y 25
Average speed = 384 km/h. 31 216
x 36.2km 36km
2. (a) In 2 minutes, he ascends = 1 metre 5 37
10 metres, he ascends in 20 minutes. 9. (a) Let original speed = S km/h
He reaches the top in 21st minute. Here, distance to be covered is constant
3. (c) We know that, the relation in time taken with two
different modes of transport is 20
S 8 (S 5)
twalk both + tride both = 2 (twalk + t ride) 3
31 25 20 100
t ride both 2 100
4 4 8S S S 25 km / h
3 3 4
25 31 19 3 10. (b) Let the distance between X and Y be x km. Then, the
t ride both – 4 hrs
2 4 4 4 x 2x
4. (d) Let the distance between each pole be x m. speed of A is km/h and that of B is km/h.
4 7
Then, the distance up to 12th pole = 11 x m
2x x km x
11x 7
km / h km / h
Speed = m/s X Y 4
24 Relative speeds of the trains
Time taken to covers the total distance of 19x
x 2x 15x
19 x 24 = km / h
= 41.45 s 4 7 28
11x
Therefore the distance between the trains at 7 a.m. =
5. (b) Rest time = Number of rest × Time for each rest
= 4 × 5 = 20 minutes x x
Total time to cover 5 km x km
2 2
5 Hence, time taken to cross each other
60 minutes + 20 minutes = 50 minutes.
10 x
6. (d) Let the average speed be x km/h. 2 x 28 14
= 60 56 min
and total distance be y km. 15x 2 15x 15
Then, 28
0.2 0.6 0.2 y Thus, both of them meet at 7 : 56 a.m.
y y y
10 30 20 x 11. (d) Let C1 takes t hrs. Then,
Distance is same.
1
x 20km / h 5
0.05 30t 45 t
7. (c) After 5 minutes (before meeting), the top runner covers 2
2 rounds i.e., 400 m and the last runner covers 1 round 15
i.e., 200 m. t hrs
2
Top runner covers 800 m race in 10 minutes.
8. (a) Let the length of the journey =x km. 15
Distance = 30× 225 km
2
1 1 12. (a) Let the distance be x km and speed of train be y km/h.
Journey rides by horse cart x 1
2 3 Then by question, we have
1 x x 30
x km. ...(i)
6 y 4 y 60
y
o
u
Speed, Time & Distance (Trains, Boat & Stream) B-149

rs
m
x x 20 11 10

a
and ...(ii) B’s speed = 5 km / h

h
y 2 y 60 2 11

b
o
On solving (i) and (ii), we get x = 3y Total distance

o
Average speed =

b
Put x = 3y in (i) we get 19. (a)
Total time

.w
3y 1

o
3 400 4 9 400 4 9

rd
y 20
y 4 2

p
88 96 89 87 360

re
Hence, distance = 20 × 3 = 60 km. = 40 metres /minute

s
13. (a) Let the speed of the goods train be x kmph.

s
20. (b) Let the speed of the train and the car be x km/h and y

.c
Distance covered by goods train in 10 hours km/h, respectively.

o
= Distance covered by express train in 4 hours.

m
10x = 4 × 90 or x = 36. 120 480
Now, 8 …(i)
So, speed of goods train = 36 kmph. x y
14. (a) Due to stoppages, it covers 20 km less .
200 400 25
20 1 and …(ii)
Time taken to cover 20km = h h x y 3
80 4
From (i),
1 120y + 480x = 8xy and …(iii)
60 min = 15 min
4 25
From (ii), 200y + 400x = xy …(iv)
a 3
15. (b) If new speed is of original speed.
b From (iii) and (iv),
Then, 120 y 480 x 3( 200 y 400 x )
b 8 25
usual time × –1 change in time
a or 15y + 60x = 24y + 48x
4 1 x 3
usual time × 1 or 12x = 9y or
3 3 y 4
1 21. (a) Let the duration of the flight be x hours. Then,
usual time = 3 1 hr
3 600 600 600 1200
16. (b) Let the distance between the two stations be x km. 200 200
x 1 x 2x 1
x
x 10 x 50 2
Then,
50 6 30 6 x (2x + 1) = 3 2x2 + x – 3 = 0 (2x + 3) (x – 1) = 0
x = 1 hr. [neglecting the –ve value of x].
x 1 x 5 22. (c) Let the speed of the cars be x km/h and y km/h
50 6 30 6 respectively.
Their relative speed when they are moving in same
x x 2
or or x 50 km direction = (x – y) km/h.
30 50 3 Their relative speed when they are in opposite
Thus distance between the station A and B = 50 km directions = (x + y) km/h.
17. (a) Distance to be covered by the thief and by the owner
is same. 70
Now, 1 or x + y = 70 ..... (i)
Let after time 't', owner catches the thief. x y
1 70
40 t 50 t – and 7 or x – y = 10 ..... (ii)
2 ( x y)
5 1 Solving (i) and (ii), we get
10t 25 t
hr 2 hr x = 40 km/h and y = 30 km/h.
2 2
18. (a) A covers 3.5 km before he meets B in 23. (b) D=S×T

66 11 45
(18 × 3.5 + 3) = 66 min = h 60 = S hr
60 10 60

11 60 60
Now, B covers a distance of 5.5 km in hours S= 80km/hr
10 45
y
o
u
B-150 Speed, Time & Distance (Trains, Boat & Stream)

rs
m
Now, new speed = 80 – 15 = 65 km/hr.
200

a
i.e., t1 ....(1)

h
Distance 60 vm – vc
Time

b
hr.

o
Speed 65 and t2 time taken to travel 200 m with the current

o
b
.w
60 200
or 60 min 55.38 min . i.e., t 2 ....(2)

o
65 vm vc

rd
Hence, Time to taken by car to travel same distance is Given : t1 – t2 = 10 min

p
re
55.38 min.
200 200

s
24. (c) Let the husband and the wife meet after x minutes. 10

s
.c
4500 metres are covered by Pradeep in 60 minutes. vm – vc vm vc

o
m
2
In x minutes, he will cover
4500
x metres. vm vc2 40vc vc2 40vc (48) 2 0
60
vc 32, 72
Similarily,
Hence, speed of the current = 32 ( vc 72) .
3750
In x minutes, his wife will cover x m. 31. (c) Their relative speeds = (4.5 + 3.75) = 8.25 km/h
60
726
Distance = 726 metres = 0.726 km
4500 3750 1000
Now, x x 726
60 60 0.726
Required time = 60 5.28 min
726 60 8.25
x 5.28 min 32. (c) Let the total distance to be travelled = x km
8250 Speed of train = v km/h
25. (b) Volume of water flowed in an hour and time taken = t hr.
= 2000 × 40 × 3 cubic metre = 240000 cubic metre
volume of water flowed in 1 minute 150 x 150
t 8 .....(1)
240000 v 3v
= 4000 cubic metre = 40,00,000 litre 5
60
26. (b) Downstream speed = 15 + 5 = 20 km/h.
510 x 510
t 4 .....(2)
Required distance 20 24 8km. v 3
60 v
5
27. (a) Man's speed in upstream = 4 – 2 = 2 km/h.
Eq (2) – Eq (1)
6
Required time 3 km / h 510 150 x 510 x 150
2 4
v v 3 3v
16 v
28. (b) Rate downstream = kmph = 8 kmph; 5 5
2 360 360 5
4 v = 60 km/hr..
16 v 3v
Rate upstream = kmph = 4 kmph. x
4 t=
60
1
Speed in still water = (8 + 4) = 6 km/h. Put in eqn (1)
2 150 x 150 x
29. (a) Let speed of the boat in still water be x km/h and speed 8
60 3 60 60
of the current be y km/h.
Then, upstream speed = (x – y) km/h 5
and downstream speed = (x + y) km/h 5 x 150 x
8
2 36 60
24 28 x 150 x 5 11
Now, 6 …(1) 8
( x y) ( x y) 36 60 2 2
30 21 13 10 x 1500 6 x 11
and …(2) 360 2
( x y) ( x y) 2
360 11
Solving (1) and (2), we get 4x– 1500 = 1980 4x = 3480
2
x = 10 km/h and y = 4 km/h
3480
30. (d) Let vm = velocity of man = 48 m/min x= km 870 km
Let vc = velocity of current 4
then t1= time taken to travel 200 m against the current. 33. (b) Let the speed of swimmer be x km/hr
When he swim with the flow
y
o
u
Speed, Time & Distance (Trains, Boat & Stream) B-151

rs
m
then speed = ( x + 3/2) km/h. by the passenger respectively.

a
t Time

h
3
l Length of bridge

b
S1 x t

o
2

o
b
When he swim against the flow of stream t l1

.w
Therefore. 1
3

o
t2 l2
then speed x– t

rd
2

p
7 280

re
3 4 2

s
S2 x– t

s
x = 160 m

.c
2

o
According to the ques 38. (a) If the rowing speed in still water be x kmph, and the

m
S1 = 2S2. distance by y km, then
y
3 3 6
x t 2 x– t x–2
2 2
y = 6 (x – 2) ...(1)
3 2x – 3 y
x t 2t and, 4
2 2 x 2
2x 3 y = 4 (x + 2) ...(2)
2x – 3 6 (x – 2) = 4 (x + 2)
2 x = 10 kmph
9 1 39. (b) Time taken by the boat during downstream
2x + 3 = 4x – 6 9 = 2x = 4 km / hr
x
2 2 50 5
journey = h
34. (c) The speeds of two persons is 108 km/h and 75 km/h. 60 6
The first person covers 1080 km in 10 hours and thus 5
he makes 12 rounds. Thus, he will pass over another Time taken by the boat in upstream journey = h
4
person 12 times in any one of the direction. 2 50 100 24
Average speed = 48 mph
5 5 50
4
1st man 's speed b b 4 6 4
5
35. (b) 2nd man 's speed
= = = 40. (b) Let the Speed of faster train be x and speed of slower
a a 1
3 train be y.
3
Now, when both the train move in same direction their
relative speed = x – y
24 3 36 6 Now, total distance covered = 130 + 110 = 240
= = =
5 10 25 5 Now, distance = speed × time
12 6 240 = ( x– y) × 60 ( 1min 60sec)
=
2nd man 's speed 5 x–y=4 …(1)
When the trains move in opposite direction
60 then their relative speed = x + y
2nd man’s speed = = 10 km/hr.. 240 = ( x + y) × 3
6
80 = x + y …(2)
36. (b) Let speed of the second train = x km/hr.
on solving eqs (1) and (2), we get x = 42 m/sec
Relative speed of trains = (50 + x) km/hr.
and y = 38 m/sec
Distance travelled by trains = (100 + 120) = 220 metres 41. (c) Note here the length of the train in which passenger is
Distance = Speed × Time travelling is not considered since we are concerned
220 6 with the passenger instead of train. So, the length of
km (50 x) km/hr. hr the bridge will be directly proportional to the time taken
1000 3600
by the passenger respectively.
220 3600 t Time
50 + x = l Length of bridge
1000 6
50 + x = 132 t l1
Therefore. 1
x = 132 – 50 = 82 km/hr t2 l2
37. (c) Note here the length of the train in which passenger is
travelling is not considered since we are concerned 7 280
with the passenger instead of train. So, the length of 4 2
the bridge will be directly proportional to the time taken x = 160 m
y
o
u
B-152 Speed, Time & Distance (Trains, Boat & Stream)

rs
m
42. (c) Let correct time to cover journey be t hours 46. (c) Let at time ‘t’ dog overtook hare

a
distance travelled by dog = 16 t

h
12 3

b
70 t = 80 t

o
60 60 1

o
distance between dog and hare 12 t 0.1 m

b
70t + 14 = 80t+ 4 60

.w
10t = 10

o
t = 1 hour 1

rd
12 t 0.1 16 t
60

p
Total Distance Covered

re
43. (b) Average Speed =
Total Time Taken 1 1

s
12t 16 t

s
.c
6+6+6+6 24 5 10

o
=

m
6 6 6 6 6 1 1 1 1 3
4t
25 50 75 150 25 50 75 150 10
24 300 3
50 km/hr t h
6 24 40
44. (a) Let speed of A and B are S1 and S2 respectively.
3
S1 + S2 = 7 ....... (1) t 3600 270 seconds
40
24 24
14 47. (a)
S1 S2
48. (a) The monkey 1 meter in 4 min. This pattern will go on till
1 1 14 he reaches 11 meters. i.e., 11 × 4 = 44 mins. After that he
or ....... (2) would have climb 1 meter and he will reach the pole.
S1 S2 24
So, the total time taken = 44 + 1 = 45 min.
S1 S2 14 7 14 49. (c) Let T be the speed of train and C be the speed of car.
S1S2 24 S1S2 24 120 480 1 4 1
=8 ...(1)
S1S2 = 12 T C T C 15
S1(7 – S1) = 12 200 400 20 1 2 1
=8 ...(2)
S12 – 7S1 + 12 = 0 T C 60 T C 24
Subtracting (2) from (1)
(S1 – 4)(S1 – 3) = 0
2 1 1
S1 = 4, 3 (2 1)
Corresponding values of S2 = 3, 4 C 15 24
As, S1 > S2 2 1
= C = 80
A’s speed is 4 km/h C 40
45. (d) B covers 40 m in 7 seconds
1 1 4
40
= –
Speed of B = m/s T 15 80
7
1 1
= T = 60
1000 7 T 60
Time taken by B to cover 1 km 175 S
40 Required ratio = 60 : 80 = 3 : 4
A’s time over the course = (175 – 7) = 168 S
y
o
u
Mensuration B-153

rs
m
a
CHAPTER

h
b
MENSURATION 12

o
o
b
.w
o
rd
p
re
s
s
.c
o
Mensuration is the branch of mathematics which deals with the (a) Equilateral triangle

m
study of different geometrical shapes, their areas and volumes in
the broadest sense, it is all about the process of measmement.
These are two types of geometrical shapes (1) 2D (2) 3D
a a
Perimeter : Perimeter is sum of all the sides. It is measured in cm,
m, etc.
Area : The area of any figure is the amount of surface enclosed
within its boundary lines. This is measured in square unit like a
cm2, m2, etc. Perimeter = 3a
Volume : If an object is solid, then the space occupied by such an 3 2
object is called its volume. This is measured in cubic unit like cm3, A a ; where a side
4
m3, etc.
(b) Right triangle
Basic Conversions :
I. 1 km = 10 hm
1 hm = 10 dam
1 dam = 10 m
1 m = 10 dm p h
1 dm = 10 cm
1 cm = 10 mm
1 m = 100 cm = 1000 mm
1km = 1000 m
b
5
II. 1 km miles 1
8 A pb and h 2 = p2 + b2 (Pythagoras triplet)
1 mile = 1.6 km 2
1 inch = 2.54 cm where p perpendicular
1 mile = 1760 yd = 5280 ft. b base
1 nautical mile (knot) = 6080 ft h hypotenuse
III. 100 kg = 1 quintal Example 1 :
10 quintal = 1 tonne Find the area of a triangle whose sides are 50 m, 78m, 112m
1 kg = 2.2 pounds (approx.) respectively and also find the perpendicular from the
IV. l litre = 1000 cc opposite angle on the side 112 m.
1 acre = 100 m2 Solution :
1 hectare = 10000 m2 (100 acre)
Here a = 50 m, b = 78 m, c = 112m
PART I : PLANE FIGURES 1
s 50 78 112 120m
B 2
TRIANGLE
s – a = 120 – 50 = 70 m
s – b = 120 – 78 = 42 m
a c s – c = 120 – 112 = 8 m
h
Area 120 70 42 8 = 1680 sq.m.
1
Area base perpendicular
Perimeter (P) = a + b + c C
b
A 2
Area (A) s s–a s–b s–c 2Area 1680 2
Perpendicular 30m.
a b c Base 112
where s and a, b and c are three sides of the triangle. Example 2 :
2
1 The base of a triangular field is 880 m and its height 550 m.
Also, A bh ; where b base Find the area of the field. Also calculate the charges for
2
h altitude
y
o
u
B-154 Mensuration

rs
RHOMBUS

m
supplying water to the field at the rate of ` 24.25 per sq.

a
hectometre.

h
d1

b
Solution :

o
a

o
Base Height d2

b
Area of the field

.w
2

o
Perimeter = 4 a

rd
880 550
242000 sq.m. = 24.20 sq.hm

p
2 1
Area d1 d 2 where a side and

re
2
Cost of supplying water to 1 sq. hm = ` 24.25

s
d1 and d2 are diagonals.

s
.c
Cost of supplying water to the whole field

o
= 24.20 × 24.25 = ` 586.85 IRREGULAR QUADRILATERAL

m
NOTE :
p q
h1
(Perimeter) 2 d
• In a rectangle, (diagonal) 2 2 Area
4 s h2 r
• In an isosceles right angled triangle,
b Perimeter = p + q + r + s
Area = 4a 2 b 2
4 1
d h1 h2
Area
where a is two equal side and b is different side 2
• In a parallelogram, TRAPEZIUM
a
Area = Diagonal × length of perpendicular on it
• If area of circle is decreased by x%, then the radius of m h n
circle is decreased by (100 10 100 x )%
b
RECTANGLE
Perimeter = a + b + m + n
1
Area a b h; where a and b are two parallel sides;
b 2
m and n are two non-parallel sides;
h perpendicular distance
between two parallel sides.
Area of pathways running across the middle of a rectangle
Perimeter = 2 ( + b)
Area = × b; where length
b breadth
a b
SQUARE

a
A=a( + b) – a2; where length
b breadth,
a width of the pathway.
a PATHWAYS OUTSIDE
Perimeter = 4 × side = 4a
Area = (side)2 = a2; where a side
PARALLELOGRAM
b
b + 2a b
a h a
a

b 2a
Perimeter = 2 (a + b) A = ( + 2a) (b + 2a) – b; where length
Area = b × h; where a breadth b breadth
b base (or length) a width of the pathway
h altitude
y
o
u
Mensuration B-155

rs
m
PATHWAYS INSIDE 2 2
BO 36.5 – 27.5 = 24 cm

a
h
b
l – 2a Hence, the other diagonal BD = 48 cm

o
o
1

b
b b – 2a Now, Area of the rhomhus AC BD

.w
2

o
rd
a 1
55 48 = 1320 sq.cm.

p
2

re
l
Example 6 :

s
A = b – ( – 2a) (b – 2a); where length

s
Find the area of a quadrilateral piece of ground, one of whose

.c
b breadth

o
diagonals is 60 m long and the perpendicular from the other

m
a width of the pathway two vertices are 38 and 22m respectively.
Example 3 : Solution :
A 5100 sq.cm trapezium has the perpendicular distance
1
between the two parallel sides 60 m. If one of the parallel Area d h1 h 2
2
sides be 40m then find the length of the other parallel side.
Solution : 1
60 38 22 1800 sq.m.
2
1
Since, A a b h CIRCLE
2

1
5100 40 x 60
2
170 = 40 + x
r
other parallel side = 170 – 40 = 130 m

Example 4 : Perimeter (Circumference) = 2 r = d


A rectangular grassy plot is 112m by 78 m. It has a gravel Area = r2; where r radius
path 2.5 m wide all round it on the inside. Find the area of the d diameter
path and the cost of constructing it at ` 2 per square metre? 22
Solution : and or 3.14
7
A = b – ( – 2a) (b – 2a) SEMICIRCLE
= 112 × 78 – (112 – 5) (78 – 5) Perimeter = r + 2r
= 112 × 78 – 107 × 73 = 8736 – 7811 1
Area r2
= 925 sq.m 2
r
Cost of construction = rate × area SECTOR OFA CIRCLE
= 2 × 925 = ` 1850
Example 5 :
The perimeter of a rhombus is 146 cm and one of its O
diagonals is 55 cm. Find the other diagonal and the area of r
the rhombus.
Solution : A
l
B
Segment
Let ABCD be the rhombus in which AC = 55 cm.
D C Area of sector OAB r2
360

m O Length of an arc (l) = 2 r


.5c 360
27
Area of segment = Area of sector – Area of triangle OAB
A 36.5cm B 1 2
r2 – r sin
146 360 2
and AB 36.5 cm
4 Perimeter of segment = length of the arc + length of segment
55 r
Also, AO 27.5m AB 2r sin
180 2
2
y
o
u
B-156 Mensuration

rs
m
RING Solution :

a
Distance to be travelled = 11km = 11000 m

h
b
3 7

o
Radius of the wheel 1 m m

o
4 4

b
R2

.w
R1 22 7

o
Circumference of the wheel 2 11 m

rd
7 4

p
In travelling 11 m, wheel makes 1 revolution.

re
s
1

s
In travelling 11000 m the wheel makes 11000

.c
Area of ring R 22 – R12 11

o
revolutions, i.e., 1000 revolutions.

m
Example 7 : Example 11 :
A wire is looped in the form of a circle of radius 28 cm. It is The circumference of a circular garden is 1012m. Find the
re-bent into a square form. Determine the length of a side of area of outsider road of 3.5 m width runs around it. Calculate
the square. the area of this road and find the cost of gravelling the road
Solution : at ` 32 per 100 sqm.
(a) Length of the wire = Perimeter of the circle Solution :
= 2 × 28 A = r2 , C = 2 r = 1012
= 176 cm2
176
Side of the square 44 cm
4
Example 8 :
The radius of a wheel is 42 cm. How many revolutions will it
make in going 26.4 km ?
Solution : 1 7
Distance travelled in one revolution = Circumference of the r 1012 161m
2 22
22 22
wheel = 2 r = 2 42cm. = 264 cm Area of garden 161 161 81466 sq. m
7 7
No. of revolutions required to travel 26.4 km Area of the road = area of bigger circle – area of the garden
329
26.4 1000 100 Now, radius of bigger circle = 161 + 3.5 m
= = 10000 2
264
22 329 329 1
Example 9 : Area of bigger circle = 85046 sq.m
7 2 2 2
Find the area of sector of a circle whose radius is 6 cm when–
(a) the angle at the centre is 35° 1 1
Thus, area of the road 85046 – 81466 3580 sqm.
(b) when the length of arc is 22 cm 2 2
Solution : 7161 32
Hence, cost Rs. = ` 1145.76
(a) Area of sector 2 100
Example 12 :
2 22 35 There is an equilateral triangle of which each side is 2m.
= r . 6 6 cm 2 11sq.cm.
360 7 360 With all the three corners as centres, circles each of radius
1 m are described.
(b) Here length of arc 22 cm.
(i) Calculate the area common to all the circles and the
triangle.
2 r 22cm. (ii) Find the area of the remaining portion of the triangle.
360 Solution :
1
Area of sector = r 2. r.2 r
360 2 360 A

1 1 60°
= r. 6 22 sq.cm 66 sq.cm.
2 2 M N
Example 10 : 60° 60°
3 B 1m C
The radius of a circular wheel is 1 m. How many revolutions
4
will it make in travelling 11 km ?
y
o
u
Mensuration B-157

rs
m
RIGHT CIRCULAR CYLINDER
60 22
r2

a
Area of each sector 1 1 It is a solid which has both its ends in the form of a circle.

h
360 360 7

b
Lateral surface area = 2 rh

o
1 22 11 2

o
m Total surface area = 2 r (r + h)

b
6 7 21

.w
Volume = r2h; where r is radius of the base and h is height

o
3 2

rd
Area of equilateral triangle a

p
4

re
s
3

s
3m 2

.c
2 2 h
4

o
m
(i) Common area = 3 × Area of each sector
11 11 r
3 1.57m 2
21 7 PYRAMID
(ii) Area of the remaining portion of the triangle = Ar. of A pyramid is a solid which can have any polygon at its base and
equilateral triangle – 3(Ar. of each sector) its edges converge to single apex.
Lateral or curved surface area
3 1.57 = 1.73 – 1.57 = 0.16m2
1 1
(perimeter of base) × slant height = pl
PART-II : SOLID FIGURE 2 2
Total surface area = lateral surface area + area of the base
CUBOID
1
A cuboid is a three dimensional box. Volume (area of the base) × height
3
Total surface area of a cuboid = 2 (lb + bh + lh)
Volume of the cuboid = lbh (i) Triangular Pyramid :
Length of diagonal l 2 b2 h2

b
l
l

Area of four walls = 2(l + b) × h


Rectangular Parallelpiped box. It is same as cuboid. Formally a
polyhedron for which all faces are rectangles.
CUBE
A cube is a cuboid which has all its edges equal.
Total surface area of a cube = 6a2 a
Volume of longest the cube = a3 a
Length of longest diagonal = 3a a (i) Area of the lateral surface of the pyramid
RIGHT PRISM a 3
1 1
A prism is a solid which can have any polygon at both its ends. = × perimeter × slant height = × 3a ×l = al
2 2 2
Lateral or curved surface area = Perimeter of base × height
Total surface area = Lateral surface area + 2 (area of the end) 1 1 3 2
(ii) Volume = × h× area of base = ×h× .a
Volume = Area of base × height 3 3 4

ha 2
=
4 3

1 3 2
(iii) Total Area of the pyramid = 3al a
2 4
(ii) Square Pyramid :

1 1
(i) Volume = h × area of base = h × a2
3 3
y
o
u
B-158 Mensuration

rs
m
Curved surface area = (r1 + r2)

a
h
Total surface area r1 r2 r12 r22 r1

b
o
o
2
where h2

b
h r1 – r2

.w
l
h

o
1
h r12 r22

rd
Volume r1r2
3

p
re
Where r1 and r2 : radii r2

s
h : height

s
.c
Example 13 :

o
a

m
The sum of length, breadth and height of a room is 19 m.
1 The length of the diagonal is 11 m. Find the cost of painting
(ii) Lateral surface area = × perimeter × slant height = the total surface area of the room at the rate of ` 10 per m2.:
2
Solution :
Let length, breadth and height of the room be , b and
1 h, respectively. Then,
× 4a × l = 2al
2 + b + h = 19 ...(i)
(iii) Total area of the pyramid = 2al + a2 = a (2l + a) and 2
b2 h 2 11
RIGHT CIRCULAR CONE 2 + b2
+ h2 = 121 ...(ii)
It is a solid which has a circle as its base and a slanting lateral Area of the surface to be painted
surface that converges at the apex. = 2( b + bh + h )
Lateral surface area = r ( + b + h)2 = 2 + b2 + h2 + 2 ( b + bh + h )
2( b + bh + h ) = (19)2 – 121 = 361 – 121 = 240
Total surface area = r ( + r)
1 2 Surface area of the room =240 m2.
Volume r h; where r : radius of the base Cost of painting the required area = 10 × 240 = ` 2400
3
h : height Example 14 :
ABCD is a parallelogram. P, Q, R and S are points on sides
: slant height
AB, BC, CD and DA, respectively such that AP = DR. If the
area of the rectangle ABCD is 16 cm2, then find the area of
the quadrilateral PQRS.
D R
C
h
Q
r

SPHERE S
It is a solid in the form of a ball with radius r.
Lateral surface area = Total surface area = 4 r2 A B
P
4 3 Solution :
Volume r ; where r is radius. Area of the quadrilateral PQRS
3
= Area of SPR + Area of PQR
1 1
PR AP PR PB
r 2 2
1 1
PR AP PB AD AB
2 2
( PR = AD and AP + PB = AB)
1 1
HEMISPHERE Area of rectangle ABCD 16 8cm 2
2 2
It is a solid half of the sphere. r Example 15 :
Lateral surface area = 2 r2 A road roller of diameter 1.75 m and length 1 m has to press
Total surface area = 3 r2 r a ground of area 1100 sqm. How many revolutions does it
make ?
2 3 Solution :
Volume r ; where r is radius
3 Area covered in one revolution = curved surface area
FRUSTUM OFACONE Total area to be pressed
Number of revolutions
When a cone cut the left over part is called the frustum of the Curved surface area
cone.
y
o
u
Mensuration B-159

rs
m
1100 1100
Example 19 :

a
A cylindrical bath tub of radius 12cm contains water to a

h
2 rh 22 1.75 depth of 20 cm. A spherical iron ball is dropped into the tub
2 1

b
o
7 2 and thus the level of water is raised by 6.75 cm. What is the

o
= 200

b
radius of the ball?

.w
Example 16 : Solution :
The annual rainfall at a place is 43 cm. Find the weight in

o
Volume of the spherical ball = volume of the water

rd
metric tonnes of the annual rain falling there on a hectare of displaced.

p
land, taking the weight of water to be 1 metric tonne to the

re
cubic metre. 4 3
r = (12)2 × 6.75

s
s
Solution : 3

.c
Area of land = 10000 sqm

o
144 6.75 3
r3

m
10000 43 729
Volume of rainfall = 4300 m3 4
100
or r = 9 cm
Weight of water = 4300 × 1 m tonnes = 4300 m tonnes
Example 20 :
A toy is in the form of a cone mounted on a hemisphere with
Example 17 :
The height of a bucket is 45 cm. The radii of the two circular the same radius. The diameter of the base of the conical
ends are 28 cm and 7 cm, respectively. Find the volume of portion is 6 cm and its height is 4 cm. Determine the surface
the bucket. area of the toy. (Use = 3.14).
Solution : Solution :
Here r1 = 7 cm, r2 = 28 cm and h = 45 cm 1
7cm
The radius of the hemisphere 6 3 cm
2

Now, slant height of cone 32 42 5 cm

45cm

5cm
28cm 4cm

1
Volume of the bucket h(r12 r22 r1r2 ) 3cm
3
Hence, the required volume
1 22
45(282 7 2 28 7) = 48510 cm3
3 7 The surface area of the toy
Example 18 : = Curved surface of the conical portion
A hollow cylindrical tube open at both ends is made of iron + Curved surface of the hemisphere
2 cm thick. If the external diameter be 50 cm and the length of = ( × 3 × 5 + 2 × 32) cm2
the tube be 140 cm, find the number of cubic cm of iron in it. = 3.14 × 3 (5 + 6) cm2 = 103.62 cm2.
Solution : Example 21 :
Height = 140 cm A solid is composed of a cylinder with hemispherical ends.
External diameter = 50 cm If the whole length of the solid is 104 cm and the radius of
External radius = 25 cm each of the hemispherical ends is 7 cm, find the cost of
polishing its surface at the rate of Re. 1 per dm2.
Solution :

O A B
7cm
90cm

Also, internal radius OA = OB – AB= 25 – 2 = 23 cm


Volume of iron = Vexternal – Vinternal
22 7cm
140 252 – 232 = 42240 cu. cm.
7
y
o
u
B-160 Mensuration

rs
m
Let the height of the cylinder be h cm. (a) Area of the field

a
Then h + 7 + 7 = 104

h
1 22 1 22

b
h = 90 42 35 2 21 2 2 17.5 2

o
2 7 2 7
Surface area of the solid

o
b
= 2 × curved surface area of hemisphere = 1470 + 1386 + 962.5 = 3818.5 m2

.w
+ curved surface area of the cylinder

o
Example 25 :

rd
22 22 A frustum of a right circular cone has a diameter of base 10

p
2 2 7 7 2 7 90 cm 2

re
7 7 cm, top of 6 cm, and a height of 5 cm; find the area of its

s
whole surface and volume.

s
= 616 + 3960 cm2 = 4576 cm2

.c
Solution :

o
Cost of polishing the surface of the solid

m
Here r1 = 5 cm, r 2 = 3 cm and h = 5 cm.
4576 1
=` ` 45.76
100 2
h2 r1 – r2
Example 22 :
A regular hexagonal prism has perimeter of its base as 600
2
cm and height equal to 200 cm. How many litres of petrol 52 5–3 29 cm = 5.385 cm
can it hold ? Find the weight of petrol if density is 0.8 gm/cc.
Whole surface of the frustum
Solution :
Perimeter 600 r1 r2 r12 r22
Side of hexagon 100 cm
Number of sides 6
22 22 2 22 2
5.385 5 3 5 3 = 242.25 sq.cm.
3 3 7 7 7
Area of regular hexagon 100 100 = 25950 sq.cm.
2
h 2
Volume = Base area × height Volume = (r1 r1r2 r22 )
3
= 25950 × 200 = 5190000 cu.cm. = 5.19 cu.m.
Weight of petrol = Volume × Density 22 5 2
= 5190000 × 0.8 gm/cc 5 5 3 32 = 256.67 cu. cm.
7 3
= 4152000 gm = 4152 kg.
Example 23 : Example 26 :
A right pyramid, 12 cm high, has a square base each side of A cylinder is circumscribed about a hemisphere and a cone
which is 10 cm. Find the volume of the pyramid. is inscribed in the cylinder so as to have its vertex at the
Solution : centre of one end, and the other end as its base. The volume
Area of the base = 10 × 10 = 100 sq.cm. of the cylinder, hemisphere and the cone are, respectively in
Height = 12 cm the ratio :
1 (a) 2 : 3 : 2 (b) 3 : 2 : 1
Volume of the pyramid 100 12 = 400 cu.cm.
3 (c) 3 : 1 : 2 (d) 1 : 2 : 3
Example 24 : Solution :
Semi-circular lawns are attached to both the edges of a (b) We have,
rectangular field measuring 42 m × 35m. The area of the total radius of the hemisphere = raidus of the cone
field is :
= height of the cone
(a) 3818.5 m2 (b) 8318 m2
(c) 5813 m 2 (d) 1358 m2 = height of the cylinder = r (say)
Solution : Then, ratio of the volumes of cylinder, hemisphere and
cone

1 3 2 1
r3 : r3 : r 1: : 3: 2 :1
3 3 3 3
y
o
u
Mensuration B-161

rs
m
a
h
b
o
o
b
.w
1. The length and breadth of a rectangle are in the ratio 9 : 5. If 11. A rectangular plot 15 m ×10 m, has a path of grass outside it.

o
its area is 720 m2, find its perimeter. If the area of grassy pathway is 54 m 2, find the width of the

rd
p
(a) 112 metre (b) 115 metre path.

re
(c) 110 metre (d) 118 metre (a) 4 m (b) 3 m

s
s
2. A circle and a rectangle have the same perimeter. The sides (c) 2 m (d) 1 m

.c
of the rectangle are 18 cm and 26 cm. What is the area of the 12. If the area of a circle decreases by 36%, then the radius of a

o
m
circle ? circle decreases by
(a) 88 cm2 (b) 154 cm2 (a) 20% (b) 18%
(c) 1250 cm 2 (d) 616 cm2 (c) 36% (d) 64%
3. If the perimeter and diagonal of a rectangle are 14 and 5 cms 13. The floor of a rectangular room is 15 m long and 12 m wide.
respectively, find its area. The room is surrounded by a verandah of width 2 m on all
(a) 12 cm2 (b) 16 cm2 its sides. The area of the verandah is :
(c) 20 cm 2 (d) 24 cm2 (a) 124 m2 (b) 120 m2
4. In an isoscele right angled triangle, the perimeter is 20 metre. (c) 108 m2 (d) 58 m2
Find its area. 14. A rectangular lawn 70 m × 30 m has two roads each 5 metres
(a) 100 3 2 2 m2 (b) 150 5 3 m2 wide, running in the middle of it, one parallel to the length
and the other parallel to the breadth. Find the cost of
(c) 500 m2 (d) None of these gravelling the road at the rate of ` 4 per square metre.
5. In a parallelogram, the length of one diagonal and the (a) ` 2,000 (b) ` 1,800
perpendicular dropped on that diagonal are 30 and 20 metres (c) ` 1,900 (d) ` 1,700
respectively. Find its area. 15. A cylindrical bucket of height 36 cm and radius 21 cm is
(a) 600 m2 (b) 540 m2 filled with sand. The bucket is emptied on the ground and a
(c) 680 m 2 (d) 574 m2 conical heap of sand is formed, the height of the heap being
6. The diameter of a garden roller is 1.4 m and it is 2 m long. 12 cm. The radius of the heap at the base is :
22 (a) 63 cm (b) 53 cm
How much area will it cover in 5 revolutions ? use (c) 56 cm (d) 66 cm
7
(a) 40 m2 (b) 44 m2 16. The altitude drawn to the base of an isosceles triangle is 8
(c) 48 m 2 (d) 36 m2 cm and the perimeter is 32 cm. The area of the triangle is
7. A horse is tethered to one corner of a rectangular grassy (a) 72 cm2 (b) 60 cm2
field 40 m by 24 m with a rope 14 m long. Over how much (c) 66 cm2 (d) None of these
area of the field can it graze? 17. The cross section of a canal is a trapezium in shape. If the
(a) 154 cm2 (b) 308 m2 canal is 7 metres wide at the top and 9 metres at the bottom
(c) 150 m2 (d) None of these and the area of cross-section is 1280 square metres, find the
8. From a square piece of a paper having each side equal to 10 length of the canal.
cm, the largest possible circle is being cut out. The ratio of (a) 160 metres (b) 172 metres
the area of the circle to the area of the original square is (c) 154 metres (d) None of these
nearly : 18. It is required to fix a pipe such that water flowing through it
at a speed of 7 metres per minute fills a tank of capacity 440
4 3
(a) (b) cubic metres in 10 minutes. The inner radius of the pipe
5 5
should be :
5 6
(c) (d) (a) 2 m (b) 2 m
6 7
9. A square carpet with an area 169 m2 must have 2 metres cut- 1 1
(c) m (d) m
off one of its edges in order to be a perfect fit for a rectangular 2 2
room. What is the area of rectangular room? 19. The area of a rectangular field is 144 m 2. If the length had
(a) 180 m2 (b) 164 m2 been 6 metres more, the area would have been 54 m 2 more.
(c) 152 m 2 (d) 143 m2 The original length of the field is
10. A picture 30" × 20" has a frame 2½" wide. The area of the
(a) 22 metres (b) 18 metres
picture is approximately how many times the area of the
(c) 16 metres (d) 24 metres
frame?
20. A rectangular parking space is marked out by painting three
1 of its sides. If the length of the unpainted side is 9 feet, and
(a) 4 (b) 2
2 the sum of the lengths of the painted sides is 37 feet, then
(c) 2 (d) 5 what is the area of the parking space in square feet?
y
o
u
B-162 Mensuration

rs
m2.

m
(a) 46 (b) 81 30. The surface area of a cube is 150 The length of its

a
(c) 126 (d) 252 diagonal is

h
21. A rectangular paper, when folded into two congruent parts

b
(a) 5 3m (b) 5 m

o
had a perimeter of 34 cm for each part folded along one set

o
b
of sides and the same is 38 cm when folded along the other 10

.w
set of sides. What is the area of the paper? (c) m (d) 15 m
3

o
(a) 140 cm2 (b) 240 cm2

rd
2 31. A copper sphere of radius 3 cm is beaten and drawn into a
(c) 560 cm (d) None of these

p
wire of diametre 0.2 cm. The length of the wire is

re
22. The length and breadth of the floor of the room are 20 feet (a) 9 m (b) 12 m

s
and 10 feet respectively. Square tiles of 2 feet length of

s
(c) 18 m (d) 36 m

.c
different colours are to be laid on the floor. Black tiles are
32. A plot of land in the form of a rectangle has a dimension 240

o
laid in the first row on all sides. If white tiles are laid in the

m
m × 180 m. A drainlet 10 m wide is dug all around it (outside)
one-third of the remaining and blue tiles in the rest, how
and the earth drug out is evenly spread over the plot,
many blue tiles will be there?
increasing its surface level by 25 cm. The depth of the drainlet
(a) 16 (b) 24 is
(c) 32 (d) 48
(a) 1.225 m (b) 1.229 m
23. Four equal circles are described about the four corners of a
(c) 1.227 m (d) 1.223 m
square so that each touches two of the others. If a side of 33. The water from a roof, 9 sq metres in area, flows down to a
the square is 14 cm, then the area enclosed between the
cylinder container of 900 cm2 base. To what height will the
circumferences of the circles is :
water rise in cylinder if there is a rainfall of 0.1 mm ?
(a) 24 cm2 (b) 42 cm2 (a) 1 cm (b) 0.1 metre
(c) 154 cm 2 (d) 196 cm2
(c) 0.11 cm (d) 10 cms
24. The ratio between the length and the breadth of a rectangular
34. The length of a cold storage is double its breadth. Its height
park is 3 : 2. If a man cycling along the boundary of the park is 3 metres. The area of its four walls (including the doors) is
at the speed of 12km / hr completes one round in 8 minutes,
108 m2. Find its volume.
then the area of the park (in sq. m) is:
(a) 215 m3 (b) 216 m3
(a) 15360 (b) 153600 (c) 217 m3 (d) 218 m3
(c) 30720 (d) 307200
35. How many spherical bullets can be made out of a lead
25. The water in a rectangular reservoir having a base 80 metres
cylinder 28 cm high and with base radius 6 cm, each bullet
by 60 metres is 6.5 metres deep. In what time can the water being 1.5 cm in diameter?
be emptied by a pipe whose cross section is a square of
(a) 1845 (b) 1824
side 20 cm, if the water runs through the pipe at the rate of
(c) 1792 (d) 1752
15 km per hour? 36. A rectangular reservoir is 54 m ×44 m × 10 m. An empty pipe
(a) 52 hrs (b) 26 hrs
of circular cross-section is of radius 3 cms, and the water
(c) 65 hrs (d) 42 hrs
runs through the pipe at 20 m section. Find the time the
26. The ratio of height of a room to its semi-perimeter is 2 : 5. It empty pipe will take to empty the reservoir full of water.
costs ` 260 to paper the walls of the room with paper 50 cm
(a) 116.67 hours (b) 110 .42 hours
wide at ` 2 per metre alllowing an area of 15 sq. m for doors
(c) 120.37 hours (d) 112 hours
and windows. The height of the room is: 37. A spherical ball of lead, 3 cm in diameter, is melted and recast
(a) 2.6 m (b) 3.9 m
into three spherical balls. The diameter of two of these balls
(c) 4 m (d) 4.2 m
are 1.5 cm and 2 cm respectively. The diameter of the third
27. Wheels of diameters 7 cm and 14 cm start rolling ball is
simultaneously from X and Y, which are 1980 cm apart,
(a) 2.5 cm (b) 2.66 cm
towards each other in opposite directions. Both of them
(c) 3 cm (d) 3.5 cm
make the same number of revolutions per second. If both of 38. A cube of 384 cm2 surface area is melt to make x number of
them meet after 10 seconds, the speed of the smaller wheel
small cubes each of 96 mm2 surface area. The value of x is
is:
(a) 80,000 (b) 8
(a) 22 cm / sec (b) 44 cm / sec (c) 8,000 (d) 800
(c) 66 cm / sec (d) 132 cm / sec
39. A conical vessel, whose internal radius is 12 cm and height
28. A metal cube of edge 12 cm is melted and formed into three
50 cm, is full of liquid. The contents are emptied into a
smaller cubes. If the edges of two smaller cubes are 6 cm cylindrical vessel with internal radius 10 cm. Find the height
and 8 cm, then find the edge of the third smaller cube.
to which the liquid rises in the cylindrical vessel.
(a) 10 cm (b) 14 cm
(a) 18 cm (b) 22 cm
(c) 12 cm (d) 16 cm (c) 24 cm (d) None of these
29. The length, breadth and height of a cuboid are in the ratio
40. The trunk of a tree is a right cylinder 1.5 m in radius and 10
1 : 2 : 3. The length, breadth and height of the cuboid are
m high. The volume of the timber which remains when the
increased by 100%, 200% and 200%, respectively. Then, the trunk is trimmed just enough to reduce it to a rectangular
increase in the volume of the cuboid will be :
parallelopiped on a square base is
(a) 5 times (b) 6 times
(a) 44 m3 (b) 46 m3
(c) 12 times (d) 17 times (c) 45 m 3 (d) 47 m3
y
o
u
Mensuration B-163

rs
The cost of the paint is ` 36.50 per kg. If 1 kg of paint covers

m
41. 51. A cone of height 9 cm with diameter of its base 18 cm is

a
16 square feet, how much will it cost to paint outside of a carved out from a wooden solid sphere of radius 9 cm. The

h
percentage of the wood wasted is:

b
cube having 8 feet each side?

o
(a) ` 692 (b) ` 768 (a) 25% (b) 30%

o
b
(c) ` 876 (d) ` 972 (c) 50% (d) 75%

.w
42. A right circular cone and a right circular cylinder have equal 52. A hemispherical bowl is filled to the brim with a beverage.

o
The contents of the bowl are transfered into a cylindrical

rd
base and equal height. If the radius of the base and the
vessel whose radius is 50% more than its height. If the

p
height are in the ratio 5 : 12, then the ratio of the total surface

re
area of the cylinder to that of the cone is diameter is same for both the bowl and the cylinder, the

s
volume of the beverage in the cylindrical vessel is:

s
(a) 3 : 1 (b) 13 : 9

.c
(c) 17 : 9 (d) 34 : 9

o
2 1

m
43. A reservoir is supplied from a pipe 6 cm in diameter. How (a) 66 % (b) 78 %
3 2
many pipes of 3 cms diameter would discharge the same (c) 100% (d) More than 100%
quantity, supposing the velocity of water is same ? 53. A cylindrical container of radius 6 cm and height 15 cm is
(a) 4 (b) 5 filled with ice-cream. The whole ice-cream has to be
(c) 6 (d) 7 distributed to 10 children in equal cones with hemispherical
44. A conical cavity is drilled in a circular cylinder of 15 cm tops. If the height of the conical portion is four times the
height and 16 cm base diameter. The height and the base
radius of its base, then find the radius of the ice-cream cone.
diameter of the cone are same as those of the cylinder.
(a) 2 cm (b) 3 cm
Determine the total surface area of the remaining solid.
(c) 4 cm (d) 5 cm
(a) 440 cm2 (b) 215 cm2
2 54. A cylinder is filled to 4/5th its volume. It is then filled so that
(c) 542 cm (d) 376 cm2
the level of water coincides with one edge of its bottom and
45. An ice-cream company makes a popular brand of ice-cream
top edge of the opposite side, In the process, 30 cc of the
in rectangular shaped bar 6 cm long, 5 cm wide and 2 cm
water is spilled. What is the volume of the cylinder?
thick. To cut the cost, the company has decided to reduce
(a) 75 cc (b) 96 cc
the volume of the bar by 20%, the thickness remaining the
same, but the length and width will be decreased by the (c) Data insufficient (d) 100 cc
same percentage amount. The new length L will satisfy : 55. There are two concentric circular tracks of radii 100 m and
(a) 5.5 < L < 6 (b) 5 < L < 5.5 102 m, respectively. A runs on the inner track and goes once
(c) 4.5 < L < 5 (d) 4 < L < 4.5 round on the inner track in 1 min 30 sec, while B runs on the
46. Water flows through a cylindrical pipe of internal diameter 7 outer track in 1 min 32 sec. Who runs faster?
cm at 2 m per second. If the pipe is always half full, then (a) Both A and B are equal (b) A
what is the volume of water (in litres) discharged in 10 (c) B (d) None of these
minutes? 56. A monument has 50 cylindrical pillars each of diameter 50
(a) 2310 (b) 3850 cm and height 4 m. What will be the labour charges for getting
(c) 4620 (d) 9240 these pillars cleaned at the rate of 50 paise per sq. m?
47. If the radius of a sphere is increased by 2 cm, then its surface (use 3.14)
area increases by 352 cm2. The radius of the sphere before (a) ` 237 (b) ` 157
the increase was: (c) ` 257 (d) ` 353
(a) 3 cm (b) 4 cm 57. Four sheets 50 cm × 5 cm are arranged without overlapping
(c) 5 cm (d) 6 cm to form a square having side 55 cm. What is the area of inner
48. A semicircular sheet of paper of diameter 28 cm is bent to square so formed?
cover the exterior surface of an open conical ice-cream cup. (a) 2500 cm2 (b) 2025 cm2
The depth of the ice-cream cup is (c) 1600 cm 2 (d) None of these
(a) 10.12 cm (b) 8.12 cm 58. A conical vessel of base radius 2 cm and height 3 cm is filled
(c) 12.12 cm (d) 14.12 cm with kerosene. This liquid leaks through a hole in the bottom
49. The cost of painting the walls of a room at the rate of ` 1.35 and collects in a cylindrical jar of radius 2 cm. The kerosene
per square metre is ` 340.20 and the cost of matting the floor level in the jar is
at the rate of ` 0.85 per m2 is ` 91.80. If the length of the (a) cm (b) 1.5 cm
room is 12 m, then the height of the room is : (c) 1 cm (d) 3 cm
(a) 6 m (b) 12 m 59. A garden is 24 m long and 14 m wide. There is a path 1 m
(c) 1.2 m (d) 13.27 m
wide outside the garden along its sides. If the path is to be
50. A hollow sphere of internal and external diameters 4 cm and
constructed with square marble tiles 20 cm × 20 cm, the
8 cm respectively is melted into a cone of base diamater 8
cm. The height of the cone is: number of tiles required to cover the path is
(a) 12 cm (b) 14 cm (a) 1800 (b) 200
(c) 15 cm (d) 18 cm (c) 2000 (d) 2150
y
o
u
B-164 Mensuration

rs
m
60. 2 cm of rain has fallen on a sq. km of land. Assuming that 67. There are two cones. The curved surface are aof one is

a
50% of the raindrops could have been collected and twice that of the other. The slant height of the latter is twice

h
that of the former. The ratio of their radii is

b
contained in a pool having a 100 m × 10 m base, by what

o
(a) 4 : 1 (b) 4 : 3

o
level would the water level in the pool have increased?

b
(c) 3 : 4 (d) 1 : 4

.w
(a) 15 m (b) 20 m
68. A wire is bent into the form of a circle, whose area is 154

o
(c) 10 m (d) 25 m
cm2. If the same wire is bent into the form of an equilateral

rd
61. In a swimming pool measuring 90 m by 40 m, 150 men take a triangle, the approximate area of the equilateral triangle is

p
re
dip. If the average displacement of water by a man is 8 cubic (a) 93.14 cm2 (b) 90.14 cm2

s
metres, what will be the rise in water level? (c) 83.14 cm 2 (d) 39.14 cm2

s
.c
(a) 33.33 cm (b) 30 cm 69. The radius of a right circular cone is 3 cm and its height is 4

o
m
(c) 20 cm (d) 25 cm cm. The total surface area of the cone is
62. A square is inscribed in a circle of radius 8 cm. The area of (a) 48.4 sq.cm (b) 64.4 sq.cm
the square is (c) 96.4 sq.cm (d) 75.4 sq.cm
70. A wooden box of dimension 8 metre × 7 metre × 6 metre is to
(a) 16 cm2 (b) 64 cm2
carry rectangular boxes of dimensions 8 cm × 7 cm × 6 cm.
(c) 128 cm2 (d) 148 cm2 The maximum number of boxes that can be carried in 1
63. The biggest possible circle is inscribed in a rectangle of wooden box is
length 16 cm and breadth 6 cm. Then its area is (a) 7500000 (b) 9800000
(a) 3 cm2 (b) 4 cm2 (c) 1200000 (d) 1000000
2
(c) 5 cm (d) 9 cm2 71. Two circular cylinders of equal volume have their heights in
64. If the diagonal of a square is doubled, then its area will be 22
the ratio 1 : 2; Ratio of their radii is (Take )
(a) three times (b) four times 7
(c) same (d) none of these (a) 1: 4 (b) 1: 2
65. A metal pipe of negligible thickness has radius 21 cm and (c) (d) 1 : 2
2 :1
length 90 cm. The outer curved surface area of the pipe in 72. A rectangular piece of paper of dimensions 22 cm by 12 cm
square cm is is rolled along its length to form a cylinder. The volume
(a) 11880 (b) 11680 22
(c) 11480 (d) 10080 (in cm3) of the cylinder so formed is (use )
7
66. The base of a right pyramid is an equilateral triangle of side (a) 562 (b) 412
4 cm each. Each slant edge is 5 cm long. The volume of the (c) 462 (d) 362
pyramid is 73 . A sphere is placed inside a right circular cylinder so as to
4 8 4 60 touch the top, base and the lateral surface of the cylinder. If
(a) cm3 (b) cm3 the radius of the sphere is R, the volume of the cylinder is
3 3
(a) 2 R3 (b) 4 R3
4 59 4 61 3 8 3
(c) cm3 (d) cm (c) 8 R3 (d) R
3 3 3

ANSWER KEY
1 (a) 9 (d) 17 (a) 25 (a) 33 (a) 41 (c) 49 (a) 57 (b) 65 (a) 73 (a)
2 (d) 10 (a) 18 (a) 26 (c) 34 (b) 42 (c) 50 (b) 58 (c) 66 (c)
3 (a) 11 (d) 19 (c) 27 (a) 35 (c) 43 (a) 51 (d) 59 (c) 67 (c)
4 (a) 12 (a) 20 (c) 28 (a) 36 (a) 44 (a) 52 (c) 60 (c) 68 (b)
5 (a) 13 (a) 21 (a) 29 (d) 37 (a) 45 (b) 53 (b) 61 (a) 69 (d)
6 (b) 14 (c) 22 (a) 30 (a) 38 (c) 46 (c) 54 (d) 62 (c) 70 (d)
7 (a) 15 (a) 23 (b) 31 (d) 39 (c) 47 (d) 55 (b) 63 (d) 71 (c)
8 (a) 16 (b) 24 (b) 32 (c) 40 (c) 48 (d) 56 (b) 64 (b) 72 (c)
y
o
u
Mensuration B-165

rs
m
a
h
b
o
o
b
.w
1. (a) Let the length and breadth of a rectangle are 9 xm and

o
7. (a) 14 m

rd
5 xm respectively. D C
In a rectangle, area = length × breadth

p
re
720 = 9x × 5x 14 m

s
or x2 = 16 x=4

s
24 m

.c
Thus, length = 9 × 4 = 36 m

o
and breadth = 5 × 4 = 20 m

m
Therefore, perimeter of rectangle = 2(36 + 20) = 112 m 40 m
2. (d) Perimeter of the circle = perimeter of rectangle A B
2 r 2(18 26) Area of the shaded portion
22 1
2 r 88 14 2 = 154 m2
r = 14 4
7
Area of the circle 8. (a) Area of the square = (10)2 = 100 cm2
The largest possible circle would be as shown in the
2 22
= r 14 14 616 cm 2 . figure below :
7
3. (a) In a rectangle, S R

(perimeter) 2
(diagonal) 2 2 area
4 5 cm
10 cm
2
(14)
52 2 area
4
49 = 25 + 2 × area P Q
10 cm
49 25 24
Area 12cm 2
2 2 22 2 22 25
Area of the circle 5
7 7
22 25 22 11
Required ratio
7 100 28 14
4. (a)
4
= 0.785 0.8 =
5
Perimeter of triangle = a + a + 2 a = 20m 9. (d) Side of square carpet Area 169 13m
a(2 2) 20 After cutting of one side,
Measure of one side = 13 – 2 = 11 m
2 2 and other side = 13 m (remain same)
20
a 10 2 2 m Area of rectangular room = 13 × 11 = 143 m2
2 2 2 2
10. (a)
2.5''
1
Area of triangle a a 2.5''
2
20''
1
10 2 2 10 2 2
2
= 50 (4 + 2 – 4 2 ) 30''

= 100 (3 – 2 2 )m2 Length of frame = 30 + 2.5 × 2 = 35 inch


5. (a) In a parallelogram. Breadth of frame = 20 + 2.5 × 2 = 25 inch
Area = Diagonal × length of perpendicular on it. Now, area of picture = 30 × 20 = 600 sq. inch
= 30 × 20 = 600 m2 Area of frame = (35 × 2.5) + (25 × 2.5) = 150
6. (b) Required area covered in 5 revolutions 600
x 4times
22 150
= 5 × 2 rh = 5 × 2 × × 0.7 × 2 = 44 m2
7
y
o
u
B-166 Mensuration

rs
m
11. (d) 15. (a) Volume of the bucket = volume of the sand emptied
Volume of sand = (21)2 × 36

a
h
Let r be the radius of the conical heap.

b
o
10 W

o
1 2
21 2 36

b
15 Then, r 12

.w
3
or r2 = (21)2 × 9 or r = 21 × 3 = 63

o
rd
Let the width of the path = W m 16. (b) Let ABC be the isosceles triangle and AD be the altitude.

p
then, length of plot with path = (15 + 2W) m Let AB = AC = x. Then, BC = (32 – 2x).

re
and breadth of plot with path = (10 + 2 W) m

s
s
Therefore, Area of rectangular plot (wihout path) A

.c
= 15 × 10 = 150 m2

o
m
and Area of rectangular plot (with path)
= 150 + 54 = 204 m2
Hence, (15 + 2W) × (10 + 2W) = 204 x x
4W2 + 50 W – 54 = 0
2W2 + 25 W – 27 = 0
(W – 1) (2W + 27) = 0 B D C

27 Since, in an isosceles triangle, the altitude bisects the


Thus W = 1 or base. So, BD = DC = (16 – x).
2
with of the path = 1 m In ADC, AC2 = AD2 + DC2
12. (a) If area of a circle decreased by x % then the radius of a x2 = (8)2 + (16 – x)2
circle decreases by 32x = 320 x = 10.
BC = (32 – 2x) = (32 – 20) cm = 12 cm.
(100 10 100 x )% = (100 10 100 36)%
1
Hence, required area = BC AD
(100 10 64)% 2
100 80 20%
1
13. (a) Area of the outer rectangle = 19 × 16 = 304 m2 12 10 cm 2 60 cm 2 .
2
2m
17. (a) 7m

12
2m 2m
9m
15
Let the length of canal = h m.
2m
Then,
Area of the inner rectangle = 15 × 12 = 180 m2 1
Required area = (304 – 180) = 124 m2 area of canal h(9 7)
2
14. (c)
5m 1
or 1280 h(16)
2
5m 30 m 1280 2
h 160 m
16
18. (a) Let inner radius of the pipe be r.

Then, 440 22 2
70 m r 7 10
7
Total area of road
40
= Area of road which parallel to length + Area of road or r2 2
which parallel to breadth – overlapped road 22 10
= 70 × 5 + 30 × 5 – 5 × 5 or r 2m
= 350 + 150 – 25
= 500 – 25 = 475 m2 19. (c) Let the length and breadth of the original rectangular
Cost of gravelling the road field be x m and y m respectively.
= 475 × 4 = ` 1900 Area of the original field = x × y = 144 m2
y
o
u
Mensuration B-167

rs
m
Then, 2 (3x + 2x) = 1600 or x = 160.
144

a
x … (i) Length = 480 m and Breadth = 320 m.

h
y Area = (480 × 320) m2 = 153600 m2.

b
o
If the length had been 6 m more, then area will be 25. (a) Volume of the water running through pipe per hour

o
b
(x + 6) y = 144 + 54

.w
20 20
(x + 6) y = 198 … (ii) 15000 600 cubic metre
100 100

o
Putting the value of x from eq (i) in eq (ii), we get

rd
60 6.5 80

p
144 Required time 52 hours

re
6 y 198 600

s
y

s
26. (c) Let h = 2x metres and (l + b) = 5x metres.

.c
144 6y 198

o
Total cost 260

m
6y = 54 y= 9 m Length of the paper = Rate per m m = 130 m.
2
Putting the value of y in eq (i) we get x = 16 m
20. (c) Clearly, we have : l = 9 and l + 2b = 37 or b = 14. 50
Area = (l × b) = (9 × 14) sq. ft. = 126 sq. ft. Area of the paper = 130 m 2 65 m 2 .
100
21. (a) When folded along breadth, we have :
Total area of 4 walls = (65 + 15) m2 = 80 m2.
l 2(l + b) × h = 80 2 × 5x × 2x = 80
2 b 34 or l + 2b = 34 ...(i)
2 x2 = 4 x = 2.
When folded along length, we have : Height of the room = 4 m.
27. (a) Let each wheel make x revolutions per sec. Then,
b
2 l+ 38 or 2l + b = 38 ....(ii) 7
2 2 x (2 7 x) 10 1980
2
Solving (i) and (ii), we get :
l = 14 and b = 10. 22 22
Area of the paper = (14 × 10) cm2 = 140 cm2. 7 x 2 7 x 198
22. (a) Area left after laying black tiles 7 7
= [(20 – 4) × (10 – 4)] sq. ft. = 96 sq. ft. 66x = 198 x = 3.
Distance moved by smaller wheel in 3 revolutions
1
Area under white tiles 96 sq. ft = 32 sq. ft. 22 7
3 2 3 cm = 66 cm.
Area under blue tiles = (96 – 32) sq. ft = 64 sq. ft. 7 2

64 66
Number of blue tiles = 16. Speed of smaller wheel = cm/s = 22 cm/s.
(2 2) 3
28. (a) Let the edge of the third cube be x cm.
23. (b) Then, x3 + 63 + 83 = 123
7 7 x3 + 216 + 512 = 1728
x3 = 1000 x = 10.
7 7 Thus the edge of third cube = 10 cm.
29. (d) Let the length, breadth and height of the cuboid be x,
7 7 2x and 3x, respectively.
Therefore, volume = x × 2x × 3x = 6x3
New length, breadth and height = 2x, 6x and 9x,
respectively.
New volume = 108x3
The shaded area gives the required region.
Thus, increase in volume = (108 – 6)x3 = 102 x3
Area of the shaded region = Area of the square – area
of four quadrants of the circles Increase in volume 102x 3
17
1 Original volume 6x 3
= (14)2 – 4 (7)2
4 30. (a) In a cube,
Area = 6 (side)2
22
196 – 49 196 – 154 42 cm 2 or 150 = 6 (side)2
7
24. (b) Perimeter = Distance covered in 8 min. side = 25 5m

12000 Length of diagonal = 3 side 5 3 m


8 m 1600 m.
60 31. (d) Let the length of the wire be h cm.
Let length = 3x metres and breadth = 2x metres. and radius of sphere and wire are R and r respectively.
y
o
u
B-168 Mensuration

rs
Then, volume of sphere = volume of wire (cylinder)

m
37. (a) Let radius of the 3rd spherical ball be R,

a
4

h
3 3
or R3 r 2h 4 3 4 3 4 4 3

b
3 (1)3 R

o
3 2 3 4 3 3

o
b
4 3
or R r 2h

.w
3 3 3
3 3

o
R3 13

rd
4 2 4
or (3)3 (0.1) h 2

p
3

re
s
3
4 (3)3 27 27 125 5

s
108 5

.c
h 3600 cm = 36 m 1 R 1.25
2 0.03 8 64 64 4

o
3 (0.1) 4

m
32. (c) Let the depth of the drainlet be h metres. Diameter of the third spherical ball
Volume of the earth dug from the drainlet 10 m wide = 1.25 × 2 = 2.5 cm.
= h[260 × 200 – 240 × 180] = 8800 h cu. m. 38. (c) Let 'A' be the side of bigger cube and 'a' be the side of
Now this is spread over the plot raising its height by smaller cube
25 cm, Surface area of bigger cube = 6 A2
1 or 384 = 6A2
i.e., m. A = 8 cm.
4
Surface area of smaller cube = 6 a 2
1 96 = 6a2
8800 h = 240 × 180 ×
4 a = 4 mm = 0.4 cm
60 180 27 Volume of bigger cube
h= So, Number of small cube
8800 22 Volume of smaller cube
h = 1.227 m.
33. (a) Let height will be h cm. (8)3 512
Volume of water in roof = Volume of water in cylinder 3
8,000
(0.4) 0.064
9 10000 0.1
h 39. (c) Volume of the liquid in the cylindrical vessel
900 10 = Volume of the conical vessel
h = 1 cm
34. (b) Let be the length and b be the breadth of cold 1 22
= 12 12 50 cm3
storage. 3 7
L = 2B, H = 3 metres
Area of four walls = 2[L × H + B × H] = 108 22 4 12 50
= cm3.
6BH = 108 7
L = 12, B = 6, H = 3
Let the height of the liquid in the vessel be h.
Volume = 12 × 6 × 3 = 216 m3
35. (c) Volume of cylinder = ( × 6 × 6 ×28)cm3 = (36 × 28) cm3. 22 22 4 12 50
Then, 10 10 h
4 3 3 3 7 7
Volume of each bullet = cm3
3 4 4 4 4 12 50
or h = 24 cm.
9 10 10
= cm3.
16 40. (c) B
Volume of cylinder
Number of bullets =
Volume of each bullet 1.5 m
16 O
A C
= (36 28) 1792. 1.5 m 1.5 m
9
36. (a) Volume of water in the reservoir
= area of empty pipe × Empty rate × time to empty
2 D
1 From AOB ,
or 54 × 44 × 10 3 20 empty time
100
AB = 1.52 1.52 2.25 2.25 = 4.50
54×44×10×100×100×7
or Empty time = sec. Area of the square base of the trunk of the tree
22×20×9
= 4.50 4.50 = 4.50 m2
54 44 10 100 100 7 Volume of the timber = Area of base × height
hrs = 116.67 hours.
22 20 9 3600 = 4.50 × 10 = 45 m3
y
o
u
Mensuration B-169

rs
Surface area of the cube = (6 ×82) sq. ft. = 384 sq. ft.

m
41. (c) Slant height of cone = radius of the sheet = 14 cm
142 = (4.45)2 + h2

a
384

h
kg 24 kg. or h2 = 196 – 19.80 = 176.20

b
Quantity of paint required =
16

o
h = 13.27 cm

o
Cost of painting = ` (36.50 × 24) = ` 876.

b
49. (a) Let length, breadth and height of the room be , b and

.w
42. (c) Let the radius of the base are 5k and 12k respectively
h, respectively.

o
Total surface area of the cylinder

rd
Then, area of four walls of the room

p
Total surface area of the cone

re
340.20
2 b h 252m 2

s
2 1.35
2 r h 2 r

s
.c
= ( + b)h = 126 ...(i)
r r2 h2 r2

o
m
91.8
2h 2r 24k 10k And b 108
= + 0.85
r2 h2 r 25k 2 144k 2 5k 12 × b = 108 ( = 12 m)
34k 34k 17 b= 9m
= = 126
13k 5k 18k 9 Using (i), we get, h 6m
43. (a) Number of discharge pipe 21
50. (b) Volume of material in the sphere
Volume of water supply pipe
Volume of water in each discharge pipe 4 4
= (4)3 (2)3 cm3 56 cm3.
3 3
(3)2 1 Let the height of the cone be h cm.
4 Since the velocity of water is same
2
3 1 4
1 4 4 h 56
2 Then,
3 3
44. (a) Total surface area of the remaining solid = Curved
surface area of the cylinder + Area of the base + Curved 4 56
h 14 cm.
surface area of the cone 4 4
= 2 rh + r2 + r
= 2 × 8 × 15 + × (8)2 + × 8 × 17 4
51. (d) Volume of sphere = 9 9 9 cm3.
= 240 + 64 + 136 3
= 440 cm2
45. (b) L × B × 2 = 48 1
Volume of cone = 9 9 9 cm3.
L × B = 24 3
Now, 6 – 6 × 10% = 5.4, Volume of wood wasted
5 – 5 × 10% = 4.5 and
Therefore, 5.4 × 4.5 = 24.3 4 1
= 9 9 9 9 9 9 cm3 .
Clearly, 5 < L < 5.5 3 3
22 7 7 = ( × 9 × 9 × 9) cm3
46. (c) Volume of one coin = 200 cm3 =7700 cm3.
7 2 2
Volume of water flown in 10 min. = (7700 × 60 ×10) cm3
9 9 9
Required percentage = 100 %
7700 60 10 4
= litres 9 9 9
1000 3
= 4620 litres.
47. (d) 4 (r + 2)2 – 4 r2 = 352 3
= 100 % 75%.
7 1 4
(r + 2)2 – r2 = 352 28. 52. (c) Let the height of the vessel be x.
22 4
Then, radius of the bowl = radius of the vessel = x/2.
(r + 2 + r)(r + 2 – r) = 28
3
28 2 x 1 3
2r + 2 = 2r + 2 = 14 r = 6 cm Volume of the bowl, V1 x .
2 3 2 12
48. (d) Circumference of the base of ice-cream cup
2
= Diameter of the sheet = 28 cm x 1 3
Volume of the vessel, V2 x x .
2 r 28 2 4
14 Since V2 > V1, so the vessel can contain 100% of the
r cm 4.45 cm beverage filled in the bowl.
y
o
u
B-170 Mensuration

rs
m
53. (b) Volume of the cylinder container Now, labour charges at the rate of 50 paise
= × 62 × 15 cu. cm

a
…(1) per sq. m = 314 × 0.5 = 157.0

h
Let the radius of the base of the cone be r cm, ` 157

b
o
then, height of the cone = 4r cm 57. (b)

o
b
Volume of the 10 cylindrical cones of ice-cream

.w
with hemispherical tops

o
rd
1 2 3
10 r 2 4r 10 r

p
3 3

re
s
40 3 20 3

s
.c
r r = 20 r3 cu. cm …(2) Side of the inner square = 55 – 10 = 45
3 3

o
Area of inner square = 45 × 45 = 2025 sq. m.

m
Since the whole ice-cream in the cylindrical container
58. (c) Let the kerosene level of cylindrical jar be h.
is distributed among 10 children in cones with
hemispherical tops, 1 2
Now, Volume of conical vessel = r h
(1) and (2), gives 3
× 62 × 15 = 20 r3 Since, radius (r) = 2 cm and height(h) = 3cm of conical
36 15 vessel.
r3 = = 27 r = 3 cm
20 1
Volume = 4 3 4
54. (d) Let the original volume of cylinder be V . 3
4 4 Now, Volume of cylinderical jar = r2h
When it is filled , then it’s volume = V = (2)2h
5 5 =4 h
When cylinder is filled, the level of water coincides Now, Volume of conical vessel = Volume of cylindrical
with opposite sides of bottom and top edges then Jar
1 4 =4 h
Volume become = V h = 1cm
2
Hence, kerosene level in Jar is 1 cm.
Since, in this process 30 cc of the water is spilled,
59. (c) Given, length of garden = 24 m and
therefore
breadth of garden = 14 m
4 1 Area of the garden = 24 × 14 m2 = 336 m2.
V - 30 = V
5 2 Since, there is 1 m wide path outside the garden
Area of Garden (including path)
4 1 = ( 24 + 2) × ( 14 + 2) = 26 × 16 m2 = 416 m2.
Þ V - V = 30
5 2 Now, Area of Path = Area of garden( inculding path)
– Area of Garden
Þ V (3/10) = 30 = 416 – 336 = 80 m2.
Þ V = 100 cc Now , Area of Marbles = 20 × 20 = 400 cm 2
55. (b) Radius of the inner track = 100 m
and time = 1 min 30 sec 90 sec. Area of Path
Marbles required = Area of Marbles
Also, Radius of the outer track = 102 m
and time = 1 min 32 sec 92 sec.
Now, speed of A who runs on the inner track 80, 0000
= 2000
400
2 (100) 20
= 6.98 m/s 60. (c) Volume of rain that is to be collected
90 9
10 1
And speed of B who runs on the outer track in a pool = 2 1 10
2
2 (102) 51 = 1010 cm3 = 10 4 meter3
= 6.96 m/s
90 23 Volume of Pool = L × B × h
Since, speed of A > speed of B 104 = 100 × 10 × h
A runs faster than B.
56. (b) Curved surface area of cylinder = 2 rh 104
h= 10 m .
Surface area of 50 cylindrical pillars = 50 × 2 rh 100 10
Now, Diameter of each cylindrical pillar = 50 cm 61. (a) Let the rise in water level = x m
50 Now, volume of pool = 40 × 90 × x = 3600 x
Radius = = 25 cm = 0.25 m When 150 men take a dip, then displacement of
2 water = 8m3
Also, height = 4m
Surface area = 50 × 2 × 3.14 × 0.25 × 4 3600 x 900
8 x 2 x 0.33m
= 314 × 1 sq m. 150 150
= 314 sq. m. x = 33.33 cm
y
o
u
Mensuration B-171

rs
m
68. (b) Let r be the radius of circle.
r2 = 154 cm2

a
h
62. (c)
D C

b
154

o
m r2 = 7 = 49
8c

o
22

b
O

.w
m r = 7 cm
8c

o
length of wire = circumference of circle

rd
A

p
B 22

re
=2 7 = 44 cm
7

s
s
Now, Perimeter of equilateral triangle = 44 cm

.c
Diagonal of square = Diameter of circle

o
44

m
2 side of square = 16 cm side = cm
Squaring on both sides 3
2 2
2 sides of square 16 2 3 44
Area of equilateral triangle =
4 3
16 16
(side of square)2 = 484 3
2 = = 91.42 cm2
Area of square = 128 sq. cm 9
63. (d) The area of circle is 9 cm2. Area of equilateral triangle is nearly equal to
64. (b) Diagonal of a square (d) = 90.14 cm2
2 × side of square (a).
Hence, option (b) is correct.
d 69. (d) Total surface area of cone = r(l + r)
d= 2a a
2 22
S 3 32 42 3
2
7
d
Area of square a2 = 22 528
2 3 8
Now, diagonal gests doubled 7 7
S = 75.4 sq.cm
(2d )
a 800 700 600 cm3
2 70. (d) Maximum number of boxes
8 7 6 cm3
2
2d d = 1000000
Area of square 4
2 2 71. (c) r12 h1 r22 h 2

d2 r1 h2 2
is area of square
2 r2 h1 1
Therefore, Area will be four temes.
65. (a) Curved surface area of cylinder = 2 rh r1 : r2 2 :1
72. (c) 2 r = 22 cm
22
2 21 90 11880 sq.cm
7 22 7 7
r cm
67. (a) C1 = 2C2 2 22 2
r 1 l1 = 2 r 2 l2 Height, h = 12 cm
also, l2 = 2l1 22 7 7
r 1 l 1 = 2 × 26 r 2 l 1 Volume of cylinder 12 462 cm3
7 2 2
r1 4 73. (a) Radius of cylinder = Radius of sphere = R
r2 1 Height of cylinder = 2 R
Volume of cylinder = R2 × (2R) = 2 R3
y
o
u
B-172 Trigonometry

rs
m
a
CHAPTER

h
b
13

o
TRIGONOMETRY

o
b
.w
o
rd
p
re
s
s
.c
In this chapter we intend to study an important branch of 5

o
mathematics called ‘trigonometry’. It is the science of measuring r1 s and r2 s

m
angle of triangles, side of triangles. 3 12
Angle : - 5
r1 r2
B 3 12
4r1 = 5r 2 r1 : r2 = 5 : 4
Trigonometric ratios :
The most important task of trigonometry is to find the remaining
O A side and angle of a triangle when some of its side and angles are
given. This problem is solved by using some ratio of sides of a
Consider a ray OA if this ray rotate about its end point O and takes the triangle with respect to its acute angle. These ratio of acute angle
position OB then we say that the angle AOB has been generated. are called trigonometric ratio of angle. Let us now define various
Measure of an angle : The measure of an angle is the amount of trigonometric ratio.
rotation from initial side to the terminal side.
Y
NOTE :
P
Relation between degree and radian measurement
radians = 180 degree r
17 y
radian measure = ×degree measure
180
180
degree measure = ×radian measure A x M X
1° = 60’ (60 minutes) Consider an acute angle YAX = with initial side AX and terminal
1’ = 60’’ (60 seconds) side AY. Draw PM perpendicular from P on AX to get right angle
Example 1 : triangle AMP. In right angle triangle AMP.
Find radian measure of 270°. Base = AM = x
Solution : Perpendicular = PM = y and
Hypotenuse = AP = r.
3
Radian meausre = 270 r2 = x2 + y2
180 2
Example 2 : r x2 y2
5 We define the following six trigonometric Ratios:
Find degree measure of .
9
Solution : Perpendicular y
sin =
180 5 Hypotenuse r
degree measure 100
9 Base x
Example 3 : cos =
If the arcs of same length in two circles subtend angles of Hypotenuse r
60° and 75° at their centres. Find the ratio of their radii.
Solution : Perpendicular y
tan =
c c Base x
60 60 and Hypotenuse r
180 3 cosec =
c c Perpendicular y
5
75 75
180 12 Hypotenuse r
sec =
c
Base x
s 5 s s
and Base x
3 r1 12 r2 r cot =
Perpendicular y
y
o
u
Trigonometry B-173

rs
Important formula:-

m
1. sin2 + cos2 = 1 2. cosec2 – cot2 =1 3. sec2 + tan2 = 1

a
h
b
4.

o
o
b
0° 30° 45° 60° 90°

.w
T-ratio

o
rd
sin 0 1 1 3 1

p
re
2 2 2

s
s
.c
3 1 1
cos 1 0

o
2

m
2 2

1 Not
tan 0 1 3
3 defined

Not 2
cosec 2 2 1
defined 3

2 Not
sec 1 2 2
defined
3

Not 1
cot 3 1 0
defined
3

5. sin (90° – ) = cos . 6. cos (90° – ) = sin . 7. tan (90° – ) = cot cot (90° – ) = tan .
8. cosec (90° – ) = sec . 9. sec (90° – ) = cosec .

RELATION AMONG T-RATIONS

sin cos tan cot sec cosec

tan 1 sec 2 1 1
sin sin 1 cos 2 2 2 cosec
1 tan 1 cot sec

1
1 cot 1
cos cos cosec 2 -1
1 sin 2 1 tan 2
1 cot 2
sec cosec

sin 1 cos 2 1 1
tan 2 tan sec 2 1
1 sin cos cot cosec 2 1

1 sin 2 cos 1 1
cot 2 cot cosec2 –1
sin 1 cos tan sec 2 1

1 1 1 cot 2 cosec
sec 2 1 tan 2 sec
1 sin cos cot cosec 2 –1

1 1 1 tan 2 sec
cosec 2 1 cot 2 cosec
sin 1 cos tan sec 2 1
y
o
u
B-174 Trigonometry

rs
m
Example 4 : Now

a
In a ABC right angled at B if AB = 12, and BC = 5 find BC 4 5

h
sin A and tan A, cos C and cot C sin C = , cosec C =

b
AC 5 4

o
Solution :

o
b
3 AB 5
, sec C =

.w
C cos C =
5 AC 3

o
rd
13 AB 4 3
tan C = , cot C = .

p
5 AC 3 4

re
s
Example 6 :

s
.c
B 12 A Find the value of 2 sin2 30° tan 60° – 3cos2 60° sec2 30°

o
Solution :

m
AC = (AB) 2 + (BC) 2 2 2 2
1 1 2
2 3 3
2 2 2 2 3
= 12 5
= 144 25 1 1 4 3 3 2
=2 3 3 1
4 4 3 2 2
= 169 13 Example 7 :
When we consider t-ratios of A we have
Base AB = 12 Find the value 2 sin 2 = 3
Perpendicular = BC = 5 Solution :
Hypotenuse = AC = 13
3
Perpendicular 5 sin 2 =
sin A = 2
Hypotenuse 13 2 = 60
= 30°
Perpendicular 5
tan A = Example 8 :
Base 12
Find the value of x.
When we consider t-ratios of C, we have tan 3x = sin 45° cos 45° + sin 30°
Base = BC = 5
Solution :
Perpendicular = AB = 12
Hypotenuse = AC = 13 1 1 1
tan 3x =
2 2 2
Base 5
cos C = 1 1
Hypotenuse 13 = 1
2 2
Base 5 tan 3x = 1 tan 3x = tan 45°
cot C =
Perpendicular 12 3x = 45°
Example 5 : x = 15°
3 Example 9 :
In a right triangle ABC right angle at B if sin A = find all If is an acute angle tan + cot = 2 find the value of tan 7
5
the six trigonometric ratios of C + cot7 .
Solution : Solution :
tan + cos = 2
Perpendicular 3
sin A = 1
Hypotenuse 5 tan + 2
tan
C tan2
+ 1 = 2 tan
tan2
– 2 tan + 1 = 0
5 – 1)2 = 0
(tan
3 tan
=1
= 45°
4 A Now, tan7 + cot7 .
B
= tan7 45° + cot7 45°
2 2 =1+ 1=2
Base = Hypotenuse – Perpendicualr Example 10 :

= 52 32 cos37º
Find the value of
sin53º
= 25 9 = 16 4
y
o
u
Trigonometry B-175

rs
m
Solution : Solution :

a
We have We have

h
b
cos 37 cos(90 53 ) sin 53 1 sin 1 sin

o
1 1 1

o
sin 53 sin 53 sin 53 = 1 sin 1 sin

b
1 sin 1 sin

.w
o
Example 11 : 2 2
2sec 2 .

rd
Find the value of = 2 2
1 sin cos

p
re
sin 36 sin 54

s
Example 16 :

s
cos 54 cos 36

.c
o
Solution : 1 sin

m
We have Find the value of
1 sin
sin 36 sin 54 Solution :
=
cos 54 cos 36
1 sin 1 sin 1 sin
sin(90 54 ) sin(90 36 )
= 1 sin 1 sin 1 sin
cos 54 cos 36
cos 54 cos 36 1 sin
2
=
cos 54 cos 36 =
1 sin 2
=1–1=0
1 sin 1 sin
Example 12 : = = = sec – tan .
cos cos cos
Evaluate the cot 12° cot 38° cot 52° cot 60° cot 78°
Solution : Example 17 :
We have Find the value of [(1 + cot ) – cosec ] [1 + tan + sec ]
cot 12° cot 38° cot 52° cot 60° cot 78° Solution :
= (cot 12° cot 78°) (cot 38° cot 52°) cot 60° (1 + cot – cosec ) (1 + tan + sec )
= [cot12° cot (90° – 12°)] [cot 38° cot (90° – 38°)] cot 60° cos 1 sin 1
= [cot 12° tan 12°] [cot 38° tan 38°] cot 60° = 1 1
sin sin cos cos
1 1
=1 1 sin cos 1 cos sin 1
3 3 =
sin cos
Example 13 :
If tan 2 = cot ( + 6°), where 2 and + 6° are acute angles 2
sin cos 1
find the value of . =
Solution : sin cos
We have
tan 2 = cot ( + 6°) sin 2 cos 2 2sin cos 1
=
cot (90° – 2 ) = cot ( + 6°) sin cos
90 – 2 = + 6°
3 = 84° 1 2sin cos 1 2 sin cos
= = =2
= 28° sin cos sin cos
Example 18 :
Example 14 :
3
Find the value of (1 – sin 2 ) sec2 . If sin , find the value of sin cos .
Solution : 5
We have, Solution :
(1 – sin2 ) (sec2 ) 3
= cos2 sec2 sin =
5
1
= cos2 × cos 1 sin 2
cos 2
=1 2
3 4
= 1
Example 15 : 5 5

1 1 3 4 12
find its value sin × cos =
1 sin 1 sin 5 5 25
y
o
u
B-176 Trigonometry

rs
m
Example 19 : them easily, we can determine these by using trigonometric ratios.

a
2 sec Line of Sight

h
1 .
If cos = , find the value if

b
The line of sight or the line of vision is a straight line from our eye
2 1 tan 2

o
to the object we are viewing.

o
Solution :

b
If the object is above the horizontal from the eye, we have to lift

.w
1 up our head to view the object. In this process, our eye move

o
cos

rd
2 through an angle. This angle is called the angle of elevation of

p
sec = 2 the object.

re
s
2sec 2sec 2 2

s
1

.c
2 2 sec 2
1 tan sec

o
m
Example 20 :
12 1 sin
If tan , find the value of
5 1 sin
Solution :
12
tan Eye
5 Angle of elevation

sec 1 tan 2 If the object is below the horizontal from the eye, then we have to
turn our head downwards to view the object. In this process, our
2 eye move through an angle. This angle is called the angle of
12 13
= 1 = depression of the object.
5 5

5 Angle of depression
cos
13
12 Eye
sin 1 cos 2 Li
13 ne
of
sig
12 25 ht
1
1 sin 13 13
thus 25
1 sin 12 1
1
13 13
Example 21 :
a Ball
If sin 0 90 find the value of
2 2
a b
tan . Example 22 :
Solution : A person observed the angle of elevation of the top of a
tower is 30°. He walked 40 m towards the foot of the tower
a along level ground and found the angle of elevation of the
sin
2 top of the tower as 60° Find the height of tower.
a b2
Solution :
cos 1 sin 2 Let height of tower AB = x m and BC = y m, DC = 40 m.
In ABC,
a2 b2 b
cos 1 A
2 2 2 2 2
a b a b a b2
a
sin a 2
b2 a x
tan =
cos b b
a2 b2 30° 60°
D 40m C y B
HEIGHT AND DISTANCE
Sometimes, we have to find the height of a tower, building, tree,
distance of a ship, width of a river, etc. Though we cannot measure
y
o
u
Trigonometry B-177

rs
m
AB x x y x

a
tan 60 3 y ....(i) In AOB, cot 30

h
BC y 3 100

b
o
o
AB y x 100 3 y 100 3 x

b
Now In rt ABD, tan 30

.w
BD
y 100 3 100 [using (i)]

o
rd
x 1

p
40 y y 100( 3 1)
3

re
s
y 100(1.732 1) = 100 0.732 = 73.20 m.

s
x

.c
3x 40 y 3x 40 [using (i)] Example 24 :

o
3

m
A 25 m long ladder is placed against a vertical wall of a
building. The foot of the ladder is 7 m from the base of the
3x 40 3 x 3x x 40 3 2x 40 3 building. If the top of the ladder slips 4 m, then the foot of
the ladder will slide by how much distance.
x 20 3m
Solution :
Example 23 : Let the height of the wall be h.
As observed from top of a light house 100 m. high above
sea level, the angle of depression of a ship sailing directly P
toward it changes from 30° to 45°. The distance travelled by
the ship during the period of observation is 4
Solution :
Let ‘y’ be the required distance between two positions O 25
h
and C of the ship
In rt. ABC,
R
A Top Q 7 x S
30°
45°
Now, h 252 – 7 2
Light house
100 m 576 24m

30° QS 625 – 400


45°
O y C x B 225 15m
Required distance, x = (15 – 7) = 8 m
x
cot 45° = x 100 …(i)
100
y
o
u
B-178 Trigonometry

rs
m
a
h
b
o
o
b
.w
o
rd
1. If tan = 1, then find the value of
1 cosec 2 sec 2

p
8sin 5sin 10. If tan = , then ?

re
3 3 7 cosec2 sec2

s
sin 2 cos 7 cos

s
5 3

.c
1 (a) (b)

o
(a) 2 (b) 2 7 7

m
2
1 3
4 (c) (d)
(c) 3 (d) 12 4
5
2. If be a positive acute angle satisfying cos2 + cos4 = 1, 2sin
then the value of tan2 + tan4 is 11. If y then 1 cos sin is equal to
1 cos sin 1 sin
3 (a) 1/y (b) y
(a) (b) 1
2 (c) 1 – y (d) 1 + y
1 12. A person, standing on the bank of a river, observes that the
(c) (d) 0 angle subtended by a tree on the opposite bank is 60°; when
2 he retreates 20m from the bank, he finds the angle to be 30°.
3. The value of tan4°. tan43°. tan47°. tan86° is The height of the tree and the breadth of the river are –
(a) 0 (b) 1
(a) 10 3 m, 10 m (b) 10 ; 10 3 m
1
(c) 3 (d) (c) 20 m, 30 m (d) None of these
3
8
4. If tan 15° = 2 3 , the value of tan 15°. cot 75° + tan 75° . 13. If is an acute angle such that tan 2 = , then the value of
7
cot 15° is.
(a) 14 (b) 12 1 sin 1 sin
is
(c) 10 (d) 8 1 cos 1 cos
5. The value of (sin 2 1° + sin 2 3° + sin 2 5° + .......+
sin 2 85° + sin 2 87° + sin 2 89°) 7 8
(a) (b)
1 8 7
(a) 21 (b) 22
2 7 64
(c) (d)
1 1 4 49
(c) 22 (d) 23
2 2 14. If 3 cos = 5 sin , then the value of
7 5sin 2 sec3 2 cos
6. If sin – cos = and 0 < < 90°, then the value of sin is equal to
13 5sin 2sec 3
2 cos
+ cos is.
271 376
17 13 (a) (b)
(a) (b) 979 2937
13 17
542
1 1 (c) (d) None of these
(c) (d) 2937
13 17
7. The minimum value of cos 2 + cos for real values of is– 15. If x sin 3 y cos 3 sin cos an d x sin y cos ,
(a) – 9/8 (b) 0
(c) –2 (d) None of these then x 2 y 2
5sin 4 cos (a) 1 (b) 2 (c) 0 (d) None
8. If 5 tan – 4 = 0, then the value of ? 16. If 1 + sin2 A = 3 sin A cos A, then what are the possible
5sin 4 cos
values of tan A?
5 5 (a) 1/4, 2 (b) 1/6, 3
(a) (b) (c) 1/2, 1 (d) 1/8, 4
3 6
1 cos3 20 cos3 70º
(c) 0 (d) 17. The value of is
6 sin 3 70 sin 3 20
9. The value of tan 6 20° – 33 tan4 20° + 27 tan 2 20° is :
(a) 2 (b) 3 1 1
(a) (b) (c) 1 (d) 2
(c) 4 (d) 5 2 2
y
o
u
Trigonometry B-179

rs
m
x cosec2 30 . sec2 45 tan tan

a
18. If = tan2 60° – tan2 30º, then x = ? 28. is equal to

h
8 cos 2 45 . sin 2 60 sec 1 sec 1

b
o
(a) 2 tan (b) 2 sec
(a) 1 (b) – 1

o
(c) 2 cosec (d) 2 tan . sec .

b
(c) 2 (d) 0

.w
29. If a cos + b sin = m and a sin – b cos = n, then
a2 + b2 =

o
19. If + = , what is the value of ( 3 tan )

rd
6 (a) m2 – n2 (b) m2 n2

p
2
(c) n – m 2 (d) m2 + n2

re
( 3 tan ) ? 30. The angular elevation of a tower CD at a place A due south

s
s
(a) 1 (b) – 1 of it is 60° ; and at a place B due west of A, the elevation is

.c
(c) 4 (d) – 4 30°. If AB = 3 km, the height of the tower is

o
m
20. If is an acute angle such that sec2 = 3, then (a) 2 3 km (b) 3 6 km
2 2
tan cosec
2 is 3 3 3 6
tan cosec 2 (c) km (d) km
2 4
4 3 2 1
(a) (b) (c) (d) 2
7 7 7 7 31. If sin + cos = 2 (0°< < < 90°), then sin
3
21. What should be the height of a flag where a 20 feet long
ladder reaches 20 feet below the flag (The angle of elevation
of the top of the flag at the foot of the ladder is 60°)? (a) sin (b) cos
2 3
(a) 20 feet (b) 30 feet
2
(c) 40 feet (d) 20 2 feet (c) sin (d) cos
3 3
22. If & 2 – 45° are acute angles such that sin = cos (2 – 45°)
then tan is equal to 2
(a) 1 (b) – 1 32. If cos4 – sin4 , then the value of 2 cos2 – 1 is
3
1 (a) 0 (b) 1
(c) 3 (d) 2 3
3
(c) (d)
23. If 5 & 4 are acute angles satisfying sin 5 = cos 4 then 2 3 2
sin 3 – 3 tan 3 = ? 33. If sin sec (30° + ) = 1 (0 < < 60°), then the value of
sin + cos 2 is
(a) 1 (b) 0
1 2 3
(c) – 1 (d) (a) 1 (b)
3 2 3
24. A vertical pole with height more than 100 m consists of two (c) 0 (d) 2
parts, the lower being one-third of the whole. At a point on 34. The minimum value of 2 sin 2 + 3 cos2 is
a horizontal plane through the foot and 40 m from it, the (a) 0 (b) 3
1 (c) 2 (d) 1
upper part subtends an angle whose tangent is . What is
2 1
the height of the pole ? 35. If cosec 39° = x, the value of + sin2 39°+ tan2 51°
cosec 2 51
(a) 110 m (b) 200 m
(c) 120 m (d) 150 m 1
25. If sec + tan = x, then sec = ? is
sin 51 sec 2 39
2

x2 1 x2 1 (a) (b)
(a) (b) x2 1 1 x2
x 2x 2
(c) x – 1 (d) 1 – x2
36. 2 4
If A = sin + cos , for any value of , then the value of A is
x2 1 x2 1
(c) (d) 3
2x x (a) 1 < A < 2 (b) <A< 2
26. The correct value of the parameter ‘t’ of the identity 4
2( sin6x + cos6x) + t (sin4x + cos4x) = –1 is: 13 3 13
(a) 0 (b) –1 (c) <A<1 (d) <A<
16 4 16
(c) –2 (d) –3
37. If tan 2 , tan 4 = 1, then the value of tan 3 is
27. If a cos – b sin = c, then a cos + b sin = ?
(a) 3 (b) 0
(a) a 2 b2 c2 (b) a 2 b 2 c2
1
(c) 2 2 2 (d) None of these (c) 1 (d)
c a b 3
y
o
u
B-180 Trigonometry

rs
m
2 48. If sec tan 3 , then the positive value of sin is

a
38. If tan ( 1 + 2) = 3 and sec ( 1 – 2) , then the value

h
3 1

b
(a) 0 (b)

o
of sin 1 + tan 3 2 is equal to (assume that 0 < 1– 2< 1 2

o
b
+ 2 < 90°)

.w
(a) 0 (b) 3 3
(c) (d) 1

o
(c) 1 (d) 2 2

rd
49. The radian measure of 63 14 51 is

p
1

re
39. If sec = x (0° < < 90°), then sec + tan is equal to
4x c c

s
2811 3811

s
(a) (b)

.c
x 8000 8000

o
(a) (b) 2x

m
2
c c
4811 5811
1 (c) (d)
(c) x (d) 8000 8000
2x
40. If x = a sec cos , y = b sec sin , z = c tan , then, the cos 4 sin 4 cos 4 sin 4
2 2 2
50. If 1 , then the value of
x y z cos 2 sin 2 cos 2 sin 2
value of 2 2 2 is :
a b c is
(a) 1 (b) 4
1
(c) 9 (d) 0 (a) 4 (b) 0 (c) (d) 1
8
sec tan 5
41. If , then sin is equal to : sin cos 1
sec tan 3 51. where is equal to
sin cos 1 2
1 1 2 3
(a) (b) (c) (d) 1 sin 1 sin
4 3 3 4 (a) (b)
cos cos
42. If 0 and sec 2 tan 2 7 , then is 1 cos 1 cos
2 (c) (d)
sin sin
5
(a) radian (b) radian 52. The angles of elevation of the top of a tower standing on a
12 3 horizontal plane from two points on a line passing through
the foot of the tower at a distance 9 ft and 16 ft respectively
(c) radian (d) radian are complementary angles. Then the height of the tower is
5 6
(a) 9 ft (b) 12 ft
43. The simplest value of (c) 16 ft (d) 144 ft
sin2x + 2 tan2x – 2 sec2 x + cos2 x is 53. If sin2 = cos3 , then the value of (cot6 – cot2 ) is
(a) 1 (b) 0 (a) 1 (b) 0
(c) –1 (d) 2 (c) – 1 (d) 2
44. A kite is flying at a height of 50 metre. If the length of string 54. The simplified value of
is 100 metre then the inclination of string to the horizontal (1 + tan + sec ) (1 + cot – cosec ) is
ground in degree measure is (a) – 2 (b) 2
(a) 90 (b) 60 (c) 1 (d) – 1
(c) 45 (d) 30
sin 53° cot 65
45. From the top of a light-house at a height 20 metres above 55. The value of is
sea-level, the angle of depression of a ship is 30°. The cos 37° tan 25
distance of the ship from the foot of the light-house is (a) 2 (b) l
(c) 3 (d) 0
(a) 20 m (b) 20 3 m
cos 60° + sin 60°
(c) 30 m (d) 30 3 m 56. The value of is
cos 60° – sin 60°
a2 b2 (a) – 1 (b) 3 +2
46. If x = a sin and y = b tan then prove that 2 2 is
x y (c) – (2 + 3 ) (d) 3 –2
(a) 1 (b) 2 57. The value of
(c) 3 (d) 4
cot 5 .cot10 .cot15 .cot 60 .cot 75 .cot 80 .cot 85
47. If 2y cos = x sin and 2x sec – y cosec = 3, then the is
relation between x and y is (cos 2 20 cos2 70 ) 2
(a) 2x2 + y2 = 2 (b) x2 + 4y2 = 4
9 1 1 3
(c) x2 + 4y2 = 1 (d) 4x2 + y2 = 4 (a) (b) (c) (d)
3 9 3 9
y
o
u
Trigonometry B-181

rs
m
58. In a triangle, the angles are in the ratio 2 : 5 : 3. What is the x

a
value of the least angle in the radian ? 64. If sin 17° = , then sec 17° – sin 73° is equal to

h
y

b
o
(a) (b)

o
20 10 y2

b
y

.w
2 (a) (b) x y2 x2
(c) (d) y2 x2

o
5 5

rd
59. If x = a cos – b sin , y = b cos + a sin then find the

p
x x2

re
value of x2 + y2.

s
(a) a2 (b) b2 (c) y y2 x2 (d) y y2 x2

s
.c
a2

o
(d) a2 + b2

m
(c) 2 65. If is a positive acute angle and cosec + cot = 3 , then
b
the value of cosec is
60. If tan + cot = 2, then the value of tan 7 + cot7 is
(a) 2 (b) 16 1
(c) 64 (d) 128 (a) (b) 3
3
61. From 125 metre high towers, the angle of depression of a
car is 45°. Then how far the car is from the tower ? 2
(a) 125 metre (b) 60 metre (c) (d) 1
3
(c) 75 metre (d) 95 metre
66. If cos + sec = 3 , then the value of cos3 + sec3 is
cos3 sin3 cos3 sin 3 (a) 2 (b) 1
62. The value of + is equal to
cos sin cos sin (c) 0 (d) 4
(a) –1 (b) 1 67. If sin + cos = 2 cos , then the value of cot is
(c) 2 (d) 0
63. The shadow of a tower standing on a level plane is found to (a) 2 1 (b) 2 1
be 30 m longer when the Sun's altitude changes from 60° to (c) 3 1 (d) 3 1
45°. The height of the tower is 68. The value of sin21° + sin22° + sin23° + .... + sin289° is
(a) 15 3 3 m (b) 15 3 1 m (a) 22 (b) 44
1
(c) 15 3 1m (d) 15 3 3 m (c) 22 (d) 44
2 2

ANSWER KEY
1 (a) 11 (b) 21 (b) 31 (b) 41 (a) 51 (a) 61 (a)
2 (b) 12 (a) 22 (a) 32 (c) 42 (b) 52 (b) 62 (c)
3 (b) 13 (a) 23 (b) 33 (a) 43 (c) 53 (a) 63 (a)
4 (a) 14 (a) 24 (c) 34 (b) 44 (d) 54 (b) 64 (d)
5 (c) 15 (a) 25 (b) 35 (c) 45 (b) 55 (b) 65 (c)
6 (a) 16 (c) 26 (d) 36 (b) 46 (a) 56 (c) 66 (c)
7 (a) 17 (c) 27 (b) 37 (c) 47 (b) 57 (d) 67 (a)
8 (c) 18 (a) 28 (c) 38 (d) 48 (b) 58 (d) 68 (d)
9 (b) 19 (c) 29 (d) 39 (b) 49 (a) 59 (d)
10 (d) 20 (d) 30 (d) 40 (a) 50 (d) 60 (a)
y
o
u
B-182 Trigonometry

rs
m
a
h
b
o
o
b
.w
1. (a) tan = 1

o
2

rd
2 1 2 3
= (2 3)
1 tan 2

p
sec 2 3 2 3

re
= 1 1

s
2

s
2 3
3) 2

.c
= 2 = (2
4 3

o
m
1
cos = (2 3)2 (2 3) 2
2
2
sin 1 cos2 =2 2 ( 3)2 [ (a + b)2 + (a – b)2 = 2 (a2 + b2)]
2 = 2 ( 4 + 3) = 2 × 7 = 14
1 1 5. (c) To find total number of terms
= 1
2 2 First term = 1, last term = 89, common diff = 2.
8sin 5 sin an = a1 + (n – 1)d
Now, 3 3 89 = 1 + (n – 1)2
sin 2 cos 7 cos
88 = (n – 1)2
1 1 n – 1 = 44
8 5 45 terms.
2 2
= 3 3 Now, sin 2 1° + sin2 3° + sin2 5°+.......... + sin2 85°
1 1 1 + sin2 87° + sin 2 89°
2 7
2 2 2 = (sin 1° + sin 89°) + (sin 3° + sin2 87°) + ..... 22 terms
2 2 2
+ sin2 45°
8 5 (8 5) 2 2 2 2
= (sin 1° + cos 1°) + (sin 3° + cos 3°) + ..... 22 terms
2 2 13 2
= = = 2 1
2
1 7 2 1 2 14 13 +
2 2 2 2 2 2 2 2
2. (b) Given, cos2 + cos4 = 1 1
= (1 + 1 + ........... 22 terms) +
or, cos4 = 1 – cos2 [ sin2 + cos2 = 1] 2
4
cos = sin .2
1 1
2
= 22 = 22
sin 1 2 2
or, 1 = . 6. (a) (sin + cos )2 + (sin – cos )2
2
cos cos 2
2 2 = (sin2 + cos2 ) + 2 sin . cos + (sin2 + cos2 )
tan . sec =1
– 2 sin . cos .
or, tan 2 . (1 + tan 2 ) = 1 [ sec2 – tan2 =1]
=1+ 1=2
or, tan 2 tan 4 1 So, (sin + cos )2 + (sin – cos )2 = 2
2
3. (b) tan4°. tan 43°. tan 47°. tan 86° 7
or, (sin + cos )2 + 2
= tan 4° . tan 43° . tan (90° – 43°) tan (90° – 4°) 13
= tan 4° . tan 43°. cot 43° cot 4° [ tan (90 – ) = cot ]
49 289
1 1 1 or, (sin – cos )2 = 2 –
= tan 4° × tan 43° × cot 169 169
tan 43 tan 4 tan
=1 2
17 17
4. (a) tan 15° . cot 75° + tan 75° . cot 15° sin + cos = .
= tan 15 . cot (90° – 15°) + tan (90° – 15°) cot 15° 13 13
= tan 15° . tan 15° + cot 15° . cot 15°
= (tan 15)2 + (cot 15)2 7. (a) Let S = cos 2 + cos = 2 cos 2 1 cos

1 2 1 1 1
= (tan 15°)2 + = 1 2 cos cos
2 2 16 8
tan15
2
Putting the value of tan 15° = 2 3 9 1 9
= 2 cos
2 8 4 8
2 1
= (2 3) So, the minimum value S = – 9/8
2 3
y
o
u
Trigonometry B-183

rs
m
8. (c) Given, 5 tan – 4 = 0
2sin (1 sin ) 2 sin

a
4 = (1 sin ) (1 cos sin ) = 1 cos =y

h
sin

b
tan =
5

o
12. (a) From right angled s ABC and DBC,

o
b
5sin 4 cos we have C

.w
cos 5 tan 4 BC BC

o
Expression, tan 60° = and tan 30°

rd
5sin 4 cos 5 tan 4 AB DB

p
cos

re
h

s
3

s
4 x h

.c
5 4

o
5 1 h

m
=
4 and 60°
5 4 3 x 20 30°
5
D A B
h=x 3 20 m x
4 4 0
= 0
4 4 8 x 20
and h =
3
3 tan 20 tan 3 20
9. (b) 3 tan 60 tan(3 20 )
1 3 tan 2 20 x 20
x 3 3x = x + 20 x = 10 m
3
9t 2 t 6 6t 4
Squaring, 3 , tan 20 t Putting x = 10 in h = 3 x, we get h = 10 3
1 9t 4 6t 2
Hence, height of the tree = 10 3 m and the breadth of
t 6 33t 4 27t 2 3
the river = 10 m.
tan 6 20 33 tan 4 20 27 tan 2 20 3 1 sin 1 sin 1 sin 2
13. (a)
1 1 cos 1 cos 1 cos 2
10. (d) Given, tan =
7 cos 2 1 1 7
= 2 = .
2 sin tan 2 8 8
2 1 8 7
sec = 1 tan 1
7 7 3
14. (a) Given, 3 cos = 5 sin tan = .
5
8
sec 2
7 8 2 3 25 9 34
cosec = tan sec = 1 tan 1 = .
1 5 25 5
7 In expression, dividing the numerator & denominator
by cos ,
2
2 8
8 5 tan 2 sec4 2
2 2 7 =
cosec sec 4
5 tan 2 sec 2
cosec 2 sec2 2 8
2
8 4
7 3 34
5 2 2
5 5
= 4
8 1 6
8 8 1 3 34
7 7 7 6 3 5 2 2
= 5 5
8 1 8 = 8 4
8 8 1
7 7 7 1156
3 22 5 2312
1 cos sin 625 625
1 cos sin = =
11. (b) = 1 cos sin . 3 2
1156
2 1 2312
1 sin 1 sin 1 cos sin 625 625
(1 sin )2 cos 2 813 271
= =
(1 sin ) (1 cos sin ) 2937 979
(1 sin 2 2sin ) (1 sin 2 ) 15. (a) We have, x sin 3 y cos 3 sin cos ....(i)
=
(1 sin ) (1 cos sin ) and x sin y cos ...(ii)
y
o
u
B-184 Trigonometry

rs
m
Equation (i) may be written as
3 tan 3 tan 1 tan tan ...(1)

a
2 3

h
x sin . sin y cos sin cos

b
( 3 tan ) ( 3 tan )

o
y cos sin 2 y cos 3

o
sin cos

b
=3 3 tan 3 tan tan tan

.w
y cos (sin 2 cos 2 ) sin cos = 3 + 1 – tan tan + tan tan = 4

o
rd
y cos sin cos 20. (d) sec2 = 3 sec = 3

p
re
y sin ...(iii) tan2 = sec2 – 1 = 3 – 1 = 2

s
s
Putting the value of y from (iii) in (ii), we get sec 2

.c
3
cosec2

o
x sin sin . cos x cos ...(iv) = 2 2

m
tan
Squaring (iii) and (iv), and adding , we get
3
x 2 y 2 cos 2 sin 2 1 tan 2 cosec2
2
Now, 2
16. (c) In the given equation, tan 2 cosec2 3
1 + sin2 A = 3 sin A cos A 2
2
Dividing both sides by cos2A,
1
1 sin 2 A sin A 1
We get 3. 2
cos 2 A cos 2 A cos A =
7 7
2
sec 2 A tan 2 A 3 tan A
21. (b) In ABD, D
1 tan 2 A tan 2 A 3 tan A
BD
tan 60
2 tan 2 A 3 tan A 1 0 AB 20
2 tan2 A – 2 tan A – tan A + 1= 0
h
2 tan A (tan A – 1) – 1(tan A – 1) = 0 3 h
AB C
(2tan A – 1) (tan A – 1) = 0
h 20
1 AB
tan A ,1 3
2 60°
17. (c) cos 20° = cos(90° – 70°) = sin 70° A B
h
cos 70° = sin 20° AB 3
3
cos3 20 cos3 70 sin3 70 sin 3 20 Now, in ABC
= 1 AC2 = AB2 + BC2
sin 3 70 sin3 20 sin3 70 sin 3 20
2
h
2 2 2 20 2 (h 20) 2
x 2 .( 2) 2 1 3
18. (a) ( 3)
2 2 3
1 3 h2 + 3h2 – 120h = 0
8 4h2 – 120h = 0
2 2
h (h –30) = 0
8x 9 1 h = 0 or 30
x 4 2 1
or, 3 h = 0 not possible
1 3 3 3 3
8 h = 30 ft
2 4 22. (a) sin = cos ( 2 – 45°)
8 8 or, cos (90° – ) = cos (2 – 45°)
or, x 90° – = 2 – 45°
3 3
= 45°
x 1 tan = tan 45° = 1
23. (b) Given, sin 5 = cos 4 = sin (90° – 4 )
5 = 90° – 4
19. (c) Given that + =
6 = 10°
2 sin 3 – 3 tan 3
tan ( + ) = tan
6
= 2 sin 30° – 3 tan 30°
tan tan 1
1 1
1 tan tan 3 =2× – 3 = 1 – 1 = 0.
2 3
y
o
u
Trigonometry B-185

rs
2 – 6 sin2x cos2x + t – 2t sin2x cos2x = – 1

m
24. (c) Let h be the height of pole, upper portion CD subtend

a
angle at A. = t (1 – 2 sin 2x cos2x) = – 3 (1 – 2 sin 2x cos2x)

h
t = – 3.

b
1

o
Then, tan 27. (b) (a cos – b sin )2 + (a cos + b sin )2

o
2

b
= a2 cos2 + b2 sin2 – 2ab sin cos + a2

.w
Let lower part BC subtend angle at A then cos2 + b2 sin2 + 2ab cos . sin .

o
In ABC, = a (cos2 + sin2 ) + b2 (sin2 + cos2 )
2

rd
= a2 × 1 + b2 × 1

p
D

re
= a2 + b2.

s
(a cos – b sin )2 + (a cos + b sin )2 = a2 + b2 .

s
2h/3

.c
c2 + (a cos + b sin )2 = a2 + b2

o
m
C a cos + b sin = ± a 2 b2 c 2
h/3 1 1
28. (c) tan
sec 1 sec 1
A B
40
sec 1 sec 1 2 sec
BC h /3 h = tan = tan
tan = (sec 1)(sec 1) sec 2 1
AB 40 120
2sec 2 sec 2
In ABD, = tan =
tan 2 tan cos
sin
BD
tan( ) cos
AB
2
tan tan h = = 2 cosec .
sin
1 tan tan 40 29. (d) (a cos + b sin )2 + (a sin – b cos )2 = m2 + n2.
a2 cos2 + b2 sin 2 + 2ab cos . sin + a2 sin 2 + b2
1 h cos2 + 2 ab sin . cos = m2 + n2.
h 2
or a (cos 2 + sin 2 ) + b 2 (sin 2 + cos 2 )
2 120
h 40 2
=m +n . 2
1
240
or a 2 b2 m2 n2
2(60 h) h
(240 h) 40
30. (d) In ACD, we get AC = h cot 60° = h. 1 / 3 , In BCD,
80 (60 + h) = 240 h – h2 4800 + 80 h = 240 h – h2 D
2 BC = h cot 30° = h 3 .
h – 160 h + 4800 = 0 (h – 120) (h – 40) = 0
h = 120 Therefore, from right-
h
[h = 40 is discarded, since h > 100 is given] angled triangle BAC,
25. (b) Given, sec + tan = x .....(i) we have
C
sec2 – tan2 = 1 BC2 = AB2 + AC2
(sec – tan ) (sec + tan ) = 1 2
30°
2 h 60°
1 1 h 3 (3) 2
or sec – tan = .....(ii) 3 B A
sec tan x 3 km
Adding (i) & (ii), we get h2 8 2
3h 9 h 9
2 3 3
1 x 1
2 sec = x +
x x 27
2 h2 =
x 1 8
sec
2x
3 3 3 6
26. (d) Given identity h= km = km
2(sin6x + cos6x) + t (sin4x + cos4x ) = – 1 2 2 4
2[(sin2x + cos2x)3 – 3 sin2x cos2x (sin2x + cos2x) 31. (b) sin + cos = 2
+ t[(sin2x + cos2x)2 – 2sin 2x cos2x] = – 1 sin < 1 : cos < 2
[ (a + b)3 = a3 + b3 + 3ab (a + b) = 90° ; = 0°
and (a + b)2 = a2 + b2 + 2ab
2 180
where a = sin 2x, b = cos2x] sin = sin
2[1– 3sin 2x cos2x] + t[1 – 2 sin 2x cos2x] = – 1 3 3
y
o
u
B-186 Trigonometry

rs
m
3 1 1 3

a
= sin 60° sin2 +cos4

h
2 2 4 4

b
o
When = 30°

o
3

b
cos cos 30 1 9 13

.w
3 2 sin2 + cos4
4 16 16

o
2

rd
32. (c) cos4 – sin4 = 1

p
3 37. (c) tan 20 = cot 4

re
tan 4
2

s
s
(cos2 + sin 4 ) (cos2 –sin2 ) = tan 2 = tan (90° – 4 )

.c
3 2 = 90° – 4

o
m
2 6 = 90° = 15°
cos2 – sin2 = tan 3 = tan 45° = 1
3
2 38. (d) tan ( 1 + 2) 3 = tan 60°
cos2 – (1– cos2 )
3 1 + 2 = 60° and sec ( 1 – 2)

2 2
cos2 – 1 = sec 30°
3 3
sin 1 – 2 = 30°
1
33. (a) cos 30 1 = 45° and 2 = 15°
sin 2 1 + tan 3 2
sin = sin 90° + tan 45°
1 =1+1=2
sin 90 – 30

sin 4 x2 1
1 39. (b) sec
sin 60 – 4x
sin = sin (60° – ) tan sec 2 1
= 60° –
2 = 60° = 30° 2
sin + cos 2 4x2 1
1
= sin 30° + cos 60° 4x
1 1
1 2 2
2 2 4x2 1 4x
34. (b) 2 sin2 + 3cos2 2
= 2 sin2 + 2cos2 + cos2 4x
= 2 (sin2 + cos2 ) + cos2
= 2 + cos2 2x 1 2 x 1 4x2 1
Minimum value of cos = – 1 4x 4x
Required minimum value = 2 + 1 = 3
1 4x2 1 4x2 1
35. (c) sin 2 39 tan 2 51 sec + tan
cosec 2 51 4x 4x

1 4 x2 1 4 x 2 1
–1 2 2
sin 51 .sec 39 4x
= sin 2 51° + sin2 39° + tan2(90° – 39°)
8 x2
1 2x
4x
sin 2 (90 – 39 ).sec 2 39 40. (a) x = a sec . cos y = b sec . sin ; z = c tan
1
= cos2 39° + sin 2 39° + cot2 39° x2 y2 z2
2 2
cos 39 .sec 39
a2 b2 c2
[ sin (90° – ) = cos , tan (90° – ) = cot ]
= 1 + cot2 39° – 1 a 2 sec2 cos 2 b sec 2 sin 2 c 2 tan 2
= cosec2 39° – 1 = x2 – 1 –
a2 b2 c2
36. (b) When = 0°
sin2 + cos4 = 1 = sec2 . cos2 + sec2 . sin2 – tan2
When = 45°, = sec2 (cos2 + sin2 ) – tan2
= sec2 – tan2 = 1
y
o
u
Trigonometry B-187

rs
m
sec tan 5 y

a
41. (a) 2 x sec 3

h
sec tan 3 sin

b
o
5 sec – 5 tan = 3 sec + 3 tan
2x yx

o
2 sec = 8 tan 3

b
cos 2 y cos

.w
tan 2 1

o
3 x

rd
sec 8 4 3cos x cos
2 2

p
re
sin 1 Now sin2 + cos2 = 1
cos

s
cos 4

s
.c
x2
y2 1

o
1

m
sin 4
4
42. (b) sec2 + tan2 = 7 4 y 2 x2 4
1 + tan2 + tan2 = 7 48. (b) sec2 – tan2 = 1
( 1 + tan2 = sec2 ) (sec + tan ) (sec – tan ) = 1
6 1
tan2 = =3 3(sec tan ) 1 sec tan ...(1)
2 3
tan 3 sec tan 3 (Given) ...(2)
tan = 3 or tan = – 3 Adding eqns. (1) and (2)
As 0 < < /2 1 4 2
2sec 3 2sec sec
= tan–1 3 3 3
3
3 1
cos sec
3 2 cos
43. (c) sin 2x + 2 tan2 – 2sec2x + cos2x
sin2x + cos2 x – 2 (sec2 x – tan2 x) Therefore, sin 1 cos 2
1 – 2 (1) = –1 3 1
1
44. (d) 4 2
'
51
100m 50m 49. (a) 63 14' [1 minute = 60 seconds]
60
' '
17 297 297
63 14 63 63º
20 20 20 60
50m 1
sin = = [1 degree = 60 minutes]
100m 2
= 30° c
75897 75897 2811
30° radian
20 1200 1200 180 8000
45. (b) tan 30°
x
20m cos 4 sin 4
1 20 50. (d) 1
cos 2 sin 2
3 x 30° cos4 sin2 + sin4 cos2 = cos2 sin2
x cos4 (1 – cos2 ) + cos2 (1 – cos2 )2 = cos2
x 20 3m
(1 – cos2 )
a2 b2 a2 b2 cos4 – cos4 cos2 + cos2 – 2 cos2
46. (a) –
x2 y2
a 2 sin 2 b2 tan 2 cos2 + cos4 cos2 = cos2 – cos4
cos 4 – 2 cos2 cos2 + cos4 = 0
cosec2 – cot2 = 1 (cos 2 – cos2 )2 = 0
47. (b) 2y cos = x sin cos 2 = cos2
2y sin = sin2
2
sin cos
x cos 4 sin 4
And 2x sec – y cosec = 3 Then,
cos 2 sin 2
y
o
u
B-188 Trigonometry

rs
m
cos2 cos 2 sin 2 sin 2 (sin cos )2 1 sin 2 cos2 2sin cos 1

a
h
2 2 sin cos sin cos
cos sin

b
o
cos2 + sin2 =1

o
2sin cos

b
2
sin cos 1

.w
51. (a) sin cos

o
sin cos 1

rd
Dividing Numerator and Denominator by cos sin 53º cot 65º
55. (b)

p
cos37º tan 25º

re
sin cos 1

s
tan 1 sec sin 53º tan 25º sin 53º tan 25º

s
cos cos cos

.c
sin cos 1 tan 1 sec cos 37º cot 65º cos(90º –53º ) cot(90º –25º )

o
m
cos cos cos
sin 53º tan 25º
2 2 1
(tan sec ) – (sec tan ) sin 53º tan 25º
tan sec 1 [ cos (90º – ) = sin and cot (90º – ) = tan ]
(tan sec )[1 sec tan ] 1 3
tan sec
tan sec 1 cos 60º sin 60º 2 2 1 3 1 3
56. (c) cos 60º – sin 60º
sin 1 1 sin 1 3 1– 3 1 3

cos cos cos 2 2
52. (b) In ABC
A (1 3)2 1 3 2 3 4 2 3
h 2 2
tan ...(1) 1 – ( 3) 1– 3 –2
9
In ABD h –2(2 3)
(2 3)
h 2
tan
16 cot 5º.cot10º.cot15º.cot 60º.cot 75º.cot 80º .cot 85º
+ = 90° (given) B 9 C 57. (d)
16 (cos2 20 cos2 70º ) 2
= 90 –
cot(90º –85º ).cot(90º –80º ).cot(90º –75º ).cot 60º .cot 75º .cot 80º cot 85º
h
since tan (cos 2 (90º –70º ) cos 2 70º ) 2
16
1
h h 16
tan(90 ) cot or tan ...(2) cot 60º 3 1 3 3
16 16 h
(1 2) 3 3 3 3 9
From eqn. (1) and (2)
h 16 58. (d) Let angles are 2x, 5x and 3x.
h2 16 9 h 12 feet. 2x + 5x + 3x = 180º
9 h (sum of interior angle of triangles is 180º)
53. (a) If sin 2 cos 3 10x =18º
x = 18º
tan 2 cos ...(1) Least angle in degree = 2x = 2 × 18 = 36º
Now consider, cot6 – cot2
In radian = 36º
1 1 1 180º 5
Since cot
tan 6 tan 2 tan 59. (d) x = a cos – b sin
Substituting for tan 2 with cos from (1) above y = b cos + a sin
equation will be x 2 + y2 = (a cos – b sin )2 + (b cos + a sin )2
a2 cos2 + b2 sin2 – 2 ab cos sin + b2 cos2
1 1 1 cos 2 sin 2 tan 2 + a2 sin2 + 2ab cos sin
1
cos 3
cos cos 3
cos3 cos (a2 + b2) cos2 + (a2 + b2) sin2
54. (b) (1 + tan + sec ) (1 + cot – cosec ) a2 + b2 (cos2 + sin2 )
a2 + b2. (1) a2 + b2
sin 1 cos 1 60. (a) tan + cot = 2
1 1
cos cos sin sin
1
tan + 2 tan2 + 1 = 2 tan
sin cos 1 sin cos 1 tan
cos sin tan2 – 2 tan + 1 = 0
tan2 – tan – tan + 1 = 0
tan (tan – 1) – 1 (tan – 1) = 0
(tan – 1) (tan – 1) = 0
y
o
u
Trigonometry B-189

rs
m
tan =1 sec 17° – sin 73°

a
= sec 17° – cos 17°

h
1
Now, tan7 + cot7 (tan )7 1 1 2

b
(tan ) 7

o
y y2 x2

o
b
A = y

.w
61. (a) Tower y2 x2
= 45º

o
rd
y2 y2 x2 x

p
re
= 2 2 =
y y x y y2 x2

s
s
125 m

.c
65. (c) cosec + cot =

o
3

m
1 cos
= 45º C + = 3
sin sin
B Car
In ABC 1 cos
= 3
AB 125 125 sin
tan tan 45º 1
BC BC BC
BC = 125 m 2 cos2
Hence, car is 125 m from the tower. 2
= 3
2sin cos
(cos sin )(cos 2 sin 2 sin cos ) 2 2
62. (c)
(cos sin )

sin )(cos 2
(cos sin 2 sin cos ) cot = 3
2
+
(cos sin )
1
= 2 cos2 + 2sin2 – sin cos + sin cos tan = ; = 30°; = 60°
=2 2 3 2
A
2
cosec = cosec 60° =
3
h 66. (c) cos + sec = 3
63. (a)
taking cube both sides
45° 60°
D x B cos3 + sec3 + 3 cos sec (cos + sec ) = 3 3
30 m C
h cos3 + sec3 +3 3 = 3 3
In ABC, tan60° =
x cos3 + sec3 =0
h 67. (a) sin + cos = 2 cos
x= ...(1)
3
sin = ( 2 1) cos
h
In ABD, tan 45° =
30 x 1
cot =
h 2 1
1= or h = 30 + x
30 x
Putting value of x from (1) 1 2 1
cot = 2 1
h 2 1 2 1
h = 30
3 68. (d) (sin2 1° + sin2 89°) + (sin2 2° + sin 2 88°) + ...
+ (sin 2 44° + sin 2 48°) + sin 2 45°
( 3 1)
or h = 30 h = 15 (3 3) m = (sin 1° + cos 1°) + (sin 2 2° + cos2 2°) + ...
2 2
3 + (sin2 44° + cos2 44°) + sin 2 45°
x 1
64. (d) sin17° = = 1 + 1 + .... + 1 (44 times) +
y 2
x2 y2 x2 1
cos 17° = 1 2 = = 44
y y 2
y
o
u
B-190 Geometry

rs
m
CHAPTER

a
h
b
14

o
GEOMETRY

o
b
.w
o
rd
p
re
s
s
.c
INTRODUCTION Angles : An angle is the union of two non-collinear rays with a

o
m
Line : A line has length. It has neither width nor thickness. It can common origin. The common origin is called the vertex and the
be extended indefinitely in both directions. two rays are the sides of the angle.
A
Ray : A line with one end point is called a ray. The end point is
called the origin.
Origin ABC with vertex B
Line segment : A line with two end points is called a segment.

B C
Parallel lines : Two lines, which lie in a plane and do not intersect,
are called parallel lines. The distance between two parallel lines is Congruent angles : Two angles are said to be congruent, denoted
constant. by , if their measures are equal.
P Q Bisector of an angle : A ray is said to be the bisector of an angle
A B if it divides the interior of the angle into two angles of equal
measure.
We denote it by PQ || AB.
Perpendicular lines : Two lines, which lie in a plane and intersect TYPES OFANGLE
each other at right angles are called perpendicular lines. 1. A right angle is an angle of 90° as shown in [fig. (a)].
2. An angle less than 90° is called an acute angle [fig. (b)].
3. An angle greater than 90° but less than 180° is called an
obtuse angle [fig (c)].
4. An angle of 180° is a straight line [fig. (d)].
5. An angle greater than 180° but less than 360° is called a
reflex angle [fig.(e)].
m

We denote it by m.
PROPERTIES
• Three or more points are said to be collinear if they lie on
a line, otherwise they are said to be non-collinear. Fig. (a) Fig. (b) Fig. (c)
• Two or more lines are said to be coplanar if they lie in the
same plane, otherwise they are said to be non-coplanar.
Fig. (d) Fig. (e)
• A line, which intersects two or more given coplanar lines
in distinct points, is called a transversal of the given lines.
• A line which is perpendicular to a line segment, i.e., PAIRS OF ANGLES
Adjacent angles : Two angles are called adjacent angles if they
intersect at 90° and passes through the mid point of the
have a common side and their interiors are disjoint.
segment is called the perpendicular bisector of the segment.
Q R
• Every point on the perpendicular bisector of a segment is
equidistant from the two endpoints of the segment.
• If two lines are perpendicular to the same line, they are
parallel to each other.
• Lines which are parallel to the same line are parallel to
P S
each other.
QPR is adjacent to RPS
y
o
u
Geometry B-191

rs
Linear Pair : Two angles are said to form a linear pair if they have

m
Alternate angles : In the above figure, 3 and 5, 2 and 8 are

a
a common side and their other two sides are opposite rays. The Alternative angles.

h
sum of the measures of the angles is 180°.

b
When two lines are itnersected by a transversal, they form two

o
o
N
pairs of alternate angles.

b
.w
The pairs of alternate angles thus formed are congruent. i.e.

o
rd
3= 5 and 2 = 8

p
re
A B Interior angles : In the above figure, 2 and 5, 3 and 8 are
M

s
Interior angles.

s
AMN + BMN = 180°

.c
When two lines are intersected by a transversal, they form two

o
Complementary angles : Two angles whose sum is 90°, are

m
complementary, each one is the complement of the other. pairs of interior angles.
A The pairs of interior angles thus formed are supplementary. i.e.
P
2 + 5 = 3 + 8 = 180°

60°
Example 1 :
30°
B C In figure given below, lines PQ and RS intersect each other
Q R
ABC + PQR = 90° at point O. If POR : ROQ = 5 : 7, find all the angles.
Supplementary angles : Two angles whose sum is 180° are
supplementary, each one is the supplement of the other. Solution :
L X POR + ROQ = 180° (Linear pair of angles)
But POR : ROQ = 5 : 7 (Given)

P S
60° 120°
M N Z
Y
LMN + XYZ = 60° + 120° = 180°
Vertically Opposite angles : Two angles are called vertically O
opposite angles if their sides form two pairs of opposite rays.
Vertically opposite angles are congruent. R Q
A D

O 5
POR 180 75
12

7
C B Similarly, ROQ 180 105
12
AOD = COB and AOC = BOD
Corresponding angles : Here, PQ || LM and n is transversal. Now, POS = ROQ = 105° (Vertically opposite angles)
Then, 1 and 5, 2 and 6, 3 and 7 and 4 and 8 are and SOQ = POR = 75° (Vertically opposite angles)
corresponding angles. Example 2 :
When two lines are intersected by a transversal, they form four
pairs of corresponding angles. In fig. if PQ || RS, MXQ = 135° and MYR = 40°, find
The pairs of corresponding angles thus formed are congruent. XMY.
i.e. 1 = 5; 2 = 6; 4 = 8; 3 = 7. P X Q
n
135°
4 1
P Q
3 2 M

8 5
L M
7 6 40°
R Y S
y
o
u
B-192 Geometry

rs
m
Solution : Solution :

a
Here, we need to draw a line AB parallel to line PQ, through POR and QOR for a linear pair

h
b
point M as shown in figure. POR + QOR = 180° (Linear pair axiom)

o
o
P X Q or a + b = 180° ........ (i)

b
.w
135° But a – b = 80° ......... (ii) [Given]

o
Adding eqs. (i) and (ii), we get

rd
M

p
A B 260

re
2a = 260° a 130
2

s
s
.c
Substituting the value of a in (1), we get
40°

o
130° + b = 180°

m
R Y S
b = 180° – 130° = 50°
Now, AB || PQ and PQ || RS AB || RS
PROPORTIONALITY THEOREM
Now, QXM + XMB = 180° The ratio of intercepts made by three parallel lines on a transversal
( AB || PQ, interior angles on the same side of the is equal to the ratio of the corresponding intercepts made on any
transversal) other transversal by the same parallel lines.
But QXM = 135° 135° + XMB = 180° If line a || b || c, and lines l and m are two transversals, then
XMB = 45° ........(i) PR QS
Now, BMY = MYR ( AB || RS, alternate angles) RT SU
l m
BMY = 40° ........(ii)
Adding (i) and (ii), we get
XMB + BMY = 45° + 40° a
P Q
i.e. XMY = 85°
Example 3 :
An angle is twice its complement. Find the angle. b
R S
Solution :
If the complement is x, the angle = 2x
2x + x = 90°
3x = 90° x = 30° c
T U
The angle is 2 × 30° = 60°
Example 4 :
The supplement of an angle is one-fifth of itself. Determine
the angle and its supplement. Example 6 :
Solution : In the figure, if PS = 360, find PQ, QR and RS.
Let the measure of the angle be x°. Then the measure of its y R S
P x Q
supplementary angle is 180° – x°.
1
It is given that 180 x x
5
5 (180° – x) = x
900 – 5x = x 900 = 5x + x
900
900 = 6x 6x = 900 x 150 A B C D
6 60 90 120
Supplementary angle is 180° – 150° = 30° Solution :
PA, QB, RC and SD are perpendicular to AD. Hence, they
Example 5 : are parallel. So the intercepts are proportional.
In figure, POR and QOP form a linear pair. If a – b = 80°,
AB PQ 60 x
find the values of a and b.
BD QS 210 360 – x
R
2 x 720
x 80
7 360 – x 9
b
P O Q PQ = 80
So, QS = 360 – 80 = 280
y
o
u
Geometry B-193

rs
m
BC QR TRIANGLES
Again,

a
The plane figure bounded by the union of three lines, which join

h
CD RS

b
three non-collinear points, is called a triangle. A triangle is denoted

o
90 y 3 y by the symbol .

o
b
120 280 – y 4 280 – y The three non-collinear points, are called the vertices of the

.w
y = 120 triangle.

o
rd
QR = 120 and SR = 280 – 120 = 160 In ABC, A, B and C are the vertices of the triangle; AB, BC, CA

p
are the three sides, and A, B, C are the three angles.
Example 7 :

re
A

s
s
In figure if || m , n || p and 1 = 85° find 2.

.c
o
n p

m
m
1 3 B C
Sum of interior angles : The sum of the three interior angles of a
triangle is 180°.
2
A + B + C = 180°
Exterior angles and interior angles
Solution : Z
n || p and m is transversal A
1 = 3 = 85° (Corresponding angles)
Also, m || & p is transversal
2 + 3 = 180° ( Consecutive interior angles)
2 + 85° = 180°
B
2 = 180° – 85° C Y
2 = 95° X
Example 8 : (i) The measure of an exterior angle is equal to the sum of the
From the adjoining diagrams, measures of the two interior opposite angles of the triangle.
calculate x, y, z and w. x y ACY = ABC + BAC
Solution : 70° CBX = BAC + BCA and BAZ = ABC + ACB
y = 70° (ii) The sum of an interior angle and adjacent exterior angle is
x + 70 = 180° 180°.
z w
..... (vertical opp. angle) i.e. ACB + ACY = 180°
x = 180 – 70 = 110° ABC + CBX = 180° and BAC + BAZ = 180°
.... (adjacent angles on a st. line or linear pair) Example 10 :
z = 70° .....(corresponding angles) If the ratio of three angles of a triangle is 1 : 2 : 3, find the angles.
z + w = 180° ..... (adjacent angles on a st. line or Solution :
linear pair) Ratio of the three angles of a = 1 : 2 : 3
70 + w = 180° Let the angles be x, 2x and 3x.
w = 180° – 170° = 110° x + 2x + 3x = 180°
E
Example 9 : 6x = 180° x = 30°
From the adjoining diagram Hence the first angle = x = 30°
Find (i) x (ii) y D 70° C
y The second angle = 2x = 60°
Solution : The third angle = 3x = 90°
x = EDC = 70°
(corresponding angles) CLASSIFICATION OF TRIANGLES
x B Based on sides :
Now, ADB = x = 70° A
[AD = DB] CBA=90° Scalene triangle : A triangle in which none of the three sides is
In ABD, equal is called a scalene triangle.
ABD = 180 – x – x Isosceles triangle : A triangle in which at least two sides are
= 180 – 70 – 70 = 40° equal is called an isosceles triangle.
BDC = ABD = 40° (alternate angles) Equilateral triangle : A triangle in which all the three sides are
y = 40° equal is called an equilateral triangle. In an equilateral triangle, all
the angles are congruent and equal to 60°.
y
o
u
B-194 Geometry

rs
m
Based on angles : SIMILARITY OF TRIANGLES

a
Right triangle : If any one angle of a triangle is a right angle, i.e., For a given correspondence between two triangles, if the

h
b
90° then the triangle is a right-angled triangle. corresponding angles are congruent and their corresponding sides

o
are in proportion, then the two triangles are said to be similar.

o
Acute triangle : If all the three angles of a triangle are acute, i.e.,

b
less than 90°, then the triangle is an acute angled triangle. Similarlity is denoted by ~.

.w
(i) AAA Similarlity : For a given correspondence between
Obtuse triangle : If any one angle of a triangle is obtuse, i.e.,

o
two triangles, if the two angles of one triangle are congruent

rd
greater than 90°, then the triangle is an obtuse-angled triangle.
to the corresponding two angles of the other triangle, then

p
re
SOME BASIC DEFINITIONS the two triangles are similar.

s
(ii) SSS Similarity : If the corresponding sides of two triangles

s
1. Altitude (height) of a triangle : The perpendicular drawn

.c
from the vertex of a triangle to the opposite side is called an are proportional, their corresponding angles are equal and

o
m
altitude of the triangle. hence the triangles are similar.
2. Median of a triangle : The line drawn from a vertex of a (iii) SAS Similarity : If one angle of a triangle is equal to one
triangle to the opposite side such that it bisects the side, is angle of the other and the sides including these angles are
called the median of the triangle. proportional, the triangles are similar.
• A median bisects the area of the triangle. PROPERTIES OF SIMILAR TRIANGLES
3. Orthocentre : The point of intersection of the three altitudes 1. If two triangles are similar,
of a triangle is called the orthocentre. The angle made by Ratio of sides = Ratio of height = Ratio of Median = Ratio of
any side at the orthocentre = 180°– the opposite angle to angle bisectors = Ratio of inradii = Ratio of circumradii.
the side. If ABC ~ PQR
4. Centroid : The point of intersection of the three medians of AB AD BE
a triangle is called the centroid. The centroid divides each
PQ PS QT
median in the ratio 2 : 1. P
5. Circumcentre : The point of intersection of the A
perpendicular bisectors of the sides of a triangle is called
the circumcentre. T
6. Incentre : The point of intersection of the angle bisectors E
of a triangle is called the incentre.
(i) Angle bisector divides the opposite sides in the ratio R
of remaining sides B D C Q S
The ratio of the areas of two similar triangles is equal to the
BD AB c ratio of the squares of the corresponding sides.
Example :
DC AC b If ABC ~ PQR, then
(ii) Incentre divides the angle bisectors in the ratio 2 2
(b + c) : a, (c + a) : b and (a + b) : c Ar( ABC) AB BC (AC) 2
Ar( PQR) (PQ) 2 (QR)2 (PR)2
CONGRUENCY OF TRIANGLES
Two triangles are congruent if the sides and angles of one triangle PYTHAGORAS THEOREM
are equal to the corresponding sides and angles of the other In a right triangle, the square of the hypotenuse is equal to the
triangle. sum of the squares of the other two sides.
(i) SAS Congruence rule : Two triangles are congruent if two
C
sides and the included angle of one triangle are equal to the
sides and the included angle of the other triangle.
(ii) ASA Congruence rule : Two triangles are congruent if two
angles and the included side of one triangle are equal to
two angles and the included side of other triangle.
(iii) AAS Congruence rule : Two triangles are congruent if any
two pairs of angles and one pair of corresponding sides are
equal. B A
(iv) SSS Congruence rule : If three sides of one triangle are If a right triangle ABC right angled at B. Then,
equal to the three sides of another triangle, then the two By Pythagoras theorem, AC2 = AB2 + BC2
triangles are congruent.
(v) RHS Congruence rule : If in two right triangles, the BASIC PROPORTIONALITY THEOREM (BPT)
hypotenuse and one side of the triangle are equal to the If a line is drawn parallel to one side of a triangle, to intersect the
hypotenuse and one side of the other triangle, then the two other two sides in distinct points, the other two sides are divided
triangles are congruent. in the same ratio.
y
o
u
Geometry B-195

rs
If ABC in which a line parallel to BC intersects AB to D and AC

m
Solution :
at E. Then,

a
If the interior angle is x, exterior angle is 2x.

h
A

b
AD AE P

o
By BPT,

o
DB EC

b
.w
o
rd
p
re
D

s
E

s
2x

.c
x
Q S

o
B C R

m
MID-POINT THEOREM x + 2x = 180°
The line segment joining the mid-points of two sides of a triangle 3x = 180°
is parallel to the third side and half of it. x = 60°
Exterior angle = 120°
In ABC, if P and Q are the mid-points of AB and AC respectively
Hence sum of the other two angles of triangle = 120°
1 (Exterior angle is the sum of two opposite interior angles)
then PQ || BC and PQ BC
2
Example 11 :
A In figure, find F.
A
E
80° 12cm

3.8cm 3 3 cm
P Q F
6 3 cm

7.6cm
60°
B C B C
6 cm D
Solution :
INEQUALITIES INATRIANGLE In triangles ABC and DEF, we have
(i) If two sides of a triangle are unequals, the angle opposite AB 3.8 1
to the longer side is larger. Conversely, DF 7.6 2
In any triangle, the side opposite to the larger angle is longer.
BC 6 1 AC 3 3 1
A Similarly, and , i.e.,
FE 12 2 DE 6 3 2
in the two triangles, sides are proportional.
ABC ~ DEF (by SSS Similarity)
B = F (Corresponding angles are equal)
But B = 60° (Given)
B C F = 60°
Example 12 :
If AB > AC then C > B In the given figure, find BAC and XAY.
(ii) The sum of any two side of a triangle is greater than the
A
third side.
P

30° 40°
X Y
B C
Solution :
Q R AXB = XAB = 30° ( BX = BA)
ABC = 30° + 30° = 60° (Exterior angle)
PQ + PR > QR; PQ + QR > PR and QR + PR > PQ
Example 10 : CYA = YAC = 40° ( CY = CA)
The interior and its adjacent exterior angle of a triangle are ACB = 40° + 40° = 80° (Exterior angle)
in the ratio 1 : 2. What is the sum of the other two angles of BAC = 180° – (60° + 80°) = 40° (Sum of all angles of a
the triangle ? triangle is 180°.
XAY = 180 – (30 + 40) = 110°
y
o
u
B-196 Geometry

rs
m
Example 13 : Solution : A

a
In the fig., PQ || BC, AQ = 4 cm, PQ = 6 cm and BC = 9 cm. In APC and ABC

h
b
Find QC. ACP = ABC

o
A

o
A= A 6 cm

b
.w
ACP ~ ABC P
8 cm

o
AP PC AC

rd
p
AC BC AB

re
P Q
AP 8 6

s
s
B C

.c
6 10 AB 10 cm

o
B C

m
8 60
Solution : AP 6 4.8 and AB 7.5
10 8
AQ PQ AP = 4.8 cm and AB = 7.5 cm
By BPT,
QC BC
4 6
ACP CP 2 82
QC = 6 cm = 0.64
QC 9 ABC BC2 102
QUADRILATERALS
Example 14 : A figure formed by joining four points is called a quadrilateral.
Of the triangles with sides 11, 5, 9 or with sides 6, 10, 8; A quadrilateral has four sides, four angles and four vertices.
which is a right triangle ?
S
Solution :
(Longest side)2 = 112 = 121;
52 + 92 = 25 + 81 = 106
112 52 + 92 R
So, it is not a right triangle.
P
Again, (longest side)2 = (10)2 = 100;
62 + 82 = 36 + 64 = 100
102 = 62 + 82
It is a right triangle. Q

Example 15 : In quadrilateral PQRS, PQ, QR, RS and SP are the four sides; P, Q,
R and S are four vertices and P, Q, R and S are the four
In figure, DBA = 132°
angles.
and EAC = 120°. • The sum of the angles of a qudrilateral is 360°.
Show that AB > AC. E
P + Q + R + S = 360°
Solution : A 120°
As DBC is a straight line, TYPES OF QUADRILATERALS :
132° + ABC = 180° 1. Parallelogram : A quadrilateral whose opposite sides are
ABC = 180° – 132° = 48° parallel is called parallelogram.
For ABC, D
132° C
EAC is an exterior angle
120° = ABC + BCA D B C

(ext. = sum of two opp. interior s)


120° = 48° + BCA
BCA = 120° – 48° = 72°
Thus, we find that BCA > ABC A B
AB > AC (side opposite to greater angle is greater)
Properties :
Example 16 : (i) Opposite sides are parallel and equal.
From the adjoining (ii) Opposite angles are equal.
diagram, calculate (iii) Diagonals bisect each other.
(i) AB (ii) AP (iv) Sum of any two adjacent angles is 180°.
(iii) ar APC : ar ABC (v) Each diagonal divides the parallelogram into two
triangles of equal area.
y
o
u
Geometry B-197

rs
m
2. Rectangle : A parallelogram, in which each angle is a right Example 17 :

a
angle, i.e., 90° is called a rectangle. The angle of quadrilateral are in the ratio 3 : 5 : 9 : 13. Find all

h
b
A B the angles of the quadrilateral.

o
o
Solution :

b
.w
Let the angles of quadrilateral are 3x, 5x, 9x, 13x.

o
3x + 5x + 9x + 13x = 360°

rd
(Sum of the angles of quadrilateral)

p
re
30x = 360°

s
x = 12°

s
.c
D C Hence angles of quadrilateral are :

o
m
Properties : 3x = 3 × 12° = 36°
(i) Opposite sides are parallel and equal. 5x = 5 × 12° = 60°
(ii) Each angle is equal to 90°. 9x = 9 × 12° = 108°
(iii) Diagonals are equal and bisect each other. 13x = 13 × 12° = 156°
3. Rhombus : A parallelogram in which all sides are congruent
(or equal) is called a rhombus. Example 18 :
ABCD is a parallelogram. E is the mid point of the diagonal DB.
DQ = 10 cm, DB = 16 cm. Find PQ.
Solution :
EDQ = EBP (Alternate angles)
A P
B

Properties :
(i) Opposite sides are parallel. E
(ii) All sides are equal.
(iii) Opposite angles are equal.
(iv) Diagonals bisect each other at right angle.
D Q C
4. Square : A rectangle in which all sides are equal is called a
square. DEQ = BEP (opposite angles)
DEQ BEP (By ASA congruency)
PE = EQ
(EQ)2 = (DQ)2 – (DE)2
= 102 – 82 = 100 – 64 = 36
EQ = 6 cm and PQ = 12 cm.
Example 19 :
Use the information given in figure to calculate the value of x.
Properties :
(i) All sides are equal and opposite sides are parallel. D C
(ii) All angles are 90°. 80° x
(iii) The diagonals are equal and bisect each other at right
angle.
5. Trapezium : A quadrilateral is called a trapezium if two of
the opposite sides are parallel but the other two sides are 73° 105°
not parallel.
E A B
Solution :
Since, EAB is a straight line
DAE + DAB = 180°
73° + DAB = 180°
i.e., DAB = 180° – 73° = 107°
Properties : Since, the sum of the angles of quadrilateral ABCD is 360°
(i) The segment joining the mid-points of the non-parallel 107° + 105° + x + 80° = 360°
sides is called the median of the trapezium. 292° + x = 360°
1 and, x = 360° – 292° = 68°
Median sum of the parallel sides
2
y
o
u
B-198 Geometry

rs
m
Example 20 : CIRCLE

a
In the adjoining kite, diagonals The collection of all the points in a plane, which are at a fixed

h
D

b
intersect at O. If ABO = 32° distance from a fixed point in the plane, is called a circle.

o
and OCD = 40°, find

o
The fixed point is called the centre of the circle and the fixed

b
(i) ABC distance is called the radius (r).

.w
(ii) ADC 40° Chord : A chord is a segment whose endpoints lie on the circle.

o
A C
O

rd
(iii) BAD AB is a chord in the figure.

p
Solution :

re
Given, ABCD is a kite.

s
s
(i) As diagonal BD bisects ABC, 32°

.c
r

o
ABC = 2 ABO = 2 × 32° = 64°

m
(ii) DOC = 90° B d
O
[diagonals intersect at right angles]
ODC + 40° + 90° = 180° [sum of angles in OCD] A B
ODC = 180° – 40° – 90° = 50°
As diagonal BD bisects ADC,
Diameter : The chord, which passes through the centre of the
ADC = 2 ODC = 2 × 50° = 100°
circle, is called the diameter (d) of the circle. The length of the
(iii) As diagonal BD bisects ABC
diameter of a circle is twice the radius of the circle.
OBC = ABO = 32°
BOC = 90° [diagonals intersect at right angles] d 2r
OCB + 90° + 32° = 180° [sum of angles in OBC] Secant : A secant is a line, which intersects the circle in two
OCB = 180° – 90° – 32° = 58° distinct points.
BCD = OCD + OCB = 40° + 58° = 98° Tangent : Tangent is a line in the plane of a circle and having one
BAD = BCD = 98° [In kite ABCD, A = C) and only one point common with the circle. The common point is
called the point of contact.
POLYGON
A plane figure formed by three or more non-collinear points joined
by line segments is called a polygon.
Q
A polygon with 3 sides is called a triangle. O
A polygon with 4 sides is called a quadrilateral.
A polygon with 5 sides is called a pentagon.
A polygon with 6 sides is called a hexagon. P
A polygon with 7 sides is called a heptagon. M T N
A polygon with 8 sides is called an octagon. PQ is a secant
A polygon with 9 sides is called a nonagon. MN is a tangent. T is the point of contact.
A polygon with 10 sides is called a decagon. Semicircle : Half of a circle cut off by a diameter is called the
Regular polygon : A polygon in which all its sides and angles are semicircle. The measure of a semicircle is 180°.
equal, is called a regular polygon. Arc : A piece of a circle between two points is called an arc. A
Sum of all interior angles of a regular polygon of side n is given minor arc is an arc less than the semicircle and a major arc is an arc
by (2n – 4) 90°. greater than a semicircle.
P
(2n – 4)90
Hence, angle of a regular polygon
n
Sum of an interior angle and its adjacent exterior angle is 180°.
Sum of all exterior angles of a polygon taken in order is 360°.

Example 21 :
The sum of the measures of the angles of regular polygon
A B
is 2160°. How many sides does it have?
Solution : Q
Sum of all angles = 90° (2n – 4) AQB is a minor arc and APB is a major arc.
2160 = 90 (2n – 4) Circumference : The length of the complete circle is called its
2n = 24 + 4 circumference (C).
n = 14
C 2 r
Hence the polygon has 14 sides.
y
o
u
Geometry B-199

rs
m
Segment : The region between a chord and either of its arcs is D

a
called a segment.

h
b
C

o
o
b
.w
o
Major sector 30°

rd
O 70°

p
Major segment A

re
O B

s
Solution :

s
Minor

.c
ACB = 90° [Angle in a semi-circle]
sector

o
In ABC, BAC + ACB + ABC = 180° [Sum of the

m
Minor segment s of is 180°]
BAC + 90° + 70° = 180°
BAC = (180° – 160°) = 20°
Sector : The region between an arc and the two radii, joining the
Now, ABCD being a cyclic quadrilateral, we have
centre to the endpoints of the arc is called a sector.
ABC + ADC = 180°
(Opposite s of a cyclic quad. are supplementary]
70° + ADC = 180°
Equal chords of a circle subtend equal angles at the centre. ADC = (180° – 70°) = 110°
Now, in ADC, we have
The perpendicular from the centre of a circle to a chord CAD + ADC + ACD = 180°
bisects the chord. (Sum of the s of a is 180°)
Equal chords of a circle are equidistant from the centre. 30° + 110° + CD = 180°
The angle subtended by an arc at the centre is double the ACD = (180° – 140°) = 40°
angle subtended by it at any point on the remaining part Hence, BAC = 20° and ACD = 40°
of the circle. Example 23 :
With the vertices of ABC as centres, three circles are
Angles in the same segment of a circle are equal.
described, each touching the other two externally. If the
Angle in a semicircle is a right angle. sides of the triangle are 9 cm, 7 cm and 6 cm. find the radii of
The tangent at any point of a circle is perpendicular to the the circles.
radius through the point of contact. Solution :
Let AB = 9 cm, BC = 7 cm A
The length of tangents drawn from an external point to a x
circle are equal. and CA = 6 cm x
Let x, y, z be the radii of y
CYCLIC QUADRILATERAL circles with centres z
A, B, C respectively. B y z C
If all the four vertices of a quadrilateral lies on a circle then the
quadrilateral is said to be cyclic quadrilateral. Then, x + y = 9, y + z = 7
• The sum of either pair of the opposite angles of a cyclic and z + x = 6
quadrilateral is 180°. Adding, we get 2 (x + y + z) = 22
x + y + z = 11
i.e. A + C = 180°
x = [(x + y + z) – (y + z)] = (11 – 7) cm = 4 cm.
B + D = 180° Similarly, y = (11 – 6) cm = 5 cm and z = (11 – 9)cm = 2 cm.
Hence, the radii of circles with centres A, B, C are 4 cm, 5 cm,
and 2cm respectively.
D C Example 24 :
In the adjoining figure, 2 circles with centres Y and Z touch
each other externally at point A.

B
A

• Conversely, if the sum of any pair of opposite angles of X


quadrilateral is 180°, then the quadrilateral must be cyclic.
Example 22 : Y A Z
In the adjoining figure, C and D are points on a semi-circle
described on AB as diameter. If ABC = 70° and
B C
CAD = 30°, calculate BAC and ACD.
y
o
u
B-200 Geometry

rs
m
Another circle, with centre X, touches the other 2 circles PQ is a tangent to a circle with centre O at a point A, AB is

a
internally at Band C. If XY = 6cm, YZ = 9 cm and ZX = 7 cm, chord and C,D are points in the two segments of the circle

h
then find the radii of the circles. formed by the chord AB. Then,

b
o
Solution : BAQ = ACB

o
b
Let X, Y, Z be the radii of the circle, centres X, Y, Z respectively BAP = ADB

.w
YAZ, XYB, XZC are straight lines (Contact of circles)

o
XY = X – Y = 6 ..... (1) COMMON TANGENTS FOR A PAIR OF CIRCLE

rd
XZ = X – Z = 7 ..... (2) (A) Length of direct common tangent

p
re
YZ = Y + Z = 9 ..... (3)
(C1C 2 ) 2 – (R1 – R 2 ) 2

s
(1) + (2) + (3) L1

s
.c
2X = 22 X = 11, Y = 5, Z = 4

o
m
The radius of the circle, centre X, is 11 cm.
The radius of the circle, centre Y, is 5 cm. R R
The radius of the circle, centre Z, is 4 cm.
C C
SOME IMPORTANT THEOREMS
I. If two chords of a circle intersect inside or outside the circle,
then the rectangle formed by the two parts of one chord is where C1C2 = Distance between the centres
equal in area to the rectangle formed by the two parts of the (B) Length of transverse common tangent
other.
L2 (C1C 2 ) 2 – (R1 R 2 ) 2 ;
B
D
B where C1C2 = Distance between the centres, and
A R1 and R2 be the radii of the two circles.
P
P
A

C R R
D
C C C
(i) (ii)

Two chords AB and CD of a circle such that they intersect


each other at a point P lying inside (fig. (i)) or outside
Example 25 :
(fig. (ii)) the circle.
Find the angle marked as x in each of the following figures
PA.PB PC.PD where O is the centre of the circle.
II. If PAB is a secant to a circle intersecting it at A and B, and
PT is a tangent, then PA.PB = PT2. x
O O
T x

O (a) (b)

P A B
O
III. Alternate segment theorem :
If a line touches a circle and from the point of contact a x
chord is drawn, the angle which this chord makes with the
given line are equal respectively to the angles formed in the
(c)
corresponding alternate segments.
Solution :
We know that the angle subtended by an arc at the centre is
C
B double the angle subtended by it at any point on the
O
remaining part of the circle.
1
(a) x = 2 × 25° = 50° (b) x 110 55
2
D
1
(c) x 70 35
P A Q 2
y
o
u
Geometry B-201

rs
m
Example 26 : Solution :

a
In the figure, RS = 12 cm and radius of the circle is 10 cm. CAD = DBC = 55° (Angles in the same segment)

h
b
Find PB. DAB = CAD + BAC = 55° + 45° = 100°

o
But DAB + BCD = 180° (Opposite angles of a cyclic

o
b
A quadrilateral)

.w
BCD = 180° – 100 = 80°

o
R S

rd
P
Example 29 :

p
re
O In figure, ABC = 69°, ACB = 31°, find BDC.

s
s
.c
A D

o
m
B

Solution :
B 69° 31°
RP = PS = 6 cm C
OS2 = PO2 + PS2
102 = PO2 + 62 Solution :
PO2 = 100 – 36 = 64 In ABC,
PO = 8 cm ABC + ACB + BAC = 180°
PB = PO + OB = 8 + 10 = 18 cm 69° + 31° + BAC = 180°
Example 27 : BAC = 180° – 100°
In the figure, AB = 16 cm, CD = 12 cm and OM = 6 cm. BAC = 80°
Find ON. But BAC = BDC
D (Angles in the same segment of a circle are equal)
N Hence BDC = 80°
C
Example 30 :
O Find the length of the tangent from a point which is at a
distance of 5 cm from the centre of the circle of radius 3 cm.
A
Solution :
M
B Let AB be the tangent. ABO is a right triangle at B.
Solution :
1
MB AB = 8 cm (perpendicular from the centre of the
2
O
circle bisects the chord)
OB2 = OM2 + MB2 5cm 3cm
OB2 = 62 + 82 = 36 + 64 = 100
OB = 10 cm A
B
OB = OD = 10cm (Radii)
OD2 = ON2 + ND2 By pythagoras theorem,
OA2 = AB2 + BO2
102 = ON2 + 62
52 = AB2 + 32
ON2 = 100 – 36 = 64
25 = AB2 + 9
Hence ON = 8 cm AB2 = 25 – 9 = 16
Example 28 : AB = 4
In figure, ABCD is a cyclic quadrilateral in which AC and Hence, length of the tangent is 4 cm.
BD are its diagonals. If DBC = 55° and BAC = 45°,
find BCD. COORDINATE GEOMETRY
D The Cartesian Co-ordinate System : Let X'OX and YOY' be two
perpendicular straight lines meeting at fixed point O then X'OX is
A
called X axis Y'OY is called the axis of y or y axis point ‘O’ is
called the origin. X axis is known as abscissa and y - axis is
known as ordinate.
B Distance Formula: The distance between two points whose

C co-ordinates are given : (x 2 x1 ) 2 (y 2 y1 ) 2


y
o
u
B-202 Geometry

rs
m
Then, we have
Distance from origin : ( x 0) 2 ( y 0) 2

a
x1 x2 2, x 2 x3 8, x 3 x1 4

h
b
o
Y and, y1 y 2 2, y 2 y3 2, y 3 y1 6

o
b
From the above equations, we have

.w
2nd Quadrant 1st Quadrant
x1 + x2 + x3 = 7 and y1 + y2 + y3 = 5

o
rd
X' X Solving together, we have x1 = –1, x2 = 3, x3 = 5

p
O

re
and y1 = 3, y2 = –1, y3 = 3
3rd Quadrant 4th Quadrant

s
Therefore the coordinates of the vertices are (–1, 3), (3, –1)

s
.c
Y' and (5, 3).

o
m
1 3 5 3 1 3 7 5
m1x 2 m 2 x1 Hence, the centroid is , i.e. , .
Section Formula : x 3 3 3 3
m1 m 2 Alternatively:
m1 y 2 m 2 y1 The coordinates of the centroid of the triangle formed by
(Internally division) y joining the mid points of the sides of the triangle are
m1 m 2
coincident
These points divides the line segment in the ratio m1 : m2.
TRIANGLE 1 2 4 1 3 1
The centroid has coordinates ,
Suppose ABC be a triangle such that the coordinates of its 3 3
vertices are A(x1, y1), B(x2, y2) and C(x3, y3). Then,
area of the triangle 7 5
i.e. , .
3 3
1
x1 (y 2 y3 ) x 2 (y3 y1 ) x 3 (y1 y 2 )
2 Example 33 :
Centroid of triangle : The coordinates of the centroid are If distance between the point (x, 2) and (3, 4) is 2, then the
x1 x2 x 3 y1 y2 y3 value of x =
, Solution :
3 3
Example 31 : 2 = (x 3)2 (2 4) 2 2 = (x 3)2 4
Find the distance between the point P (a cos , a sin ) and Squaring both sides
Q (a cos , a sin ). 4 = (x – 3)2 + 4 x – 3 = 0 x = 3
Solution :
Example 34 :
d2 = (a cos – a cos )2 + (asin – a sin )2
Find the co-ordinates of a point which divides the line
= a2 (cos – cos )2 + a2 (sin – sin )2
segment joining each of the following points in the given
2 2 ratio :
2
= a 2sin sin a 2 2 cos sin (a) (2, 3) and (7, 8) in the ratio 2 : 3 internally
2 2 2 2
(b) (–1, 4) and (0, –3) in the ratio 1 : 4 internally.
Solution :
= 4a2 sin2 sin 2 cos 2 (a) Let A(2, 3) and B(7, 8) be the given points.
2 2 2
Let P(x, y) divide AB in the ratio 2 : 3 internally.
Using section formula, we have,
= 4a2 sin2 d = 2a sin
2 2 2 7 3 2 20
x 4
Example 32 : 2 3 5
The coordinates of mid-points of the sides of a triangle are
(1, 1), (2, 3) and (4, 1). Find the coordinates of the centroid. 2 8 3 3 25
and y 5
Solution : 2 3 5
P(4, 5) divides AB in the ratio 2 : 3 internally.
(2, 3) (x3, y3) C (b) Let A (–1, 4) and B (0, –3) be the given points.
(x1, y1) A Let P(x, y) divide AB in the ratio 1 : 4 internally
Using section formula, we have
1 0 4 ( 1) 4
x
(1, 1) (4, 1) 1 4 5
1 ( 3) 4 4 13
and y
B (x2, y2) 1 4 5
4 13
Let the coordinates of the vertices be A(x1, y1), B(x2, y2) and P , divides AB in the ratio 1 : 4 internally..
C(x3, y3). 5 5
y
o
u
Geometry B-203

rs
m
Example 35 : Solution :

a
Find the mid-point of the line-segment joining two points (3, 4) Area of quadrilateral = Area of ABC + Area of ACD

h
b
and (5, 12). D (–5,–4)

o
C(7,–6)
Solution :

o
b
Let A(3, 4) and B(5, 12) be the given points.

.w
Let C(x, y) be the mid-point of AB. Using mid-point formula,

o
rd
3 5 4 12

p
we have, x 4 and y 8

re
2 2

s
C(4, 8) are the co-ordinates of the mid-point of the line

s
.c
segment joining two points (3, 4) and (5, 12).

o
A (–3,2) B (5,4)

m
Example 36 :
1
The co-ordinates of the mid-point of a line segment are (2, 3). If So, Area of ABC= | ( 3)(4 6) 5( 6 2) 7(2 4) |
co-ordinates of one of the end points of the line segment are 2
(6, 5), find the co-ordiants of the other end point. 1 1
Solution : = | 30 40 14 | | 84 | 42 sq. units
2 2
Let other the end point be A(x, y) So, Area of ACD
It is given that C (2, 3) is the mid point
1
= | 3( 6 4) 7( 4 2) ( 5)(2 6) |
x 6 y 5 2
We can write, 2 and 3
2 2
1 1
or 4 = x + 6 or 6=y+5 = | 6 42 40 | | 76 | 38 sq. units
2 2
or x = – 2 or y= 1
So, Area of quadrilateral ABCD = 42 + 38 = 80 sq. units.
A (–2, 1) be the co-ordinates of the other end point. Example 39 :
Example 37 : In the figure, find the value of x°.
A
The area of a triangle is 5. Two of its vertices are (2, 1) and
(3, –2). The third vertex lies on y = x + 3. Find the third vertex. x°
Solution : 25° E
Let the third vertex be (x3, y3), area of triangle
1
= | [x1 (y 2 y3 ) x 2 (y 3 y1) x 3 (y1 y 2 )] |
2 35° 60°
As x1 2, y1 1, x 2 3, y2 2 , Area of =5 Solution : B C D

1 In the ABC, A + B + ACB = 180°


5 | 2( 2 y3 ) 3(y3 1) x 3 (1 2) | 25° + 35° + ACB = 180°
2
10 = | 3x3 + y3 – 7 | 3x3 + y3 – 7 = ± 10 ACB = 120°
Taking positive sign, Now, ACB + ACD = 180° (linear pair)
3x3 + y3 – 7 = 10 3x3 + y3 = 17 ......... (i) or 120° + ACD = 180°
or ACD = 60° = ECD
Taking negative sign Again in the CDE, CE is produced to A.
3x3 + y3 – 7 = – 10 3x3 + y3 = –3 ......... (ii) Hence, AED = ECD + EDC
Given that (x3, –y3) lies on y = x + 3 x = 60° + 60° = 120°
So, –x3 + y3 = 3 ......... (iii) Example 40 :
7 13 Find the equation of the circle whose diameter is the line
Solving eqs. (i) and (iii), x 3 , y3 joining the points (– 4, 3) and (12, – 1). Find the intercept
2 2
made by it on the y-axis.
3 3 Solution :
Solving eqs. (ii) and (iii), x 3 , y3 .
2 2 The equation of the required circle is
7 13 3 3 (x + 4) (x – 12) + (y – 3) (y + 1) = 0
So the third vertex are , or , On the y-axis, x = 0
2 2 2 2
– 48 + y2 – 2y – 3 = 0 y2 – 2y – 51 = 0
Example 38 :
y= 1 ± 52
Find the area of quadrilateral whose vertices, taken in order,
are A (–3, 2), B(5, 4), C (7, – 6) and D (–5, – 4). Hence the intercept on the y-axis = 2 52 4 13
y
o
u
B-204 Geometry

rs
m
Example 41 : Solution :

a
(a) The triangle PQR is isosceles

h
In figure, if || m , then find the value of x.

b
MN || QR by converse

o
Solution :

o
of Proportionally theorem

b
.w
As || m and DC is (b) Again by converse of proportionally theorem, MN || QR

o
transversal P

rd
D + 1 = 180° D 60°

p
m

re
60° + 1 = 180° 1 M N

s
s
C
1 = 120°

.c
2

o
Here, 2 = 1 = 120° 25° x°

m
A B
(vertically opposite angles) Q R
In the ABC Example 43 :
A + B + C = 180° The point A divides the join the points (– 5, 1) and (3, 5) in
25° + x° + 120° = 180° the ratio k : 1 and coordinates of points B and C are (1, 5) and
or x = 35° (7, –2) respectively. If the area of ABC be 2 units, then find
the value (s) of k.
Solution :
Example 42 :
3k 5 5k 1
M and N are points on the sides PQ and PR respectively of A , , Area of ABC = 2 units
k 1 k 1
a PQR. For each of the following cases state whether MN
is parallel to QR : 1 3k 5 5k 1 5k 1
(5 2) 1 2 7 5 =±2
(a) PM= 4, QM = 4.5, PN = 4, NR = 4.5 2 k 1 k 1 k 1
(b) PQ = 1.28, PR = 2.56, PM = 0.16, PN = 0.32 14k – 66 = ± 4 (k +1) k = 7 or 31/9

1. In triangle ABC, angle B is a right angle. If (AC) is 6 cm, and 3. ABCD is a square of area 4, which is divided into four non
D is the mid-point of side AC. The length of BD is overlapping triangles as shown in the fig. Then the sum of
the perimeters of the triangles is
A
A B

D
D C
(a) 8(2 2) (b) 8(1 2)
B C
(c) 4(1 2) (d) 4(2 2)
(a) 4 cm (b) 6cm 4. The sides of a quadrilateral are extended to make the angles
(c) 3 cm (d) 3.5 cm as shown below :
2. AB is diameter of the circle and the points C and D are on
the circumference such that CAD 30 . What is the 75°

measure of ACD ? x°

C
D

70°
A B 115°

90°
(a) 40° (b) 50° What is the value of x ?
(c) 30° (d) 90° (a) 100 (b) 90
(c) 80 (d) 75
y
o
u
Geometry B-205

rs
m
5. AB BC and BD AC. And CE bisects the angle C.
D

a
A = 30º. The, what is CED.

h
b
A B

o
o
30

b
.w
E D E

o
rd
B C

p
(a) 30° (b) 60° A

re
C

s
(c) 45° (d) 65°

s
.c
6. Instead of walking along two adjacent sides of a rectangular
(a) 1 : 4 (b) 1: 2

o
field, a boy took a short cut along the diagonal and saved a

m
(c) 1 : 3 (d) Insufficient data
distance equal to half the longer side. Then the ratio of the
14. Find the co-ordinates of the point which divides the line
shorter side to the longer side is
segment joining the points (4, –1) and (–2, 4) internally in
(a) 1/2 (b) 2/3
the ratio 3 : 5
(c) 1/4 (d) 3/4
7. In a triangle ABC, points P, Q and R are the mid-points of 6 7 4 8
(a) , (b) ,
the sides AB, BC and CA respectively. If the area of the 4 2 7 7
triangle ABC is 20 sq. units, find the area of the triangle
PQR 7 7 7 8
(c) , (d) ,
(a) 10 sq. units (b) 5.3 sq. units 4 8 12 4
(c) 5 sq. units (d) None of these
8. PQRS is a square. SR is a tangent (at point S) to the circle AD 3
15. In ABC, DE | | BC and . If AC = 5.6 cm, find AE.
with centre O and TR = OS. Then, the ratio of area of the DB 5
circle to the area of the square is
A

P S D E
O

T B C

Q R (a) 2.1 cm (b) 3.1 cm


(c) 1.2 cm (d) 2.3 cm
(a) /3 (b) 11 / 7 16. In the adjoining figure, AC + AB = 5 AD and AC – AD = 8.
(c) 3 / (d) 7 / 11 Then the area of the rectangle ABCD is
9. Two circles touch each other internally. Their radii are 2 cm D C
and 3 cm. The biggest chord of the outer circle which is
outside the inner circle is of length
(a) 2 2 cm (b) 3 2 cm A B
(a) 36 (b) 50
(c) 2 3 cm (d) 4 2 cm
(c) 60 (d) Cannot be answered
10. A triangle and a parallelogram are constructed on the same 17. In the given fig. AB | | QR, find the length of PB.
base such that their areas are equal. If the altitude of the
parallelogram is 100 m, then the altitude of the triangle is : P
(a) 100 m (b) 200 m
(c) 100 2 m (d) 10 2 m
11. The sum of the interior angles of a polygon is 1620°. The
6c

A
3cm. B
m.

number of sides of the polygon are :


(a) 9 (b) 11
(c) 15 (d) 12
12. From a circular sheet of paper with a radius of 20 cm, four
Q 9 cm. R
circles of radius 5cm each are cut out. What is the ratio of
the uncut to the cut portion? (a) 3 cm (b) 2 cm
(a) 1 : 3 (b) 4 : 1 (c) 4 cm (d) 6 cm
(c) 3 : 1 (d) 4 : 3 18. In ABC, AD is the bisector of A if AC = 4.2 cm., DC = 6
13. In the adjoining the figure, points A, B, C and D lie on the cm., BC = 10 cm., find AB.
circle. AD = 24 and BC = 12. What is the ratio of the area of (a) 2.8 cm (b) 2.7 cm
the triangle CBE to that of the triangle ADE (c) 3.4 cm (d) 2.6 cm
y
o
u
B-206 Geometry

rs
m
19. Two circles of radii 10 cm. 8 cm. intersect and length of the 28. The figure shows a rectangle ABCD with a semi-circle and

a
common chord is 12 cm. Find the distance between their a circle inscribed inside it as shown. What is the ratio of the

h
centres. area of the circle to that of the semi-circle?

b
o
(a) 13.8 cm (b) 13.29 cm

o
A B

b
(c) 13.2 cm (d) 12.19 cm

.w
20. ABCD is a cyclic quadrilateral in which BC || AD,

o
ADC = 110° and BAC = 50° find DAC

rd
D C
(a) 60° (b) 45°

p
re
(c) 90° (d) 120° (a) ( 2 1) 2 (b) 2( 2 1)2

s
21. The length of a ladder is exactly equal to the height of the

s
.c
wall it is resting against. If lower end of the ladder is kept on (c) ( 2 1) 2 / 2 (d) None of these

o
m
a stool of height 3 m and the stool is kept 9 m away from the 29. If ABCD is a square and BCE is an equilateral triangle, what
wall the upper end of the ladder coincides with the tip of the is the measure of the angle DEC?
wall. Then, the height of the wall is
(a) 12 m (b) 15 m A B
(c) 18 m (d) 11 m
22. In a triangle ABC, the internal bisector of the angle A meets
E
BC at D. If AB = 4, AC = 3 and A 60 , then the length of
AD is
D C
12 3
(a) 2 3 (b) (a) 15° (b) 30°
7
(c) 20° (d) 45°
3 3 30. AB and CD two chords of a circle such that AB = 6 cm
(c) 15 (d) 6
8 7 CD = 12 cm. And AB || CD . The distance between AB and
23. In a quadrilateral ABCD, B = 90° and AD2 = AB2 + BC2 CD is 3 cm. Find the radius of the circle.
+ CD2, then ACD is equal to : . (a) (b)
3 5 2 5
(a) 90° (b) 60°
(c) 30° (d) None of these (c) 3 4 (d) 5 3
24. How many sides a regular polygon has with its sum of 31. ABCD is a square, F is the mid-point of AB and E is a point
interior angles eight times its sum of exterior angles? on BC such that BE is one-third of BC. If area of FBE = 108
(a) 16 (b) 24 m2, then the length of AC is :
(c) 18 (d) 30
(a) 63 m (b) 36 2 m
25. The Co-ordinates of the centroid of the triangle ABC are
(6, 1). If two vertices A and B are (3, 2) and (11, 4) find the (c) 63 2 m (d) 72 2 m
third vertex
32. On a semicircle with diameter AD, chord BC is parallel to the
(a) (4, –3) (b) (2, 1)
diameter. Further, each of the chords AB and CD has length
(c) (2, 4) (d) (3, 3)
2, while AD has length 8. What is the length of BC?
26. In given fig, if BAC = 60° and BCA = 20° find ADC
A
B C
60°
B

A D
(a) 7.5 (b) 7
20° D (c) 7.75 (d) None of the above
33. The line x + y = 4 divides the line joining the points (–1, 1)
C and (5, 7) in the ratio
(a) 60° (b) 45° (a) 2 : 1 (b) 1 : 2
(c) 1 : 2 externally (d) None of these
(c) 80° (d) 90°
27. In a triangle ABC, the lengths of the sides AB, AC and BC 34. If the three vertices of a rectangle taken in order are the
are 3, 5 and 6 cm, respectively. If a point D on BC is drawn points (2, –2), (8, 4) and (5, 7). The coordinates of the fourth
such that the line AD bisects the angle A internally, then vertex is
(a) (1, 1) (b) (1, –1)
what is the length of BD ?
(a) 2 cm (b) 2.25 cm (c) (–1, 1) (d) None of these
(c) 2.5 cm (d) 3 cm 35. The centroid of a triangle, whose vertices are (2, 1), (5, 2)
y
o
u
Geometry B-207

rs
m
and (3, 4) is such that BA ED and EF BC, then find value of

a
ABC + DEF.

h
8 7 10 7

b
(a) , (b) , A

o
3 3 3 3 D

o
P

b
.w
10 7 10 7
(c) , (d) , E

o
3 3 3 3

rd
36. If O be the origin and if the coordinates of any two points

p
B C

re
Q1 and Q2 be (x1, y1) and (x2, y2) respectively, then

s
OQ1.OQ2 cos Q1OQ2 =

s
.c
(a) x1x2 – y1y2 (b) x1y1 – x2y2 F

o
(c) x1x2 + y1y2 (d) x1y1 + x2y2 (a) 120º (b) 180º

m
37. In the given figure, AB || CD, BAE = 45º, DCE = 50º and (c) 150º (d) 210º
CED = x, then find the value of x. 43. In the figure AG = 9, AB = 12, AH = 6, Find HC.
A
B D

H
xº Q
45º 50º 108°
B 72° C
A C (a) 18 (b) 12
(c) 16 (d) 6
(a) 85º (b) 95º
44. In the figure given below, AB is a diametre of the semicircle
(c) 60º (d) 20º APQB, centre O, POQ = 48º cuts BP at X, calculate AXP.
38. If the coordinates of the points A, B, C be (4, 4), (3, – 2) and
P Q
(3, – 16) respectively, then the area of the triangle ABC is:
X
(a) 27 (b) 15
(c) 18 (d) 7
48º
39. Arc ADC is a semicircle and DB AC. If AB = 9 and
BC = 4, find DB. A O B
(a) 6 (b) 8 (a) 50º (b) 55º
(c) 10 (d) 12 (c) 66º (d) 40º
40. In the given figure given below, E is the mid-point of AB 45. OA is perpendicular to the chord PQ of a circle with centre
and F is the midpoint of AD. if the area of FAEC is 13, what O. If QR is a diametre, AQ = 4 cm, OQ = 5 cm, then PR is
is the area of ABCD ? equal to

A E B P A Q

F
O

R
D C (a) 6 cm (b) 4 cm.
(c) 8 cm (d) 10 cm
(a) 19.5 (b) 26 46. In the cyclic quadrilateral ABCD BCD =120º , m(arc DZC)
(c) 39 (d) None of these = 7º, find DAB and m (arc CXB).
41. Given the adjoining figure. Find a, b, c C
Z X
C D B
50º
b D
c a
36º 70º B
A
A
(a) 74º, 106º, 20º (b) 90º, 20º, 24º
(c) 60º, 30º, 24º (d) 106º, 24º, 74º (a) 60º, 70º (b) 60º, 40º
42. In the given figure, ABC and DEF are two angles (c) 60º, 50º (d) 60º,60º
y
o
u
B-208 Geometry

rs
m
NT 9

a
47. In the figure , if = and if MB = 10, find MN.

h
AB 5 E

b
o

o
M

b
.w
54º

o
D

rd
zº yº Y
B x

p
re
A 950 A C B

s
40º

s
.c
850 650 (a) 20°, 27°, 86° (b) 40°, 54°, 86°

o
m
N T ° °
(c) 20 , 27 , 43 ° (d) 40°, 54°, 43°
53. The distance between two parallel chords of length 8 cm
(a) 5 (b) 4
each in a circle of diameter 10 cm is
(c) 28 (d) 18
(a) 6 cm (b) 7 cm
48. The perimeter of the triangle whose vertices are (– 1,4), (c) 8 cm (d) 5.5 cm
(– 4, –2), (3, –4), will be : 54. In a quadrilateral ABCD, the bisectors of A and B meet
(a) 38 (b) 16 at O. If C = 70° and D = 130°, then measure of AOB is
(c) 42 (d) None of the above (a) 40° (b) 60°
49. In the figure, AB = 8, BC = 7 m, ABC = 1200. Find AC. (c) 80° (d) 100°
A 55. In ABC, E and D are points on sides AB and AC
respectively such that ABC = ADE. If AE = 3 cm, AD =
2 cm and EB = 2 cm, then length of DC is
8 (a) 4 cm (b) 4.5 cm
1200
M B C (c) 5.0 cm (d) 5.5 cm
56. In a circle with centre O, AB is a chord, and AP is a tangent
(a) 11 (b) 12 to the circle. If AOB = 140°, then the measure of PAB is
(c) 13 (d) 14 (a) 35° (b) 55°
50. Give that segment AB and CD are parallel, if lines , m and n (c) 70° (d) 75°
intersect at point O. Find the ratio of to ODS 57. In ABC, A < B. The altitude to the base divides vertex
angle C into two parts C1 and C2, with C2 adjacent to BC.
m n Then
A P B (a) C1 + C2 = A + B (b) C1 – C2 = A – B
X (c) C1 – C2 = B – A (d) C1 + C2 = B – A
58. If O is the in-centre of ABC; if BOC = 120°, then the
O
measure of BAC is
2x
(a) 30° (b) 60°
y 2y (c) 150° (d) 75°
R C Q D S 59. Two parallel chords of a circle of diameter 20 cm are 12 cm
and 16 cm long. If the chords are in the same side of the
(a) 2 : 3 (b) 3 : 2 centre, then the distance between them is
(c) 3 : 4 (d) Data insufficient (a) 28 cm (b) 2 cm
51. In the given figure, AB is chord of the circle with centre O, (c) 4 cm (d) 8 cm
BT is tangent to the circle. The values of x and y are AB BD
60. In a ABC, , B = 70° and C = 50°, then
P AC DC
BAD = ?
y
(a) 60° (b) 20°
B O (c) 30° (d) 50°
61. In a ABC, AD, BE and CF are three medians. The perimeter
X
32º of ABC is always

T A
(a) equal to AD BE CF

(a) 52º, 52º (b) 58º, 52º (b) greater than AD BE CF


(c) 58º, 58º (d) 60º, 64º
52. In the given figure, m EDC = 54°. m DCA = 40°. Find (c) less than AD BE CF
x, y and z.
(d) None of these
y
o
u
Geometry B-209

rs
m
62. In a ABC, AD, BE and CF are three medians. Then the ratio 73. Two circles intersect each other at P and Q. PA and PB are

a
two diameters. Then AQB is

h
AD BE CF : AB AC BC is (a) 120° (b) 135° (c) 160° (d) 180°

b
o
3 3 74. O is the centre of the circle passing through the points A, B

o
(a) equal to (b) less than

b
4 4 and C such that BAO = 30°, BCO = 40° and AOC = x°.

.w
3 1 What is the value of x ?

o
(c) greater than (d) equal to (a) 70° (b) 140° (c) 210° (d) 280°

rd
4 2
75. A and B are centres of the two circles whose radii are 5 cm
63. Two circles with radii 25 cm and 9 cm touch each other

p
and 2 cm respectively. The direct common tangents to the

re
externally. The length of the direct common tangent is
circles meet AB extended at P. Then P divides AB.

s
(a) 34 cm (b) 30 cm (c) 36 cm (d) 32 cm

s
(a) externally in the ratio 5 : 2

.c
64. If AB = 5 cm, AC = 12 and AB AC then the radius of the

o
(b) internally in the ratio 2 : 5

m
circumcircle of ABC is (c) internally in the ratio 5 : 2
(a) 6.5 cm (b) 6 cm (c) 5 cm (d) 7 cm (d) externally in the ratio 7 : 2
65. The radius of the circumcircle of the triangle made by x- 76. A, B, P are three points on a circle having centre O. If
axis, y-axis and 4x + 3y = 12 is OAP = 25° and OBP = 35°, then the measure of AOB is
(a) 2 unit (b) 2.5 unit (c) 3 unit (d) 4 unit (a) 120° (b) 60° (c) 75° (d) 150°
66. The length of the circum-radius of a triangle having sides 77. Side BC of ABC is produced to D. If ACD = 140° and
of lengths 12 cm, 16 cm and 20 cm is ABC = 3 BAC, then find A.
(a) 15 cm (b) 10 cm (c) 18 cm (d) 16 cm (a) 55° (b) 45° (c) 40° (d) 35°
67. If D is the mid-point of the side BC of ABC and the area of 78. The length of tangent (upto the point of contact) drawn
ABD is 16 cm2, then the area of ABC is from an external point P to a circle of radius 5 cm is 12 cm.
(a) 16 cm2 (b) 24 cm2 (c) 32 cm2 (d) 48 cm2 The distance of P from the centre of the circle is
68. ABC is a triangle. The medians CD and BE intersect each (a) 11 cm (b) 12 cm (c) 13 cm (d) 14 cm
other at O. Then ODE : ABC is 79. ABCD is a cyclic quadrilateral, AB is a diameter of the circle.
(a) 1 : 3 (b) 1 : 4 (c) 1 : 6 (d) 1 : 12 If ACD = 50°, the value of BAD is
69. If P, R, T are the area of a parallelogram, a rhombus and a (a) 30° (b) 40° (c) 50° (d) 60°
triangle standing on the same base and between the same 80. Two circles of equal radii touch externally at a point P. From
parallels, which of the following is true? a point T on the tangent at P, tangents TQ and TR are
(a) R < P < T (b) P > R > T drawn to the circles with points of contact Q and R
(c) R = P = T (d) R = P = 2T respectively. The relation of TQ and TR is
70. AB is a diameter of the circumcircle of APB; N is the foot (a) TQ < TR (b) TQ > TR
of the perpendicular drawn from the point P on AB. If (c) TQ = 2TR (d) TQ = TR
81. When two circles touch externally, the number of common
AP = 8 cm and BP = 6 cm, then the length of BN is
tangents are
(a) 3.6 cm (b) 3 cm (c) 3.4 cm (d) 3.5 cm
(a) 4 (b) 3 (c) 2 (d) 1
71. Two circles with same radius r intersect each other and one 82. D and E are the mid-points of AB and AC of ABC. If
passes through the centre of the other. Then the length of A = 80°, C = 35°, then EDB is equal to
the common chord is (a) 100° (b) 115° (c) 120° (d) 125°
3 83. If the inradius of a triangle with perimeter 32 cm is 6 cm, then
(a) r (b) 3r (c) r (d) 5r the area of the triangle in sq. cm is
2
72. The bisector of A of ABC cuts BC at D and the (a) 48 (b) 100 (c) 64 (d) 96
circumcircle of the triangle at E. Then 84. If two circles of radii 9 cm and 4 cm touch externally, then
(a) AB : AC = BD : DC (b) AD : AC = AE : AB the length of a common tangent is
(c) AB : AD = AC : AE (d) AB : AD = AE : AC (a) 5 cm (b) 7 cm
(c) 8 cm (d) 12 cm

ANSWER KEY
1 (c) 12 (c) 23 (a) 34 (c) 45 (a) 56 (c) 67 (c) 78 (c)
2 (a) 13 (a) 24 (c) 35 (b) 46 (c) 57 (c) 68 (d) 79 (b)
3 (b) 14 (c) 25 (a) 36 (c) 47 (d) 58 (b) 69 (d) 80 (d)
4 (c) 15 (a) 26 (c) 37 (a) 48 (d) 59 (b) 70 (a) 81 (b)
5 (b) 16 (c) 27 (b) 38 (d) 49 (c) 60 (c) 71 (b) 82 (b)
6 (d) 17 (b) 28 (d) 39 (a) 50 (c). 61 (b) 72 (d) 83 (d)
7 (c) 18 (a) 29 (a) 40 (b) 51 (c) 62 (c) 73 (d) 84 (d)
8 (a) 19 (b) 30 (a) 41 (a) 52 (b) 63 (b) 74 (b)
9 (d) 20 (a) 31 (b) 42 (b) 53 (a) 64 (a) 75 (a)
10 (b) 21 (b) 32 (b) 43 (b) 54 (d) 65 (b) 76 (a)
11 (b) 22 (b) 33 (b) 44 (c) 55 (d) 66 (b) 77 (d)
y
o
u
B-210 Geometry

rs
m
a
h
b
o
o
b
.w
o
1 2
1. (c) In a right angled , the length of the median is the x

rd
2 y x2 y2

p
2

re
1

s
length of the hypotenuse . Hence BD AC 3cm.
x2

s
2

.c
y2 xy x2 y2

o
2. (a) D 180 B 180 70 110 4

m
ACD 180 D CAD
x2 4xy 4x 2
180 110 30 40
y 3
3. (b) A B 4xy 3x 2 4y 3x
x 4
a 7. (c) Consider for an equilateral triangle. Hence ABC
O
D C consists of 4 such triangles with end points on mid
a points AB, BC and CA
ABCD is square a 2 A
4 a 2
AC BD 2 2
perimeters of four triangles P R
AB BC CD DA 2(AC BD)

8 2(2 2 2 2 ) 8(1 2 )
B Q C
4. (c) Sum of all the exterior angles of a polygon taken in 1
order is 360°. ar ( ABC) = ar ( PQR)
4
75° ar ( PQR) = 5 sq. units

8. (a)
P a S r
O
r
a a T
r
115° Q a R
90°
In SOR , a 2 r2 ( 2r ) 2 4r 2
i.e. x + 90 + 115 + 75 = 360
or x = 360° – 280° = 80° or x = 80° a2 3r 2 or a 3r
5. (b) In ABC, C 180 90 30 60
22 2
r
60 Area of circle 7 22 22
DCE 30
2 Area of square ( 3r) 2 7 3 21 3
Again in DEC , CED 180 90 30 60
6. (d) D C 9. (d)
2 cm
x2 y2
y O'
O
1 cm 3 cm
A B
C B A
According to question, 2 2 cm
x
( x y) x2 y2
2
x AB = 32 12 2 2 cm
( x y) x2 y2
2
AC = 4 2 cm
y
o
u
Geometry B-211

rs
m
10. (b) Let the common base be x m. D a C

a
Now, area of the triangle = area of the parallelogram 16. (c)

h
b b

b
1

o
x Altitude of the triangle = x × 100

o
2 A a B

b
.w
Altitude of the triangle = 200 m
AC AB 5AD or AC a 5b ....(i)
11. (b) The sum of the interior angles of a polygon of n sides

o
AC AD 8 or AC b 8 ....(ii)

rd
p
is given by the expression (2n – 4) Using (i) and (ii) , a b 8 5b or a 8 4b ...(iii)

re
2 Using Pythagorous theorem,

s
s
.c
2n – 4 1620 a2 b2 (b 8) 2 b2 64 16b

o
2 180

m
or a 2 16b 64 (4b 8) 2 16b 2 64 64b
1620 2
2n – 4 18 [From (iii)]
180
or 2n = 22 16b 2 80b 0 or b 0 or 5
or n = 11 Putting b = 5 in (iii), a = 4b – 8 = 20 – 8 = 12
Area of rectangle = 12 5 60
Ratio of uncut portion ( 20 20) (100 ) 17. (b) PAB ~ PQR
12. (c)
Ratio of cut portion (4 5 5)
PB PR PB 6
300 3 AB QR 3 9
100 1 PB 2 cm
13. (a) AD 24, BC 12 18. (a) A
In BCE & ADE
since CBA CDA (Angles by same arc)
BCE DAE (Angles by same arc)
? 4.2 cm
BEC DEA (Opp. angles)
BCE & DAE are similar s
with sides in the ratio 1 : 2
Ratio of area = 1:4 ( i.e square of sides)
14. (c) Here x1 4, x 2 2, y1 1, y 2 4 B 4 cm D 6 cm C
and m1 3 and m 2 5 using angle bisector theorem
AC DC 4.2 AB
m1x 2 m 2 x1 3( 2) 5( 4) 7
x AB BD 6 4
m1 m 2 3 5 4
AB 2.8 cm
m1 y 2 m 2 y1 3(4) 5( 1) 7 19. (b) Here, OP = 10 cm; O'P = 8 cm
and y
m1 m 2 3 5 8 P

7 7 10
The required point is , 8
4 8
O L O'
15. (a) In ABC, DE | | BC
By applying basic Proportionality theorem,
AD AE Q
PQ = 12 cm
DB EC
1
AD 3 PL = 1/2 PQ 12 PL PL 6 cm
But (Given) 2
DB 5 In rt. OLP , OP 2 OL2 LP 2
AE 3 (using Pythagoras theorem)
AE 3 AE 3
or or
EC 5 EC AE 5 3 AC 8 (10 ) 2 OL2 (6) 2 OL2 64 ; OL = 8

AE 3 In O' LP, (O ' L) 2 O' P 2 LP 2 = 64 – 36 = 28


or 8AE = 3 × 5.6 AE = 3 × 5.6 /8
5.6 8
O'L2 = 28 O' L 28
AE = 2.1 cm.
y
o
u
B-212 Geometry

rs
m
O'L = 5.29 cm
3 4 2 12 3

a
OO' OL O' L = 8 + 5.29 y x 3

h
2x 7 1 7

b
O O' 13.29 cm

o
23. (a) We have, AD = AB + BC + CD2
2 2 2

o
20. (a) ABC + ADC = 180° (sum of opposites angles of

b
A D

.w
cyclic quadrilateral is 180°)

o
rd
A D

p
re
50° 110°

s
s
.c
B C

o
90°

m
B C
In ABC
AC2 = AB2 + BC2
ABC + 110° = 180° AD2 = AC2 + CD2 CD = 90°
( ABCD is a cyclic quadrilateral ) 24. (c) Let n be the number of sides of the polygon
ABC = 180 – 110 ABC = 70° ( AD || BC) Now, sum of interior angles = 8 × sum of exterior angles
ABC + BAD = 180° (Sum of the interior angles
on the same side of transversal is 180°) i.e. (2n – 4) 8 2
70° + BAD = 180° 2
BAD = 180° – 70° = 110° or (2n – 4) = 32
or n = 18
BAC + DAC = 110° 50° + DAC = 110°
25. (a) Let the third vertex be ( x, y)
DAC = 110° – 50 ° = 60° The centroid of the triangle is given (6, 1).
21. (b)
x1 x 2 x3 3 11 x
6 6 14 x 18
3 3
x=4
x 3+x
y1 y2 y3 2 4 y
and 1 1 6 y 3
3 3
y = –3
Third vertex is (4, –3)
3 3m
26. (c) In ABC, B = 180° – (60° + 20°) (By ASP)
9m
A
Using pythagoras , x2 + 81 = (3 + x)2
or x 2 81 9 x 2 6 x 6x 72 or x 12m 60°
Height of wall = 12 + 3 =15 m D
22. (b) B
A

4 30 30 3 20°

y
B C C
D B = 100°
x
Using the theorem of angle of bisector, But B + D = 180°
( ABCD is a cyclic quadrilateral;
BD AB 4 4 3 Sum of opposite is 180°)
BD x & DC x
DC AC 3 7 7 100° + D = 180° ADC = 80°
27. (b) As AD biseets CA, we have A
In ABD, by sine rule, sin 30 sin B
......(i) BD DC
4/ 7x y
AB AC
sin 60 sin B DC 5 3 5
In ABC, by sine rule; or
x 3 BD 3
3 sin 30.y DC 5
or [putting the value of sin B from (i)] or 1 1
2x 4 / 7x 3 BD 3
B D 6 C
y
o
u
Geometry B-213

rs
m
DC BD 5 3 A F B

a
or

h
BD 3

b
o
E

o
BC 8
or

b
.w
BD 3

o
BC 3 6 3 9

rd
or BD = 2.25 cm

p
8 8 4

re
28. (d) Let the radius of the semi- circle be R and that of the D C

s
s
circle be r, then from the given data, it is not possible x2

.c
1 x x
Area of FEB

o
to express r in terms of R. Thus option (d) is the correct
2 3 2 12

m
alternative.
29. (a) A B x2
Now, 108
12
x2 = 108 × 12 = 1296
E
In ADC, we have
60 AC2 = AD2 + DC2
D C = x2 + x2 = 2x2
In DEC , DCE 90 60 150 = 2 × 1296 = 2592

180 150 or AC 2592 36 2


CDE DEC 15 32. (b) B C
2
30. (a) Draw OE CD and OF AB
2 4

6cm A E O F D
A B 4
F BO = radius = 4 = AO
C D
E
O 22 42 42 2 1
AE = 2 cos A 2
2 2 4 4 2

1 1
AB || CD (Given) BC = AD – AE – FD 8 7 ( AE = FD)
2 2
Let ‘r’ be the radius of the circle
Now in rt. OED, 1 1 4 1
33. (b) Ratio
5 7 4 2
(OD) 2 (OE) 2 (ED) 2
(using Pythagoras theorem) x 8 2 5
34. (c) Let fourth vertex be (x, y), then
1 1 2 2
r2 x2 (6 ) 2 ED CD 12 6cm
2 2 y 4 2 7
and x 1, y 1
2 2
r2 x2 36 ....(i)
2 5 3 10 1 2 4 7
In rt. OFB, (OB) 2 (OF) 2 (FB) 2 35. (b) x and y
3 3 3 3
r2 ( x 3) 2 (3) 2 r2 x2 6x 9 9 36. (c) From triangle OQ1Q2, by applying cosine formula.
r2 x2 6 x 18 ....(ii) Y

From (i) and (ii), we get x 2 36 x 2 6 x 18


36 6x 18 36 18 6x
Q2 (x 2 , y2 )
18 6x 3 x
For (i), r2 = (3)2 + (6)2
r2 = 9 + 36 r2 = 45 Q1 (x1 , y1 )
r 45 r 3 5 cm.
31. (b) Let the side of the square be x, then
x and x
BE BF X
3 2 O
y
o
u
B-214 Geometry

rs
m
46. (c) m DAB + 180º – 120º = 60º
Q1Q22 OQ12 OQ22 2OQ1.OQ 2 cos Q1OQ 2

a
(Opposite angles of a cyclic quadrilateral)

h
m (arc BCD) = 2m DAB = 120º.

b
or (x1 x 2 ) 2 (y1 y 2 ) 2

o
x12 y12 x 22 y 22 2OQ1.OQ2 cos

o
b
Z C

.w
X
or x1x 2 y1y2 OQ1.OQ2 cos Q1OQ 2

o
37. (a) EDC = BAD = 45º (alternate angles) D B

rd
x = DEC = 180º – (50º + 45º) = 85º.

p
re
1

s
38. (d) [4 (2 16) 3 ( 16 4) 3 (4 2)]

s
2

.c
o
1

m
= [56 – 60 + 18] = 7 A
2
39. (a) m ADC = 90º
(Angle subtended by the diameter on a circle is 90°) m (arc CXB) = m (BCD) – m (arc DZC)
= 120º – 70º = 50º .
D
47. (d) MBA = 180º – 95º = 85º
AMB = TMN ...(Same angles with different names)
MBA – MNT ......(AA test for similarity)
A B C
MB AB
ADC is a right angled triangle. = .......(proportional sides)
(DB)2 = BA × BC .. MN NT
(DB is the perpendicular to the hypotenuse) 10 5 90
= 9 × 4 = 36 = MN = = 18.
MN 9 5
DB = 6
40. (b) As F is the mid-point of AD, CF is the median of the 48. (d) The three length AB, BC, AC will be
triangle ACD to the side AD. AB [( 1 4)2 (4 2)2 ] 45
Hence area of the triangle FCD = area of the triangle
ACF.
BC [( 4 3)2 ( 2 2) 2 ] 72 22 53
Similarly area of triangle BCE = area of triangle ACE.
Area of ABCD = Area of (CDF + CFA + ACE + BCE)
AC 4 2 82 80
= 2 Area (CFA + ACE) = 2 × 13 = 26 sq. units.
Perimeter = AB + BC + AC
41. (a) a + 36º + 70º = 180º (sum of angles of triangle)
49. (c) m ABM = 180º –120º = 60º
a = 180º – 36º – 70º = 74º
AMB is a 30º – 60º – 90º triangle.
b = 36º + 70º(Ext. angle of triangle ) = 106º
c = a – 50º (Ext. angle of triangle ) =74º – 50º = 24º. 3 3
42. (b) Since the sum of all the angle of a quadrilateral is 360º AM AB = ×8=4 3
2 2
We have ABC + BQE + DEF + EPB = 360º
ABC + DEF = 180º [ BPE = EQB = 90º ] 1 1
MB = AB = x 8 = 4
43. (b) m AHG = 180 – 108 = 720 2 2
AHG = ABC .....(same angle with different
names) (AC)2 = (AM)2 + (MC)2 = (4 3 )2 + (4 +7)2
AHG – ABC .....(AA test for similarity) = 48 + 121 = 169 ; AC = 169 = 13.
AH AG 6 9 50. (c) Let the line m cut AB and CD at point P and Q
= ; = respectively
AB AC 12 AC
DOQ = x (exterior angle)
12 x 9 Hence, Y + 2x (corresponding angle)
AC = = 18 y=x
6
HC = AC – AH = 18 – 6 = 12 ...(1)
Also . DOQ = x (vertically opposite angles)
1 In OCD, sum of the angles = 180°
44. (c) b = (48º)
2 y + 2y + 2x + x =180°
( at centre = 2 at circumference on same PQ) 24º 3x + 3y = 180°
AQB = 90º ( In semi- circle) x + y = 60
QXB = 180º – 90º – 24º ( sum of ) = 66º ...(2)
From (1) and (2) x = y = 30 = 2y = 60
45. (a) AO = OQ2 AQ2 = 52 42 = 9 = 3
ODS = 180 – 60 = 120°
Now, from similar s QAO and QOR = 180 – 3x = 180 – 3(30) = 180 – 90 = 90°.
OR = 2OA = 2× 3 = 6 cm. The required ratio = 90 : 120 = 3 : 4.
y
o
u
Geometry B-215

rs
51. (c) Given AB is a circle and BT is a tangent, BAO = 32º

m
A + B + C + D = 360

a
Here, OBT = 90º A + B = 360 – (130 + 70) = 160°

h
[ Tangent is to the radius at the point of contact]

b
A B

o
OA = OB [Radii of the same = 80° ...(1)

o
circle] 2 2

b
.w
OBA = OAB = 32º In AOB,

o
[Angles opposite to equal side are equal]

rd
A B
OBT = OBA + ABT = 90º or 32º + x = 90º . 0 = 180°

p
x = 90º – 32º = 58º . 2 2

re
Also, AOB = 180º – OAB – OBA 0 = 180° – 80° = 100°

s
s
= 180º – 32º – 32º = 116º 55. (d)

.c
P

o
1 56. (c) In AOB, A + B + O = 180°

m
Now Y = AOB A + B = 180 – 140° = 40°
2
[Angle formed at the center of a circle is double the A = B = 20° {AO = BO}
angle formed in the remaining part of the circle] PAO = 90° A B
1 PAB + BAO = 90° 60°
= × 116º = 58º . PAB = 90° – 20° = 70° O
2
1 57. (c) C
52. (b) m ACD = M(are CXD) = m DEC
2
C1 C2
m DEC = x = 40°
1
m ECB = m (are EYC) = m EDC A B
2 D
m ECB = y = 54º In ADC,
54 + x + z = 180° (Sum of all the angles of a triangle) A + D + C1 = 180°; A + C1 = 180° – 90° = 90°
54 + 40 + z = 180°
In BDC,
z = 86°. B + D + C2 = 180°; B + C2 = 180° – 90° = 90°
B A + C1 = B + C 2
53. (a) C1 – C2 = B – A
M 58. (b)
D
O 59. (b)
A 6 cm C 6 cm
B
N
8 cm 8 cm
A
C D
Two parallel chords AB & CD & AB = CD = 8 cm cm
O 10
Diameter of circle = AD = 10 cm.
10
radius = AO = OD = 5 cm
2 In ADO,

AM = MB =
AB OD = (AO)2 AD2
= 4 cm.
2
AOM is Right angle , = 100cm 2 64cm 2 = 6 cm
AO2 = AM2 + OM2 In BCO,
52 = 42 + OM2
OM2 = 25 – 16 = 9 OC = OB2 CB2
OM = 3 cm.
Similarly, = 100cm 2 36cm 2 = 8 cm
OM = ON = 3 cm distance between chords = OC – OD = 2cm
Distance between parallel chords = MN
= OM + ON 60. (c) A
= 3 + 3 = 6 cm

54. (d) A B

O
130° 70° 70° 50°
D C B D C
y
o
u
B-216 Geometry

rs
BC2 = (5)2 + (12)2 A

m
AB BD 12

a
Given, BC2 = 25 + 144

h
AC DC BC2 = 169 5

b
B C

o
According to angle bisector theorem which states that O

o
the angle bisector, like segment AO, divides the sides BC = 169 = 13 cm

b
.w
of the triangle proportionally. Therefore, A being BC 13
Radius of triangle 6.5 cm

o
the bisector of triangle.

rd
2 2
In ABC,

p
65. (b) Putting x = 0 in 4x + 3y = 12 we get y = 4

re
A + B + C = 180° Putting y = 0 in 4x + 3y = 12 we get x = 3

s
s
A = 180° – 70° = 60° The triangle so formed is right angle triangle with points

.c
(0, 0) (4, 0) (0, 3)

o
60

m
BAD = = 30° So diameter is the hypotenus of triangle 16 9
2
= 5 unit
A radius = 2.5 unit
61. (b)
abc
66. (b) Circum Radius (R)=
4 Area of triangle
[where a, b and c are sides of triangle]
F E
Area of Triangle = s(s a)(s b)(s c)
a b c
s 24
B C 2
D
Area of Triangle 24 12 8 4 = 8 × 3 × 4 cm2
Let ABC be the triangle and D, E and F are midpoints 12 16 20
of BC, CA and AB respectively. R 10 cm
4 8 3 4
Hence, in ABD, AD is median
AB + AC > 2 AD ...(1) 67. (c) Area of ABD = 16 cm2
Similarly, we get Area of ABC = 2 × Area of ABD [ In triangle, the
BC + AC > 2 CF ...(2) midpoint of the opposite side, divides it into two
BC + AB > 2 BE ...(3) congruent triangles. So their areas are equal and each
On adding the above in equations, we get is half the area of the original triangle]
(AB + AC + BC + AC + BC + AB) > 2 (AD + BE + CF) 32 cm2
2 (AB + AC + BC) > 2 (AD + BE + CF) 1
68. (d) Area of ODE OK DE A
AB + AC + BC > AD + BE + CF 2
Thus, the perimeter of triangle is greater than the sum
of the medians. 1 1
BC OK K
62. (c) 2 2 D E
P 1 O
63. (b) Q [BC (AO AK)]
9R 4
16 B F C
B A 1 2 1
BC AF AF
C 4 3 2

Let the two circles with centre A, B and radii 25 cm and 1 1 1 1


AF BC = area of ABC = 1 : 12
9 cm touch each other externally at point C. 4 3 2 12
Then AB = AC + CB = 25 + 9 = 34 cm. 69. (d) Parallelogram Area = l × b
Let PQ the direct common tangent. i.e., BQ PQ and Rhombus Area = l × b
AP PQ . Draw BR AP. Then BRQP is a rctangle. l b
Triangle Area
(Tangent radius at point of contact) 2
In ABR, Therefore R = P = 2T.
AB2 = AR2 + BR2 70. (a) Since AB is a diameter. Then APB = 90° (angle in the
(34)2 = (16)2 + (BR)2 semicircle)
BR2 = 1156 – 256 = 900 BPN ~ APB
BR = 900 = 30 cm So, BN = BP2 / AB
64. (a) In ABC, 6 6
BC2 = AB2 + AC2 BN = 3.6cm
10
y
o
u
Geometry B-217

rs
A A

m
71. (b) In AOM 76. (a) In OBP.
r
r2 = AM2 + x2

a
r OB = OP( radius)

h
AM2 = r2 – x2 ...(1) OBP = OPB = 35º

b
x 25º

o
In AMO' O M O' In AOP

o
b
r2 = (r – x)2 + AM2 OA = OP ( radius)

.w
O
AM2 = r2 – (r – x)2 ...(2) OAP = OPA = 25º
B

o
From eqns. (1) & (2) Now, APB = OPA + OPB

rd
35º
r2 – x2 = r2 – (r – x)2 = 25º + 35º = 60º B P

p
re
2rx = r2 Hence, AOB = 2 APB

s
(Angle be substended by are at centre is twice)

s
r

.c
x = 2 × 60º = 120º

o
2

m
From eq. (1) A
77. (d)
2
r 3 2
AM 2 r2 r
2 4

3
AM = r 140º
2
B C D
3
Length of chord AB = 2AM = 2 r 3r
ACB + ACD = 180º (linear pair)
2
72. (d) ACB = 180º – 140º = 40º
In ABC
P BAC + ABC + ACB = 180º
BAC + 3 BAC + 40º = 180º
73. (d)
O O 4 BAC = 180º – 40º
140
BAC = 35º
4
A Q B
78. (c) A 12 cm
P
5 cm

AQP = (Angle in the semicircle is 90°)


2
O
BQP = (Angle in the semicircle is 90°)
2
AP is a tangent and OA is a radius.
AQB = AQP + BQP = + or 180°
2 2 Therefore, OA is at AP.
74. (b) In AOB A So, In OAP
AO = BO (radii of circles) OP2 = 52 + 122
OP2 = 25 + 144 = 169
30º
ABO = BAO = 30º xº D C
O OP = 13 cm
In BOC 50º
40º
BO = CO (radii of circles) B C 79. (b) In ABC, ACB = 90º
BCO = OBC = 40º ACB + ACD A B
ABC = ABO + OBC 90º + 50º = 140º O
ABC = 30° + 40° = 70° As angle mode by triangle
2 × ABC = AOC x° = 140 in semicircle is equal to 90º.
75. (a) R In quad. ABCD BAD + BCD = 180º
Q angle of (opp. pair of quad is equal to 180º)
BAD = 180º – 140º = 40º
A B P
S T
80. (d)
Q R
T
PQB and PRA are similar triangle by AAA criteria.
P
AP AR 5
BP BQ 2
P divides AB externally in the ratio of 5 : 2
y
o
u
B-218 Geometry

rs
m
TP = TQ [The length of tangents drawn from 83. (d) Area of triangle = Inradius × Semi-perimeter

a
an external point to a circle are equal] = 6 ×16 = 96 sq. cm

h
Similarly, TP = TR

b
o
Using both equation, we get A

o
84. (d)

b
TQ = TR

.w
C B
The relation of TQ and TR is TQ = TR. 9 cm

o
4 cm

rd
O'

p
O 9 cm 4 cm

re
81. (b) D F C

s
s
.c
o
m
In figure, AC = AO – CO
A E B = 9 cm – 4 cm = 5 cm {CO = BO'}
Also, CB = OO = 13 cm
In ABC
There are three common tangents AB, CD and EF
82. (b) DE is parallel to BC A AB = CB2 AC 2
So AED = C = 35º
80º = (13cm)2 (5cm)2
Since A = 80º
Then ADE = 65º = 12 cm
EDB is supplement to ADE. D E
So, EDB = 180º – ADE
= 180º – 65º = 115º
35º
B C
y
o
u
Data Interpretation B-219

rs
m
a
CHAPTER
DATA

h
b
15

o
o
b
.w
INTERPRETATION

o
rd
p
re
s
s
.c
Data Interpretation questions are based on the information given in which the magnitudes are represented by the length or

o
m
in the tables and graphs. height of rectangle, whereas width of rectangles are
Classification of Data Interpretation : immaterial. Thus, a bar is just one dimensional as only the
length of the bar is to be considered and not the width. All
the bars drawn in a diagram are generally of uniform width
DI which depends on the number of bars to be constructed
and the availability of the space.
Tables Graphs Types of Bar Graphs are–
(i) Simple Bar Graph : It is used to represent only one
dependent variable.
Bar Graph Line Graph Pie Graph Combined (ii) Sub-divided Bar Graphs : These are used to represent
Graph
the break down of a total into its component bars. A
bar is divided into different segments, each segment
1. BG+ LG
Simple Bar Subdivided 2. BG + PG respresents a given component. Different shades,
Multiple
Graph Bar Bar 3. LG + PG colours, designs etc. are used to distinguish the
Graph Graph etc. various components. An index is given to represent
the various components. To compare, the order of
various components in the different bars is same.
TABLES (iii) Multiple Bar Graph (MBG) : When a combination
A table is one of the easiest way for summarising data. of inter-related variables are to be represented
A statistical table is the logical listing of related quantitative data graphically, multiple bar diagrams are used. These
in vertical columns and horizontal rows of numbers with sufficient are extended form of simple bar diagrams. In M.B.G.
explanatory and qualifying words, phrases and statements in the many aspects of the data are presented
form of titles, heading and notes to make clear the meaning of simultaneously with separated bars or various shades
data. of colours. An index is given to explain the shades or
colours used.
2. Line Graph(LG) : LG are used to show how a quantity
changes, very often the quantity is measured as time
changes. If the line goes up, the quantity is increasing
Sum of all items
Average = and the line goes down, the quantity is decreasing. If the
Total number of items
line is horizontal, the quantity is not changing.
= The bars are drawn proportional in length to the total
3. Pie Graph(PG) : Pie graph is a pictorial representation of
and then divided in the ratios of their components.
numerical data by non-intersecting adjacent sectors of a
% change (increase or decrease)
circle sector’s area of each sector is proportional to the
Final value _ Initial value magnitude of the data represented by the sector.
= 100
Initial value
360
1% of total value = 3.6
100
GRAPHS
The % of components parts can be converted to degrees
Graphs are a convenient way to represent information. The graphs by multiplying 3.6°.
should be labelled properly to show what part of the graphs Degree of any component part
shows what a value.
1. Bar Graph - Bar diagram consists of a number of component value
= ×360 .
equidistant rectangles. One for each category of the data total value
y
o
u
B-220 Data Interpretation

rs
m
a
h
b
o
o
b
.w
DIRECTIONS (Qs.1-5): Study the following table to answer the 6. What is the average percentage growth of sales of

o
rd
given questions: Assembled PCs for the given years?

p
Percentage of marks obtained by seven (a) 30 (b) 20

re
students in six subjects (c) 40 (d) 35

s
s
7. If the Branded PCs sold in 2011 were 100000, how many

.c
Subject Eng His Com MathsScience Econ
Branded PCs were sold in 2014?

o
(Max, Marks

m
Students (100) (100) (100) (100) (100) (100) (a) 202800 (b) 156000
Meera 100 80 50 90 90 60 (c) 234000 (d) 300000
Subodh 80 70 80 100 80 40 8. What is the difference between total Branded and total
Kunal 90 70 60 90 70 70
Assembled PCs sold for the given years?
Soni 60 60 65 80 80 80
(a) 75000 (b) 750000
(c) 175000 (d) Cannot be determined
Richu 50 90 62 80 85 95
9. In which year is the difference in the growth between
Irene 40 60 64 70 65 85
Branded and Assembled PCs lowest’?
Vgay 80 80 35 65 50 75
(a) 2010 (b) 2013
1. What is the total marks obtained by Meera in all the subject? (c) 2014 (d) None of these
(a) 448 (b) 580 10. For Assembled PCs sale, which year is the per cent growth
(c) 470 (d) 74.67 the highest compared to previous year’?
2. What is the average marks obtained by these seven students (a) 2014 (b) 2011
in History? (rounded off to two digits) (c) 2013 (d) Cannot be determined
(a) 72.86 (b) 27.32 DIRECTIONS (Qs. 11-15): Read the following table and answer
(c) 24.86 (d) 29.14 the questions.
3. How many students have got 60% or more marks in all the
subjects? Internet owners in our country (Approximate)
(a) One (b) Two Year Government Private
(c) Three (d) Four 2007-2008 3900 –
4. What is the overall percentage of Kunal ? 2008-2009 29400 –
(a) 64 (b) 65 2009-2010 90000 –
(c) 75 (d) 64.24 2010-2011 230000 12000
5. In which subject is the overall percentage the best ? 2011-2012 520000 120000
(a) Maths (b) Economics 2012-2013 1060000 450000
(c) History (d) Science 2013-2014 1550000 950000

DIRECTIONS (Qs. 6-10): Study the following graph carefully 11. In which period the percentage increase in the total internet
and answer the questions given below: owners is least to that over the earlier period?
The following graph shows the percentage growth of (a) 2009-2010 (b) 2010-2011
Branded and Assembled PCs (c) 2011-2012 (d) 2013-2014
12. What is the total number of fresh internet owners in the
period 2013-14?
60 (a) 54900 (b) 549000
(c) 990000 (d) 99000
50 Branded Assembled 13. What is the proportion of Government internet owners to
the Private internet owners in the period 2011-12?
40 (a) 13 : 4 (b) 13 : 3
(c) 3 : 13 (d) 4 : 13
30
14. What is the approximate percentage increase in the Private
internet owners in the period 2013-14 over that in the period
20
2010-11?
(a) 5000 (b) 6000
10
(c) 8000 (d) 4000
15. What is the approximate percentage of Private internet
0
owners in the total internet owners in 2010-11?
2010 2011 2012 2013 2014
(a) 20 (b) 5
(c) 10 (d) 15
y
o
u
Data Interpretation B-221

rs
If the income of company C in the year 2013 was ` 35 lakhs,

m
DIRECTIONS (Qs.16-21): Study the following table to answer 22.

a
the given questions: what was its expenditure in that year?

h
Production (in crore units) of six (a) ` 24 lakhs (b) ` 21 lakhs

b
o
companies over the year (c) ` 25 lakhs (d) Can’t be determined

o
b
23. If, in the year 2014, total expenditure of companies B and C

.w
Company Years Total was ` 48 lakhs, then what was their total income in the same

o
2009 2010 2011 2012 2013 2014 year?

rd
TP 103 150 105 107 110 132 707 (a) ` 32 lakhs (b) ` 28.6 lakhs

p
re
ZIR 75 80 83 86 90 91 505 (c) ` 34.2 lakhs (d) Can’t be determined

s
AVC 300 300 300 360 370 340 1970 24. If, in the year 2013, expenditure of Company C was ` 32

s
.c
CTU 275 280 281 280 285 287 1688 lakhs, what was the income of the company in the same

o
m
PEN 25 30 35 40 42 45 217 year?
SIO 85 87 89 91 92 96 540 (a) ` 44.2 lakhs (b) ` 44.80 lakhs
Total 863 927 893 964 989 991 5627 (c) ` 46.4 lakhs (d) ` 38 lakhs
25. If the expenditures of Company E in the years 2013 and
16. The production of Company AVC in 2012 is approximately 2014 were the same, what was the ratio of the incomes of
what per cent of its average production over the given years?
the company in the same years respectively?
(a) 300 (b) 110 (c) 136 (d) 18.25
(a) 19 : 21 (b) 11 : 12
17. For SIO, which year was the per cent increase or decrease
in production from the previous year, the highest? (c) 29 : 31 (d) 9 : 11
(a) 2013 (b) 2010 26. The income of Company D in the year 2013 was ` 31 lakhs.
(c) 2014 (d) 2012 What was the earned profit?
18. Which company has less average production in the last (a) ` 11 lakhs (b) ` 20 lakhs
three years compared to that of first three years? (c) ` 17 lakhs (d) ` 12 lakhs
(a) No company (b) CTU DIRECTIONS (Qs. 27-31): Study the following table carefully
(c) ZIR (d) Non of these and answer the questions given below:
19. The total production of the six companies in the first two Number of students studying in different faculties in seven
given years is what per cent of that of last two given years? institutions
(round off up to two decimal places) Faculty
(a) 87.08 (b) 104.55 Ins tit- Arts Com- S cience Engine- Mana-
(c) 90.40 (d) 10.62 ution merce ering gement
20. For ZIR, which of the following is the difference between A 125 187 216 98 74
production in 2014 and that in 2013? B 96 152 198 157 147
(a) 10,00,00,000 (b) 1,00,00,000 C 144 235 110 164 127
(c) 10,00,000 (d) 40,00,000 D 165 138 245 66 36
21. For how many companies did the production increase every E 215 196 287 86 66
year from that of the previous year? F 184 212 195 112 97
(a) One (b) Two
G 255 206 182 138 89
(c) Three (d) Four
27. What is the percentage of students studying science in the
institue G with respect to the total number of students
DIRECTIONS (Qs. 22-26): These questions are based on the studying in the institute G?
graph given below: (a) 17.20 (b) 12.70
Per cent profit earned by six companies during 2013 and
1 2
2014 (c) 21 (d) 21
Profit = Income – Expenditure 3 3
28. Out of the total students of the institute ‘D’, approximately
Income – Expenditure
% Profit = ×100 what percentage of students study Management?
Expenditure (a) 6 (b) 8
Year 2013 Year 2014 (c) 12 (d) 10
70 29. The total number of students studying Arts in institutes A,
B and C together is approximately what per cent of the total
Per cent profit earned

60 number of students studying commerce in institutes D, E, F


50 and G together?
40 (a) 50 (b) 45
30 (c) 42 (d) 55
30. What is the percentage of students studying Engineering
20 in institute C with respect to the total students of all
10 institutions studying Engineering? (rounded to the nearest
0 integer)
A B C D E F (a) 19 (b) 20
Company (c) 18 (d) 21
y
o
u
B-222 Data Interpretation

rs
31. In which institution, the percentage of students studying

m
38. The marked price of the book is 20% more than the CP. If the
Commerce with respect to the total students of the institution

a
marked price of the book is `30, then what is the cost of the

h
is maximum? paper used in a single copy of the book ?

b
o
(a) F (b) E (a) `6 (b) `5

o
(c) C (d) A

b
(c) `4.5 (d) `6.50

.w
39. Which two expenditures together will form an angle of 108°

o
DIRECTIONS (Qs. 32 - 36) : These questions are to be answered at the centre of the diagram ?

rd
on the basis of the pie chart given below showing how a person’s (a) A and E (b) B and E

p
monthly salary is distributed over different expense heads.

re
(c) A and D (d) D and E

s
40. If the difference between the two expenditures are represented

s
.c
by 18 degrees in the diagram, then these expenditures must

o
Food

m
15% be :
House Rent (a) B and E (b) A and C
Petrol 30%
10% (c) A and D (d) D and E
Entertainment
DIRECTIONS (Qs. 41 - 44) : Study the following pie-charts
15% Clothes carefully to answer the questions that follow :
Savings 7% Percentage of Students in a College, studying in various
23% subjects and the percentage of girls out of these
Total students: 1800 (1200 girls + 600 boys)
32. For a person, whose monthly salary is Rs 6,000 p.m., how Percentage of students in various subjects
many items are there on which he has to spend more than
` 1,000 p.m ? Biology Law
(a) 1 (b) 2 13% 20%
(c) 3 (d) 4 Arts
12%
33. The annual saving for such a person will be approximately:
(a) ` 5,000 (b) ` 10, 000 Computers
Political 15%
(c) ` 15, 000 (d) ` 16, 560 Science

Ma %
34. The monthly salary for a person who follows the same 35%

5
ths
expense pattern, but has a petrol expense of Rs 500 p.m., is
(a) ` 2,500 (b) ` 3, 000 Total Girls : 1200
(c) ` 5, 000 (d) ` 6,500 Percentage of girls in various subjects
35. The percentage of money spent on clothes and savings is
equal to which other single item of expense ? Maths
(a) Petrol (b) House rent 2%
(c) Food (d) Entertainment
36. The angle made at the centre of the pie chart by the sector
representing the expense on petrol is : ts
Ar
(a) 30° (b) 45°
%
14

(c) 36° (d) 90°


DIRECTIONS (Qs. 37 - 40) : In the chart given below , A, B, C, Law Biology
D and E represent the costs of paper, printing, binding, 30% 14%
miscellaneous and the royalty, respectively in publishing a book.
E A
ter
15% 20% Compu
10% Political Science
D 30%
10%

41. The number of girls studying art in college is


B (a) 242 (b) 168
25% (c) 120 (d) 276
C
30% 42. For which subject is the number of boy the minimum?
(a) Law (b) Biology
37. What is the angle of the pie chart showing the expenditure (c) Arts (d) Maths
incurred on royalty ? 43. For Political Science, what is the respective ratio of boys
(a) 15° (b) 24° and girls?
(c) 48° (d) 54° (a) 4 : 3 (b) 3 : 4
(c) 2: 3 (d) 4 : 5
y
o
u
Data Interpretation B-223

rs
m
44. The number of girls studying art is what per cent more 50. In the case of which soft drink was the average annual sale

a
than the number of boys studying art? maximum in the given period?

h
(a) 170% (b) 150%

b
(a) Pep-up only (b) Cool-sip only

o
(c) 80% (d) 250% (c) Dew-drop only (d) Cool-sip and Dew-drop

o
b
DIRECTIONS (Qs. 45 - 48) : Answer the Questions based on the 51. In the case of Cool-sip drink, what was the approximate per

.w
Pie chart. cent increase in sale in 2013 over its sale in 2012?

o
rd
Nati onal Budget Expenditure
(a) Less than 20 (b) 20-25

p
(percentage al location) (c) 25 (d) 31-35

re
International 52. In the year 2011, what was the difference between the number

s
9%

s
of ‘Pep-up’ and ‘Cool-sip’ bottles sold?

.c
Interest on (a) 50,00,000 (b) 5,00,000

o
Debit 9%

m
(c) 50,000 (d) 10,00,000
Military
53. What was the approximate per cent drop in sale of Pep-up
59% in 2011over its sale in 2010?
Veterans
16% (a) 5 (b) 12
(c) 14 (d) 20
DIRECTIONS (Qs. 54 - 58) : The following pie-chart shows the
Other monthly expenditure of a family on food, house rent, clothing,
7%
education, fuel and miscellaneous. Study the pie-chart and
45. Approximately how many degree should there be in the answer questions that follow:
angle of the sector for military expenditure?
(a) 312.4 (b) 341.4
(c) 241.4 (d) 212.4 30%
46. What is the ratio of military expenditure to veterans Food
expenditure approximately?
(a) 59 : 6 (b) 1 : 10 12%
(c) 54 : 10 (d) None of these Clothing
5% Miscellaneous
47. If India had a total expenditure of Rs. 120 billion, 18% 15%
approximately how many billions did it spend on interest Education Fuel
on debt?
(a) ` 8.10 billion (b) `12.9 billion
(c) `10.8 billion (d) `11.12 billion 54. If the expenditure for food is ` 9000, then the expenditure
48. If `9 billion were spent for veterans, what would have been for education is
the total expenditure for that year? (a) ` 5000 (b) ` 5200
(a) ` 150 billion (b) ` 56. 25 billion (c) ` 5400 (d) ` 6000
(c) ` 140 billion (d) ` 145 billion 55. The central angle of the sector for the expenditure on fuel
(in degrees) is
DIRECTIONS (Qs. 49-53): Study the following graph carefully (a) 50.4 (b) 54
and answer the questions given below: (c) 57.6 (d) 72
56. If the expenditure on fuel is ` 3000, then the total expenditure
Dew-drop Cool-sip Pep-up excluding expenditure on house rent and education is
(a) ` 11600 (b) ` 12000
40 35 (c) ` 12400 (d) ` 12500
35 30 57. If the percentage of expenditure on food is x% of the total
30 30
Sale In lakh bottles

30
30 percentage of expenditure on clothing, education and fuel,
25 25 25 25 25
25 then x equals
20 20 20 20
20 1
15 15 (a) 66 (b) 66
15 3
10
10 7 2
5 (c) 66 (d) 67
3
0 58. Total percentage of expenditure on house rent, clothing
2009 2010 2011 2012 2013 2014 and fuel is greater than the percentage of expenditure on
Year food by
49. In which year was the sale of ‘Pep-up’ the maximum? (a) 16 (b) 17
(a) 2011 (b) 2012 (c) 18 (d) 20
(c) 2013 (d) 2010
y
o
u
B-224 Data Interpretation

rs
m
DIRECTIONS (Qs. 59-61): The following graph shows the DIRECTIONS (Qs. 64-65): The adjacent histogram shows the

a
expenditure incurred in bringing a book, by a magazine average pocket money received by 60 students for a span of one

h
b
producer. Study the graph and answer question. month. Study the diagram and answer the question.

o
o
b
.w
20

o
rd
Cost of

p
Printing 16

re
35%

s
No. of students

s
.c
Binders Royalty 12

o
Charge 15%

m
15% 8
Misc
Cost of Adv. 4%
Paper charges 4
16% 15%

0
50 80 110 140 170 200
Pocket Money (in `)
59. What should be the central angle of the sector for the cost
of the paper ?
(a) 57.6° (b) 54.4°
64. Maximum number of students received pocket money
(c) 56.7° (d) 54.8°
between
60. If the miscellaneous charges are ` 6,000, the cost of paper is
(a) 50 – 80 (b) 140 –170
(a) ` 12,000 (b) ` 18,000 (c) 80 – 110 (d) 110 – 140
(c) ` 15,000 (d) ` 24,000 65. The number of students who received pocket money upto
61. If 5500 copies are published, miscellaneous expenditures ` 140 is
amount to ` 1,848, find the cost price of 1 copy. (a) 20 (b) 32
(a) ` 10.40 (b) ` 9.40 (c) 48 (d) 56
(c) ` 12.40 (d) ` 8.40 DIRECTIONS (Qs. 66-68): Study the following table and answer
DIRECTIONS (Qs. 62-63): The pass percentage for an the questions that follow:
examination in a school is shown in the adjoining bar diagram, School No. of Percentage No. of
for males and females separately for four years. Study the students of students students
diagram and answer the question. scoring scoring appeared
50 marks marks
Male less than more than
Percentage of students

40 50% 50%
Female A 240 55 600
30 B 220 40 400
C 300 20 375
20 D 280 10 350
E 210 25 300
10
66. The ratio of the total number of students scoring marks less
than 50% to that of of scoring marks exactly 50% is
2007 2008 2009 2010 (a) 50 : 3 (b) 25 : 2
Years (c) 25 : 4 (d) 35 : 2
67. Which school has the highest number or students scoring
62. The maximum percentage of students passed in the year is
exactly 50% marks?
(a) 2007 (b) 2008
(c) 2009 (d) 2010 (a) D (b) E
63. The year in which the difference of pass percentage between (c) B (d) A
male and female is maximum, is 68. The total number of students scoring 50% or more marks is
(a) 2010 (b) 2009 (a) 1250 (b) 875
(c) 2008 (d) 2007 (c) 775 (d) 675
y
o
u
Data Interpretation B-225

rs
m
DIRECTIONS (Qs. 69-72): Study the following graph which DIRECTIONS (Qs. 73-77) : The following pie-chart represents

a
shows income and expenditure of a company over the years the profits earned by a certain company in seven consecutive

h
2005-2009 and answer questions. years. Study the pie-chart carefully and answer the question.

b
o
o
24° Years

b
.w
1991

o
1992

rd
42°

p
1993

re
60°

s
66° 1994

s
.c
o
54° 1995

m
72°
1996
1997
73. If the expenditure in the year 1993 was 30% more than the
69. The difference in profit (` in crores) of the company during expenditure in the year 1991, then the income in the year
2006 and 2007 is 1993 exceeds the income in the year 1991 by 30% of
(a) 10 (b) 15 (a) the income in the year 1991
(c) 20 (d) 25 (b) the expenditure in the year 1993
70. In how many years was the income of the company less (c) the income in the year 1993
than the average income of the given years? (d) the expenditure in the year 1991
(a) 4 (b) 3 74. If x% of the total of profits earned in all the given years is
(c) 2 (d) 1 same as the profit earned in the year 1994, then x is
71. The percentage increase in expenditure fo the company 2 1
(a) 16 (b) 33
from 2007 to 2008 is 3 3
(a) 20 (b) 25
1 2
(c) 30 (d) 35 (c) 12 (d) 11
72. Profit of the company was maximum in the year 2 3
(a) 2009 (b) 2008 75. The ratios of expenditures and incomes in the years 1992,
(c) 2006 (d) 2005 1994 and 1996 are given to be 6 : 5 : 8 and 2 : 3 : 4 respectively.
The ratio of the income in the year 1996 to the total
expenditure in the years 1992 and 1994 is
(a) 40 : 11 (b) 10 : 7
(c) 20 : 11 (d) 20 : 13
76. The year in which the profit is nearest to the average of the
profits earned in all the given years is
(a) 1991 (b) 1995
(c) 1993 (d) 1994
77. If the income in the year 1997 was 5 times the expenditure
made in the same year, then the ratio of the profit earned in
the year 1991 to the expenditure in the year 1997 was
(a) 11 : 28 (b) 44 : 7
(c) 28 : 11 (d) 7 : 44

ANSWER KEY
1 (c) 10 (c) 19 (c) 28 (a) 37 (d) 46 (d) 55 (b) 64 (a) 73 (d)
2 (a) 11 (d) 20 (b) 29 (a) 38 (b) 47 (c) 56 (c) 65 (c) 74 (a)
3 (a) 12 (c) 21 (c) 30 (b) 39 (c) 48 (c) 57 (c) 66 (b) 75 (c)
4 (c) 13 (b) 22 (c) 31 (c) 40 (d) 49 (d) 58 (b) 67 (c) 76 (b)
5 (a) 14 (c) 23 (d) 32 (b) 41 (b) 50 (a) 59 (a) 68 (b) 77 (c)
6 (d) 15 (b) 24 (b) 33 (d) 42 (a) 51 (c) 60 (d) 69 (c)
7 (d) 16 (b) 25 (c) 34 (c) 43 (b) 52 (d) 61 (d) 70 (c)
8 (d) 17 (c) 26 (a) 35 (b) 44 (d) 53 (c) 62 (c) 71 (b)
9 (d) 18 (d) 27 (d) 36 (c) 45 (d) 54 (c) 63 (d) 72 (c)
y
o
u
B-226 Data Interpretation

rs
m
a
h
b
o
o
b
.w
1. (c) Total marks obtained by Meera
2 100

o
= 100 + 80 + 50 + 90 + 90 + 60 = 470 2012 = = 2.24%

rd
2. (a) Average marks obtained by seven students in History 89

p
re
80 70 70 60 90 60 80 1 100
2013 = =1.09%

s
= 72.86

s
7 91

.c
o
3. (a) Only Kunal and Soni got 60% or more marks in all the 4 100

m
subjects. 2014 = =4.35%
92
4. (c) Average percentage of Kunal You can solve it with simple rough work. See the
90 70 60 90 70 70 difference of produced units between two consecutive
= 75% years. The difference is maximum for 2013 to 2014, and
6
production during all these years is almost same.
6. (d) Average percentage growth of Assemble PCs
Hence, in the year 2014 SIO registered maximum
20 25 25 50 55 175 increase in production over the previous year.
35%
5 5 18. (d) Sum of the productions of the companies in first three
7. (d) Number of Branded PCs sold in 2014 years and the last three years in ` crore is as follows:

30 Company First three years Last three years


100000 300000 TP 358 349
10
ZIR 238 267
8. (d) Difference between Assembled and Branded PCs
AV C 900 1070
2010 2011 2012 2013 2014
10% 15% 5% 20% 25% CTU 836 852
10. (c) Per cent growth of Assembled PCs is PEN 90 127
1996 1997 1998 1999 SIO 261 279
5% No change 25% 5% 19. (c) Total production of the six companies in first two given
11. (d) Percentage increase in the total Internet owners years = 863 + 927 = 1790
Again, total production of the six companies in last
2008-09 2009-10 2010-11 2011-12 2012-13 2013-14
two given years = 989 + 991 =1980
653.85 206.12 168.89 164.46 135.94 65.56 1790 100
12. (c) Reqd number Therefore, required per cent = 90.40%
1980
= (1550000 + 950000) – (1060000 + 450000) = 990000 20. (b) The required difference
13. (b) Reqd ratio = 520000 : 120000 = 13 : 3 = (91 – 90) crore units =1×10000000= 10000000 units
14. (c) Reqd percentage increase 21. (c) Those companies are:
950000 -12000 ZIR PEN and SIO
= ´100 » 8000%
12000
22. (c) Expenditure of Company C in 2013
12000
15. (b) Reqd percentage = ´100 » 5% 100
230000 + 12000 35 = ` 25 lakhs
140
16. (b) Production of company AVC in 2012 =360 crore units
23. (d) Here, the percentage profits of Companies B and C in
Average production of AVC over the given years
2014 were not the same. Therefore, can’t be determined
1970 is the correct choice.
= 24. (b) Income of Company C in 2013
6
360 6 140
Hence, required per cent = 100 = 32 = ` 44.80 lakhs
1970 100
= 109.64% 110% 25. (c) Reqd ratio = 145:155 = 29 : 31
17. (c) Approximate per cent increase or decrease in 26. (a) Expenditure of Company D in 2013
production from the previous year for SIO are as
follows: 100
= 31 = ` 20 lakhs
155
2
2010 = ×100 = 2.35% Profit = Income – Expenditure = 31 – 20 = ` 11 lakhs
85 27. (d) Total no. of students studying in institute G
2 100 = 225 + 206 + 182 + 138 + 89 = 840
2011 = = 2.29%
87
y
o
u
Data Interpretation B-227

rs
m
182 2 For (Qs. 41-44)
Reqd % 100 21 %

a
h
840 3 Subjects Students Girls Boys

b
28. (a) Total students in institute D

o
Art 216 168 48

o
= 165 + 138 + 245 + 66 + 36 = 650 Biology 234 168 66

b
.w
36 Law 360 360 Nil
Reqd % 100 6%

o
650 Computers 270 120 150

rd
Maths 90 24 66

p
125 96 144

re
29. (a) Reqd % 100 48.50% Political Science 630 360 270

s
138 196 212 206

s
Total 1800 1200 600

.c
164

o
30. (b) Reqd % 100 20%

m
821 41. (b) 168
187 42. (a) There is no boy studying Law in the college.
31. (c) Percentage of institute A 100 26.71% 43. (b) Required ratio = 270 : 360 = 3 : 4
700
168 48
152 44. (d) Required percentage 100 250% .
% of institute B 100 20.26% 48
750
% of institute C = 30.12 % 59
45. (d) Required degree = 360 212.4
% of institute D = 21.23 % 100
% of institute E = 23.05 %
59
% of institute F = 26.5 % and G = 23.67 % 46. (d) Required ratio = 59 : 16
Hence, maximum % is in institute C. 16
9
32. (b) On two items, savings and house rent, he has to invest 47. (c) Spend on interest on debt = 120 = 10.8 billion
100
more than Rs 1000.
33. (d) Savings per month = 6000 × 23% = Rs 1380 9
48. (b) Total expenditure = 100 = 56.25 billion
Annual savings = 1380 × 12 = Rs 16,560 16
34. (c) 10% = 500 100% = Rs. 5000 49. (d) Sale of Pep-up was the maximum in the year 2010.
35. (b) Money spent on clothes + Savings = Money spent on 50. (a) Avg annual sale of Dew-drop
house rent.
10 15 25 15 30 25
36. (c) Angle subtended by the Petrol sector = = 20 lakhs
6
10 Avg. annual sale of Cool-sip
= 360 36
100
25 + 7 + 20 + 20 + 25 + 30
37. (d) % of Royalty = 15% = = 21.16 lakhs
6
15 Avg. annual sale of Pep-up
Angle made by Royalty = 360 54
100
30 + 35 + 30 + 25 + 20 + 20
38. (b) Marked price = 1.2 × Cost price = 26.66 lakhs
6
30
Cost price = = Rs. 25 25 20
1.2 51. (c) Reqd % = × 100 = 25%
20
Cost of paper = 25 × 20% = Rs. 5 52. (d) Reqd no. = 30 – 20 = 1000000
39. (c) We have, 360° = 100%
35 30
53. (c) Reqd % drop = 100 14%
100% 35
108 108 30%
360 9000
54. (c) Expendicture for eduction 18 `5, 400
From the given pie chart. (A + D) = 30% 30
55. (b) Central angle of the sector for the expenditure on fuel
18
40. (d) 18 100% 5% 360
360 15 54º
100
So, the difference in percentages of expenditures be 56. (c) Expenditure excluding rent and eduction
5% 3000
Thus, required expenditures are C and B or D and E. (100 – 20 –18)
15
or A and B or A and E.
= 200 × 62
= ` 12400
y
o
u
B-228 Data Interpretation

rs
m
57. (c) 30 = x% of (12 + 18 + 15) 40 60 50 65 70 285

a
70. (c) Average Income = = 57

h
x 5 5
45

b
30 =

o
100 Income during 2005 and 2007 is less than average

o
b
200 2 50 40

.w
x 66 71. (b) Required % = 100 = 25%
3 3 40

o
rd
58. (b) Required difference = (20 + 12 + 15) – 30 72. (c) It is clear from the graph.

p
= 47 – 30

re
For Qs. 73-77

s
= 17 . Profit percentage in given years.

s
.c
59. (a) Central angle of the sector for cost of the paper

o
42
1991 100 11.67%

m
Cost of paper % 16 360
360º 360º 57.6º
100 100
24
60. (d) If the cost of paper = ` x, then from the given pie-chart 1992 100 6.67%
360
Cost of paper 16% 42
Miscellaneous charges 4% 1993 100 11.67%
360
x 16 16 6000 60
x = ` 24000 1994 100 16.67%
6000 4 4 360
62. (c) By observing the graph, we can say that yr. 2009 has 54
maximum percentage of students passed in the year. 1995 100 15%
360
63. (d) Year 2007.
65. (c) 20 + 12 + 16 = 48 72
1996 100 20%
66. (b) Number of students scoring less than 50% 360
= (240 + 220 + 300 + 280 + 210) = 1250 66
Number of student scoring exact 50% 1997 100 18.33%
360
= (30 + 20 + 0 + 35 + 15) = 100 74. (a) x % of 100 = 16.67%
Ratio = 1250 : 100 = 25 : 2
68. (c) Number of students scoring 50% or more marks 2
x 16 %
= (600 – 240) + (400 – 220) + (375 – 300) + (350 – 280) 3
+ (300 – 210) 76. (b) Average of year 1995
= 360 + 180 + 75 + 70 + 90 = 775 11.67 6.67 16.67 15 20 18.33
69. (b) Profit during 2006 = (60 – 35) = 25 =
7
Profit during 2007 = (50 – 40) = 10
= 14.28%
Difference = 25 – 10 = 15
y
o
u
rs
SECTION - C : ENGLISH COMPREHENSION

m
a
h
b
o
o
b
CHAPTER

.w
1

o
GRAMMAR

rd
p
re
s
s
.c
o
m
Grammar is the key to attaining success in a competitive examination. If you want success in a competitive exam, it is important to have
a good command of grammar and its applications. No matter how hard you prepare for your competitive exams, you can never be too
sure of your success.
Before sitting for an examination, it is important to evaluate yourself, where you stand and how much chances of your success are.
In the English section of a competitive exam, the kind of questions likely to be asked can be categorised in three parts
(1) Fill in the blanks;
(2) Identifying errors in sentences and
(3) Correcting the sentences.
The questions can be dealt with easily and you can score well if you get your basics clear and right.
HOW THIS BOOK WILL HELP YOU?
This book will help you to understand how language and its components work. It is oriented towards making you more confident user of English
by giving you an insight into the correct usage. The material provided is user-friendly with adequate examples and ‘practice exercises’.
If you make a concentrated effort, it will not only prepare you for the forthcoming competitive exams but also fine-tune your
communication skills.
READING: To supplement your efforts, you should build up reading habits. The material can be of any kind - magazines, newspapers
or novels. But, one should consciously look at the usage. Good reading habits will definitely build up your understanding of
grammatical usage and help you succeed in competitive exams.
We wish you all the best in your endeavour.

NOUNS Here, the singular and plural form of the noun Deer is the
same. Like Deer there are other nouns that have the same
A Noun is a word used as a name of a person, place or thing. form in singular as well as plural form. For example: sheep,
There are five kinds of Noun : deer, apparatus, species, series, hundred, dozen, hair etc.
(a) Proper Noun Preceding adjectives and articles decide whether the word
(b) Common Noun is used in the singular form or plural form.
(c) Collective Noun For example :
(d) Abstract Noun (a) He paid eight hundred rupees for this pair of shoes.
(e) Material Noun (b) India again lost the series.
GRAMMAR REGARDING NOUNS THAT WORLD BE Nouns denoting large numbers are used both in singular
USEFUL IN A COMPLETE EXAM : and plural form.
1. Proper nouns are sometimes used as common nouns. 3. For example:
For example : (a) Three hundred people attended the function.
(a) Amitabh is Gandhiji of our class. (Incorrect) (b) Hundreds of people attended the party.
(b) Amitabh is the Gandhiji of our class, (Correct) In sentence a), ‘hundred’ is preceded by number ‘three’. So
Here Gandhiji does not mean Mahatma Gandhi. The word ‘hundred’ will take no plural form. Word ‘three hundred’
here stands for the possessor of the qualities that Gandhiji indicates plurality . But in sentence b), ‘hundred’ is not preceded
is most known for - truth and non-violence. Thus Gandhiji by any number. So to indicate plurality , we will write ‘hundreds’.
is being used as a metaphorical common noun. Some nouns So, rule is that when words like hundred, dozen, thousand,
have the same form both in singular as well as plural forms. pair, score are not preceded by any word denoting number
FOLLOWINGARE RULES REGARDING THE NUMBER OF then they take the plural form. Otherwise not.
THE NOUN Consider some more examples :
2. For example : (a) Coca-Cola paid lakhs of rupees to Aamir Khan for
(a) Deer was caught promoting their product.
(b) Deer were caught. (b) I brought two dozen bananas.
y
o
u
C-2 Grammar

rs
m
4. Tell which sentence is correct: Vocabulary

a
(a) Since long no news has been heard. Other nouns having different meanings in the singular and

h
plural form are:

b
(b) Since long no news have been heard.

o
Sentence (a) is correct. The reason is that some nouns are

o
Singular with meaning Plural with meaning

b
always used as singular though they look like plural nouns.

.w
Advice - counsel Advices - information
That’s why we should never use the ‘plural verb with these

o
Respect - regard Respects - compliments

rd
words. Other similar words are politics, mathematics,
Compass - extent Compasses - instrument or range

p
physics, gallows, means, billiards, ethics, summons, innings.

re
Custom - habit Customs - duties levied on
For example :

s
Ground - Earth Grounds - reasons

s
(a) Politics is not my cup of tea.

.c
Iron - metal Irons - fetters made of iron
(b) I received summons.

o
Mean - average Means - way or method

m
(c) Sachin once again played a superb innings.
5. Tell which sentence is correct: Respect - regard Respects - polite greetings
(a) The spectacles that you are wearing are really nice. Colour - hue Colours - appearance
(b) The spectacles that you are wearing is really nice. Physic - medicine Physics - natural science
Sentence (a) is correct. The reason being that some noun FOLLOWING ARE RULES REGARDING GENDER OF THE
words are always used in the plural form. NOUN :
For example : trousers, arms, drawers, assets, scales, alms, 9. Go through the following singulars and plurals as plural
thanks, cards; ashes, riches, premises, scissors, credentials, forms are commonly known but their singular forms are
proceeds. not commonly kown.
6. Tell which sentence is correct:
Singular Form Plural form
(a) The cattle was grazing in the field.
(b) The cattle were grazing in the field. Agendum Agenda
Sentence (b) is correct .The reason being that some nouns Alumnus Alumni
are always used as plurals though they look like singular. Index Indices
Other nouns like this are—public, people, folk, mankind, Phenomenon Phenomena
poultry, sheep, and police. gentry, peasantry, bulk, majority. Criterion Criteria
For example: Radius Radii
(a) The majority are with the leader. Formula Formulae
(b) Police, though late, have come. Memorandum Memoranda
(c) Public wants results. 10. Some noun words have two plurals with different
7. Tell which sentence is correct. meanings. So that plural form should be selected which will
(a) This project will lead to lots of expenditures. convey the right meaning.
(b) This project will lead to lots of expenditure. For example :
Sentence (b) is correct. The reason is that some nouns are (a) I have one brother and one sister (meaning- sons of
always used as singular. Preceding adjectives or the verb the same parents).
form indicates the singularity or plurality. Other nouns (b) Why should only select brethren be allowed to attend
are–expenditure, furniture, information, machinery, issue, the meeting? (meaning - members of same society,
offspring, alphabet, scenery, poetry. organisation)
For example : (c) I took off my shoes and clothes (meaning- things that
(a) All the furniture was bought last year. people wear).
(b) All the Information was given to him. (d) Cotton, Nylon, Silk are different kinds of cloths
8. Meaning of some nouns in plural form is very different (meaning- kinds or pieces of cloth).
from the meaning of nouns in singular form. Hence, that Other nouns having two plurals with different meanings are.
form should be used which will convey the right meaning.
Singular Plural with different meaning
For example:
(a) I opened the letter and read its contents. Die Dies - stamps
(b) Her mouth was fixed in a smile of pure content. Dice - small cubes used in games
(c) The conflict between good and evil is age old. Genius Geniuses-persons of great talent
(d) We must produce goods at competitive prices. Genie - spirit
(e) Delhites breathe the most polluted air in the world. Quarter Quarter - fourth part
Quarter(s) - lodging
(f) She was just putting on airs when she came to visit us/me.
Manner Manner - Method
(g) We should renounce the use of force to settle our
Manners - Correct behaviour
dispute.
Pain Pain - Suffering
(h) Families of people who died as a result of services in
Pains - Careful efforts
the forces should not be ignored. Spectacle Spectacle - sight
(i) I was very excited on my return to my home village. Spectacles - eye-glasses
(j) Early returns in the ballot indicate majority for Penny Pence-indicate amount of money
opposition. Pennies number of coins
y
o
u
Grammar C-3

rs
m
11. Collective nouns, even when they denote living beings, are (c) Plural nouns ending in (s) like boys, cows: add(’) (an

a
considered to be of the neuter gender. apostrophe) to regular plurals.

h
For example :

b
• The boys’ football club, the students union, the

o
(a) Lalu had a herd of cows. He kept a herdsman to look labours’ movement.

o
b
after her. (d) Plural nouns not ending in (s) like men, children: (’s) is

.w
(b) Lalu had a herd of cows. He kept a herdsman to look after it. added after the word.

o
Sentence (b) is correct. Though herd consists of cows • men’s club, children’s wear

rd
(females), herd is not a feminine noun as it a collective noun. (e) ’s is added primarily after the living things and

p
re
12. Young children and the lower animals are also referred to as personified objects. For example: Governor ’s

s
of the neuter gender. bodyguard, horse’s head, Nature’s law, Fortune’s

s
.c
For example : favourite.

o
(a) The baby loves his toys. (Incorrect)

m
(f) ’s is not used with inanimate or non-living things. For
(b) The baby loves its toys. (correct) example:
(c) The mouse lost his tail when the cat pounced on him. table’s leg (incorrect)
(Incorrect) leg of the table (correct)
We are often uncertain regarding the gender of the animals. book’s cover (incorrect)
The mouse here may be a male or a female. So, English language cover of the book (correct)
prefers the easy way out: treat it as of the neuter gender. (g) But nouns that denote time, distance or weight, (’s) is
13. When objects without life are personified they are used. For example: a stone’s throw, in a year’s time,
considered of: the earth’s surface.
(i) The masculine gender, if the object is remarkable for (h) Some other common phrases where (’s) is used are : to
strength and violence. Ex. : Sun, Summer, Winter, Time, his heart’s content, at his wit’s end, out of harm’s way.
Death etc. (i) When a noun consists of several words, the
(ii) The feminine gender, if the object is remarkable for possessive sign is attached only to the last word.
beauty, gentleness and gracefulness. Ex: Earth, Moon, For example:
Spring, Nature, Mercy, etc. (a) The Queen’s of England reaction is important in
For example: the Diana episode. (Incorrect)
(a) The Sun came from behind the clouds and with (b) The Queen of England’s reaction is important in
her brilliance tore the veil of darkness. (Incorrect) the Diana episode. (Correct)
(b) The Sun came from behind the clouds and with Do not be mistaken that since it is the Queen’s reaction,
his brilliance tore the veil of darkness. (Correct)
the (’s) should come after Queen. You might think that
Convention does not see brilliance as a womanly
putting it after England would make the reaction
quality, but a manly one.
England’s and not the Queen’s. This is shortsightedness.
(a) Nature offers his lap to him that seeks it. Do not see Queen and England in isolation, Queen of
(Incorrect) England is one whole unit and the apostrophe should
(b) Nature offers her lap to him that seeks it. (Correct) come at its end.
The offering of a lap is usually the mother’s role. Hence, (j) When two nouns are in apposition, the possessive
Nature here should be treated as a feminine noun. sign is put to the latter only.
Tell which sentence is correct. For example :
(a) The earth goes round the sun in 365 days. Can (a) I am going to Stephen Hawking’s the scientist’s
you calculate her speed? country. (Incorrect)
(b) The earth goes round the sun in 365 days. Can (b) I am going to Stephen Hawking the scientist’s
you calculate its speed? country. (Correct)
Sentence (b) is correct. The error being made here is that (k) When two or more nouns show joint possession, the
personification is being brought where it does not exist. In possessive sign is put to the latter only.
the above statement the earth is being treated as a body (a For example:
thing), not a person. The scientist here is not concerned (a) Amitabh and Ajitabh are Bachchanji’s sons. So
with the womanly qualities of the planet. So, neuter gender Bachclaanji is Amitabh’s and Ajitabh’s father.
should be applied. (Incoreect)
(b) Amitabh and Ajitabh are Bachchanji’s sons. So
FOLLOWINGARE RULES REGARDINGAPOSTROPHE Bachchanji is Amitabh and Ajitabh’s father.
14. Rules regarding apostrophes—s: (Correct)
(a) Singular noun: We add(s) (apostrophe s’) to singular (l) When two or more nouns show separate possession,
nouns and to irregular plurals that don't end in – s: the possessive sign is put with both.
• Barry’s house, the college’s faculty, the women's For example.
liberation movement. (a) The audience listened to Javed and Vajpayee’s
(b) Singular noun: Only an apostrophe is added when poems. (Incorrect)
there are too many hissing sounds. For example: (b) The audience listened to Javed’s and Vajpayee’s
Moses’ laws, for goodness’ sake, for justice’ sake. poems. (Correct)
y
o
u
C-4 Grammar

rs
m
PRONOUNS RULES REGARDING PERSONAL PRONOUNS

a
8. Tell which sentence is correct:

h
A pronoun is a word used instead of a noun. (a) The presents are for you and me.

b
o
NOW CONSIDER THE FOLLOWING CASES (b) The presents are for you and I.

o
b
1. Since a pronoun is used instead of a Noun, it must be of the Sentence (a) is correct. Pronoun has to agree with the case.

.w
same number, gender and person as the noun for which it Here it is the objective case. So, ‘me’ should be used instead

o
of ‘I’. For example : My uncle asked my brother and me to

rd
stands. For example: Those beggars are idle. They refuse
dinner.

p
to work for their living.

re
2. Consider the following two sentences. 9. Tell which sentence is correct:

s
s
(a) After a few anxious moments; the jury gave its verdict (a) He loves you more than I.

.c
(b) He loves you more than me.

o
(Pronoun ‘its is used in place of noun jury’).

m
(b) The jury were divided in their opinions. (Pronoun Sentence (a) is correct ‘Than’ is a conjunction joining
‘their’ is used in place of noun ‘jury’) clauses. And the case of the pronoun to be used may be
found by writing the clauses in full. So, in sentence (a) two
You must be wondering why different pronoun its’ and
clauses joined by ‘than’ are ‘He loves you more’ and ‘I love
‘their’ is used in place of the same noun ‘jury’ The reason is
you’. Being a subjective case, ‘I’ should be used.
when a pronoun stands for a collective noun it must be in
For example:
the singular number and neutral gender. (Sentence (a). But
(a) He is taller than I (am).
when collective noun conveys the idea of separate
(b) He loves you more than (he loves) me.
individuals comprising the whole, the pronoun standing
10. When a pronoun refers to more than one noun or pronouns
for it must be of the plural number. In sentence (b), it is clear
of different persons, it must be of the first person plural in
that members of the jury are not behaving as a whole. preference to the second and of the second person plural in
For example : preference to the third.
(a) The committee is reconsidering its decision. For example :
(b) The committee decided the matter without leaving their (a) You and I, husband and wife, have to look after your
seats. home. (Incorrect)
PRONOUNS IN SENTENCES FOUND BY CONJUNCTION (b) You and I, husband and wife, have to look after our
home. (Correct)
3. When two or more singular nouns are joined by ‘and’, the
Now, common sense tells us that if we are a couple, wife
pronoun used for them must be plural.
and husband, the feeling of togetherness is expressed by
For example : Rama and Hari work hard. Their teachers
our home, not your home. And so does grammar.
praise them.
Rule: 123. I stands for first person, 2 for second person
But when two Singular nouns joined by ‘and’ refer to the
and 3 for third person. The order of precedence is: 1 before
same person or thing, the pronoun should be singular.
2 and 2 before 3. In the given example, we have 2 and 1. So
For example : The Secretary and Treasurer is negligent of
I will apply; that is, first person. The number, of course, will
his duty. be plural.
Here the same person is acting as Secretary and Treasurer. Let us take another example.
That’s why singular pronoun is used. (a) You and Hari have done their duty. (Incorrect)
4. When two singular nouns joined by ‘and’ are preceded by (b) You and Hari have done your duty. (Correct)
‘each’ or ‘every’, then the pronoun must be singular Applying 123 rule. You = 2 and Hari =3. So, 2. Second person
For example : Every soldier and every sailor was in his plural gives ‘your’.
place. Similarly, when all the three persons are taken into account,
5. When two or more singular nouns are joined by ‘or’, it has to be I; that is, first person plural.
‘either...or’, ‘neither...nor’, the pronoun is generally (a) You, he and I have not forgotten your roots.
singular. (Incorrect)
For example : (b) You, he and I have not forgotten our roots. (Correct)
(a) Neither Abdul nor Rehman has done his lessons. 11. Each, either and neither are always singular and are
(b) Either Rama or Hari must help his friend. followed by the verb in the singular.
6. When a plural and a singular noun are joined by ‘or’ or For example :
‘nor’, the pronoun must be in the plural (a) Neither of the accusations is true.
(b) Each boy took his turn.
For example : Either the manager or his assistants failed in
(c) Each of the lady performs her duty well.
their duty.
12. (A) Please consider the following sentences.
7. When two things which have been already mentioned are (a) This is the boy. He works hard. (He subjective case)
referred to; ‘this’ refers to the thing last mentioned and (b) This is the boy. His exercise is done well. (His is
‘that’ to the thing first mentioned. possessive case)
For example : Alcohol and Tobacco are both injurious: this (c) This is the boy. All praise him. (Him is objective
perhaps less than that. case)
y
o
u
Grammar C-5

rs
m
13. An apostrophe is never used in ‘its’, ‘yours’ and ‘theirs’. 22. Which is used for inanimate things and animals. Which is

a
14. The complement of the verb be, when it is expressed by a used for both singular as well as plural noun.

h
b
pronoun should be in the nominative form. For example :

o
For example. (a) I have found the book which I had lost last week.

o
b
(a) It was he (not him), (b) The horse which won the race yesterday, is my

.w
(b) It is I (not me) that gave the prizes away. favourite.

o
rd
(c) It might have been he (not him). 23. When ‘which’ is used for selection, it may refer to a person

p
15. The case of a pronoun following than or as is determined as well as things.

re
by mentally supplying the verb. For example :

s
s
For example : (a) Which of the packets is yours?

.c
o
(a) He is taller than I (am). (b) Which of the boys has not done his homework?

m
(b) I like you better than he (likes you). 24. Who, Which, Whom, That, Whose should be placed as
(c) They gave him as much as (they gave) me. near to the antecedent as possible.
16. A pronoun must agree with its Antecedent in person, For example :
number and gender. (a) I with my family reside in Delhi, which consists of my
For example: wife and parents. (Incorrect)
(a) All passengers must show their (not his) tickets. This sentence is wrong as which relates to ‘my’ family’.
(b) I am not one of those who believe everything they (not I) So ‘which’ should be placed as near to family’ as
hear. possible. So, the correct sentence is.
RULES REGARDING DEMONSTRATIVE PRONOUNS (b) I with my family which consists of my wife and parents,
reside in Delhi.
17. That is used
25. Who is used in the nominative cases and whom in the
A. After adjectives in the superlative degree.
objective cases.
For example.
For example :
(a) This is the best that we can do.
(a) There is Mr. Dutt, who (not whom) they say is the best
(b) He is the best speaker that we ever heard.
B. After the words all, same, any, none, nothing, only painter in the town.
For example: (b) The Student, whom (not who) you thought so highly
(a) Man is the only animal that can talk. of, has failed to win the first prize.
(b) He is the same man that he has been. 26. When the subject of a verb is a relative pronoun, the verb
C. After two antecedents, one denoting a person and the should agree in number and person with the antecedent of
other denoting an animal or a thing. the relative.
For example : The man and the pet that met with an For example :
accident yesterday died today. (a) This is one of the most interesting novels that have
18. What and That refer to persons as well as things. (not has) appeared this year. (Here, antecedent of
relative pronoun that is novels and not one)
RULES REGARDING RELATIVE PRONOUNS (b) This is the only one of his poems that is (not are)
19. On combining each of the above pairs into one sentence worth reading. (Here the antecedent of that is one and
(a) This is the boy who works hard. not poems. Note the difference between sentence
(Who in place of He) (a) and (b).
(b) This is the boy whose exercise is done well.
(Whose in place of His) OTHER USEFUL RULES
(c) This is the boy whom all praise. 27. None is used in the singular or plural as the sense may
(Whom in place of Him) require.
The above sentences show when to use who, whose and For example:
whom. Who is the subjective case, Whose the possessive (a) Each boy was acompanied by an adult but there were
case and Whom the objective case. none with the orphan. (Incorrect)
20. Who is used for persons only. It may refer to a singular or (b) Each boy was accompanied by an adult but there was
plural noun. none with the orphan. (Correct)
For example : (c) I am used to many guests everyday but there was none
(a) He who hesitates is lost. today. (Incorrect)
(b) Blessed is he who has found his work. (d) I am used to many guests everyday but there were
21. Whose can be used for persons as well as things without none today. (Correct)
life also. 28. When ‘one’ is used as pronoun, its possessive form ‘one’s’
For example : should follow instead of his, her etc.
(a) This is the hotel whose owner is a criminal. For example : One must put one’s best efforts if one wishes
(b) This is the person whose willpower is extraordinary. to succeed.
y
o
u
C-6 Grammar

rs
m
29. With let objective case of the pronoun is used. 2. The different tenses and the verb forms used in each tense

a
For example : Let you and me do it. are given below :

h
b
30. If a pronoun has two antecedents, it should agree with the

o
Singular with meaning Plural with meaning

o
nearer one.

b
Name of Tenses Verb form used in Tenses

.w
For example : Pres ent s imple / indefinite Verb + s/es

o
(a) I hold in high esteem everything and everybody who

rd
Pres ent continuous / Is/am/are + verb + ing
reminds me of my failures.

p
Progres sive

re
(b) I hold in high esteem everybody and everything, which

s
Pres ent perfect Has / have + third form of verb

s
reminds me of my failures.

.c
31. In referring to anybody, everybody, anyone, each etc., the Pres ent perfect continuous Has/have + been + verb + ing

o
m
pronoun of the masculine or the feminine gender is used Past simple / indefinite Second form of verb
according to the context. Past continuous / W as /were + verb + ing
For example. Progres sive
(a) I shall be glad to help everyone of my boys in his Past perfect Had + third form of verb
studies. Past perfect continuous Had been + verb + ing
(b) I shall be glad to help everyone of my girls in her Future s imple / indefinite Shall / will + verb
studies.
Future continuous / Shall / will + be + verb + ing
(c) I shall be glad to help everyone of my students in his Progres sive
studies. Future perfect Shall/will + Have + past participle
But when gender is not determined, the pronoun of the
Future perfect continuous Shall/will + have been + verb + ing
masculine gender is used as in sentence (c).
32. (A) The pronoun one should be used throughout, if used 3. The simple Present tense is used
at all. A. To express a habitual action.
For example: For example : I get up every day at five o’clock.
(a) One must use one’s best efforts if one wishes to B. To express general truths.
succeed. For example : Fortune favours the brave.
C. In vivid narrative, as substitute for the simple past.
(b) One should be careful about what one says.
For example : Immediately the Sultan hurries to his
(B) Plural is commonly used with none. capital.
For example. D. To indicate a future event that is part of a fixed
(a) None of his poems are well known. programme or time table.
(b) None of these words are now current. For example : The train leaves at 5:20 am.
In case of ‘none’ the verb can be both singular Note: We can also use will leave in place of leaves.
and plural (according to the sense) E. It is used to introduce quotations.
33. Anyone should be used when more than two persons or For example : Keats says, ‘A thing of beauty is a joy
things are spoken of. forever’.
For example : She was taller than anyone of her five sisters. F. In exclamatory’ sentences beginning with here and
there to express what is actually taking place in the
TENSES present.
1. Tense is the form taken by a verb to indicate time and For example : Here comes the bus!
continuance or completeness of action. The continuance G. When two actions of the future are being talked about,
or completeness of action is denoted by four subcategories. one dependent on the other, the former action is
represented by present simple and the latter by future
(a) Simple Tense: It is need for habitual or routine actions simple.
in the Present Tense, action which is over in the Past For example : We shall go when the child comes back
Tense & action to happen in the Future Tense. home.
The action is mentioned simply. Nothing is said about 4. The Present Continuous tense is used
whether the action is complete. (I) For an action going on at the time of speaking.
(b) Continuous Tense : The action is incomplete or For example : The boys are playing cricket in the
continuous or going on. ground.
(c) Perfect Tense: The action is complete, finished or (II) For a temporary action that may not be actually
perfect with respect to a certain point of time. happening at the time of speaking but was happening
(d) Perfect Continuous Tense: The action is going on in the recent past and is still happening in recent future.
continuously over a long period of time and is yet to For example : I am reading Sidney Sheldon now-a-
be finished. days.
y
o
u
Grammar C-7

rs
m
(III) To express changing or developing situations. (V) To describe the past events when we think more of

a
For example : India is progressing day by day. their effect in the present than of the action itself.

h
(IV) For an action that is planned or arranged to take place

b
For example : I have cut my finger.

o
in the near future. (VI) For long actions and situations which started in the

o
b
For example : I am going to cinema tonight. near past and went on until very recently.

.w
Note: But it is not good to use the present continuous For example : I have read three chapters since this

o
for slightly distant future. So, don’t say

rd
morning.
(a) I am going to cinema next week.

p
6. The Present Perfect Continuous tense is used for an action,

re
Rather, use the future simple. So, it is better if you
which began at sometime in the past and is still continuing.

s
say

s
.c
(b) I will go to cinema next week. With the present perfect continuous tense an adverb or

o
(V) When the reference is to a particularly obstinate habit, phrase that expresses time is used.

m
the present continuous is used instead of present For example :
simple. An adverb like always, continually, constantly (a) I have been reading this book since morning.
is also used. (b) They have been building the bridge for several months.
For example : It is no use scolding him; he always
7. The Simple Past tense is used
does what is forbidden. (Incorrect)
Note: that his doing what is forbidden has become a (I) To indicate an action completed in the past. Generally,
die-hard habit. The habit persists in spite of advice or adverbs or adverb phrases of past time are used in the
warning. So, we should use the present continuous. past simple tense.
For example : It is no use scolding him; he is always doing For example:
what is forbidden. (Correct) (a) The steamer sailed yesterday.
5. The Present Perfect tense is used (b) He went home sometime back.
(I) To indicate the completed activities in the immediate (II) To express imaginary present situations or imaginary
past.
future events that may not happen.
For example : He has just gone out.
For example :
(II) Action completed in the immediate past or an action of
the past whose effect lingers in the present. (a) If I had longer holidays, I would be very happy.
For example : I wrote three books. ( Incorrect) (b) If I got rich, I would travel all over the world.
The given sentence appears to be incomplete. The (III) When this tense is used without an adverb of time,
reader of the sentence immediately queries. ‘When did then time may be either implied or indicated by the
you write three books?” It would be a different case if context.
you said For example : I didn’t sleep well. (i.e., last night)
For example : I wrote books. (IV) For past habits ‘used to’ is added to the verb.
Then the reader would infer that you wrote books in
For example : She carried an umbrella.
the past as a profession or hobby. But when you are
being so specific as to say “three books”, we immediately 8. The Past Continuous tense is used;
feel the need of a time frame. Since no time frame is (I) To denote an action going on at some time in the past.
mentioned, we assume it to be ‘by now’. So, we have The time of the action may or may not be indicated.
something to the effect. For example :
For example : I have written three books by now. (a) It was getting darker.
This ‘by now’ is implied and need not be written. So, (b) We were listening to the radio all evening.
For example : I have written three books. (Correct) (II) When a new action happened in the middle of a longer
(III) The present perfect is never used with adverbs of past action. In this case Past Simple and Past Continuous
time. In such cases the past simple should be used.
are used together. Past Simple is used for the new
For example : India has won the match last week.
(Incorrect) action.
“Last week” is not immediate past. You may therefore For example : The Light went out while I was reading.
be tempted to use the present perfect. But remember (III) For persistent habits in the past.
that the immediate past here does not go unindicated. For example : She was always chewing gum.
Last week is being used as an adverb of past time. So, 9. The Past Perfect tense is used when two actions happened
For example : India won the match last week. (Correct) in the past. In this case it is necessary to show which action
(IV) To express past actions whose time is not given and happened earlier than the other. Here past perfect is used
not definite - actions with their effect continuing in the for the action, which happened earlier.
present.
For example :
For example :
(a) I have never known him to be angry. (a) When I reached the station the train had started.
(b) Have you read ‘Gulliver’s Travels’? (b) I had done my exercise when Hari came to see me.
y
o
u
C-8 Grammar

rs
m
10. The past perfect continuous is used for an action that began ARTICLES

a
before a certain point in the past and continued up to that

h
1. ‘A’ or ‘an’ does not refer to a particular person or thing. It

b
time. A time expression like since last year, for the last few

o
leaves indefinite the person or thing spoken of.
days is generally put after perfect continuous tense.

o
For example : I saw a doctor. (means I saw any doctor)

b
For example : At that time he had been writing a novel for

.w
2. An is used before a word beginning with vowel sound
two months.

o
(please note that the word begins… with vowel sound and

rd
11. The simple future is used for an action that has still to take place. not necessarily a vowel itself).

p
For example : For example : an ass, an enemy, an inkstand, an orange, an

re
(a) I shall see him tomorrow.

s
umbrella, an hour.

s
(b) Tomorrow will be Sunday.

.c
3. An is placed before an abbreviation if the first letter of an

o
12. The Future Continuous tense abbreviation is F, H. L, M, N, R, S or X.

m
(I) Represents an action as going on at sometime in the For example :
future. (a) An MBA was required for the post.
For example : I shall be reading the paper then. (b) An SAO is an officer of high rank.
(II) Represents the future events that are planned. 4. A is used before a word beginning with a consonant sound.
For example : He will be meeting us next week. For example : a boy, a woman a horse, a one-rupee note, a
13. The Future Perfect tense is used to indicate the completion university, a European (both university and European begin
of an event by a certain future time. with a consonant sound of ‘yu’ ).
For example : I shall have written my exercise by that time. 5. A and an are used with words ‘few’ and ‘little’ if they refer to
14. The Future Perfect Continuous tense indicates an action a small number or a small amount. Words ‘few’ and ‘little’
represented; it being in progress over a period of time that without the articles means almost none.
will end in the future. Generally time period is mentioned For example:
along with it. (a) We have little time to spare. (means almost no time)
(b) We have a little time to spare. (means some time)
For example : By next July we shall have been living here
(c) Few persons were present at the meeting. (means
for four years.
almost no one was present)
15. Event occurring at the same time must be given in the same tense.
(d) A few persons were present at the meeting. (means
For example : When he fainted his brother was with him.
some were present)
16. Will or Shall cannot be used twice in the same sentence 6. A is used in the following senses :
even if both the actions refer to the future tense. (A) In its original numerical sense of one.
For example : For example:
(a) I shall come if he will call me. (Incorrect) a) Not a word was said.
(b) I shall come if he calls me. (Correct) b) A word to the wise is sufficient.
17. With the phrases as if and as though the past tense (B) In the vague sense of a certain time.
and plural form of the verb should be used. For example : One evening a beggar came to my door.
For example : (C) In the sense of any, to single out an individual as the
(a) He behaves as if he is a king. (Incorrect) representative of a class.
(b) He behaves as if he were a king. (Correct) For example : A pupil should obey his teacher.
18. With the word ‘wish’ four verbs are used namely were, had, (D) To make a common noun of a proper noun.
could, would. ‘Were’ is used when the wish seems to be For example : A Daniel came to judgement. (A Daniel =
unrealisable. A very wise man)
For example : I wish I were a king. 7. ‘The’ points out some particular person or thing or someone
‘Had’ is used when our wish is a lament over the past or something already referred to.
happening. For example :
For example : I wish I had accepted that job. ‘Would’ is (a) I saw the doctor. (means I saw some particular doctor)
used when we refer to the future. (b) The book you want is out of print.
For example : I wish I would get a ticket. 8. ‘The’ is used with names of gulfs, rivers, seas, oceans,
‘Could’ is used when we wish that something that has
groups of islands and mountain ranges.
happened already should have happened otherwise.
For example :
For example : He did not go because he was busy yesterday.
The Persian Gulf, The Red Sea, The Indian Ocean, The British
I wish he could go with you.
Isles, The Alps.
19. ‘For’ is used for a period of time.
9. ‘The’ is used before the name of certain books.
For example : He has been working for two hours.
For example : The Vedas, The Puranas, The Ramayana.
‘Since’ is used with a point of time.
For example : He has been working since morning. But we never say ‘The Valmiki’s Ramayana’. The is not
20. In case of conditional sentences ‘had’ and ‘would have’ are used. used when the name of a book is mentioned along with the
For example : If I had met him I would have invited him. author’s name. So, ‘Valmiki’s Ramayana’ is correct.
y
o
u
Grammar C-9

rs
m
10. The is used before the names of things unique of their kind. is as great as Newton and the second sentence means that

a
For example : the sun, the sky, the ocean, the sea. Mumbai is a great manufacturing City like Manchester.

h
21. No articles are used before a common noun used in its

b
11. The is used before a plural common noun if it refers to a

o
widest sense.

o
particular group among the class and not the whole class.

b
For example :

.w
For example : Drive away the cows from the field.
(a) The science has developed much in the past hundred

o
12. The is used before a proper noun only when it is qualified years. (Incorrect)

rd
by an adjective. (b) Science has developed much in the past hundred years.

p
re
For example : The great Rani of Jhansi, the immortal Kalidas. (Correct)

s
13. The is used before the superlatives. 22. No article is used before the noun following ‘Kind of’:

s
.c
For example : For example :

o
m
(a) Sachin is the best batsman in the world today. (a) What kind of a hobby is this? (Incorrect)
(b) The best person should win. (b) What kind of hobby is this? (Correct)
14. The noun if emphasis is laid on the use of such a noun. 23. No article is used before abstract nouns.
For example :
Here, noun can be proper or abstract noun
(a) Wisdom is the gift of heaven.
(a) the time for doing it.
(b) Honesty is the best policy.
(b) occasion to help the distressed. But consider the following examples where an article is used
15. The is used with ordinals. before an abstract noun.
For example : (a) The wisdom of Solomon is famous.
(a) He was the first student to finish his homework. (b) I cannot forget the kindness with which he treated me.
(b) The second chapter of the book is very interesting. Here the article is used before the abstract noun as the
16. The is used before an adjective when the noun is
abstract noun has been qualified by an adjective or adjectival
understood.
clause.
For example :
24. No article is used before languages, subject of arts and
(a) The poor are always with us. (Here poor mean poor
science.
people, which is understood.)
For example :
(b) The weak and the strong. (Here weak means weak
people and strong means strong people.) (a) We are studying English.
17. No article is used before a common noun when it refers to (b) Geometry is the toughest subject I ever studied.
all the members of the class. 25. No article is used before words such as school, college,
For example : church, bed, table, hospital, market, prison.
(a) Man is mortal. For example :
(b) Fish has high protein content. (a) I went to school till last year.
(c) What kind of flower is it? (b) I have never been to hospital.
18. The is used before a common noun to give it the meaning of But an article is used before these words when reference is
an abstract noun. made to a definite place or some particular purpose. For
For example : The devil in him begins its misdeeds now example: I am going to the school. (= means I am going to
and then. my child’s school for some particular purpose i.e., to deposit
19. No article is used before the names of materials such as the fee).
gold, stone, wine, iron, wheat, wood, cloth. 26. No article is used before the name of relations like father,
For example : mother, aunt, uncle.
(a) Gold is a precious metal. For example : Mother would like to see you.
(b) Wheat grows in Uttar Pardesh, Haryana and Madhya But if someone else’s mother is being talked about then the
Pardesh. should be used.
(c) Iron is a useful metal. For example : The mother would like to see you.
Note: But it is correct to say. 27. Article should not be used before positions that are held at
For example : An iron is a useful gadget. one time by one person only.
Because here we are not taking about material iron, but the For example :
object which is used to make clothes smooth. (a) Pranab Mukherjee is was elected the President of the
20. No article is used before proper nouns. country. (Incorrect)
For example : (b) Pranab Mukherjee is elected President of the country.
(a) Delhi is the capital of India. (Correct)
(b) Newton was a great philosopher. 28. Consider this sentence :
But consider the following examples where an article is used (a) I have a black and white cat.
before a proper noun. Here I mean that I have one cat that is partly black and
(a) This man is a second Newton. partly white.
(b) Mumbai is the Manchester of India. Now, consider this sentence
Here Newton and Manchester is not used as a proper noun For example : I have a black and a white cat.
but a common noun. The first sentence means that this man Here I mean that I have two cats one is black and the other
y
o
u
C-10 Grammar

rs
m
white. Hence the rule is that when two or more adjectives ADJECTIVES OF QUANTITY AND ITS RULES

a
qualify the same noun, the article is used before the first 6. Some is used in affirmative sentences to express quantity

h
b
adjective only. But when they qualify different nouns, the or degree.

o
article is used before each adjective separately. For example : I shall buy some bananas.

o
b
Consider one more example. Any is used in the negative or interrogative sentences to

.w
(a) The President and Chairman is absent. express quantity or degree.

o
rd
(b) The President and the Chairman are present. For example :

p
Sentence (a) means that only one person is acting as (a) I shall not buy any bananas.

re
president as well as chairman. Sentence (b) means that two (b) Have you bought any bananas?

s
s
different persons are acting as the President and the But some is an exception to the above rule. Some is used in

.c
interrogative sentences, which are commands or requests.

o
Chairman and both the persons are absent.

m
For example : Will you please lend me some money?
ADJECTIVES 7. Few is used for countable objects and little is used for non-
Adjectives are the words that describe the qualities of a noun or countable objects.
pronoun in a given sentence. 8. Little means not much. So use of the word little has a
negative meaning.
CONSIDER THE FOLLOWING For example :
1. Tell which sentence is correct: (a) There is little hope of his recovery.
(a) Flowers are plucked freshly. (b) He has little appreciation of hard work.
(b) Flowers are plucked fresh. A little means some though not much. So, use of a
Sentence (b) is correct as, adjective is correctly used with little has a positive meaning.
a verb when some quality of the subject rather than verb is For example :
to be expressed. Here, fresh describes the word Flowers (a (a) There is a little hope of his recovery.
noun) and not plucked (a verb). (b) He has a little appreciation of hard work.
The little means not much but all there is.
DEMONSTRATIVE ADJECTIVE AND ITS RULES For example :
2. This and that are used with the singular nouns and these (a) The little information he had was quite reliable.
and those are used with plural nouns (b) The little knowledge of management he possessed was
For example : not sufficient to stand him in good stead.
(a) This mango is sour. 9. Few means not many. So use of the word few has a negative
(b) These mangoes are sour. meaning.
(c) That boy is industrious For example : Few men are free from faults.
(d) Those boys are industrious. A few means some. So use of word a few has a positive
3. This and these indicate something near to the speaker while meaning.
that and those indicate something distant to the speaker. For example : A few men are free from faults.
For example : The few mean not many, but all there are.
(a) This girl sings. For example : The few remarks that he made were very
(b) These girls sing. good.
(c) That girl sings. 10. Only uncountable nouns follow much, little, some, enough,
(d) Those girls sing. sufficient and whole.
For example :
DISTRIBUTIVE ADJECTIVES AND ITS RULES (a) I ate some rice.
4. Each is used when reference is made to the individuals (b) There are not enough spoons.
forming any group. Each is also used when the number of
the group is limited and definite. INTERROGATIVE ADJECTIVE AND ITS RULES
For example : 11. What is used in the general sense and which is used in a
(a) Five members were seated on each chair. selective sense.
(b) I was in Shimla for five days and it rained each day. For example :
Every is used when reference is made to total group or (a) Which of you haven’t brought your book?
when the number is indefinite. (b) What manner of man is he?
For example :
(a) Every seat was taken. DEGREES OF COMPARISON OF ADJECTIVES AND ITS
(b) I go to the movies every week.
RULES
(c) Leap year falls in every fourth year.
12. The comparative form ending in ‘er’ is used when we are
5. Each, either, neither and every are always followed by the
comparing one quality in two persons.
singular noun.
For example : Anjali is wiser than Rahul.
For example :
But if we wish to compare two qualities in the same person
(a) Each boy must take his turn.
then the comparative form ending in ‘er’ is not used.
(b) Every word of it is false.
For example : Anjali is wiser than brave.
(c) Neither accusation is true.
y
o
u
Grammar C-11

rs
m
13. When two objects are compared with each other, the latter 20. When the comparative form is used to express selection

a
term of comparison must exclude the former. from two of the same kind or class, it is followed by ‘of’ and

h
For example :

b
preceded by ‘the’.

o
(a) Delhi is bigger than any other city in India.

o
For example :

b
If we say,

.w
(a) Ramesh is stronger of the two boys.
(b) Delhi is bigger than any city in India.

o
Then we are saying Delhi is bigger than Delhi, as any city in 21. When than or as is followed by the third person pronoun,

rd
India includes Delhi also. And this is obviously wrong. the verb is to be repeated.

p
re
14. In a comparison by means of a superlative the latter term For example : Ram is not as clever as his brother is.

s
should include the former. 22. When than or as is followed by first or second person

s
.c
For example : pronoun, the verb can be omitted.

o
(a) Delhi is the biggest of all cities in India.

m
For example : He is more intelligent than you.
(b) Of all men he is the strongest.
23. In comparing two things or classes of things the comparative
Kindly note the difference in this and previous rule.
15. Later and latest refer to time. should be used.
For example : For example :
(a) He is later than I expected. (a) Of two evils choose the lesser (not least).
(b) This is the latest news. (b) Which is the better (not best) of the two?
Latter and last refer to position. 24. A very common form of error is exemplified in the following
For example : sentence.
(a) The last player could not bat as he was injured.
(a) The population of London is greater than any town in
(b) The latter chapters are very interesting.
Latter is used when there are two only, last when there are India.
more than two. (b) The population of London is greater than that of any
For example : town in India.
(a) Of Manohar, Syam and Joshi, the latter is a driver. Sentence (b) is correct as the comparison is between the
(Incorrect) population of London and the population of any town in
(b) Of Manohor, Syam and Joshi, the last is a driver. India.
(Correct)
25. Double comparatives and superlatives should be avoided.
16. Elder and eldest are used only of persons (usually members
of the same family). For example :
For example : (a) Seldom had the little town seen a more costlier funeral.
(a) My elder sister is doing MBA from IIM Ahemdabad. (Incorrect)
(b) My eldest brother is getting married today. (b) Seldom had the little town seen a costlier funeral.
Older and oldest are used of both persons and things. (Correct)
For example : (c) Seldom had the little town seen a more costly funeral.
(a) This is the oldest building in the city.
(Correct)
(b) Anthony is the oldest boy in the class.
17. Further means more distant or advanced whereas farther 26. Preferable has the force of comparative and is followed
means additional. by to. Phrase ‘more preferable’ should not be used.
For example : For example :
(a) No one discussed the topic further. (a) Coffee is more preferable to tea. (Incorrect)
(b) Kolkata is farther from the equator than Colombo. (b) Coffee is preferable to tea. (Correct)
18. The comparative degree is generally followed by ‘than’, 27. Less refers to quantity whereas fewer refers to number.
but comparative adjectives ending in is or are followed by
For example :
the preposition ‘to’.
For example : (a) No fewer than fifty miners were killed in the explosion.
(a) Akshay is inferior to Aamir in intelligence. (b) We do not sell less than ten kg of tea.
(b) Aamir is superior to Akshay in intelligence. 28. Certain adjectives do not really need comparison because
(c) He is junior to me. their meaning is already superlative. Such words are–unique,
(d) Who was captain prior to Azhar ? Ideal, perfect, complete, universal, entire, extreme, chief,
19. Adjectives such as square, round, perfect, eternal, universal, full, square, round. Therefore phrases like most unique,
unique do not admit of different degrees. So they cannot be more round, fullest, chiefest, etc. are wrong.
compared. Thus strictly speaking we cannot say that a thing 29. If there is a gradual increase, it is generally expressed with
is more square more round or more perfect. But sometimes two comparatives and not with positives.
we do make exceptions to this rule. For example :
For example : This is the most perfect specimen I have (a) It grew hot and hot. (Incorrect)
seen. (b) It grew hotter and hotter. (Correct)
y
o
u
C-12 Grammar

rs
m
OTHER COMMON RULES persons, the verb agrees in person with the nearest one.

a
30. ‘Verbal’ means ‘of or pertaining to words’ whereas ‘oral’ For example :

h
(a) Either he or I am mistaken.

b
means ‘delivered by word of mouth or not written’. Hence

o
(b) Neither you nor he is to blame.

o
the opposite of written is oral, not verbal.

b
5. When words are joined to a singular subject by ‘with’,

.w
For example :
‘together with’, ‘in addition to’, ‘as well as’, then also

o
(a) His written statement differs in several important

rd
number of the verb remains singular.
respects from his oral (not verbal) statement.

p
For example : The Chief with all his men, was massacred.
(b) The boy was sent with an verbal message to the doctor.

re
6. Following examples exemplify the common mistakes

s
31. ‘Common’ means shared by all concerned. If a fact is a

s
committed:

.c
common Knowledge, it means the knowledge of the fact is
(a) His diet was abstemious, his prayers long and fervent.

o
shared by all. Everyone knows about it. ‘Mutual’ means in

m
(Incorrect as subjects are not in the same number.)
relation to each other. If you and I are mutual admirers, it
(b) His diet was abstemious, his prayers were long and
means I admire you and you admire me. We might also have
fervent. (Correct)
a common admirer who admires both of us.
(c) He never has and never will, take such strong
(a) We stopped smoking on the advice of a mutual friend.
(Incorrect) measures. (Incorrect)
(b) We stopped smoking on the advice of a common friend. (d) He never has taken, and never will take, such strong
(Correct) measures. (Correct)
It is apparent that there are two or more than two of us. (e) Ten new members have been enrolled and seven
Apart from us, there is a person (friend). Since he is a friend resigned. (Incorrect)
to all of us, this friend is being shared by all of us. So, he is (f) Ten new members have been enrolled and seven have
a common friend. Now, look at this sentence. resigned. (Correct)
For example : We stopped smoking on mutual advice. (g) Being a very hot day, I remained in my tent.
It means I advised, you not to smoke and you advised me (Incorrect as participle being is referring to none)
not to smoke. (h) It being a very hot day, I remained in my tent. (Correct)
(i) Sitting on the gate, a scorpion stung him.
OTHER COMMON ERRORS
(Incorrect as participle sitting is not
32. Other common errors. referring to any word)
(a) These kind of questions is often asked in the
(j) While he was sitting on the gate, a scorpion stung him.
examinations. (Incorrect)
(Correct)
(b) This kind of question is often asked in the examinations.
7. The verb lay (lay, laid, laid) is transitive and is always
(Correct)
followed by an object. The verb lie (lie, lay, lain) is intransitive
(c) He is as good if not better than his brother. (Incorrect)
(d) He is as good as if not better than his brother. and cannot have an object.
(Correct) For example :
(e) The future do not hold much for you. (a) Lay the child to sleep.
(Incorrect) (b) Let me lie here.
(f) The future does not hold much for you. (c) I laid the book on the table.
(Correct) AGREEMENT OF THE SUBJECT WITH THE VERB
VERB 1. A verb must agree with its subject in number and person.
Often due to “Error of Proximity”: the verb is made to agree
1. Two or more singular subjects connected by ‘and’ usually
take a verb in the plural. in number with a noun near it, instead of with its proper
For example : Hari and Rama are there. subject.
2. If two singular nouns refer to the same person or thing, the For example :
verb must be singular. (a) The quality of the mangoes were not good. (Incorrect
For example : My friend and benefactor has come. since subject is quality, a singular and not mangoes.)
3. If two subjects together express one idea, the verb may be (b) The quality of the mangoes was not good (Correct).
in the singular. (c) His knowledge of Indian vernaculars are far beyond
For example : The horse and the carriage is at the door. the common. (Incorrect)
4. Two or more singular subjects connected by ‘or’, ‘nor’, (d) His knowledge of Indian vernaculars is far beyond the
either... or, neither...nor take a verb in the singular.
common. (Correct)
For example : Neither he nor I was there.
2. Verb should be singular even when some words are joined
But when subjects joined by ‘or’, ‘nor’ are of different
numbers, the verb must be plural, and the plural subject to a singular subject by ‘with’, ‘as well as’ etc,
must be placed next to the verb. For example :
For example : Rama and his brothers have done this. (a) The chairman, with the directors, is to be present.
When the subjects joined by ‘or’, ‘nor’ are of different (b) Silver, as well as cotton, has fallen in prices.
y
o
u
Grammar C-13

rs
m
3. Two or more singular subjects connected by ‘or’, ‘nor’ 11. When a plural noun denotes some specific quantity or

a
require singular verb. amount considered as a whole, the verb is generally singular.

h
b
For example : For xample :

o
(a) One hundred paise is equal to one rupee.

o
(a) No nook or corner was left unexplored.

b
(b) Our happiness or our sorrow is largely due to our own (b) Six miles is a long distance.

.w
actions. (c) Fifty thousand rupees is a large sum.

o
rd
4. If two singular nouns express one idea, the verb is in the
ADVERBS

p
singular.

re
A word that modifies the meaning of a verb as called an Adverb.

s
For example :

s
.c
(a) Bread and Butter are essential for one’s life. SOME IMPORTANT RULES

o
(Incorrect) 1. Adverbs of manner such as well, fast, quickly, carefully,

m
(b) Bread and Butter is essential for one’s life. calmly etc. are placed after the verb if there is no object and
(Correct) after the object if there is one.
For example :
5. Either, neither, each, everyone, many a must be followed
(a) It is raining heavily.
by a singular verb. (b) She speaks English well.
For example : 2. Adverbs of time such as always, often sometimes, never,
(a) Neither of the two men was very strong. generally, ever merely, seldom etc., are placed before the
(b) Every one of the prisons is full. verb they qualify.
(c) Many a man has done so. For example :
(d) He asked whether either of the applicants was suitable. (a) I seldom meet him. (Correct)
6. When the subjects joined by ‘or’, ‘nor’ are of different (b) I meet him seldom. (Incorrect)
numbers, the verb must be plural, and the plural must be Adverbs of degree refer to words which show “how much”, “in
placed next to the verb. what degree” or “to what extent” does the action taken place.
For example : CONSIDER THE FOLLOWING :
(a) Neither Rekha nor her friends was present at the party. 3. Meaning of too is more than enough. Too denotes some
(Incorrect) kind of excess.
(b) Neither Rekha nor her friends were present at the party. For example :
(Correct) (a) He is too weak to walk.
7. When a plural noun denotes some specific quantity or (b) It is never too late.
amount considered as a whole, the verb is generally singular. Hence, use of very in place of too is wrong.
For example : For example : Instead of saying that:
(a) Five hours are too short a time to judge one’s character. (a) Cow’s milk is too nutritious.
We should say that
(Incorrect)
(b) Cow’s milk is very nutritious.
(b) Five hours is too short a time to judge one’s character. 4. Enough is placed after the word it qualifies.
(Correct) For example : Everyone should be strong enough to support
This is so because five hours is considered as one chunk. one’s family.
8. Two nouns qualified by each or every, even though It will be wrong if we write ‘Everyone should be enough
connected by ‘and’ require a singular verb. strong to support one’s family’.
For example : Every boy and every girl was given a packet of 5. Much is used with past participles.
sweets. For example :
9. ‘None’ though singular commonly takes a plural verb. (a) He was much disgusted with his life.
For example : None are so deaf as those who will not hear. (b) The news was much surprising.
Very is used with present participles.
10. Tell which sentence is correct.
For example :
(a) Put in to bat first, a huge total was expected from India. (a) It is very disgusting to ask him for a loan.
(b) Put in to bat first, India was expected to pile up a huge (b) The news is very surpising.
total. 6. When very and much are used to qualify superlative form
Now: who has been put in to bat first? A huge total of of adjectives/adverbs, they should be put before the word
India? Common sense tells us it must be India. But the ‘very’ and after the word ‘much’.
sentence (a), as it stands, appears otherwise. So, sentence For example :
(b) is correct. (a) Rim is the very best boy in his class.
(a) Being a rainy day, I decided to take my umbrella. (b) Rim is much the best boy in his class.
Adverbs of Affirmation or Negation refer to words that
(b) It being a rainy day, I decided to take my umbrella.
assert the action emphatically.
The sentence (a), as it stands, gives us the impression that
Consider these examples :
being a rainy day qualifies I. This is simply not true. I am (a) He certainly was a winner among them.
not a rainy day. So sentence (b) is correct. (b) Luckily he survived the crash.
y
o
u
C-14 Grammar

rs
CONSIDER THE FOLLOWING

m
5. With often denotes the instrument and by the agent. For

a
7. No sooner should always be followed by than. example :

h
For example :

b
(a) He killed two birds with one shot.

o
(a) No sooner I saw him I trembled with fear. (Incorrect) (b) He was stabbed by a lunatic with a dagger.

o
b
(b) No sooner did I see him than I trembled with fear. 6. Since is used before a noun or phrase denoting some point

.w
(Correct) of time and is preceded by a verb in the perfect tense.

o
8. Not should not be used with the words, which have

rd
For example :
negative meaning if we want the sentence to be negative.

p
(a) I have eaten nothing since yesterday.

re
For example : (b) He has been ill since Monday last.

s
s
(a) I received no letter neither from him nor from her. From is also used before a noun or phrase denoting some

.c
(Incorrect)

o
point of time but is used with non-perfect tense.

m
(b) I received letter neither from him nor from her. (Correct) For example :
9. Of course is used to denote a natural consequence. It should (a) I commenced work from 1st January.
not be used in place of certainly, undoubtedly. (b) He will join school from tomorrow.
For example : For is used with a period of time. For example :
(a) Of course he is the best player. (Incorrect) (a) He has been ill for five days.
(b) He is certainly the best player. (Correct)
(b) He lived in Bombay for five years.
7. Use of in before a period of time means at the end of period,
FOLLOWING ARE COMMON RULES OF ADVERBS IN
but use of within before a period of time means before the
GENERAL
end of period.
10. Only is used before the word it qualifies.
For example :
For example :
(a) I shall return in an hour. (means I shall return at the end
(a) Only I spoke to him.
(b) I only spoke to him. of an hour).
(c) I spoke to him only. (b) I shall return within an hour. (means I shall return before
11. Else is followed by but and not by than. the end of an hour).
For example : It is nothing else but hypocrisy. 8. Scarcely should be followed by ‘when’ and not by ‘but’.
12. ‘As’ is often used in a sentence though there is no need for For example : Scarcely had he gone, when (not than) a
it. For example : policeman knocked at the door.
(a) He is elected as the President. (Incorrect) 9. The phrase ‘seldom or ever’ is wrong ‘Seldom or never’ is right.
(b) He is elected President. (Correct) For example : Such goods are made for export, and are
13. ‘Perhaps’ means possibly whereas ‘probably’ means most seldom or never used in this country.
likely. For example : 10. Examine the following sentence:
(a) Where is Govinda? Perhaps he is not here. (Incorrect) (a) This is as good, if not better than that. (Incorrect)
(b) Where is Govinda? Probably he is not here. (Correct) (b) This is as good as, if not better than, that. (Correct)
(c) This is as good as that, if not better. (Correct)
PREPOSITION 11. Beside means at the side of while besides means in addition
1. In is used with the names or countries and large towns; at is to. For example :
used when speaking of small towns and villages. For (a) Beside the ungathered rice he lay.
example : (b) Besides being fined, he was sentenced to a term of
(a) I live in Delhi. imprisonment.
(b) I live at Rohini in Delhi. 12. Above and below merely denote position while over and
2. In and at are used in speaking of things at rest; to and into under also carry a sense of covering or movement.
are used in speaking of things in motion. For example : (a) The bird flew above the lake. (Incorrect)
(a) He is in bed. (b) The bird flew over the lake. (Correct)
(b) He is at the top of the class. Here over is used as besides denoting upward position,
(c) He ran to school. movement is also involved.
(d) He jumped into the river. 13. During is used when reference is made to the time within
(e) The snake crawled into its hole. which something happens. For is used when we are talking
3. On is often used in speaking of things at rest; and upon for about how long something lasts.
the things in motion. For example: (a) There are few incidents of irregularity for the
(a) He sat on a chair. emergency years. (Incorrect)
(b) The cat sprang upon the table. (b) There are few incidents of irregularity during the
4. Till is used of time and to is used for place. emergency years. (Correct)
For example : 14. Compare is followed by to when it shows that two things
(a) He slept till eight o’clock. are alike. It is followed by with when we look at the ways in
(b) He walked to the end of the street. which two things are like and unlike each other.
y
o
u
Grammar C-15

rs
m
For example : 5. Lest is used to express a negative purpose and is equivalent

a
(a) Sanath Jayasuria’s bowling may be compared to the to ‘in order that… not’, ‘for fear that’.

h
sales of a useful book, they score right from the

b
For example :

o
beginning. (Incorrect) (a) He lied lest he should be killed.

o
b
(b) Sanath Jayasuria’s batting may be compared with the (b) I was alarmed lest we should be wrecked.

.w
sales of a useful book; they score right from the 6. While is used to mean:

o
beginning. (Correct) (A) During that time, as long as.

rd
(c) If we compare Delhi University with the regional ones, For example : while there is life there is hope.

p
re
we find the former to be much more efficient. (Incorrect) (B) At the same time that.

s
(d) If we compare Delhi University to the regional ones,

s
For example : While he found fault, he also praised.

.c
we find the former to be much more efficient. (Correct)
7. Only means except that, but, were it not that.

o
m
CONJUNCTIONS For example :
(a) A very pretty woman, only she squints a little.
1. Since as conjunction means: (b) The day is pleasant, only rather cold.
(A) From and after the time when. 8. The conjunctions after, before, as soon as, until are not
For example : followed by clause in the future tense. Present simple or
(a) Many things have happened since I left the present perfect tense is used to express a future event.
school. For example :
(b) I have never seen him since that unfortunate (a) I will phone you after I arrive here.
event happened. (b) I will phone you after I have arrived here.
(B) Seeing that,
9. As if used in the sense of as it would be is generally followed
For example :
by a subject + were + complement.
(a) Since you wish it, it shall be done.
For example :
(b) Since that is the case, I shall excuse you.
2. Or is used (a) He loves you as if you were his own child.
(A) To introduce an alternative. (b) Sometimes she weeps and sometimes she laughs as if
For example : she were mad.
(a) You must work or starve. 10. The clause that begins with as if should be put into the
(b) You may take this book or that one. past simple tense, if the preceding clause expresses a past
(c) He may study law or medicine or engineering or action. But if it expresses a past action it should be followed
he may enter into trade. by the past perfect tense.
(B) To introduce an alternative name or synonym. For example :
For example : The violin or fiddle has become the (a) He behaves as if he were a lord.
leading instrument of the modern orchesta. (b) He behaved as if he had been a lord.
(C) To mean otherwise. 11. While as long as is used to express time in sense of how
For example : We must hasten or night will overtake long, until is used to express time in sense of before.
us. For example :
3. If is used to mean: (a) Until you work hard you will improve. (Incorrect)
(A) On the condition or supposition that. (b) As long as you work hard you will improve.(Correct)
For example : (c) He learnt little as long as he was 15 years old. (Incorrect)
(a) If he is here, I shall see him. (d) He learnt little until he was 15 years old. (Correct)
(b) If that is so, I am content. 12. No sooner should be followed by (verb + subject) and than
(B) Admitting that. should begin another clause.
For example : If I am blunt, I am at least honest. For example :
(C) Whether. (a) No sooner had I reached the station than the train left.
For example : I asked him if he would help me.
(b) No sooner did the bell ring than all the students rushed
(D) Whenever.
in.
For example : If I feel any doubt I enquire.
4. That is used: 13. When as well as is used, finite verb should agree in number
(A) To express a reason or cause. and person with the first subject.
For example : For example : He as well as us is innocent.
(a) Not that I loved Caesar less but that I loved Rome 14. As well as should never be used in place of and if the first
more. subject is preceded by the word ‘both’.
(b) He was annoyed that he was contradicted. For example :
(B) To express a purpose and is equivalent to in order (a) Both Rani as well as Kajol came. (Incorrect)
that. (b) Both Rani and Kajol came. (Correct)
For example : He kept quiet that the dispute might 15. Because is generally used when the reason is the most
cease. important part of a sentence.
(C) To express a consequence, result or effect. For example : Some people like him because he is honest
For example : He bled so profusely that he died. and hard working.
y
o
u
C-16 Grammar

rs
m
Since is used when the reason is already known or is less (a) Not only the students but also the teacher were

a
important than the chief statement. responsible for what happened in the class. (Incorrect)

h
For example : Since you refuse to cooperate, I shall have to (b) Not only the students but also the teacher was

b
o
take legal steps. responsible for what happened in the class. (Correct)

o
24. Such ... as is used to denote a category whereas such ...that

b
For is used when reason given is an afterthought.

.w
For example : The servant must have opened the box, for emphasises the degree of something by mentioning its
consequence.

o
no one else had the key. For never comes at the beginning

rd
For example :
of the sentence and for is always preceded by a comma.

p
(a) Each member of the alliance agrees to take such action

re
16. Scarcely should be followed by when and not by than, that it deems necessary. (Incorrect)

s
(a) Scarcely had he arrived than he had to leave again. (b) Each member of the alliance agrees to take such action

s
.c
(Incorrect) as it deems necessary. (Correct)

o
(b) Scarcely had he arrived when he had to leave again. Here “it seems necessary” is not a consequence of “such

m
(Correct) action”. The sentence wants to imply that the action
17. Conjunctions such as either... or, neither... nor, not only... belongs to the category “as it deems necessary”. In
but also, both... and, whether, or etc. always join two words other words, what kind of action? Such action as it
or phrases belonging to the same parts of speech. deems necessary.
For example : (a) She looked at him in such distress as he had to look
away. (Incorrect)
(a) Either he will ask me or you. (Incorrect)
(b) She looked at him in such distress that he had to look
(b) He will ask either me or you. (Correct) away. (Correct)
(c) Neither he reads nor writes English (Incorrect) Here, “he had to look away” is a consequence of “she looked
(d) He neither reads nor writes English. (Correct) at him in such distress.” In other words, the degree of the
(e) Either you shall have to go home or stay here. distress of looking at him was such that (not as) he had to
(Incorrect) look away.
(f) You shall have either to go home or stay here. (Correct)
18. Conjunctions like neither...nor, either..or, should be followed PHRASAL VERBS
by the same part of speech.
Phrasal Verbs are a particular kind of expression, wherein the
For example :
verb is made of two or more components. Mostly the combining
(a) He neither agreed to my proposal nor to his. (Incorrect) components are verbs and prepositions. When divided these
(b) He agreed neither to my proposal nor to his. (Correct) components will have a meaning of their own but would not
19. Conjunction is not used before an interrogative adverb or suggest anything about the meaning of the phrasal verb.
interrogative pronoun in the indirect narration. Consider the following sentences.
For example : (a) This sword has been handed down from father to son in the
(a) He asked me that where I stayed. (Incorrect) family for many generations.
(b) He asked me where I stayed. (Correct) (b) I have been looking forward to meeting you since long now.
20. Although goes with yet or a comma in the other clause. (c) The patient came out of the delirium only when given
For example : tranquilizers.
(a) Although Manohar is hardworking but he does not (d) We had almost decided to give up on the search when we
get a job. (Incorrect) made the discovery.
(b) Although Manohar is hard working, yet he does not Phrasal verbs are idiomatic expressions and have a particular meaning
get a job. (Correct) different from that of the combining verbs and prepositions.
21. Nothing else should be followed by but not by than, Following are some phrasal verbs with their meanings.
For example : (a) sit in : to attend or take part as a visitor
(a) Mr. Bureaucrat! This is nothing else than red-tapism. (b) sit out : to stay till the end of
(c) come round : to accept circumstances and adjust
(Incorrect)
yourself to them.
(b) Mr. Bureaucrat! This is nothing else but red-tapism. (d) get on : to manage one’s life
(Correct) (e) turn out : to have a particular result
22. The correlative conjunctions indeed... but are used to (f) turn up : to arrive unexpectedly
emphasise the contrast between the first and the second (g) show off : to brag or boast
parts of the statement. (h) sort out : to successfully deal with a problem
For example : (i) hand in : to give something to someone in authority
(a) I am indeed happy with my school but it produces (j) sit down : to take a seat
famous men. (Incorrect) (k) sit up : to rise from a supine position
(b) I am indeed happy with my school but it does not (l) give in : to yield to some pressure
produce famous men. (Correct) (m) come forth : to find something
(c) I am indeed happy with my school that it produces (n) switch on : to start something
famous men. (Correct) (o) turn down : to refuse or reject an offer
23. In a “not only ... but also...” sentence, the verb should agree (p) turn in : to expose
with the noun or pronoun mentioned second, that is; the (q) look in : probe, or investigate a matter
one after ‘but also’, because this is the part being (r) look after : take care of
emphasised. (s) take off : to remove something
For example : (t) put out : to end
y
o
u
Grammar C-17

rs
m
(u) try on : to wear some clothes for first time IV If the verb in the main sentence is an active verb without
(v) turn down : lower the volume, refuse to accept

a
any auxillary verb, then the verb used in the question tag

h
(w) turn on : to start a machine will be the form of verb ‘do’ that corresponds with the tense

b
o
(x) put in : to invest something (matter or abstract) in the main sentence.

o
(y) look out : be careful of some danger.

b
• He knows it’s true, doesn’t he?

.w
• You wanted to come with me, didn’t you?
Following are some sentences using Phrasal Verbs

o
• I told you so, didn’t I?

rd
• Don’t throw away your opportunity to enter this University.
• Many people believe in astrology and tarrot cards now-a- • She never informed us, did she?

p
V If the main sentence has an auxillary then it is used in the

re
days.

s
• Quickly get in the car, we’re getting late. question tag, but with opposite affirmation, i.e., a positive

s
.c
• You can put forward your point in today’s meeting. auxiliary in the main sentence transforms to a negative

o
• To sit through his speech was very difficult. auxiliary in the question tag and vice-versa e.g.,

m
• I don’t understand why you put up with his insolent • He will be coming, won’t he?
behaviour. • You were there at the party, weren’t you?
• I could see through his intentions the first time I met, but • You would appear for this exam, wouldn’t you?
kept quiet to give him a chance to reform. • He didn’t call us, did he?
• Please, fill in all the necessary information in this form. • She doesn’t live here anymore, does she?
• I am sure you will not let me down, I’ve full faith in your
capabilities. MODALS
• Why are you taking it out on me? I’m not the one
responsible for the mishap. The verbs like can, could, may, might, would, shall, should and
ought are called modal verbs or modals. They are used with
QUESTION TAGS ordinary verbs to express possibility, permission, certainly, etc.
(1) Can usually expresses ability or capacity
Consider the following examples:
I can swim across the river
(1) You wanted that, didn’t you? Can you lift this table?
(2) He is coming tonight, isn’t he? (2) Can is also used to express permission
(3) You wouldn’t report me, would you? You can go now.
Now, look at the last part of all the above sentences preceded (3) May is a more formal modal used to express permission:
by the comma. These are very small questions added to the You may come in.
sentence and are called question Tags. Remember only the May I leave the room now?
question tag is a question and not the entire sentence. So, (4) May is also used to suggest possibility in an affirmative
one can say that a Question Tag is an added brief question sentence.
to a statement. Usually a question tag consists of two He may be at home.
words—an auxillary verb in the positive or negative form It may rain tomorrow.
and a pronoun. (5) Can is used to suggest possibility in negative/interrogative
sentence.
How to form question tags?
Can this be true?
Three things are to be kept in mind while making a question
It cannot be so.
tag :
(6) May when used in a negative sentence suggests an
(a) The right auxiliary verb to be used in the question.
improbability whereas can suggests impossibility.
(b) The right pronouns to be used in the tag. He may not come today.
Both (a) and (b) should be in agreement with the verb and She cannot sing.
noun in the main statement. (7) Could and might are used as past forms of ‘can’ and ‘may’.
(c) Whether the verb in the question tag should be positive or I could swim across the river when I was young.
negative. I thought he might be at home.
Rules to form Question Tags (8) Might suggests less possibility or probability than may.
I. If the main statement is positive, the auxiliary verb will be I might go to Bangalore next week suggests the probability
negative and vice-versa e.g., of going is less than a sentence with ‘may’ will suggest.
• He saw that, didn’t he? (9) Could is used as a polite form of seeking permission or
• But he isn’t going to England, is he? making a request.
II. If there is a single subject/noun/pronoun in the main Could you pass me the plate ?
sentence, the corresponding pronoun/same pronoun will Could I please talk to Mr. Grover?
be used in the question tag. e.g., (10) Shall is used with first person and will in all the persons to
• You are coming with us, aren’t you? express future.
• Reena is leaving tonight, isn’t she? I shall need the money tomorrow.
III. If there are more than one noun/pronoun in the main When will you come next?
sentence then the corresponding pronoun to the active (11) Shall is used with the second and third person to express
subject will be used in the question tag. e.g., command, promise or threat.
• After all this time you’d think he’d have forgotten, You shall never come near my child.
wouldn’t you? You shall be punished for this.
• You would’nt refuse me, would you? We shall go for a picnic this Sunday.
y
o
u
C-18 Grammar

rs
Following are the rules for transformation of voice.

m
(12) Will You? indicates an invitation or request.

a
h
Will you dine with us tonight? Tense Active voice Passive Voice

b
o
Will you lend me your car for a week? Simple present take/takes is/am/are taken

o
(13) Should and would are used as past forms of shall and will.

b
Present is/am/are is/am/are being taken

.w
I expected that I would get a first class. continuous taking

o
She would sit for hours listening to the radio.

rd
Present perfect has/have taken has/have been taken

p
(14) Should is used to express duty or obligation.

re
We should obey the laws. Simple past took was/were taken

s
s
You should keep your premise.

.c
Past continuous was/were was/were being taken

o
(15) Should is used to express a supposition taking

m
If it should rain, they will not come. Past perfect had taken had been taken
(16) Should can also be used to express probability.
Simple future will/shall take will/shall be taken
He should be in the library.
(17) Must is used to express necessity. When changing a passive voice sentence to an Active voice
You must improve your spelling. sentence if the agent is absent in the given sentence you
(18) Must is also need to express obligation, and is a stronger can use any vague agents such as someone, they, people
word than should.
etc. e.g.,
We must follow the law.
My pen has been stolen. (Passive)
(19) Must is also used to express logical certainty.
Someone has stolen my pen. (Active)
Living alone in such a big city must be difficult.
I was asked my name. (Passive)
(20) Ought is used to express moral obligation and is stronger
They asked me my name. (Active)
than both should and must.
English is spoken all over the world. (Passive)
We ought to love our parents.
People speak English all over the world. (Active)
(21) Ought is also used to express probability sometime when
Exercise
the probability is very strong.
Change the voice in the following sentences.
The book ought to be very useful.
1. He was refused admittance.
VOICE TRANSFORMATION 2. All desire wealth and some acquire it.
Compare the following sentences 3. Why should I be suspected by you?
– Ram helps Hari. 4. He made his wife do the work.
– Hari is helped by Ram. 5. The public will learn with astonishment that war is imminent.
While both sentences express the same meaning, there is a 6. The people regarded him as an imposter and called him a
difference in their construction, the difference of voice. villain.
In sentence I, the subject Ram is the doer of the action and thus 7. Do not insult the weak.
it is in Active voice, the verb is in the Active Voice. 8. My watch was lost.
In sentence II, the subject is Hari to whom the action is done and 9. The legends tell us how the castle received its name.
thus the verb is in the Passive Voice.
10. We elected Babu captain.
Following are some examples of active and passive voice
Key to Voice Transformation
sentences
1. Admittance was refused to him.
Active Voice Passive Voice 2. Wealth is desired by all but acquired by some.
The peon opened the The gate was opened by 3. Why should you suspect me?
gate. the peon. 4. His wife was made to do the work by him.
5. It will be learnt by the public with astonishment that war is
Some boys were The old man was being
helping the old man. helped by some boys. imminent.
6. He was regarded as an imposter and called a villain by the
He will finish the work The work will be finished people.
in a fortnight. by him in a fortnight. 7. Let the weak not be insulted.
Why did your brother Why was such a letter 8. I lost my watch.
write such a letter? written by your brother ? 9. How the castle received its name is told to us by the leg-
ends.
He handed her a chair. A chair was handed to her
10. Babu was elected captain by us.
by him.
y
o
u
Grammar C-19

rs
m
POINTS TO REMEMBER

a
h
b
o
o
b
1. Abstract Noun: Abstract noun refers to quality, action or state of 12. Concrete Noun: Concrete noun is the opposite of abstract noun.

.w
a thing that can only be felt by us.
Concrete noun refers to a thing that can be identified or sensed by

o
For example: Laughter, greatness, faith, poverty, courage, kindness,

rd
our senses.
fear, bravery, childhood etc. Abstract noun is always uncountable

p
For example : House, Brick, Telephone, Rose.

re
and has no plural form.

s
2. Accusative Case: See Objective case. 13. Countable Noun: As the name suggests, a countable noun is one

s
.c
3. Active Voice: A verb is in the active voice when its form shows that can be counted.

o
that the person or thing denoted by the subject does something or, For example : ten girls, 25 rupees. Depending upon how the

m
in other words, is doer of the action. plural form of a countable noun is obtained, countable noun can be
4. Adjectives: A word that names an attribute, used to describe a categorised as Regular countable noun and Irregular countable noun.
noun or pronoun.
14. First Person: First person denotes the person or persons
For example:
speaking.
A small house.
A big mansion. First Person (Masculine or Feminine)
Adjective is used with the noun to describe or point out the person,
Case Singular Plural
animal, place or thing the noun names, or to tell the number or
quantity. Nominative I We
5. Adverbs: Adverb is a word that modifies the meaning of a verb, an Possessive My, mine Our, ours
adjective or another adverb.
Objective Me Us
For example :
(a) P. T. Usha runs fast. 15. Intransitive Verb: When a verb is so used in a sentence that its
(b) Govinda reads quite clearly. effect is limited to its subject or doer only, it is called intransitive.
6. Antecedent: Antecedent is a noun or noun-equivalent to which a For example : Compare these two sentences.
relative pronoun refers. (a) This boy is eating.
For example : The cloud that thunders does not rain. ‘Cloud’ is
(b) This boy is eating mango.
antecedent in the sentence.
In sentence (a) the effect of eating is limited to subject, boy only.
7. Apposition: When one noun follows another to describe it, the
noun which follows is said to be in apposition to the noun which But in sentence (b) the effect of eating passes from subject (boy)
comes before it. to an object (mango). If we get an answer to ‘who eats what?’
For example : In the sentence, Stephen Hawking, the scientist, has Mango, then it is transitive verb. Hence sentence (a) uses
written A Brief History of Time, the noun scientist is in apposition intransitive verb and sentence (b) falls into transitive verb category.
to the noun Stephen Hawking. 16. Irregular Countable Noun: Plural form of these countable nouns
8. Case: The use of different forms of a noun or pronoun to show is not obtained by adding ‘s’, ‘es’ or ‘ies’ after the word.
its relation to the remaining sentence is called case. Three different
For example : plural of person is people, tooth is teeth.
types of cases are Nominative case, Objective or Accusative case
17. Nominative Case: Here noun or pronoun is used as the subject
and Possessive or Genitive case.
9. Collective Noun: Collective noun refers to a group of similar of a verb. To find the nominative put who or what before the verb.
persons or things. Though collective noun refers to more than one 18. Noun: A noun is a word used as the name of a person, place, thing
thing, it is always singular in form. or idea. A noun can be a common noun or a proper noun, an
For example : Army, Family, Herd, and Committee. abstract noun or a concrete noun, a countable noun or non-countable
10. Common Noun. Common noun is a name that can be applied to noun and a collective noun.
all the members of a class. In other words it refers to all the
19. Object: Also called Predicate. The part which tells something
persons and things of the same kind. Like proper noun it does not
about the subject is called object.
refer to a particular—person or thing.
For example : man, woman, elephant, village, crowd, army, family, 20. Objective Case: Also called Accusative Case. Here noun or
nation. pronoun is used as the object of the verb. To find the objective
11. Complement: Complement of the verb is the word or words case put whom? or what? before the verb and its subject.
which are used to make the sense of, the sentence complete. 21. Passive Voice: A verb is in the passive form when its form shows
For example : that something is done to the person or thing denoted by the
(a) They made him. subject.
(b) They made him king.
22. Personal Pronoun: Personal pronoun refers to an individual or
Sentence (a) carries no complete sense or meaning. But when the
Individuals. Personal pronouns are of three different types—First
word king is added to it, the sentence becomes sensible. So, here
king is the complement. person, Second person and Third person.
y
o
u
C-20 Grammar

rs
m
23. Possessive Case: In this form of the noun, ownership or 32. Subject: The part which names the person or thing we are

a
possession is shown. Possessive case is also used to denote speaking about is called subject of the sentence.

h
b
ownerships, origin, kind etc. The possessive case: answers the 33. Third Person: Third person denotes the person or persons spoken

o
question ‘whose?’ of Third Person.

o
b
24. Predicate : Please see object.

.w
25. Preposition: A preposition is a word placed before a noun or Case Singular Plural

o
pronoun to show in what relation the person or thing denoted by Masculine Feminine Neuter All Genders

rd
it stands in regard to something else. Nominative He She It They

p
26. Pronoun: A pronoun is a word used instead of a noun. Pronouns

re
are classified as personal, relative, reflexive, demonstrative, Possessive His Her, hers Its They, their

s
s
indefinite, interrogative, reciprocal pronoun. Objective Him Her It Them

.c
27. Proper Noun: Unlike common noun, proper noun refers to a

o
particular member of class. Proper noun is the name of some 34. Transitive Verb: When an action/word or verb is so used in a

m
particular person or thing. Proper nouns are always written with sentence that its effect is not limited to its subject only but passes
a capital letter at the beginning. For example : names of all people, to another person or thing, it is called Transitive verb.
places. For example : A boy is eating a mango. For details kindly see
28. Regular Countable Noun: Plural form of these words is obtained
definition of Intransitive Verb.
by adding ‘s’, ‘es’ or converting ‘y’ to ‘ies’ after the word. For
35. Uncountable Noun: Unlike countable nouns it can not be counted.
example : plural form of book is books, city is cities.
For example : Water, Milk, Sand, News, information. But if an
29. Relative Pronoun: Relative pronoun refers or relates two clauses.
uncountable thing is placed in a thing that can be counted, then the
Relative pronoun refers to some noun which is called its
antecedent. uncountable noun can be counted.
For example : I met Hari who has just returned. For example : One bottle of milk. Uncountable nouns can never be
30. Second Person.: Second person denotes the person or persons plural, though some uncountable nouns may appear to be plurals.
spoken to. For example : News
Second Person(Masculine or Feminine) 36. Voice: Voice is that form of a verb which shows whether what is
denoted by the subject does something or has something done to it.
Case Singular Plural
For example :
Nominative You You
(a) Rama helps Hari.
Possessive Your, yours Your, yours
(b) Hari is helped by Rama.
Objective You You In sentence (a), the form of the verb denotes that the person
31. Sentence: Sentence is a group of words which makes complete denoted by the subject, Rama, does something. In sentence (b) the
sense. In a sentence we name some person or thing and say form of the verb shows that something is done to the person
something about that person or thing. denoted by the subject, Hari.
y
o
u
rs
m
a
CHAPTER

h
b
2

o
VOCABULARY

o
b
.w
o
rd
p
re
s
s
.c
o
Every student at some time or the other +faces the question ''How (d) Thesaurus-the viable alternative - If using a dictionary seems

m
do I increase my vocabulary? '' This is because, people who might to be too boring to be considered for any length of time,
otherwise be very fluent in spoken English do not really take care using a thesaurus may be a more interesting alternative. In
to use new words, since, for the purpose of communication, the theory, it is the reverse of a dictionary, and basically gives
current register of words is deemed enough. An unfortunate fallout the various synonyms and the types of usage of a word e.g.,
of this nonchalant complacency is that when these people actually as a verb, adjective, noun etc.. So it gives a lot of information
face questions examining their vocabulary and its extent, they are about each word. If the student can remember even some of
all at sea. An easy example will bear this out. Everyone knows that it, it will be a great advantage. The most commonly available
when we are asked to name the tip of a mountain or the highest thesaurus in the market is the Roget’s Thesaurus, usually
point of something, we use the word ‘peak’. But not everyone now used as a generic name by most publishers.
would know that words like zenith, apogee, crest and summit may (e) The Word List - The Word List is a comprehensive
be replacements for the same word. So the same meaning used in compendium of the words most commonly asked or used in
the form of another word might leave the student totally perplexed. the Management Examination question papers. Studying the
Therefore, it is important to start working on one’s vocabulary as Word List will also give the student a lot of information about
soon as possible for success in any competitive exam. the origin of various words, their roots etc. This is a
Given below are a few tips on the accepted methods and practices particularly useful method of studying because knowledge
used to improve one’s vocabulary: of etymology helps the student gauge the meaning and usage
(a) The practice of reading - This is, sadly, lacking in most of other words having the same roots, regardless of whether
aspirants. With the advent of Television, reading has almost the student has come across the word earlier or not.
become passe. Reading is important not just because it (f) Flip-Cards - We strongly advocate this technique which is
increases general knowledge. That it definitely does; in another tool to memorise words from the list. These are blank
addition it helps a student to get into the habit of reading. It
cards approximately double the size of your standard visiting
is also important to read a variety of subjects, because each
card. After isolation of the exceptionally difficult words from
subject has its own register of language and words are used
the Word List, the student should write approximately five
with differing connotations in each register. So, for success
words on one side of the card and the corresponding
in competitive exams, perusal of a few different sources of
meanings on the other. The advantages of using this type of
reading material is mandatory. The sources are:
tool are that (1) Cards are portable and the student can carry
1) General magazines e.g., India Today, Frontline, Outlook,
Reader’s Digest, Time, etc. a card around with him/her any where and glance at it anytime.
2) Business magazines e.g., Business Today, Business (2) The embarrassment factor which carrying a Word List
India, Business World, etc. around entails is absent here, and (3) At the time of the final
3) The Daily Newspaper e.g., The Times of India, The run-up to the examination, the student, instead of revising
Hindu, The Indian Express, etc. the whole Word List, can just go through the set of Flip
(b) Using a dictionary-the almost-extinct habit - Even though Cards that he or she has collected. The only problem is getting
the idea of using a dictionary does nothing to enthuse the started and, to mix metaphors, once that initial hurdle is
common student, every one owns a dictionary but treats it overcome the rest is smooth sailing!
like a sacred cow, not to be touched and defiled; of course, (g) Gauging meanings - This logically follows from the earlier
not that they are to be blamed too much for it; it is almost a method. It is advisable for the student to try and gauge the
habit now; but should be considered a necessary evil. A meaning of a word from the context of the sentence. This is
dictionary should be kept with the student while reading an extremely effective method and very frequently, it is
anything, so that an incomprehensible word can be looked possible to find out the meaning of a newly seen word just
up immediately. Prefer a standard dictionary which explains by reading the whole sentence and getting the meaning of
the different use of a same word. the sentence as a whole. e.g., in the sentence ‘All of us tried
(c) Self-help books - Quite a few self-help books claiming to our best to persuade him but he remained adamant’. Even if
improve Verbal Ability are to be found in the open market, a student does not know the meaning of the word adamant
and one or two are actually helpful in this regard. Students per se, it is still possible to gauge the meaning from the
are welcome to consult these books but are advised to do so context of the sentence i.e., unmoved, firm, intractable, etc.
after consulting discriminating people who have experience It should be kept in mind by the students that none of the
in this regard e.g., experienced English teachers or the English above methods are absolute in themselves. It is a combination
faculty, since they have better idea of the relative efficacy of of all these, or at least some of these, which will give one the
these books. best results.
y
o
u
C-22 Vocabulary

rs
m
ONE WORD - A SMALL COLLECTION Armistice - (or cease-fire or truce) An agreement to stop

a
fighting

h
Abdicate - Renounce a throne or high office Ascetic - One who avoids physical pleasures and

b
o
Abolish - Do away with comforts

o
Accelerate - Move faster

b
Astrology - The study of the movements and relative

.w
Accomplice - One associated with another especially in positions of celestial bodies interpreted as

o
wrong-doing having an influence human affairs and the

rd
Acoustics - Science of the production, transmission, natural world.

p
reception and effects of sound

re
Astronomy - Scientific study of heavenly bodies
Acrobat - One who performs gymnastic feats

s
Backlog - An accumulations of uncomplete work or

s
.c
Adolescence - The period of life from puberty to maturity matters needing to be dealt with

o
Actuary - One who calculates insurance and annuity Backwater - A part of a river out of the main stream, where

m
premium etc the water does not move
Adulterate - Make impure by the addition of inferior Barbarian - An uncivilised person
substance Barbecue - A metal flame on which meat etc., is cooked
Aggression - Unprovoked attack of one country or person over an open fire
by another Barometer - An instrument for measuring the air pressure
Amnesty - General pardon Beverage - A liquid for drinking
Abattoir - A building where animals are killed for meat Bibliography - A list of the books of a specific author or
(or slaughterhouse) publisher or on a specific subject.
Ad hoc - Created or done for a particular purpose as Biennial - Happening once every two years
necessary. Bigamy - The offence of marrying someone while
Aeronautics - The science of the operation and flight of already married to another person.
aircraft Biodata - Biographical details
Aesthete - A person with a highly developed sense of Black Hole - A region of space having a gravitational field
beauty aesthetics so intense that no matter or radiation can
escape.
Agnostic - One who believes that nothing can be known
Bleach - Make white or pale by means of chemicals or
about God
sunlight
Agoraphobia - Fear of open spaces, public places
Blue Blood - The quality of being a noble person by birth
Alibi - It is Latin for elsewhere. It is actually a plea
Blueprint - The word originated in the engineering
of having been elsewhere at the time of industry where it means the final stage of
commission of an act. But it is now used in paper design. So it may mean the final plan or
the sense of an excuse. Example: He offered layout. Example: The blueprint of the Five-
no alibi for his absence from duty. Year Plan is ready.
Alimony - Compensatory allowance given to wife after Bonsai - The art of growing a plant in a pot that is
divorce prevented from reaching its natural size
Allergic - Caused by or relating to allergy e.g., an Bon Vivant - One who likes good wine and food and
allergic reaction. cheerful companions, pleasure lover
Altruist - One who is habitually kind to others, selfless Bookworm - (or nerd) One who is too fond of reading and
concern for the well-being or others. study
Alumnus - A former male student of a school or college Bottleneck - It is a narrow passage, a place, stage or
Ambivalent - The word means simultaneous attraction condition that checks progress. Example: We
towards and repulsion from an object, person must remove all bottlenecks in the swift
or action. Example : The attitude of educated implementation of policies.
Indians to love-marriages is ambivalent Boulevard - A broad street having trees on each side
Anachronism - That which appears to be old fashioned and Bourgeois - Belonging to the middle class
does not belong to the present time Bric-a-brac - Small objects of little value kept for
Anarchy - Lawlessness and disorder caused by absence decoration
of control Bullion - Bars of gold or silver
Anecdote - A short interesting or amusing story Bust - A piece of sculpture showing a person’s head,
Anthology - A collection of poems or writings shoulders, and upper chest
Aphorism - (or maxim) A wise saying in a few words which Cabal - A small group of people who make secret
contains a general truth plans for political faction
Aphrodisiac - A food, drink, or other thing that stimulates Calligraphy - The art of decorative writing by hand
sexual desire Canine - Of a dog
Cannibal - One who eats human flesh
Apiary - A place where bees are kept
Cardiac - Connected with the heart
Apprentice - A person who works under someone to learn
Catch-22 - A situation from which one is prevented
that person’s skill
from escaping by something that is part of
Arboreal - Those who live in trees
the situation itself
y
o
u
Vocabulary C-23

rs
m
Celestial -Of the sky or heaven Debacle - A sudden complete failure

a
Cerebral -Connected with the brain Decanter - A container for holding alcoholic drinks,

h
Chalet -A wooden house with a steeply sloping roof especially wine

b
o
Charlatan -One who deceives others by falsely Defeatism - The practice of thinking in a way that shows

o
b
claiming to have a skill an expectation of being unsuccessful

.w
Celibacy - One who does not indulge in carnal Deficit - The amount by which something is less

o
pleasure than what is needed

rd
Clean sweep - A complete victory Déja vu - The feeling of remembering something that

p
in fact one is experiencing for the first time

re
Cloak-and-Dagger - Involving or characterised by mystery,
Depression - A lon g per iod of seriously reduced

s
intrigue or espinoage, e.g., a cloak-and-

s
business activity and high unemployment

.c
dagger operation.

o
Clot - A half-solid mass or lump formed from a Designate - Chosen for an office but not yet officially

m
liquid, especially blood placed in it
Disarmament - Reduction of weapons by a government
Clubfoot - A badly-shaped foot twisted out of position
Dissection - Cutting up the body of a plant or animal for
from birth
studying
Coagulate - Change from a liquid into a solid by chemical
Dividend - The money which is divided among
action
shareholders
Cold war - Severe political struggle between countries, Dormitory - A large room containing a number of beds
without actual fighting Down-and-out - One who is suffering from lack of money,
Colloquial - Suitable for ordinary, informal, or familiar work, etc, and is unable to change the
conversation situation
Colonnade - A row of pillars supporting a roof or arches Dragnet - A system of connected actions and
Coma - A prolon ged state of deep methods for catching criminals
unconsciousness, caused especially by Dregs - Sediment in a liquid that sinks to the bottom
severe injury or illness, e.g., she went into a and is thrown away
coma. Drudgery - Hard uninteresting work
Combustible - (or Inflammable) That can catch fire and Dutch - Of the Netherlands (Holland)
burn easily Eagle-eyed - Looking with very keen attention and
Comrade - A close companion who shares difficult noticing small details
work Eaves - The edges of a roof which come out beyond
Congenital - A disease or physical abnormality present the walls
from birth. Eddy - A circular movement of water, wind, dust, etc.
Connotation - The feeling or ideas that are suggested by Elastic - Able to spring back into shape after being
a word stretched
Consortium - A combination of several companies, banks, Electrocute - To kill by passing electricity through the
etc. for a common purpose body
Consul - A person appointed by a government to Embargo - An official order forbidding trade with
protect and help its citizens and its interests another country
in trade in a foreign city Empirical - Based on practical experience of the world
Contemporary - A person living at the same time as another we see and feel
Contretemps - A minor dispute or disagreement e.g., she Enigmatic - That which is mysterious and very hard to
had occasional contretemps with her staff. understand
Entomology - The scientific study of insects
Corinthian - Typical of the most richly decorated style
Epic - A long narrative poem
of ancient Roman buildings
Epicurean - Lover of physical/material
Corrigendum - Something to be corrected in a printed book
Ergonomics - The study of the conditions in which
Counterfeit - Made exactly like something real in order to people work most effectively with machines
deceive Estuary - The wide lower part or mouth of a river
Countervailing - Acting with equal force but opposite effect Evaporate - To change into steam and disappear
Couture - The design and manufacture of fashionable Evolution - Gradual development from simpler forms
clothes to a client’s specific requirements Excise - Tax on goods produced and used inside a
and measurements. country
Criminology - The scientific study of crime and criminals Expletive - An often meaningless word used for
Crossroads - A point at which an important decision must swearing
be taken which will have far-reaching Expressionism - A style of painting which expresses feelings
consequences rather than describing objects and
Cruise - A sea voyage for pleasure experiences
Cuisine - A style of cooking Extrovert - An outgoing, socially confident person
Daredevil - One who is prepared to take dangerous risks Facet - Any of the many flat sides of a cut jewel
D-day - A day on which an important operation is Faeces - The solid waste material passed from the
to begin bowels
y
o
u
C-24 Vocabulary

rs
m
Fallacy - A false idea or belief Hangar - A big building where aircraft are kept

a
Farce - A light humorous play full of silly between flights

h
happenings Harpoon - A spear with a long rope, used for hunting

b
o
Farrier - One who makes and fits shoes for horses large sea animals

o
b
Febrile - Of or caused by fever Haven - A place of calm and safety

.w
Felony - A serious crime such as murder or armed Headgear - A covering for the head

o
robbery Headstrong - Determined to do what one wants in spite

rd
Fiance - (feminine fiancée) The person one is going of all advice

p
re
to marry Heat-stroke - Fever and weakness caused by too much

s
Filament - A thin thread heat

s
.c
Flogging - Severe beating with a whip or stick Heirloom - A valuable object passed on for generations

o
Flora - All the plants of a particular place, country, Herbivore - A plant-eating animal

m
or period Hide - An animal’s skin, when removed, to be used,
Fluvial - Of, found in, or produced by rivers for leather
Foible - A small rather strange and stupid personal Hinterland - The inner part of a country
habit Histrionics - Behaviour resembling a theatrical performance
Foolscap - A large size of paper, especially writing Holocaust - Great destruction and the loss of many lives
paper Holster - A leather holder for a pistol
Foray - A sudden attack into enemy country Hooligan - A noisy rough person who causes trouble
Foreman - A skilled and experienced worker in charge Hothead - One who does things too quickly, without
of other workers thinking
Fourth Estate - Newspapers and their writers, considered Hub - The central part of a wheel
with regard to their political influence Hump - A lump on the back of a camel
Freckle - A small flat brown spot on the skin Ideology - A set of ideas on which a political or
Freight - Goods carried by ship, train, plane, etc. economic system is based
Frill - A decorative edge to a piece of material Idolatry - The worship of idols
Frontispiece - A picture or photograph at the beginning Illegible - Difficult or impossible to read
of a book Immortal - That which will never die
Fumigate - To clear of disease, bacteria etc. by means Implacable - Impossible to satisfy, change, or make less
of chemical smoke angry
Furrow - A long narrow track cut by a plough Improvident - One who does not save for the future
Galaxy - A large group of stars Incarnate - In physical form rather than in the form of a
Gastronomy - The practice of choosing cooking and spirit or idea
eating good food Incorporeal - Without a body or form
Gelatine - A clear substance used for making jellies Inedible - Not suitable for eating
Geocentric - Having the Earth as the central point Inflate - To fill with air or gas until swelled
Gigolo - A man who is paid to be a woman’s lover Ingest - To take into the stomach
Glacier - A mass of ice moving very slowly down a Innate - That which one is born with
mountain valley Inseminate - To put male seed into a female
Glut - An excessively abundant supply of Intelligentsia - Those who are highly educated and often
something concern themselves with ideas and new
Goatee - A little pointed beard on the bottom of the developments
chin Intestate - Not having made a will
Go-Getter - One who is forceful, determined, and likely Invective - A forceful attacking speech used for
to succeed in getting what one wants blaming someone
Good Samaritan - One who helps others in trouble, without Invoice - A list of goods supplied, stating quantity
thinking of oneself and price
Gorge - A deep narrow valley with steep sides Irreproachable - So good that no blame at all could be given
Graffiti - Drawings or writing on a wall Journal - A serious magazine produced by a specialist
Grange - A large country house with Farm buildings society
Green Belt - A stretch of land, around a town or city, Junta - A council or assembly that deliberates in
where building is not allowed, so that fields, secret upon the affairs of government.
woods, etc. remain intact
Juxtapose - To place side by side or close together
Grunt - Short deep rough sound of a pig
Kimono - A long loose garment made of silk
Gubernatorial - Of a governor
Knuckle - The joint between the finger and the hand
Guinea pig - A person who is subject of some kind of
Lackey - One who behaves like a servant by always
test
obeying
Halitosis - A condition in which one has bad breath
Lead Time - The time taken in planning and producing a
Handbook - A short book giving all the most important
new product
information about a subject
y
o
u
Vocabulary C-25

rs
m
Lecher - One who continually looks for sexual Miniature - A very small painting

a
pleasure Mirage - The appearing of objects which are not

h
Leonine - Of or like a lion really there

b
o
Levee - An embankment beside a river or stream or Misnomer - A name wrongly or mistakenly applied

o
b
an arm of the sea, to prevent floods Moccasin - A simple shoe made of leather

.w
Levy - An official demand and collection, Modus Operandi - A method of doing something typical of

o
especially of a tax someone

rd
Libertarian - One who believes that people should have Mogul - A person of very great power, wealth and

p
re
freedom of expression importance

s
Lien - A legal claim or hold on employment or Monarchy - Rule by a king or queen

s
.c
property, as security for a debt or charge Monomaniac - One who keeps thinking of one particular

o
Limerick - A humorous short poem with five lines idea only

m
Linchpin - An important member which keeps the Moralistic - Having unchanging narrow ideas about
whole group together right and wrong
Literati - People with great knowledge of literature Morbid - Having or expressing a strong interest in
Livery - Uniform of a special type for servants sad or unpleasant things
Locale - A place where something particular Motto - A few words taken as the guiding principle
happens Multinational - A company having operations in many
Logger - One whose job is to cut down trees different countries
Loom - A machine on which thread is woven into Mundane - Dull / Ordinary
cloth Mycology - The scientific study of fungi (plural of
Lore - Old beliefs, not written down, about a fungus)
particular subject Namesake - A person with the same name as yours is
Lowbrow - One who has no interest in literature, art, your namesake
etc. Nautical - Of sailors, ships, or sailing
Lullaby - A pleasant song used for causing children Necromancy - The practice which claims to learn about
to sleep the future by talking with the dead
Machete - A knife with a broad heavy blade Nemesis - Just and unavoidable punishment
Magnum Opus - A great work of art, theatre, film, etc. Newfangled - New (idea. machine etc.) but neither
Malady - That which is wrong with a system necessary nor better
Malaise - A feeling of pain without any particular pain Nihilism - The belief that nothing has meaning or
or appearance of disease value
Malcontent - One who is dissatisfied with the existing Nodding - Giving consent
state of affairs Acquaintance - A very slight familiarity
Male Chauvinist - A man who believes that men are better than Nosegay - A small bunch of flowers, to be carried or
women worn on a dress
Malign - To speak evil of, especially to do so falsely Notary - A public official who makes written
and severely statements official
Mane - The long hair on the back of a horse’s neck No-win Situation - That which will end badly whichever choice
Manual - A book giving information about how to do one makes
something Nursery - A place where small children are taken care
Market Forces - The free operation of business and trade of or where young plants are grown for sale
without govt. controls Oar - A long pole used for rowing a boat
Mascot - Chosen as a symbol or thought to bring Obstetrics - The branch of medicine concerned with
good luck childbirth
Massacre - The unnecessary and indiscriminate killing Obtrude - To be pushed or to push oneself into
of human beings undue prominence
Materialism - Too great interest in money and material, Obtrusive - Tending to be pushed or to push oneself
etc, rather than spiritual matters into undue prominence
Mechanics - The science of the action of forces on Obviate - To clear away or provide for, as an objection
objects or difficulty
Megalomania - The belief that one is more important or Odoriferous - Having a smell
powerful than one really is Off-White - White with some grey or yellow
Mercantile - Of trade and business Oligarchy - A collective government formed by a few
Meteorology - The scientific study of weather conditions persons
Midriff - The part of the human body between the One-Upmanship - The art of getting an advantage over others
chest and the waist without actually cheating
Militia - Those trained as soldiers but not belonging Ontology - The branch of philosophy concerned with
to a regular army the nature of existence
y
o
u
C-26 Vocabulary

rs
m
Operational - The study of how bussinesses are organised Portend - To indicate as being about to happen,

a
Research in order to make them more efficient especially by previous signs

h
Opprobrium - The state of being scornfully reproached Post-Haste - In a great hurry

b
o
or censured Pot-Boiler - A book of low quality produced quickly to

o
b
Orderly - A soldier who attends an officer make money

.w
Ornithology - The scientific study of birds Powder Keg - Something dangerous that might explode

o
Ostentation - A display dictated by vanity and intended Précis - A shortened form of a piece of writing

rd
to invite applause or flattery Prescient - Able to imagine or guess what will probably

p
re
Ostracism - The state of not being included in a group happen

s
Outcast - One who is forced from one’s home or Prevaricate - To use ambiguous or evasive language for

s
.c
without friends the purpose of deceiving or diverting

o
Overhaul - Thorough examination and repair if attention

m
necessary Prey - An animal that is hunted and eaten by
Pacemaker - A small machine that regularises heartbeats another
Palaeography - The study of ancient writing systems Prima Donna - The main woman singer in an opera
Panacea - A remedy or medicine proposed for or company
professing to cure all diseases Prodigal - One who is wasteful or extravagant,
Panache - Being able to do things in a confident and especially in the use of money or property
elegant way Profile - A side view of someone’s head /face
Panegyric - A speech or a piece of writing praising Projection - Something that sticks out from a surface
somebody or something Propellant - An explosive for firing a bullet or a rocket
Pariah - One who is not accepted by society Protagonist - First actor in a play. It means one who takes
Parricide - Act of murdering one’s father, mother or the leading part in a drama, novel or any
other close relative other sphere
Parting Shot - A last remark made at the moment of leaving Proscribe - To reject, as a teaching or a practice, with
Passive Smoking - The breathing in of smoke from the condemnation or denunciation
cigarettes that others are smoking Prosody - The rules by which the patterns of sounds
Patent - The right to make or sell a new invention and rhythms are arranged in poetry
Paunch - A man’s fat stomach Postscript / P.S. - A note added at the end of a letter
Peanuts - Too small a sum of money Pulmonary - Of or having an effect on the lungs
Peeping Tom - One who secretly looks at others when they Punctilious - Strictly observant of the rules or forms
are undressing prescribed by law or custom
Penance - Making oneself willingly suffer for one’s Punter - One who makes a bet on horserace results
wrongs Pus - A thick yellowish liquid produced in an
Perdition - Everlasting punishment after death infected wound
Perjury - A lie told on purpose in court Putsch - A sudden secretly planned attempt to
Persona on Grata - One who is not acceptable or welcome remove a government by force
Petrology - The scientific study of rocks Palmistry - The art of telling one’s character or future
Phonetics - The study and science of speech sounds by examining one’s hands and palms
Phylum - A main division of animals or plants Quartet - Four singers or musicians performing
Pigment - The natural colouring matter of plants and together
animals Quixotic - Trying to do the impossible, often so as to
Pillion - A seat for a second person on a motorcycle help others, while getting oneself into
Pithead - The entrance to a coalmine danger
Placate - To bring from a state of angry or hostile Raconteur - One who is good at telling stories in an
feeling to one of patience or friendliness interesting way
Plaintiff - One who brings a charge against someone Raillery - Friendly joking at someone’s weakness
in a court Ranger - The keeper of a forest
Platitude - A written or spoken statement that has Real Estate - Property in the form of land and houses
been made often befor e and is not Realpolitik - Politics based on practical facts rather than
interesting on moral or ideological aims
Platonic - A friendly intimate and affectionate Rebuff - A peremptory or unexpected rejection of
relationship but not sexual advances or approaches
Plebeian - Of the lower social classes Recant - To withdraw formallyone’s belief (in something
Poetaster - A writer of inferior quality poems previously believed or maintained)
Poker Face - A face that shows nothing of what one is Recumbent - Lying down on the back or side
thinking or feeling Red-Handed - In the act of doing something wrong
Porcine - Of or like a pig Redundant - Means exceeding what is natural, usual or
Pork - Meat from pigs necessary
y
o
u
Vocabulary C-27

rs
m
Reflation - A govt. policy of increasing the amount of Shaman - A priest believed to have magical powers

a
money used to increase the demand for and able to cure people

h
goods or services Shibboleth - A once-important custom which no longer

b
o
Relic - Something old that reminds us of the past has much meaning

o
b
Renaissance - A renewal of interest in some particular kind Shoot - A new growth from a plant

.w
of art, literature, etc, a period of revival Short-change - To give back less than what actually should

o
during 15th and 16th centuries in Europe be given back

rd
Renal - Of the kidneys Siamese twins - Those joined together from birth at some

p
re
Requisition - An official demand or request part of their bodies

s
Rescind - To make void, as an act, by the enacting Side Effect - An unwanted effect happening in addition

s
.c
authority or by a superior authority to the intended one

o
Resonance - Sound produced in one object by sound Signatory - Any of those who sign an agreement

m
waves from another Sill - The flat piece at the base of a window
Retribution - Punishment inflicted on someone as Singsong - A repeated rising and falling of the voice in
vengeance for a wrong act speaking
Revisionism - The questioning of the main beliefs of an Skyscraper - A very tall modern city building
already existing political system Sleeping Partner- A partner in a business who takes no active
Rhyme - To end with the same sound, including a part
vowel Slip-up - A slight unintentional mistake
Right-hand Man - One’s most useful and valuable helper Small Fry - A young or unimportant person
Ringleader - One who leads others to do wrong or make Smokestack - The tall chimney of a factory or a ship
trouble
Snippet - A short piece from something spoken or
Riviera - A warm stretch of coast on the
written
Mediterraneon sea popular with holiday
Socialite - A person well known for going to
makers
fashionable parties
Rodent - A small herbivore with strong sharp long
Sociology - The scientific study of societies and human
front teeth
behaviour in groups
Rolling Stone - A person who is unwilling to settle for long
Solidarity - Loyal agreement of interests, aims, or
in one place
principles among a group
Rosary - A string of beads used for counting prayers
Somnambulism - The habit of sleep walking
Roving eye - Sexual interests that pass quickly from one
SOS - An urgent message from someone in trouble
person to another
Souvenir - An object kept as a reminder of something
Rubber Stamp - One who acts only to make official the
Spatial - Connected with space
decisions already made by another
Spectacle - A grand public show or scene
Ruling - An official decision of a court
Rung - Any of the cross-bars that form the steps Spindle - A machine part round which something
of a ladder turns
Saboteur - One who practices sabotage Splinter - A small sharp-pointed piece of wood broken
Salve - (or Ointment) An oily substance for putting off something
on a cut, wound, etc. Sportsmanship - A spirit of honest fair play
Sapient - Wise and full of deep knowledge Sprig - A small end of a stem or branch with leaves
Scaffolding - A structure built from poles and boards for Standard-bearer - An important leader in a moral argument or
workmen to stand on movement
Scalp - The skin on the top of the human head Statesman - A political leader who is respected as being
Sceptical - Unwilling to believe a claim or promise wise, honourable, and fair-minded
Scraps - Pieces of food not eaten at a meal and Stellar - Of the stars
thrown away Sticking Point - Something that prevents an agreement
Scuba - An instrument used for breathing while Stock Broker - One whose job is buying and selling shares
swimming underwater and debentures for others
Seam - A line of stitches joining two pieces of cloth, Stoic - One who is indifferent to joys/sorrows
leather, etc. Stooge - One who habitually does what another
Sedentary - Anything done while sitting down person wants
Seer - One who can see into the future Stratagem - A trick to deceive an enemy
Seismic - Of or caused by earthquakes Strategist - A person skilled in planning, especially of
Seller’s Market - Where there are not many goods for sale military movements
Sensationalism - The intentional producing of excitement or Stride - A long step in walking
shock Strobe Light - A light which goes on and off very quickly
Septic - Infected with disease bacteria Subcutaneous - Beneath the skin
Sexagenarian - One who is between 60 and 69 years old Sub Judice - A legal case being considered in court
Sexism - The belief that one sex is not as good as the Subsidy - Money paid by the government to reduce
other prices
y
o
u
C-28 Vocabulary

rs
m
Superannuated - Too old for work Turf - A surface made up of earth and a thick

a
Surety - One who takes responsibility for another’s covering of grass

h
b
performance of an undertaking Tutelage - The act of training or the state of being

o
Surreal - Having a strange dreamlike unreal quality under instruction

o
b
Swarm - A large group of insects moving in a mass Tyrant - A ruler with complete power, who rules

.w
Sweet Tooth - A liking for sweet and sugary things cruelly and unjustly

o
rd
Sword of - Something bad that may happen at any time Tyro - One slightly skilled in or acquainted with

p
Damocles any trade or profession

re
Underling - A person of low rank in relation to another

s
Tactile - Of the sense of touch

s
Undermanned - Not having enough workers

.c
Take-home Pay - Wages left after all taxes, deductions, etc,

o
Unguent - A thick oily substance used on the skin to
have been made

m
heal it
Tannery - A Place where animal skin is made into
Unisex - Of one type used by both male and female
leather
Upholstery - A comfortable covering and filling for a seat
Tarot - A set of 22 cards used for telling the future
Valise - A small bag used while travelling
Tautology - The saying of same thing twice over in Vertebrate - A living creature which has a backbone
different words. Example: audible to the ear, Vicissitude - A change, especially a complete change,
return back, One after another in succession, of condition or circumstances, as of fortune
etc. Vinous - Of or pertaining to wine e.g., a vinous smell
Taxonomy - The system of putting plants and animals Voluntary - Done willingly, without being forced
into various classes Wade - To walk through water
Technocrat - A highly skilled specialist in charge of an Walkout - Leaving a meeting as an expression of
organisation disapproval
Teller - One who is employed to receive and pay Wardrobe - A large cupboard in which one hangs up
out money in a bank clothes
Tenure - The act, right, or period of holding land or a Wasteland - Empty, unproductive, usually barren land
job Waterloo - A severe defeat after a time of unusual
Territorial waters - The sea near a country’s coast over which success
it has legal control Weakling - One who lacks physical strength or
Testamentary - Of or done according to a will strength of character
Thatch - Roof covering of straw, reeds, etc. Wean - To transfer (the young) from dependence
Thermal - Of heat on mother ’s milk to another form of
nourishment
Thorax - The part between the neck and the
Weather-beaten - Marked or damaged by the force of wind.
abdomen
sun, rain. etc.
Thrombosis - Having a clot in a blood vessel or the heart
Wheeler-dealer - One who is skilled at making profitable or
Topiary - The art of cutting trees and bushes into successful deals
decorative shapes of animals and birds Whirlpool - A place with circular currents of water, which
Touchstone - Something used as a test or standard can pull objects down into it
Tract - A short piece dealing with a religious or Wholesale - The business of selling goods to shopkeepers
moral subject Wit - The ability to say clever and amusing things
Traitor - One who is disloyal to one’s country Wizard - One who has magic powers
Transient - Lasting only for a short time Word Blindness - (or dyslexia) Difficulty in seeing the
Transmogrify - To change completely as if by magic difference between letter shapes
Transpire - To happen or occur or become known. Workaholic - One who likes to work too hard
Example: It transpired at the meeting that Working - Enough practical knowledge to do
he was going to be our next President knowledge something
Treatise - A serious book or article that examines a Wreckage - The broken parts of a destroyed thing
particular subject Wretch - An unfortunate or unhappy person
Tribunal - A court of people officially appointed to Xenophobia - Fear of strange or foreign people, customs,
deal with special matters etc.
Troglodyte - One who lives in a cave Yeoman service - Great and loyal service, help, or support
Yuppie - A young person in a professional job with
Trousseau - The personal possessions that a woman
a high-income
brings with her when she marries
Zeitgeist - The intellectual and moral tendencies that
Tunnel Vision - A condition in which one can see only
characterize any age or epoch
straight ahead Zoology - The scientific study of animals
y
o
u
Vocabulary C-29

rs
Cancel : Annual, withdraw, revoke, delete

m
WORD LIST

a
Candid : Sincere, straightforward, frank

h
Given below is a list of words placed in alphabetical order. Each Captive : Prisoner, confined, jailed, bonded

b
o
word is followed by a few of its synonyms. Note these words Cause : Make, originate, induce, generate,

o
whenever you come across them. You should be familiar with

b
create

.w
most of the words for which synonyms are given if you have done Censor : Cut off, prohibit, ban

o
all the exercises till this point thoroughly. So, this list will be Censure : Blame, condemn, reprove, reprimand

rd
giving you synonyms for the words which you know. Thus
Character : Personality, emblem, type, nature,

p
learning will be easier.

re
disposition, quality

s
A

s
Charity : Philanthropy, benevolence

.c
Chaste : Pure, immaculate, virgin, refined

o
bandon : Leave, desert, forsake

m
Chatter : Babble, ramble, talk, discourse
Abase : Degrade, disgrace, humiliate Cheat : Defraud, gull, outwit, dupe
Abhor : Hate, loathe, detest Cite : Quote, mention, name, adduce
Abridge : Shorten, abbreviate Clothes : Apparel, attire, dress, garb
Absolute : Unalterable, unrestricted, unconditional Colossal : Huge, gigantic, enormous, big
Absurd : Ridiculous, silly, foolish Commence : Begin, start
Abundant : Ample, plentiful
Commensurate : Equivalent, suitable, applicable,
Accessory : Additional, auxiliary, subsidiary
proportionate
Adept : Proficient, skilled, expert
Adherent : Follower, stickler Conceal : Hide, cover, shelter, disguise
Adhesive : Sticky, glue, gum Confess : Admit, acknowledge, reveal, agree
Admire : Praise, adore, esteem Confuse or
Adore : Respect, idolise, worship, admire confound : Mix, perplex, astonish, amaze, bewilder
Adversity : Misery, misfortune Consequent : Following, resultant, outcome
Affliction : Distress, sorrow, sadness Conspiracy : Plot, intrigue, treason
Alien : Foreign, stranger, unknown Convict : Felon, culprit, criminal, guilty
Alive : Lively, vivacious, living Cowardly : Craven, dastardly, fearful, poltroon
Alleviate : Relieve, lighten, ease Coy : Modest, shy, reserved
Alms : Gratuity, donation, grant Crafty : Artful, adroit, dexetrous, cunning,
Amend : Improve, change, emend deceitful
Amicable : Suitable, friendly, lovable, amiable Crazy : Mad, insane, silly
Anxiety : Eagerness, misgiving, worry
Credence : Belief, faith, trust, confidence
Apathy : Indifference, neutrality
Crisis : Turning point, emergency, decisive
Appalling : Terrific, terrifying, dreadful, horrible
Apposite : Apt, suitable, well chosen moment
Appraise : Evaluate, estimate Criterion : Test, touchstone, standard, yardstick
Apprehend : Seize, fear, arrest Criticism : Analysis, review, stricture
Arbitrary : Despotic, wayward Cruel : Brutal, unmerciful, beastly, savage
Assent : Agree, consent, acquiesce Cynical : Captious, incredulous, sarcastic,
Astonish : Astound, surprise, amaze, bewilder morose

D
Audacious : Bold, courageous, daring
Aversion : Dislike, detestation, hostility, hatred anger : Hazard, risk, peril

B ase : Mean, low, ignoble Dash


Dastardly
:
:
Run, rush, fly
Cowardly, invaliant, afraid, fearful
Beg : Implore, ask, beseech, solicit Dawn : Daybreak, appear, (sunrise), begin
Behaviour : Conduct, deportment, way, demeanour Deadly : Fatal, lethal, destructive
Brave : Courageous, intrepid, bold, daring, Dearth : Scarcity, lack, want
valiant Debase : Degrade, defame, disparage,
Brisk : Active, fast, quick, busy, alert humiliate
Brittle : Frail, fragile Decay : Decompose, rot, decline in power,
Brutal : Animal, savage, beastly, cruel wealth, waste, wither, fade
Burglar : Thief, bandit, brigand, stealer Decease : Death, demise, end
Bystander : Spectator, onlooker, beholder Deceit : Fraud, cheating, forgery

C
Decipher : Translate, interpret, solve, explain
alculate : Estimate, count, reckon, compute Decorum : Decency, etiquette, propriety, gravity
Decree : Law, edict, ordinance, mandate, judgement
Callous : Hard, indifferent, cold-blooded Defamation : Calumny, disparagement, debasement
Calm : Cool, confident, quiet, serene, tranquil Defection : Abandonment, desertion
y
o
u
C-30 Vocabulary

rs
m
Defer : Postpone, delay Distorted : Blurred, maligned, changed, disguised,

a
Deference : Respect, reverence, honour deformed, misrepresented

h
b
Deformity : Disfigurement, malformation, ugliness Distress : Affliction, depression, misery

o
Dejected : Depressed, distressed, downhearted, Divert : Turn aside, deflect, deviate

o
b
downcast Divine : Heavenly, metaphysical, godlike

.w
Delectable : Charming, delightful, pleasant Divulge : Reveal, uncover, disclose

o
rd
Delegate : Commission, depute, authorise Docile : Amenable, tractable, submissive

p
Deliberate : Knowingly done, intentional, forcible Doctrine : Precept, principle, teaching

re
Delicacy : Softness, nicety, slenderness, refinement, Dogmatic : Categorical, authoritative, firm, preachy

s
s
purity Dolt : Blockhead, stupid, fool, idiot, dullard

.c
Delusion : Illusion, fancy, error, false belief Domicile : Dwelling, home, residence

o
m
Demeanour : Behaviour, conduct, bearing Dotage : Senility, imebcility
Demise : Death, decease Downright : Simple, unquestionable, blunt, frank
Demolish : Break, destroy, annihilate Dread : Apprehend, fear
Demure : Modest, coy, humane Drench : Soak, wet
Denomination : Name, appellation, designation Drowsy : Sleepy, comatose, lazy, lethargic
Denounce : Accuse, malign, criticise, defame, Dubious : Suspicious, doubtful, unreliable
condemn Ductile : Pliant, yielding, flexible
Deny : Contradict, refuse, disavow, withhold Dupe : Cheat, befool, steal
Deride : Ridicule, mock, taunt Dwindle : Shrink, diminish, decrease

E
Descant : Discourse, expatiate, enlarge
Desire : Wish, long for, crave, covet arnest : Eager, ardent, intent, anxious, sincere
Desolate : Lonely, deserted, solitary, devastated
Despise : Condemn, dislike Eccentric : Irregular, anomalous, abnormal, odd
Despondency : Despair, dejection, hopelessness Economise : Save, retrench
Despotic : Arbitrary, tyrannical, illegal Ecstasy : Trance, enchantment, rapture
Destitute : Needy, poor, miserable, indigent Efface : Blot out, obliterate, destroy
Destruction : Ruin, demolition, ravage Effeminate : Womanly, weak, unmanly
Detain : Lock in, arrest, hold, custody Efficacy : Energy, virtue, potence
Detest : Despise, abhor, dislike Egotistic : Self-centered, egoist, self-conceited
Dethrone : Depose, remove (from office) Egregious : Conspicuously bad, sinful, monstrous,
Devastate : Ruin, demolition, ravage shocking
Devoid : Lacking, empty, vacant Elaborate : Explain, discuss, elucidate
Devout : Religious, reverent Elevated : Elated, promoted, upgraded, risen
Dexterity : Adroitness, cleverness, skill Eliminate : Remove, replace, dismiss, discard
Diabolical : Fiendish, devilish, wicked Eloquence : Oratory, rhetoric, finery (of speech)
Diatribe : Tirade, denunciation fluency of expression
Dictatorial : Tyrannical, arbitrary, despotic Emanate : Originate, proceed, spring, issue
Diffident : Hesitating, doubtful, distrusting Emancipate : Free, deliver, liberate
Digression : Excursion, deviation, misguidance Embarrass : Vex, confuse, entangle
Diligence : Care, industry, effort Embezzle : Steal, peculate, cheat
Dire : Terrible, awful, horrible; miserable Embody : Incorporate, include, comprise
Disapprove : Condemn, reject, disallow Emolument : Salary, wage, remuneration
Disavow : Deny, refuse Emulate : Compete, rival, vie against, copy
Disciple : Follower, learner, student Enchant : Charm, bewitch, hypnotise
Disclose : Reveal, tell, uncover, divulge Encompass : Surround, encircle
Disconsolate : Sad, cheerless, miserable Encounter : Come across, combat, fight
Discredit : Disbelieve, doubt, disgrace Encroach : Trespass, intrude, invade
Disgust : Abhorrence, dislike, detestation Endeavour : Attempt, effort, aspiration
Dismay : Disappointment, discouragement Endorse : Back, approve, ratify
Disorder : Disease, illness, untidiness, uncleanliness Endurance : Patience, continuance, fortitude
Disown : Disclaim, deny, renounce Enfranchise : Liberate, free, (also: give right to vote)
Disparage : Debase, decry, defame Enlighten : Illuminate, edify, elaborate
Dispose : Adjust, arrange, incline Enmity : Hostility, hatred, animosity
Dispute : Argument, controversy, altercation Enormous : Big, huge, colossal, gigantic
Disregard : Neglect, overlook, disrespect Enrage : Infuriate, madden, incense, irritate
Dissolute : Corrupt, mean, lax, licentious Ensue : Succeed, follow, result
Distaste : Abhorrence, dislike, detestation Entangle : Ravel, involve, perplex
y
o
u
Vocabulary C-31

rs
m
Enterprise : Undertaking, venture, endeavour Figurative : Typical, imaginative, emblematic,

a
Enthusiasm : Zeal, ardour, interest metaphorical

h
b
Entice : Allure, tempt, seduce, attract Filthy : Dirty, foul, nasty

o
Entreat : Beseech, implore, beg Fissure : Crevice, rift, narrow opening

o
b
Entwine : Encircle, surround, encompass Flaccid : Soft, loose, weak

.w
Enumerate : Count, specify, recount, list, mention Flatter : Adore, please, praise

o
rd
Enunciate : Declaire, publish, propound, reveal Fleece : Rob, despoil, cheat

p
Envoy : Delegate, messenger, ambassador, deputy Flexible : Variable, pliable, changeable

re
Epoch : Era, time, age Flimsy : Transparent, thin, trivial, tenuous

s
s
Equivocal : Doubtful, ambiguous, uncertain Flounder : Stumble, falter, wallow, struggle

.c
Eradicate : Root out, extirpate, annihilate Fluctuate : Undulate, waver, vacillate

o
m
Erroneous : Wrong, false Flutter : Flip, quiver, ruffle, agitate
Erudite : Learned, scholarly, lettered Folly : Absurdity, silliness, imprudence,
Esteem : Admire, appreciate, adore, respect foolishness
Eulogy : Laudation, praise, extolling, felicitation Fondle : Caress, touch, rub
Evidence : Testimony, proof, witness Foray : Incursion, inroad, venture
Evince : Show, manifest, demonstrate Forbearance : Abstaining, refraining
Exact : Extort, oppress, loot Forbid : Prohibit, disallow, debar
Exaggerate : Amplify, overstate Forebode : Foretell, indicate, augur
Excerpt : Extract, quotations Forlorn : Disconsolate, cheerless, distressed,
Exile : Expulsion, banishment, expatriation abandoned, lonely
Exonerate : Acquit, absolve, release Formidable : Dreadful, difficult, hard to overcome
Exorbitant : Excessive, too much, very high Fragile : Weak, feeble, slender, delicate
Extinguish : Quench, terminate, destroy, put out Fragrant : Odorous, balmy, soothing
Extravagant : Excessive, lavish, stylish Frailty : Weakness, delicacy, fragileness
Exuberant : Abundant, plentiful Frantic : Frenzied, mad, distracted
Exult : Triumph, rejoice, delight Fraudulent : Dishonest, cheating, deceitful

F
Fray : Battlefield, combat, brawl
able : Story, legend, myth, fiction Frisk : Skip, dance, caper, frolic
Frivolous : Vain, foolish, trivial
Fabricate : Construct, forge, invent Frugal : Economical, thrifty
Fabulous : Fictitious, mythical, exaggerated Futile : Useless, hopeless, in vain
Facile : Fluent, ready, glib (of writing), pliable,

Faction :
docile, tractable
Clique, cabal, discord, section
G aiety : Hilarity, jollity, festivity
Fallacy : Deception, illusion, mistake Gainsay : Contradict, dispute, controvert, deny
Falter : Waver, hesitate, delay, flounder Gallantry : Courage, bravery, heroism
Famine : Hunger, starvation, scarcity of food Garbage : Filth, waste, useless, throwaway, trash
Fanatical : Bigoted, enthusiastic Garner : Accumulate, collect, gather
Fancy : Liking, conception, craving, whim Garrulous : Prattling, chattering
Farcical : Droll, comic, extravagant Gawky : Awkward, clumsy
Fascinate : Charm, bewitch, attract Gay : Happy, merry, joyous
Fastidious : Particular, over-nice, squeamish Generous : Noble, magnanimous, kind, liberal
Fatal : Deadly, lethal, mortal Genteel : Well-bred, well-cultured, polite, refined
Fatigue : Weakness, exhaustion, tiredness Genuine : True, authentic, real
Feeble : Weak, frail, dim Ghastly : Horrible, horrifying, horrific
Felicitate : Congratulate, compliment Ghost : Phantom, spirit, spectre
Felicity : Joy, happiness, good luck Glimmer : Shine, flash, gleam
Felon : Criminal, sinner, guilty, bandit Glimpse : Glance, (quick) look, (brief) view
Ferment : Excite, agitate Glisten : Shine, beam, glow
Ferocity : Fierceness, vehemence, fanaticism Gloom : Depression, melancholy, loneliness
Fervent : Glowing, heated, impassioned Glutinous : Sticky, viscous
Fervour : Warmth, glow, vehemence Gluttonous : Greedy, gorging, voracious
Festivity : Gaiety, merry-making Grandeur : Splendour, magnificence, glory
Fetter : Shackle, bind, imprison Grapple : Grasp, clutch, seize
Feud : Dispute, rivalry, hostility Gratification : Satisfaction, enjoyment
Fickle : Changeable, variable, volatile, unsteady Gravity : Seriousness, importance, calmness
Fiendish : Devilish, diabolical, malignant Greed : Avarice, ravenous, envious, covetous
y
o
u
C-32 Vocabulary

rs
m
Grievance : Hardship, complaint, trouble Inarticulate : Voiceless, indistinct, inexpressive

a
Grotesque : Horrifying, contorted, bizzare, whimsical Incense : Infuriate, enrage, anger

h
b
Grudge : Grievance, begrudge, resent Incessant : Unceasing, continual

o
Gullible : Simple, easy, pliable, credulous Incognito : (Identity) Concealed, secretly, stealthily

o
b
H Inculcate : Instil, enforce, pass on, generate

.w
allucination : Delusion, illusion, nightmare Inculpate : Blame, incriminate

o
rd
Incumbent : Compulsory, obligatory, binding
Hamper : Impede, block, fetter, bind

p
Incursion : Inroad, foray, venture

re
Hapless : Unfortunate, unlucky
Indefatigable : Tireless, assiduous

s
Harangue : a lengthy speech, oration

s
Indict : Accuse, charge

.c
Hardship : Trouble, adversity, difficulty

o
Indiscriminate : Mixed, undistinguished, confused,

m
Haughty : Arrogant, overbearing, imperious
Havoc : Devastation, destruction, ruin wanton
Hearty : Sincere, warm, ardent Indolence : Apathy, inactivity, laziness, sluggishness,
Heave : Raise, lift lethargy
Hedge : Fence, hem Industrious : Busy, active, tireless
Heed : Advise, note, consider, mind Inevitable : Unavoidable, certain, sure
Heinous : Atrocious, odious, wicked Inexorable : Relentless, indefatigable
Hem : Border, edge, fringe, outskirt Infallible : Unfailing, unerring, certain
Herculean : Colossal, laborious, excessive Infectious : Catching, contaminating, corrupting,
Heterogeneous: Dissimilar, unlike, different, diverse, transmittable
varied Infer : Gather, conclude, deduce
Hideous : Terrific, horrible, filthy Influence : Authority, effect, power
Hilarious : Exceedingly, funny, boisterously merry, Infringe : Break, violate, transgress, encroach
amusing, joyous Ingredient : Constituent, component, element
Hindrance : Impediment, hampering, obstruction, Inherent : Inborn, innate, built-in
obstacle Inhibition : Restraint, check
Histrionic : Theatrical, dramatic Iniquitous : Unjust, wrong, unfair
Hoax or Initiate : Start, begin, inchoate
Humbug : Dupe, cheat, befool
Innocuous : Harmless, mild, innocent
Homage : Respect, salute, worshipping, tribute
Insanity : Madness, lunacy, mania
Horizon : Verge, limit, skyline
Insidious : Deceitful, treacherous
Hostile : Adverse, opposing, inimical
Hover : Hang around, remain suspended, linger Instantaneous : Immediate, sudden, quick
Humane : Compassionate, caring, bvenevolent Instigate : Arouse, misguide, provoke
Humiliate : Abase, insult, mock, defy Intact : Untouched, unscathed, whole, unbroken,
Humorous : Amusing, laughable, comical undamaged, unimpaired, entire
Hypocrisy : Pretence, imposture, deceit Integrity : Oneness, entirety, completeness,
honesty, wholeness, soundness
I dolise : Adore, worship, admire Intellectual
Intentional
:
:
Intelligent, rational, learned
Deliberate, intended,
Illegal or Illicit : Unlawful, prohibited Intercourse : Intimacy, association, communication
Illogical : Inconsequent, irrational Intermission : Suspension, stoppage, pause, cessation,
Imitate : Copy, follow, mimic, emulate interval
Immaculate : Spotless, stainless, perfect Intervene : Interpose, mediate
Immature : Crude, childish, unseasoned
Intimate : Close, tender, friendly or informal
Imminent : Impending, near, due, threatening
Intolerable : Unendurable, unbearable
Immodest : Indecent, indelicate, unchaste
Intransigent : Uncompromising, inflexible
Immortal : Everlasting, undying, endless
Intrepid : Brave, fearless
Impassioned : Fervent, frenzied, fanatical
Impeach : Indict, charge, accuse Intricate : Complex, difficult, complicated
Implicit : Implied, assumed, inferred Intuition : Insight, premonition, instinct
Impostor : Cheat, conman, charlatan Inveterate : Habitual, deep-rooted
Impracticable : Unachievable, impossible, out of the Involuntary : Compulsory, unwilled, reflex
question Irksome : Annoying, disagreeable, tedious
Impressive : Striking, affecting, extraordinary Irresolute : Wavering, confused, vacillating
Impunity : Exemption (from punishment), immunity Itinerant : Travelling (on a circuit), wandering,
Inane : Empty, silly, idiotic nomadic
y
o
u
Vocabulary C-33

rs
J

m
Munificent : Extraordinarily generous, magnanimous

a
aded : Tired, weary Myriad : A vast indefinite number, multitude

h
N

b
o
Jargon : Cant, technical language, idiom

o
adir : The lowest point, the depths

b
Jocular : Humorous, funny, witty

.w
Jovial : Merry, cheerful, happy Nefarious : Wicked in the extreme, villainous

o
Judicious : Prudent, wise, sensible Negligent : Apt to omit what ought to be done

rd
Juxtaposition : Contrast, comparision, proximity Neophyte : Having the character of a beginner

p
re
K
Noxious : Hurtful, toxic, harmful

s
Nugatory : Having no power or force, worthless

s
iosk : Stand (open on one side), booth, stall

.c
O

o
Knead : Work up (into dough), press, squeeze

m
bdurate : Impassive to feelings of humanity or pity

L acerate : Tear (tissue toughly), mangle Obfuscate


Oblique
:
:
To darken; to obscure
Slanting; said of lines, diagonal
Lachrymose : Given to shedding tears, weaping Obstreperous : Boisterous
Lackadaisical : Listless, careless, lazy Odious : Hateful, repulsive, foul
Laconic : (Briskly) Short, concise, pithy Odium : A feeling of extreme repugnance, or of
Languid : Listless, spiritless dislike and disgust
Languish : Die (with hunger or desire), weaken, Ominous : Portentous, threatening
droop Onerous : Burdensome or oppressive
Lascivious : Lustful, wanton, dirty Onus : A burden or responsibility, mantle
Lassitude : Weariness, tiredness
Latent
Laudable
:
:
Hidden, concealed
Praiseworthy, admirable, worthy
P alate : The roof of the mouth, savour
Legacy : A bequest, inheritance Palatial : Magnificent, Luxurious
Legitimate : Genuine, lawful Palliate : To cause to appear less guilty
Levity : Frivolity, carefreeness, mirth Palpable : Perceptible by feeling or touch
Libel : Defamation, character assassination Panoply : A full set of armor
Licentious : Wanton, dissolute, depraved Paragon : A model of excellence
Lithe : Supple, graceful, limber Pariah : A member of a degraded class; a social
Loquacious : Talkative, verbose, profouse outcast
Lustrous : Shining, gleaming, scientillating Paroxysm : A sudden outburst (of any kind of
activity)

M alaise : A condition of uneasiness or ill-being,


Paucity
Pellucid
Penchant
:
:
:
Fewness
Translucent
A bias in favor of something
weariness, unhappiness Penurious : Excessively sparing in the use of money
Malevolence : Ill will, hostility, hatred Penury : Indigence, extreme/direpoverty
Malleable : Pliant, ductile, soft Peremptory : Precluding question or appeal
Mawkish : Sickening or insipid, sentimental Perfidy : Treachery, deceit, betrayal
Mellifluous : Sweetly or smoothly flowing, soothing Perfunctory : Half-hearted, brief, hasty
Mendacious : Untrue, lying, deceitful Peripatetic : Walking about, nomadic, migrant
Mendicant : A beggar, vagabond Perjury : A solemn assertion of a falsity
Mesmerise : To hypnotize, spellbind, bewitch Permeate : To pervade, spread through
Meticulous : Over-cautious, careful, diligent Pernicious : Tending to kill or hurt, destructive
Mettle : Courage, moral fibre, resolve Persiflage : Banter, repartee
Mien : The external appearance or manner of a Perspicacity : Acuteness or discernment
person, attitude, expression Perturbation : Mental excitement or confusion
Moderation : Temperance, self-discipline Petulant : Displaying impatience
Modicum : A small or token amount, fragment Phlegmatic : Not easily roused to feeling or action
Mollify : To soothe, pacify, calm down Pique : To excite a slight degree of anger in
Mordant : Biting, cutting, sarcastic Plea : An argument (to obtain some desired
Moribund : On the point of dying, near the end action)
Morose : Gloomy, bad-tempered, moody Plenary : Entire, unconditional
Multifarious : Having great diversity or variety, diverse Plethora : Excess; superabundance
Mundane : Worldly, as opposed to spiritual or Poignant : Severely painful or acute to the spirit
celestial Polyglot : Speaking several tongues
y
o
u
C-34 Vocabulary

rs
R

m
Ponderous : Unusually weighty or forcible

a
Portent : Anything that indicates what is to abble : Throng (of the vulgar), crowd, proletariat

h
b
happen

o
Rabid : Furious, mad, fanatical

o
Pragmatic : Practical(values), empirical

b
Raconteur : A person skilled in telling stories
Precarious : Critical, dangerous

.w
Raillery : Jesting (language), banter, ridicule
Preclude : To prevent, ruleout, stop

o
Ramify : To divide or subdivide into branches

rd
Precocious : Advanced (in development), overforward,
or subdivisions

p
premature

re
Rampant : Rife, widespread

s
Predilection : Preference, partiality, inclination Ramshackle : Dilapidated, tumbledown, rickety

s
.c
Predominate : To be chief in importance, quantity, or Rapacious : Sieze by force, avaricious

o
degree

m
Raucous : Harsh, sharp, hoarse
Preposterous : (Very) Absurd, ridiculous Recalcitrant : Stubborn, refractory
Prerogative : (Special) Right, privilege Recluse : One who lives in r etirement or
Presage : To foretell, prophery, augur seclusion
Prescience : Knowledge of events before they take Recondite : Incomprehensible to one of ordinary
place understanding
Preternatural : Extraordinary, uncommon, phenomenal Recuperate : To recover, bounce back
Prim : Stiffly proper, formal, prudish Redoubtable : Formidable, alarming, fearsome
Pristine : Primitive, perfect, unspoilt Redundant : Wordy, repetitious, superfluous, needless
Probity : Virtue or integrity tested and confirmed Refractory : Not amenable to control, stubborn
Proclivity : A natural inclination Regale : To give unusual pleasure, fascinate
Regicide : The killing of a king or sovereign
Procrastination : Delay, hesitation
Reiterate : To say or do again and again
Prodigious : Large, immense
Relapse : To suffer a return of a disease after
Profligate : Immoral, wanton, reckless, dissolute,
partial recovery
licentious Relegate : Assign a lower position, banish, demote
Profuse : Produced or displayed in overabundance Repast : A meal; figuratively, any refreshment
Prolix : Verbose, lengthy, prolonged Repine : To indulge in fretfulness and faultfinding
Promiscuous : Indiscriminate, impure, casual Reprisal : (Injury in) Return, retaliation, revenge
Propinquity : Nearness, proximity Reprobate : One abandoned to depravity and sin
Propitious : Kindly disposed, favourable Repudiate : Disavow, disclaim
Prosaic : Unimaginative, dull, dry Resilience : The power of springing back to a former
Protagonist : Leading character), hero(ine) position, flexibility, elasticity
Providential : Fortunate, lucky Resonance : Able to reinforce sound by sympathetic
Prudence : Caution, wisdom, sagacity vibrations
Puerile : Childish Respite : Interval of rest, delay, suspension
Pugnacious : Quarrelsome, aggresive Restive : Resisting control, restless, tense
Puissant : Powerful, influential, mighty Revere : To regard with worshipful veneration,
Punctilious : Exact (in formalities), ceremonious, admire
conscientious Reverent : Humble, respectful, adoring
Rotund : Round from fullness or plumpness,
Pungency : The quality of affecting the sense of
chubby, fat
smell
Ruffian : A lawless or recklessly brutal fellow
Pusillanimous : Without spirit or bravery
Ruminate : To chew over again, as food previously
Putrefy : Decay, rot, decompose swallowed and regurgitated
Pyre : A heap of combustibles arranged for
burning a dead body
S agacious : Wise, shrewd, astute

Q ualm : Misgiving, doubt, anxiety Salacious


Salubrious
:
:
Obscene, foul, indecent, lecherous
Healthful; promoting health
Quandary : Doubt, dilemma, plight Salutary : (Morally) Healthy, salubrious, beneficial
Quibble : An utterly trivial distinction or objection, Sanguine : Ardent, confident, optimistic
protest, grouse Sardonic : Ironical, scornful, derisive
Quiescence : Being quiet, still, or at rest; inactive Satiate : Gratify (fully), surfeit, saturate
Quirk : Twist, quibble, deviation Satyr : A very lascivious person
Quixotic : (Foolishly) Chivalrous, unrealistic, Savor : To perceive by taste or smell
whimsical Schism : Disjunction, split
y
o
u
Vocabulary C-35

rs
m
Scribble : Hasty, careless writing Trepidation : Nervous uncertainty of feeling

a
Sedition : Plotting (against government), incitement, Trite : Made commonplace by frequent

h
b
insurgence repetition

o
Sedulous : Persevering in effort or endeavor Truculent : Having the character or the spirit of a

o
b
Severance : Separation, cut off savage

.w
Sinecure : Any position (having emoluments with Turbid : In a state of turmoil; muddled

o
rd
few or no duties) Turgid : Swollen, bombastic, pompous

p
Sinuous : Curving in and out, zig-zag, bending Turpitude : Depravity, immorality, corruptness

re
Sluggard : A person habitually lazy or idle
U

s
s
biquitous : Being present everywhere

.c
Solace : Comfort in grief, trouble, or calamity

o
Solvent : Having sufficient funds to pay all debts

m
Umbrage : A sense of injury, take something
Somniferous : Tending to produce sleep personally
Somnolent : Sleepy, tired, lethargic Unctuous : Fawning, oily, self-abasing
Soporific : Causing sleep; also, something that Undulate : To move like a wave or in waves
causes sleep Untoward : Unexpected, unpredictable, unfavourable
Sordid : Filthy, morally degraded Upbraid : To reproach as deserving blame
Specious : Plausible but worry, misleading
Spurious
Squalid
:
:
Not genuine, pretended, fabricated
Having a dirty, mean, poverty-stricken
V agary : A sudden desire or action, ecentricity

appearance, filthy, wretched Vainglory : Excessive, pretentious, and demonstrative


Stanch : To stop the flowing of; to check vanity
Stingy : Cheap, unwilling to spend money Valorous : Courageous
Stolid : Expressing no power of feeling or Vapid : Having lost sparkling quality and
perceiving flavour
Submerge : To place or plunge under water Variegated : Having marks or patches of different
Subterfuge : Evasion, deceit, dishonesty colours; also, varied
Succinct : Concise, compact, to the point Vehement : Very eager or urgent
Sumptuous : Rich and costly, lavish, grand
Venal : Mercenary, corrupt
Supercilious : Exhibiting haughty and careless
Veneer : Outside show or elegance
contempt, arrogant, superior
Venial : That may be pardoned or forgiven, a
Superfluous : Being more than is needed, left over
Supersede : To displace, replace, substitute forgivable sin
Supine : Lying on the back, stretched out Veracious : Habitually disposed to speak the truth
Supplicate : To beg, urge, ask, request Veracity : Truthfulness, accuracy, faithfulness
Suppress : To prevent from being disclosed or Verbiage : Use of many words without necessity
punished, subdue, repress Verbose : Wordy, lengthy
Surcharge : An additional amount charged Verdant : Green with vegetation
Surfeit : To feed to fullness or to satiety Veritable : Real; true; genuine
Susceptibility : Vulnerability, sensitivity, liability Vestige : (A visible) trace, mark, or impression

T aciturn : Disinclined to conversation, unresponsive Virago :


(of something absent, lost, or gone)
Loud talkative women, strong statured
Taut : Stretched tight, flexed, strained women
Temerity : Foolhardy disregard of danger; Virtu : Rare, curious, or beautiful quality
recklessness Visage : The face, countenance, or look of a
Terse : Pithy, abrupt, brief person
Timorous : Lacking courage, fearful, shy Vitiate : To contaminate
Torpid : Dull; sluggish; inactive Vituperate : To overwhelm with wordy abuse
Torrid : Excessively hot, sultry, dry Vivify : To endue with life
Tortuous : Abounding in irregular bends or turns Vociferous : Making a loud outcry
Tractable : Easily led or controlled
Volatile : Changeable, unpredictable
Transgress : To break a law
Voluble : Having great fluency in speaking
Transitory : Existing for a short time only
Travail
Travesty
:
:
Hard or agonising labour
A grotesque imitation W himsical : Capricious, mischievous
Trenchant : Cutting deeply and quickly Winsome : Attractive, appealing, enchanting
y
o
u
C-36 Synonyms

rs
m
a
CHAPTER

h
b
3

o
SYNONYMS

o
b
.w
o
rd
p
re
s
s
.c
o
m
DIRECTIONS: Pick out the nearest correct meaning or synonym 14. GULLIBLE
of the words given below: (a) credible (b) believable
(c) credulous (d) fallible
1. ADVICE 15. BRAVERY
(a) council (b) counsel (a) onslaught (b) arrogant
(c) practice (d) proposal (c) fortitude (d) nepotism
2. MISERABLE 16. JEALOUS
(a) object (b) obstruct (a) obvious (b) atrocious
(c) abject (d) abstract (c) envious (d) ferocious
3. QUOTE 17. PATRONAGE
(a) sight (b) sigh (a) donation (b) support
(c) sue (d) cite (c) espionage (d) beneficiary
4. HARMONY 18. TRAVERSE
(a) cemetery (b) ceremony (a) mingle (b) frustrate
(c) symmetry (d) hierarchy (c) take (d) cross
5. UNLAWFUL 19. FEROCIOUS
(a) elicit (b) draw (a) fierce (b) bitter
(c) litigation (d) illicit (c) danger (d) enmity
6. HAUGHTY 20. RECUPERATE
(a) imperial (b) arrogant (a) recapture (b) delight
(c) adamant (d) empire (c) recover (d) overcome
7. WISE 21. ALMS
(a) momentous (b) pragmatic (a) blessings (b) charity
(c) judicious (d) delay (c) prayers (d) worship
8. LOQUACIOUS 22. ATTENUATE
(a) Victorian (b) bombastic (a) repent (b) make thin
(c) verbose (d) ambiguous (c) force (d) divide
9. COURAGEOUS 23. VINDICTIVE
(a) fickle (b) insipid (a) revengeful (b) triumphant
(c) timorous (d) fearless (c) strategic (d) demonstrative
10. WATCHFULNESS 24. DISCOMFIT
(a) supervision (b) custody (a) litigate (b) embarrass
(c) superintendence (d) vigil (c) conflict (d) frustrate
11. ATTACHMENT 25. WRATH
(a) affinity (b) influence (a) violence (b) anger
(c) causation (d) appendage (c) hatred (d) displeasing
12. WEARY 26. ABSTINENCE
(a) sad (b) fatigued (a) synchronic (b) torrential
(c) sentimental (d) emotional (c) restraint (d) gluttony
13. BEQUEST 27. ERUDITE
(a) parsimony (b) matrimony (a) execute (b) expanse
(c) heritage (d) patrimony (c) academic (d) settle
y
o
u
Synonyms C-37

rs
m
28. TACITURNITY 48. PHENOMENAL

a
(a) dumbness (b) changeableness (a) incidental (b) eventful

h
(c) hesitation (d) reserved (c) natural (d) extraordinary

b
o
29. ANIMATE 49. PARADIGM

o
b
(a) kill (b) dead (a) solution (b) model

.w
(c) energise (d) calm (c) discovery (d) invention

o
30. NIGGARDLY 50. HONORARY

rd
(a) penurious (b) generous (a) honest (b) dignified

p
re
(c) liberal (d) nimbus (c) unpaid (d) praiseworthy

s
31. BLITHE 51. FACULTY

s
.c
(a) joyless (b) grudging (a) privilege (b) desire

o
(c) somnolent (d) indifferent (c) branch (d) ability

m
32. CAPTIVATE 52. FORESEE
(a) repel (b) subjugate (a) contemplate (b) visualise
(c) dangerous (d) fascinate (c) assume (d) hypothesis
33. REDEEM 53. ANNEX
(a) extend (b) fulfil (a) add (b) low
(c) reconsider (d) recover (c) copy (d) initial
34. BLAND 54. MENAGE
(a) unpleasant (b) irritating (a) suffocation (b) system
(c) affable (d) tasteless (c) law (d) household
35. VISIONARY 55. DILEMMA
(a) dreamy (b) savant (a) darkness (b) freedom
(c) philosopher (d) saint (c) trap (d) confusion
36. REVOKE 56. RIGMAROLE
(a) repudiate (b) repeal (a) short-cut (b) lengthy procedure
(c) impute (d) force (c) unnecessary burden (d) happy responsibility
37. REPRISAL 57. TRANSCEND
(a) denial (b) reluctance (a) lower (b) climb
(c) unequivocal (d) retaliation (c) energise (d) cross
38. FASTIDIOUS 58. IMPERATIVE
(a) faint (b) dainty (a) order (b) command
(c) delicious (d) dormant (c) suggestion (d) necessity
39. ATTRIBUTE 59. EXEMPT
(a) infer (b) impute (a) duty (b) provide
(c) inhere (d) inundate (c) relieve of (d) forgive
40. DIALECTIC 60. INFIRMITY
(a) argumentative (b) instructive (a) disease (b) malady
(c) constructive (d) destructive (c) weakness (d) slimness
41. GERMINATE 61. IMMINENT
(a) decay (b) breed (a) eminent (b) immediate
(c) produce (d) sprout (c) future (d) impending
42. EFFICACY 62. CHASTE
(a) delicacy (b) ruthlessness (a) filthy (b) lewd
(c) efficiency (d) solemnity (c) immoral (d) noble
43. MAGNATE 63. FASCINATE
(a) tycoon (b) senior executive (a) captivate (b) irritating
(c) non-magnetic (d) symbolic (c) fashionable (d) impulsive
44. FACET 64. CURVATURE
(a) sweet (b) tap (a) angularity (b) straightness
(c) deceit (d) aspect (c) short-cut (d) streamline
45. PERNICIOUS 65. SUMMIT
(a) deadly (b) curious (a) base (b) slope
(c) gorgeous (d) expensive (c) declivity (d) peak
46. PERSUADE 66. WEAN
(a) assure (b) opinionated (a) introduce (b) withdraw
(c) convince (d) cheat (c) detach (d) alienate
47. FORTIFY 67. MENDACIOUS
(a) topple (b) destroy (a) irritating (b) misleading
(c) reproduce (d) strengthen (c) provocative (d) untruthful
y
o
u
C-38 Synonyms

rs
m
68. OSTRACISED 87. ADAGE

a
(a) hated (b) shut out from the society (a) proverb (b) youth

h
(c) criticised (d) applauded by the majority (c) supplement (d) hardness

b
o
69. SPURIOUS 88. ENSUE

o
b
(a) false (b) harmful (a) compel (b) plead

.w
(c) poisonous (d) foreign (c) remain (d) follow

o
70. OBNOXIOUS 89. ZENITH

rd
(a) clever (b) shrewd (a) lowest point (b) middle

p
(c) disagreeable (d) outdated

re
(c) compass (d) summit

s
71. PERNICIOUS 90. HYPOTHETICAL

s
.c
(a) radical (b) baneful (a) magical (b) theoretical

o
(c) scientific (d) negative (c) visual (d) two-faced

m
72. RECTIFY 91. SUPERFICIAL
(a) to command (b) to correct (a) shallow (b) aged
(c) to destroy (d) to build (c) unsually fine (d) proud
73. CORDON 92. DISPARAGE
(a) pile of logs (a) separate (b) belittle
(b) heavy cloak (c) compare (d) imitate
(c) line of people placed as guard 93. LUDICROUS
(d) none of these (a) profitable (b) ridiculous
74. CONCERT (c) excessive (d) undesirable
(a) beauty (b) power 94. INTREPID
(c) agreement (d) none of these (a) moist (b) rude
75. MITIGATE (c) tolerant (d) fearless
(a) to heal (b) soothen 95. FILCH
(c) to pardon (d) to send on a mission (a) hide (b) steal
76. DEVOID (c) swindle (d) covet
(a) evasive (b) hopeless 96. URBANE
(c) lacking (d) stupid
(a) well-dressed (b) friendly
77. RESOLVED
(c) polished (d) prominent
(a) summarised (b) dispelled
97. DECANT
(c) determined (d) hanged
(a) bisect (b) pour off
78. APPOSITE
(c) speak widly (d) bequeath
(a) appropriate (b) foolish
98. ANTITHESIS
(c) paiuful (d) none of these
(a) contract (b) examination
79. BUOYANT
(c) conclusion (d) opposite
(a) childlike (b) sturdy
99. HAVOC
(c) brisk (d) light-hearted (a) festival (b) sea battle
80. INFIRM (c) disease (d) ruin
(a) sturdy (b) anxious 100. REJUVENATE
(c) patient (d) feeble (a) reply (b) judge
81. INTELLECT (c) renew (d) age
(a) rationality (b) imbecility 101. OBNOXIOUS
(c) insanity (d) reverie (a) dreamy (b) daring
82. MANIAC (c) offensive (d) visible
102. VERBATIM
(a) lunatic (b) deft
(a) word for word (b) in secret
(c) sober (d) dunce (c) at will (d) in summary
83. OMEN 103. ENTICE
(a) augury (b) superstition (a) inform (b) attract
(c) imagery (d) imagination (c) observe (d) disobey
84. SPECTACLE 104. ACCLAIM
(a) pageant (b) show (a) discharge (b) divide
(c) mystification (d) panorama (c) excel (d) applaud
85. TURBULENCE 105. SOLILOQUY
(a) treachery (b) triumph (a) figure of speech (b) isolated position
(c) commotion (d) overflow (c) historical incident (d) monologue
86. DEFER 106. STUPEFY
(a) discourage (b) minimize (a) lie (b) make dull
(c) postpone (d) estimate (c) talk nonsense (d) overread
y
o
u
Synonyms C-39

rs
m
107. ADMONISH 127. AFFABLE

a
(a) polish (b) distribute (a) friendly (b) cheerful

h
(c) escape (d) caution (c) helpful (d) neutral

b
o
108. ATROPHY 128. EXORBITANT

o
b
(a) wither (b) grow (a) odd (b) ridiculous

.w
(c) soften (d) spread (c) excessive (d) threatening

o
109. COMPREHEND 129. TERRIFIC

rd
(a) agree (b) reprieve (a) big (b) excellent

p
re
(c) settle (d) understand (c) tragic (d) terrible

s
110. SUFFICE 130. ABNEGATION

s
.c
(a) endure (b) be adeqaute (a) self-denial (b) self-sacrifice

o
(c) annex (d) eat up (c) self-praise (d) self-criticism

m
111. PERSONABLE 131. AFFLUENT
(a) self-centered (b) initimate (a) prosperous (b) poor
(c) attractive (d) sensitive (c) talkative (d) close
112. ANALOGY 132. INFREQUENT
(a) similarity (b) distinction (a) never (b) usual
(c) transposition (d) variety (c) rare (d) sometimes
113. INTRIGUE 133. MASSACRE
(a) request (b) poison (a) stab (b) slaughter
(c) plot (d) veto (c) murder (d) assassinate
114. DEBONAIR 134. DISTINGUISH
(a) gay (b) extravagant (a) darken (b) abolish
(c) corrupt (d) healthful (c) differentiate (d) confuse
115. PONDEROUS 135. GRATIFY
(a) conceited (b) heavy (a) frank (b) appreciate
(c) shameless (d) abundant (c) pacify (d) indulge
116. CHARGIN 136. TERMINATE
(a) delight (b) caution (a) suspend (b) dismiss
(c) deceit (d) vexation (c) end (d) interrupt
117. DEFAMATION 137. OBJECT
(a) slander (b) debt (a) disobey (b) challenge
(c) infeciton (d) deterioration (c) deny (d) disapprove
118. APLOMB 138. ADVERSITY
(a) caution (b) shortsightedness (a) crisis (b) misfortune
(c) timidity (d) self-assurance (c) failure (d) helplessness
119. FORTITUDE 139. STUBBORN
(a) wealth (b) loudness (a) easy (b) obstinate
(c) courage (d) luck (c) willing (d) pliable
120. MERCENARY 140. TACITURNITY
(a) poisonous (b) unworthy (a) reserve (b) hesitation
(c) serving only for pay (d) luring by false charms (c) changeableness (d) dumbness
121. DEIFY 141. BLITHE
(a) face (b) worship (a) graceful (b) joyous
(c) flatter (d) challenge (c) giddy (d) other worldly
122. TYRANNY 142. ELICIT
(a) misrule (b) power (a) induce (b) extract
(c) madness (d) cruelty (c) divulge (d) instil
123. CONNOISSEUR 143. PORTRAY
(a) ignorant (b) interpreter (a) communicate (b) paint
(c) delinquent (d) lover of art (c) express (d) draw
124. WRATH 144. ARTIFACT
(a) jealousy (b) hatred (a) synthetic (b) man-made
(c) anger (d) violence (c) natural (d) exact copy
125. REBATE 145. PILFER
(a) loss (b) refund (a) destroy (b) damage
(c) compensation (d) discount (c) steal (d) snatch
126. PROLIFIC 146. LETHAL
(a) plenty (b) competent (a) dreary (b) dreadful
(c) fertile (d) predominant (c) deadly (d) strange
y
o
u
C-40 Synonyms

rs
m
147. TEDIOUS 167. OUTRE

a
(a) painful (b) troublesome (a) fair (b) traditional

h
(c) lengthy (d) tiresome (c) real (d) eccentric

b
o
148. OBSCENE 168. TEDIOUS

o
b
(a) objectionable (b) indecent (a) tiresome (b) painful

.w
(c) displeasing (d) condemnable (c) troublesome (d) lengthy

o
149. UNIFORMITY 169. ASSIMILATE

rd
(a) routine (b) continuity (a) absorb (b) arrange

p
re
(c) stability (d) constistency (c) receive (d) assemble

s
150. MYSTIQUE 170. COTERIE

s
.c
(a) fame (b) reputation (a) mob (b) group

o
(c) admirable quality (d) popularity (c) family (d) institution

m
151. RESILIENT 171. OBEISANCE
(a) flexible (b) proud (a) insult (b) obedience
(c) separable (d) rigid (c) indifference (d) disrespect
152. DUBIOUS 172. FATIGUE
(a) straight (b) sincere (a) weariness (b) sweating
(c) zig zag (d) doubtful (c) tension (d) drowsiness
153. CAVIL 173. FERFIDY
(a) appreciate (b) amuse (a) debauchery (b) deceit
(c) quibble (d) munch (c) treachery (d) conceit
154. RESTITUTE 174. MASTICATE
(a) help (b) avenge (a) devour (b) drink
(c) revenge (d) repair (c) chew (d) swallow
155. RETRIBUTION 175. BIZARRE
(a) contempt (b) revenge (a) colourful (b) strange
(c) punishment (d) discount (c) exotic (d) comical
156. INTRINSIC 176. ZENITH
(a) introvert (b) intricate (a) top (b) bright
(c) complicated (d) secret (c) wonderful (d) smart
157. APPRAISAL 177. CURSORY
(a) estimation (b) praise (a) penetrating (b) informal
(c) approval (d) investigation (c) superficial (d) angry
158. LUMINARY 178. DISMAL
(a) bright (b) lightning (a) deformed (b) impolite
(c) famous (d) dashing (c) bleak (d) watery
159. STRINGENT 179. NEMESIS
(a) shrill (b) regorous (a) punishment (b) victory
(c) dry (d) strained (c) adventure (d) reward
160. JEREMAI 180. CONNIVE
(a) friction (b) incident (a) threaten (b) shield
(c) trouble (d) accident (c) instigate (d) disregard
161. WHIMPER 181. ERSATZ
(a) prevent (b) cry (a) liveliness (b) imitation
(c) instigate (d) pacify (c) freshness (d) pleasure
162. HARASS 182. ANNOTATION
(a) grieve (b) injure (a) translation (b) prologue
(c) excite (d) annoy (c) quip (d) explanatory note
163. GADFLY 183. EXACERBATE
(a) harror (b) naisance (a) irritate (b) enlighten
(c) gain (d) blessing (c) aggravate (d) exaggerate
164. HYBRID 184. THRIVE
(a) unusual (b) hackneyed (a) hurt (b) persuade
(c) pedigreed (d) crossbred (c) push (d) flourish
165. MAIM 185. VAPID
(a) disfigure (b) slit (a) virtuous (b) vital
(c) severe (d) slash (c) priceless (d) dull
166. EXCTUCIATE 186. FRANTIC
(a) refifne (b) torture (a) urgent (b) excited
(c) extract (d) imprison (c) novel (d) painful
y
o
u
Synonyms C-41

rs
m
187. EGREGIOUS 207. PREVARICATE

a
(a) social (b) shocking (a) anticipate (b) lie

h
(c) common (d) plain (c) delay (d) authentiacate

b
o
188. MAMMOTH 208. EXUDE

o
b
(a) greedy (b) wild (a) ooze (b) wither

.w
(c) straight (d) huge (c) over flow (d) evaporate

o
189. HUMDRUM 209. PRECARIOUS

rd
(a) thoughtful (b) musical (a) brittle (b) perilous

p
re
(c) unnatural (d) commonplace (c) critical (d) cautious

s
190. MANIA 210. MALAISE

s
.c
(a) fame (b) greatness (a) stagnation (b) spite

o
(c) fear (d) illusion (c) curse (d) sickness

m
191. OBLOQUY 211. VOGUE
(a) lethargy (b) burial service (a) fashion (b) rejection
(c) verbal abuse (c) vulgar joke (c) order (d) satisfaction
192. ENTHRAL 212. IMPREGNATE
(a) inspire (b) charm (a) conceal (b) suffer
(c) glorify (d) annoy (c) affect (d) conclude
193. BAFFLE 213. DELEGATE
(a) insult (b) frustrate (a) officer (b) participant
(c) defame (d) antagonise (c) member (d) representative
194. DAUNT 214. ABANDON
(a) detain (b) annoy (a) admit (b) refrain
(c) abuse (d) intimidate (c) abstain (d) forsake
195. BEHOLDEN 215. AVER
(a) upright (b) lovable (a) assert (b) confess
(c) grateful (d) obliged (c) impress (d) trust
196. SOLICIT 216. YAW
(a) beseech (b) require (a) dedicate (b) soar
(c) claim (d) demand (c) arouse (d) drift
197. CLUMSY 217. DELECTABLE
(a) adroit (b) dexterous (a) attractive (b) delightful
(c) rough (d) ungraceful (c) desirable (d) delicate
198. FRICASSEE 218. HINDER
(a) grill (b) decorate (a) obstruct (b) challenge
(c) stew (d) to baste (c) damage (d) ruin
199. HINDER 219. REITERATE
(a) create (b) protect (a) reassess (b) rewrite
(c) vindicate (d) impede (c) repeat (d) stutter
200. ABIDE 220. FEIGN
(a) hold (b) encourage (a) pretend (b) attend
(c) accept (d) comment (c) condemn (d) condone
201. MONOLITHIC 221. VITUPERATE
(a) short-sighted (b) black & white (a) appreciate (b) abuse
(c) repetitive (d) very large (c) appraise (d) encourage
202. SYMBIOSIS 222. NAUSEATE
(a) transformation (b) close association (a) tempt (b) sicken
(c) cure- all (d) similarity (c) despise (d) detest
203. EGRESSION 223. INIQUITOUS
(a) digression (b) effusion (a) unequal (b) curious
(c) departure (d) hostility (c) biased (d) wicked
204. PERNICIOUS 224. ECSTATIC
(a) relevant (b) vigilant (a) animated (b) enraptured
(c) destructive (d) minute care (c) bewildered (d) fitful
205. IRRUPTION 225. REPLENISH
(a) hate (b) bursting in (a) fill (b) supply
(c) interference (d) altercation (c) provide (d) restore
206. ECHELON 226. WALLOW
(a) rank (b) opponent (a) luxuriate (b) suffer
(c) follower (d) identity (c) sacrifice (d) prosper
y
o
u
C-42 Synonyms

rs
m
227. INNATE 247. UMBRAGE

a
(a) unique (b) important (a) sensitive (b) shabbiness

h
(c) inborn (d) essential (c) premature (d) resentment

b
o
228. FURTIVE 248. EMULATE

o
b
(a) baffling (b) fleeing (a) trying to do as well

.w
(c) hasty (d) stealthy (b) enable

o
229. ACCOLADE (c) likely to be late (d) inspite to win

rd
(a) welcome (b) award 249. AMNESTY

p
re
(c) affection (d) arrival (a) revolt (b) privilege

s
230. INDIGENCE (c) farewell (d) pardon

s
.c
(a) poverty (b) prosperity 250. HIATUS

o
(c) suffering (d) scarcity

m
(a) gap (b) contempt
231. DISTINCTION (c) tight (d) narrow
(a) degree (b) difference 251. KITTY
(c) diffusion (d) disagreement (a) romantic (b) cheap
232. STALEMATE (c) pooled fund (d) drowsy
(a) deadly (b) dead-end 252. FRAGMENT
(c) deadlock (d) dead-drunk
(a) dissection (b) cut
233. REQUITE
(c) crumble (d) scrap
(a) repay (b) demand
253. CADGE
(c) refuse (d) requisition
(a) beg (b) hide
234. COMPENDIUM
(c) bicker (d) imprison
(a) glossary (b) reference
254. EXORCISE
(c) index (d) summary
235. VAPID (a) expel (b) expose
(a) virtuous (b) priceless (c) explain (d) mock
(c) dull (d) vital 255. AMBITION
236. OVERSTRUNG (a) plan (b) proclamation
(a) active (b) energetic (c) desire (d) decision
(c) concerned (d) too sensitive 256. TAINT
237. FRATERNISE (a) soil (b) stain
(a) associate (b) organise (c) dirty (d) corrupt
(c) expel (d) cheat 257. PROMISCUOUS
238. CATALOGUE (a) casual (b) selective
(a) menu (b) record (c) discriminate (d) pure
(c) list (d) pamphlet 258. KEMP
239. OVERSTRUNG (a) professional (b) parasite
(a) concerned (b) active (c) tent (d) champion
(c) sensitive (d) energetic 259. UNDULATE
240. GREGARIOUS (a) retard (b) decrease
(a) sociable (b) turbulent (c) wave (d) flood
(c) pugnacious (d) clumsy 260. VORACIOUS
241. ACCOST (a) quick (b) angry
(a) hesitate (b) speculate (c) hungry (d) wild
(c) insult (d) address 261. PESTER
242. DEFUNCT (a) console (b) disturb
(a) active (b) clever (c) gratify (d) torture
(c) alive (d) extinct 262. REDUNDANT
243. SAGACITY (a) rude (b) brutish
(a) morality (b) wisdom (c) superfluous (d) coarse
(c) sanity (d) uprightness 263. INEVITABLE
244. ANGST (a) unavoidable (b) probable
(a) anxiety (b) pride (c) expected (d) fixed
(c) modesty (d) simplicity 264. VENDETTA
245. KULAK (a) feud (b) friendship
(a) fortress (b) priest (c) mortal (d) threat
(c) insane (d) farmer 265. SMEAR
246. SALACITY
(a) encourage (b) quarrel
(a) recession (b) indecency
(c) avoid (d) mark
(c) depression (d) bliss
y
o
u
Synonyms C-43

rs
m
266. CALUMNIATE 275. TRIVIAL

a
(a) approve (b) slander (a) crucial (b) significant

h
(c) aarn (d) dealy

b
(c) vital (d) ordinary

o
267. PARVENU 276. PERSIST

o
b
(a) pretender (b) privileged (a) Resist (b) Leave

.w
(c) royal (d) intelligent (c) Quit (d) Insist

o
268. ATTRITION 277. EVENTUALLY

rd
(a) friction (b) decline (a) previously (b) briefly

p
re
(c) suffering (d) attraction (c) finally (d) successfully

s
269. MURKY 278. IMPECCABLE

s
.c
(a) ugly (b) dishonest (a) remarkable (b) unbelievable

o
(c) dull (d) lazy

m
(c) flawless (d) displeasing
270. CONDONE 279. SCORN
(a) Forgive (b) Support (a) concise (b) despise
(c) Forget (d) Defend (c) bias (d) fierce
271. ANALOGY 280. CATASTROPHE
(a) Difference (b) Comparison (a) tragedy (b) anger
(c) Addition (d) Deletion (c) violence (d) hatred
272. ALLURE 281. ABJURE
(a) Extol (b) Excite (a) renounce (b) announce
(c) Entice (d) Elicit (c) pronounce (d) denounce
273. VOCIFEROUS 282. ASSESS
(a) violent (b) loud (a) overload (b) measure
(c) secret (d) true (c) permit (d) enter
274. FICTIONAL 283. ELASTIC
(a) genuine (b) authentic (a) free (b) liberal
(c) fanciful (d) real (c) flexible (d) broad
y
o
u
C-44 Synonyms

rs
m
ANSWER KEY

a
h
1 (b) 43 (a) 85 (c) 127 (a) 169 (a) 211 (a) 253 (a)

b
o
2 (c) 44 (d ) 86 (c) 128 (c) 170 (b) 212 (c) 254 (c)

o
b
.w
3 (d) 45 (a) 87 (a) 129 (d) 171 (b) 213 (d) 255 (d)

o
4 (c) 46 (c) 88 (d) 130 (a) 172 (a) 214 (d) 256 (a)

rd
p
5 (d) 47 (d ) 89 (d) 131 (a) 173 (c) 215 (a) 257 (d)

re
6 (b) 48 (d ) 90 (b) 132 (d) 174 (c) 216 (d) 258 (b)

s
s
.c
7 (c) 49 (b ) 91 (a) 133 (b) 175 (b) 217 (b) 259 (c)

o
m
8 (c) 50 (c) 92 (b) 134 (c) 176 (a) 218 (a) 260 (c)
9 (d) 51 (d ) 93 (b) 135 (c) 177 (c) 219 (c) 261 (b)
10 (d) 52 (b ) 94 (d) 136 (c) 178 (c) 220 (a) 262 (c)
11 (a) 53 (a) 95 (b) 137 (d) 179 (a) 221 (b) 263 (a)
12 (b) 54 (d ) 96 (c) 138 (b) 180 (d) 222 (b) 264 (a)
13 (c) 55 (d ) 97 (b) 139 (b) 181 (b) 223 (d) 265 (d)
14 (c) 56 (b ) 98 (d) 140 (a) 182 (d) 224 (b) 266 (b)
15 (c) 57 (d ) 99 (d) 141 (b) 183 (a) 225 (d) 267 (a)
16 (c) 58 (d ) 100 (c) 142 (b) 184 (d) 226 (a) 268 (a)
17 (b) 59 (c) 101 (c) 143 (d) 185 (d) 227 (c) 269 (a)
18 (d) 60 (c) 102 (a) 144 (b) 186 (b) 228 (d) 270 (a)
19 (a) 61 (d ) 103 (b) 145 (c) 187 (b) 229 (b) 271 (b)
20 (c) 62 (d ) 104 (d) 146 (c) 188 (d) 230 (a) 272 (c)
21 (b) 63 (a) 105 (d) 147 (d) 189 (d) 231 (b) 273 (b)
22 (b) 64 (a) 106 (b) 148 (b) 190 (d) 232 (c) 274 (c)
23 (a) 65 (d ) 107 (d) 149 (d) 191 (c) 233 (a) 275 (d)
24 (b) 66 (d ) 108 (a) 150 (c) 192 (b) 234 (b) 276 (d)
25 (b) 67 (d ) 109 (d) 151 (a) 193 (b) 235 (c) 277 (c)
26 (c) 68 (b ) 110 (a) 152 (d) 194 (d) 236 (d) 278 (c)
27 (c) 69 (a) 111 (c) 153 (c) 195 (d) 237 (a) 279 (b)
28 (d) 70 (c) 112 (a) 154 (c) 196 (a) 238 (c) 280 (a)
29 (c) 71 (b ) 113 (c) 155 (c) 197 (d) 239 (c) 281 (a)
30 (a) 72 (b ) 114 (a) 156 (a) 198 (c) 240 (a) 282 (b)
31 (d) 73 (d ) 115 (b) 157 (a) 199 (d) 241 (d) 283 (c)
32 (d) 74 (c) 116 (d) 158 (c) 200 (c) 242 (d)
33 (d) 75 (a) 117 (a) 159 (b) 201 (c) 243 (b)
34 (d) 76 (c) 118 (d) 160 (c) 202 (b) 244 (a)
35 (a) 77 (c) 119 (c) 161 (b) 203 (c) 245 (d)
36 (b) 78 (a) 120 (c) 162 (d) 204 (c) 246 (b)
37 (d) 79 (d ) 121 (b) 163 (b) 205 (b) 247 (a)
38 (b) 80 (d ) 122 (d) 164 (d) 206 (a) 248 (d)
39 (b) 81 (a) 123 (b) 165 (a) 207 (b) 249 (a)
40 (a) 82 (a) 124 (c) 166 (b) 208 (a) 250 (c)
41 (d) 83 (a) 125 (d) 167 (d) 209 (c) 251 (d)
42 (c) 84 (a) 126 (c) 168 (a) 210 (d) 252 (a)
y
o
u
Synonyms C-45

rs
m
a
h
b
o
o
b
.w
1. (b) counsel also means ‘legal adviser’. one ticket available and he did not want to miss the

o
2. (c) abject means helpless, miserable, despicable situation, match for the sake of his friend. When Yadu came to

rd
p
obstruct means to hinder, abstract is the literal opposite know of this, he felt aggrieved. Out of vindictiveness,

re
of concrete, real. Yadu bought only one ticket for the next match and

s
s
3. (d) sight means scene, sue means to file a Suit. left Mohan alone. In other words, Yadu took a revenge

.c
4. (c) cemetery means graveyards, hierarchy means a system on Mohan.

o
m
of society in which people are graded into different 24. (b) If you are discomfited by something, it causes you to
classes according to certain norms, harmony is when feel slightly embarrassed or confused.
many things work together in the same mode. 25. (b) Wrath is extreme anger.
5. (d) illicit means not sanctioned by law, elicit means to draw 26. (c) If you abstain from something, you deliberately do
a response with difficulty, illegitimate means illegal/ not do it. Abstinence, however, is a particular kind of
unlawful abstaining —that from alcoholic drink, sex etc, often
6. (b) arrogant means revealing an exaggerated sense of own for health or religious reasons. If you abstain from
importance, adamant means firmly or stubbornly, drinking, you do not get drunk.
determinedly. 27. (c) An erudite person is scholarly. He or she is full of learning.
7. (c) judicious means having good sense, momentous Now, where does a learned person get his or her learning
means most important, pragmatic means practical, from? From the world of education or books. In other
treating in a sensible and realistic way. words, the person has an academic orientation.
8. (c) Verbose means using or containing more words than 28. (d) A taciturn person is not dumb. A dumb person can’t
are needed, ambiguous means having more than one speak. A taciturn person can speak but does not want
meaning. to. In other words, he maintains a reserve.
9. (d) fearless means lack of fear, fickle means unstable, often 29. (c) As an adjective, animate simply means “having life”.
changing, not constant, not faithful, Insipid means For example: Plants and animals are animate objects.
tasteless Now, as a verb: If you animate something, you make it
15. (c) Onslaught means ‘attack’; arrogant means ‘full of lively or more cheerful. Thus you put energy into it. In
pride’ other words, you energise it.
16. (c) Obvious means ‘clear’, ‘atrocious’ means ‘cruel’, 30. (a) If someone is niggardly, he shows lack of generosity.
ferocious means full of anger. Thus, generous would be its antonym. But we are
17. (b) Classical music today is in need of patronage. Earlier, looking for a synonym. None of the words is a fit
its patrons were the maharajas. Classical musicians synonym. In such cases we go for the word closest in
were honoured in the royal courts. They were given all meaning. Both penurious (very poor) and niggardly
the support necessary to let their art flourish. imply a lack of money, though in different senses.
18. (d) How did you traverse the river? That is, how did you 31. (d) Blithe refers showing casual and cheerful indifference
go from one side of it to the other? In other words, considered to be callous or improper.
how did you cross it? If you traverse an area of land or 32. (d) You are captivated by something attractive; It
water, you go across it. Traverse is a literary word. fascinates you.
19. (a) Fierce means displaying a violent or ferocious a violent 33. (d) I have lost my reputation and nothing can redeem it.
aggresiveness. That is, much though I may try, no good action on my
20. (c) Once I fall ill, it takes me long to recuperate. That is, part can bring my lost reputation back. In other words,
I take a long time to recover my health or strength after I cannot recover my reputation.
I have been ill. 34. (d) That which is bland is rather dull and unexciting.
21. (b) When a cyclone affects an area, the people are deprived Doordarshan is infamous for its bland presentations
of their belongings. You then give them money, clothes, of sports events. Similarly, food can be bland. That is,
food etc. Such gifts to the poor and needy are called it has very little flavour. In other words, it is tasteless.
alms. Since they are given out of charity (kindness 35. (a) A visionary is one who has vision for the future. He
and tolerance), these gifts are also known as charity. comes up with strong, original ideas. He can foresee the
22. (b) You are familiar with the word tenuous. Attenuate also changes to come and shapes the future, bringing about
comes from the same Latin root tenuis (= thin). If you an improvement. All visionary ideas are dreams at one
attenuate something, you make it slender or thin; you stage. But all dreams do not mature. Most of them remain
reduce it in force or value. pipedreams. A visionary is a dreamy person.
23. (a) Mohan and Yadu were friends. One day, however, 36. (b) When a law is revoked, it stands cancelled. We can
Mohan went to a cricket match alone. There was only also say that the law has been repealed.
y
o
u
C-46 Synonyms

rs
m
37. (d) When an army operation was launched to bomb the desired to edit this book, he would then have to be the

a
terrorists, they took the army chief hostage in reprisal. honorary editor of this book. That is, he cannot receive

h
b
That is, they mounted pressure on the army because any payment as wages for editing the book. An

o
o
the army mounted pressure on them. In other words, honorary job is thus unpaid for.

b
they retaliated. 51. (d) Your faculties are your physical and mental abilities.

.w
38. (b) If you are fastidious, you pay great attention to details. We have all been endowed with the faculty of

o
rd
You like everything to be very neat, accurate, and imagination. Most of us, however, kill it through an

p
orderly. A dainty object is small, delicate and pretty. exercise of constant neglect.

re
The nearness of meaning lies in that fastidious persons 52. (b) If you visualise something, you imagine what it is like

s
s
.c
like things to be dainty. by forming a mental picture of it. You may visualise

o
39. (b) If A is the cause of B, you attribute B to A. usually; if B future events as well, your wedding for example. You

m
is something bad (blame, charge etc.), we use impute thus foresee your wedding.
instead of attribute. Sample the following : When Sri 53. (a) From Akbar to Aurangzeb, every Mughal emperor went
Lanka beat Pakistan, the Sri Lankans attributed their on annexing one kingdom or another to what they
success to their batsmen. The Pakistanis imputed the inherited. In other words, each of them added to his
blame of their defeat to their sloppy fielding. father’s empire.
40. (a) In philosophy, dialectics is a method of reasoning and 54. (d) A menage is a group of people living together in one
reaching conclusions by considering theories and house. In other words, a household.
ideas together with ones that contradict them. Such 55. (d) If you are in a dilemma, you do not know what to do. In
contradictions lead to argumentation and often other words, you are in confusion.
degenerate into arguments. 56. (b) A rigmarole is a lengthy and complicated procedure
41. (d) When a seed germinates, it puts forth shoots. In other which you do not like.
words, it sprouts. 57. (d) If you ascend, you go up. If you descend, you go
42. (c) Efficacy is effectiveness. Efficiency is the ability to do down. If you transcend, you go across. In other words,
a thing successfully. Your efficacy is judged by how you cross.
efficient you are. 58. (d) An imperative is something that is extremely important
43. (a) John D Rockefeller was an oil magnate. Onassis was a and must be done. In other words, it is a necessity.
shipping tycoon. Both Rockefeller and Onassis were Revision of obsolete laws has become an imperative
big businessmen, their spheres being oil (petroleum) for the 50-year-old country.
and shipping respectively. Both magnate and tycoon 59. (c) Certain categories of companies have been exempted
refer to businessmen who have made it big, their from paying tax. That is, they do not have to shoulder
success making them rich and powerful. the burden of tax-paying. In other words, their burden
44. (d) A facet of something is a single part or aspect of it. has been relieved of.
45. (a) Violence in films has a pernicious influence on our 60. (c) If something is firm, it is strong enough to maintain its
children. That is, the influence is harmful. Similarly, a balance. Infirmity therefore suggests weakness.
deadly situation has harmful consequences. Is heinous 61 (d) Imminent means about to happen.
a synonym of pernicious? No. Heinous crimes are 62. (d) Filth means dirt, Lewd means characterless. A chaste
extremely evil and horrible. When one calls a crime person does not have sex with anyone or has it only
heinous, one is rather disgusted with its horror than with his or her spouse.
concerned about its harmfulness. 63. (a) Fasinate refers to attract the strong attention and
46. (c) If you persuade me to do something, you try to interest of someone.
convince me that it should be done. 65. (d) Peak. Summit is the highest point.
47. (d) When you fortify a place, you make it as if it were a 66. (d) Alienate means to withdraw.
fort. A fort has a wall or ditch round it and is not easy 67. (d) Mendacious is something untruthful
to attack. Thus, if you fortify something, you 68. (b) Ostracised is shutout from the society
strengthen it. 69. (a) Spurions is false
48. (d) There has been a phenomenal increase in the number 70. (c) disagreeable, unpleasant, nasty, offensive
of dish antennas in recent years. That is, the rise in 71. (b) Pernicious is ruinous, injurious, hurtful. Baneful is
number is much, much more than ordinarily expected. destructive.
The increase is thus extraordinary — “more than 72. (b) Rectify is to correct.
ordinary”. 73. (d) Cordon means a line of policemen or soldiers which
49. (b) A paradigm is a model for something which explains it guards or prevents people entering or leaving an area
or shows how it can be produced. As it is not covered any of the options, answer should
50. (c) Government servants are not allowed to receive money be ‘none of these.
from any other job. Suppose an income-tax officer 74. (c) Concert means together.
y
o
u
Synonyms C-47

rs
m
75. (a) Mitigate means to heal, to reduce, to release someone 96. (c) Urbane means to show high degree of refinement such

a
of his sufferings. as polished behavior. Svelte is another synonym.

h
b
76. (c) evasive means careless, stupid means foolish. 97. (b) To decant means to make a liquid flow out from a

o
77. (c) Resolved is determined. container.

o
b
78. (a) Apposite is suitable, pertinent, relevant. 98. (d) Antithesis is something exactly opposite.

.w
79. (d) Sturdy means strong, brisk means energetic. Buoyant 101. (c) Something causing disapproval or protest is obnoxious

o
rd
is cheerful; not easily depressed. 102. (a) Verbatim means precisely the same words used by a

p
80. (d) Infirm is feeble or weak in body or health. writer or speaker

re
81. (a) Imbecility means mental weakness, Insanity means 103. (b) Entice means to provoke someone to do something

s
s
madness, Reverie means day -dreaming, Vacuity means

.c
through (often false or exaggerated) promises or

o
emptiness. persuasion. Other synonyms are lure, tempt

m
82. (a) Lunatic means mad. Deft means skilful, Dunce means 104. (d) To acclaim means to praise vociferously. Other
highly ignorant. synonyms are hail, herald, plaudit
83. (a) Augury is an omen, token, or indication. 106. (b) To make dull or stupid or muddle
84. (a) Mystification means ‘deliberately making mysterious, 107. (d) To admonish means to warn or to caution. Other similar
difficult to understand, Tableau means dramatic scene.
words are Reprimand; express disapproval etc.
85. (c) Commotion means a disorderly outburst or tumult. It
110. (a) Suffice also means to answer or to serve
is very close to turbulence which means unstable flow
270. (a) condone and forgive are similar in meaning.
of a liquid or gas. Turbulence also refers to a state of
271. (b) Analogy denotes comparison.
disturbance.
272. (c) Allure and entice both denote provoking someone to
86. (c) Other synonyms are prorogue, put off, set back, shelve
87. (a) An adage is a proverb or byword do something through (often false or exaggerated)
88. (d) Ensue is a verb which means to happen afterwards as promises or persuasion.
a consequence or result. Example of use as adjective is 273. (b) The meaning of word Vociferous (Adjective) is :
"the ensuant response to his appeal" outspoken, blunt.
89. (d) Zenith means summit, top, celestial point. Its synonym should be : Loud.
90. (b) Hypothetical means something based primarily on 274. (c) The meaning of word Fictional (Adjective) is :
surmise rather than adequate evidence. Other Imaginary, unreal, fabricated, mythical
synonyms are conjectural, divinatory, suppositious. Its synonym should be : fanciful
91. (a) Superficial means something shallow, not deep
275. (d) The meaning of word Trivial (Adjective) is : not
intellectually or emotionally. Trivial is another
important.
synonym.
92. (b) Disparage means to express negative impression about Its synonym should be : ordinary.
something.Belittle is closest synonym. Disparagement 276. (d) 'Persist' means 'to insist'.
is communication that belittles somebody or 277. (c) 'Eventually' means 'finally'.
something. When you dispraise or derogate somebdoy, 278. (c) 'Impeccable' means 'flawless'.
you are disparaging. 279. (b) Scorn means contempt toward something.
93. (b) Ludicrous is something which invites ridicule. 280. (a) Catastrophe means an event causing great and usually
Something absurd. Other synonyms are farcical,
sudden damage or suffering.
derisory, idiotic, laughable, nonsensical.
281. (a) Abjure means solemnly renounce.
94. (d) Those who are invulnerable to some kind of fear are
intrepid. Other synonyms are brave, dauntless, 282. (b) Assess means to get measure of.
unfearing, hardy. 283. (c) Elastic means able to encompass much variety and
95. (b) To filch means to steal. Pilfer is another synonym. change; flexible and adaptable.
y
o
u
C-48 Antonyms

rs
m
a
CHAPTER

h
b
4

o
ANTONYMS

o
b
.w
o
rd
p
re
s
s
.c
o
m
DIRECTIONS: Pick out the opposite meaning or antonym of 14. TERSE
the words given below: (a) lengthy (b) scarce
1. IMPLICATE (c) diffuse (d) headless
(a) appease (b) exonerate 15. MULTIPLICITY
(c) adore (d) advocate (a) finite (b) uniformity
2. VACILLATING (c) magnitude (d) infinite
(a) fascinating (b) fanaticism 16. NAIVE
(c) indolence (d) resolute (a) subtle (b) energise
3. RECKLESS (c) spotless (d) clever
(a) modest (b) awkward 17. ANACHRONISTIC
(c) celebrated (d) cautious (a) formerly (b) present
4. INSULT (c) futuristic (d) non-existing
(a) humiliation (b) credulity 18. TRAIT
(c) degradation (d) honour (a) symbol (b) uncharacteristic
5. ABANDON (c) habit (d) identity
(a) roost (b) forfeit 19. GARISH
(c) quit (d) forsake (a) tasteful (b) green
(c) scenic (d) contrasting
6. OFFEND
20. GEOLOGICAL
(a) angry (b) hate
(a) astral (b) solar
(c) force (d) respect
(c) galactic (d) heavenly
7. INDICT
21. REJUVENATION
(a) condemn (b) reprimand (a) sexual (b) perfect
(c) acquit (d) allege (c) killing (d) magical
8. OBSCENE 22. ORIGIN
(a) decent (b) objectionable (a) ointment (b) detergent
(c) condemnable (d) jealousy (c) remnant (d) comfort
9. LIBERATE 23. APATHETIC
(a) imprison (b) enclose (a) agitated (b) happy
(c) liberal (d) conceal (c) concerned (d) surprised
10. MELANCHOLY 24. HOMOGENISED
(a) depressed (b) prejudiced (a) set type (b) multi-coloured
(c) reckless (d) cheerful (c) different (d) rejected
11. LEGITIMATE 25. PROVE
(a) valid (b) extend (a) vapid (b) assume
(c) unlawful (d) distinguished (c) disincline (d) atone
12. VACILLATE 26. ACCOLADE
(a) amplify (b) stimulate (a) balcony (b) outer garment
(c) consistent (d) eradicate (c) drink (d) criticism
13. HINDER 27. INSTANTLY
(a) expidite (b) protect (a) repeatedly (b) lately
(c) devote (d) create (c) gradually (d) awkwardly
y
o
u
Antonyms C-49

rs
m
28. FRAIL 47. CONSCIENTIOUS

a
(a) worried (b) strong (a) cruel (b) licentious

h
b
(c) nervous (d) wily (c) careless (d) whip

o
29. CRUDE

o
48. PERIGEE

b
(a) classical (b) graceful

.w
(a) apogee (b) hybrid
(c) natural (d) polished (c) descent (d) night

o
rd
30. RETRIBUTION 49. FLUCTUATE

p
(a) compensation (b) forgiveness (a) conceive (b) stabilise

re
(c) contempt (d) grudge

s
(c) energise (d) emancipate

s
31. PROCLAIM

.c
50. RADICAL
(a) denounce (b) pretend

o
(a) superficial (b) slow

m
(c) attend (d) distend
32. SUMPTUOUS (c) narrow (d) simple
(a) irritable (b) meagre 51. ACCORD
(c) fancy (d) sad (a) concord (b) policy
33. FEIGN (c) dissent (d) act
(a) condone (b) attend 52. HAPLESS
(c) willing (d) original (a) lucky (b) kind
34. INSIPID (c) helpful (d) futile
(a) witty (b) meagre 53. FRIVOLOUS
(c) wily (d) lucid (a) trivial (b) significant
35. SALUBRIOUS (c) fearless (d) permissive
(a) sticky (b) soft 54. INTEGRAL
(c) famous (d) malaise (a) minor (b) major
36. REFULGENT (c) essential (d) independent
(a) angry (b) dull 55. HOLISTIC
(c) sad (d) lament (a) negative (b) piecemeal
37. INNOCUOUS
(c) impure (d) inadequate
(a) offensive (b) harmless
56. EXTENSION
(c) organic (d) anger
(a) diminution (b) condensation
38. AFFECTATION
(a) sincerity (b) humility (c) deletion (d) subtraction
(c) stirring (d) affluent 57. INDIGENTLY
39. LUMINOUS (a) richly (b) awfully
(a) dark (b) ludicrous (c) completely (d) diligency
(c) unsteady (d) provoking 58. AUDACITY
40. INTRICACY (a) quivering (b) patricide
(a) ornate (b) simplicity (c) bravado (d) cowardice
(c) distance (d) cordiality 59. ELEVATION
41. AMELIORATE (a) depression (b) deflation
(a) amend (b) gyrate (c) depreciation (d) recession
(c) sweeten (d) worsen 60. PROFANE
42. LACKADAISICAL (a) pious (b) kitten
(a) abundant (b) energetic (c) energy (d) wild
(c) theatrical (d) actual 61. UNFATHOMABLE
43. CAPRICIOUS
(a) comprehensible (b) sinkable
(a) thoughtful (b) specious
(c) uncomfortable (d) infallible
(c) carcinogenic (d) capacious
62. TERMINATION
44. PERFIDIOUS
(a) loyal (b) treacherous (a) endeavouring (b) beginning
(c) religious (d) humane (c) amendment (d) phasing
45. ENNUI 63. INSPIRED
(a) sticky (b) activity (a) discouraged (b) extracted
(c) start (d) yearly (c) negated (d) admired
46. LASCIVIOUS 64. PARTICIPATE
(a) devout (b) fluid (a) precipitate (b) change
(c) chaste (d) stable (c) disengage (d) boycott
y
o
u
C-50 Antonyms

rs
m
65. EGALITARIAN 84. CORPULENT

a
(a) unequal (b) socialist (a) sallow (b) co-operative

h
b
(c) capitalist (d) liberal (c) enterprising (d) emaciated

o
66. DEFICIENCY 85. OCCIDENTAL

o
b
(a) abundance (b) deficit (a) oriental (b) accidental

.w
(c) ill (d) profit (c) coincidental (d) confidential

o
rd
67. FLOURISH 86. FLACCID

p
(a) improve (b) retard (a) upright (b) taut

re
(c) hamper (d) stop (c) rough (d) even

s
s
68. VENEER 87. NEPOTISM

.c
(a) exterior (b) interior (a) midnight (b) partiality

o
m
(c) impression (d) armour (c) impartiality (d) chauvinism
69. DICTATORSHIP 88. ABSTEMIOUS
(a) democracy (b) tyranny (a) fastidious (b) punctilious
(c) aristocracy (d) self-rule (c) dissipated (d) prodigal
70. EVENTUALLY 89. CHUIERICAL
(a) primarily (b) resultantly (a) numerical (b) real
(c) initially (d) objectively (c) obvious (d) heavenly
71. PHENOMENAL 90. VERBOSE
(a) ordinary (b) experiential (a) laconic (b) talkative
(c) natural (d) spiritual (c) vent (d) suspense
72. NONCHALANT 91. DIMINUTIVE
(a) fearful (b) cowardly (a) enlarged (b) bright
(c) patriotic (d) excited (c) small (d) admonitory
73. ABOMINATE 92. IMMUNITY
(a) love (b) loathe (a) obligatory (b) impassive
(c) abhor (d) despise (c) impervious (d) susceptibility
74. INGENUOUS 93. AMALGAMATE
(a) cunning (b) stupid (a) synthesise (b) bubble
(c) naive (d) young (c) separate (d) moderate
75. EGREGIOUS 94. CAPTIOUS
(a) notorious (b) splendid (a) capable (b) detailed
(c) abortive (d) maturity (c) tolerant (d) classical
76. DISSIPATE 95. MANIFOLD
(a) waste (b) conserve (a) hidden (b) enrolled
(c) organise (d) unite (c) simple (d) exact
77. STURDY 96. WARP
(a) important (b) rich (a) plush (b) web
(c) weak (d) vigorous (c) alienate (d) straighten
78. SACROSANCT 97. JOCOSE
(a) irreligious (b) unethical (a) dull (b) humorous
(c) irreverent (d) open (c) regulated (d) brief
79. CELIBATE 98. PAMPER
(a) reprobate (b) prodigal (a) neglect (b) scold
(c) profligate (d) married (c) scorn (d) discourage
80. OBSOLETE 99. NEGLIGENCE
(a) rare (b) useless (a) diligence (b) punctuality
(c) recent (d) conducive (c) integrity (d) meticulousness
81. MAGNANIMOUS 100. INSTANTLY
(a) small (b) generous (a) repeatedly (b) lately
(c) naive (d) selfish (c) immediately (d) slowly
82. EVACUATE 101. OBVIOUSLY
(a) admit (b) emerge (a) obscurely (b) surely
(c) abandon (d) invade (c) indefinitely (d) certainly
83. SANGUINE 102. EXTRAORDINARY
(a) bloody (b) thin (a) exceptional (b) unusual
(c) happy (d) gloomy (c) dull (d) plain
y
o
u
Antonyms C-51

rs
m
103. SPREAD 122. AFFECTIONATE

a
(a) express (b) prohibit (a) cold (b) indifferent

h
b
(c) contain (d) contradict (c) hostile (d) unfriendly

o
104. PROHIBIT

o
123. ERUDITE

b
(a) grant (b) agree

.w
(a) ignorant (b) unknown
(c) permit (d) accept (c) illiterate (d) unfamiliar

o
rd
105. RUTHLESS 124. PRETENTIOUS

p
(a) gracious (b) compassionate (a) unassuming (b) calm

re
(c) generous (d) malicious

s
(c) secretive (d) cowardly

s
106. CHAGRIN

.c
125. POLTROON
(a) tempt (b) ascetic

o
(a) plutocrat (b) hero

m
(c) swollen (d) pleased
107. BEGET (c) amateur (d) partisan
(a) forget (b) fade 126. ABOLISH
(c) harm (d) abort (a) remove (b) reside
108. AUSTERE (c) confront (d) establish
(a) painful (b) comfortable 127. RETALIATION
(c) lavish (d) plentiful (a) disintegration (b) wholesale
109. APPROPRIATE (c) admonition (d) reconciliation
(a) unqualified (b) unskilled 128. ABET
(c) unable (d) unsuitable (a) aid (b) risk
110. WILD (c) pacify (d) prevent
(a) arrogant (b) humble 129. REPREHENSIBLE
(c) tamed (d) rude (a) commendable (b) fearful
111. SUBSERVIENT (c) ignorant (d) culpable
(a) aggressive (b) straightforward 130. SEDATE
(c) dignified (d) supercilious (a) addicted (b) excited
112. CONFESS
(c) shy (d) inebriate
(a) deny (b) refuse
131. ABATE
(c) contest (d) contend
(a) gamble (b) dilute
113. ENGAGE
(a) abstain (b) liberate (c) increase (d) discourage
(c) release (d) join 132. MOROSE
114. JUDICIOUS (a) docile (b) boorish
(a) imprudent (b) silly (c) diffuse (d) cheerful
(c) separation (d) sagacious 133. SALIENT
115. UPROARIOUS (a) emphatic (b) striking
(a) tumultuous (b) upright (c) important (d) incline
(c) posture (d) calm 134. PRECEPT
116. GRATEFUL (a) discernment (b) instruction
(a) quick (b) beholden (c) important (d) incline
(c) unappreciative (d) convincing 135. PALPABLE
117. TOIL (a) innovative (b) fresh
(a) laborious task (b) sloth (c) imaginary (d) creative
(c) strive (d) vivid 136. MALIGNANT
118. INDISCRIMINATE (a) swallow (b) prune
(a) promiscuous (b) selective
(c) benign (d) virulent
(c) undistinguished (d) broad
137. POLEMIC
119. FICKLE
(a) aggressive attack (b) warlike
(a) steadfast (b) independent
(c) unwise (d) esoteric (c) logically argued (d) controversial
120. TARDY 138. DELIRIOUS
(a) sluggish (b) dilatory (a) large (b) calm
(c) reluctant (d) prompt (c) insane (d) responsive
121. FABLE 139. JUVENILE
(a) truth (b) fact (a) mature (b) youthful
(c) reality (d) actuality (c) blind (d) control
y
o
u
C-52 Antonyms

rs
m
140. MORIBUND 159. DEARTH

a
(a) restored (b) healthy (a) extravagance (b) scarcity

h
b
(c) wholesome (d) growing (c) abundance (d) sufficiency

o
141. ANALYSIS 160. TRANSPARENT

o
b
(a) synthesis (b) substitution (a) coloured (b) childlike

.w
(c) emphasis (d) replacement (c) opaque (d) imminent

o
142. GENUINE

rd
161. EXHIBIT

p
(a) innocent (b) spurious (a) conceal (b) prevent
EXERCISE 2 : ANTONYMS

re
(c) real (d) plutonic (c) withdraw (d) concede

s
s
143. DISSENT 162. HAUGHTY

.c
(a) ascent (b) accent (a) pitiable (b) scared

o
m
(c) agreement (d) convergence (c) humble (d) cowardly
144. DEFICIT 163. VIRTUE
(a) implicit (b) explicit (a) vice (b) fraud
(c) surplus (d) superfluous (c) wickedness (d) crime
145. RETREAT 164. ERUDITE
(a) advance (b) recede (a) professional (b) immature
(c) entice (d) caputre (c) unimaginative (d) ignorant
146. AUTONOMY 165. HAPLESS
(a) submissiveness (b) dependence (a) lucky (b) kind
(c) subordination (d) slavery (c) helpful (d) futile
147. SHALLOW 166. ACQUITTED
(a) high (b) hidden (a) entrusted (b) convicted
(c) deep (d) hollow (c) burdened (d) freed
148. OVERT 167. LACONIC
(a) deep (b) shallow (a) prolix (b) profligate
(c) secret (d) unwritten (c) prolifie (d) bucolic
149. SYNTHETIC 168. ABSOLUTE
(a) cosmetic (b) plastic (a) scarce (b) limited
(c) affable (d) natural (c) prolific (d) bucolic
150. PRECARIOUS
169. MAGNIFY
(a) dangerous (b) safe
(a) induce (b) diminish
(c) cautious (d) easy
(c) destroy (d) shrink
151. DEEP
170. BOOST
(a) elementary (b) superficial
(a) hinder (b) obstruct
(c) shallow (d) perfunctory
(c) discourage (d) rebuke
152. LEND
171. SMOOTH
(a) hire (b) pawn
(a) ugly (b) awkward
(c) cheat (d) borrow
(c) hard (d) rough
153. PAUCITY
172. BASE
(a) surplus (b) scarcity
(a) roof (b) height
(c) presence (d) richness
(e) want (c) top (d) climax
154. MINOR 173. MOIST
(a) heavy (b) tall (a) parched (b) dry
(c) major (d) big (c) hard (d) crisp
155. APPROPRIATE 174. DEMON
(a) unskilled (b) unsuitable (a) charitable (b) kind-hearted
(c) unqualified (d) unable (c) angel (d) fair-minded
156. OPAQUE 175. DELETE
(a) misty (b) covered (a) impound (b) insert
(c) clear (d) transparent (c) inspire (d) injure
157. RUTHLESS 176. ONEROUS
(a) mindful (b) compassionate (a) straight-forward (b) easy
(c) majestice (d) merciful (c) complex (d) plain
158. VIOLENT 177. BRIDGE
(a) tame (b) humble (a) divide (b) bind
(c) gentle (d) harmless (c) release (d) open
y
o
u
Antonyms C-53

rs
m
178. ATTRACT 197. CONCEAL

a
(a) repulse (b) reject (a) unfold (b) reveal

h
b
(c) repel (d) distract (c) open (d) discover

o
179. GRATUITY 198. SELDOM

o
b
(a) annuity (b) stipend (a) rarely (b) laily

.w
(c) discount (d) wages (c) often (d) never

o
199. PERTINENT

rd
180. KNOWLEDGE
(a) indifferent (b) detached

p
(a) ignorance (b) illiteracy

re
(c) foolishness (d) backwardness (c) determined (d) irrelevant

s
s
181. NADIR 200. AMALGAMATE

.c
(a) progress (b) liberty (a) generate (b) repair

o
m
(c) zenith (d) modernity (c) materialise (d) separate
182. FOREIGNER 201. AMICABLE
(a) national (b) stranger (a) cunning (b) shy
(c) native (d) alien (c) hostile (d) crazy
183. LANGUID 202. CLARITY
(a) smart (b) energetic (a) exaggeration (b) candour
(c) fast (d) ferocious (c) confusion (d) reserve
184. STRINGENT 203. CHALLENGE
(a) magnanimous (b) lenient (a) admire (b) accept
(c) vehement (d) general (c) favour (d) praise
185. ALIENATE 204. TEDIOUS
(a) gather (b) identify (a) pleasant (b) lovely
(c) assemble (d) unite (c) lively (d) gay
186. MALICIOUS 205. SUPERFICIAL
(a) boastful (b) indifferent (a) artificial (b) deep
(c) kind (d) generous (c) shallow (d) real
187. SPURIOUS 206. COMMEND
(a) false (b) genuine (a) suspend (b) admonish
(c) simple (d) systematic (c) hate (d) dislike
188. LISSOME 207. DAUNTLESS
(a) ungainly (b) huge (a) cautious (b) thoughtful
(c) pungent (d) crude (c) weak (d) adventurous
189. HINDRANCE 208. EXODUS
(a) agreement (b) cooperation (a) restoration (b) return
(c) persuasion (d) aid (c) home-coming (d) influx
190. JEER 209. INNOCENT
(a) mourn (b) praise (a) sinful (b) guilty
(c) mock (d) sneer (c) deadly (d) corruption
191. PROHIBIT 210. DEAR
(a) accept (b) permit (a) cheap (b) worthless
(c) agree (d) grant (c) free (d) priceless
192. ROUGHLY 211. APPOSITE
(a) exactly (b) completely (a) inappropriate (b) intemperate
(c) pointedly (d) largely (c) inconsistent (d) irregular
193. MISERLY 212. JETTISON
(a) generous (b) liberal (a) rejoice (b) surrender
(c) spend thrift (d) charitable (c) accept (d) defend
194. WONDER 213. SCOLD
(a) stock (b) amusement (a) enamour (b) rebuke
(c) expectation (d) surprise (c) criticise (d) praise
195. DENSITY 214. PODGY
(a) brightness (b) clarity (a) short (b) thin
(c) intelligence (d) rarity (c) weak (d) slim
196. CONTENTED 215. VIRTUOUS
(a) rash (b) narrow-minded (a) scandalous (b) vicious
(c) gloomy (d) disappointed (c) wicked (d) corrupt
y
o
u
C-54 Antonyms

rs
m
216. GRIM 236. CAPITULATE

a
(a) serious (b) satisfying (a) conquer (b) venerate

h
(c) delightful (d) painful

b
(c) destroy (d) surrender

o
217. DEPLETE 237. MUNDANE

o
b
(a) refund (b) replenish (a) extraordinary (b) superb

.w
(c) fulfil (d) recover (c) heavenly (d) excellent

o
218. CONSPICUOUS

rd
238. INDOLENT
(a) indifferent (b) harmless

p
(a) desirable (b) adequate

re
(c) insignificant (d) unknown (c) energetic (d) consistent

s
219. CONFESS

s
239. COSSET

.c
(a) deny (b) refuse
(a) neglect (b) divert

o
(c) contest (d) contend

m
220. HOARD (c) pamper (d) pinch
(a) deposit (b) supply 240. DOLEFUL
(c) satisfy (d) accumulate (a) poor (b) happy
221. PROVOCATION (c) rich (d) steady
(a) destruction (b) peace 241. MORTAL
(c) pacification (d) vocation (a) eternal (b) spiritual
222. FACT (c) immortal (d) divine
(a) fable (b) story 242. MASTICATE
(c) illusion (d) fiction (a) gobble (b) conceal
223. NATIVE (c) chew (d) review
(a) alien (b) foreigner 243. UNDERHAND
(c) newcomer (d) stranger (a) cruel (b) quiet
224. MODICUM (c) secret (d) open
(a) simplicity (b) a large amount 244. NEAT
(c) brazenness (d) immodestry
(a) sloppy (b) fragrant
225. MASK
(c) spruce (d) prodigal
(a) deface (b) injure
(c) expose (d) hit 245. TERSE
226. FICKLE (a) concise (b) detailed
(a) diseased (b) fast (c) expressive (d) descriptive
(c) constant (d) quick 246. ILLUSTRIOUS
227. CHOICE (a) uneducated (b) uncivilised
(a) refusal (b) dilemma (c) unintelligent (d) unknown
(c) harm (d) approval 247. PASTEL
228. DAINTY (a) urban (b) delicate
(a) splendid (b) aggressive (c) bright (d) sweet
(c) vigorous (d) towering 248. LETHAL
229. NIMBLE (a) safe (b) playful
(a) giant (b) clumsy (c) virulent (d) forgiving
(c) quick (d) frank 249. ENCOURAGE
230. RESERVED (a) warn (b) discourage
(a) likeable (b) talkative (c) dampen (d) disapprove
(c) popular (d) companionable
250. INNOCENCE
231. INVETERATE
(a) crime (b) mischief
(a) stupid (b) uneducated
(c) ignorant (d) inexperienced (c) guilt (d) sin
232. OBSOLETE 251. DIABOLIC
(a) conducive (b) rare (a) patient (b) generous
(c) useless (d) recent (c) kind (d) simple
233. INSANITY 252. FRUGAL
(a) sanity (b) normality (a) gaudy (b) generous
(c) lucidity (d) sobriety (c) extravagant (d) charitable
234. CHURLISH 253. TENTATIVE
(a) accomodating (b) polite (a) immediate (b) urgent
(c) helpful (d) happy (c) developed (d) final
235. ARTIFICIAL 254. PARALLEL
(a) solid (b) truthful (a) divergent (b) curved
(c) authentic (d) natural (c) random (d) wavy
y
o
u
Antonyms C-55

rs
m
255. STUBBORN 275. PERSUASIVE

a
(a) willing (b) consenting (a) demoralising (b) false

h
(c) pliable (d) easy (c) discouraging (d) unconvincing

b
o
256. NOVEL 276. BANISH

o
b
(a) formal (b) ancient (a) abandon (b) harbour

.w
(c) customary (d) traditional (c) intrude (d) drop

o
257. LIABILITY 277. REWARD

rd
(a) treasure (b) debt (a) demotion (b) forfeiture

p
re
(c) assets (d) property (c) penalty (d) retribution

s
258. MANAGE 278. REJECT

s
.c
(a) direct (b) avail (a) agree (b) accept

o
(c) bungle (d) sild (c) embrace (d) adopt

m
259. ILLUSORY 279. FOSTER
(a) deceptive (b) real (a) repress (b) curb
(c) imaginary (d) certain (c) check (d) control
260. ARID 280. THEORY
(a) plentiful (b) productive (a) imagination (b) fact
(c) humid (d) agreeable (c) chance (d) thought
261. DISPERSE 281. INIMICAL
(a) collect (b) assemble (a) friendly (b) cheerful
(c) hoard (d) save (c) neutral (d) emotional
262. IMPULSIVE 282. PROSCRIBE
(a) cautious (b) considerate (a) interdict (b) allow
(c) clever (d) cunning (c) extend (d) betray
263. AUSPICIOUS 283. INTRICATE
(a) spicy (b) unfavourable (a) foolish (b) sample
(c) conspicuous (d) condemnatory (c) straight forward (d) easy
264. ENGULFED 284. MEAGRE
(a) encircled (b) groped (a) average (b) plentiful
(c) disfigured (d) detched (c) extravagant (d) excessive
265. CONVEX 285. DORMANT
(a) flat (b) protuberant (a) active (b) modern
(c) full (d) indented (c) permanent (d) transient
266. LUXURIANT 286. ADHERENT
(a) barren (b) small (a) rival (b) alien
(c) ghastly (d) decaying (c) detractor (d) enemy
267. GLIB 287. EQUANIMITY
(a) unwilling (b) hesitant (a) excitement (b) duplicity
(c) dumb (d) modest (c) dubiousness (d) resentment
268. ZEST 288. TURBID
(a) restive (b) callous (a) easy (b) hazy
(c) indifference (d) distate (c) clear (d) distinct
269. BAROQUE 289. OBFUSCATE
(a) direct (b) straight (a) simplify (b) explain
(c) plain (d) strong (c) describe (d) clarify
270. REPEL 290. BEGUILE
(a) attract (b) concentrate (a) persuade (b) cheat
(c) attend (d) continue (c) flatter (d) smile
271. CAPRICIOUS 291. HAMSTRING
(a) satisfied (b) scattered (a) strengthen (b) enlarge
(c) steadfast (d) insured (c) stimulate (d) awaken
272. FACTITIOUS 292. SCEPTICAL
(a) ridiculous (b) genuine (a) inquisitive (b) hopeful
(c) engineered (d) magnificent (c) indictive (d) intuitive
273. SAGACIOUS 293. DESTINY
(a) casula (b) cunning (a) vulnerability (b) chance
(c) foolish (d) false (c) self-dependence (d) fate
274. EPILOGUE 294. PERDITION
(a) conversation (b) dialogue (a) excitement (b) reward
(c) dramatic (d) prologue (c) inspiration (d) salvation
y
o
u
C-56 Antonyms

rs
m
295. SHAME 315. CONSOLIDATE

a
(a) glorify (b) exalt (a) isolate (b) weaken

h
(c) dignify (d) enshrine (c) divide (d) identify

b
o
296. RESCUE 316. INSOLENT

o
b
(a) extricate (b) waver (a) agreeable (b) coward

.w
(c) bind (d) desert (c) polite (d) considerate

o
297. AGONY 317. LIABILITY

rd
(a) pleasure (b) bliss (a) assumption (b) exemption

p
re
(c) ecstasy (d) fear (c) consumption (d) presumption

s
298. REQUISITE 318. FABRICATE

s
.c
(a) dispensable (b) random (a) unearth (b) construct

o
(c) inappropriate (d) chaotic (c) demolish (d) renovate

m
299. VIE 319. GREGARIOUS
(a) guard (b) maintain (a) sociable (b) societal
(c) discover (d) yield (c) unsociable (d) solitary
300. FLURRY 320. PRAGMATIC
(a) disclose (b) soothe (a) indefinite (b) vague
(c) pelt (d) achieve (c) optimistic (d) idealistic
301. SUBSERVIENT
321. CULTIVATED
(a) aggressive (b) dignified
(a) crude (b) genteel
(c) straight forward (d) supercilious
302. JUBILANT (c) suave (d) refined
(a) scared (b) disturbed 322. IMPERTINENT
(c) gloomy (d) quiet (a) insolent (b) impudent
303. LUXURY (c) cheeky (d) courteous
(a) sadness (b) treachery 323. DIVULGE
(c) duplicity (d) austerity (a) disseminate (b) dissemble
304. FORBID (c) publicize (d) transmit
(a) provoke (b) appreciate 324. APPRECIATION
(c) celebrate (d) permit (a) aspersion (b) admiration
305. DEFECTION (c) commendation (d) compliment
(a) resignation (b) invitation 325. SUPPLE
(c) joining (d) co-operation (a) pliant (b) pliable
306. DUSKY (c) rigid (d) flexible
(a) visible (b) fair
326. PREDILECTION
(c) obscure (d) shadowy
(a) predicament (b) afterthought
307. PUNCTILIOUS
(a) irregular (b) fussy (c) aversion (d) postponement
(c) careless (d) curious 327. POMPOUS
308. CAPRICIOUS (a) uppish (b) humble
(a) fixed (b) solid (c) meek (d) grandiose
(c) firm (d) reliable 328. SERENE
309. SHAMEFUL (a) calm (b) angry
(a) naked (b) brazen (c) ruffled (d) bitter
(c) wanton (d) unblushing 329. SAFE
310. FLAGITIOUS (a) rash (b) insecure
(a) frivolous (b) ignorant (c) beneficial (d) harsh
(c) vapid (d) innocent 330. REDUNDANT
311. BENEVOLENCE (a) repentant (b) surplus
(a) contempt (b) malevolence (c) singular (d) required
(c) hatred (d) derision
331. FAIR
312. KNACK
(a) untrue (b) unjust
(a) dullness (b) balance
(c) coarse (d) harsh
(c) talent (d) dexterity
332. BOISTEROUS
313. CELIBATE
(a) serenity (b) calm
(a) profligate (b) reprobate
(c) cheerful (d) courageous
(c) extravagant (d) prodigal
314. DITHER 333. SUBSTANTIAL
(a) flimsy (b) hefty
(a) cry (b) refer
(c) actual (d) excess
(c) decide (d) defer
y
o
u
Antonyms C-57

rs
m
A NSWER KEY

a
h
1 (b) 41 (d) 81 (d ) 121 (b) 161 (c) 201 (c) 2 41 (c) 281 (a) 321 (a)

b
o
2 (d) 42 (b) 82 (a) 122 (c) 162 (a) 202 (c) 2 42 (a) 282 (b) 322 (d)

o
b
3 (d) 43 (a) 83 (d ) 123 (c) 163 (d) 203 (b) 2 43 (d) 283 (c) 323 (b)

.w
o
4 (b) 44 (a) 84 (d ) 124 (a) 164 (a) 204 (c) 2 44 (a) 284 (b) 324 (a)

rd
5 (a) 45 (b) 85 (a) 125 (b) 165 (b) 205 (b) 2 45 (b) 285 (a) 325 (c)

p
re
6 (d) 46 (c) 86 (b ) 126 (d) 166 (c) 206 (d) 2 46 (d) 286 (c) 326 (c)

s
s
7 (c) 47 (c) 87 (c) 127 (d) 167 (a) 207 (d) 2 47 (c) 287 (a) 327 (b)

.c
o
8 (a) 48 (a) 88 (c) 128 (d) 168 (b) 208 (d) 2 48 (a) 288 (c) 328 (c)

m
9 (a) 49 (b) 89 (b ) 129 (a) 169 (b) 209 (b) 2 49 (b) 289 (d) 329 (b)
10 (d) 50 (a) 90 (a) 130 (b) 170 (a) 210 (a) 2 50 (c) 290 (a) 330 (c)
11 (c) 51 (c) 91 (a) 131 (c) 171 (d) 211 (a) 2 51 (c) 291 (a) 331 (b)
12 (d) 52 (a) 92 (d ) 132 (d) 172 (c) 212 (c) 2 52 (c) 292 (b) 332 (b)
13 (a) 53 (b) 93 (c) 133 (d) 173 (b) 213 (d) 2 53 (d) 293 (c) 333 (a)
14 (a) 54 (d) 94 (c) 134 (a) 174 (c) 214 (b) 2 54 (a) 294 (d)
15 (b) 55 (b) 95 (c) 135 (c) 175 (b) 215 (b) 2 55 (c) 295 (b)
16 (d) 56 (d) 96 (d ) 136 (c) 176 (b) 216 (c) 2 56 (d) 296 (d)
17 (c) 57 (a) 97 (a) 137 (b) 177 (d) 217 (b) 2 57 (c) 297 (c)
18 (b) 58 (d) 98 (a) 138 (b) 178 (c) 218 (c) 2 58 (c) 298 (a)
19 (a) 59 (a) 99 (d ) 139 (a) 179 (d) 219 (a) 2 59 (b) 299 (d)
20 (d) 60 (a) 100 (d ) 140 (d) 180 (a) 220 (b) 2 60 (c) 300 (b)
21 (c) 61 (a) 101 (a) 141 (a) 181 (c) 221 (c) 2 61 (b) 301 (b)
22 (c) 62 (b) 102 (d ) 142 (b) 182 (c) 222 (d) 2 62 (a) 302 (c)
23 (c) 63 (a) 103 (c) 143 (c) 183 (b) 223 (a) 2 63 (b) 303 (d)
24 (c) 64 (d) 104 (c) 144 (c) 184 (b) 224 (b) 2 64 (d) 304 (d)
25 (b) 65 (a) 105 (b ) 145 (a) 185 (b) 225 (c) 2 65 (d) 305 (c)
26 (d) 66 (a) 106 (d ) 146 (b) 186 (d) 226 (c) 2 66 (a) 306 (b)
27 (c) 67 (b) 107 (d ) 147 (c) 187 (b) 227 (b) 2 67 (c) 307 (c)
28 (b) 68 (b) 108 (c) 148 (c) 188 (a) 228 (c) 2 68 (d) 308 (d)
29 (d) 69 (a) 109 (d ) 149 (d) 189 (d) 229 (b) 2 69 (c) 309 (b)
30 (b) 70 (c) 110 (c) 150 (b) 190 (b) 230 (b) 2 70 (a) 310 (d)
31 (a) 71 (a) 111 (d ) 151 (c) 191 (b) 231 (d) 2 71 (c) 311 (b)
32 (b) 72 (d) 112 (a) 152 (d) 192 (a) 232 (d) 2 72 (b) 312 (a)
33 (d) 73 (a) 113 (a) 153 (a) 193 (c) 233 (a) 2 73 (c) 313 (b)
34 (a) 74 (a) 114 (a) 154 (c) 194 (c) 234 (b) 2 74 (d) 314 (c)
35 (d) 75 (b) 115 (d ) 155 (b) 195 (d) 235 (d) 2 75 (d) 315 (b)
36 (c) 76 (b) 116 (c) 156 (d) 196 (d) 236 (a) 2 76 (b) 316 (c)
37 (b) 77 (c) 117 (b ) 157 (d) 197 (b) 237 (c) 2 77 (c) 317 (b)
38 (b) 78 (d) 118 (b ) 158 (c) 198 (c) 238 (c) 2 78 (b) 318 (a)
39 (c) 79 (d) 119 (a) 159 (c) 199 (d) 239 (a) 2 79 (a) 319 (c)
40 (b) 80 (c) 120 (d ) 160 (a) 200 (d) 240 (b) 2 80 (b) 320 (a)
y
o
u
C-58 Antonyms

rs
m
a
h
b
o
o
b
.w
o
1. (b) Exonerate means declared free from blame, to implicate 24. (c) Homogenised means when every part of the group,

rd
means to show that someone is involved in something mixture or collection has been made similar or same,

p
re
especially a crime, to appease means to pacify, to adore variety means the opposite, i.e difference.

s
means to love deeply and respect highly. 25. (b) Prove means to state that a statement or theory is

s
.c
2. (d) Fascinating means charming, fanaticism means correct after giving valid and logical reasons while

o
m
madness, especially in religious or political matters, assumption is something which is believed without
indolence means laziness. any proofs or evidence.
3. (d) Modest means humble, awkward means clumsy, 26. (d) Accolade is a token of respect and admiration or kind
celebrated means distinguished. of reward given to appreciate, criticism is to find faults.
4. (b) Credulity 27. (c) Instantly means something that happens in an instant
5. (a) Roost or very quickly, gradually means to happen step by
6. (d) offend means not to show proper respect or make step. slowly has not been considered as the apt choice
someone angry, so the opposite will be respect. because instantly suggests in one step and gradually
7. (c) Indict means to charge someone with a crime or means in several steps so it is a more appropriate
something wrong. Accuse also means the same, acquit antonym.
means to free of charge. 28. (b) Frail means something or someone very weak, usually
8. (a) Obscene means in decent, objectionable and due to illness.
condemnable mean similar to obscene. 29. (d) Crude means raw, undeveloped or unsophisticated,
9. (a) Liberate is to make someone free and imprison is to graceful can also be an antonym to crude but polished
put in prison, conceal is to hide and vacillation is to is more exactly opposite to crude.
move from one place or idea to another. 30. (b) Retribution is punishment, contempt is feeling of
10. (d) Melancholy means depressed and is therefore opposite disgust and grudge is an ill-feeling.
to cheerful. 31. (a) Proclaim is to announce usually in favour of, denounce
11. (c) Legitimate means legal or lawful, distinguished means means to speak against.
eminent, and courteous means polite. 32. (b) Sumptuous means sufficient or more in quantity,
12. (d) Vacillate means to move from one place to another and meagre means very little.
consistent means to stay the same, eradicate means to 33. (d) Feign means to pretend, that is show what is not real
remove. or original, condone is to forgive and condemn is to
13. (a) Hinder means to prevent the growth or progress of express disapproval of something.
something while expedite is to make fast the process 34. (a) Insipid means unintelligent, plain or flat, witty means
or facilitate. Vindicate means to justify. clever intelligent.
14. (a) Terse means brief. Diffuse means spread widely or 35. (d) Salubrious means health giving, malaise is something
thinly. that causes discomfort or pain, like a disease.
15. (b) Multiplicity is presence of multiple or many identities, 36. (c) Refulgent means bright, lament is to repent.
while uniformity means same everywhere, infinite 37. (b) Innocuous is harmless or inoffensive.
means endless. 38. (b) Affectation means haughtiness or pride while humility
16. (d) Naive means inexperienced, someone who will not be means humble, polite behaviour.
very sharp or clever owing to lack of experience. 39. (c) Luminous means bright or giving out a lot of light.
17. (c) Anachronistic is something that belongs to a time gone 40. (b) The carpets are priced so high due to the intricacy of
by, while futuristic is something that belongs to the patterns. An intricate pattern is made up of many small
future or suggests how the future would be. details. It is complicated, not simple. The noun from
18. (b) Trait means characteristic. simple is simplicity.
19. (a) Garish means gaudy or in bad taste. 41. (d) If you ameliorate a situation, you make it better or easier
20. (d) Geological means belonging to or related to the earth, in some way. The opposite of better is worse. If you
while galactic is that which belongs to the galaxy. make a situation worse, you worsen it.
Heavenly is belonging to heaven so opposite to 42. (b) The fielding of the Indian cricket team is lackadaisical.
earthly. Heavenly is also called unearthly. That is, the fielders do not show much interest or
21. (c) Rejuvenation means rebirth and regeneration or to live enthusiasm; they are rather lazy. Jadeja, however, is an
again. exception. He is Energetic.
22. (c) Origin means the starting point and remnant is what is 43. (a) He was fed up with the capricious behaviour of his
left after finishing or distrubution. wife. It began to prove difficult for him to fulfil her
23. (c) Apathetic is one who does not care or is indifferent. caprices. A caprice is an unexpected action or decision;
y
o
u
Antonyms C-59

rs
m
It has no strong reason or purpose. In fact, the principles of holism. Holism is the belief that everything

a
husband could not tolerate her. He was a thoughtful in nature is connected in some way. A piecemeal

h
approach, on the other hand, deals with only one part

b
man; his actions were based on reason.

o
44. (a) One who is perfidious is treacherous or untrustworthy. at a time.

o
b
He may be with you today and against you tomorrow. 56. (d) There has been an extension of STD facility to 2000

.w
On the other hand, a local person remains Firm in his more villages. In other words, there has been an

o
or her friendship or support. addition to the number of villages under STD coverage.

rd
Now, the opposite of addition is subtraction.

p
45. (b) Ennui is a feeling of tiredness, boredom and

re
dissatisfaction caused by lack of interest and having 57. (a) One who is indigent is very poor. The opposite of

s
poor is rich.

s
nothing to do. Only activity (something to do) can be

.c
its antidote. [An antidote is a substance that prevents 58. (d) Audacity is audacious behaviour. If you are audacious,

o
m
the bad effects of a disease.] you take risks in order to achieve something. So it is a
46. (c) One who is lascivious shows an unnaturally strong kind of bravery. Now, the opposite of bravery is
interest in sex. On the other hand, a chaste person cowardice.
does not have sex with anyone, or has it only with his 59. (a) An elevation is a ‘piece of ground that is higher
or her spouse. (elevated) than the area around it. A depression in a
47. (c) Rachna is very conscientious about her actions. That surface is an area which is lower (depressed) than the
is, she is very careful to do her work properly. Now, the parts surrounding it.
opposite of careful is careless. 60. (a) Cows are sacred to the Hindus. Killing of cows is a
48. (a) The perigee is the point where the path of an object profane act for them. That is, they consider the act to
through space is closest to the Earth. The apogee is be a disrespect for the religion, and therefore sinful.
the point where the path is farthest from the Earth. Pious (very religious and moral) Hindus would plug
49. (b) How do you rectify voltage fluctuations ? With the their ears even if they heard of it.
help of a voltage stabiliser. If something fluctuates, it 61. (a) The Vedas are unfathomable; that is, one cannot fully
changes a lot in an irregular way. If you stop this, it understand the depth of their meaning. The
becomes stable. commentaries, however, are simplified versions easy
50. (a) Radical comes from Latin radix (= root). If you make a to understand. In other words, they are comprehensible.
62. (b) The controversy led to a termination of the contract.
radical change in something, the change goes to its
That is, the contract came to an end. The opposite of
very roots, thus affecting the entire thing. On the other
End is beginning.
hand, there are changes which do not disturb the entire
63. (a) My success is due to my elder brother, who has
thing; the changes take place only on the surface. In
inspired me at every point. That is, he encouraged me
other words, they are superficial changes.
to go ahead with my work. The opposite of encouraged
51. (c) Four of the judges were in accord with one another
is discouraged.
while one dissented. That is, four of them were in
64. (d) Only 76 countries participated in the meet. The rest
agreement with one another on the judgement. There
boycotted it. That is, they refused to participate (take
was no conflict between their opinions. One judge,
part) in it because they disapproved of it.
however, did not share the majority opinions. He was
65. (a) Some people believe that reservations are against the
of a different opinion. In other words, he dissented. idea of an egalitarian society. An egalitarian society
52. (a) All reservations have been abolished from this year! supports or follows the idea that all people are equal;
Hari Paswan, a scheduled caste candidate, has fallen a they should have the same right and opportunities. So
hapless victim to the new law. That is, he was unlucky why give special rights to some? Others, however, feel
not to have taken the exam last year. His elder brother, that the society, as it stands today, is unequal. Only
Barkhu, was lucky enough to have made it last year. reservations can bring an equilibrium and make the
53. (b) Farming on such a barren land! It is a frivolous project. society egalitarian.
That is, it would only entail a wastage of time and 66. (a) A deficiency of iodine leads to goitre. In other words,
money. It is not useful. It would make no significant iodine is riot present in enough quantity. On the other
(important) contribution to ameliorating the plight of hand, if it be present in large quantities, that is more
the poor. than enough (usually), we say it is in abundance.
54. (d) Kashmir is an integral part of India. That is, it is an 67. (b) The economy flourishes when there are lesser
essential part of India without which the country restrictions. That is, it grows quickly and strongly.
cannot be complete. It cannot be separated from the Imposing of restrictions makes the growth of the
country. That is what most of the people believe. economy retard. That is, it gets slower, as if under
However, a handful of people believe it should be brakes.
independent - that it has no connection whatsoever 68. (b) The veneer of something is the coating from outside.
with the rest of India. It is the exterior which often gives a false impression
55. (b) Holistic medicine treats the whole person, not just the of the reality inside. The inside of something is its
diseased part. That which is holistic is based on the interior.
y
o
u
C-60 Antonyms

rs
m
69. (a) What happens in a dictatorship? All the decisions are town, or area. They will be admitted to the House again

a
made by the dictator — a ruler who has complete power when it gets repaired.

h
in a country. In effect, it becomes a one-man rule. 83. (d) We are sanguine about the outcome of this dispute. That

b
o
Countries like India, however, believe in a rule “by the is, we are cheerful and confident that things will turn in

o
b
people”. In other words, we believe in democracy. In a our favour. Now, the opposite of cheerful is gloomy.

.w
democracy, power is not centred in one person. The 84. (d) If a person is very fat, you euphemistically call him or

o
ruler is merely the representative or the people. her corpulent. Emaciated, on the other hand, means

rd
70. (c) Eventually means “at the end”; initially means “in the extremely thin or weak because of illness or lack of food.

p
re
beginning”. 85. (a) Occidental means “of the West”. That is, it relates to

s
71. (a) Dhirubhai Ambani has had a phenomenal (unusually the countries of Europe and America. The Britishers

s
.c
big) success. Such a success is not usually had by brought with them Occidental culture, which greatly

o
m
ordinary persons. influenced the Oriental values. Oriental means “of the
72. (d) A nonchalant person seems not to worry or care about East”, and is associated with eastern Asia, especially
things. He or she seems very calm. Not at all excited. China and Japan.
73. (a) If you abominate something, you hate it very much. 86. (b) If a part of someone’s body is flaccid, it is unpleasantly
The opposite of hate is love. soft, and not hard or firm. On the other hand, if
74. (a) An ingenuous person is innocent, trusting, and something is tout, it is stretched very tight. A taut
incapable of deceiving anyone. He or she lays bare body is very lean with firm muscles.
their tricks. Will a cunning person ever do that? 87. (c) If Mr. Mhatre comes to power, his son-in-law will easily
75. (b) That which is egregious is shocking, extremely had. If get the licence for a petrol pump. This is what is called
something is extremely good, you call it splendid. nepotism. That is, power is used unfairly in order to get
76. (b) Arjuna Ranatunga does not dissipate his energy by jobs or other benefits for your family or friends. In other
running fast every time he takes a run. That is, he does words, nepotism promotes partiality towards your near
not waste it in a foolish way. When he sees that the and dear ones. The antonym, therefore, is impartiality.
ball is distant enough for him to easily reach the crease 88. (c) If you are abstemious, you avoid doing too much of
for a single, and that no matter how fast he runs he something enjoyable. For example, you eat or drink in
cannot take more than a single, he simply ambles moderation. The antonym therefore will describe a
across the pitch. He thus conserves (saves) energy person who goes for physical pleasures in excess. Such
and utilises it when there is need for a quick run. a person is dissipated. He or she spends a lot of time
77. (c) Let him play with this table; it is sturdy enough to bear drinking alcohol and enjoying other physical
his kicks. That which is sturdy looks strong; it is pleasures, and is probably unhealthy because of this.
unlikely to be easily injured or damaged. The antonym 89. (b) A chimera is an imaginary monster. So the opposite of
of strong is weak. chimerical is real.
78. (d) You think you will persuade him to live a comfortable 90. (a) Verbose comes from Latin verbum (= word). A verbal
life? Impossible! A Spartan way of life is sacrosanct test is thus associated with words. The non-verbal
for him. If something is sacrosanct, it is too holy or section does not deal with words; its questions are
important to be allowed any harm. The opposite of based on figures. Now, verbose means wordy. So if
holy is unholy. you are verbose, you use too many words, more than
79. (d) One who is celibate does not marry or have sex, because what is necessary. On the other hand, laconic reply
of one’s religious beliefs. (1), (3) and (4) are close uses as few words as possible.
choices. But we reject (1) and (3) because they have 91. (a) Diminutive comes from Latin minuo (= lessen). A minute
other meanings too. Profligate and reprobate give a particle is very small. If something diminishes, it
general sense of wastefulness. Such persons may be becomes smaller. A diminutive object is thus very small.
given to vices, sex being one of them. Thus (1) and (3) Obviously, the antonym is enlarged.
can be focussed as an antonym only in absence of (4). 92. (d) When one’s immunity to a disease is over, one becomes
A married man, on the other hand, is categorically not susceptible to it. That is, one is then very likely to be
celibate. affected by it.
80. (c) Recent advances in technology have rendered older 93. (c) Chemistry students should be familiar with amalgam.
equipment’s obsolete (no longer in need). Amalgam is a mixture of mercury and another metal,
81. (d) A magnanimous person has a “‘big heart”. Well, not usually silver, that is used in dentistry to make fillings.
literally. What I mean is that he or she is filled with love In general, an amalgam is a mixture of two or more
for everyone. The pressures of modern live have made things. So when two or more organisations amalgamate
magnanimous persons an endangered species. It is they become one large organisation. When their
getting more and more difficult to think beyond one’s relations deteriorate, they separate.
self in other words, people are getting selfish. 94. (c) A captious person is too critical, too ready to find
82. (a) The residents of Katynyan House have been fault. He or she cannot tolerate anything that goes
evacuated. If you evacuate people, you send them to even a fraction against them. Such persons are not
a place of safety, away from a dangerous building, tolerant at all.
y
o
u
Antonyms C-61

rs
m
95. (c) The difficulties are manifold. That is, they are of many 109. (d) Was it appropriate for a man of your calibre to work at

a
different kinds. If so, they simply cannot be simple. so low a remuneration? I don’t think so. The money

h
96. (d) If something warps, it becomes damaged by bending that you got did not suit the talent you had. In other

b
o
or curving, often because of the effect of heat or water. words, it was unsuitable.

o
b
In order to bring it back to as it was, you have to make 110. (c) That which is wild lacks discipline and control. When

.w
it straight. In other words, you have to straighten it. it is brought under control, it becomes tamed.

o
97. (a) That which is jocose is meant to cause amusement. If 111. (d) If you are subservient, you do whatever someone

rd
it does not do so, it is dull. wants you to do. On the other hand, if you are

p
re
98. (a) If you pamper someone, you pay too much attention supercilious, you behave in a scornful way towards

s
to make him comfortable and happy; you treat him too

s
others. You think you are superior to them. So, you

.c
kindly. On the other hand, if you do not pay attention think, they should be at your command, not you at

o
to someone, you neglect him.

m
theirs.
99. (d) A good book should not show negligence in editing. 112. (a) Seven men were charged of a murder. Six of them
No part of it should be neglected. It should be read admitted that they were a party to the murder. In other
and re-read, with corrections, till it becomes words, they confessed their guilt. But the seventh man
presentable. A good editor is meticulous about every
denied. He said, “No, I know nothing about the murder.
word that goes into the book. That is, he does things
The charge is totally baseless.
very carefully and with great attention to detail.
113. (a) Usually, at this time of the year, I would be engaged in
100. (d) Instantly means “at once, without any delay”. Its
debates and dramas. That is, I used to be actively
antonym therefore will be slowly. If you are tempted to
say lately, take care! If you have the meaning “with involved in them. This year, however, I have my Board
delay” in mind, note that this meaning is given by the exams. So I am abstaining (keeping away) from them,
word late. Lately (also of late) means – in the recent 114. (a) He must have lost if he stood for the elections. Not to
past, not long ago stand was thus a judicious move. That is, the move
101. (a) If something is obvious, it is clearly seen. If it is obscure, showed good judgement on his part. Had he stood for
it is hidden. the elections, it would have shown a lack of careful
102. (d) Some women are extraordinary in their beauty. That is, thinking and proper judgement. The move would then
their beauty surpasses the ordinary level. Not every have been imprudent.
beautiful woman can attain that degree of beauty. They 115. (d) An uproarious laughter is very noisy. The opposite of
are just plainly beautiful. noisy is calm.
103. (c) Computers are now spreading in every part of the world. 116. (c) I helped both Ravi and Santosh equally. Ravi was
Doomsayers believe they are a threat to mankind. So grateful. He said to me, “Thank you. Thank you very
they wish to check its spread. If you check something much. I will be indebted to you for this support.” On
from spreading, you contain it. the other hand, Santosh took the support for granted.
104. (c) The ‘No Smoking’ sign prohibits you from smoking at His work being done, Santosh didn’t even look at me.
that place. That is, it does not allow you to smoke. On I can’t even think of any appreciation from him. He
the other hand, if someone allows you to smoke, he was simply unappreciative of my support.
permits you to do so. 117. (b) In a highly competitive world, you can’t succeed unless
105. (b) Human beings often show ruthless behaviour towards you toil (work hard). But sloth gets the better of
animals. Such a behaviour shows a complete lack of Avinash; so success eludes him forever. Sloth is
pity and sympathy. Organisations such as the Society
laziness, especially with regard to work.
for Prevention of Cruelly to Animals (SCPA) ask you
118. (b) FAO has warned India of the indiscriminate use of
to be compassionate towards them. That is, they ask
pesticides. That is, farmers are using pesticides without
you to show pity and sympathy towards them, to show
understanding for their suffering. giving any careful thought or exercising a careful
106. (d) Chagrin is a feeling of annoyance or disappointment. choice. Now, pesticides are not meant to be sprayed in
If something happens to your chagrin, you are not every field and without any restrictions. A good user
pleased. must carefully select the fields that really need them.
107. (d) When a man begets a child, he becomes its father. If In other words; he should be selective.
you beget something, you cause it to happen or be 119. (a) If you are fickle, you keep changing your mind. That
created. On the other hand, if something is stopped in happens when you are unsure of purpose. When you
the process of its creation, it is aborted. We are familiar have a concrete target in mind, you become steadfast.
with abortion — the deliberate ending of a pregnancy. That is, you are convinced that what you are doing is
108. (c) Mohan leads an austere life. He eats simple food, does right. You refuse to change your mind.
not have furniture, and prefers walking to riding a car. 120. (d) The mess bills were brought out in time. But the tardy
He thus saves money and sends it to his son. His city- students are yet to pay the bills. The result: there is no
dwelling son lives lavishly on that money. He throws fund to run the mess. Unless the students become
parties every week, wears the best of designer wear, prompt in payment, we shall have to face the crunch
and travels mostly by air. again and again.
y
o
u
C-62 Antonyms

rs
m
121. (b) A fable is a false story or account; a fact is a true 130. (b) When the doctor gives you a sedative, what happens?

a
account. It makes you sedate (calm, quiet). It prevents you from

h
getting excited

b
122. (c) One who is affectionate shows affection towards you.

o
Affection is love. The opposite of love is hate. Now, 131. (c) After three days of massive destruction, the cyclone

o
b
who can hate you? Not a friend, but an enemy. In other has now abated. That is, it has become much less

.w
words, one who is hostile towards you. strong. Its intensity has decreased. Now, the antonym

o
of decrease is increase.

rd
123. (c) (a) is rejected because ignorance is the opposite of
318. (a) Fabricate means make up something artificial or untrue

p
knowledge. Now, knowledgeable is not the same as

re
erudite. Knowledge can come from any source. There while demolish means destroy completely which is just

s
opposite.

s
are knowledgeable persons who have never touched

.c
319. (c) Gregarious denotes tending to form a group with others

o
a book in their lives. Erudition, on the other hand, is

m
of the same species and unsociable is the opposite.
scholarly learning for which books are essential. An 320. (a) Pragmatic means concerned with practical matters while
erudite person is thus highly literate. An illiterate indefinite denotes vague or not clearly defined or stated.
person is one who does not know how to read. 321. (a) The meaning of word cultivated (Adjective) is :
124. (a) If you are pretentious, you have a desire to show off Eductated.
On the other hand, if you are unassuming, you do not It’s antonym should be : Crude.
wish to be noticed. 322. (d) The meaning of word Impertinent (Adjective) is : Ill
125. (b) A poltroon is a coward. One who is brave is a hero. mannered, disrespectful.
126. (d) Sati was abolished by William Bentick. That is, he It’s antonym should be : courteous.
brought Sati to an end. When you bring something 323. (b) The meaning of word Divulge (verb) is : Reveal, make
into being, you establish it. known.
127. (d) What is retaliation? If someone slaps your cheek, you Its antonym should be : Dissemble.
slap his in return. Thus, enmity breeds enmity. And a 324. (a) The meaning of word Appreciation (Noun) is :
vicious cycle gets started. Both of you become Thankfullness.
enemies. There are others who prefer reconciliation — It’s antonym should be : Aspersian.
becoming friends again after a quarrel. They make 325. (c) The meaning of word Supple (Adjective) is : Flexible.
compromises and iron out their differences. It’s Antonym should be : Rigid.
128. (d) On the one hand, we have powers that abet terrorism. 326. (c) The opposite of 'predilection' is 'aversion'.
That is, they help its growth. On the other hand, there 327. (b) The opposite of 'pompous' is 'humble'.
are forces that check its growth. In other words, these 328. (c) The opposite of 'serene' is 'ruffled'.
forces prevent its growth. 329. (b) Safe means free from harm. Its opposite is insecure.
129. (a) The manner in which our leaders conduct themselves 330. (c) Redundant means excessive. Its opposite is singular.
in the parliament is reprehensible. It deserves to be 331. (b) Fair means impartial. Its opposite is unjust.
condemned. However, some of them still behave 332. (b) Boisterous means noisy and mischievous. Its opposite
decently and deserve to be praised. In other words, is calm.
their behaviour is commendable. 333. (a) Flimsy means insubstantial and easily damaged.
y
o
u
rs
m
a
CHAPTER

h
b
5

o
IDIOMS/PHRASES

o
b
.w
o
rd
p
re
s
s
.c
o
Some Commonly Used Idioms To give someone the slip (to dodge someone who is looking

m
Beat back (to compel to retire) : The firemen were beaten for you) : The police had nearly got the dacoits when the
back by angry flames and the building was reduced to ashes. latter gave them the slip in the Chambal ravines.
Boil down to (to amount to) : His entire argument boiled To go on a fool’s errand (to go on an expedition which
down to this that he would not join the movement unless leads to a foolish end) : Many people earlier believed that
he saw some monetary gain in it. going to the moon was like going on a fool’s errand
Cast aside (to reject, to throw aside) : Men will cast aside To go to the wall (to get the worst in a competition) : In the
truth and honesty for immediate gains. struggle of life, the weakest goes to the wall.
Cry down (to deprecate) : Some of the Western powers did To go to rack and ruin, to go to the dogs (to be ruined) : If
their best to cry down India’s success in the war. a big war comes, our economy will go to the dogs.
To have one’s hands full (to be very busy) : Pakistan could
To cut off with a shilling (to give someone a mere trifle in
hardly expect active help from the U.S.A. as her hands were
the will) : The father was so angry with the son over his
already full with Vietnam, Laos and West Asia problems.
marriage that he cut him off with a shilling.
To have a bone to pick with one (to have a difference with
Egg on (to urge on) : Who egged you on to fight a
a person which has not yet been fully expressed) : The
professional boxer and get your nose knocked off?
extreme leftists have a bone to pick with the police and if
Gloss over (explain away) : Even if you are an important
ever they come to power there may be unpleasantness
person your faults cannot be glossed over.
between the two.
To laugh in one’s sleeves (to be secretly amused) : While I
To have the whip hand of (to have mastery over) : After the
was solemnly reading my research paper to the audience,
split in the party Mrs. Gandhi has the whip hand of the
my friends were laughing in their sleeves for they knew
Congress.
what it was worth.
To have too many irons in the fire (to have so much work
Play off (to set one party against another for one’s own
in hand that some part of it is left undone or is done very
advantage) : It best serves the interests of the super powers
badly) : Let the Government not go in for nationalisation so
to play off one poor nation against another.
fast. If they have too many irons in the fire they are bound
Pull one through (to recover, to help one recover) : Armed
to fare badly.
with the latest medicines, the doctor will pull him through. To have the tree or right ring (To be genuine) : Nixon’s
Cost a slur upon (by word or act to cast a slight reproach pronouncements on world peace do not have the right ring.
on someone) : Many a man casts a slur on his own good To have two strings to one’s bow (to have an alternative
name with some mean act. means of achieving one’s purpose) : A wife always has two
To catch a Tartar (to encounter a strong adversary) : When strings to her bow if coaxing fails to achieve the desired
Hitler marched in to Russia he little knew that he would end; tears succeed.
catch a Tartar in the tough people of that country. To have an axe to grind (have personal interests to serve) :
To come off with flying colours (to come out of a conflict Bigger nations supply arms to the smaller ones primarily
with brilliant success) : The 1971 election outcome was because they (the bigger nations) have their own axe to grind
uncertain but finally the congress came off with flying To keep the wolf from the door (to keep away extreme
colours. poverty and hunger) : Lakhs in India have to struggle
To come off second best (to be defeated in every contest) : everyday to keep the wolf from the door.
Be it an election or a tambola, I have always come off the To make short work of (to bring to sudden end) : The locusts
second best. made short work of the ripe standing corn.
To cut the Gordian knot (to remove a difficulty by bold or To make amends for (to compensate for damage) : By his
unusual measures) : The Parliament threw out the Bill for kindness today he has made amends pr his past insolence.
Abolition of Privy Purses. The Government cut the Gordian To make common cause with (to unite, to co-operate with) :
knot by abolishing the privy purses through an ordinance. During the last elections the princes made a common cause
To fall to one’s lot (to become one’s fate) : It fell to the lot of with the rightist parties. Both went down.
Mujib and his colleagues to reconstruct the shattered To make a virtue of necessity (to do a very disagreeable
economy of their nation. thing as though from duty but really because you must do
To get into hot water (to get into difficulty) : The it) : When a minister knows that he is going to be booted
businessman got into hot water with the Income-tax out of the cabinet he makes a virtue of necessity and resigns
authorities for concealing his income from ancestral property. on health grounds.
y
o
u
C-64 Idioms/Phrases

rs
m
To make much ado about nothing (make a great fuss about To bear the brunt of (to endure the main force or shock of) :

a
a trifle) : Demonstrations and protests over the change in The infantry has to bear the brunt of a battle.

h
the timing of news bulletins over AIR was making much To beard the lion in his den (to oppose someone, in his

b
o
ado about nothing. stronghold) : The Indian Army broke through strong

o
b
To make a cat’s paw or a tool of someone (to use someone Pakistani fortifications in the Shakargarh area and bearded

.w
as a means of attaining your object) : The super-powers the lion in his own den.

o
have made a cat’s paw of the smaller nations of Asia in their To bid fair to (to give fair prospect of) : His health is so

rd
game of power politics. good that he bids fair to live till he is sixty.

p
re
To play into the hands of someone (to act as to be of To blow one’s own trumpet (to parade one’s own good

s
advantage to another) : By raising the slogan ‘Indira Hatao’ deeds) : Modesty does not pay. Only if you blow your own

s
.c
the opposition played into her hands and Mrs. Gandhi trumpet, you can succeed.

o
won the elections hands down (easily). To blunt the edge of (to make something less effective) :

m
To play second fiddle’ (to take a subordinate part) : With Time blunts the edge of grief.
Mrs. Gandhi as the undisputed leader of the Congress and To build castles in the air (to indulge in reveries or
the nation, everyone else is content to play second fiddle visionary schemes) : There is nothing wrong if you build
to her. castles in the air; now put foundations under them.
To put the cart before the horse (to begin at the wrong end to To burn the candle at both ends (to use too much energy) :
do a thing) : Preparing the blue print of a project without the Our resources are limited. Let us use them judiciously and
provision of funds is like putting the cart before the horse. not burn the candle at both ends.
To put one’s shoulder to the wheel (to make great efforts To buy a pig in a poke (to purchase a thing without previously
ourselves) : No amount of foreign aid will pull us out of the examining it) : Buying shares in a new Company started by
economic morass; we have to put our own shoulders to the unknown entrepreneurs is like buying a pig in a poke.
wheel. To cross or pass the Rubicon (to take a decisive step
To set store by (to value highly) : India, surely sets much forward) : The Government will have to think of many things
store by the Indo Soviet Treaty of Friendship. before nationalising the textile industry for once they cross
To set the Thames on fire (to do something extraordinary) : He the Rubicon there will be no going back.
is a steady worker but never likely to set the Thames on fire.
To cry over spilt milk (to nurse unnecessary regrets) : We
To set one’s house in order (to arrange one’s affairs) : Let
have failed to build up a sizeable total against England’s
Pakistan set her own house in order before talking of the
meagre first innings total. It is no use crying over spilt milk
welfare of the Kashmiris.
now.
To take into one’s head (to occur to someone) : The Manager
To err on the safe side (to choose a course which may in
look it into his head that by shutting off the electricity for
fact be inaccurate, but which will keep you safe from risk or
a few hours daily he could save on refrigeration costs.
harm) : In going in for mixed economy rather than wholesale
To take the bull by the horns (to grapple with a problem
nationalisation the Government were erring on the safe side.
courageously instead of avoiding it) : There is no short cut
to prosperity. We have to take the bull by the horns and To flog a dead horse (waste one’s energies) : We are
make people work like slaves. flogging a dead horse if we are trying to make Sanskrit the
To take a leap in the dark (to do a hazardous thing without national language of India.
any idea of what it may result in) : You took a leap in the To feather one’s nest (to provide for oneself through
dark in going into partnership with that man. dishonest means) : Many tax collectors make a point of
To throw cold water upon (to discourage something) : The feathering their own nests well while they have opportunity.
doctor threw cold water upon my plans for a world tour by To Eat one’s heart out (to brood over one’s sorrows or
declaring that I could never stand the strain of it. disappointments) : Don’t eat your heart out over failure in
To throw up the sponge (to give up a contest) : Faced with this competition.
stiff competition from big companies, many a small company To eat humble pie (to have to humiliate oneself) : Since
will throw up the sponge. none came to his support he had to eat humble pie and give
To turn over a new leaf (to change one’s course of action in to their demands.
completely) : After a long career of crime the convict To eat one’s words (to retract one’s assertions under
suddenly turned over a new leaf and became a model citizen. compulsion) : It is hard for a haughty man to have to eat his
To turn tail (to retreat ignominiously) : The enemy turned words.
tail in the face of heavy onslaughts on its key positions. To throw down the gauntlet, to take up the gauntlet (to
To turn the tables (to reverse someone’s success or offer or give a challenge, to accept a challenge) : It is not for
superiority) : Pakistan started war with a blitz on our a small country to throw down the gauntlet to the right and
positions but the superior tactics of our Armed Forces soon the left.
turned the tables on them. To run the gauntlet (to undergo severe criticism or ill
To cook or doctor an account (to tamper with or falsify the treatment) : Most trend-setting books have to run the
account) : From the balance sheet presented to the gauntlet of the literary critics.
shareholders, the company seemed to be flourishing, but it To burn one’s fingers (to get oneself into unexpected
afterwards turned out that the Secretary had cooked the trouble) : They were happily placed in the woollen industry.
accounts. But they went in for cosmetics and burnt their fingers.
y
o
u
Idioms/Phrases C-65

rs
To rule the roast or roost (to lord it over others in a party or

m
To force one’s hands (to compel one to do something

a
unwillingly or earlier than he wished to do it) : The group) : In almost every party there is some overbearing

h
Government wanted to do all that they could to meet the person who tries to rule the roost.

b
o
workers’ demands. But the violence by the strikers forced To run in the same groove (to move forward on the same

o
path, to advance in harmony) : It is clear that the ideas of

b
their hands to declare a lockout.

.w
To haul over the coals (to scold a man, reprove him) : If both reformers run in the same groove.

o
your bad habits become known, you will get hauled over To run in the blood (a peculiarity which clings to certain

rd
the coals and richly deserve it. families) : Snobbery runs in the blood of the Englishmen.

p
To scatter to the winds (to waste, to scatter abroad) : We

re
To let the grass grow under your feet (to be inert and passive
have scattered to the winds what we had gained by our

s
to things around) : The authorities should listen to students’

s
independence.

.c
grievances. By being indifferent they would only let the To be on the right scent (to be on the right track) : The

o
grass grow under their feet till it will be too late to turn

m
customs have decided to patrol the Kerala seas to nab
these young people take away from the path of violence. smugglers from Dubai. They are on the right scent (Its
To put in a nutshell (this is said of a thing which is capable, opposite is to be on the wrong scent or wrong track)
of, or presented in, brief expression) : His conduct is weird. To see how the wind blows (to observe what influence,
To put in a nutshell be is insane. The explanation of his favourable or adverse, is likely to affect the existing state of
conduct can be put in a nutshell - he is insane. things) : In party-politics people sitting on the fence keep
To let loose the dogs of war (to set in motion the destructive on watching how the wind is blowing before deciding on
forces of war) : Pakistan has let loose the dogs of war in their options.
Kashmir, through organised terrorism. To see a thing through coloured glasses (to regard
To lord it over someone (to domineer over someone, to act something favourably because of one’s prejudice) : Pakistan
as a lord) : The love of power is so strong in human nature, has for long looked at India through coloured glasses and
that when a man becomes popular he seeks to lord it over never trusted even the most genuine gestures for peace.
his fellows. (The world is a place of strife and one should not see it
To mind one’s Ps and Qs (to be punctilious) : The manager through coloured glasses.)
suspects his chief clerk of dishonesty, and if the clerk does To show the white feather (to show signs of cowardice) :
not mind his Ps and Qs, he will soon find himself without a The agitators shouted and gesticulated but the moment the
police appeared on the scene they seemed to show the
job.
white feather.
To muster in force (to assemble in large numbers) : The
To sow broadcast (to scatter widely or without stint) : The
citizens mustered in force to welcome their beloved leader. emissaries of the banished king were sowing sedition
To pay one back in one’s own coin (to give tit for tat, to broadcast.
retaliate) : Howsoever revengeful you may be, unless you To split hairs (to make subtle and useless distinctions) : As
are strong enough you cannot pay him back in his own the drought played havoc in Bihar, the authorities were
coin. busy splitting hairs trying to decide whether it was ‘scarcity
To plough a lonely furrow (to work without help or support) : conditions’ or famine.
In the organised society of today no individual or nation To steal a march (to gain an advantage over another
can plough a lonely furrow. stealthily) : While we were still debating the desirability of
To poison the ears or mind (to prejudice another person) : joint ventures with foreign concerns, Singapore and
A judge must not allow anyone to poison his mind against Malaysia stole a march over us and opened their gates to
either the plaintiff or the defendant. foreign investment in a big way.
To rest on one’s laurels (to rest satisfied with honours already To steer clear of (to avoid) : India decided on non-alignment
won, and to make no attempt to gain further distinction) : to steer clear of the hazards of alignment with one block or
Even if he wins the biggest award, a film star will never rest the other.
on his laurels. He will try to rise higher and higher. To stick at nothing (the phrase implies readiness to stoop
To rest on one’s oars (to suspend efforts after something to baseness or deception to reach one’s end) : An ambitious
has been attained) : The agitators have been vigorously at politician will stick at nothing if he can only serve himself.
To strain every nerve (to use one’s utmost efforts) : We
work during the winter, but at present they seem to be resting
have to strain every nerve to get over the poverty line.
on their oars.
To strike while the iron is hot (to take advantage of the
To harp on the same string (to keep repeating the same opportunity when it arises) : If you want to succeed in life,
sentiment over and again) : This gentleman keeps harping you must strike the iron while it is hot. In going in for
on the same string: he is from Oxford and deserves this and general elections immediately after the war, the Congress
deserves that etc. struck while the iron was hot.
To rise like a phoenix from its ashes (the phoenix was a To swallow the bait (to catch others by guile, by offering
fabulous Arabian bird. It had no mate but when about to them large promises) : The candidate offered the people
die, made a funeral pile of wood and aromatic gums and on everything on earth and in the heavens if selected. The
it burned itself to ashes. From the ashes a young phoenix people swallowed the bait and elected him.
was believed to rise) : Germany was completely decimated To talk shop (to use the phrases peculiar to one’s
in the Second World War. But she has risen like a phoenix circumstances) : Except for the undertakers, people of the
from its ashes. same professions always talk shop at parties.
y
o
u
C-66 Idioms/Phrases

rs
m
To tie one’s hands (to restrain one from action) : The Good offices: (recommendation) : One can get a good job

a
Government’s hands are already tied with problem plants. only through the good offices of some one in power.

h
It would not like to go in for nationalisation in a big way. A good Samaritan: (one who be-friends a stranger or a

b
o
To tread on the heels of (follow close behind) : Famine friendless person) : Centuries ago, India played a good

o
b
treads on the heels of drought. Samaritan to the hapless Parsees fleeing their native land.

.w
To fish in troubled waters (to make personal profit out of a The green-eyed monster: (jealousy) : The green-eyed

o
disturbance) : The super powers are there in West Asia to monster strikes a woman the moment she sees her husband

rd
fish in troubled waters. talking to a pretty woman.

p
re
To pour oil on troubled waters (to say or do anything A Herculean task (a job requiring great efforts) : Eradication

s
which soothes and calms angry passions) : The government of poverty is a Herculean task requiring the collective

s
.c
poured oil on troubled waters by announcing a judicial efforts of the entire country.

o
enquiry into the firing. Lynch Law: (the practice of punishing people where the

m
To win or gain laurels or to bear away palm (to achieve punishment is inflicted by unauthorised persons and
success in a contest) : The Indian Cricket Team won laurels without judicial trial) : Mob law denotes the same thing
on two successive occasions once in West Indies and then when carried out by a mob. In African countries they often
in England. resort to lynch laws.
To worship the rising sun (to pay respect to the man who is A maiden speech (the first speech of a new member in a
rising in power the influence) : The newly appointed manager public body as in Town Hall or in Parliament) : Amitabh’s
has taken over and his clerks worship the rising sun. maiden speech was very impressive.
Argus-eyed (jealously watchful) : The husband of a pretty A nine day’s wonder (a fascinating but temporary
wife has got to be Argus-eyed. phenomenon): Beauty is, proverbially, a nine day’s wonder.
Aegean stables: (to clean Aegean stables, To correct a An open question : (a matter for discussion and not yet
great abuse, from the stables of king Agues of Greece, whose decided): As far as India is concerned, Kashmir is no longer
stables had not been cleaned for thirty years) : The law an open question.
against prostitution has cleaned no Aegean stables; it has A red-letter day: (an auspicious, fortunate or important day):
merely pushed it underground. The 26th January, 1950 is a red-letter day in India’s history.
Backstairs influence (influence exerted secretly and in a Scot-free: (exempt from payment, unhurt, safe): Because he
fashion not legitimate) : The moneyed people do exercise had influential connections, the culprit went scot-free.
backstairs influence on Parliament. A sheet anchor: (the chief safety, the last refuge for safety):
Bad blood : (active enmity) : There has been bad blood One’s faith in God is one’s sheet anchor in times of stress
between India and Pakistan since 1947. and strain.
A bone of contention : (subject of dispute) : Kashmir Tall Talk: (boastful language): If we have no real
continues to be a bone of contention between India and accomplishments, we indulge in tall talk to delude ourselves
Pakistan since 1947. and others too.
A bosom friend (A very intimate and trusted friend) : Bosom A white elephant (an unprofitable possession) : The upper
friends never betray one another. Houses are white elephants and should be abolished.
A bull in a China shop: (Someone who destroys everything A white lie: (an evasion, a harmless and non-malicious
at the same time he happens to be in) : The plainsmen proved untruth) : Professional members often indulge in white lies.
to be a bull in a China shop in the hills, ruining the hill A wild goose chase (a foolish, wild, unprofitable adventure) :
people in all ways. Attempts towards stabilisation of prices in a developing
A close shave : (a narrow escape from collision accident): economy, is a wild goose chase.
The bus had a close shave as its driver swerved to the right An apple of discord: (a subject of envy and strife) : Kashmir
a split second before the on-coming truck could run into it. continues to be the apple of discord between India and
A cold comfort: (something calculated to cause pain or Pakistan.
irritation) : The promise of a better future is only cold comfort Cock and bull story (a silly improbable story) : That India
to the frustrated youth of today. wanted to break up West Pakistan was a cock and bull
A dog in the manger policy: (said of a person who cannot story published by the U.S.A.
himself use what another wants, and yet will not let that A fish out of water : (a person in uncomfortable
other have it) : The affluent nations are a dog-in-the manger, surroundings) : An Indian may earn tons of money in the
destroying what they can’t use themselves than giving it Western countries, but he will always feel like a fish out of
to the poor nations of Asia and Africa. water there.
Elbow room: (opportunity for freedom of action) : Only The gift of the gab: (fluency of speech) : The gift of the gab
give him elbowroom and he will succeed. combined with a slight cunning makes for a successful
A fair-weather Friend: (one who deserts you in difficulties) : politician.
A fair-weather friend disappears the moment your money Lion’s share: (an unfairly large share) : The big nations
disappears. continue to have the lion’s share of world trade.
French leave: (absence without permission.) : He went on A mare’s nest : (a discovery that turns out to be false or
a french leave and was summoned by the direction the next worthless) : There was much fanfare about the solar cooker.
day he went to office. Later it turned out to be a mare’s nest.
y
o
u
Idioms/Phrases C-67

rs
m
The milk of human kindness: (kindly feelings a phrase used To be Greek or double Dutch to one: (unintelligible) : He

a
by Shakespeare.) : With all their poverty, Indians do not spoke so fast that all he said was double Dutch to the

h
lack the milk of human kindness.

b
audience.

o
Penelope’s web : (a work which seems to be going on and To be with in an ace of (to be very nearly) : He was within

o
b
yet never comes to an end.) : A housewife’s chores are a an ace of being shot.

.w
penelope’s web. To be at the back and call: (to be always ready to serve) :

o
The pros and cons of a question: (arguments for and against You must not expect me to be at your back and call, I have

rd
a thing) : They discussed the pros and cons of the matter my own business to attend to.

p
re
before taking a decision. To be at daggers drawn : (in bitter enmity) : With every

s
The skin of one’s teeth: (a phrase used when one escapes

s
passing year the hostility between the Arabs and the Israelis

.c
losing everything except life.) : The storm broke up the ship has grown more bitter. They have always been at daggers

o
but the sailors escaped by the skin of their teeth.

m
drawn.
A snake in the grass: (a secret foe.) : China has certainly To be at sea: (contused, uncertain of mind) : I am quite at
been a snake in the grass for India. Even in the heyday of
sea in Mathematics.
Hindi Chini bhai-bhai, she was quietly devouring bits of
To be at one’s wits end: (perplexed) : With the master
our territory.
shouting from the bathroom and the mistress from the
A stone’s throw: (very near.) : The Taj Hotel is at a stone’s
kitchen the servant was at his wits end as to whom to attend
throw from the Gateway of India.
All moonshine: (foolish, idle, untrue statement.) : The talk first.
about welfare of the poor is all moonshine. To be in one’s element: (to be in agreeable company or
Behind the scenes : (of a person having secret or private work) : Shaw is in his element when he is writing about the
information and influence) : The dismissed Secretary, having social ills of his time.
been behind the scenes, has made some strange revelations To be on wane: (to be on the decline) : After the second
as to the way in which the business is managed. World War, the British Empire was on the wane.
Between two fires : (assailed or shot at from two sides) : A To be on the carpet: (to be summoned to one’s employer’s
man, arbitrating between the mother and wife, is to be room for reprimand) : The unpunctual clerk was repeatedly
between the two fires, for his decisions can rarely please on the carpet.
both. To be on the last legs: (about to collapse) : With science
In a body: (together) : The striking workers went in a body dominating life more and more, religion seems to be on its
to the Manager to present their demands. last legs.
Wide off the mark or beside the mark: (irrelevant) : ‘Beside Chip of the old block (a son who is very like his father) :
the mark reasoning or argument’. The younger Nawab of Pataudi has proved to be a chip of
Cheek by jowl: (in the same position) : There was a lawyer the old block. He is as good a batsman as his father.
who never had a client cheek by jowl with a doctor who To bring under the hammer: (to sell it by auction) : If a
never had a patient. person goes insolvent, his creditors will bring everything
Out at elbows: (destitute) : The rising prices and the new that he owns under the hammer to recover their money.
taxes may soon see most of us out at elbows. To pay one’s way:( not get into debt) : While at college, he
Part and Parcel : (integral part of a society, community paid his way by working as a newspaper vendor.
etc.) Some customs and traditions are a part and parcel of To weather the storm: (to come out of a crisis successfully) :
Indian culture. In a crisis it is unity which helps a nation to weather the
A storm in a tea cup: (a great fuss about a trifle) : The storm.
crackers fired by Diwali revellers caused a storm in the tea To sail before the wind: (to go in the direction towards in
cup when minority communities thought it to be a bomb which the wind is blowing) : An opportunist is he who sails
attack by the other community.
before the wind (Its opposite is to sail close to the wind
A fly in the ointment : (a trifling circumstance which mars
i.e., to break a law or principle)
enjoyment) : It was a wonderful picnic, the only fly in the
To be in the same boat (To be equally exposed with a person
ointment being the absence of shady trees at the picnic
to risk or misfortune) : In a nuclear war, the rich and the
spot.
Not worth his salt: (good for nothing) : A soldier who poor nations will be in the same boat. None will be able to
shivers at the boom of guns is not worth his salt. protect themselves.
With a pinch of salt: (to take a statement with a grain of salt To sail under false colours: (To pretend to be what one is
is to feel some doubt whether it is altogether true) : Shaw’s not, to try to deceive) : In our blessed country, a smuggler
claim of having remained a celibate even after marriage has sailing under the false colours of a socialist will never be
to be taken with a pinch of salt. exposed.
Null and void: (Invalid, valueless, no longer in force) : The To take the wind out of one’s sails: (Frustrating him by
court declared the appointment to be null and void. anticipating his arguments, take away his advantage
To be posted up: (well acquainted with) : I want to be posted suddenly) : Before the U.S. could spread the canard about
up in Indian History. India’s intention to destroy West Pakistan after “capturing”
To be worth its weight in gold: (extremely valuable) : In the Bangladesh, India took the wind out of their sails by
desert a bottle of water is often worth its weight in gold. declaring a unilateral cease-fire.
y
o
u
C-68 Idioms/Phrases

rs
m
Game is not worth the candle : (The advantage or enjoyment Render unto Caesar what is Caesar’s : (to be wise)

a
to be gained is not worth the time spent in gaining it) : Look before you leap : (don’t be reckless and impulsive)

h
b
Journey to the moon is an elaborate and costly affair and Make hay while the sun shines : (to make/ill use of the

o
some people with a pragmatic approach feel the game is not

o
given opportunity)

b
worth the candle.

.w
Never look a gift horse in the mouth : (there can be no
Not fit to hold a candle to: (One is inferior) : For all his

o
choice about things given in charity)

rd
pious platitudes and political stunts, Mr. Nixon is not fit to Beggars can’t be choosers : (no choice in scarcity)

p
hold a candle to Lincoln or Roosevelt.

re
Nearer the Church, farther from heaven : (the more
Hope springs eternal in the human breast : (one never loses

s
opportunity you have, the less you benefit from it)

s
hope).

.c
Every cock fights best on his own dung hill : (one is very

o
Fools rush in where angels fear to tread : (said of reckless

m
brave and confident in one’s own place)
persons)
A rolling stone gathers no moss : (an aimless person cannot
He who pays the piper calls the tune : (one has to act
succeed)
according to the wishes of one’s master)
Rome was not built in a day : (things take time to complete
You cannot make a silk purse out of a sow’s ear : (said of
and to mature)
something impossible)
A bird in hand is worth two in the bush : (right use of the One swallow does not make a summer : (one person can ‘t
present opportunity) do everything)
One man’s meat is another man’s poison : (what is good for Apparel proclaims the man : (you judge a man’s worth by
one may he harmful for another person) his clothes)
Out of the frying pan into the fire : (from one trouble to To run with the hare, to hunt with the hound : (to be insincere
another) to someone)
The last straw breaks the camel’s back : (the smallest Sweet are the uses of adversity : (sufferings are to be
addition to an already heavy task makes it intolerable) welcomed)
Distance lends enchantment to the old : (things look nice Uneasy lies the head that wears the crown : (with power
and beautiful when they are not within reach) and authority come worries and responsibilities)

DIRECTION: Choose the correct meaning of an idom/ phrase. 6. To talk through one’s hat :
1. To turn over a new leaf : (a) To speak fluently
(a) To change completely one’s course of action (b) To talk nonsense
(b) To shift attention to new problems (c) To talk wisdom
(c) To cover up one’s faults by wearing new marks (d) To speak at random
(d) To change the old habits and adopt new ones
7. To snap one’s fingers :
2. To wrangle over an ass’s shadow :
(a) To act in a foolish way (a) To speak abruptly
(b) To quarrel over trifles (b) To accept immediately
(c) To waste time on petty things (c) To grasp eagerly
(d) To do something funny (d) To become contemptuous of
3. All agog : 8. To take the bull by the horns :
(a) Everybody (b) All ready (a) To punish a person severly for his arrogance
(c) Restless (d) Almighty (b) To grapple courageously with difficulty that lies in our
4. To take with a grain of salt : way
(a) To take with some reservation (c) To handle it by fierce attack
(b) To take with total disbelief (d) To bypass the legal process and take action according
(c) To take whole heartedly
to one’s own whims
(d) To take seriously
9. To be in abeyance :
5. Hobson’s choice :
(a) Feeling of insecurity (a) To be in trouble
(b) Accept or leave the other (b) Dual minded
(c) Feeling of strength (c) In a fighting mood
(d) Excellent choice (d) Insuspense
y
o
u
Idioms/Phrases C-69

rs
m
10. To cast pearls before a swine : 21. To turn the tables :

a
(a) To spend recklessly (a) To defeat

h
(b) To spend a lot of money on the unkeep of domestic hogs (b) To oppose

b
o
(c) To waste monkey over trifles (c) To create chaos

o
b
(d) To offer to a person a thing which he cannot appreciate (d) To change the sorry scheme

.w
11. To take people by storm : (e) To change completely the position of disadvantage

o
(a) To put people in utter surprise 22. To keep the ball rolling :

rd
(b) To captivate them unexpectedly (a) To earn more and more

p
re
(c) To exploit people’s agitation (b) To work constantly

s
(d) To bring out something sensational attracting people’s (c) To keep the conversation going

s
.c
attention (d) To make the best use of

o
12. Harp on : 23. All and sundry :

m
(a) To comment (a) Greater share
(b) To criticise (b) All of a sudden
(c) To keeep on talking (c) Completion of work
(d) To keep on insulting (d) Everyone without distinction
13. To bring one’s eggs to a bad market : 24. To worship the rising sun :
(a) To face on humiliating situation (a) To honour a man who is coming into office
(b) To bring one’s commodities to a market where there is (b) To honour to promising people
no demand for them (c) To indulge in flattery
(c) To show one’s talents before audience which is (d) To welcome the coming events
incapable of appreciating them 25. To play fast and loose :
(d) To fail in one’s plans because one goes to the wrong (a) To beguile others
people for help (b) To be winning sometimes and losing at other times
14. To give/get the bird : (c) To play with someone’s feeling
(a) To get the awaited (d) To play tricks
(b) To have good luck 26. To have brush with :
(c) To send away (a) To start painting
(d) To get the impossible (b) To have good and pleasing terms
15. To save one’s face : (c) To be impressed
(a) To hide oneself (d) To have a slight encounter
(b) To oppose 27. To pull one’s socks up :
(c) To avade disgrace (a) To prepare
(d) To say plainly (b) To try hard
16. To split hours : (c) To get ready
(a) To sidetrack the issue (d) To depart
(b) To quarrel over trifles 28. Dog in the manger :
(c) To indulge in over-refined arguments (a) An undersized bull almost the shape of a dog
(d) To find faults with other (b) A dog that has no kennel of its own
17. Will o’ the wisp : (c) A person who puts himself in difficulties on account
(a) Anything which eludes or deceives of other people
(b) To act in a childish way (d) A person who prevents others from enjoying
(c) To act in a foolish way something useless to himself
(d) To have desires unbacked by efforts 29. To set the people by ears :
18. To read between the lines : (a) To box the people
(a) To concentrate (b) To insult and disgrace the people
(b) To read carefully (c) To punish heavily
(c) To suspect (d) To excite people to a quarrel
(d) To grasp the hidden meaning 30. To give chapter and verse for a thing :
19. To flog a dead horse : (a) To produce the proof of something
(a) To act in a foolish way (b) To eulogize the qualities of a thing
(b) To waste one’s efforts (c) To make publicity of a thing
(c) To revive interest in an old subject (d) To attach artificial value to a thing
(d) To revive old memories 31. To plough the sands :
20. A tall order : (a) To work hard to achieve one’s aim
(a) A task difficult to perform (b) To afford hope or ground for expecting a good result
(b) A bid problem (c) To busy oneself in a way which cannot lead to any
(c) A royal summon profitable result
(d) A big demand (d) To advance one’s position in life
y
o
u
C-70 Idioms/Phrases

rs
m
32. To take umbrage : 43. Hard-pressed :

a
(a) To feel depressed (a) Bewildered

h
(b) To be offended (b) Insulted

b
o
(c) To be satisfied (c) Hard discipline

o
b
(d) To be pleased (d) In difficulties

.w
33. To drink like a fish : 44. To be at one’s finger’s end :

o
(a) To drink little (a) To be hopeless

rd
(b) To drink alone (b) To be highly perplexed

p
re
(c) To be a drunkard (c) To be completely conversant with

s
(d) To drink in the company of others (d) To count things

s
.c
34. To fight to the bitter end : 45. To pull strings :

o
(a) To fight with poison-tipped arrows (a) To exert hidden influence

m
(b) To fight to the last paint of enemy position (b) To tease someone
(c) To die fighting (c) To speed up
(d) To carry on a contest regardless of consequences (d) To start something
35. To give a false colouring : 46. A green horn :
(a) To misrepresent (a) An envious lady
(b) To submit the false report (b) A trainee
(c) To be dishonest (c) An inexperienced man
(d) To conceal the facts (d) A soft-hearted man
36. To show the white feather : 47. To be old as the hills :
(a) To show signs to cowardice (a) To be very ancient
(b) To seek peace (b) To be wise tand learned
(c) To show arrogance (c) To be old but foolish
(d) To become polite (d) Not being worth the age
48. To pour oil in troubled water :
37. Spick and span :
(a) To forment trouble
(a) Neat and clean
(b) To add to the trouble
(b) Outspoken
(c) To instigate
(c) A ready-made thing
(d) To calm a quarrel with sooting words
(d) Garrulous
49. To mind one’s P’s and Q’s :
38. To take the wind out of another’s sails :
(a) To be cautious
(a) To manouevre ot mislead another on the high seas
(b) To be accurate and precise
(b) To cause harm to another
(c) To be careful of one’s accounts
(c) To defeat the motives of another (d) To be careful of one’s personality
(d) To anticipate another and to gain advantage over him 50. To break the ice :
39. To carry the coal to newcastle : (a) To start quarreling
(a) To work hard (b) To end the hostility
(b) To finish a jab (c) To start to conversation
(c) To do unnecessary things (d) To end up partnership
(d) To do menial jobs 51. To cool one’s heels :
40. The pros and cons : (a) To rest for sometime
(a) For and aginst a thing (b) To give no importance to someone
(b) Foul and fair (c) To remain in a comfortable position
(c) Good and evil (d) To be kept waiting for sometime
(d) Former and latter 52. A fool’s errand :
41. A baker’s dozen : (a) A blunder
(a) Twelve (b) An impossible task
(b) Charity (c) A useless undertaking
(c) Thirteen (d) None of these
(d) Allowance 53. Swan-song :
42. A bull in a china shop : (a) Music as sweet as a song of birds
(a) A person who is very ugly but loves the beautiful (b) A melodious song in praise of someone
things of life (c) Praise of a woman by her lover
(b) A person who takes a sadistic delight in harming (d) Last work of a poet or musician before death
innocient people 54. To oil the knocker :
(c) A person who becomes too excited where no excitment (a) To instigate a person to do a job
is warranted (b) To do a work with increased pace
(d) A person who is rought and clumsy where skill and (c) To tip the office-boy
care are required (d) To revive old enmity
y
o
u
Idioms/Phrases C-71

rs
55. By the rule of thumb :

m
67. To have one’s heart in one’s boots :

a
(a) By the use of force (a) To be deeply depressed

h
(b) By the use of trickery (b) To be frightened

b
o
(c) By cheating and deception (c) To get angry

o
b
(d) By practical experience which is rather rough (d) To keep a secret

.w
56. To live fast : 68. To strike one’s colours :

o
(a) To lead a life of dissipation

rd
(a) To fight vigorously
(b) To accomplish a purpose

p
(b) To work hard

re
(c) to do a task hurriedly
(c) To surrender

s
(d) To use up one’s income as fast as it comes in

s
(d) To make a rude gesture

.c
57. To hold somebody to ransom :

o
(a) To keep captive and demand concession 69. To ride hell for Leather :

m
(b) To humiliate somebody (a) To ride with furious speed
(c) To offer bribe (b) To adopt flase means to succeed
(d) To blackmail and extract money (c) To work hard for a small accomplishment
58. To hit the jackpot : (d) To earn money by all means
(a) To gamble
70. To speak daggers :
(b) To get an unexpected victory
(c) To be wealthy (a) To abuse someone
(d) To make money unexpectedly (b) To indulge in varacious talks
59. To pay off old scores : (c) To speak to a person with hostility
(a) To have one’s revenge (d) None of these
(b) To settle a dispute 71. Be in the mire :
(c) To repay the old loan (a) Be under dept
(d) None of these (b) Be in love
60. To take a leaf out of somebody’s book : (c) Be in difficulties
(a) To take him as a model
(d) Be uneasy
(b) To steal something valuable
(c) To follow the dictates of someone 72. To give a rap on the knuckles :
(d) To conform to other’s standard (a) To eulogise
61. To set the thames on fire : (b) To chatise
(a) To do something remarkable (c) To condemn
(b) To try to do the impossible (d) To rebuke
(c) To burn something to ashes 73. To be above board.
(d) To fling inonical remarks (a) To have a good height
62. To cast pearls before a swine :
(a) To offer some a thing which he cannot appreciate (b) To be honest in any business deal
(b) To bring something good before the eyes of a greedy (c) Having no debts
person (d) To try to be beautiful
(c) To spend racklessly on a useless fellow 74. To cry wolf.
(d) To indulge in fruitless endeavours
(a) To listen eagerly (b) To give false alarm
63. To bear the palm :
(a) To win (c) To turn pale (d) To keep off starvation
(b) To accept a challenge 75. He is on the wrong side of seventy.
(c) To accept defeat (a) more than seventy years old
(d) To endure something (b) less than seventy years old
64. To change colour :
(c) seventy years old
(a) To change appearance
(b) To shift allegiance to (d) eighty years old
(c) To favour the wrong person 76. To have an axe to grind.
(d) None of these (a) a private end to serve
65. To cut the gordian knot : (b) to fail to arouse interest
(a) To solve a difficult problem
(b) To be victorious (c) to have no result
(c) To break ties with someone (d) to work for both sides
(d) To defeat to powerful person 77. To drive home.
66. To wrangle over an ass’s shadow : (a) To find one’s root
(a) To act in a foolish way
(b) To return to place of rest
(b) To do something funny
(c) To quarrel over trifles (c) Back to original position
(d) To quarrel over the possession of an ass (d) To emphasise
y
o
u
C-72 Idioms/Phrases

rs
m
DIRECTIONS (Qs. 78-92) : In the following questions four (c) to understand the inner meaning

a
alternatives are given for the idiom / phrase and bold italicised (d) to read carefully

h
b
in the sentence. Choose the alternative which best expresses the 86. Where discipline is concerned I put my foot down.

o
o
meaning of the idiom / phrase and mark it in the Answer-Sheet. (a) take a firm stand

b
(b) take a light stand

.w
78. Once the case reached the court, the police washed their
(c) take a heavy stand

o
hands off it.

rd
(d) take a shaky stand
(a) waited for a response to

p
87. The convict claimed innocence and stood his ground in

re
(b) claimed credit for

s
spite of the repeated accusations.

s
(c) disassociated themselves from

.c
(a) knelt

o
(d) seemed eager to continue (b) surrendered

m
79. She wanted to go hitch-hiking but her mother put her foot (c) kept standing
down and now she’s going by bus. (d) refused to yield
(a) took a firm stand 88. We must work with all our might and main, otherwise we
(b) expressed her displeasure cannot succeed.
(c) scolded her badly (a) full force (b) complete trust
(d) got irritated (c) exceptional skill (d) full unity
80. Adolescence is a period of halcyon days. 89. The sailors nailed their colours to their mast.
(a) hard days (b) of mental pressure (a) put up a colourful mast
(c) happy days (d) days of preparation (b) refused to climb down
81. My sincere advice to my maidservant fell on stony ground. (c) took over the ship
(a) was counter productive (b) had a strong impact (d) decided to abandon the ship
(c) made on stubborn (d) had little success 90. We had (had) better batten down the hatches. The weather
is unpredictable.
82. He has all his ducks in a row; he is complacent.
(a) stay in-door
(a) has everything ready (b) is well organised
(b) prepare for a difficult situation
(c) always scores a zero (d) never gets confused
(c) go somewhere safe
83. With great difficulty, he was able to carve out a niche for (d) face the obstacles
himself. 91. It is difficult to have a sensible discussion with her as she
(a) became a sculptor flies off at a tangent.
(b) did the best he could do
(a) gets carried away
(c) destroyed his career
(b) starts discussing something irrelevant
(d) developed a specific position for himself
84. You will succeed if you follow my advice to the letter. (c) loses her temper easily
(a) about writing letters (b) written in the letter (d) does not really understand anything
(c) in every detail (d) very thoughtfully 92. The students found it hard to go at equal speed with the
85. A critic's work is to read between the lines. professor.
(a) to comprehend the meaning (a) get away from (b) put up with
(b) to appreciate the inner beauty (c) keep up with (d) race against

ANSWER KEY
1 (d) 2 (b) 3 (c) 4 (a) 5 (b) 6 (b) 7 (d) 8 (b)
9 (d) 10 (d) 11 (b) 12 (c) 13 (d) 14 (c) 15 (c) 16 (c)
17 (a) 18 (d) 19 (b) 20 (a) 21 (d) 22 (c) 23 (d) 24 (a)
25 (d) 26 (d) 27 (b) 28 (d) 29 (d) 30 (a) 31 (c) 32 (b)
33 (c) 34 (d) 35 (a) 36 (a) 37 (a) 38 (d) 39 (c) 40 (a)
41 (c) 42 (d) 43 (d) 44 (c) 45 (a) 46 (c) 47 (a) 48 (d)
49 (b) 50 (c) 51 (d) 52 (c) 53 (d) 54 (c) 55 (d) 56 (a)
57 (a) 58 (b) 59 (b) 60 (a) 61 (b) 62 (a) 63 (a) 64 (b)
65 (a) 66 (c) 67 (a) 68 (c) 69 (a) 70 (c) 71 (c) 72 (d)
73 (b) 74 (b) 75 (a) 76 (a) 77 (d) 78 (c) 79 (a) 80 (c)
81 (d) 82 (b) 83 (d) 84 (c) 85 (c) 86 (a) 87 (d) 88 (a)
89 (b) 90 (b) 91 (b) 92 (b)
y
o
u
rs
m
a
CHAPTER

h
b
6

o
SPELLING TEST

o
b
6

.w
o
rd
p
re
s
s
.c
In this type of questions there are four different spelling of Ex. 1 (a) Comittee (b) Commitee

o
m
common word. The student is required to choose the correct (c) Committee (d) Committey
spelling of the given word. Ans. (c) Committee is correctly spelt.

DIRECTIONS: (Qs. 1-143) Choose the correct spelling of the 20. (a) Sattellite (b) Satellite
given word. (c) Sattelite (d) Satelite
1. (a) Efflorascence (b) Efflorescence 21. (a) Ocasion (b) Ocassion
(c) Efllorescence (d) Eflorescence (c) Occasion (d) Occassion
2. (a) Aliennate (b) Allienate 22. (a) Posesion (b) Possession
(c) Alienate (d) Alienatte (c) Posession (d) Possesion
3. (a) Forefiet (b) Forefeit 23. (a) Greivance (b) Greievance
(c) Forfeit (d) Forfiet (c) Griveance (d) Grievance
4. (a) Comemorate (b) Commemmorate 24. (a) Beligrent (b) Beligerent
(c) Momemmorate (d) Commemorate
(c) Belligrent (d) Belligerent
5. (a) Exampli (b) Exampel
25. (a) Coruppt (b) Curropt
(c) Example (d) Exampal
(c) Corrupt (d) Currupt
6. (a) Psychology (b) Sycology
(c) Psykology (d) Sychology 26. (a) Dielectic (b) Deallectic
7. (a) Accessible (b) Accesibel (c) Dilectic (d) Dialectic
(c) Accessible (d) Acessible 27. (a) Achievment (b) Acheivment
8. (a) Tresspass (b) Trespass (c) Achievement (d) Achevement
(c) Tresspas (d) Trespas 28. (a) Coreander (b) Coriander
9. (a) Argumant (b) Arguemant (c) Coriandar (d) Coreandor
(c) Argument (d) Arguement 29. (a) Bouquete (b) Bouquet
10. (a) Aproched (b) Aproached (c) Boquet (d) Bouquette
(c) Appraoched (d) Approached 30. (a) Translucent (b) Translusent
11. (a) Comentry (b) Commentry (c) Transluscent (d) Tranclucent
(c) Commentery (d) Commentary 31. (a) Hetrogenous (b) Hetrogeneous
12. (a) Coimmision (b) Comision (c) Heterogenous (d) Heterogeneous
(c) Comission (d) Commision 32. (a) Foreigner (b) Forienor
13. (a) Guerila (b) Gurilla (c) Foriegnor (d) Foreiner
(c) Gorila (d) Gorilla
33. (a) Addultration (b) Adultration
14. (a) Pasanger (b) Pessenger
(c) Addulteration (d) Adulteration
(c) Pesanger (d) Passenger
15. (a) Tariff (b) Tarriff 34. (a) Gorgette (b) Georgette
(c) Tarif (d) Tarrif (c) Gorgete (d) Georget
16. (a) Jewelery (b) Jewellry 35. (a) Examplary (b) Exemplary
(c) Jwellry (d) Jewellery (c) Examplery (d) Exemplery
17. (a) Grametic (b) Grammetic 36. (a) Sobotage (b) Sobbotage
(c) Grammatic (d) Gramatic (c) Sobotaze (d) Sabatage
18. (a) Blisfull (b) Blissful 37. (a) Colaboration (b) Collaberation
(c) Blisful (d) Blissfull (c) Colaberation (d) Collaboration
19. (a) Embarasment (b) Embarassment 38. (a) Exagerate (b) Exadgerate
(c) Embarrasment (d) Embarrassment (c) Exaggerate (d) Exeggerate
y
o
u
C-74 Spelling Test

rs
m
39. (a) Acurrate (b) Accurate 67. (a) Remunaration (b) Ramunration

a
(c) Acurate (d) Accuratte (c) Remeuneration (d) Remuneration

h
40. (a) Buisness (b) Bussiness 68. (a) Coitment (b) Comittment

b
o
(c) Business (d) Buisiness (c) Committment (d) Commitment

o
b
41. (a) Forecast (b) Forcaust 69. (a) Fullfil (b) Fulfil

.w
(c) Forcast (d) Forecaste (c) Fulfill (d) Fullfill

o
70. (a) Genelogy (b) Genealogy

rd
42. (a) Liesure (b) Leisure

p
(c) Leasure (d) Lesiure (c) Geneology (d) Genalogy

re
43. (a) Colliberation (b) Calliberation 71. (a) Defendant (b) Defendent

s
s
(c) Callibration (d) Calibration (c) Defandent (d) Defandant

.c
o
44. (a) Adviceable (b) Advisable 72. (a) Recommand (b) Recommend

m
(c) Advicable (d) Adviseable (c) Reccomend (d) Recomend
45. (a) Interegnum (b) Intrenum 73. (a) Varstile (b) Verstile
(c) Interregnum (d) Interregnim (c) Versatile (d) Vorstyle
46. (a) Aforstation (b) Aforestation 74. (a) Correspondant (b) Corraspondent
(c) Afforstation (d) Afforestation (c) Corraspondant (d) Correspondent
47. (a) Entreprenuer (b) Entrepraneur 75. (a) Finnancer (b) Financier
(c) Entrapreneur (d) Entrepreneur (c) Financer (d) Financeir
48. (a) Harasment (b) Harassment 76. (a) Etiquete (b) Etiquette
(c) Harrassment (d) Harrasment (c) Ettiquete (d) Ettiquette
49. (a) Perseverance (b) Preservarance 77. (a) Overlapped (b) Overlaped
(c) Preseverence (d) Preserverence (c) Overleped (d) Overelaped
50. (a) Sovereignty (b) Soveriegnty 78. (a) Necessary (b) Necesarry
(c) Sovereignity (d) Soveriegnity (c) Necesary (d) Neccessary
51. (a) Receding (b) Receeding 79. (a) Inocculate (b) Inoculate
(c) Reeceding (d) Recedeing (c) Innoculate (d) Innocculate
52. (a) Illusteous (b) Illistrious 80. (a) Benefeted (b) Benifited
(c) Ilustrious (d) Ilustrious (c) Benefitted (d) Benifitted
53. (a) Skillful (b) Skillfull 81. (a) Supreintendent (b) Superintendent
(c) Skilful (d) Skilfull (c) Suprintendent (d) Supereintendent
54. (a) Septaganarian (b) Septagenarian 82. (a) Temparament (b) Temprament
(c) Septuaganarian (d) Septuagenarian (c) Temperament (d) Temperamant
55. (a) Pitiaeble (b) Pitiable 83. (a) Homeopathy (b) Homoepathy
(c) Pitiaable (d) Pitiabale (c) Homiopathy (d) Homoeopathy
56. (a) Incumbent (b) Incumbant 84. (a) Numrator (b) Numerator
(c) Incambant (d) Incambent (c) Numerater (d) Numrater
57. (a) Budgetary (b) Budgetry 85. (a) Ommission (b) Omision
(c) Budgetery (d) Bugetary (c) Omission (d) Ommision
58. (a) Paralelled (b) Parallelled 86. (a) Mountaineous (b) Mountaneous
(c) Paralleled (d) Parralleled (c) Mountainous (d) Mountanous
59. (a) Arangement (b) Arrangement 87. (a) Giraf (b) Gerraffe
(c) Arrangment (d) Arangement (c) Geraff (d) Giraffe
60. (a) Cemetry (b) Cemetery 88. (a) Legendary (b) Legendery
(c) Cemetary (d) Cemmetery (c) Legendry (d) Legandery
61. (a) Vetniary (b) Vetarinary 89. (a) Aquaintence (b) Acquaintance
(c) Veteninary (d) Veterinary (c) Aquaintance (d) Acquaintence
62. (a) Lauriate (b) Laureat 90. (a) Arodrome (b) Airodrome
(c) Laureate (d) Lauriat (c) Aerodrom (d) Aerodrome
63. (a) Comander (b) Commander 91. (a) Forein (b) Fariegn
(c) Commandor (d) Comandar (c) Foriegn (d) Foreign
64. (a) Tranquillity (b) Trenquillity 92. (a) Aprentic (b) Aperentic
(c) Trenquility (d) Tranquility (c) Apprentic (d) Apperentice
65. (a) Alitration (b) Alliteration 93. (a) Indigenous (b) Indigenuous
(c) Alitration (d) Aliteration (c) Indiginous (d) Indeginuous
66. (a) Chaufer (b) Chauffer 94. (a) Coleagu (b) Colleagu
(c) Chaufeur (d) Chauffeur (c) Coleague (d) Colleague
y
o
u
Spelling Test C-75

rs
m
95. (a) Lieutinant (b) Lieutenant 124. (a) Gaurantee (b) Guarantee

a
(c) Leutenant (d) Liutenant (c) Garuntee (d) Guaruntee

h
b
96. (a) Paralelogram (b) Paralellogram 125. (a) Parlamentary (b) Parliamentary

o
(c) Parallelogram (d) Parallellogram (c) Parlimentary (d) Parliamentery

o
b
97. (a) Corigendum (b) Coridgendum 126. (a) Tentaitive (b) Tentative

.w
(c) Corrigendum (d) Corridgendum (c) Tentitive (d) Tantitive

o
98. (a) Prevalent (b) Prevalent

rd
127. (a) Surveilance (b) Survellance
(c) Prevelant (d) Prevelent

p
(c) Surveillance (d) Survaillance

re
99. (a) Milennium (b) Millenium 128. (a) Careir (b) Career

s
s
(c) Millennium (d) Milenium (c) Caireer (d) Carreer

.c
100. (a) Acomodation (b) Accomodation

o
129. (a) Resteraunt (b) Restaurent

m
(c) Acommodation (d) Accommodation
(c) Rastaurent (d) Restaurant
101. (a) Inimitable (b) Innimatible
130. (a) Damage (b) Dammage
(c) Inimitible (d) Inimmitable
(c) Damaige (d) Dammege
102. (a) Refrigerator (b) Refriderater
(c) Refridgerator (d) Referigerator 131. (a) Farenheit (b) Fahrenhiet
103. (a) Apallation (b) Appellation (c) Farenhiet (d) Fahrenheit
(c) Appelation (d) Appallation 132. (a) Recommandation (b) Recomandation
104. (a) Catlogg (b) Catlog (c) Recommendation (d) Recomendation
(c) Catlogue (d) Catalogue 133. (a) Accessible (b) Accissible
105. (a) Obssesion (b) Obssession (c) Acessible (d) Aceesible
(c) Obsession (d) Obsessien 134. (a) Tempereture (b) Tempearature
106. (a) Alottee (b) Allotee (c) Temperature (d) Tamperature
(c) Alotte (d) Allottee 135. (a) Rennaisance (b) Rennaissance
107. (a) Seperable (b) Separable (c) Renaissance (d) Renaisance
(c) Saperable (d) Saparable 136. (a) Milennium (b) Millenium
108. (a) Hindrence (b) Hindrance (c) Milleneum (d) Millennium
(c) Hinderence (d) Hinderance 137. (a) Ocassion (b) Occassion
109. (a) Proprietry (b) Propriatory (c) Occasion (d) Occation
(c) Proprietary (d) Proprietory 138. (a) Garulous (b) Garrulous
110. (a) Strategam (b) Strategem (c) Garullous (d) Garrullous
(c) Stratagem (d) Stratagam 139. (a) Marquee (b) Markue
111. (a) Reharsal (b) Rehersal (c) Marquei (d) Marquie
(c) Rehearsal (d) Rehearsel 140. (a) Puissant (b) Puiscant
112. (a) Cancelation (b) Cancelletion (c) Puiscent (d) Puissent
(c) Cancellation (d) Cancellasion 141. (a) Disconncerting (b) Disconserting
113. (a) Assasination (b) Asasination (c) Discuncerting (d) Disconcerting
(c) Assassination (d) Asassination 142. (a) Exilarate (b) Exhilerate
114. (a) Millonare (b) Millionaire (c) Exsilarate (d) Exhilarate
(c) Millionare (d) Millonaire 143. (a) Plebeian (b) Plibeian
115. (a) Draut (b) Drout (c) Plebian (d) Plebiean
(c) Drougt (d) Drought
116. (a) Fabullous (b) Fablous DIRECTIONS (Qs. 144-220) : Choose the mis-spelt word out of
(c) Fabulous (d) Fabuleous given options.
117. (a) Accomplish (b) Ackmplesh 144. (a) Designation (b) Amature
(c) Accompalish (d) Acomplush (c) Controversy (d) Burglar
118. (a) Simaltanious (b) Simultaneous 145. (a) Deceive (b) Conneive
(c) Simultanious (d) Simaltaneous (c) Perceive (d) Acheive
119. (a) Turnament (b) Tournamant 146. (a) Penance (b) Menace
(c) Tournament (d) Turnamant (c) Tendancy (d) Governance
120. (a) Tomarow (b) Tomarrow 147. (a) Prejudicial (b) Affectionate
(c) Tomorrow (d) Tommorow (c) Indispensible (d) Assiduous
121. (a) Neighbur (b) Neighber 148. (a) Kitten (b) Cumulative
(c) Neighbour (d) Neibur (c) Justified (d) Inediable
122. (a) Fasist (b) Facict 149. (a) Hillock (b) Vilify
(c) Facist (d) Fascist (c) Mileage (d) Hillarious
123. (a) Occurrance (b) Occurrence 150. (a) Predilection (b) Discipline
(c) Occurance (d) Occurence (c) Indigenous (d) Preferrable
y
o
u
C-76 Spelling Test

rs
m
151. (a) Suicide (b) Suiteable 179. (a) Parasology (b) Possibility

a
(c) Summarize (d) Superficial (c) Potassium (d) Preamble

h
b
152. (a) Neice (b) Neither 180. (a) Careful (b) Untill

o
(c) Neigh (d) Rein

o
(c) Fulfil (d) Final

b
153. (a) Light (b) Fight 181. (a) Essential (b) Rehabilitation

.w
(c) Hight (d) Might (c) Appreciation (d) Concomitant

o
rd
154. (a) Control (b) Confusion 182. (a) Knack (b) Knee

p
(c) Confrence (d) Committee (c) Kneed (d) Kneel

re
155. (a) Instantanious (b) Intermediate 183. (a) Amend (b) Commend

s
s
(c) Intermittent (d) Interference

.c
(c) Assend (d) Comprehend

o
156. (a) Breakage (b) Brevity 184. (a) Monotonous (b) Monastry

m
(c) Breathless (d) Briliance (c) Monitor (d) Moraine
157. (a) Nuptial (b) Nickers 185. (a) Adultration (b) Adulation
(c) Nephew (d) Nuisance (c) Advancement (d) Admittance
158. (a) Explaination (b) Extermination
186. (a) Soothe (b) Sponser
(c) Exaggeration (d) Expectation
(c) Stepney (d) Struggle
159. (a) Covetous (b) Coherent
187. (a) Resembelance (b) Recuperate
(c) Consice (d) Consent
(c) Recommend (d) Resource
160. (a) Plateau (b) Plebian
(c) Plaedge (d) Pollinate 188. (a) Rapport (b) Substance
161. (a) Cruelly (b) Truly (c) Gravyard (d) Resistible
(c) Verbally (d) Rudely 189. (a) Hack (b) Hawl
162. (a) Oversear (b) Oscillate (c) Halve (d) Hark
(c) Ossicle (d) Obnscure 190. (a) Deep (b) Heap
163. (a) Cruise (b) Deduse (c) Sleep (d) Weap
(c) Truce (d) Bruise 191. (a) Semester (b) Senesent
164. (a) Democracy (b) Aristocracy (c) Sensory (d) Salacious
(c) Advocacy (d) Courtecy 192. (a) Solution (b) Tuition
165. (a) Hyphen (b) Hygene (c) Circutous (d) Fruition
(c) Hurdle (d) Haphazard 193. (a) Stolen (b) Eaten
166. (a) Believe (b) Relieve (c) Beaten (d) Writen
(c) Drieve (d) Decieve 194. (a) Device (b) Demice
167. (a) Juice (b) Jersy (c) Revise (d) Advice
(c) Jaggery (d) Japanese 195. (a) Illegal (b) Illitrate
168. (a) Envelope (b) Enthuse (c) Illegitimate (d) Illegible
(c) Eratic (d) Emigrant 196. (a) Concession (b) Reconciliation
169. (a) Freshner (b) Forlorn
(c) Humilliation (d) Polliation
(c) Foreign (d) Fruity
197. (a) Anxiety (b) Ankel
170. (a) Democracy (b) Beaureaucracy
(c) Accommodation (d) Allergy
(c) Prophecy (d) Aristocracy
198. (a) Anaemia (b) Attitude
171. (a) Spurious (b) Studious
(c) Subsidary (d) Sensible (c) Admiralty (d) Acquarium
172. (a) Charade (b) Studious 199. (a) Nemesis (b) Necessary
(c) Cartrige (d) Sensible (c) Nectar (d) Nieghbour
173. (a) Ballistic (b) Baloon 200. (a) Trachaea (b) Tragedy
(c) Bulletin (d) Bullock (c) Trajectory (d) Training
174. (a) Winner (b) Winning 201. (a) Chancellor (b) Trailor
(c) Writer (d) Writting (c) Traitor (d) Inheritor
175. (a) Bataliion (b) Believe 202. (a) Handicapped (b) Hankerchief
(c) Bewilder (d) Bogie (c) Handiwork (d) Handicraft
176. (a) Asterisk (b) Audience 203. (a) Auspicious (b) Available
(c) Acrue (d) Almighty (c) Attainable (d) Auxilliary
177. (a) Bearer (b) Governor 204. (a) Butcher (b) Burrow
(c) Conquerer (d) Admirer (c) Buoyant (d) Buletin
178. (a) Ceremony (b) Agression 205. (a) Procession (b) Profession
(c) Schedule (d) Knowledge (c) Possession (d) Occassion
y
o
u
Spelling Test C-77

rs
m
206. (a) Enterance (b) Elementary 213. (a) Termination (b) Culmination

a
(c) Endeavour (d) Eccentricity (c) Determination (d) Defination

h
214. (a) Voluntary (b) Vomitting

b
207. (a) Mischeif (b) Belief

o
(c) Vaocabulary (d) Vicisitude

o
(c) Thief (d) Grief

b
215. (a) Mortgaze (b) Manicure

.w
208. (a) Vacuum (b) Vaccinate (c) Miraculous (d) Mesmeric

o
(c) Vacilate (d) Varnish 216. (a) Pellicle (b) Pilgrimage

rd
209. (a) Jeopardy (b) Jealosy (c) Portrait (d) Parepet

p
re
(c) Jaundice (d) Javelin 217. (a) Sceptic (b) Agnostic

s
(c) Prolific (d) Assetic

s
210. (a) Curiculum (b) Contingency

.c
218. (a) Miscelleneous (b) Mathematics

o
(c) Cauliflower (d) Cooperate

m
(c) Cautiously (d) Competition
211. (a) Despair (b) Desperate
219. (a) Soldier (b) Beautiful
(c) Maintain (d) Maintainance (c) Infinite (d) Clearify
212. (a) Photostate (b) Palmistry 220. (a) Sandwitch (b) Sculpture
(c) Posthumous (d) Polytechnic (c) Secrecy (d) Sovereign

ANSWER KEY
1 (b) 2 (c) 3 (c) 4 (d) 5 (c) 6 (a) 7 (c) 8 (b)
9 (c) 10 (d) 11 (d) 12 (d) 13 (d) 14 (d) 15 (a) 16 (d)
17 (c) 18 (b) 19 (d) 20 (b) 21 (c) 22 (b) 23 (b) 24 (d)
25 (c) 26 (d) 27 (c) 28 (b) 29 (b) 30 (a) 31 (d) 32 (a)
33 (d) 34 (b) 35 (b) 36 (a) 37 (d) 38 (c) 39 (b) 40 (b)
41 (a) 42 (b) 43 (d) 44 (b) 45 (c) 46 (d) 47 (d) 48 (b)
49 (a) 50 (a) 51 (a) 52 (d) 53 (c) 54 (d) 55 (b) 56 (a)
57 (a) 58 (c) 59 (b) 60 (b) 61 (b) 62 (c) 63 (b) 64 (a)
65 (b) 66 (d) 67 (d) 68 (d) 69 (c) 70 (b) 71 (a) 72 (b)
73 (c) 74 (d) 75 (b) 76 (b) 77 (a) 78 (a) 79 (b) 80 (c)
81 (b) 82 (c) 83 (d) 84 (b) 85 (c) 86 (c) 87 (d) 88 (a)
89 (b) 90 (d) 91 (d) 92 (d) 93 (a) 94 (d) 95 (b) 96 (c)
97 (c) 98 (b) 99 (c) 100 (d) 101 (a) 102 (a) 103 (b) 104 (d)
105 (c) 106 (d) 107 (b) 108 (b) 109 (c) 110 (c) 111 (c) 112 (c)
113 (c) 114 (b) 115 (d) 116 (c) 117 (a) 118 (b) 119 (c) 120 (c)
121 (c) 122 (d) 123 (b) 124 (b) 125 (b) 126 (b) 127 (c) 128 (b)
129 (d) 130 (a) 131 (d) 132 (c) 133 (a) 134 (c) 135 (c) 136 (d)
137 (c) 138 (b) 139 (a) 140 (a) 141 (d) 142 (d) 143 (b) 144 (b)
145 (d) 146 (c) 147 (c) 148 (d) 149 (d) 150 (d) 151 (b) 152 (a)
153 (c) 154 (c) 155 (a) 156 (d) 157 (b) 158 (a) 159 (c) 160 (b)
161 (b) 162 (a) 163 (b) 164 (d) 165 (b) 166 (d) 167 (b) 168 (c)
169 (a) 170 (b) 171 (c) 172 (c) 173 (b) 174 (d) 175 (a) 176 (c)
177 (c) 178 (b) 179 (a) 180 (b) 181 (d) 182 (c) 183 (c) 184 (b)
185 (a) 186 (b) 187 (a) 188 (c) 189 (b) 190 (d) 191 (b) 192 (c)
193 (d) 194 (b) 195 (b) 196 (c) 197 (b) 198 (d) 199 (d) 200 (a)
201 (b) 202 (b) 203 (d) 204 (d) 205 (d) 206 (a) 207 (a) 208 (c)
209 (b) 210 (a) 211 (d) 212 (a) 213 (d) 214 (d) 215 (a) 216 (d)
217 (d) 218 (a) 219 (d) 220 (a)
y
o
u
C-78 Spotting Errors

rs
m
CHAPTER

a
h
b
SPOTTING ERRORS 7

o
o
b
.w
o
rd
p
re
s
s
.c
o
m
DIRECTIONS (Qs. 1-90 ): In each of the following sentences 8. He relieved the monotony of book peddling by
four words or phrases have been underlined. Only one under-
lined part in each sentence is not acceptable in Standard a b
English, Pick up the part a, b, c or d. becoming an assistant to a conjuror or to other similar
c d
1. I have seen as bad or worse scenes of disorder at the engagements.
a b 9. You have been more able to withstand the treaties than
English fair than in any other Australian mining town. a b c
c d did he .
2. The officers are now perfectly happy fishing, boating, d
a b 10. It is hoped that with the medical fee being withdrawn and
shooting, playing cricket and other sports . a
c d men can be tested and finally accepted , a large number
3. While in conversation with a high military officer b c
a of people will come forward .
he told me that at the headquarters nothing was known . d
b c d 11. Several guests noticed Mr. Peter fall back in his chair
4. The fear of an impending invasion has more to do a b c
a b and gasping for breath.
than even the debasing of the coinage with the financial d
c d
difficulties. 12. A long life is good if one be happy and has friends.
a b c c
5. His assistants have and are still doing excellent work
a b c 13. The entire party is for the Bill ready to stake their po-
a b c d
for the organization.
litical existence.
d
14. He has suffered from a similar attack but milder as
6. There is something more in the fact that the guns have
a b c
a b c
that of last year..
or are about to be sent abroad.
d
d
15. Our strongest reason for siding with the communists
7. The deceased Lord along with his brother started a
a b c
a b c
is because our father is a communist.
business in India .
d
d
y
o
u
Spotting Errors C-79

rs
m
16. Such startling never have neither and never will be 26. The two men were not interested in the same thing and

a
h
a b c d a

b
o
offered again. the work which each of them had to do was of

o
b
17. The persecutors laid a heavy fine on an old man who b c

.w
o
a b different nature .

rd
d

p
being unable to pay, the soldiers were dragging him to

re
c d 27. Surely there was nothing very profound in this remark

s
s
.c
the prison. a b

o
m
18. He feels his troubles as much or even more than but received as thought it were Solomon’s.
s.
a b c c d

they . 28. I am sorry that a previous engagement will prevent


d a b

19. She told them if they heard any one knocking me being present at that meeting.
c d
a b c
29. The fields and meadows looked a picture with sheep
not to open the door..
a b c
d
and cattle feeding on green grass .
20. For this delightful excursion it was felt by the members d
a b c
30. On entering the room she was found hanging from the
that it was due to the secretary.. a b c d
d ceiling.
21. The use of public roads is seriously threatened by the 31. Within twenty four hours of that order being given the
a b a b

freedom given or taken by the owners and drivers of setting up of T.V. stations will being in this country..
c d
c d
motor cars. 32. He laid for half an hour in an unconscious state until
a b c d
22. The red blossoms accented the prevailing pink of the
a police man picked him up.
a b c
33. The journey by car is as quick and perhaps quicker
house front . a b c
d
than by train.
23. What an innovation the team received as they stepped d
a b c
34. The pilot revealed that his airbus was carrying not
on to the field. a b
d less than three hundred passengers last evening .
24. The sea coast is disappearing under the steady c d
a b 35. This year 's crisis seems to be , if not more ,
a b c
erosion of the sea .
c d equally dangerous as last year’s.
d
25. He wishes to tax not only the luxuries of the rich but
a b c 36. He told the members of his staff that every one of them
a b
also the necessity of the poor..
should carry out his task oneself .
d
c d
y
o
u
C-80 Spotting Errors

rs
m
37. He was disciplined in his actions such as in the case of 48. He always practices the justice and cares for moral

a
h
a b a b c d

b
principles.

o
being late for an appointment.

o
49. He generally maintains that the wife bringing a dowry

b
c d

.w
38. odd days off if not half the satisfaction to me a b
Taking

o
is an evidence of her freedom .

rd
a b c

p
c d

re
of a real holiday..
50. Those who are desirous of applying for the post they

s
d

s
.c
a b c
39. A necklace of diamonds, presented by the friends of the

o
should collect forms soon supplied by the office.

m
a
d
bride were among the presents on view .
51. The monument has been erected to the memory of
b c d
a b
40. This year the monsoons failed which caused a terrible
John who was accidentally shot by his brother
a b c
c
famine in the country..
as a mark of affection .
d
d
41. He is trying his best to please his master whom he fears
52. He visited the battlefield where Napoleon was defeated
a b
a b c
may be seriously offended by what has happened.
in his holidays .
c d
d
42. Most of the critics , all over the world, agree that this
53. Some one had said that the English excelled all other na-
a b
a
is one of the most interesting novels that has recently
tions, the Dutch being avaracious , the French, a set of
c d
appeared. b c
43. The building, the work of the most famous among sycophants, the Germans drunk and gluttonous and the
a b Spaniards were proud and insolvent .
d
the living architects , which has been completed after
c 54. Advertising enables the public to know the value of
many years is to be formally inaugurated . a b
d what they are far more than did the people of the past .
44. Hearing the strange noises, it at once occurred to me c d
a b c 55. I think that education and health and those sort of things
that thieves had entered the house. a b
d are two of the main advantages of modern travel .
45. I was much excited being the first time for at least a c d
a b c 56. The statesman was as his speeches inferred so
few years that I had travelled by sea. a b
d imbued with ambition as to be positively dangerous.
46. The new managing committee arranged everything c d
a 57. If this man would not have been poor he
except the weekly meetings were discontinued . a b
b c d would not have worked so hard .
47. Unlike the children of the last century there is a c d
a b 58. Though he has been working very hard supposing if
compulsory education class now for every child upto the a b c
c d he fails what he will do .
age of fifteen. d
y
o
u
Spotting Errors C-81

rs
m
59. There were present Mr. John, Mr. Davis and Mr. Williams 71. She , who had been the apple of his eye, he now began

a
h
a a b c

b
o
the latter was full as ever of good spirits . to regard with something like distrust.

o
b
b c d d

.w
60. The Finance Minister has not and probably will not in-

o
72. Such goods are made for export , and are seldom or ever

rd
a b a b c

p
re
troduce the bill in this session of the Parliament . used in the country..

s
s
c d d

.c
o
61. Whatever your views may be I still hold that the purpose 73. He advised each of them to look after their own in-

m
a b c a b c
of education is to succeed in the life . terests in this matter..
d d
62. Your information may be correct but this type of 74. Walking along the street, my hat was blown off .
a b a b c d
behaviour from a father is hard to be believed . 75. The wise people, who are very few in our country,,
c d a b
63. None can deny that every scientific invention has they think before they act .
a b c d
proved more harmful to humanity than a beneficial . 76. My reason for taking his side is because he has
c d a b c
64. The whole block of buildings including two furniture
no friend .
a b
d
stores and a draper establishment were destroyed in the
77. I have eaten nothing from yesterday evening .
c d
a b c d
fire.
65. I hope you will excuse me leaving early , as I have to 78. People do not entertain guests like they used to in
a b c a b c d
the past.
attend to a patient.
79. The student who you thought so highly of
d
a b c
66. The marquis with his son now at Oxford were present
has failed .
a b c
d
at the function.
80. Neither he pays now nor does he promise to pay later .
d
a b c d
67. The team was now in the field and about to take their
81. He refused to give alms to the beggar
a b c d
places. a b c
68. I can well afford to disregard he who is capable of with a frown on his face
.
a b c d
making such statements. 82. He was very fond of her ; he thought of marrying her
d a b c
69. Such rules do not apply to you and I . more than once .
a b c d d
70. He said that, if he were me he would certainly 83. He does not know how to move with women.
a b c a b c d
undertake it .
d
y
o
u
C-82 Spotting Errors

rs
m
84. She requested me to sit down an d offered me 100. Can you (a) / repair my watch (b) / until Tuesday? (c) / No

a
error (d)

h
a b c

b
101. This picture (a) / is the best (b) / of the two (c) / No error (d)

o
wine and smoke . 102. The drawing room was a mess (a) / with all the furnitures (b)

o
b
d / scattered in total disarrar (c) / No error (d)

.w
85. You cannot skip it over ; it is only a part and parcel 103. No sooner did (a) / the doctor enter the house (b) / then the

o
rd
a b c patient died. (c) / No error (d)

p
104. As I prefer coffee than tea (a) / my friends always take the

re
of the ceremony
. trouble (b) / to get me a cup of coffee, whenever I visit them.

s
d

s
(c) No error (d)

.c
86. Many of the utterances beginning from the presidential

o
105. I did ask him (a) / where you were (b) / but he didn't tell me

m
a b (c) / No error (d)
address were marked with eloquence . 106. Those who are excessively careful (a) / for their health (b) /
c d are not genQrally healthy. (c) No error (d)
107. There has been (a) / little change in the patient's condition
87. The government has a soft corner for the backward (b) / since he was moved to a special ward in the hospital.
a b (c) / No error (d)
classes; it wants to pull them up . 108. Once we have agreed on (a) / the fundamentals, there will
c d hardly be (b) / anything left to discuss about. (c) / No error
(d)
88. They have devoted most of their time in gossiping
109. They cook meals, (a) / lay the table and wash up, (b) / clean
a b c
the house and mend the clothes. (c) / No error (d)
with friends . 110. I will wait for you (a) / at the office (b) until you will finish
d your work. (c) / No error (d)
89. It is wise to marry away the girls before they reach 111. The gap between what he preaches (a) and what he
a b c practises is too wide (b) / to accept by any sensible person.
(c) / No error(d)
the age of puberty..
112. The king was embarrassed (a) / to find evidences (b) / against
d his own queen. (c) / No error (d)
90. Barbarian chiefs and barons carved out 113. While flying over India (a) / we had glimpses of the two
a b sources of her culture (b) / Ganges and Himalayas. (c) No
error (d)
among themselves the fairest positions. 114. They begged him (a) / not to go (b) yet he was determined.
c d (c) / No error (d)
DIRECTIONS (Qs. 91-154): In these quesitons, some of the 115. I don't remember having met him before, (a) / although the
sentences have errors and some are correct as they are. Find voice sounds (b) rather familiar to me. (c) / No error (d)
out which part of a sentcnce has an error. If a sentence is free 116. A lot of travel delay is caused (a) / due to the inefficiency
from error, write (d) as the answer. and, lack of good management (b) / on behalf of the railways.
91. A free press is not a privilege (a) / but the organic necessity (c) /No error (d)
(b) / in a free society (c) / No error (d) 117. The manager called the clerk and said whether (a) / he was in
92. Where he (a) / to see you, (b) / he would have been surprised. the habit of (b) / sleeping at home as well. (c) / No error (d)
(c) / No error. (d) 118. The crew were on board (a) / and they soon busied
93. The number of marks carried by each questions (a) / are themselves (b) / in preparing to meet the storm. (c) / No
error (d)
indicated (b) / at the end of the question. (c) / No error (d)
119. The statement made by the writer (a) / appears to be
94. It is tme (a) / we did something (b) / to stop road accidents
incorrect (b) / as Gandhiji was not born in Ahmedabad. (c) /
(c) / No error. (d)
No error (d)
95. An animal (a) / can be just as unhappy in a vast area (b) / or
120. With the advancement of winter, (a) / the days grow shorter
in a small one. (c) / No error (d)
(b) / while the nights grow longer. (c) / No error (d)
96. This is an excellent site (a) / for a stadium (b) / and which we
121. Amar introduced me (a) / to his friend (b) / who is a scientist
should like to acquire (c) / No error (d)
and an engineer. (c) / No error (d)
97. If you absent from (a) / college your name is likely (b) / to
122. Students should work (a) / hard in order to (b) / build their
be struck off the rolls (c) / No error (d) carrier. (c) / No error (d)
98. When I lived in England (a) / it was natural for me (b) / to 123. There is still (a) / little tea (b) left in the cup. (e) / No error (d)
speak a lot of English. (c) / No error (d) 124. I will spend (a) / my remaining life (b) in the village. (e) / No
99. Neither he is (a) / nor his wife is (b) / coming. (c)/ No error (d). error (d)
y
o
u
Spotting Errors C-83

rs
m
125. The future of food companies (a) /seems quite secure(b) / 141. The accused refused (a)/ to answer to the policeman (b)/ on

a
owed to ever-growing demand(c) / No error (d). duty. (c)/ No error (d).

h
b
126. The vaccine (a) / when hit the Indian market (b) / is dogged 142. What is (a)/ the use of me (b)/ attending the session? (c)/

o
o
by controversy (c) / No error (d) No error (d).

b
.w
127. His son (a) / is working (b) / very hardly (c) / No error (d) 143. We met our prospective employer (a)/ for a briefing session

o
128. Do you know that it was I (a) / who has done (b) / this piece (b)/ in the Taj Hotel. (c)/ No error (d).

rd
of beautiful work (c) / No error (d) 144. Because of the severe snow storm and the road blocks, (a)/

p
re
the air force dropped food and (b)/ medical supplies close
129. The company has ordered (a) / some (b) / new equipments

s
to the city. (c)/ No error (d).

s
(c) / No error

.c
145. If I would have realised (a)/ what a bad shape our library is

o
130. If you had told me (a)/ I would have helped you (b) / solve

m
in (b)/ I would have done something, to arrest the
the problem (c) / No error (d)
deterioration. (c)/ No error (d).
131. The Arabian Nights(a) / are indeed (b)/ an interesting book
146. He has been (a)/ enhanced in position (b)/ as result of his
(c)/ No error (d)
diligence and integrity. (c)/ No error (d).
132. He (a)/ loved her (b)/ dispite of himself (c)/ No error (d) 147. It is I (a)/ who is responsible (b)/ for the delary. (c)/ No error
133. Of all the models (a)/ Sessica is (b)/ the more good -looking (d).
on (c)/ No error (d) 148. There is only one cure (a)/ to the evils which newly (b)/
134. When I went there (a)/ Charles is playing (b)/ a game of acquired freedom produces and that cure in freedom. (c)/
chess (c)/ No error (d). No error (d).
135. Mohans' eyes (a)/ reflect a hope (b)/ for a better future in 149. He flew (a)/ over extensively (b)/ the Pacific last winter (c)/
No error (d).
Microsoft.(c)/ No error (d).
150. In 1906 a earthquake (a)/ destroyed much (b)/ of San
136. He went to Mumbai (a)/ with a view (b)/ to secure a job. (c)/ Francisco (c)/ No error (d).
No error (d).
151. His parents does not (a)/ approve of (b)/ his business (c)/
137. The Headmaster with all his senior teachers (a)/ have come No error (d).
(b)/ to attend the meeting. (c)/ No error (d).
152. The college library is (a)/ not only equipped with (b)/ very good
138. The teacher said that (a)/ the building adjacent with his books but also with the latest journals (c)/ No error (d).
house (b)/ needed repairs. (c)/ No error (d).
153. The lovers walked (a)/ besides each other (b)/ in silence (c)/
139. Grapes (a)/ cannot gathered (b)/ from thistles. (c)/ No error (d). No error (d).
140. Air pollution, together with littering, (a)/ are causing many 154. Men are wanted (a)/ for the army (b)/ and the navy, and the
problems (b)/ in our cities. (c)/ No error (d). air force, (c)/ No error (d).
y
o
u
C-84 Spotting Errors

rs
m
ANSWER KEY

a
h
1 (c) 26 (d) 51 (d) 76 (c) 101 (b) 126 (c) 151 (a)

b
o
2 (d) 27 (c) 52 (d) 77 (c) 102 (b) 127 (c) 152 (d)

o
b
3 (b) 28 (c) 53 (d) 78 (c) 103 (c) 128 (b) 153 (b)

.w
4 (c) 29 (b) 54 (c) 79 (b) 104 (a) 129 (c) 154 (c)

o
rd
5 (a) 30 (c) 55 (b) 80 (a) 105 (b) 130 (a)

p
re
6 (c) 31 (d) 56 (b) 81 (d) 106 (b) 131 (b)

s
s
7 (c) 32 (a) 57 (b) 82 (d) 107 (d) 132 (c)

.c
o
8 (d) 33 (b) 58 (c) 83 (c) 108 (c) 133 (c)

m
9 (d) 34 (c) 59 (b) 84 (d) 109 (d) 134 (b)
10 (b) 35 (c) 60 (a) 85 (b) 110 (c) 135 (b)
11 (d) 36 (d) 61 (d) 86 (d) 111 (c) 136 (c)
12 (c) 37 (b) 62 (d) 87 (d) 112 (b) 137 (b)
13 (d) 38 (b) 63 (d) 88 (c) 113 (c) 138 (b)
14 (c) 39 (b) 64 (d) 89 (b) 114 (d) 139 (b)
15 (d) 40 (b) 65 (a) 90 (c) 115 (b) 140 (b)
16 (c) 41 (b) 66 (c) 91 (b) 116 (c) 141 (b)
17 (d) 42 (d) 67 (a) 92 (a) 117 (a) 142 (b)
18 (d) 43 (a) 68 (b) 93 (b) 118 (a) 143 (c)
19 (d) 44 (a) 69 (d) 94 (d) 119 (d) 144 (d)
20 (c) 45 (b) 70 (b) 95 (c) 120 (a) 145 (a)
21 (c) 46 (b) 71 (c) 96 (c) 121 (c) 146 (b)
22 (b) 47 (a) 72 (c) 97 (a) 122 (c) 147 (b)
23 (a) 48 (c) 73 (c) 98 (d) 123 (b) 148 (b)
24 (c) 49 (b) 74 (a) 99 (a) 124 (b) 149 (b)
25 (d) 50 (c) 75 (c) 100 (c) 125 (c) 150 (a)

1. (c) In place of ‘than’ it will be ‘as’. 8. (d) or by taking up other


2. (d) and indulging in other sports. Or is a Co-ordinating Conjunction that joins together
3. (b) I was told two statements or clauses of equal rank or importance.
The subject should come before the verb. 9. (d) he did
4. (c) with the debasing of the coinage than 10. (b) men being tested
5. (a) have done The present (passive) participle, being tested ought to
The present perfect tense is used to denote an action be used here to maintain the uniformity in the Sentence.
beginning at some time in the past and continuing upto 11. (d) gasp
the present moment as the words “are still doing” in The sentence demands the use of the simple present
the sentence reveal. tense.
6. (c) have been 12. (c) is
The present perfect tense is used to express past ac- The simple present is used to express general truths.
tions whose time is not given and not definite. 13. (d) it’s
7. (c) had started A collective noun takes a singular verb when the col-
The past perfect tense helps in telling us which action lection is thought of as one whole; a plural verb when
happened earlier than the other when both the actions the individuals of which it is composed are thought
referred to happen in the past. The simple past is used of, as, The Committee has issued its report. The Com-
in one clause and the past perfect in the other. mittee are divided on one minor point.
y
o
u
Spotting Errors C-85

rs
m
14. (c) but milder than 36. (d) himself

a
‘Than’ (her used as a preposition) shows the relation 37. (b) except in the case

h
between this year and last year’s attack. It is more ap- 38. (b) has not

b
o
propriate than ‘as’ which generally shows the relation The Preposition ‘if is not needed.

o
b
between two similar things. 39. (b) was. (singular subject)

.w
15. (d) that 40. (b) and

o
The use of the conjunction ‘that’, to introduce the The conjunction ‘and’ is needed to join the two simple

rd
adverb clause of reason or cause, is more apt here as sentences in this compound sentence.

p
re
because means ‘for the reason that’. 41. (b) who

s
16. (c) The-adverb of time, ‘never’ and not the conjunction Who/What fears?

s
.c
‘neither’ is to be used. This is also done to never have 42. (d) that have

o
the construction in the same series. When the subject of a verb is a relative pronoun, care

m
17. (d) was being dragged by the soldiers should be taken to see that the verb agrees in number
All qualifying words, phrases and clauses should be and person with the antecedent of the relative. In this
placed as near as possible to the words to which they case the antecedent is “novels”.
refer. 43. (a) The building, which is the work
18. (d) them The adjective clause must be joined by the relative pro-
The Accusative Case of the third person plural ‘them’ noun ‘which’, which is used for things without life and
and not the Nominative ‘they’ is to be used as the for animals.
other people are the object of the sentence. 44. (a) Having heard
19. (d) they were not to open The perfect participle generally precedes a sentence in
The infinitive ‘to open’: like a noun needs a subject the past tense.
which the verb ‘were’ fulfils. 45. (b) it being
20. (c) the members felt 46. (b) but
The subject usually comes before the verb in a sen- Except is generally followed by the preposition for.
tence. In English, owing to the fewness of the inflec- 47. (a) Unlike
tions, the order of the words in a sentence is of the The education class is being compared and not the chil-
first importance. dren.
21. (c) given to 48. (c) The article is to be omitted
22. (b) accentuated The article is omitted before abstract nouns used in a
The word above means to emphasize or make promi- general sense.
nent. Note— An abstract noun, when it is qualified by an
23. (a) ovation adjective or an adjectival phrase or clause
The word above means an enthusiastic reception es- may have the article; as ‘the wisdom of Solomon is
pecially spontaneous and sustained applause. famous’.
24. (c) action 49. (b) a wife bringing a dowry
The indefinite article will be used as he is not referring
The word erosion should be followed by the prepo-
to any particular wife.
sition ‘by’.
50. (c) The personal pronoun “they” is redundant (superfluous).
25. (d) necessities
51. (d) The phrase indicated by (d) should follow (a), ideally or
The word above is the plural of the noun ‘necessity’.
be eliminated.
26. (d) a different nature.
52. (d) The phrase indicated by (d) should be placed at the
The article ‘a’ is used before a common noun, singular beginning of the sentence or be eliminated.
number. 53. (d) proud and insolvent.
27. (c) it was received When one noun follows another to describe it, the noun
The verb has to have an object. which follows is said to be in apposition to the noun
28. (c) my being which comes before it. Part (d) is changed to have the
The speaker himself is being prevented, hence the pos- series in the same construction.
sessive case of the first person (i.e. my) is used. 54. (c) far better
29. (b) looked picturesque The comparative ‘better is needed here.
30. (c) hung 55. (b) such sort of things
The past tense of ‘hang’ is to be used as the verb The adjective ‘such’ is needed here to qualify ‘sort of
‘found’ indicates. things’ and not the demonstrative pronoun ‘those’.
31. (d) begin in 56. (b) Speeches indicated \ revealed.
32. (a) lay Inferred means to deduce from facts, hence the speeches
The past tense of the verb ‘lie’ is lay. can’t ‘infer’.
33. (b) as quick as 57. (b) had not been
34. (c) no fewer than 58. (c) yet
35. (c) if not more than 59. (b) last
More is a comparative adjective and the sentence must 60. (a) has not introduced
specify which two things are being compared. The past tense of the verb is needed here.
y
o
u
C-86 Spotting Errors

rs
m
61. (d) in life 85. (b) a

a
The article is omitted before a noun used in its widest Only in the sense of merely is not to used as “part and

h
sense, as; Man is mortal. What kind of flower is it? parcel” (an idiomatic expression) means an essential part

b
o
62. (d) to believe and the “it” referred to in the sentence is apparently

o
b
The Present Infinitive ‘to be believed’, is not needed. important enough not to be missed.

.w
63. (d) harmless 86. (d) by eloquence

o
‘Than’ as a conjunction follows adjectives and adverbs 87. (d) uplift them

rd
in the comparative degree. The opposite of ‘harmful’ is The phrase ‘pull up’ means to stop or cause to stop

p
re
‘harmless’ and not beneficial. moving.

s
64. (d) was 88. (c) gossiping/not to gossiping

s
.c
The subject is the “block”. 89. (b) marry

o
65. (a) will excuse my The use of the adverb away is not apt here.

m
The pronoun or noun governing a Gerund should be 90. (c) for themselves
put in the Possessive case. The word ‘leaving’ is a Ger- The preposition ‘for’ is needed here and not ‘among’.
und (i.e. Verbal Noun). Therefore it must be preceded 91. (b) Replace ‘the’ by ‘an’
by the possessive form.
92. (a) The correct form is ‘If he were to .....’
66. (c) was
93. (b) Replace ‘are by ‘is’
67. (a) The team were
94. (d) No error
68. (b) him
95. (c) Replace ‘or’ by ‘as’
The object of a verb / preposition, when it is a pronoun
96. (c) Replace ‘should’ by ‘would’
should be in the Objective / Accusative form.
69. (d) me 97. (a) Add ‘yourself’ afer ‘absent’
70. (b) The complement of the verb ‘to be’ when it is expressed 98. (d) No error
by a pronoun should be in the Nominative form, as; If I 99. (a) Remove ‘is’
were he, I wouldn’t do it. 100. (c) Replace ‘until’ by ‘by’
I am she whom you want. 101. (b) Replace 'the best' by 'better'
71. (c) has now begun 102. (b) Replace 'furnitures' by 'furniture'
72. (c) seldom or never 103. (c) Replace 'then' by 'than'
Seldom or never and seldom if ever are both correct but 104. (a) Replace 'than' by 'to'
seldom or ever is incorrect 105. (b) Replace 'where you were' by 'where he was'
73. (c) his 106. (b) Replace 'for' by 'of'
74. (a) a 107. (d) No error
The indefinite article is to be used as the sentence 108. (c) Remove 'to discuss'
doesn’t specify the particular street. 109. (d) No error
75. (c) think 110. (c) Remove 'will'
The word ‘they’ should be omitted as the relative pro- 111. (c) Replace 'to accept' by 'to be accepted'
noun ‘who’ already refers or relates (i.e. carries us back) 112. (b) Replace 'evidences' by 'evidence'
to the noun before it i.e. it’s antecedent (wise people). 113. (c) Add 'the' before 'Ganges' and 'Himalayas'
76. (c) that 114. (d) No error
The conjunction ‘that’ is more apt here to express the 115. (b) Replace 'although' by 'though'
reason / cause. 116. (c) Replace ‘on hehalf’ by ‘on the part of ‘behave’ means
77. (c) since ‘in place of someone’
since is used before a noun or phrase denoting some 117. (a) replace ‘said’ by ‘asked’
point of time and is preceded by a verb in some perfect 118. (a) Replace ‘meet’ by ‘face’
tense. From is used with all tenses. 119. (d) No error
78. (c) as they 120. (a) Replace ‘advancement’ by ‘advent’
The subordinating conjunction ‘as’ here introduces the 121. (c) The correct form is ‘who is both a scientist and an
Adverb Clause of Time and not the word ‘like’ as a engineer’ or ‘who is a scientist as well as an engineer’
preposition. 122. (c) Replace 'carrier' by 'career'
79. (b) whom you 'Carries' is something that carries
80. (a) Neither does he pay 123. (b) a little
Care should be taken, when using correlative conjunc- 124. (b) Replace 'my remaining life' by 'the rest of my life'
tions, that they are followed by the same part of speech. 125. (c) Owing to (preposition) means : Because of “hence
81. (d) Part (d) should either be placed after part(a) or be elimi- owing to ever growing demand” should be used here
nated. 126. (c) “Was dogged by controversy” should be used.
82. (d) Part (d) should be placed after part (b).
127. (c) ‘very hard’ should be used
The phrase ‘more than once’ qualifies the thought of
128. (b) “who have done” should be used
marrying her and not the action of marrying her.
83. (c) interact 129. (c) The word equipment is uncountable noun hence
84. (d) wine and snacks “equipment” shold be used in place of “equipments.”
y
o
u
Spotting Errors C-87

rs
m
130. (a) ‘Told’ should be replaced as ‘asked’ statement is 'topper'. Correct application of word

a
131. (b) “The Arabian Nights” name of book and is singular enriches the meaning of the sentence.

h
b
hence “ is indeed should be used in place of “are indeed” 147. (b) Part 'b' of the sentence is wrong. The correct form of

o
o
132. (c) “dispite himself” should be used because dispite of sentence is 'It is I am who should be responsible for

b
himself is improper word.

.w
the delay'.
133. (c) more than one degree can not be used for single

o
148. (b) Part 'b' of the sentence is wrong. The preposition 'of' is

rd
subjects. missing in that part. 'Of' is applicable after the word

p
134. (b) “Charles was playing” because events shows past

re
'evils'.
time.

s
s
149. (b) Part 'b' of the sentence is wrong. It will be arranged as

.c
135. (b) Here Mohan's eyes reflect means that it was Mohan's
'over extensively'. In the part 'B' 'extensively over' is

o
habit which is not the case. Hence, it should be as

m
Mohan's eyes reflected ..... denoting wrong application of words which is acting
136. (c) With a view to (doing) something because you are as a breach to the meaning of the sentence.
planning to do something in the future. Ex: We bought 150. (a) In 1906 an earthquake destroyed much of San
the house with a view to retiring there. Francisco. Instead of 'a' before earthquake, article 'an'
137. (b) 'has come' is the correct option because the verb will should be used because it is followed by a vowel letter.
agree with the first subject. 151. (a) His parents don't approve of his business. In English,
138. (b) adjacent always takes the preposition 'to'. don't is used when speaking in the first and second
139. (b) Grapes cannot be the subject of the sentence; hence it person plural and singular and the third person plural
should be 'Grapes cannot be gathered......' ("I," "you," "we," and "they").Doesn't, on the other
140. (b) ‘are’ is replaced by ‘is’ hand, is used when speaking in the third person
141. (b) Remove ‘to’ before the policeman singular only ("he," "she," and "it").
142. (b) ‘me’ is replaced by ‘myself’ 152. (d) No error.
143. (c) ‘in’ is replaced by ‘at’ 153. (b) The lovers walked beside each other in silence. (the
144. (d) No error preposition besides means in addition to, whereas
145. (a) Part 'a' of the statement is wrong. It will be as 'If I had beside means next to)
realized'. The Past perfect tense suits here and it denotes 154. (c) Men are wanted for the army, the navy and the air
the sentence to be in Active voice. force. When joining two or more grammatically similar
146. (b) Part 'b' of the statement is wrong. The word 'enhanced' expressions, and is used followed in the end and
given here is wrong. The correct word that will suit the commas are used to distinguish the other expressions.
y
o
u
C-88 Sentence Improvement

rs
m
CHAPTER

a
h
SENTENCE

b
8

o
o
b
.w
IMPROVEMENT

o
rd
p
re
s
s
.c
o
m
DIRECTIONS (Qs. 1-219): In each of these questions, in the 6. Since we are living in Bombay for five years, we are reluctant
given sentences, a part of the sentence is underlined. Beneath to move to another city.
each sentence, four different ways of phrasing the underlined (a) Being that we living
part are indicated. Choose the best alternative. In case no (b) Since we were living
improvement is needed, your answer would be ‘No improvement’. (c) Since we have been living
1. Expect for you and I, everyone brought a present to the (d) Being that we have been living
party. 7. As a child, my parents took me to Jammu to visit my
(a) With the exception of you and I, everyone brought grandmother.
(b) Except for you and I, everyone had brought (a) When I was a child, my parents took me to Jammu to
(c) Except for you and me, everyone brought visit my grandmother
(d) Except for you and me, everyone had brought (b) My parents took me, as a child, to Jammu to visit my
2. Although I calculate that he will be here any minute, I cannot grandmother
wait much longer for him. (c) My parents took me to Jammu to visit my grandmother
(a) Although I reckon that he will be here as a child.
(b) Although I think that he will be here (d) A child, my parents took me to Jammu to visit my
(c) Because I am confidant that he will be here grandmother
(d) Because I calculate that he will be here 8. Anyone interested in the use of computer can learn much if
3. We want the teacher to be him who has the best rapport you have access to a state-of-the-art microcomputer.
with the students.
(a) if he has access to (b) if access is available to
(a) We want the teacher to be he
(c) by access to (d) from access to
(b) We want him to be the teacher
(c) We desire the teacher to be him 9. Start the motor, and then you should remove the blocks.
(d) We anticipate the teacher to be him (a) Start the motor, then removing the blocks
4. Today this is a totally different world than we have seen in (b) Starting the motor, the blocks should then be removed
the last decade. (c) Start the motor and then remove the blocks
(a) than what we seen (d) Starting the motor remove the blocks
(b) then we have seen 10. The Vice President of the local bank spoke for a half an hour
(c) from what we seen and told his colleague that he, his colleague, must consider
(d) from what we have seen finding a new job, or accept a reduction in salary.
5. Although he was the most friendly of all present and (a) for a half hour and told his colleague that he must
different from the others, he hadn’t hardly any friends consider to find a new job
except me. (b) for half an hour and told his colleague that the
(a) different from the others, he hardly had any friends colleague must get employed by a new bank
except I (c) for half an hour and told his colleague to consider
(b) different than the others, he hardly had any friends
finding, a new job
except me
(d) no change
(c) different than the others, he hardly had any friends
11. He does not smoke, nor he drinks.
except I
(d) different from the others, he hardly had any friends (a) nor he does drink (b) neither he does drink
except me (c) nor does he drink (d) no change
y
o
u
Sentence Improvement C-89

rs
m
12. She wished that her career could be as glamorous as the 21. British Airspace has been focusing on building European links.

a
other women but not willing to work as they had. (a) concentrating on creating European links

h
b
(a) as glamorous as the other women’s (b) focussing on forging European links

o
(b) more glamorous than the career of the other women/

o
(c) stressing on building European connectivity

b
(c) glamorous

.w
(d) no change (d) stressing on building European links

o
22. In Southern Bengal, many banyan trees are planted at the

rd
13. Although he is liable to make political enemies with the

p
decision, the Finance Minister will propose severe tax cuts end of a village or at the border of a district, for providing

re
that may both stimulate business and reduce the availability excellent shade at Summers, and shelter during Winter from

s
s
of loans. the bitter winds.

.c
o
(a) liable from (b) able to (a) to provide excellent shade in Summer and, in Winter,

m
(c) of a mind to (d) no change shelter from the bitter winds
14. No matter what experience you have had with forest fires, if (b) providing excellent shades of summer, and shelters of
you would have witnessed the fire roaring down through bitter winter winds
the canyon, you would have been terrified.
(c) in order to provide shade in excellent summer, and
(a) if you witnessed (b) if you had witnessed
shelter from the winds of bitter winter
(c) if you could witness (d) no change
15. Eaten in Portugal only, the Indians viewed the potato with (d) for providing excellent shades at summer, and shelter
suspicion for they assumed it had poisonous properties from bitter winds in summer
since only the white skinned people consumed it. 23. Standards of reportage in the profession have actually risen
(a) Indians viewed the potato with suspicion for they in recent years, with specialists lending credibility for the
(b) Indians were suspicious of the potato and they printed word.
(c) potato was viewed with suspicion by Indians who (a) with specialists who have lend credibility to the printed
(d) potato was suspicious to Indians, and it was word
16. Though he was more faster than his opponent on the field, (b) with specialist lending credibility to the printed word
his chances of winning the race was low as he lacked the
(c) by specialist lending credibility in the printed word
killer instinct.
(a) Though he was more faster than his opponent on the field (d) as specialists lending credibility to the printed word
(b) As he was more faster than his opponent on the field 24. Knowing the area was prone to earthquakes, all the
(c) Though he was more fast from his opponent on the field buildings were reinforced with additional steel and concrete.
(d) Though he was faster than his opponent on the field (a) Having known that the area was prone to earthquakes
17. The local library has recommended that the books put up (b) Since they knew the area was prone to earthquakes
for the used book sale should be in good condition and (c) Since the area was known to be prone to earthquakes
should have no writing in them or be underlined.
(d) Being prone earthquakes
(a) and should have no writing in them or be underlined
(b) and should not have writing in them or not be 25. Visiting Agra for the first time, the Taj Mahal in moonlight
underlined was truly impressive.
(c) and contain no writing or underlining (a) Visiting Agra for the first time, the Taj Mahal in
(d) without containing writing or underlining moonlight was truly impressive
18. The news of her employment soon circulated around the (b) Visiting Agra for the first time, I found the Taj Mahal
small town. truly impressive in the moonlight
(a) circulated round the small town (c) Visiting Agra for the first time, I found the Taj Mahal in
(b) circulated in the small town moonlight truly impressive
(c) was circulating across the small town
(d) Visiting Agra the first time, I find the Taj Mahal in
(d) was circulating within the small town
19. It is the craziness for speeding that is maddening that is moonlight was truly impressive
responsible for many motor accidents. 26. The policemen attempted to tactfully pacify the quarreling
(a) the craziness for speeding that is mad families.
(b) the mad craze for speed (a) The policemen attempted to tactfully pacify the
(c) the mad craze for speeding quarreling families
(d) the craze for speeding that is maddening (b) The policemen tactfully attempted to pacify the
20. If they cooperate together by dividing up the work, they quarreling families
shall be over with the work faster. (c) The policemen attempted tactfully to pacify the
(a) if they cooperate together by dividing the work
quarrelsome families.
(b) if they cooperate by dividing up the work
(d) The policemen attempted to pacify tactfully the
(c) if they cooperate by dividing the work
(d) if they cooperate together by division of work quarreling families
y
o
u
C-90 Sentence Improvement

rs
m
27. All the allegations levelled against him were found to be 41. The summer has set out. and the days are getting warm.

a
baseless. (a) set up (b) set in

h
b
(a) levelled for (b) level with (c) set off (d) No improvement

o
(c) level against (d) No improvement

o
42. She could not help but laugh.

b
28. He did many mischiefs. (a) laughing (b) but laughing

.w
(a) made many a mischiefs (c) laugh (d) No improvement

o
rd
(b) made much mischief 43. Im sorry, but I don’t believe what you say.

p
(a) I will not believe (b) I am not believing

re
(c) committed many mischiefs
(c) I will not be believing (d) No improvement

s
(d) No improvement

s
.c
29. He has said so out of affection, do not take it to heart. 44. Let’s buy a new sari with the annual bonus, can we?

o
(a) can’t we (b) don’t we

m
(a) in heart (b) it in the heart
(c) by the heart (d) No improvement (c) shall we (d) No improvement
45. Since 1986, there is no earthquakes here.
30. I would gladly accompany your sister if you had asked.
(a) have been (b) are
(a) would have gladly accompanied
(c) were being (d) No improvement
(b) was to have gladly accompanied
46. It is time the six year old is learning how to read and write.
(c) will gladly accompany
(a) has learned (b) learnt
(d) No improvement
(c) was learning (d) No improvement
31. What are needed are not large houses but small cottages.
47. He plays cricket and tennis also.
(a) were (b) was (a) both (b) besides
(c) is (d) No improvement (c) too (d) No improvement
32. Sumit would have been looked smart in traditional clothes. 48. I am used to hard work.
(a) was looked (b) would be looked (a) hard working (b) work hard
(c) had looking (d) would have looked (c) work hardly (d) No improvement
33. Hardly had I finished writing the letter before Anil arrived. 49. Practically every part of the banana tree is used by man.
(a) then (b) while (a) most part (b) any part
(c) when (d) as (c) each part (d) No improvement
34. The main point of his speech was well understood. 50. No sooner I saw the tiger than I ran away.
(a) that he spoke (b) in the speech of his (a) No sooner I had seen (b) No sooner did I see
(c) made when he spoke (d) No improvement (c) As soon as I saw (d) No improvement
35. His father won’t be able to leave for Varanasi until they 51. Try to be diplomatic when you refuse any invitation, so for
have arrived. not to cause bad feeling.
(a) until they arrive (a) as far not (b) so as not
(b) until they will arrive (c) if as far not (d) No improvement
(c) until they will have arrived 52. The need of the hour was some fast action on the part of
(d) No improvement the leaders.
36. He returned with two burning mugs of coffee and set them (a) needed (b) needing
down on her desk. (c) needs (d) No correction required
53. The indecisive man was readily persuaded to change his
(a) heating (b) steaming
mind again.
(c) blazing (d) No improvement
(a) was persuaded ready (b) was ready to persuade
37. People are tiring very soon in an activity which they don’t like.
(c) was ready persuaded (d) No improvement
(a) tried (b) trying
54. The doctor examined my pulse.
(c) tired (d) tyring
(a) observed (b) saw
38. They are working for the upliftment of their village. (c) felt (d) No improvement
(a) uplift of (b) uplifting of 55. He doesn’t hesitate to do whatever his brother does.
(c) uplifting (d) No improvement (a) will do (b) would do
39. They felt humiliated because they realised that they had (c) shall do (d) No improvement
cheated. 56. The teacher asked the intruder who was he and why was he
(a) had been cheating (b) had been cheated occupying his chair.
(c) have been cheated (d) were to be cheated (a) who he was and why he was
40. Is there any place for me to sit? (b) who he was and why was he
(a) space (b) room (c) who he had been and why he had been
(c) area (d) No improvement (d) No improvement
y
o
u
Sentence Improvement C-91

rs
m
57. Bad habits must be nipped at the bud. 73. They have stopped from constructing new buildings.

a
(a) nipped off the bud (b) nipped in the bud (a) to contruct (b) at constructing

h
b
(c) nipped onthe bud (d) No improvement (c) constructing (d) No improvement

o
58. It is no good to cry over spilt milk.

o
74. All over Russia, Indian films are more popular than those in

b
(a) It is no good crying (b) It is of no good to cry any other country.

.w
(c) It is of no good crying (d) No improvement (a) in (b) that in

o
rd
59. To various practices and norms for bank’s transactions are (c) that of (d) those of

p
laid down by the Reserve Bank of India. 75. Wise men catch time by the forelock.

re
(a) are laid up (b) are led down

s
(a) hold (b) seize

s
.c
(c) are lead up (d) No improvement (c) take (d) No improvement

o
60. No one needs to worry about me. 76. Seeing the injustice done to them by the Britishers, the

m
(a) will worry (b) need worry sepoys broke into open revolt.
(c) shall worry (d) No improvement (a) broke to (b) broke upto
61. When the soldiers realised that they had been fooled, they (c) tesorted to (d) No improvement
rush back. 77. The cloud of misfortunes appears to have blown out.
(a) had rush (b) have rush (a) away (b) over
(c) had rushed (d) rushed (c) up (d) No improvement
62. The officer looked on my complaint. 78. Anand has the guts to rise from the occasion and come out
(a) looked of (b) looked into successful.
(c) looked to (d) No improvement (a) to rise against (b) to rising from
63. To actress said that it sometimes took her two hours to put (c) to rise to (d) to rise with
her make -up. 79. A bird in hand is worth two in bush.
(a) put over (b) put up (a) two in the bush (b) two at a bush
(c) put on (d) No improvement (c) two on bush (d) No improvement
64. The matter must be considered in every point of view. 80. Scarcely had he entered the room than the phone rang.
(a) with (b) from (a) when (b) while
(c) at (d) No improvement (c) as (d) No improvement
65. He enjoys to tell stories to children. 81. Five years ago today, I am sitting in a small japanese car,
(a) how to tell stories (b) telling stories driving across Poland towards Berlin.
(c) to narrate stories (d) No improvement (a) was sitting (b) have been sitting
66. Manisha purchased the very good of all the saris kept in (c) sat (d) No improvement
the shop. 82. The custom of many centuries ago origin is slowly
(a) the better (b) the very better disappearing.
(c) the most good (d) the best (a) which was originated ago many centuries
67. On seeing the lion she felt too much afraid. (b) originating for many centuries
(a) very much (b) excessively (c) which orginated many centuries ago
(c) much (d) No improvement (d) with many centuries of origin
68. Last evening I sent to the optician and bought spectacles. 83. If you had told your problem yesterday, we might had helped
(a) a spectacle (b) two spectacles you.
(c) a pair of spectacles (d) No improvement (a) would have (b) might have been
69. Education is a strong instrument for modulding the character (c) would have been (d) No correction required
of the young.
84. He stayed back so that it can look as if he was unaware of
(a) striking (b) powerful the entire incident.
(c) potent (d) No improvement (a) may look (b) would look
70. One of the men gave first aid to Hitesh who is injured in a
(c) will look (d) No improvement
road accident.
85. When he heard the rhetorical speech of the leader, he was
(a) who had injured (b) who was injured
carried along by his enthusiasm.
(c) which was injured (d) as he was to injure
(a) was carried away (b) was carried down
71. Mrs. Johnson had staying here since 1954 and has made
(c) was carried aloft (d) No improvement
India her home
86. The members of his family are coming in this train.
(a) will stay (b) was to stay
(a) with (b) by
(c) is staying (d) has been staying
(c) on (d) No improvement
72. The problems of translation are still remain.
87. If he had time he will call you.
(a) are remain (b) will remained
(a) would have (b) would have had
(c) will still remain (d) No improvement
(c) has (d) No improvement
y
o
u
C-92 Sentence Improvement

rs
m
88. The intruder stood quietly for few moments. 103. The huge Negro would have been a comic figure if he had

a
(a) for few time (b) for a few moments not been terrific, for he was dressed in a very loud grey

h
b
(c) for the few moments (d) No correction required check suit with a flowing salmon coloured tie.

o
o
89. A lot depends on your early brought up in the family. (a) must have been (b) may have been

b
(a) bringing up (b) bringing on (c) should have been (d) No improvement

.w
(c) upbringing (d) No improvement 104. Not a word they spoke to the unfortunate wife about it.

o
rd
90. Many believed that girls who received western education (a) they had spoken (b) did they speak

p
would make slaves of their husbands. (c) they will speak (d) No improvement

re
(a) could receive (b) had received 105. Young men and women should get habituated to reading

s
s
.c
(c) have received (d) No improvement and writing about corrent affairs.

o
91. I have not written many letter to him since my father had (a) prepared (b) trained

m
died. (c) used (d) No improvement
(a) died (b) was dead 106. The bill will be paid at sight or on demand
(c) has died (d) No improvement (a) at a sight or on a demand
92. The manager was unhappy at Gaurav because he did not (b) on sight or at demand
complete the work in time. (c) at sight or demand
(a) is unhapy at (b) is to be unhappy at (d) No improvement
(c) was unhappy with (d) No correction required 107. It is difficult for a simple person to see through her artificial
93. In the modern world it is difficult to live through one’s ideals. schemes.
(a) to live upto (b) to live by (a) artistic (b) artful
(c) to live for (d) No improvement (c) aesthetic (d) No improvement
94. It is long since I last saw you. 108. You ought not to have gone there, but you did.
(a) long time (b) long before (a) be going (b) have been
(c) long ago (d) No improvement (c) go (d) No improvement
95. She gave most of her time to music. 109. Being worked for the whole day, you should have taken
(a) devoted (b) spend some rest and started the work tomorrow
(c) lent (d) No improvement (a) After having work (b) Having worked for
96. When I was fourteen, I sat the entrance examination for (c) Working for (d) After being worked for
senior secondary school. 110. Ravi has got many friends because he has got much money.
(a) sat for (b) sat in (a) enough money (b) a lot of money
(c) sat at (d) No improvement (c) bags of money (d) No improvement
97. He has been working off and on for several years to compile 111. You must try making him to understand.
a dictionary. (a) make him understand
(a) on or off (b) on and off (b) to making him understand
(c) regularly (d) No correction required (c) to make him understand
98. Rajni unnecessarily picked up a quarrel with Kashish and (d) No improvement
left the party hurriedly. 112. He has cooked that meal so often he can do it with his eyes
(a) picking up (b) picked on closed.
(c) picked (d) has picked up (a) mind blank (b) eyes covered
99. She did not like the movie, nor I did. (c) hands full (d) No improvement
(a) nor did I (b) nor I like it 113. Because of his mastery in this field, his suggestions are
(c) nor I liked it (d) No improvement wide acepted.
100. It will take two hours to walk across the forest. (a) are widely accepted (b) are wide acceptance
(a) over (b) between (c) have widely accepted (d) have been wide accepted
(c) away (d) through 114. The fact finding committee has so far not made any
101. The unemployment rate has dropped sharplly this month, advancement.
through it may only be temporary. (a) progress (b) improvement
(a) but it may be only temporary (c) stride (d) No improvement
(b) but the drop may only be temporary 115. Who does not know that this was broadcasted ten days
(c) even though the drop may only be temporary ago?
(d) No improvement (a) had broadcast (b) was broadcast
102. In accordance to your instructions, we have remitted the (c) was broadcasting (d) No improvement
amount in the bank. 116. You are worned against committing the same mistake again.
(a) by (b) with (a) for committing (b) against to commit
(c) for (d) No improvement (c) to commit (d) No improvement
y
o
u
Sentence Improvement C-93

rs
m
117. The teacher gave the students some advice. 133. Can you tell me why did you not speak the truth?

a
(a) advise (b) advises (a) why did not you speak

h
b
(c) advices (d) No improvement (b) that why did you not speak

o
118. You must find someone to accompany you to Bombay.

o
(c) why you did not speak

b
(a) no one (b) everyone (d) No improvement

.w
(c) anyone (d) No improvement 134. The boy which come to see me this morning had come from Agra.

o
rd
119. No sooner he reached home than all the villagers gathered (a) that (b) whom

p
at his home to listen to his story. (c) who (d) No improvement

re
(a) would he reach (b) did he reach 135. Not till he got home he did realise that he had lost his keys

s
s
(c) had he reached (d) No improvement

.c
in the theatre
120. He is the kindest and generous of all my friends.

o
(a) did he realise (b) he did not realise

m
(a) more generous among all my frieds
(c) he realised (d) No improvement
(b) more generous of all my frieds
136. He has a house for letting.
(c) the most generous of all my friends
(a) for letting out (b) to let
(d) No improvement
(c) ro rent out (d) No improvement
121. None of the gusests were introduced to the bride
137. Had we been on time, we would not have missed the train.
(a) introduced (b) was introduced
(c) have been introduced (d) No improvement (a) Will not have (b) Might have
122. One should exercise their right to vote (c) Would have been (d) No improvement
(a) his (b) our 138. When we saw hism last, he ran to catch a bus.
(c) one’s (d) No improvement (a) was running (b) has run
123. He stopped to work an hour ago (c) had run (d) No improvement
(a) to working (b) to have worked 139. If you get confused while performing the act, just don’t
(c) working (d) No improvement stop to try.
124. Your results depend not only on how much you have (a) keep your try on (b) continue your tries
studied but how long you have read. (c) not give up trying (d) keep on trying
(a) but also how long 140. He insisted on he was innocent.
(b) but also on how long (a) insisted that (b) insisted on that
(c) but also on how much long (c) insists that (d) No improvement
(d) No improvement 141. It was indeed a shock for her, but she has later recovered
125. She cut a sad figure in her first performance on the stage. from it.
(a) made a sorry figure (b) cut a sorry face (a) afterwards (b) since
(c) cut a sorry figure (d) No improvement (c) then (d) No improvement
126. When the professor fell off the stool, the students could 142. I must study my books today as I have to give an examination
not avoid to laugh. tomorrow.
(a) avoid laughing (b) refrain to laugh (a) for I have to give (b) since I have to give
(c) avoid laughter (d) stop laughter (c) as I have to take (d) No improvement
127. What does agonise me mot is not this criticism, but the
143. Any body would like assist you for improvement in the
trivial reason behind it.
work.
(a) I most agonised (b) most agonising me
(a) you like to assist (b) like assist to you
(c) agonises me most (d) No improvement
(c) like to assist you (d) No correction required
128. All his answers were correct.
144. Two hours have elapsed since he had fallen asleep.
(a) His every answers (b) His all answers
(c) All of his answers (d) No improvement (a) fell (b) has fallen
129. He gave me an old scissor. (c) was fallen (d) No improvement
(a) old scissor (b) and old scissors 145. Due to these reasons we are all in favour of universal
(c) a pair of old scissors (d) No improvement compulsory education.
130. His trousers are three sizes large (a) by these reasons (b) for these reasons
(a) three sizes too (b) three size (c) out of these reasons (d) no improvement
(c) three sizes over (d) No improvement 146. He has very good command on English.
131. He offered me tea but I denied it. (a) of (b) over
(a) refuted (b) reglected (c) in (d) No improvement
(c) declined (d) No improvement 147. I am looking forward to see you soon.
132. There is a shortage of well qualified teachers of msot (a) looking forward towards seeing
subjects, specially of English and Economics. (b) looking forward for seeing
(a) especially of (b) especially in (c) looking forward to seeing
(c) in special of (d) No improvement (d) No improvement
y
o
u
C-94 Sentence Improvement

rs
m
148. His colleagues could not tell me why did he not come to 162. To get one’s name in the Rowland Ward’s book of hunting

a
office yesterday. records was he hot ambition of every serious hunter.

h
b
(a) why not had he come (b) why he did not come (a) burning (b) extreme

o
(c) why had he not come (d) No improvement (c) high (d) No improvement

o
b
149. I waited for half an hour for my fried, but he didn’t turn in. 163. She did not ask any question to him.

.w
(a) turn about (b) turn around (a) any question from him (b) him any question

o
(c) to him any question (d) No improvement

rd
(c) turn up (d) No improvement
164. On some evenings I leave office as late as seven o’clock.

p
150. I shall not go until I am invited.

re
(a) if not I am invited (b) till I am invited (a) In some evenings (b) Some evenings

s
(c) On some of the evenings (d) No improvement

s
(c) unless I am invited (d) No improvement

.c
165. Hold hands of your child while crossing the road.

o
151. Our departure from this place now depends on their coming here.

m
(a) they come (b) them coming (a) your child’s hands (b) your child’s hand
(c) hand of your child (d) No improvement
(c) they coming (d) their’s coming
166. There is a dearth of woman doctor in our state. We shall
152. He should not had done it.
have to recruit some from the other states.
(a) should had not (b) should not have (a) women doctor (b) woman doctors
(c) should have (d) had not (c) women doctors (d) No improvement
153. The assistant was asked that why he was generally late. 167. He he was likely to win the elections by the sweeping
(a) why was he (b) why that he was majority.
(c) why he was (d) that why he had been (a) by a sweeping majority
154. After the heavy rains last week, the water in the lake raised (b) with sweeping a majority
another two feet. (c) with the sweeping majority
(a) would raise another two feet. (d) No improvement
(b) rised another two feet 168. The policeman stopped him for jumping the red light but let
(c) rose another two feet him out.
(d) No improvement (a) Let him off (b) Let him through
155. It was quite clear that the runner could be able to improve (c) Let him by (d) No improvement
upon his own record. 169. You will be late if you do not leave now.
(a) will be able (b) should be able (a) will not leave (b) did not leave
(c) would be able (d) No improvement (c) left (d) No correction required
170. They invite I and my friend to tea.
156. They were all charmed by the sweet and sothing music
(a) my friend and I (b) my friend and me
(a) sweet and soothingly (b) sweetly but soothing
(c) to me and my friend (d) No improvement
(c) sweetened and soothing (d) No improvement 171. Every time I go in a life to my sixth floor apartment, I remember
157. His father wrote to him, “it is high time your start preparing the calm and serenity of my ancestral home in the village.
for the forthcoming examination”. (a) move in a lift (b) take a lift
(a) would start (b) started (c) ascend in a lift (d) No improvement
(c) hard started (d) No improvement 172. The shopkeeper said graciously, “I II quote no price; the
158. The standard of living of the working class is higher than article is yours for asking.”
was the case thirty years ago. (a) for the asking (b) if you ask
(a) than it was before thirty years (c) for only asking (d) No improvement
(b) than that of thirty years ago 173. Rohi assured Sunita that he would look at her work while
(c) than what is was thirty years ago she was on leave.
(d) No improvement (a) would overlook (b) would look after
159. The child has been in the sick bed for the last one week. (c) would look down upon (d) will look
(a) for the past (b) for past 174. He told us the story in a nutshell.
(c) since the past (d) No improvement (a) in nutshell (b) in the nutshell
160. They are social insects, living in communities,regulated by (c) putting it in a nut (d) No improvement
definite laws, ech member of society bearing a well-defined 175. It was hard to believe that he was dead for two years.
and separate part in the work of a colony. (a) is dead (b) has been dead
(a) living amoung a community (c) had been dead (d) No improvement
(b) who are living in communities 176. He perfers to walk than to ride.
(c) who lives with a community (a) walking than riding (b) to walk over riding
(d) No improvement (c) walking to riding (d) No improvement
161. He has been growting weaker and his life now hangs like a 177. The accused now flatly denies have admitted his guilt in
thread. this first statement.
(a) hangs with threads (b) hangs by a thread (a) had admitted (b) having admitted
(c) hung on a thread (d) No improvement (c) had admitted (d) have been admitting
y
o
u
Sentence Improvement C-95

rs
m
178. The whole town was being decorated for the tourist week. 194. He compensated the loss to me.

a
(a) is to be decorated (b) will be decorated (a) He compensated the loss for me.

h
b
(c) was to be decorated (d) No improvement (b) He compensated me to the loss

o
179. The little child was knocked out by a car.

o
(c) He compensated me for the loss.

b
(a) up (b) away

.w
(d) No improvement.
(c) down (d) No improvement

o
195. As employees, we are accountable for our stakeholders.

rd
180. He asked me if I can help him to lift the box.
(a) accountable with (b) accountable to

p
(a) may (b) would

re
(c) will (d) No improvement (c) accountable against (d) No improvement

s
196. Recently he had insured for a mediclaim policy.

s
181. She tried to eat another leaf of lettuce but she could no

.c
longer fight the stench. (a) He had recently insured for

o
m
(a) face (b) keep (b) Recently he insured for
(c) stand (d) No improvement (c) He insured recently for
182. Have you not reached in time, we would have lost our lives. (d) No improvement
(a) Had you not reach 197. Everyday, we usually had lunch at 1.30 p.m.
(b) If you have not reached (a) we have had usually (b) we have usually
(c) Had you not reached
(c) we usually have (d) No improvement
(d) If you would not have reached
183. I wish that I read this book before the seminar. 198. All nations must first become agricultural strong.
(a) had read (b) can read (a) become agricultural strong
(c) was reading (d) No improvement (b) become strong agriculture
184. The rules to chess require that one made only one move at (c) become agriculture strong
a time. (d) No improvement
(a) makes (b) will make 199. An orangutan’s intelligence is as superior to that of man.
(c) make (d) No improvement (a) is more superior to
185. Let us partake a meal before we start. (b) is superior to
(a) partake of (b) partake off
(c) is superior than that of
(c) partake in (d) No improvement
(d) No improvement
186. Do take an umbrella with you lest you do not get wet.
(a) lest you might not get wet 200. You shall have attended if the court had instructed you to do so.
(b) lest you should not get wet (a) would have had to attend
(c) lest you should get wet (b) would attend
(d) No improvement (c) would have to
187. They will be coming to attend the farewell party, isn’t it? (d) No improvement
(a) aren’t they? (b) wouldn’t they? 201. The relics of Greece over which such a great deal of evidence
(c) won’t they? (d) No improvement has been collected should be preserved.
188. Taxpayers are to be conscious of their privileges. (a) from which (b) on which
(a) need (b) have to (c) ascent which (d) No improvement
(c) ought to (d) No improvement 202. When the beverage was ready, they drank possibly as much
189. You can’t go that way, I’m afraid, as the road is in repairs. as they could.
(a) under repairs (b) under repair (a) as much as they possibly could
(c) on repairs (d) No improvement (b) as much as possibly they could
190. The climate of Karnataka is cooler than Tamil Nadu. (c) as much as they could possibly
(a) is cooler to (b) is cooler than of (d) No improvement
(c) is cooler than that of (d) No improvement 203. A citizen is expected to give allegiance to his country of origin.
(a) homage (b) loyalty
191. The Tsunami victims suffered of cholera.
(c) obedience (d) No improvement
(a) suffered from (b) suffered under
204. We were with daggers drawn despite attempts to understand
(c) suffered in (d) No improvement
each other.
192. I gave to Sana the keys. (a) in (b) on
(a) I gave (b) I gave to the (c) at (d) No improvement
(c) I gave the (d) No improvement 205. Why should you be despaired of your success of your
193. If he smokes less he might get rid of his cough. undertaking?
(a) If he smoked less he would get rid of his cough. (a) you despair of the success of your undertaking
(b) If he had smoked less he might get rid of his cough. (b) you despair of success of undertaking
(c) If he smokes less he might have got rid of his cough. (c) you be despaired of the success of your undertaking
(d) No improvement. (d) No improvement
y
o
u
C-96 Sentence Improvement

rs
m
206. As Rees was going to town in the High Street a savage dog 212. I prevailed on him to vote for you.

a
attacked him and bit him. (a) to (b) at

h
b
(a) going to town a savage dog attacked him and bit him (c) upon (d) No improvement

o
in the High Street 213. Eager to pass his final exams, studying was the students

o
b
(b) in the High Street a savage dog attacked him and bit top priority.

.w
him in the town (a) the student's top priority was studying.

o
(b) the student made studying his top priority.

rd
(c) going to town in the High Street a savage dog bit him
(c) the top priority of studying was made by the student.

p
and attacked him

re
(d) No improvement (d) No improvement.

s
s
207. Something is pretty here that Vineeta can wear to the party. 214. Mr. Dev will not go to the wedding reception without being called.

.c
(a) if he is not invited (b) till he is invited

o
(a) Something here is pretty

m
(b) Something is here pretty (c) unless he is invited (d) No improvement
215. He will revise it when he is comes back.
(c) Here is something pretty
(a) when he come back (b) on coming back
(d) No improvement
(c) when he came back (d) No improvement
208. I have dreamt all my life to own a beautiful maroon coloured car.
216. The members of the student's union did not give the
(a) of owning (b) to owning
examination in protest.
(c) at owning (d) No improvement
(a) did not write (b) did not sit for
209. Sitting on the hill top, the sun went down watching before him. (c) did not show up for (d) No improvement
(a) he watched the sun go down. 217. She was kept from her assignment because of her loquacious
(b) the sun went down with him watching room-mate.
(c) the sun went down when he watched (a) noisy (b) irritating
(d) No improvement (c) talkative (d) No improvement
210. The office is soon to be closed. 218. James epitomizes everything that a leader should be.
(a) just to (b) about to (a) worships (b) loves
(c) immediately to (d) No improvement (c) adores (d) No improvement
211. He has achieved nothing out of his way worth mentioning. 219. The businessman is respectively connected.
(a) out of the way (b) by the way (a) respectfully (b) respectably
(c) in a big way (d) No improvement (c) receptively (d) No improvement

ANSWER KEY
1 (c) 26 (a) 51 (b) 76 (d) 101 (d) 126 (a) 151 (b) 176 (c) 201 (c)
2 (a) 27 (d) 52 (d) 77 (b) 102 (b) 127 (c) 152 (b) 177 (b) 202 (a)
3 (a) 28 (c) 53 (d) 78 (c) 103 (d) 128 (d) 153 (c) 178 (d) 203 (b)
4 (d) 29 (d) 54 (c) 79 (a) 104 (b) 129 (c) 154 (c) 179 (c) 204 (c)
5 (d) 30 (a) 55 (d) 80 (d) 105 (c) 130 (a) 155 (c) 180 (b) 205 (a)
6 (c) 31 (c) 56 (b) 81 (a) 106 (d) 131 (c) 156 (d) 181 (c) 206 (d)
7 (a) 32 (d) 57 (b) 82 (c) 107 (b) 132 (b) 157 (b) 182 (c) 207 (c)
8 (a) 33 (c) 58 (a) 83 (a) 108 (d) 133 (c) 158 (c) 183 (a) 208 (a)
9 (c) 34 (d) 59 (d) 84 (b) 109 (b) 134 (c) 159 (a) 184 (b) 209 (a)
10 (c) 35 (a) 60 (b) 85 (a) 110 (b) 135 (a) 160 (d) 185 (a) 210 (b)
11 (c) 36 (b) 61 (d) 86 (b) 111 (c) 136 (b) 161 (b) 186 (c) 211 (a)
12 (a) 37 (c) 62 (b) 87 (c) 112 (d) 137 (d) 162 (c) 187 (c) 212 (c)
13 (d) 38 (a) 63 (c) 88 (b) 113 (a) 138 (a) 163 (b) 188 (b) 213 (b)
14 (b) 39 (b) 64 (b) 89 (c) 114 (a) 139 (d) 164 (d) 189 (b) 214 (c)
15 (c) 40 (a) 65 (b) 90 (b) 115 (c) 140 (a) 165 (b) 190 (c) 215 (b)
16 (d) 41 (b) 66 (d) 91 (a) 116 (d) 141 (b) 166 (c) 191 (a) 216 (b)
17 (c) 42 (a) 67 (a) 92 (c) 117 (d) 142 (c) 167 (a) 192 (a) 217 (d)
18 (b) 43 (d) 68 (c) 93 (a) 118 (b) 143 (c) 168 (a) 193 (a) 218 (d)
19 (c) 44 (a) 69 (b) 94 (d) 119 (b) 144 (a) 169 (d) 194 (c) 219 (a)
20 (c) 45 (a) 70 (b) 95 (a) 120 (c) 145 (b) 170 (b) 195 (b)
21 (a) 46 (b) 71 (d) 96 (b) 121 (b) 146 (a) 171 (b) 196 (a)
22 (a) 47 (c) 72 (c) 97 (b) 122 (c) 147 (c) 172 (a) 197 (c)
23 (b) 48 (d) 73 (c) 98 (c) 123 (c) 148 (b) 173 (b) 198 (a)
24 (c) 49 (d) 74 (d) 99 (a) 124 (b) 149 (a) 174 (d) 199 (b)
25 (b) 50 (b) 75 (b) 100 (d) 125 (c) 150 (c) 175 (c) 200 (a)
y
o
u
Sentence Improvement C-97

rs
m
a
h
b
o
o
b
.w
1. (c) When the word ‘except’ is used ‘me’ is used with it 12. (a) The comparison is between the careers of the two

o
rd
instead of I, so the most appropriate form of this phrase women and not a woman and the career of another

p
will be except for you and me. women. The subject is absent from the sentence but

re
2. (a) ‘Calculate’ is not an appropriate word for this sentence, if you complete the sentence you will find it should

s
s
calculation is done on the basis of available facts with

.c
be- she wished that her career was as glamorous as
certain fixed rules, whereas according to the sense of

o
the other woman’s career. In sentences such as these

m
the sentence the person is only making a guess or a
where the noun is hidden, it is helpful to complete the
supposition, thus reckon is the most appropriate word
that can be used to replace calculate. noun to find the correct answer.
3. (a) ‘him’ is not the right usage because it is a possessive 13. (d) Liable to means, it is an event that will happen. This
pronoun, and in the sentence it is followed by who, is the only phrase which would fit in with the sense
which is used to refer. So the pronoun he should be of the rest of the sentence. So there will be no change.
used. He, who has the best rapport with the students. 14. (b) would is a modal and its use here is unnecessary.
4. (d) Different is used with ‘from’ e.g. ‘you are different from There is a ‘would have’ in the second part of the
Ritu’. Than is used for comparisons, e.g., The world sentence but it should not be repeated in the first one,
is more populated than it was in our time. because the use of ‘would have’ is done as a
5. (d) When we use the word ‘hardly’, it implies a negative possibility of an affect of something done in past. So
meaning, there is then no need to use not, so the most generally would have will be followed by a past perfect
suitable use is ‘he hardly had any friends’. verb.
6. (c) The tense in this sentence should be the present 15. (c) The first part of the sentence is directed at a subject,
perfect continous because it refers to an action that
which is ‘eaten in portugal only’, i.e., the potato,
started at some time in past and continues till the
therefore the comma should be followed by a phrase
present time so right use is ‘have been living in
Mumbai.’ with potato as its subject and not Indians. Of (c) &
7. (a) As a child, should be followed by a phrase that has (d), (c) is the right option because ‘suspicious to
‘child’ as its subject, i.e., as a child I was taken by my Indian’ is not an appropriate usage.
parents to visit Jammu & Kashmir or other ways, we 16. (d) The comparative degree for verb ‘fast’ is ‘faster’ and
can change the first part of the sentence, the second there is no need to add more to it.
part describes an action so the first should be an adverb 17. (c) The modal ‘should’ need not be repeated since the
clause. Thus when I was child is the most suitable conjunction ‘and’ is being used. The should in the
alternative. sentence will be used for both the parts.
8. (a) ‘Anyone’ implies a person involved. So a personal
18. (b) ‘in’ is the correct preposition to be used with the verb
pronoun will be needed to give conditions later in the
circulated.
sentence, there must be a ‘he’ or ‘one’ in the sentence.
The noun cannot be in the second person but has to 19. (c) ‘craziness’ is no word, the right noun is craze. If that
be in the third person because ‘Anyone’ refers to an is maddening is chosen it will be follwed by ‘that is
unknown third person. responsible’ which will not be appropriate ‘mad craze’
9. (c) The use of the modal ‘should’ is unnecessary in this is the correct use, and this craze is not for the noun
statement. It is a statement giving directions. ‘speed’ but the verb ‘speeding’, i.e., driving vehicles
10. (c) The part he, his colleague could easily be eliminated at high speed.
and a simpler form of to + verb can be used to make 20. (c) Co-operate has an implicit meaning of together. It means
the sentence more appropriate (a) is grammatically ‘work together’, so together must be eliminated.
incorrect since it has omitted the article ‘an’ (b) Similarly ‘dividing’ means ‘breaking up’ so ‘up’ from
changes the meaning or intent of the underlined
‘dividing’ must also be eliminated.
phrase. Thus (c) is the most suitable choice.
21. (a) ‘building’ is a verb used usually with ‘relations and
11. (c) The first part of the sentence uses the auxillary
verb‘does’, since the second part of the sentence is not links, ‘links’ are simply made or created.
an alternative to the first it must be in the same syntax 22. (a) The errors in the underlined part of the sentence are
so there must be the auxillary verb ‘does’ in the second ‘at summers’ which should be ‘in summer’; ‘during
part as well. winter’ should be ‘in winter’, keeping the system an
y
o
u
C-98 Sentence Improvement

rs
both parts same. Also the use of the infinitive, i.e., to

m
199. (b) Superior always takes preposition 'to'. Ex: Your computer

a
+ verb form is more appropriate in this sentence – ‘to is far superior to mine.

h
b
provide’ in place of ‘for providing’. 200. (a) Would have had to attend

o
23. (b) ‘lending’ is done ‘to’ and not ‘for’, so lending 201. (c) ascent which

o
b
credibility to. 202. (a) as much as they possibly could

.w
24. (c) The second part of the sentence is an action taken as 203. (b) Alternative should be ‘Loyalty’

o
rd
a result of an incident, so the first part of the sentence 204. (c) at

p
must give the incident and use the word ‘since’. (b) 205. (a) The sentence requires an improvement. The underlined

re
is not the right option because it will make the first part portion must be rectified to 'you despair of the success

s
s
active voice and second passive voice which must not

.c
of your undertaking'.

o
happen. Similarlity of voice should be maintained in the 206. (d) The statement requires no improvement. The underlined

m
sentence. portion of the statement suits best with the meaning of
25. (b) The subject is missing from the sentence so first the the statement.
subject ‘I’ should be included. Since an experience is 207. (c) The underlined portion of the statement requires an
being related to the tense of verb will be past so ‘find’ improvement. It may be rectified as 'here is something
cannot be right, thus (d) is eliminated. Whatever pretty that Vinita can wear to the party’.
phrase follows the subject Taj Mahal will act as the 208. (a) The underlined portion requires an improvement. It may
adjective attatched to the verb ‘found’, the remaining be rectified as 'of owning'. So, the sentence will be as 'I
expression will then become only supportive or have dreamt all my life of owning a beautiful maroon
additional information. That is in (b) I found the Taj coloured car'.
Mahal truly impressive is the chief part of the sentence 209. (a) The underlined portion in the sentence requires an
and in the moonlight is additional information, without improvement. It may be rectified as 'he watched the sun
which also the sentence makes sense. In (c) I found go down'. So, the sentence will be 'sitting on the top of
the Taj Mahal in moonlight will be the chief sentence the hill he watched the sun go down'.
which is not the appropriate usage. 210. (b) The underlined portion of the sentence has to be
26. (a) There is no error in the underlined part. rectified. 'Soon to' may be replaced with 'about to'. So,
190. (c) Here we compare the climate of Karnataka with the the sentence will be as 'the office is about to close'.
climate of Tamil Nadu and not with the Tamil Nadu itself; 211. (a) The underlined portion of the sentence needs to be
hence we use 'than that of'. rectified. It may be replaced with 'out of the way'. So, the
191. (a) If someone suffers from an unpleasant or difficult sentence will be as 'He has achieved nothing out of the
experience or situation, then we use 'suffer from.' Ex: way worth mentioning'.
Shiela is suffering from ill health. Lately factories are 212. (c) 'On' may be replaced with 'upon'. So, the sentence will
suffering from a desperate shortage of labours. be as 'I prevailed upon him to vote for you'.
192. (a) The correct arrangement of sentence is - I gave Sana 213. (b) The underlined portion may be replaced with 'the
the keys. student made studying his top priority'. So, the sentence
193. (a) When you are using if to talk about something that is
will be as 'Eager to pass his final exams the student
unlikely to happen or is impossible, use the past tense
made studying his top priority'.
in the if-clause, not present. Ex: If someone gave me
214. (c) The underlined portion in the sentence may be replaced
(NOT gives or would give me) the money, I'd buy a car
with 'unless he is invited'. So, the sentence will be as
tomorrow.
'Mr.Dev will not go to the wedding reception unless he
194. (c) 'He compensated me for the loss.' is the correct answer.
195. (b) The proposition 'to' is complementary with accountable. is invited'.
196. (a) Recently denotes not long ago and thus usually takes 215. (b) He will revise it on coming back.
perfect tense. 216. (b) The members of the student's union did not sit for the
197. (c) Present tense shows what exits or happening now. It examination in protest.
also denotes a habit which in this case is the timing of 217. (d) She was kept from her assignment because of her
having lunch. loquacious roommate.
198. (a) Here 'agriculturally' is the adverb that adds to the 218. (d) James epitomises everything that a teacher should be.
meaning of the adjective 'strong'. Other examples are Epitomises means to personify, typify the traits/
'slowly' in 'He ran slowly', 'very' in 'It's very hot', or characteristics.
'naturally' in 'Naturally, we want you to come.' 219. (a) The businessman is respectfully connected.
y
o
u
Sentence Completion C-99

rs
m
CHAPTER

a
h
SENTENCE

b
9

o
o
b
.w
COMPLETION

o
rd
p
re
s
s
.c
o
m
DIRECTIONS (Qs. 1-277): In the following questions, sentences 12. The police arrived .......... the scene half an hour after the
are given with blanks to be filled with appropriate word(s). accident.
Choose the correct alternative form the given options and (a) on (b) to
indicate it. (c) into (d) in
1. The match having ended in a draw, the first prize was 13. If only you had spoken clearly, you
shared........... (a) would not be misunderstood
(a) between Usha and I (b) by Usha and (b) would not have been misunderstanding
(c) by Usha and me (d) among me and Usha. (c) would not have been misunderstood
2. It is not time for the cinema to begin ..........? (d) would not have misunderstood
(a) so far, is it? (b) yet, isn’t it? 14. He agreed .......... my plan in general, but objected ..........
(c) already, is it? (d) before, isn’t it? some details.
3. If you are really not feeling well. you .......... a doctor. (a) onto, to (b) for, for
(a) should better see (b) may Sec (c) with, about (d) with, to
(c) had better see (d) would rather see 15. Do not stand .......... the rail: the paint is ..........wet.
4. If only I.......... his address, I would most certainly have told (a) against, still (b) on, yet
you.
(c) at, very (d) onto, not
(a) know (b) knew
16. If I.......... enough experience, I would have got the
(c) had known (d) off
5. The marathon race is intended to test one’s endurance appointment.
more........... (a) bad (b) would have had
(a) than his speed (b) than how fast one runs (c) had had (d) were with
(c) than one’s speed (d) lain off 17. I would rather that she .......... me tomorrow.
6. She expects me to type the letter in five minutes .......... is (a) might see (b) can see
impossible. (c) sees (d) will see
(a) that (b) which 18. It is high time that he .......... himself.
(c) what (d) but (a) had reformed (b) will reform
7. Remember to switch .......... the lights before you leave the (c) has to reform (d) reformed
room. 19. .........., a bus almost ran over him.
(a) on (b) down (a) Running across the road
(c) off (d) s o (b) Running on the road
8. Because of the power cut, many workers were........... (c) When he ran across the road
(a) laid of (b) lay out (d) When he was running through the road
(c) laid off (d) would have known 20. My brother has never been to Delhi, ..........
8. He is .......... handsome boy that he is very popular with
(a) and so have I (b) and neither have I
girls.
(c) nor me (d) and I also have not
(a) such a (b) a such
(c) such (d) rather than the speed. 21. I have lived..........this street..........ten years.
10. We hope to complete the job........... (a) on, during (b) in, for
(a) around a week or more (b) within a week or more (c) near, since (d) with, for
(c) within a week or two (d) before a week or two 22. He was ill.......... a full week, and .......... the entire period, his
11. He is .......... of the doctor not to smoke. wife never left his bedside.
(a) in strict orders (b) under strict orders (a) during, in (b) in, in
(c) over strict orders (d) with orders (c) since, for (d) for, during
y
o
u
C-100 Sentence Completion

rs
m
23. He was .......... parking his car .......... a no parking area. 39. You wouldn’t report me, .......... ?

a
(a) while, on (b) on, in (a) don’t you (b) would you

h
b
(c) for, in (d) to, within (c) wouldn’t you (d) do you

o
24. He advised the crowd to work .......... the good ..........their 40. He isn’t going to like this, .......... ?

o
b
country, the crowd received his advice .......... shouts .......... (a) didn’t he (b) did he

.w
contempt. (c) won’t he (d) is he

o
rd
(a) to, for, with, in (b) for, of, with, of 41. These essays are intellectually .............. and represent

p
(c) towards, for, by, of (d) on, of, among, of various levels of complexity.

re
25. Go back .......... the hotel, and wait there .........., I call for you. (a) revealing (b) modern

s
s
(a) to, before (b) in, so that (c) superior (d) persistent

.c
(c) to, until (d) towards, after

o
42. The soldiers are instructed to .......... restraint and handle

m
26. I was so afraid .......... missing the train that I took a the situation peacefully.
taxi..........the station. (a) exercise (b) control
(a) of, to (b) about, upto (c) enforce (d) remain
(c) for, towards (d) over, into 43. Since one connot read every book, one should be content
27. I object .......... being kept waiting. Why are you always so with making a ........ selection.
late .......... appointments? (a) normal (b) standard
(a) to, for (b) for, for (c) sample (d) moderate
(c) about, towards (d) on, over 44. Some people ........ themselves into believing that they are
28. If you do not comply.......... traffic regulations, you will get indispensable to the organisation they work for.
.......... trouble.......... the police. (a) keep (b) fool
(a) about, into, for (b) with, into, with
(c) force (d) denigrate
(c) over, in, with (d) on, to, for
45. How do you expect that country to progress when her
29. Since you are .......... eighteen, this regulation does not apply
government is corrupt, ..... and still lergely feudal?
.......... you.
(a) devalued (b) dwinding
(a) about, for (b) on, with
(c) demobilised (d) demeaning
(c) under, to (d) above, towards
46. The truck was ...... the trafic and the policeman asked the
30. I have heard such a lot .......... him that I’m looking forward
driver to move off.
.......... seeing him eagerly.
(a) failing (b) obstructing
(a) on, for (b) on, to
(c) obviating (d) hiding
(c) about, for (d) about, to
47. The paternalistic attitude is so ingrained to the
31. You wanted that, .......... ?
managements that they will have to ........ try to change it.
(a) would you (b) didn’t you
(c) wouldn’t you (d) do you (a) casually (b) slowly
32. He saw that .......... ? (c) subtly (d) inadvertently
(a) is he (b) won’t he 48. His interest in the study of human behaviour is indeed very
(c) didn’t he (d) doesn’t he .........
33. You know that’s right .......... ? (a) strong (b) large
(a) would you (b) wouldn’t you (c) broad (d) vast
(c) don’t you (d) didn’t you 49. The improvement made by changes in the system was ....
34. He wil be coming .......... ? and id not warrant the large expenses.
(a) is he (b) did he (a) large (b) small
(c) doesn’t he (d) won’t he (c) minute (d) marginal
35. After all this time you’d think he’d have forgotten .......... ? 50. He is too ..... to be deceived easily.
(a) didn’t you (b) wouldn’t you (a) strong (b) modern
(c) don’t you (d) do you (c) kind (d) intelligent
36. The amount he is suffering from hay fever he needs to see 51. There has been a ...... lack of efficiency in all the crucial
a doctor, .......... ? areas of the working of Public Sector Undertakings.
(a) doesn’t he (b) did he (a) positive (b) surprising
(c) won’t he (d) is he (c) conspicuous (d) stimulative
37. You may think you know the answer but you don’t .......... ? 52. I will be leaving for Delhi tonight and ........... to return by
(a) don’t you (b) would you this week end.
(c) wouldn’t you (d) do you (a) waiting (b) plan
38. After working so hard he didn’t deserve to fail the exam (c) going (d) likely
.......... ? 53. Ravi’s behaviour is worthy of ....... by all the youngsters.
(a) doesn’t he (b) did he (a) trial (b) emulation
(c) won’t he (d) is he (c) following (d) exploration
y
o
u
Sentence Completion C-101

rs
m
54. The only way to ...... the country from the evils of (a) joint (b) conflicting

a
communalism is to enforce the rule of law. (c) dual (d) contradictory

h
(a) eradicate (b) mobilise 70. It ws the help he got from his friends which ....... him through

b
o
(c) extricate (d) purge the tragedy.

o
b
55. Even at the risk of economic loss, he ...... refused to take the (a) helped (b) boosted

.w
beaten track. (c) perked (d) sustained

o
(a) repeatedly (b) stead fastly 71. The criminals managed to escape from the prison even

rd
(c) regularly (d) continuously through two armed policemen were ....... vigil over them.

p
re
56. On his sudden demise, may emotions were so complicated (a) taking (b) putting

s
that it was ......... how I felt. (c) guarding (d) keeping

s
.c
(a) unreasonable (b) impossible 72. The speaker did not properly use the time as he went on .....

o
(c) inexplicable (d) unimaginable on one point alone.

m
57. Two of the fugitives managed to remain free by adeptly (a) dilating (b) devoting
avoiding the ....... of the police. (c) deliberating (d) diluting
(a) torture (b) pursuit 73. Ravi had to drop his plan of going to picnic as he had
(c) discovery (d) following certain ..... to meet during that period.
58. Experts fail to understand the ..... behind the decision to move (a) preparations (b) observations
coal by road when there is enough rail capacity in this sector. (c) urgencies (d) commitments
(a) ideology (b) judgement 74. Even in today’s modern society, people ....... god to bring rains.
(c) rationale (d) politics (a) provoke (b) evoke
59. Automobile manufacturers are reviving up to launch a (c) propitiate (d) superimpose
compaign designed to increase consumer ........ about the 75. The good is often ....... with their bones.
new emmission control. (a) buried (b) covered
(a) production (b) education (c) exhumed (d) interred
(c) capacity (d) awareness 76. Eight scientists have ..... the national awards for outstanding
60. His logic ......... everyone, including the expects. contribution and dedication to the profession.
(a) teased (b) defied (a) bestowed (b) picked
(c) surprised (d) confounded (c) bagged (d) conferred
61. The factory went into a state of suspended ....... today with 77. It was ...... hot that day and the cable suffered the brunt of
all its workers on strike. the heat.
(a) symbiosis (b) animation
(a) treacherously (b) acceptably
(c) ways (d) condition
(c) unfailingly (d) unbelievably
62. It is not fair to cast ....... on honest and innocent persons.
78. Sachin was to reach that afternoon but was ....... up at Delhi
(a) aspiration (b) aspersions
for some personal work.
(c) inspiration (d) adulation
(a) kept (b) held
63. You must ....... your career with all seriousness.
(a) direct (b) complete (c) delayed (d) stayed
(c) follow (d) pursue 79. No country can .......... to practice a constant, rigid foreign
64. The villagers .......... the death of their leader by keeping all policy in view of the world power dynamics.
the shops closed. (a) obliviate (b) anticipate
(a) announced (b) protested (c) afford (d) envisage
(c) mourned (d) consoled 80. The prisoner was released on ....... for good behaviour.
65. These medicines are ........ for curing cold. (a) probation (b) bail
(a) proper (b) real (c) parole (d) grounds
(c) effective (d) capable 81. ....... eye witnesses, the news reporter gave a graphic
66. The poor ones continue to ........... out a living inspite of description of how the fire broke.
economic liberalisation in that country. (a) Reporting (b) Observing
(a) find (b) go (c) Seeing (d) Quoting
(c) bring (d) manage 82. The library expects you to return each and every book that
67. I will write a letter to you tentatively ....... the dates of the you have ...........
programme. (a) demanded (b) taken
(a) involving (b) urging (c) lent (d) handed
(c) guiding (d) indicating 83. This article tries to ......... us with problems of poor nations
68. Contemporary economic development differs ....... form the so that we halp them more effectively
Industrial Revolution of the 19th century. (a) allow (b) enable
(a) naturally (b) usually (c) convince (d) acquaint
(c) literally (d) markedly 84. The skill and ease with which he repaired the machine
69. The word gharana points to the ...... concepts of stylistic proved tha he is a/an ...... mechanic.
individuality and handing down of tradition within family (a) able (b) handy
confines. (c) nimble (d) competent
y
o
u
C-102 Sentence Completion

rs
m
85. They have decided to meet the Prime minister in order to 101. The President called upon politicians not to ...... themselves

a
have their ...... heard. with communal and parochial forces.

h
(a) agony (b) apathy (a) counter (b) favour

b
o
(c) woes (d) sorrow (c) cope (d) align

o
b
86. After a recent mild paralytic attack, his movements are ..... 102. A glue produced by bees to ...... their hives appears to

.w
restricted, otherwise he is still very active. contain antibiotic substances.

o
(a) entirely (b) nowhere (a) collect (b) design

rd
(c) not (d) slightly (c) build (d) decorate

p
re
87. Freedom and equality are the ...... rights of every human 103. He very successfully ..... all the allegations levelled against him.

s
being. (a) extricated (b) eradicated

s
.c
(a) inalienable (b) inscrutable (c) retaliated (d) rebutted

o
(c) incalculable (d) institutional 104. The passengers and crew members of the aeroplane had a

m
88. The new industrial policy is a result of the confidence the ....... escape when it was taking off from the runway.
government has in the ....... of the Indian industry. (a) narrow (b) large
(a) opinion (b) existence (c) little (d) brief
(c) status (d) maturity 105. ........ by people’s perception it seems that democracy has
89. The most obvious reason for his lack of popularity was his succeeded in India.
........ to find fault with other. (a) Following (b) Going
(a) propensity (b) reluctance (c) Making (d) Planned
(c) notoriety (d) ability 106. Deepak put on a very .....face when he said something silly
90. Modern public sector managers have become quite..... in and the contrast made everyone laugh.
shifting their loyalities. (a) funny (b) comic
(a) overt (b) inept (c) serious (d) great
(c) learned (d) adept 107. I am given to ...... that you are going abroad.
91. The conference was successful, your style of presentation (a) learn (b) think
was ......, everyone liked it. (c) predict (d) understand
(a) voluable (b) difficult 108. Man power is the ...... means of converting other resources
(c) conducive (d) wanderful to mankind’s use and benefit.
92. A large crowd used to ....... to listen to his speech. (a) insuperable (b) inimitable
(a) follow (b) gather (c) indivisible (d) indispensable
109. The petition before the Court prayed for ...... the
(c) deliver (d) collect
appointment orders issued by the management.
93. Most of the issues discussed in the meeting were trivial
(a) granting (b) removing
and only a few were .....
(c) posting (d) quashing
(a) practical (b) complex
110. In the departmental inquiry, it was denied that the police
(c) significant (d) irrelevant
had committed any ....... on people under their custody.
94. Rajeev is too ....... as far as his good habits are concerned.
(a) injuries (b) crime
(a) enloyable (b) fastidious
(c) blunder (d) excesses
(c) curious (d) involved
111. How much did it ..... to reach Bombay by car?
95. My father keeps all his........ papers in a lock and key. (a) charge (b) price
(a) required (b) necessary (c) cost (d) estimate
(c) useful (d) confidential 112. Every one should ....... himself against illness since medical
96. Freedom is not a ....... but out birth right. care has now become expensive
(a) sin (b) gift (a) vaccinate (b) insure
(c) farce (d) illusion (c) brace (d) ensure
97. There are ....... views on the issue of giving bonus to the 113. Research has also ......... the illusion that childhood dreasms
employees. ae pure innocence.
(a) Independent (b) divergent (a) accepted (b) observed
(c) modest (d) adverse (c) dispelled (d) discovered
98. He has ..... people visiting him at his house because he 114. Dowry is no longer permitted by law even in ........... marriage
fears it will cause discomfort to neighbours (a) love (b) conventional
(a) curtailed (b) requested (c) natural (d) bigamous
(c) stopped (d) warned 115. After reaching New York, Ramakant will have to ......... himself
99. The defending champion justified his top ...... by clinching to the new surroundings.
the title (a) mix (b) develop
(a) skill (b) form (c) submit (d) adapt
(c) technique (d) billing 116. Belying his mother ’s worries, Rajesh’s behaviour
100. He applied for and was ....... legal aid by the Labour Ministry. throughout the function was .......
(a) offered (b) granted (a) immodest (b) impeccable
(c) allowed (d) awarded (c) imaginable (d) imperial
y
o
u
Sentence Completion C-103

rs
m
117. There is no doubt that one has to keep ........... with the 134. I have lost my purse. Can I ........ some money from you?

a
changing times. (a) borrow (b) hire

h
(a) aside (b) oneself

b
(c) demand (d) loan

o
(c) pace (d) himself 135. The more your action and thought are allied and ........, the

o
b
118. Kamal’s friends had nothing to offer him other than ....... in happier you grow.

.w
his grief. (a) invincible (b) divergent

o
(a) friendship (b) kindness (c) integrated (d) unravelled

rd
(c) solution (d) consolation 136. You must ........ facts and not run away from the truth.

p
re
119. Rubbers often act in ........... with the police. (a) realise (b) notice

s
(a) connection (b) conformity

s
(c) face (d) know

.c
(c) collusion (d) Co-ordination
137. Some students are ........ and want to take only the courses

o
120. It is impossible for an ordianty mortal to ......... all these

m
figures to memory for which they see immediate value.
(a) keep (b) commit (a) indolent (b) foolish
(c) retain (d) hoard (c) pragmatic (d) theoretical
121. Many of the advances of civilisation have been conceived 138. A cheerful man ....... all difficulties and hardships with a
by young people just on the ......... of adulthood. smile on his face.
(a) horizon (b) boundary (a) challenges (b) embraces
(c) peak (d) threshold (c) resists (d) endures
122. The accused ....... that he had met the deceased before. 139. Such ....... clothes did not appeal to her nor did they suit her.
(a) remarked (b) refused (a) simple (b) gaudy
(c) exclaimed (d) denied (c) ornate (d) fatuous
123. His father-in-law ........... his up in business. 140. He requested all his office ........ to join him at the party.
(a) made (b) put (a) comrades (b) companions
(c) built (d) set (c) colleagues (d) collaborators
124. This was the first time Roshan had been found guilty. He 141. He knew what he was doing was right and so their catcalls
had no ...... convictions. did not ...... him.
(a) past (b) earlier (a) worry (b) threaten
(c) previous (d) former (c) faze (d) disconcert
125. The .......... to distinguish differences is the basic of science
142. I listened, but I had no idea what he was ........ about.
and art.
(a) saying (b) telling
(a) power (b) tact
(c) talking (d) discussing
(c) strength (d) wishdom
126. Speeding and blocking are traffic offences which lead to 143. The doctor was overcome with .........when he came to know
.......... accidents. that the patient had died due to negligence on his part.
(a) winsome (b) urban (a) conscience (b) remorse
(c) gruesome (d) minor (c) humiliation (d) emotion
127. However, the group’s long term strategy is to ....... on core 144. Although I have been interested in photography, yet I am
sector businesses connected with infrastructure and energy only a/an ............
(a) depend (b) breed (a) novice (b) amateur
(c) develop (d) concentrate (c) apprentice (d) unprofessional
128. He begged leave of the king on the ......... that he had to 145. You will have to face some practical problems when you
attend his ill wife. start ........... this plan.
(a) offer (b) excuse (a) prosecuting (b) projecting
(c) pardon (d) pretext (c) prescribing (d) proscribing
129. Finding that there was no use moving any further, the officer 146. There are various hobbies for us to ........ in our leisure hours.
order the army’s .......... (a) pursue (b) follow
(a) withdrawal (b) retreat (c) absorb (d) contribute
(c) restoration (d) revival 147. Every Shakespearean hero has an internal ........ in his
130. Kavita ........ a sum of Rs 200 out of the bank every Monday.
character.
(a) extracts (b) draws
(a) fault (b) defect
(c) obtains (d) pulls
(c) flaw (d) weakness
131. We offered a ......... of incentives to attract a new banker.
(a) bundle (b) assurance 148. Statistics are ........... as a means of determining public
(c) gift (d) package opinion.
132. These facts has been ............ into them. (a) unreliable (b) dubious
(a) digested (b) dictated (c) uncertain (d) phoney
(c) drummed (d) spoken 149. He was abole to ........ his small income by working in a hotel
133. Every individual wishes to lead a .......... life. at night.
(a) respected (b) respectful (a) amplify (b) supplement
(c) respectable (d) respective (c) expand (d) multiply
y
o
u
C-104 Sentence Completion

rs
m
150. The solution to the problem ........... me. 165. They are refugees in need of .........

a
(a) outgrew (b) outraged (a) renovation (b) recapitulation

h
(c) evaded (d) undermined (c) restoration (d) rehabilitation

b
o
151. This test .......... to previous knowledge of the subject : any 166. The teacher promised to ........... a prize to the boy who stood

o
b
graduate is expected to complete it successfully. first in the class.

.w
(a) concerns (b) presupposes (a) reward (b) award

o
(c) assummes (d) necessitates (c) promote (d) secure

rd
152. The clever politician .......... his way to the ministerial position 167. If our friends are not able to take us in their car, we must

p
re
in a short time. make ......... arrangements to go to the airport.

s
(a) faked (b) wangled (a) possible (b) alternate

s
.c
(c) scaled (d) moved (c) another (d) alternative

o
153. The rain water that does not flow to the rivers, ........ beneath 168. Please do not ....... the offer made by the Chairman.

m
the soil to form underground water. (a) refrain (b) refute
(a) flows (b) penetrates (c) refuse (d) refuge
(c) seeps (d) percolates 169. It is generally believed that the rats are the first to leave a
154. The brilliant students will be ............. scholarships. ...... ship.
(a) honoured (b) awarded (a) moving (b) anchored
(c) rewarded (d) forwarded (c) sinking (d) drowning
155. According to corporate circles, Datta is pushing through 170. My mother upset the kettle of boiling water and ...... her
the merger to create a financially ............... company in the right hand badly.
processed foods business, the group’s the processed foods (a) wounded (b) sizzled
business, the group’s thrust area for the 1990s. (c) scorched (d) scalded
(a) straight (b) powerful 171. Legislation was passed to punish brokers who ....... their
(c) leading (d) acceptable clients’ funds.
156. A stone that goes on rolling ............ no mass (a) devour (b) defalcate
(a) gets (b) accumulates (c) devastate (d) embezzle
(c) collects (d) gathers 172. The figures were barely ........ in the dim light.
157. The doctor gave the woman a .................. to calm her down (a) obvious (b) perceived
(a) tonic (b) sedative (c) perceptive (d) susceptible
(c) antiseptic (d) antidote 173. It is my considered opinion that English education in the
manner it has been given has ....... the English educated
158. Several of our players were injured, so our losing the match
Indians
was almost ............
(a) emasculated (b) exocommunicated
(a) necessary (b) indispensable
(c) castrated (d) intimidated
(c) inevitable (d) inexcusable
174. ........... learning does not help the menttal growth of a
159. There could have been a war on it, but in the end reason
student.
..........
(a) guided (b) rote
(a) persisted (b) counted
(c) assisted (d) fast
(c) prevailed (d) survived
175. The accused denied having committed the crime but could
160. My friend says that he drinks tea because it is the best ..... advance no sound arguments to ........... the charges
in the world. (a) refuse (b) repress
(a) fluid (b) drink (c) refute (d) reproof
(c) beverage (d) liquid 176. Manuring in argicultural fields increases ............
161. Every human being in the developed country ....... over five (a) income (b) fertility
pounds of solid wastes per day in various forms. (c) profitability (d) productivity
(a) contributes (b) fosters 177. Life is to death as pleasure is to .........
(c) produces (d) develops (a) suffering (b) pain
162. By sheer hard work, Sushil has ........ out a niche for himself (c) poverty (d) anguish
in the field of journalism. 178. Orchestral players are ......... behind screens, and at no time
(a) carved (b) worked do the judges bear their names
(c) derived (d) accomplished (a) confined (b) conditioned
163. The President of the Students’ Union played into the hands (c) visualised (d) auditioned
of some political leaders and ....... the students to go on a 179. The patient was cheered ........ by the news that she was
strike. likely to be discharged in a day or two.
(a) invited (b) ignited (a) on (b) up
(c) instigated (d) inspired (c) out (d) down
164. We should not mix with those men who have an ........ 180. The thieves had driven ten miles before the police caught
reputation ........them.
(a) unsafe (b) unsavoury (a) on with (b) upon
(c) unsteady (d) unsanctified (c) up to (d) up with
y
o
u
Sentence Completion C-105

rs
m
181. As soon as the visitor’s dishonest purpose was discovered 198. Though young, he proved a worthy rival ........ his opponent

a
he was ........ the door. who had much more experience.

h
(a) show with (b) shown to (a) to (b) against

b
o
(c) shown (d) shown out of (c) for (d) with

o
b
182. As soon as my attention was ........ the dangerous state of 199. He was of a charitable disposition, but did not like a number

.w
the staircase, I got it repaired. of his relatives trying to live ........ him without trying to earn

o
(a) drawn for (b) drawn upon their living.

rd
(c) drawn near (d) drawn to (a) with (b) near

p
re
183. ........ pompous, he was an entertaining companion. (c) off (d) through

s
(a) Before (b) Never 200. The by-election, ........ closely fought, resulted ........ a surprise

s
.c
(c) Though (d) Despite landslide for the ruling party.

o
184. I wish my brother ........ here to listen to this entertaining lecture. (a) if, at (b) since, with

m
(a) would be (b) has been (c) whether, in (d) though, in
(c) is (d) were 201. You cannot be too sensitive........ criticism, if you have
185. Did you think you ........ somewhere before? chosen a political career.
(a) have seen me (b) saw me (a) about (b) with
(c) had seen me (d) would see me (c) for (d) to
186. John is great ........ organizing meetings. 202. ........ we are good friends his views differ ........mine on many
(a) for (b) to issues.
(c) at (d) in (a) Despite, with (b) Because, off
187. He has ........, he deals both in books and curios. (c) Although, from (d) Nevertheless, form
(a) two bows to his string (b) two strings to his bow 203. A peculiar custom prevailing........ Toads is the sacrifice of
(c) two arrows to his bow (d) two bows and strings. buffaloes on ceremonial occasions.
188. The way he is currying favour ........ his rich neighbour is (a) over (b) with
sickening. (c) by (d) among
(a) of (b) with 204. ........ the four children, Raju is ........
(c) for (d) to (a) Between, taller (b) Amidst, tallest,
189. Because of his distrust........ every one, he could never gain (c) Among, the tallest (d) In, the tallest.
any good friend.
205. Having placed ........ proposals before you, I now ........ your
(a) by (b) of
decision.
(c) with (d) in
(a) alternate, waited for (b) different, wait to
190. The new Twenty Point Programme is designed to be a
(c) alternative, await (d) many, am waiting
renewed assault ........ poverty.
206. The train to Bombay ........ from Madras Central at 9.30 pm.
(a) for (b) into
(a) commences (b) begins
(c) on (d) upon
(c) leaves (d) starts
191. If you have a good project but are lacking ........finance, you
207. Einstein was awarded the Nobel Prize for his ........ the Theory
should enter into partnership ........an affluent person.
(a) of, with (b) for, with of Relativity.
(c) in, with (d) in, of (a) inventing (b) invention of
192. He was operated ........ an abscess in his leg. (c) promulgation of (d) discovery of
(a) for (b) against 208. He is a good ........ and has specialised in painting walls of
(c) on for (d) on against houses.
193. Living a simple life ........ one’s won resources is preferable (a) artist (b) artiste
........ leading a luxurious life on borrowed funds. (c) artisan (d) artefact
(a) with, than (b) within, to 209. However poor one may be, one can be happy only if one
(c) within, against (d) on, from has the right attitude to life; happiness lies........ contentment.
194. I doubt whether the accommodation in that place is (a) through (b) in
adequate........our needs. (c) with (d) over
(a) according to (b) with 210. Being himself very quick ........ arithmetical calculations, he
(c) for (d) against did not need a calculator.
195. The teacher warned the students to desist ........making noise. (a) ahout (b) for
(a) from (b) about (c) over (d) in
(c) with (d) by 211. Though accused of partiality........ his home team, the umpire
196. It was customary ........ devotees going to that temple to had a clear conscience and believed in the correctness of
take a bath in the nearby tank first. his decisions.
(a) with (b) on (a) with (b) towards
(c) for (d) about (c) for (d) against
197. Absorbed ........ his own thought, he paid scant attention to 212. Though I would not recommend it, I have no objection........
what was happening ........ him. your going to that movie.
(a) with, about (b) in, around (a) for (b) against
(c) by, to (d) of, besides (c) to (d) upon
y
o
u
C-106 Sentence Completion

rs
m
213. Do not force me to ........ you on this issue; I am not at all 229. If I were you, I ............... be careful with my words.

a
convinced. (a) will (b) would

h
(a) agree upon (b) concur with

b
(c) shall (d) should

o
(c) join over (d) equate with 230. Of the two assistants we employed last month, I find

o
b
214. The court acquited him ........ all the charges. Raman .............. hard working.

.w
(a) from (b) against (a) most (b) more

o
(c) for (d) of (c) least (d) only

rd
215. There is no point in counting........ his support he is

p
231. She is so fastidious that ............... of the three houses was

re
notoriously undependable. liked by her.

s
(a) for (b) on

s
(a) neither (b) either

.c
(c) with (d) through (c) none (d) no one

o
216. His claims of close acquaintance........ the high and the mighty

m
232. .............. a very long time this city has been prosperous.
are unbelievable. (a) Since (b) For
(a) about (b) towards (c) From (d) Till
(c) with (d) from 233. The mother of the dead child was overwhelmed .............
217. He spoke for over forty minutes, but the speech was wholly grief.
irrelevant the subject of the meeting.
(a) by (b) with
(a) on (b) to
(c) from (d) for
(c) at (d) for
234. William Shakespeare was .............. greatest playwright of
218. Compared........China, India’s progress in the agricultural field
his time.
has been commendable.
(a) against (b) over (a) a (b) an
(c) towards (d) to (c) the (d) thee
219. The court found him guilty and imposed ........ him a fine of 235. There is no use discussing .............. prohibition.
Rs.500. (a) on (b) about
(a) to (b) for (c) of (d) for
(c) upon (d) against 236. It is earth’s gravity which .............. people their weight.
220. It is believed that hypertension is most often the cause (a) gives (b) give
........ heart attack. (c) giving (d) given
(a) towards (b) with 237. Total weight of all the ants in the world is much greater
(c) in (d) of than ............... .
221. Because of low marks, he could not get admission........ any (a) to all human beings (b) that of all human beings
college. (c) is of all human beings (d) that of the all human beings
(a) in (b) with 238. It is good form to use the name of the person ............... .
(c) into (d) for (a) who are greeting
222. Which competitor was selected for the first prize, the former (b) you are greeting
or ........? (c) which you are greeting
(a) later (b) latter (d) greeting for you
(c) the latter (d) the second 239. .............. that increasing numbers of compact disc players
223. He was greatly disappointed when his proposal, on which will be bought by consumers in the years to come.
he had worked very hard, was turned........ by his boss. (a) They are anticipated (b) In anticipation
(a) up (b) over (c) Anticipating (d) It is anticipated
(c) down (d) off 240. .............. business, a merger is a combination of two or more
224. The child said she had seen someone in the garden, but corporations under one management.
when we made a search we did not find ........ (a) at (b) in
(a) someone (b) no one (c) the (d) on
(c) anyone (d) her
241. A ............... firms are having trouble with industrial relations.
225. ........ Robert are expected to attend the party tonight.
(a) considerable amount of (b) great deal of
(a) as well as (b) together with
(c) large part of (d) great many
(c) and (d) accompanied by
242. He was frightened ...............
226. ........ more I think ........ your suggestion, the less I like it.
(a) to be killed (b) to being killed
(a) Even, on (b) The, about
(c) for being killed (d) of being killed
(c) If, about (d) Some, on
227. Having ........ only in salt water before, I found it a little 243. Capitalist society .............. profit as a valued goal.
difficult to swim in fresh water. (a) which regards (b) regarding
(a) swam (b) s

Das könnte Ihnen auch gefallen